Sei sulla pagina 1di 1225

WILEY ACING THE GATE

ELECTRICAL ENGINEERING
WILEY ACING THE GATE
ELECTRICAL ENGINEERING

Dr. Debashis Chatterjee


Professor
Department of Electrical Engineering
Jadavpur University
Kolkata

Dr. J. S. Lather
Professor
Department of Electrical Engineering
National Institute of Technology
Kurukshetra

Dr. Lalita Gupta


Assistant Professor
Maulana Azad National Institute of Technology
Bhopal
WILEY ACING THE GATE
ELECTRICAL ENGINEERING
Copyright © 2016 by Wiley India Pvt. Ltd., 4435-36/7, Ansari Road, Daryaganj, New Delhi-110002.

All rights reserved. No part of this book may be reproduced, stored in a retrieval system, or transmitted in
any form or by any means, electronic, mechanical, photocopying, recording or scanning without the written
permission of the publisher.

Limits of Liability: While the publisher and the author have used their best efforts in preparing this book,
Wiley and the author make no representation or warranties with respect to the accuracy or completeness of
the contents of this book, and specifically disclaim any implied warranties of merchantability or fitness for
any particular purpose. There are no warranties which extend beyond the descriptions contained in this
paragraph. No warranty may be created or extended by sales representatives or written sales materials.

Disclaimer: The contents of this book have been checked for accuracy. Since deviations cannot be
precluded entirely, Wiley or its author cannot guarantee full agreement. As the book is intended for
educational purpose, Wiley or its author shall not be responsible for any errors, omissions or damages
arising out of the use of the information contained in the book. This publication is designed to provide
accurate and authoritative information with regard to the subject matter covered. It is sold on the
understanding that the Publisher is not engaged in rendering professional services.

Other Wiley Editorial Offices:


John Wiley & Sons, Inc. 111 River Street, Hoboken, NJ 07030, USA
Wiley-VCH Verlag GmbH, Pappellaee 3, D-69469 Weinheim, Germany
John Wiley & Sons Australia Ltd, 42 McDougall Street, Milton, Queensland 4064, Australia
John Wiley & Sons (Asia) Pte Ltd, 1 Fusionpolis Walk #07-01 Solaris, South Tower Singapore 138628
John Wiley & Sons Canada Ltd, 22 Worcester Road, Etobicoke, Ontario, Canada, M9W 1L1

First Edition: 2016


ISBN: 978-81-265-5087-6
ISBN: 978-81-265-8235-8 (ebk)
www.wileyindia.com
PREFACE

Wiley’s Acing the GATE Examination in Electrical Engineering is intended to be the complete book for
those aspiring to compete in the Graduate Aptitude Test in Engineering (GATE) in Electrical Engineering discipline.
It comprehensively covers all the topics as prescribed in the GATE 2016 syllabus in terms of study material, quick
reference material and an extensive question bank, complete with solutions. The book offers a number of useful features
and the approach is logical concept building rather than only formula based, as offered by the other books generally
published in this domain.
The objective has been to structure this book as a complete reference covering fundamental aspects of theory before
proceeding to relevant questions. A three tier approach has been adopted to create this manuscript, that is, coverage of
the basic building blocks for each of the subjects followed by the exhaustive solved examples, and then practice exercises
with one and two marks questions. Lastly the set of questions from GATE previous years’ papers starting 2003 to cur-
rent have been provided along with their solutions and explanations as a supplement to the formal subject coverage.
The book presumes basic understanding of the fundamentals of Electrical Engineering and related basic electronics.
The book is divided into 10 chapters based on the units of the syllabus, wherein each chapter constitutes a subject. The
chapters are divided into various sections which are self-sufficient and easy to read and understand. The sequence of the
chapters has been arranged in such a way that almost no cross-referencing to subsequent chapter is needed. The system-
atic coverage of the book by the aspirant enhances their knowledge with valuable insights into problem solving approach.
The authors are of the view that the systematic coverage of this text will not only enhance the GATE cracking skills
but also impart a level of knowledge to the students which might have been missed by them during formal study of
the subjects in the classroom. In addition, the subject centric approach of the books trains the reader to crack other
parallel examinations like UPSC Exams etc.
Though adequate precautions have been taken to ensure correctness of theoretical concepts, equations and related
questions, we appreciate communication regarding any inadvertent mistakes that you might come across during the
course of your study.

FM.indd 5 3/24/2016 12:36:48 PM


vi        Preface 

Last but not the least, the authors wish to convey thanks to the entire editorial and production team for being
patient to wait for the completion of manuscript by the authors and understanding the technicalities involved in the
writing such a comprehensive book and finally for all their efforts to make this dream a reality. The continuous inputs
from the editorial team of Wiley India and systematic coverage and compilation by the authors, have resulted into this
cohesive and complete reference text for GATE examination in Electrical Engineering.

 D. Chatterjee, J. S. Lather and Lalita Gupta

FM.indd 6 3/24/2016 12:36:48 PM


ABOUT THE AUTHORS

Dr. Debashis Chatterjee is Professor at the Department of Electrical Engineering, Jadavpur University, Kolkata.
He received his B.E. in Electrical Engineering from Jadavpur University, Kolkata, M. Tech in Machine Drives and
Power Electronics from IIT-Kharagpur and Ph.D. from Jadavpur University, Kolkata. Dr. Chatterjee is the author
and co-author of more than 60 journal articles and conference papers in reputed publications. His areas of interest are
Parameter Estimation and Speed Control of Induction Machines, Control of Induction Generators, Development of
Improved Harmonic Elimination Techniques of Inverters, Control of Permanent Magnet Machines.
Dr. J. S. Lather is Professor at the Department of Electrical Engineering, National Institute of Technology, Kurukshetra.
He received his B.E. in Electrical Engineering from SVNIT, Surat, M. Tech in Control Systems from NIT, Kurukshetra
and Ph.D. in the area of Robust Control from NIT, Kurukshetra. Dr. Lather has more than 21 years of teaching, research
and industrial experience to his credit. He has published more than 40 research papers in national and international
journals and in conferences proceedings. His areas of specialisation are Control Systems, Power Systems and Semantic
Computing.
Dr. Lalita Gupta is Assistant Professor at the Department of Electronics and Communication Engineering, Maulana
Azad National Institute of Technology, Bhopal. She received her B.E. in Electronics and Telecommunication from
Pt. Ravi Shankar Shukla University Raipur, M. Tech in Digital Communication from Maulana Azad National Institute
of Technology, Bhopal and Ph.D. from the same institute. Since July 2004 she has been associated as a faculty member
with the Maulana Azad National Institute of Technology, Bhopal. She is a member of IEEE, IETE, ICEIT, IE.
Dr. Gupta has 35 research publications in national and international journals of repute. Her area of specialisation is
Signal Processing.

FM.indd 7 3/24/2016 12:36:48 PM


FM.indd 8 3/24/2016 12:36:48 PM
ACING THE GATE

ABOUT GATE EXAMINATION

The Graduate Admission Test in Engineering (GATE)  is an All-India level competitive examination for engineering
graduates interested in pursuing Masters or Ph.D. programmes in India The examination tests the examinees in
General Aptitude, Engineering Mathematics and the discipline (subject) of study in the undergraduate course. The
objective of GATE is to identify meritorious and motivated candidates for higher studies in Engineering and Sciences.
The examination serves as a benchmark for normalisation of the undergraduate engineering education in the country.
The level of competitiveness can be gauged from the fact that close to ten lakh students appear in this competitive
examination every year.

Admission to Higher Learning Courses

A valid GATE score is essential to become eligible for admission to the post-graduate courses in Engineering, that is,
M.Tech, M.E. or direct doctoral programme in the Indian higher education institutes. Although qualifying the GATE
examination entitles you to apply for the higher degrees; achieving qualifying score is definitely not enough if one is
aspiring for admission to top institutes like the IITs, the NITs, the Indian Institute of Science (IISc) and some of the
high ranked universities. For this, a high GATE score is important. Needless to say, a percentile of greater than 95 is
perhaps the least one needs to secure admission to a top institute. A total of 804463 candidates appeared for GATE
2015 out of which about 125851 students belonged to Electrical branch of engineering. It is important to mention here
that only 15.05% of those who appeared could qualify according to the qualifying marks set by the GATE examination
committee.

Financial Assistance

Selected GATE qualified candidates admitted to M.Tech programmes in Colleges/Universities all over India are
eligible for obtaining financial assistance. The financial assistance is awarded to Indian nationals doing the M.Tech
programmes, subject to institute rules. It is also available in the form of Half-Time Teaching Assistantship (HTTA)

FM.indd 9 3/24/2016 12:36:48 PM


x        Acing the GATE 

and is tenable for a maximum period of 24 months. HTTA students are required to assist the department for 8 hours
of work per week related to academic activities of the department such as laboratory demonstration, tutorials,
evaluation of assignments, test papers, seminars, research projects, etc.

Jobs in Public Sector Undertakings

While admission to a top institute for the Masters programme continues to be the most important reason for working
hard to secure a good score in the GATE examination; another reason to appear and qualify GATE examination with
good score is that many Public Sector Undertakings (PSUs) are and probably in future almost all, will be recruiting
through GATE examination. And it is quite likely that even big private sector companies may start considering GATE
seriously for their recruitment as GATE score can give a bigger clue about competencies of candidates they are recruiting.
A large number of PSUs have already started recruiting on the basis of GATE score. These include companies like, Power
Grid, Delhi Development Authority (DDA), Indian Oil, Bharat Electronics (BE), Bharat Heavy Electricals Limited
(BHEL), National Thermal Power Corporation (NTPC), HPCL, DVC, NALCO, NLC, Central Electronics Limited
(CEL), BSPHCL, Vizag Steel and Gas Authority of India Limited (GAIL). In 2015, 15 PSUs signed MoUs with IIT
Kanpur to receive the GATE 2015 results across various papers.

APPLYING FOR THE EXAMINATION

Eligibility

The candidates applying for GATE examination must meet the under mentioned requirements.
1. A candidate is allowed to appear only in one paper. The first step therefore is to select the paper you wish to
appear for.
2. The next step is to choose the city for appearing in the examination. There are three choices to be given in the
order of preference. The candidate can choose a particular city as the first choice for appearing in GATE exami-
nation. Having done that, he/she would know the zone the chosen first preference city belongs to. The candidate
can then choose his/ her second choice only from the cities available in that zone. As an additional option, a third
choice was also introduced from GATE 2014. The list of third choice cities will be as specified by each zone. Note
that this third choice city may either be from the zone to which the first and second choice cities belong or from
some other zone. The third choice will be considered only when the candidate can not be accommodated either
in first or second choice cities. The tentative zone wise list of cities for GATE every year is given in Examination
Cities. However, the GATE Committee reserves the right to add a new city or remove an existing city and allot
a city that may not be any of the choices of a candidate.
3. Minimum qualification for appearing in GATE EE examination of a given year is B.E/B.Tech (currently in 4th
year or already completed), Integrated M.E./M.Tech, Post B.Sc. (currently in 2nd, 3rd or 4th year or already com-
pleted), Integrated M.E./M.TECH or Dual degree post 10 + 2 or Diploma (currently in 4th or 5th year or already
completed) and Professional Society Examinations equivalent to B.E./B.Tech (completed section-A or equivalent
of such courses).
4. Candidates who are likely to complete the qualifying examination during the year of the GATE examination or
later have to submit a certificate from their college Principal. They have to obtain a signature from their principal
along with the seal on the “Certificate from the Principal” format that will be printed on the application PDF file
which is generated after completion of the online application submission.
5. Candidates who have appeared in the final semester/year exam in the year immediately preceding the year of GATE
examination (for GATE-2016, it will be 2015), but with a backlog (arrears/failed subjects) in any of the papers in their
qualifying degree should (a) submit a copy of any one of the marks sheets of the final year, or (b) have to obtain a
signature from their Principal along with the seal on the “Certificate from the Principal” format that will be printed
on the application PDF file which will be generated after completion of the online application submission.

Official Website

All announcements regarding GATE can be seen on the official website of the current organizing institute. There are
a large number of other websites that contain GATE relevant information.

FM.indd 10 3/24/2016 12:36:48 PM


Acing the GATE          xi

STRUCTURE OF THE EXAMINATION

The GATE examination is conducted for 22 disciplines (papers) that are listed in GATE brochure and also available
on official GATE website. The syllabus for each of these is also given separately in detail. The candidate is expected to
select and appear in the appropriate paper as per the discipline of his qualifying degree. However, he is free to choose
any paper, depending on the plan for admission into higher degree and the eligibility requirements of the same.

Examination Pattern

GATE examination consists of a single paper of 3 hour-duration. There are a total of 65 questions for 100 marks belonging
to the following sections:
•  General aptitude: Comprises of 10 questions, out of which five questions are of 1 mark each and five for 2 marks
each. These are designed to check the language and analytical skills of the aspirants.
•  Subject paper: Comprises of 25 questions of 1 mark each and 30 questions of 2 marks each with Engineering
Mathematics constituting 13—15% of the total marks. These are designed to check the subject knowledge of the
aspirants.
The questions are a mix of multiple choice and numerical type:
•  Multiple choice questions (MCQs): These questions will have single option correct
•  Numerical answer questions (NAQs): These come with no choices and candidates are expected to answer using
a virtual keypad. The numerical answer will be a real number, signed or unsigned, with due consideration being
given for a range during the answer evaluation.

Marking Scheme

For 1 mark questions, 1/3 Mark is deducted for a wrong answer. For a 2 mark question, 2/3 mark is deducted for a
wrong answer. There is no negative marking for numerical answer type questions.

Mode of Examination

The GATE examinations for the papers of all streams are held on-line. These include papers with codes AE, AG, AR,
BT, CE, CH, CY, GG, MA, MN, MT, PH, TF, XE, XL, CS, EC, EE, IN, ME and PI.

UNDERSTANDING GATE RELATED INFORMATION

Pre-Examination

Pre-examination related information is covered in detail under different headings in the previous pages. Before starting
the application process, you must:
1. Ensure you are eligible for the relevant GATE examination.
2. Determine the GATE paper you wish you appear for (You can appear in only one paper).
3. Choose at least two cities that are convenient for you to write the exam.
4. Application for appearing in GATE has to be made online only.
5. All Supporting documents should be sent online only. No hard copy will be accepted.
6. Payments have to be made through debit/ATM cards, credit cards or internet banking and e-challan only
7. Your choice of exam paper will determine date, and choice of available cities.

Post-Examination (Normalisation of GATE Score)

Examination for CE, CS, EC, EE and ME papers is generally held in multi-sessions. Hence, for these papers,
a suitable normalisation is applied to take into account any variation in the difficulty levels of the question
papers across different sessions. The normalisation is done based on the fundamental assumption that “in all

FM.indd 11 3/24/2016 12:36:48 PM


xii        Acing the GATE 

multi-session GATE papers, the distribution of abilities of candidates is the same across all the sessions”. This
assumption is justified since the number of candidates appearing in multi-session papers in GATE 2015 is large
and the procedure of allocation of session to candidates is random. Further it is also ensured that for the same
multi-session paper, the number of candidates allotted in each session is of the same order of magnitude. For the
above mentioned papers; GATE score will be computed based on the normalized marks and not the actual mark
obtained in the examination. For all other papers, actual marks obtained in the examination will be used for
computation of GTE score.

ATTRIBUTES FOR SUCCESS IN THE EXAMINATION

As PSUs began to accept GATE score as shortlisting criteria, the number of candidates appearing for the Electrical
Engineering branch has greatly increased. In 2010, 52,246 students appeared for GATE in Electrical Engineering; while
in 2015, the number rose to 125,851. Even as the number of seats for M.Tech programs in IITs and NITs increase, it
is marginal as compared to the increase in the number of candidates. This has increased level of competition and has
brought focus on the methodical planning that students must embrace to succeed in such tough environment. There
are many ingredients to being successful in this examination. With an aim to offer you the leading edge, we suggest the
following attributes so as to enhance your chance of success.

Preparing for the Examination

Knowing your Subject

The syllabus for GATE covers all the subjects essential to Electrical Engineering. It takes both time and strategic
planning to go through the entire syllabus which was covered in 6—7 semesters during B.Tech/B.E. courses.
1. The focus of GATE is on core concepts. Most university syllabi include advanced topics, which are not included
in GATE. Have a clear demarcation of the topics that need to be mastered.
2. GATE is based mostly on numerical problem solution. There is no rote learning involved. Practice numericals to
understand concepts, especially from the subjects that have a higher weightage.
3. Be aware of the standard notations and assumptions, for those are typically not provided in the statement of the
questions.
4. Build on your strengths. Work on the subjects you like first. As you start solving GATE level problems from those
subjects, it will boost your confidence and then you can pick up subjects that you may not be very comfortable with.

Preparing a Study Plan

Analyse the past few years’ papers. This will give you a clear idea of which subjects and topics carry the maxi-
mum weightage. A well-chartered study plan takes into account the time required for mastering both the concepts as
well the application of these high weightage topics. The most relevant topics for each unit and unit-wise weightage is
provided in the part opener preceding each chapter opener. These can be helpful in deriving focus areas.
1. The study plan may include a schedule of chapter-wise time allotted to conceptual understanding and problem
solving. The subjects with high weightage like Network Analysis, Electrical Machines, Control Systems and
Power Systems can be allotted extra time, whereas subjects like Digital Electronics and Measurements (which
typically have less weightage) can be done in a shorter time.
2. Extra time may be allotted to the topics you feel are challenging. For example, many students struggle with
Signals and Systems. However, GATE typically deals with the easier concepts from this course, like time shift-
ing and scaling and Fourier transforms, which are simply mathematical algorithms. Start from the very basics of
these topics and increase the level of difficulty slowly.
3. Once you have the confidence to solve all types of problems, attempt the MCQs. Time yourself and know the
average time taken to solve an MCQ. MCQs from some subjects, like Power Systems and Electrical Machines
may take somewhat longer to solve than others. As long as the average time taken is within limits, do not worry
about these.

FM.indd 12 3/24/2016 12:36:48 PM


Acing the GATE          xiii

Refining Problem-Solving Skills

The examination tests the ability to analyse information provided, make judgements, and apply the understanding of
topic to arrive at the best-suited answer.
1. Revision all the important formulae and unit summary would be useful before attempting practice tests or mock
tests. Some subjects like Network Analysis and Control Systems are required for understanding other subjects.
Clear these basics first before going towards advanced topics.
2. GATE questions very often have a lot of information, most of which is not actually needed to solve the problem.
With practice, you’ll learn to filter information down to the data required to solve the problem.
3. Re-work the type of problems that were not solvable in the first attempt. Use these to identify the common mis-
takes you make.
4. Self-evaluate using the answer keys.
•  F
 or questions that you got correct, evaluate your approach against the approach applied. Did you use the short
cuts? Can you get the same solution faster?
•  For questions that you got wrong, identify the mistake—mathematical or conceptual. Rework through the questions.
•  For questions that you could not answer, study the concept again.
5. Attempt as many previous years’ GATE questions based on the topic and follow the above approach for
self-evaluation.

Ensuring Effective Preparation and Time Management

Since the syllabus to be covered is very vast, time management plays a key role in effective preparation. Following steps
might assist you in preparing daily schedule of activities during the course of preparation.
1. Have a realistic idea of the actual time that you have. University examinations or deadlines at work will reduce
the time that you appear to have.
2. Set small weekly targets as you progress through your study plan and focus on achieving them to avoid getting
overwhelmed by the entire process.
3. Avoid skipping a day entirely. If another commitment appears, reduce the number of study hours but don’t break
the continuity.
4. Do not dwell on a problem for too long. Ask for help and sort it out, or keep it aside for later.

TAKING THE EXAMINATION

Maximise your Exam-Taking Efficiency

Towards the end of study plan, it is important to focus on exam preparedness and time management skills.
1. If possible, attempt online Mock GATEs (some will be available on the GAOPS website) and see the time taken
to attempt the full paper. Also, check the accuracy of your answers.
2. Determine if either your speed or accuracy needs more work. Focus on specific topics whose questions too
longer to solve. Practice more of these questions. If questions from a certain topic are incorrect, re-read the
concepts.
3. Do NOT over attempt! It is very tempting to attempt MCQs. But remember that there is negative marking
involved. Mark an MCQ only if you are certain of the answer.

On the Examination Day!

1. It is important to take proper rest and sleep well in the night before the examination. Eat well and keep yourself
healthy.
2. Read the instructions carefully within the allotted time, and raise any equipment/software concerns immediately
to the invigilator.

FM.indd 13 3/24/2016 12:36:48 PM


xiv        Acing the GATE 

3. Read the question statements carefully. What is the question asking for? Work towards the solution step by step.
4. The aptitude section is usually quite easy. Work through it fast and leave more time for the technical section.
5. If any question appears unfamiliar or too difficult, mark it for review. Come back to these questions if time
permits, and try another approach to see if you can work these out. Some  MCQs  may become solvable by
eliminating options.
6. Keep a track of time. If possible, set your watch to alarm after every half an hour.

All the very best!


 Authors

FM.indd 14 3/24/2016 12:36:48 PM


SYLLABUS FOR ELECTRICAL ENGINEERING (EE)

Section 1: Engineering Mathematics

Linear Algebra: Matrix Algebra, Systems of linear equations, Eigenvalues, Eigenvectors.


Calculus: Mean value theorems, Theorems of integral calculus, Evaluation of definite and improper integrals, Partial
Derivatives, Maxima and minima, Multiple integrals, Fourier series, Vector identities, Directional derivatives, Line
integral, Surface integral, Volume integral, Stokes’s theorem, Gauss’s theorem, Green’s theorem.
Differential Equations: First order equations (linear and nonlinear), Higher order linear differential equations with
constant coefficients, Method of variation of parameters, Cauchy’s equation, Euler’s equation, Initial and boundary
value problems, Partial Differential Equations, Method of separation of variables.
Complex Variables: Analytic functions, Cauchy’s integral theorem, Cauchy’s integral formula, Taylor series, Laurent
series, Residue theorem, Solution integrals.
Probability and Statistics: Sampling theorems, Conditional probability, Mean, Median, Mode and Standard
deviation, random variables, Discrete and continuous distributions, Poisson distribution, Normal distribution, Binomial
distribution, Correlation analysis, Regression analysis.
Numerical Methods: Solutions of non-linear algebraic equations, Single and Multi-step methods for differential
equations.
Transform Theory: Fourier transform, Laplace transform, Z-transform.

Section 2: Electric Circuits

Network graph, KCL, KVL, Node and Mesh analysis, Transient response of DC and AC networks, Sinusoidal steady-
state analysis, Resonance, Passive filters, Ideal current and voltage sources, Thevenin’s theorem, Norton’s theorem,
Superposition theorem, Maximum power transfer theorem, Two-port networks, Three phase circuits, Power and power
factor in AC circuits.

Section 3: Electromagnetic Fields

Coulomb’s law, Electric field intensity, Electric flux density, Gauss’s law, Divergence, Electric field and potential due to
point, line, plane and spherical charge distributions, Effect of dielectric medium, Capacitance of simple configurations,

FM.indd 15 3/24/2016 12:36:48 PM


xvi        Syllabus for Electrical Engineering (EE) 

Biot—Savart’s law, Ampere’s law, Curl, Faraday’s law, Lorentz force, Inductance, Magnetomotive force, Reluctance,
Magnetic circuits, Self and mutual inductance of simple configurations.

Section 4: Signals and Systems

Representation of continuous and discrete-time signals, Shifting and scaling operations, Linear time invariant and
Causal systems, Fourier series representation of continuous periodic signals, Sampling theorem, Applications of Fourier
transform, Laplace transform and Z-transform.

Section 5: Electrical Machines

Single phase transformer: equivalent circuit, phasor diagram, open circuit and short circuit tests, regulation and effi-
ciency; Three phase transformers: connections, parallel operation; Auto-transformer, Electromechanical energy conver-
sion principles, DC machines: separately excited, series and shunt, motoring and generating mode of operation and
their characteristics, starting and speed control of DC motors; Three phase induction motors: principle of operation,
types, performance, torque-speed characteristics, no-load and blocked rotor tests, equivalent circuit, starting and speed
control; Operating principle of single phase induction motors; Synchronous machines: cylindrical and salient pole
machines, performance, regulation and parallel operation of generators, starting of synchronous motor, characteristics;
Types of losses and efficiency calculations of electric machines.

Section 6: Power Systems

Power generation concepts, AC and DC transmission concepts, Models and performance of transmission lines and
cables, Series and shunt compensation, Electric field distribution and insulators, Distribution systems, Per-unit quanti-
ties, Bus admittance matrix, Gauss—Seidel and Newton—Raphson load flow methods, Voltage and frequency control,
Power factor correction, Symmetrical components, Symmetrical and unsymmetrical fault analysis, Principles of over-
current, Differential and distance protection; Circuit breakers, System stability concepts, Equal area criterion.

Section 7: Control Systems

Mathematical modeling and representation of systems, Feedback principle, Transfer function, Block diagrams and
signal flow graphs, Transient and steady-state analysis of linear time invariant systems, Routh—Hurwitz and Nyquist
criteria, Bode plots, Root loci, Stability analysis, Lag, lead and lead-lag compensators; P, PI and PID controllers; State
space model, State transition matrix.

Section 8: Electrical and Electronic Measurements

Bridges and potentiometers, Measurement of voltage, current, power, energy and power factor; Instrument transformers,
Digital voltmeters and multimeters, Phase, time and frequency measurement; Oscilloscopes, Error analysis.

Section 9: Analog and Digital Electronics

Characteristics of diodes, BJT, MOSFET; Simple diode circuits: clipping, clamping, rectifiers; Amplifiers: biasing,
equivalent circuit and frequency response; Oscillators and feedback amplifiers; Operational amplifiers: characteristics
and applications; Simple active filters, VCOs and timers, Combinational and sequential logic circuits, Multiplexer,
Demultiplexer, Schmitt trigger, Sample and hold circuits, A/D and D/A converters, 8085 Microprocessor: architecture,
programming and interfacing.

Section 10: Power Electronics

Characteristics of semiconductor power devices: Diode, Thyristor, Triac, GTO, MOSFET, IGBT; DC to DC conversion:
Buck, Boost and Buck-Boost converters; Single and three phase configuration of uncontrolled rectifiers, Line commu-
tated thyristor based converters, Bidirectional AC to DC voltage source converters, Issues of line current harmonics,
Power factor, Distortion factor of AC to DC converters, Single phase and three phase inverters, Sinusoidal pulse width
modulation.

FM.indd 16 3/24/2016 12:36:48 PM


CONTENTS

Preface v

About the Authors vii

Acing the GATE ix


Syllabus for Electrical Engineering (EE) xv

SECTIONS I & II: ELECTRIC CIRCUITS AND ELECTROMAGNETIC FIELDS 1

1 Electric Circuits 3

1.1 Circuit Concepts and Laws 3


1.1.1 Electrical Quantities and Units 4
1.1.2 Types of Circuits 7
1.1.3 Current and Voltage Sources 9
1.1.4 Kirchoff’s Laws 12
1.2 Circuit Analysis Techniques 14
1.2.1 Equivalent Circuits 14
1.2.2 Series and Parallel Networks for Resistors, Capacitors and Inductors 15
1.2.3 Source Transformation 17
1.2.4 Nodal and Mesh Analysis 18
1.3 Topology 21
1.3.1 Graph of a Network 21
1.3.2 Directed and Undirected Graph 22
1.3.3 Planar and Non-Planar Graph 22
1.3.4 Concept of Tree and Co-Tree 22

FM.indd 17 3/24/2016 12:36:48 PM


xviii        CONTENTS 

1.3.5 Incidence Matrix 23


1.3.6 Link Currents: Tie Set Schedules 24
1.3.7 Cut Set and Tree Branch Voltages 25
1.3.8 Interelation between Matrices 27
1.3.9 Summarised Properties 27
1.4 Network Theorems 27
1.4.1 Linearity and Superposition 27
1.4.2 Superposition Theorem 27
1.4.3 Thevenin’s Theorem 28
1.4.4 Norton’s Theorem 31
1.4.5 Maximum Power Transfer Theorem 33
1.4.6 Reciprocity Theorem 35
1.4.7 Substitution Theorem 37
1.4.8 Tellengen’s Theorem 37
1.4.9 Compensation Theorem 39
1.4.10 Millman’s Theorem 39
1.4.11 Duality Theorem 40
1.4.12 Wye(Star)-Delta Transformations 40
1.5 Laplace Transform 43
1.5.1 Properties of Laplace Transform 43
1.5.2 Laplace Transform of Periodic Function 44
1.6 Transient Response To DC and AC Networks 45
1.6.1 Initial and Steady State 46
1.6.2 DC Transients 47
1.6.3 AC Transients 57
1.7 Sinusoidal Steady State Analyses Using Phasors 62
1.7.1 Phasor 63
1.7.2 Phasor Relationship for Circuit Elements 64
1.8 Magnetically Coupled Circuits 70
1.8.1 DOT Convention 70
1.8.2 Series Connection of Coupled Inductors 70
1.8.3 Parallel Connection of Coupled Coils 71
1.8.4 Analysis of Multiwinding Coupled Circuits 71
1.8.5 Ideal Transformer 73
1.9 Resonance 73
1.9.1 Series Resonance 73
1.9.2 Parallel Resonance 75
1.10 Filters 77
1.11 Three-Phase Circuits 79
1.11.1 Balanced Three-Phase Circuits 80
1.11.2 Unbalanced Three-Phase Circuits 81
1.12 AC Power Analysis 81
1.12.1 Sinusoidal Signals 81
1.12.2 Average Power 81
1.12.3 Effective (RMS) Values of Current and Voltage 82
1.13 Two-Port Network 83
1.13.1 Types of Network Parameters 84
1.13.2 Interconnections of Two-Port Network 87

FM.indd 18 3/24/2016 12:36:48 PM


CONTENTS          xix

Important Formulas 89
Solved Examples 95
Practice Exercises 112
Answers to Practice Exercises 124
Solved Gate Previous Years’ Questions 142

2 Electromagnetic Fields 169

2.1 Coulomb’s Law 169


2.2 Electric Field Intensity 169
2.3 Electric Flux Density 170
2.3.1 Relation between Electric Flux Density and Electric Field Intensity 170
2.4 Divergence and Curl 170
2.4.1 Divergence 170
2.4.2 Curl 170
2.5 Divergence and Stokes’ Theorems 171
2.6 Gauss’s Law 171
2.7 Electric Field and Potential 171
2.7.1 Due to Point Charge 171
2.7.2 Due to Line Charge 172
2.7.3 Due to a Plane (or Infinite Sheet) of Charge 172
2.7.4 Due to Uniformly Charged Sphere 172
2.7.5 Due to Different Distributions 173
2.7.6 Effect of Dielectric Medium on Electric Field 173
2.8 Capacitance 173
2.8.1 Expression for a Parallel Plate Capacitor 173
2.8.2 Series and Parallel Combination of Capacitors 173
2.9 Magnetic Flux and Magnetic Field 174
2.10 Biot—Savart Law 174
2.11 Ampere’s Law 174
2.12 Faraday’s Law and Lenz’s Law of Electromagnetic Induction 174
2.12.1 Faraday’s First Law 174
2.12.2 Faraday’s Second Law 174
2.12.3 Derivation of Faraday’s Law 175
2.12.4 Lenz’s Law 175
2.13 Lorentz Force 175
2.14 Maxwell’s Equations 175
2.14.1 Magnetic Boundary Conditions 175
2.15 Poynting Vector 175
2.16 Inductance 175
2.16.1 Series and Parallel Combination of Inductors 176
2.16.2 Self Inductance 176
2.16.3 Mutual Inductance 176
2.17 Self and Mutual Inductance for Simple Configurations 177
2.17.1 Inductance of Solenoid 177
2.17.2 Inductance of a Coaxial Cable 178
2.18 Magnetic Circuit and Magnetomotive Force 179
2.18.1 Reluctance 179

FM.indd 19 3/24/2016 12:36:48 PM


xx        CONTENTS 

Important Formulas 179


Solved Examples 181
Practice Exercises 182
Answers to Practice Exercises 183
Solved Gate Previous Years’ Questions 184

SECTION III: SIGNALS AND SYSTEMS 189

3 Signals and Systems 191

3.1 Elementary Signals 191


3.1.1 Basic Operations of Independent Variables 195
3.2 Classification of Signals 196
3.3 System  198
3.3.1 Continuous-Time and Discrete-Time Systems 198
3.3.2 Time Invariant and Time Variant Systems 198
3.3.3 Linear and Non-Linear Systems 199
3.3.4 Causal and Non-Causal Systems 199
3.3.5 Stable and Unstable Systems 200
3.3.6 Invertible and Non-Invertible Systems 200
3.3.7 Static and Dynamic Systems 200
3.4 Time Domain Representation of Linear Time Invariant System 200
3.4.1 Convolution 201
3.4.2 Differential Equation Representation of LTI System 202
3.4.3 Difference Equation Representation of LTI System 202
3.4.4 Linear Time Invariant System with Random Input 203
3.4.5 Types of LTI Systems 203
3.5 Continuous-Time Fourier Series 204
3.5.1 Coefficient Values of Fourier Series  204
3.5.2 Exponential Form of Fourier Series 205
3.6 Discrete-Time Fourier Series 207
3.6.1 Power Density Spectrum of Periodic Signal 208
3.7 Continuous-Time Fourier Transform 209
3.7.1 Properties of Continuous-Time Fourier Transform 211
3.8 Discrete-Time Fourier Transform 216
3.8.1 Properties of Discrete-Time Fourier Transform 216
3.9 Discrete Fourier T­ ransform (DFT) and Fast Fourier ­Transform (FFT) 221
3.9.1 Discrete Fourier Transform (DFT) 221
3.9.2 Fast Fourier Transform (FFT) 222
3.10 Sampling Theorem for Continuous-Time Signal 222
3.10.1 Impulse Train Sampling of a Continuous-Time Signal 222
3.10.2 Zero-Order Hold Sampling 223
3.11 Laplace Transform 223
3.11.1 Inverse Laplace Transform 224
3.11.2 Properties of Laplace Transform 224
3.12 Z-Transform and its Application to the Analysis of LTI System 229
3.12.1 Region of Convergence 229
3.12.2 Properties of Z-Transform 229

FM.indd 20 3/24/2016 12:36:48 PM


CONTENTS          xxi

3.12.3 Inverse Z-Transform 235


3.13 Realisable LTI Systems 237
Important Formulas 239
Solved Examples 243
Practice Exercises 246
Answers to Practice Exercises 249
Solved Gate Previous Years’ Questions 252

SECTION IV: ELECTRICAL MACHINES 267

4 Electrical Machines 269

4.1 Transformer 269


4.1.1 Ideal Transformer 270
4.1.2 Three-Phase Transformer 277
4.1.3 Parallel Operations of Transformers 280
4.1.4 Per Unit System 280
4.2 DC Machines 281
4.2.1 Commutation 282
4.2.2 EMF Induction 282
4.2.3 Type of Connections 283
4.2.4 DC Generator Characteristics 284
4.2.5 Parallel Operations of DC Generators 285
4.2.6 DC Motors 286
4.2.7 Power Stages in DC Generator and Motor 290
4.2.8 Testing of DC Machines 290
4.2.9 Electrical Braking 291
4.3 Three Phase Induction Machines 291
4.3.1 Rotating Magnetic Field 292
4.3.2 Equivalent Circuit  293
4.3.3 Determination of Circuit Parameters 294
4.3.4 Power Flow Diagram of Induction Machine 295
4.3.5 Complete Torque−Speed Characteristics 295
4.3.6 Methods of Starting Three-Phase Induction Motors 297
4.4 Single-Phase Induction Motor 298
4.4.1 Double-Field Revolving Theory 298
4.4.2 Equivalent Circuit of Single Phase Induction Machine 298
4.4.3 Starting Methods for Single Phase Induction Motor 299
4.4.4 Shaded-Pole Motors 301
4.5 Synchronous Machines 302
4.5.1 Principle of Operation 303
4.5.2 Phasor Diagram 303
4.5.3 Open-Circuit and Short-Circuit Tests 303
4.5.4 Zero Power Characteristics and Potier Triangle 304
4.5.5 Generator Voltage Regulation 305
4.5.6 Two Reaction Theory of Salient-Pole Machines (Blondel’s Two-Reaction Theory) 305
4.5.7 Synchronous Motor Power Equation 306
4.5.8 Power Angle Characteristics 306

FM.indd 21 3/24/2016 12:36:49 PM


xxii        CONTENTS 

4.5.9 Synchronizing Power and Synchronizing Torque  307


4.5.10 Power Factor Control 307
4.5.11 Hunting 307
4.5.12 Slip Test 308
4.5.13 Cogging and Crawling 308
4.5.14 Comparison between Synchronous Motors and Induction Motors 308
4.6 Stepper and Servo Motors 309
4.6.1 Stepper Motor 309
4.6.2 Servo Motors 310
Important Formulas 311
Solved Examples 314
Practice Exercises 326
Answers to Practice Exercises 333
Solved Gate Previous Years’ Questions 338

SECTION V: POWER SYSTEMS 369

5 Power Systems 371

5.1 Power Generation Concepts 371


5.1.1 Some Standard Terms used in Power Systems 372
5.2 Transmission Line Parameters 372
5.2.1 Inductance 372
5.2.2 Capacitance 373
5.2.3 Resistance 374
5.3 Transmission Lines 375
5.3.1 Transmission Line Performance 375
5.3.2 Classification of Transmission Line 375
5.3.3 Combination of Networks 378
5.3.4 Ferranti Effect 379
5.3.5 Surge Impedance 379
5.4 AC and DC Transmission Concepts 379
5.4.1 Comparison of AC and DC Transmission 379
5.5 Insulated Cables 381
5.5.1 Construction of Cables 381
5.5.2 Classification of Cables 382
5.5.3 Dielectric Stress in Cables 382
5.5.4 Most Economical Size of Conductor 382
5.5.5 Grading of Cables 383
5.6 Electric Line Insulators 384
5.6.1 Electrical Stresses on External Insulation 385
5.6.2 Types of Insulators 385
5.6.3 Voltage Distribution in Suspension Insulator 385
5.6.4 Electric Field Distribution 386
5.7 Distribution Systems 387
5.7.1 Classification of Distribution Systems 387
5.7.2 AC and DC Distribution Systems 387
5.7.3 Connection Schemes 388

FM.indd 22 3/24/2016 12:36:49 PM


CONTENTS          xxiii

5.8 PU (Per Unit System) 389


5.9 Bus Admittance Matrix 389
5.9.1 Gauss—Seidel Method of Load Flow Analysis 389
5.9.2 Newton—Raphson Method for Load Flow Analysis 390
5.10 Voltage Control 391
5.10.1 Voltage Control by Reactive Power Injection 391
5.10.2 Voltage Control using Tap Changing Transformers 393
5.10.3 Voltage Control by Combination of Tap Changing Transformer and Reactive
Power Injection 393
5.11 Frequency Control 394
5.12 Power System Stability Concepts 394
5.12.1 Power-Angle Characteristics 394
5.12.2 Swing Equation 394
5.12.3 Equal-Area Criterion 395
5.13 Power Factor Correction 399
5.13.1 Power Factor Correction Formula 400
5.14 Symmetrical Components 400
5.14.1 Symmetrical Components Transformation 400
5.15 Faults 401
5.15.1 Unsymmetrical Fault Analysis 401
5.15.2 Symmetrical Fault Analysis 404
5.15.3 Open Conductor Faults 406
5.16 Relays 408
5.17 Protection 409
5.17.1 Overcurrent Protection 409
5.17.2 Distance Protection 410
5.17.3 Differential Protection 411
5.18 Circuit Breakers 411
5.18.1 Arc Phenomenon 412
5.19 High Voltage Direct Current Transmission 412
5.19.1 Comparison of HVAC and HVDC 413
5.19.2 Corona 413
5.20 Economics of Power Generation 414
Important Formulas 414
Solved Examples 419
Practice Exercises 434
Answers to Practice Exercises 441
Solved Gate Previous Years’ Questions 448

SECTION VI: CONTROL SYSTEMS 485

6 Control Systems 487

6.1 Fundamentals 487


6.1.1 Reasons for Building Control Systems  488
6.1.2 Classification of System 489
6.1.3 Modeling of Physical Systems 491
6.1.4 Effects of Negative Feedback 495

FM.indd 23 3/24/2016 12:36:49 PM


xxiv        CONTENTS 

6.1.5 Sensitivity of Control Systems 495


6.1.6 Model Simplifications using Block Diagram Algebra 496
6.1.7 Signal-Flow Graphs 498
6.2 Time-Response Analysis 499
6.2.1 Transient State Behaviour 499
6.2.2 Steady-State Behaviour 501
6.2.3 Transient Response of Second-Order Systems 504
6.2.4 Nature of System Response vs. Location of Closed-Loop Poles 505
6.3 Stability of Control Systems 507
6.3.1 Bounded Input Bounded Output Stability 507
6.3.2 Routh−Hurwitz (RH) Stability Criterion 507
6.4 Root Locus Technique 509
6.4.1 Concepts of Root Locus − Intuitive Example 509
6.4.2 Rules of Sketching Root Locus 510
6.4.3 Systems with Time Delay 513
6.4.4 Root Locus for System with Gain −K 514
6.5 Frequency Response Analysis 514
6.5.1 Polar Plots 515
6.5.2 Closed-Loop Poles and Zeros of a Negative Feedback System 516
6.5.3 Principle of Argument 516
6.5.4 Nyquist Contour 517
6.5.5 Nyquist Stability Criterion 519
6.6 Bode Plots 520
6.6.1 Advantages of Bode Plots 520
6.6.2 Measures of System Performance in Frequency Domain 523
6.7 Compensators 525
6.7.1 Proportional (P) Control 525
6.7.2 Integral (I) Control 526
6.7.3 Proportional plus Integral (PI) Control 527
6.7.4 Proportional plus Derivative (PD) Control 527
6.7.5 Proportional Integral Derivative (PID) Control 527
6.7.6 Lead Compensator 528
6.7.7 Phase Lag Compensator 529
6.7.8 Realisation using Op-amps 530
6.8 State Space Representation 531
6.8.1 Concept of State 531
6.8.2 Transforming Differential Equation Representation into State Space Representation 532
6.8.3 Time Response from State Representation 533
6.8.4 Obtaining Transfer Function Representation from State Space Representation 533
6.8.5 State Transition Matrix 533
6.8.6 Similarity Transformations 533
6.8.7 Canonical Forms 533
Important Formulas 535
Solved Examples 538
Practice Exercises 552
Answers to Practice Exercises 565
Solved Gate Previous Years’ Questions 576

FM.indd 24 3/24/2016 12:36:49 PM


CONTENTS          xxv

SECTION VII: ELECTRICAL AND ELECTRONIC MEASUREMENTS 613

7 Electrical and Electronic Measurements 615

7.1 Classification of Measuring Instruments 615


7.1.1 Indicating Type Instruments 616
7.2 Types of Indicating Instruments 616
7.2.1 Permanent Magnet Moving Coil Instruments 617
7.2.2 Moving Iron Type Instruments 618
7.2.3 Electrodynamic Type Meters 618
7.2.4 Measurement of High-Frequency Signals 619
7.3 Bridges and Potentiometers 619
7.3.1 Measurement of Inductance 620
7.3.2 Measurement of Capacitance 623
7.3.3 Measurement of Mutual Inductance 625
7.3.4 Measurement of Resistance 626
7.4 Measurement of Current and Voltage 630
7.4.1 Shunts and Multipliers 630
7.4.2 Sensitivity of Ammeter and Voltmeter 631
7.5 Measurement of Power and Energy 631
7.5.1 Measurement of Power in AC Circuits 631
7.5.2 Measurement of Power in a Three-Phase Circuit  633
7.6 Instrument Transformers 636
7.6.1 Current Transformer 636
7.6.2 Voltage Transformer 637
7.6.3 Linear Variable Differential Transformer (LVDT) 638
7.6.4 Errors in Instrument Transformers 638
7.7 Digital Voltmeters and Multimeters 639
7.8 Cathode Ray Oscilloscope 639
7.8.1 Working of CRO  640
7.8.2 CRO Controls 641
7.8.3 Measurements of Voltage 641
7.8.4 Measurement of Frequency 642
7.8.5 Measurement of Phase Difference 643
7.9 Q-Meter 643
7.10 Transducers 644
7.10.1 Strain Gauge 644
7.10.2 Velocity Transducers 644
7.10.3 Temperature Sensing Devices 645
7.10.4 Transducers for Fluid Flow Measurement 645
7.10.5 Capacitive Transducer 646
7.10.6 Piezo-Electric Transducer 646
7.10.7 Bourdon Tube 647
7.10.8 Bellows 647
7.11 Error Analysis 647
7.11.1 Types of Errors 647
7.11.2 Mean Value and Deviation 648
7.11.3 Precision and Calibration 648

FM.indd 25 3/24/2016 12:36:49 PM


xxvi        CONTENTS 

Important Formulas 648


Solved Examples 651
Practice Exercises 658
Answers to Practice Exercises 663
Solved Gate Previous Years’ Questions 668

SECTION VIIIA: ANALOG ELECTRONICS 685

8 Analog Electronics 687

8.1 Semiconductor Physics 687


8.1.1 Insulators, Semiconductors and Metals 687
8.1.2 Types of Semiconductors 688
8.1.3 Donor and Acceptor Impurities 688
8.1.4 Drift and Diffusion Currents 689
8.1.5 Mass Action Law 690
8.1.6 Energy Band Gap 691
8.1.7 Mobility 692
8.1.8 Hall Effect 692
8.2 Semiconductor Diodes 693
8.2.1 The Equilibrium p-n Junction Diode 693
8.2.2 Operational Characteristics 694
8.2.3 Voltage-Current (V-I ) Characteristics of a Diode 696
8.2.4 Diode Resistance 697
8.2.5 Diode Junction Capacitance 698
8.2.6 Diode Equivalent Circuits 700
8.2.7 Breakdown Diodes 701
8.3 Transistors 702
8.3.1 Bipolar Junction Transistor 703
8.3.2 Load Line and Modes of Operation 703
8.3.3 Transistor Configurations 705
8.3.4 Transistor as Amplifier 713
8.4 Field-Effect Transistor (JFET and MOSFET)  715
8.4.1 Junction Field Effect Transistors 715
8.4.2 MOSFETS 720
8.5 Simple Diode Circuits 729
8.5.1 Connecting Diodes in Series and Parallel 729
8.5.2 Clippers 730
8.5.3 Clampers 732
8.5.4 Rectifier Circuits 735
8.5.5 Voltage Regulator 741
8.6 Stability and Biasing 743
8.6.1 Stabilisation 743
8.6.2 Transistor (BJT) Biasing 744
8.6.3 Compensation Techniques (Bias Compensation)  747
8.6.4 Bias Stabilisation in FET (JFET and MOSFET) 748
8.7 Amplifiers 751
8.7.1 Single Stage Amplifiers 751

FM.indd 26 3/24/2016 12:36:49 PM


CONTENTS          xxvii

8.7.2 Hybrid Equivalent Model 754


8.7.3 Multistage Amplifiers 757
8.7.4 Differential Amplifier 760
8.7.5 Operational Amplifier 763
8.7.6 Feedback Amplifier 768
8.7.7 Power Amplifiers 768
8.8 Frequency Response 775
8.8.1 Low Frequency Response — BJT Amplifier 776
8.8.2 Low Frequency Response — FET Amplifier 777
8.8.3 Miller Effect Capacitance 777
8.8.4 High Frequency Response — BJT Amplifier 779
8.8.5 High Frequency Response — FET Amplifier 780
8.8.6 Multistage Frequency Effects 780
8.9 Active Filters 781
8.9.1 First-Order Low-Pass Butterworth Filter 781
8.9.2 Second-Order Low-Pass Butterworth Filter 781
8.9.3 First-Order High-Pass Butterworth Filter 782
8.9.4 Second-Order High-Pass Butterworth Filter 782
8.9.5 Band-Pass Filters 783
8.9.6 Band-Stop Filter 783
8.9.7 All-Pass Filter 784
8.10 Oscillators 785
8.10.1 Types of Oscillators 785
8.10.2 Sinusoidal Oscillator 785
8.11 Multivibrators 787
8.11.1 Astable or Free Running Multivibrator 787
8.11.2 Monostable Multivibrator 788
8.11.3 Bistable Multivibrator 789
8.11.4 Schmitt Trigger 790
8.11.5 IC-555 Timer 791
8.12 Sample and Hold Circuits 792
Important Formulas 793
Solved Examples 801
Practice Exercises 824
Answers to Practice Exercises 837
Solved Gate Previous Years’ Questions 847

SECTION VIIIB: DIGITAL ELECTRONICS 877

9 Digital Electronics 879

9.1 Number Systems 879


9.1.1 Types of Number Systems 879
9.1.2 Number Conversions 880
9.1.3 Binary and Hexadecimal Arithmetic 882
9.1.4 Fixed Precision and Overflow 882
9.1.5 Representation of Binary Numbers 883
9.1.6 Binary Codes 885

FM.indd 27 3/24/2016 12:36:49 PM


xxviii        CONTENTS 

9.2 Boolean Algebra 887


9.2.1 Boolean Function 887
9.2.2 Laws of Boolean Algebra 888
9.2.3 Simplification of Boolean Function 889
9.3 Logic Gates 893
9.3.1 Universal Gates 893
9.4 Logic Families 895
9.4.1 Characteristic Parameters 895
9.4.2 Resistor-Transistor Logic (RTL) 896
9.4.3 Diode-Transistor Logic (DTL) 897
9.4.4 Transistor-Transistor Logic (TTL) 897
9.4.5 Emitter Coupled Logic (ECL) Family 899
9.4.6 CMOS Logic Family 900
9.5 Combinational Logic Circuits 901
9.5.1 Arithmetic Combinational Circuits 901
9.5.2 Multiplexers 907
9.5.3 Demultiplexers 908
9.5.4 Decoders and Encoders 909
9.6 Memory 913
9.6.1 Classification of Memory 913
9.6.2 Read Only Memory (PROM) 913
9.7 Sequential Logic Circuits 913
9.7.1 Flip-Flops 914
9.7.2 Counters 920
9.7.3 Registers 923
9.8 D/A and A/D Converters 926
9.8.1 D/A Converters 926
9.8.2 A/D Converter 928
9.9 Microprocessors 930
9.9.1 Microprocessor Architecture 930
9.9.2 Basic Microprocessor Instructions 931
9.9.3 Assembly Language 934
9.10 8085 Microprocessors 934
9.10.1 Microprocessor Bus Architecture 936
9.10.2 Pin Diagram of 8085 Microprocessor 937
9.10.3 8085 Interrupt Structure 939
9.10.4 Addressing Modes in 8085 939
9.10.5 Machine Cycle 940
9.10.6 Execution of Program by Microprocessor 940
9.11 Memory Interfacing 942
9.11.1 Memory Structure 943
9.11.2 Address Decoding and Memory Mapping 943
Important Formulas 943
Solved Examples 946
Practice Exercises 953
Answers to Practice Exercises 959
Solved Gate Previous Years’ Questions 964

FM.indd 28 3/24/2016 12:36:49 PM


CONTENTS          xxix

SECTION X: POWER ELECTRONICS 981

10 Power Electronics 983

10.1 Power Semiconductor Devices 983


10.1.1 Basic p-n Diode Characteristics 983
10.2 Bipolar Junction Transistor 985
10.2.1 Construction and Operating Principle of BJTs 985
10.2.2 Static Characteristics of BJTs 986
10.2.3 Conduction and Switching Losses of BJT 986
10.2.4 Paralleling of BJTs and Power Darlingtons 987
10.3 Thyristors 988
10.3.1 Construction and Operating Principle of Thyristors 988
10.3.2 Static Characteristics of a Thyristor 990
10.3.3 Thyristors Turn-ON Methods 991
10.3.4 Thyristor Protection 991
10.4 Triac 992
10.4.1 Construction and Operating Principle of Triac 992
10.4.2 Static Characteristics of Triac 993
10.5 Gate Turn-OFF Thyristor (GTO) 993
10.5.1 Construction and Operating Principle of GTO 994
10.6 Power Metal Oxide Semiconductor Field Effect Transistors (MOSFETS) 994
10.6.1 Construction and Operating Principle of Power MOSFETs  995
10.6.2 Static Characteristics of a Power MOSFET 996
10.7 Insulated Gate Bipolar Transistor (IGBT) 997
10.7.1 Construction and Operating Principle of IGBT 997
10.7.2 Static Characteristics of an IGBT 998
10.8 Performance Parameters 999
10.9 Converters: AC-DC Rectifiers  1000
10.9.1 Single-Phase Uncontrolled Half-Wave Rectifier 1000
10.9.2 Single-Phase Uncontrolled Full-Bridge Rectifier with RLE Load 1002
10.9.3 Single Phase Half-Wave Controlled Rectifier with Resistive Load 1003
10.9.4 Single-Phase Half-Wave Controlled Rectifier with Resistive-Inductive Load 1004
10.9.5 Single-Phase Half-Controlled Converter with RLE Load 1005
10.9.6 Single-Phase Fully-Controlled Bridge Converter with RLE Load 1007
10.9.7 Three-Phase Half-Wave Uncontrolled Rectifier 1008
10.9.8 Three Phase Full-Wave Uncontrolled Bridge Rectifier Feeding RLE Load 1009
10.9.9 Three-Phase Half-Controlled Converter Feeding RLE Load  1010
10.9.10 Three-Phase Fully-Controlled Bridge Converter Feeding RLE Load  1011
10.10 DC to DC Conversion 1012
10.10.1 Choppers 1012
10.10.2 Types of DC to DC Converters 1015
10.10.3 Buck or Step-Down Converters 1015
10.10.4 Boost or Step-Up Converters 1015
10.10.5 Buck-Boost Converters 1016
10.11 DC Line Commutation 1017
10.11.1 Commutation with Parallel Capacitance Configuration 1017
10.11.2 Commutation by Another Load Carrying Thyristor 1018

FM.indd 29 3/24/2016 12:36:49 PM


xxx        CONTENTS 

10.12 Inverters 1019


10.12.1 Voltage Source Inverters 1019
10.12.2 Current Source Inverters 1021
10.13 Pulse Width Modulation 1022
10.13.1 Sinusoidal Pulse Width Modulation 1022
10.14 Bidirectional AC to DC Voltage Source Converter 1023
10.15 Speed Control of DC Motor Drives 1023
10.15.1 Pulse Width Modulated Converter for DC Drive System 1024
10.15.2 Closed Loop Control of DC Motors 1025
10.16 Speed Control of AC Motor Drives 1026
10.16.1 Induction Motor Speed Control 1026
10.16.2 Closed Loop Constant Voltage/Constant Frequency Controller 1026
10.16.3 Pulse Width Modulated (PWM) Converter for AC Drive System 1027
Important Formulas 1027
Solved Examples 1032
Practice Exercises 1038
Answers to Practice Exercises 1044
Solved Gate Previous Years’ Questions 1048

Solved GATE 2014 1077


Solved GATE 2015 1129
Solved GATE 2016 1161
Index I1

FM.indd 30 3/24/2016 12:36:49 PM


SECTIONS I & II: ELECTRIC CIRCUITS AND ELECTROMAGNETIC FIELDS

MARKS DISTRIBUTION FOR GATE QUESTIONS

12

10
Number of questions

8
Marks 1
6
Marks 2

4 Total number of questions

0
2009 2010 2011 2012 2013 2014 2015

Chapter 1_part opener.indd 1 3/22/2016 10:34:54 AM


2        CHAPTER 1:  ELECTRIC CIRCUITS

TOPIC DISTRIBUTION FOR GATE QUESTIONS

Year Concepts
2015 Electric and magnetic fields, Network solution and methodology, Sinusoidal steady state
analysis, Laplace transform, Two-port network
2014 Basic concepts, Electric field and potential
2013 Maximum power transfer theorem, Steady state analysis, Electric field and potential
due to point, line, plane and spherical charge distribution, Curl, Gradient, Sinusoidal
steady-state analysis, KVL, KCL, Two-port network, Resonance
2012 KVL, KCL, Network theorem, Transient and steady state analysis, Filters, Maximum
power transfer theorem, Thevenin/Norton theorems, Two-port network, AC
fundamentals, Curl and Vector
2011 RLC parallel circuit, Filters, AC circuit, Maximum power transfer theorem, Phasor
diagrams, Capacitor, Vector
2010 Transient response, KVL, KCL, AC fundamentals, Two-port network
2009 Circuit analysis, KVL, KCL, Electric circuit and transformer, Sinusoidal steady state
analysis, Network theorems

Chapter 1_part opener.indd 2 3/22/2016 10:34:54 AM


CHAPTER 1

ELECTRIC CIRCUITS

An electric circuit is an interconnection of electrical ele- Consider the following network elements (Fig. 1.1),
ments. In this chapter, we will study basic concpets of R
electrical circuits, fundamental laws that govern elec- ≅
tric circuit, methods of circuit analysis, network topol-
ogy, circuit theorems, first order RL and RC circuit, l
Laplace transform, second order RLC circuits, sinusoi- Figure 1.1 |   Conditions for network theory to be valid.
dal steady state analysis, magnetically coupled circuits,
frequency response and resonance, voltage and current Here, l <<< l, where
3 × 108
l =
sources, three-phase circuits AC power analysis two port
m/s
networks. f
If the distributed element length is very much less than
1.1  CIRCUIT CONCEPTS AND LAWS the wavelength (l) corresponding to the maximum fre-
quency of excitation, then the distributed parameters
are approximated into lumped parameters. The inter-
Network is a combination of elements and it may or may connection of such lumped parameters (elements) is
not consist of a closed path. Circuit is a combination of called electrical network. At higher frequencies we
elements and should consist of closed path. cannot construct the lumped electric circuit, and hence
the network theory is not applicable. The network
Now, under what conditions is the network or circuit theory is valid only at low frequencies, that is, upto
theory valid? 1 MHz frequencies only. For above 1 MHz frequencies,

Chapter 1 Theory 1.indd 3 3/21/2016 11:15:59 AM


4     Chapter 1:  ELECTRIC CIRCUITS 

we use field theory. The field theory approach of solv- velocity called drift velocity is defined. It is an aver-
ing the electrical network is valid for all frequencies age velocity of all the free electrons present with in
starting from zero onwards. a conductor given by vd = mE m/s where m is the
The flow chart for use of field theory and circuit mobility of free electron (m2/V-s) in a conductor and
theory for solving problems related to complicated elec- E is the applied electrical field. The direction of force
trical networks is depicted in Fig. 1.2. will be opposite to the field direction because the free
electrons have a negative charge. The net momentum of
1. Exact charge will exist and will be opposite to the direction of
Field theory
2. Complicated the field.
approach
3. Too many
(Ohms’s law In the presence of an electric field, all the free
variables
and Maxwell’s
(E, H, D, J) electrons will have the drift velocity. So that the
equations)
4. Distributed circuit kinetic energy (K.E.) associated with each electron
Complicated
electrical is given by
network
1
1. Approximate K.E. = m v2 Joules
Circuit theory 2. Simple 2 e d
approach 3. Less number The mass of electron m = 9.1 × 10−31 kg and me = effective
mass of the electrons where me > m.
(Ohm’s law, of variables
Kirchoff’s law) (V, I)
4. Lumped circuit In the presence of electric field, each electron will
appear to have reduced mass. So we will consider the
Figure 1.2 |   Application of field theory and circuit effective mass of the mobile electrons in the above
theory on analysis of electrical network. equation (me).
In the absence electric field (E), drift velocity is zero
1.1.1 Electrical Quantities and Units and hence the kinetic energy is zero. So, the total energy
(W) of electron is equal to the potential energy.
1.1.1.1 Ohm’s Law
T.E. = P.E. + K.E.
Before, the discussion on Ohm’s law, let us recapitulate W = P.E. Joules
the mechanism of energy flow through the conductor. Since the conductor is an open circuit at room tem-
The conductivity (s) of conductors is attributed to the perature (27°C or 300°K), the free electrons will acquire
presence of free electrons and their mobility through the thermal energy and they begin to move in different
the conductor. In the absence of electric field (E), there direction in a random manner. Hence the net momentum
is no net momentum of electrons in a conductor so or net flow of free electrons in any direction is zero. So
there is no net current, that is i = 0. In the presence the net flow of charge is zero and the current zero
of axial electric field (E), force is exerted on the free
dq
electrons, that is =0⇒i=0
  dt
F = E ⋅e
also J = 0 that is, when E = 0 ⇒ J = 0.
Here, e = —1.602 × 10 C and net charge Q = ne, where n
—19
The time rate of flow of electric charges is defined as
is the number of free electrons. Figure 1.3 shows the flow of an electrical current (i), that is
electrons under the influence of an electric field through the
d(Q)
conductor with length l and cross-sectional area A. i= Ampere
dt
s Since Q is negative, so the current will flow in the oppo-
⊕ ⊕ ⊕ ⊕ site direction to that of electron motion, that is, in the
E A ⊕ ⊕ ⊕ ⊕ direction of the applied electric field (E).
⊕ ⊕ ⊕ ⊕ Current density (J) is defined as the current per unit
l cross-section area and is given by
Figure 1.3 |   Conductivity under the influence of an 
 i
electric field. J = A/m2
A
In the presence of electric field, different free elec- Since A is a scalar, the direction of J is in the direction
trons will move with different velocities. But only one of the current, that is, in the direction of E.

Chapter 1 Theory 1.indd 4 3/21/2016 11:16:08 AM


1.1  CIRCUIT CONCEPTS AND LAWS     5

According to the Ohm’s law in the field theory form We have from Eq. (1.1) that
or point form, there exists a linear relation between the J = sE
current density (J) and the electric field (E), that is
= s.
i V

J ∝ E (under constant temperature) A l
   l 
J = sE  V = 
 s ⋅ A 
(1.1) i (1.3)

where s = conductivity of material = constant (Ωm)−1. Substituting for V from Eq. (1.3) in Eq. (1.2), we get
l
W
Therefore, power dissipation
Resistance R =
s ⋅A
W
J ⋅ E = (sE ) ⋅ E = sE 2  3   The resistivity of the material is defined as
 m 

r = W-m
1
The characteristic curve for J-E is shown in Fig.1.4. s
J If temperature, length of the conductor and surface area
increases, l/A is almost constant.
s = Slope When the conductivity decreases, resitance increases,
−E
hence for a conductor
0 E RT = R0 (1 + at)
s
Hence, temperature coefficents (a) is positive.
−J
Another form of Ohms law is
Figure 1.4 |   J-E Characteristic curve.
i = G ⋅ V Ampere  (1.4)

The limitations in the description of conductivity and
Ohms’ law based on network (field) theory are: where G is the conductance, expressed in Siemens or

 mho = ; s is conductivity, expressed in (W - m)−1 =
1
  1.  The Ohm’s law is valid when the proportionality
R m
constant (s) is kept constant, that is, the tem-
or Siemens m.
pearture is kept constant otherwise temperature
increases as s decreases. As           i =
dq
  2.  At a constant applied field, E, as the temperature dt
increases the free electrons will acquire extra ther- Therefore,
mal energy, which leads to the increase in collisions dq
and hence the mobility (m) falls, so conductivity V = R⋅  (1.5)
(s) decreases. dt
  3. At a constant temperatue, as E increases, the colli-
sions between the free electrons and the positive ions 1.1.1.2 Power and Energy
(larger in size) increases, which leads to the fall in the
drift velocity and hence loss in K.E. This lost energy The rate of change of energy is called power
is dissipated in the form of heat, which results in a dW dW dq
voltage drop (V) across the conductor. The amount P = = . = Vi Watts
dt dq dt
of power dissipated within the conductor is
 A  V  Using Eq. (1.2), we have
P = JE  2    = sE 2 W/m3
 m   m  P = i2R
To overcome these, we define Ohm’s law in circuit V2
P = W
theory, which states that the voltage across many types R
of conducting materials is directly proportional to the Energy is given by
dW = P .dt
current flowing through the element (material).

V ∝ i ⇒ V = Ri Volts 
(1.2) W = ∫ P ⋅ dt Joules
where the constant of proportionality R (resistance) is The energy associated with resistor is
constant, that is, temperature is constant.
∫ i R ⋅ dt
2
ER = Joules

Chapter 1 Theory 1.indd 5 3/21/2016 11:16:32 AM


6     Chapter 1:  ELECTRIC CIRCUITS 

1.1.1.3 Electric Parameters dy di


VL = = L⋅ Volts (1.7)
dt dt
Resistor (R) The current across the inductor is
We have from Ohm's law that V = iR where R is the t


1
resistance. iL = V ⋅ dt Amperes
L
The convention for representing current voltage and −∞

resistor is shown in Fig. 1.5(a) and the voltage-current Power is given by


characteristics are depicted in Fig. 1.5(b). P = Vi
i From Eq. (1.6)
V = Ri Volts
+ V −
di di
P = L⋅ ⋅ i = Li Watts (1.7)
dt dt
R Energy is given by
V = R (−i) Volts
+ V − i

W = ∫ P ⋅ dt (1.8)
(a) From Eq. (1.7)
 di 
V W = ∫ Li  dt  ⋅ dt Joules (1.9)
2
R= V Comparing Eqs. (1.8) and (1.9), we have
1
−i −2 −1
i
d 1 
=  Li2 
0 1 2 di
−1
i P = Li ⋅

dt dt  2 
−2
R = −V
−i −V Energy of inductor
d 1 2

(b) WL = ∫ P ⋅ dt = ∫ dt  2 L ⋅ i  ⋅ dt

Figure 1.5 |   (a) Voltage-current relation and 1
(b) Voltage-current characteristics. WL = L ⋅ i2 Joules
2
From the current-voltage characteristics we can see that Figure 1.7 shows the characteristic current and flux curve
the resistor is a passive, linear, bilateral and time invari- for the inductor. Thus, the energy stored in an inductor
ant element. at any instant depends on the current through the induc-
tor at that instant. From the characteristic curve, we
Inductor (L) can see that inductor is a linear, passive, bilateral and
When conductor is bound in the form of coil (Fig. 1.6), it time invariant element.

y
will exhibit an opposition, called inductance.

i
+ −i L = Slope
VL L 0 i

−y
Figure 1.6 |   Inductor.
When a time varying current flows through an inductor, Figure 1.7 |   Current-flux characteristic curve for
inductor.
it produces a time varying magnetic flux. The total flux (y)
produced by it is given by
Capacitor (C )
y = Nf Webers
Figure 1.8 shows the circuit for a capacitor.
where N is the number of turns of coil and f is flux per turn.
iC
The amount of flux produced is proportional to the +
current through the coil VC C

y ∝ i ⇒ y = Li Webers (1.6)

The voltage across the inductor is Figure 1.8 |   Capacitor.

Chapter 1 Theory 1.indd 6 3/21/2016 11:16:50 AM


1.1  CIRCUIT CONCEPTS AND LAWS     7

Current across the capacitor is So the voltage-current relation in an inductor is linear.


dq Hence, the relation
iC =
dt di
V = L⋅
q = C ⋅V
 (1.10)
dt
dV
iC = C . Amperes is another form of Ohm’s law.
dt
Voltage across the capacitor For a capacitor:
t dV

1 i=C
VC = i dt Volts dt
C
−∞ Here, i1 ← V1 and i1 ← V2
Power in the capacitor
Therefore, i1 + i2 ← V1 + V2
dV dV
P = Vi = C ⋅ ⋅ V = CV Watts So the voltage-current relationship is linear in a
dt dt
capacitor. Hence, the relation
Energy of the capacitor is given by
dV
 dV  i=C⋅
C ⋅ V   dt = CV 2 Joules
 (1.11)

1
 dt 
W = dt
2
is yet another form of Ohm’s law.
The energy stored in the capacitor at any instant depends
on the voltage across the capacitor at that instant. The Comparison of different electrical paramters in circuit
charge-voltage characteristic curve is shown in Fig. 1.9. and field theory is given in Table 1.1.

Table 1.1 |   Electric parameters in circuit and field


From the characteristic curve, we can see that capacitor
is a linear, passive bilateral and time invariant element.
theory
q
Parameter Circuit   Field Theory
C Theory
V V
R P = i2R Watts P = sE 2 W/m3

mH 2 (J/m3 )
C 1 2 1
L E= Li Joules WH =
−q 2 2
Figure 1.9 |   Charge-voltage chacateristic curve for a
CV 2 Joules WE = eE 2 (J/m3 )
1 1
capacitor. C E=
2 2
Voltage and Curent Relations in Inductors
and Capacitors Then current and voltage is given by

For an inductor:
di
i= ∫ H ⋅ dt and V = ∫ E ⋅ dt (1.12)
V = L⋅ i i
dt
From the relation given in Eq. (1.12), we can conclude
i1 V1 that:
i2 ® V2   1.  The inductor will store the energy in the mag-
di1 netic field.
V1 = L ⋅   2.  The capacitor will store the energy in the elec-
dt
tric field.
di2
V2 = L ⋅
dt
1.1.2 Types of Circuits
d(i + i2 )
V = L⋅ 1   1.  Passive circuit: A network will be called passive
dt
di1 di if it cannot generate or amplify energy, that is, if
= L⋅ + L⋅ 2 the energy which it has supplied since the begin-
dt dt ning of time cannot exceed the energy which was
⇒ V = V1 + V2 fed into it.

Chapter 1 Theory 1.indd 7 3/21/2016 11:17:24 AM


8     Chapter 1:  ELECTRIC CIRCUITS 

t n s R
∫ ∑ [Vi (t )ii (t )]dt ≥ 0
i
E(t) = S @
−∞ i =1
+ V −
l
where E(t) is the difference between the energies
entering and leaving the circuit, that is, the net (a) (b)
energy supplied to the network. If E(t) equals the Figure 1.11 |   (a) Lumped and (b) distributed
energy stored in circuit then the circuit is neither elements.
generating, nor losing energy. Such a passive circuit   8.  Bilateral and unilateral: An element is said to
is called a loss-less network. Otherwise, the passive be bilateral, if the voltage-current relationship is
circuit is lossy. For example, resistor, capacitor, the same for current flowing in either direction.
inductor, diode, bulb, transformer. Resistor, capacitor and inductor are the examples
  2.  Active circuit: If E(t) < 0, the circuit is active, of bilateral elements. An element is said to be uni-
such a circuit can supply excess energy. In other lateral, if the voltage-current relationship is dif-
words, when the elements are capable of deliver- ferent for two directions of current flow. Diode is a
ing energy independently for a long time (approxi- unilateral element.
mately infinite time) or when element is having
Note: In the linear-time invariant case, resistors, capac-
property of internal amplification then element
itor and inductors are passive if and only if R ≥ 0, C ≥ 0,
is called as active element. For example, voltage
L ≥ 0; otherwise active, that is, if R < 0, C < 0 and
sources, current sources, transistors, op-amps.
L < 0. If the ratio of voltage to current at any point
  3. Linear circuit: A network is called linear if the
on characteristic curve is negative, then the element
amplitude of the response is always directly propor-
is active; otherwise it is passive. Every linear element
tional to the amplitude of the excitation (Fig. 1.10).
is bidirectional. If the characteristic curve is similar in
t3 opposite quadrants, then the element is bidirectional;
V t2
otherwise, it is unidirectional.
t1
−I
1.1.2.1 V-I Characterictics of Elements
t1 0 I
  1. If the V-I characteristics of an element of Fig. 1.12(a)
t2
t3 −V are as shown in Fig. 1.12(b), then the element is linear,
passive and bilateral.
Figure 1.10 |   Linear element.
I V
  4.  Non-linear circuit: If the response is not pro-
portional to the amplitude of the excitation, the +
circuit is called non-linear. −I
  5.  Time-invariant and time-variant circuits: V ZL 0 I
A circuit is time-invariant if the relation between
its response and its excitation is applied; otherwise, −
the network is time-varying. −V
  6.  Lumped circuit: A circuit can be considered (a) (b)
lumped if the physical dimensions of all its compo-
nents are negligible compared with the wavelength Figure 1.12
of the electromagnetic signal inside the compo-
  2. If the V-I characteristics of an element are as shown
nent. In such a network, since Kirchhoff’s volt-
in Fig. 1.13, then the element is non-linear, passive
age and current relations hold; the current within
and unilateral. For example, diode.
any branch is the same at all points of the branch
between its terminating nodes [Fig. 1.11(a)]. V
  7.  Distributed circuit: If the physical dimensions
of the elements are comparable to the signal wave-
length, the spatial variations of voltages and cur-
rent along the wires and in the components must 0 I
be taken into consideration. In such circuits, called
distributed network, Kirchhoff’s laws are no longer
valid, and the more general laws of Maxwell must
be applied [Fig. 1.11(b)]. Figure 1.13

Chapter 1 Theory 1.indd 8 3/21/2016 11:17:30 AM


1.1  CIRCUIT CONCEPTS AND LAWS     9

  3. If the V-I characteristics of an element are as shown From the above results,
in Fig. 1.14, then the element is non-linear, passive   1.  All passive elements need not be linear elements.
and bilateral.   2.  All linear elements need not be passive elements.
  3.  All linear elements are always bilateral, but con-
V
verse need not be true.

−I
1.1.3 Current and Voltage Sources
0 I
There are two types of energy sources:
−V   1.  Independent sources
•• V (ideal, practical)
Figure 1.14 •• I (ideal, practical)
  4. If the V-I characteristics of an element are as shown   2.  Dependent sources
in Fig. 1.15, then the element is non-linear, active •• Voltage-controlled voltage source (VCVS)
and unilateral. •• Voltage-controlled current source (VCCS)
•• Current-controlled voltage source (CCVS)
V •• Current-controlled current source (CCCS)

−I 1.1.3.1 Independent Sources


0 I
Ideal Voltage Source

−V Ideal voltage source delivers energy at specified volt-


age (VS), which is independent of load current. Internal
Figure 1.15 resistance of ideal voltage source is zero [Fig. 1.18(a)]. In
an ideal voltage source, the terminal voltage is indepen-
  5.  If the V-I relation across a resistor is I = 2V 2 as
dent of the terminal current so the source will be there
shown in Fig. 1.16, then the element is non-linear,
like a non-linear element [Fig. 1.18(b)].
active, unilateral. For example, bipolar junction
transistor.
I
V Ideal
I

VS + VS
− V ZL
V = VS
−V
V
I
(a) (b)
I
Figure 1.18 |  Ideal voltage source place. (a) Circuit.
Figure 1.16
(b) V—I characteristics.
  6. If the V-I characteristics of an element are as shown
in Fig. 1.17, then the element is linear, active and
bilateral. Practical Voltage Source
Practical voltage source delivers energy at specified volt-
V age (V), which depends on current delivered by source
[Fig. 1.19(a)], therefore,

−I V = VS − IR S
0 I
If I increases, V decreases as shown in Fig. 1.19(b). Note
that the terminal voltage is a function of the terminal
−V current. Now, when I = 0, V = VS. So, the current
through a passive element can be zero so that two voltages
Figure 1.17 are equal and vice versa [Fig. 1.19(c)].

Chapter 1 Theory 1.indd 9 3/21/2016 11:17:38 AM


10     Chapter 1:  ELECTRIC CIRCUITS 

RS I V I
VS IS
VS +
− V Load
0 I 0 V
(a) (b) (b)

I
I R +

+
VS - V + 0A
VS IS 0 V RS V=0

(c) −
Figure 1.19 |   Ideal voltage source. (a) Circuit. (b) V-I (c)
characteristics.(c) Current through Figure 1.21 |   Practical current source. (a) Circuit.
passive element. (b) V—I characteristics. (c) Current flow-
Ideal Current Source ing through minimum resistance path.
Ideal current source delivers energy at specified current
(IS), which is independent of voltage across the source 1.1.3.2 Dependent Sources
[Fig. 1.20(a)]. Internal resistance of ideal current source
is infinite. The terminal current is independent of termi- All the controlled sources shown in Fig. 1.22 are the linear
nal voltage [Fig. 1.20(b)]. control sources since the control variables are the linear
variables. The controlled sources are most of the time
I
I active elements and non-bilateral in nature. For exam-
I = IS Ideal ple, (a) BJT ® CCCS, VCVS; (b) Op-amp ® VCVS;
IS
IS (c) JFET ® VCCS.
V Load

0 V
Figure 1.20 |   Ideal current source. (a) Circuit. + +
K1V1 K2V2 K3I1 K3I2
(b) V—I characteristics. − −
Practical Current Source
Practical current source delivers energy at specified cur-
rent (I), which depends on voltage across the source (a)  VCVS  (b)  VCCS  (c)  CCVS  (d)  CCCS
[Fig. 1.21(a)]. Figure 1.22 |   Controlled sources.
V
−IS + +I = 0 Inherently, the dependent sources are also non-linear in
RS nature; since the voltage and relation is non-linear. But
V
So,   I = IS − the linearity of the controlled sources is defined with
RS respect to the controlled variables if they vary in a linear
manner, their magnitude varies in a linear manner, so
As the terminal voltage increases, I will decrease these called linear controlled variables. All the linear
[Fig. 1.21(b)]. So, the terminal current will be a function of controlled sources are always bilateral (non-linear con-
the terminal voltage. When V = 0 ⇒ I = IS. So, the current trolled sources are unilateral). So, the dependent sources
always chooses a minimum resistance path [Fig. 1.21(c)]. are linear, active and bilateral elements. The presence of
I I these elements in the network makes it an active, linear
and bilateral network.

IS RS V Problem 1.1: The V-I characteristics of a network of


Fig. (a) is shown in Fig. (b). If a 10 Ω resistor is con-
Load
nected across the terminals a, b of the network, then
the terms V and I are, respectively,
(a)

Chapter 1 Theory 1.indd 10 3/21/2016 11:17:52 AM


1.1  CIRCUIT CONCEPTS AND LAWS     11

V i(t)
RS I 5
a
+ −2 0 3
VS + I
− V
− −20
b 0 3 4 6 7 t(µs)

Figure (a) Figure (b) (a) 12.5 μs  (b) 13 μs  (c) 14.5 μs  (d) 15 μs

Solution: When V = 0 V, I = −2 A, we have


Solution: We know that
dq(t)
RS I i(t) =
dt
5
+ q(t) = ∫ i(t) . dt = ∫ i(t)dt
VS − 0
3 4 5

V = VS − IRS = ∫ i1(t)dt + ∫ i2 (t)dt + ∫ i3 (t)dt


0 = VS − (−2) RS
0 3 4
   
= × 3 × 5 + 1 × 3 + × 1 × 2 + 1 × 3 + × 1 × 1
1 1 1
 VS = −2RS (i) 
    
2 2 2
When I = 0, A V = −20 V, we have =
15
+ 4 + = 15 μs
7
2 2 Ans. (d)
RS
a
+
VS − V Problem 1.3: The current through 10 Ω resistor is
shown in the following figure. Determine the power
b
dissipated in the resistor.
   VS = V
i(t)
  Substituting in Eq (i),VS = −20 V (ii)
  we have        RS = 10 Ω 10 A
Now considering the following circuit, we have

10 Ω I
a 0 1s 2s 3s t
+ Solution: Average power is given by
VS +
− V 10 Ω

Energy absorbed in one period
Pavg =
period
T
b
∫ ∫ i (t)Rdt = Irms ⋅ R
1
ER = i2 (t) ⋅ Rdt = 2 2
T
−20
0
VS V
= −1 A
Therefore,
I= = S =
10 + 10 20 20 (10 − 0)
i(t) = ⋅ t = 10t 0 ≤ t ≤ 1 0≤t≤1
(1 − 0)
So V = I × 10 Ω = −10 V 1

∫ (10t) ⋅ 10dt
2
ER =
Ans. (−10, −1)
0
1
t3
= 1000
3
Problem 1.2: Current I(t) is passing through a capac- 0
 1  1000
= 1000   =
itor as shown in the following figure. Determining
charge across the capacitor, up to the first 5 μs is  3 
J
3

Chapter 1 Theory 1.indd 11 3/21/2016 11:18:16 AM


12     Chapter 1:  ELECTRIC CIRCUITS 

Therefore, 1000 4

∫6 ⋅ 1 dt = 36 × 2 = 72 J
2
1000 E R2 =
Pavg = 3 = W 2
1 3
Peak power = (10)2 . 10 = 1000 W  di 
2

Ans. (1000)
EL1 = ∫ L ⋅ i  dt  ⋅ dt
0
2 2 2
t2
Problem 1.4: When a resistor R is connected to a cur- = ∫ 2 .3t ⋅ 3 .dt = 18 ∫ t ⋅ dt = 18
2
= 18 × 2 = 36 J
rent source it consumes a power of 18 W. When the 0 0 0

 di 
4

∫ L ⋅ i  dt  dt
same resistor R is connected to a voltage source whose
magnitude is same as the current source, the power EL2 =
absorbed is 4.5 W. The values of current source and 2

R are____________. 4
= ∫ 2 ⋅ 6 ⋅ 0 .dt = 0 J
Solution: Consider the circuits shown in Fig. (a) for 2
R connected to current source and Fig. (b) for R con- Therefore,
necte to voltage source.
E =ER 1 + E R 2 + E L1 + E L 2 = 24 + 72 + 36 + 0 = 132 J
Ans. (132)
iS VS +

R R
1.1.4 Kirchoff’s Laws
   
Figure (a) Figure (b) Kirchhoff’s laws are valid only in lumped circuits, and
at constant temperature, Ohm’s law is also valid. Some
Given that iS = VS
features of these laws are as follows:
Also,
  1.  The Ohm’s law is defined across an element, and
iS2R = 18 W
that element can be lumped (R, L, C) or distrib-
V2
= 4. 5 W uted element (J = sE); whereas Kirchhoff’s laws
R are valid only for lumped electric circuits.
= 4W
18 180
R2 = =   2.  The Kirchhoff’s laws are independent of the nature
4.5 45 of the elements; whereas the Ohm’s law is a func-
  R = 2 Ω tion of the nature of the elements.
18
iS2 = = 9 ⇒ iS = 3 A   3.  The Ohm’s law is applicable for the active elements
2 like sources (i.e., generators). Since the voltage and
Ans. (3, 2) current relation is non-linear, Kirchhoff’s laws are
applicable only for the linear and passive elements
Problem 1.5: The given figures show the inductor (R, L, C).
and the waveform of the current passing through an   4.  Kirchhoff’s laws and the lumped element models
inductor of resistance 1 ohm and inductance 2 H. The are valid only when the length of distributed ele-
energy absorbed by the inductor up to 4 s is ment is much less than the wavelength correspond-
ing to the frequency under consideration.
i(t)

6A 1.1.4.1 Kirchoff’s Current Law

For any lumped electrical network, for any of its node’s


and at any time, the algebraic sum of all the branch cur-
0 2 4 t (s)
rents entering (or leaving) the nodes is zero. A simple
Solution: Given that R = 1 Ω; 0 ≤ t ≤ 2 seconds node is one having an interconnection of only two
2 branches. The principle node is an interconnection of
∫ (3t) dt
2
E R1 = at least three branches.
0
2
For the segment of circuit, shown in Fig. 1.23, the
9×8
2
t3

Kirchhoff’s current law (KCL) is expressed as
= 9 t2 ⋅ dt = 9 = = 24 J
0
3
0
3 −i1 − i2 + i3 + i4 + i5 = 0

Chapter 1 Theory 1.indd 12 3/21/2016 11:18:36 AM


1.1  CIRCUIT CONCEPTS AND LAWS     13

i2 According to KCL
i1 i3 I1 + I2 + I3 + I 4 + 5 = 10
Using Ohm’s law
i5 i4 V V V V
I1 = , I2 = , I3 = , I 4 =
5 10 2 1
Figure 1.23 |   Branching current at the node. Therefore,
Also, it implies V V V V
+ + + + 5 = 10
  1.  i1 + i2 = i3 + i4 + i5 5 10 2 1
  2.  q1 + q2 = q3 + q4 + q5 1 1 1 
V  + + + 1 = 5
  3.  n1 + n2 = n3 + n4 + n5  5 10 2 
Some important points regarding KCL are as follows:
V = 2.78 Volts
  1.  KCL applies to any lumped electric circuits, it does
not matter whether the circuit elements are linear The voltage across 10 Ω resistor is 2.78 V and current
and non-linear, active and passive, time-varying passing through it is
and time-invarying, etc. V 2.78
I2 = = = 0.278 A
  2.  KCL is independent of the nature of the elements 10 10
(Fig. 1.24). Ans. (2.78, 0.278)

+
Problem 1.7: If R = 10 Ω, find V2 in figure given below.
2A

+ iL − + −
20 Ω V1 R V2 10 Ω
+ iR
+

iD
100V 20 Ω 30 Ω 20 Ω
iC

Figure 1.24 |   KCL independent of elements.


Applying KCL to circuit shown in Fig. 1.24, we get (a) 22 V  (b) 23.4 V  (c) 10 V  (d) 21.4 V
−2 − iL + iB + iC + iD = 0
⇒ 2 + iL = iR + iC + iD
Solution: Apply nodal analysis and KCL at node 1,
we have
  3. The current through a branch measures the rate at
V1 − 100 V1 V1 − V2
which the electric charges are flowing through the + + = 0 (i)
branch. KCL expresses that charges do not accumu- 20 20 10
late at any node, or charge is conserved at every node.
Applying KCL at node 2, we have
Note: The length of antenna is above the quarter of l corre-
sponding to the frequency of the operation, hence, it is not a V2 V2 V2 − V1
+ + = 0 (ii)
lumped element, therefore, KCL is not applicable to antenna. 30 30 10

Problem 1.6: For the circuit shown in figure below, Solving Eq. (i), we have
find the voltage across 10 Ω resistor and current pass- V1 - 100 + V1 + 2V1 + 2V2 = 0
ing through it. ⇒ 4V1 − 2V2 = 100
From Eq. (ii), we get
V2 + V2 + 3V2 − 3V1 = 0
5W 10 W 10 A 2W 1W 5A
−3V1 + 5V2 = 0
 4 −2  V1  100
   =  
−3 5 V2   0
Solution: Assuming voltage V at the node A,
V
100 −2  4 100
∆1 =   = 500, ∆2 =   = 300 and
I1 I2 I3 I4  0 5 −3 0
5Ω 10 Ω 10 A 2Ω 1Ω 5A  4 −2 
∆=  = 20 − 6 = 14
−3 5

Chapter 1 Theory 1.indd 13 3/21/2016 11:18:58 AM


14     Chapter 1:  ELECTRIC CIRCUITS 

We have consists of finding a new circuit that provides a desirable


response to a given input excitation. The concept of anal-
∆ 500 ∆ 300
V1 = 1 = = 35.7 V and V2 = 2 = = 21.4 V ysis and synthesis is depicted in Fig. 1.25.
∆ 14 ∆ 14
Ans. (d) Known
excitation Known Response = ?
1.1.4.2 Kirchoff’s Voltage Law circuit
In a lumped electric circuit, for any of its loops and
at any time, the algebraic sum of the branch voltages Known Desired
around the loop is zero. This statement is known as excitation response
Kirchhoff’s voltage law (KVL). Circuit = ?
Some important points regarding KVL are as follows:

Figure 1.25 |   Analysis and synthesis outcomes.


  1.  KVL is independent of the nature of the elements.
  2.  KVL expresses that the energy is conserved in
every loop.
  1. An analysis problem normally has a solution, by con-
trast, in design a solution may or may not exist at all.
Problem 1.8: Find voltage across 10 Ω resistor in the   2.  Analysis almost always leads to a unique solution,
circuit given below. while if a design problem can be solved, it usually
has several equivalent solutions.
V1 10 Ω V2   3.  Linear circuit analysis uses a few basic meth-
V1 10 Ω
ods, nodal and mesh analyses and state variable
V2
­analysis. Design on the other hand employs a wide
100 A 5Ω 20 Ω 200 A variety of techniques.
100 A 5Ω 20 Ω 200 A
1.2.1 Equivalent Circuits
Ω circuit can be20represented
Solution: The 5given Ω as   1.  Equivalent circuits when i = i1 = i2 are shown in
5Ω 10 Ω 20 Ω Figs. 1.26(a) and (c). Two ideal current sources are
connected in series only when the magnitudes of
500 V 10I1Ω 4000 V the sources are equal otherwise, this would violate
KCL which results in the unstability in system due
500 V 4000 V
I1 to the oscillations.
  2.  Equivalent circuit when V = V1 = V2 are shown
in Figs. 1.26(b) and (d). Two ideal voltage sources
Applying KVL in simple series circuit, we get are connected in parallel only when the magnitudes
500 − 35I1 − 4000 = 0 of the sources are equal, otherwise this would violate
−3500 KVL which results in the unstability in system due
I1 = = 100 A to the oscillations.
35
Voltage across 10 Ω = 1000 V
Ans. (1000) i2
≅ i ≅ i
i i1
1.2  CIRCUIT ANALYSIS TECHNIQUES

(a)
Analysis is important in electric domain because it is
  1.  easy to analyse.
  2.  easy to measure the electrical quantities.
V + R ≅V + + + ≅V +
  3.  easy to process the electrical signals. − − − − − only if
  4.  easy to store. V1 V2 V = V1 = V2
  5.  easy to visualise, etc.
Analysis typically involves the calculation of the response
(b)
of a known circuit or system to a given excitation. Synthesis

Chapter 1 Theory 1.indd 14 3/21/2016 11:19:09 AM


1.2  CIRCUIT ANALYSIS TECHNIQUES     15

+ I
V1 +
− +
10 V − 5Ω +
i1 i ≅ ≅ + V = V1 + V2
10 V − 10 V
− −
V2 +
= i1 + i2 −
i

(c) The voltage at 5 Ω resistor is V5Ω = 10 V, therefore


current is
− 10
+ I5W = =2A
+ 5 Ans. (2)
− + 10 V −
10 A 10 A ≅ Open − ≅ Short
+ − circuit 10 V + circuit Problem 1.10: Determine the power delivered by the
10 V source in the following circuit.
− − i 2A
+
(d)
+ 5Ω
Figure 1.26 |   Equivalent circuits.
10 V − 10 V
2A −

Problem 1.9: Determine the current through the 5 Ω Solution: The voltage at 5 Ω resistor is
resistor in the following figure.
V5Ω = 10 V
10
+ I5W = = 2A
5Ω
+ + 5
5V − − 10 V
− Then applying KCL, we have
−i - 2 + 2 = 0 ⇒ i = 0
Solution: Consider the following equivalent circuit Thus, the power delivered by the 10 V source
= 10.(0) = 0 Watts
P2A = 2 × 10 = 20 Watts (delivered)
+
5V − +
− 5Ω P5Ω = 22 × 5 = 20 Watts (absorbed)
10 V Ans. (20)

Since there is violation of the KVL in the network, 1.2.2 Series and Parallel Networks for Resistors,
the circuit connection is physically not possible. Capacitors and Inductors
Applying KVL, we get
In a series circuit, the current through the elements is
5 - 10 = 0 ⇒ −5 = 0 the same. In a parallel circuit, the voltage across the ele-
which is not possible. ments is the same.
Now consider the following equivalent circuit 1.2.2.1 Parallel Impedance and Current
Division

5Ω
+ + Two or more circuit elements are said to be in parallel
10 V − − 10 V
if the same voltage appears across each of the elements
(Fig. 1.27).
I1
Here applying KVL, we get Z1
10 + 10 = 0 ⇒ 20 = 0 I
It is also violation of KVL, so physically not I
possible. Z2
I2
The value of current can be determined using the
following circuit. Figure 1.27 |   Parallel circuit elements.

Chapter 1 Theory 1.indd 15 3/21/2016 11:19:23 AM


16     Chapter 1:  ELECTRIC CIRCUITS 

I = I1 + I2 1 VC2
   Z = ⇒ V1 =
IR2 jwC
Z = R ⇒ I1 =
C1 + C2
R1 + R2
VC1
V2 =
IR1 C1 + C2
I2 =
R1 + R2
1
I⋅ Problem 1.11: Determine equivalent capacitance in
1 jwC2 I ⋅ C1
⇒ I1 =
the circuit given below.
Z= =
jwC 1
+
1 C1 + C2 (0V)
jwC1 jwC2 CC
CC CC
I ⋅ C2
I2 = B Y
C1 + C2
CS CC
I jwL2 I ⋅ L2
Z = jwL ⇒ I1 = =
jwL1 + jwL2 L1 + L2
(0V)
I ⋅ L1 Solution: The equivalent circuit is
I2 =
L1 + L2
B Y
1.2.2.2 Series Impedance and Voltage Division CC

Two or more circuit elements are said to be in series if CC CS CC CS


they exclusively share a single node and consequently
carry the same current (Fig. 1.28).
B CC Y
I
Z1 Z2
+ − + −
(CC + CS) (CS + CC)

V
Figure 1.28 |   Series circuit elements.
CC + CS 3CC + CS
CBY = CC + =
V 2 2
I=
Z1 + Z2
Problem 1.12: Twelve 1 Ω resistors are used as edges
V ⋅ Z1
Therefore, V1 = I ⋅ Z1 = to form a cube. The Req seen between the two diago-
Z1 + Z2 nally opposite corners of cube is

VZ2 R
V2 = R R
Z1 + Z2 R
VR1
R = Z ⇒ V1 = R
R
R1 + R2 R
R
VR2 R
V2 = R
R1 + R2 R
VL1 R
 Z = jwL ⇒ V1 =
L1 + L2 I + −
V

W  (b) W  (c) 6 Ω  (d) W


VL2 6 5 5
V2 = (a)
L1 + L2 5 6 12

Chapter 1 Theory 1.indd 16 3/21/2016 11:19:56 AM


1.2  CIRCUIT ANALYSIS TECHNIQUES     17

Solution: The equivalent circuits is 1.2.3 Source Transformation


I/3 R I Source transformation states that an independent (or
dependent) voltage source VS in series with a resistance R
R is equivalent to an independent current source IS = VS/R S
I/6 R R
in parallel with a resistance RS.
I/3
I/6 or
R R
An independent (dependent) current source IS in paral-
R I/3 lel with a resistance RS is equivalent to an independent
R I/6
voltage source VS = IS R in series with a resistance RS.
R I/6 R   1.  It is a simplification technique and is applicable
I/3 I/6 only for practical sources.
I/3 R
  2.  It is impossible to convert an ideal voltage into an
I/6
I/3 R equivalent ideal current source and vice versa.


The two circuits shown in Fig. 1.29 are equal only with
I +
V respect to the performance point of view, but from the ele-
ments and connections point of view, they are not equal.
Applying KVL, we get The source transformation is applicable even for the depen-
dent sources, provided the controlled variables are outside
I I I
V − R− R− R = 0 the branches where the source transformation is applied.
3 6 3 RS
R R R 
V = I  + +  = I
5R
 3 6 3 6
VS
VS +
⇒ = Req = Ω
V 5 − RS
I 6
Ans. (b)

Note: Whenever the inductor and capacitors are pres- RS


ent in the network, an AC source of frequency (w) is
always used.
+
IS ISR −
Problem 1.13: If in the figure shown in Problem 1.12,
if all resistors are replaced by L Henry inductors,
determine the equivalent inductor between to two
diagonally opposite corners of cube. Figure 1.29 |   Source transformation.

Solution: From the corresponding equivalent circuit,


we have Problem 1.14: Simplfy the network between the ter-
minals a and b in the following circuit.
I I I
V −
Z − Z − Z =0 a
3 L 6 L 3 L
5
V = I ⋅ ⋅ ZL 2Ω
6
V 5 1A 2Ω
= ⋅ ZL +

4V
I 6
Note: If capacitors are used in the circuit, we have
6 3Ω 1Ω
Zeq = ZC
5 − +
Note: For DC input, the inductor acts as short
6V + − 3V
circuit. The capacitor will act as an open circuit in
steady state. b

Chapter 1 Theory 1.indd 17 3/21/2016 11:20:10 AM


18     Chapter 1:  ELECTRIC CIRCUITS 

Solution: The simplified circuits are: For an ideal diode Vf = 0 and Rf = 0. So,
a + −

1A 2Ω 2Ω 2A 3A 1Ω + −

2Ω 1Ω
2A 3Ω 1Ω 3

3A 1A
4 + − + −
id
+ +
b 5V − − 2V

a
5 - 2id - 1id - 2 = 0 ⇒ id = 1 A
1Ω Ans. (1)

+ a
3V − 7 Ω a
1.2.4 Nodal and Mesh Analysis
⇒3 4

15 7

Ω + 7
A
The Nodal and Mesh analyses are two network solving tech-

4 15 4
− niques that are applicable only for lumped electric circuits.
3 + 4
V − b b
4 Kirchhoff’s current law (KCL) + Ohm’s law = Nodal
b analysis (98% used).
Kirchhoff’s voltage law (KVL) + Ohm’s law = Mesh
analysis (2% used).
Problem 1.15: Determine the current through the
ideal diode (D) in the following circuit. 1.2.4.1 Mesh Analysis

4Ω D Mesh analysis provides another general procedure for


analyzing circuits using mesh currents as the circuit vari-
id ables. A mesh is a loop that does not contain any other
+
10 V 4Ω 1Ω 2A loop within it. Mesh analysis is applicable for only planar
− circuits. A planar circuit is one that can be drawn in a
plane with no branches crossing one another; otherwise,
it is non-planar. The steps involved in mesh analysis are:
Solution: Since the diode is a non-linear element, the   1.  Identifying the meshes.
network is non-linear and hence only source trans-   2.  Assigning the mesh currents.
formation is applicable (superposition theorem is not   3. Using KVL + Ohm’s laws to write the mesh equations.
applicable).
Applying mesh analysis at mesh 2 in Fig. 1.30 and
D
D assuming i2 >> i1, we get

−VR 3 − V (t) − VR 3 − V2 = 0
id
id
2Ω 1Ω
5
5 A
2Ω 1Ω
2A d
2 A
2
2A −i2R2 − V (t) − i2R3 − L (i − i ) = 0
dt 2 1

V1 C V2 R2
2Ω 1Ω
2Ω 1Ω
D iC iL iR2
D +
i(t) R1 i1 LL i2 − V(t)
+ id + R3
− −
5V + id + 2V
5V − − 2V

Figure 1.30 |   Circuit for mesh analysis.

Chapter 1 Theory 1.indd 18 3/21/2016 11:20:24 AM


1.2  CIRCUIT ANALYSIS TECHNIQUES     19

Problem 1.16: A segment of a circuit is shown in the Solving, we get i3 = 2A, i1 = 9A, i2 = 2.5A
following figure. At VR = 5 V, VC = 4 sin 2t. Find VL. Inside the super mesh, applying KCL, we get
i1 = 7 + i3 - 7 - i3 + i1 = 0
+
i1 - i3 = 7
5 Ω VR
(iii)
2A − So, the power dissipation is
+ − iC
iR
P = (i2 - i3)2 × 3 = (0.5)2.3 ⇒ P = 0.75 W
iC 1 F Ans. (0.75)
+
2 H VL Note: Since the voltage across an ideal current source
− can be of any value, it is not possible to write the
mesh equations for the meshes (1) and (2) indepen-
(a) 16 sin2t V        (b) 32 sin2t V dently. Hence, the supemesh procedure is followed.
(c) 16 cos2t V        (d) 32 cos2t V
Solution: Applying KCL, we get
1.2.4.2 Nodal Analysis
−2A - iR + iL + iC = 0
5 d Nodal analysis provides a general procedure for analys-
−2 − + iL + 1 × (4 sin 2t) = 0
5 dt ing circuits using node voltages as the circuit variables.
−2 − 1+iL + 4 cos 2t(2) = 0   1.  Identify the nodes.
iL = 3 - 8 cos 2t   2.  Assign the node voltages w.r.t. ground node.
  3.  By using KCL + Ohm’s laws, write the equations.
diL
Now, VL = L ⋅ = 2 × (0 − 8(− sin 2t) × 2) = 32 sin 2t
dt Consider the circuit shown in Fig. 1.30 above. Applying
   VL = 32 sin 2t V nodal analysis at node 2, we get
Ans. (b)    iC + iL + iR = 0

V2 − V (t)

d 1
Supermesh C (V2 − V1 ) + V2 dt + =0
dt t R2 + R3
If a current source (independent or dependent) is −∞
common between two meshes, we can create a super-
Note: Whenever the dependent sources are present in
mesh by avoiding the current source and any elements
the network always nodal analysis is used to calculate
connected in series with it.
the responses.
Problem 1.17: Find the power dissipation in 3 Ω resistor.
Supernode
If the voltage source (dependent or independent) is
connected between two non-reference nodes, the two
+ non-reference nodes form a generalised node called the
1 Ω i2 2Ω supernode. Both KCL and KVL are used to determine
− node voltages.
+ 3Ω
7V − i1
Problem 1.18: Determine V1 and V2 in the following
7A +
i3 1Ω circuit.

1Ω
+ 3Ω −

V1 V2
+−
Super
Solution: Applying KCL at mesh 2, we get node
−2i2 - 3(i2 - i3) - 1(i2 - i1) = 0
+ +
(i) 1Ω 1Ω
4A 5V 9A

The combined mesh equations for meshes (1) and (3) are 2 6

(7) (i1 - i2) - 3(i3 - i2) - 1.i3 = 0 (ii)

Chapter 1 Theory 1.indd 19 3/21/2016 11:20:33 AM


20     Chapter 1:  ELECTRIC CIRCUITS 

Solution: Applying KVL, we get Problem 1.20: Voltage Vx and current Ix in the given
circuit are ––––.
V1 V1 − V2 V2 − V1 V2
−4 + + + + −9 = 0
2Ω
4V
1 1 1 1 1 − + Ix
2 3 3 6

−4 + 2V1 + 6V2 - 9 = 0
10 A 4Ω 2 Ω VA
2V1 + 6V2 = 13 (i)

Since the current i through an ideal voltage source


can be of any value; it is not possible to write the
nodal equations at nodes (1) and (2) independently. Solution: Applying KVL at node 1 and using nodal
Hence, a supernode procedure is followed. analysis, we get
Vx Vx + 4
Inside the supernode, applying KVL, we get −10 + + =0
4 2
V1 - 5 - V2 = 0
−40 + Vx + 2Vx + 8
V1 - V2 = 5 (ii) =0
4
Solving Eqs. (i) and (ii), we get
3Vx = 32 ⇒ V x = 10.66 V
V1 = 5.375 V and V2 = 0.375 V
Ans. (5.375, 0.375) The current through concerned branch is
10.66 + 4
Ix = = 7.33 A
2
Problem 1.19: Determine the current i in the circuit. Ans. (10.66, 7.33)

2Ω V + 2Ω −

i Problem 1.21: Determine voltage V2 in the following


+
circuit.
4A 2Ω + 20 V

− 10 Ω V2 V2 10 Ω

60 V + 20 Ω 20 Ω 20 Ω
+ 5V
− −
Solution: Applying nodal analysis

V − 20
−4 + i + =0
2 Solution: Applying KVL, we have

V V − 20 V2 − 60 V2 V2 − 5
−4 + + =0 + + =0
2 2 10 10 10

2V − 20 3V2 = 65 ⇒ V2 = 21.6 V
−4 − =0
2 Ans. (21.6)

V
So, we have V = 14 V ⇒ i =
Note: A register parallel with ideal voltage source is
= 7 A.
2 always negligible, but not in power calculations.
Ans. (7)
Note: A resistor in series with an ideal current source
is always neglected from the nodal analysis. But in Problem 1.22: If R of 10 Ω connected across terminal
the power calculation, it cannot be neglected. A and B as in the following circuit what is the value
of current?

Chapter 1 Theory 1.indd 20 3/21/2016 11:20:53 AM


1.3  TOPOLOGY     21

2Ω V
A 10 + = 0 ⇒ V = 1000
100
2Ω + 5Ω V 1000
− 5V 10 A I=
100
=
100
= 10 A

R Ans. (c)
B
2Ω Problem 1.24: Determine the values of V3 and I for
the following circuit.

(a) 2 A  (b) 3.175 A  (c) 2.075  (d) 1.175 V3 I 2Ω


+ −

Solution: The given circuit can be represented as
A 2Ω + 4Ω
V3 3 Ω + +

4 V3
5Ω −
+ 4A
− 5V
5Ω 50 V

4Ω Solution: The source will have some internal voltage, so


10 Ω B
V3 V3 − (−4V3 )
−4 + + =0
3 2W
V 5 V3
−4 + 3 + =0
VA V V − 5 VA − 50 3 2
+ A + A + =0
10 4 5 7 24
⇒ V3 = Volts
140 VA + 350 VA + 280 VA − 1400 + 200 VA − 10000 17
=0 V + 4V3 5 24 60
1400 ⇒I = 3 = × = A
970VA = 11400 VA = 11.75 2 2 17 17
 24   60 
11.75 Ans.  ,  
I= = 1.175 A  17   17 
10
Ans. (d) Note: The voltage across ideal current source can be
any value and it is decided by the other elements from
the network.
Problem 1.23: Find the current through 100 Ω resis-
tance in the circuit given below.
10 V 1 1.3  TOPOLOGY

The topology is the branch of science which deals with


50 10 100 Ω the study of geometrical properties and spatial relations
4 4 unaffected by the continuous change of shape or size
of figures. Network topology is network geometry.
Topology or graph theory deals with graphs of networks
(a) 9.8 A  (b) 10.5 A  (c) 10 A  (d) 5 A and provides information that helps in the formulation
of network equations. Topological properties of a network
Solution: Applying nodal analysis KCL at node 1, depend entirely on the structure of a network.
we get
50 10 V
− + =0 1.3.1 Graph of a Network
4 4 100
If each element or a branch of a network is represented
40 V
+ =0 on a diagram by a line irrespective of the characteristic
4 100 of the element, we get a graph (Fig. 1.31).

Chapter 1 Theory 1.indd 21 3/21/2016 11:21:16 AM


22     Chapter 1:  ELECTRIC CIRCUITS 

2 1.3.4 Concept of Tree and Co-Tree


2
a b
A tree is a connected subgraph of a network, which consists
1 3 of all the nodes of the original graph but no closed path.
1 3
c d 1.3.4.1 Properties of a Tree
4
A tree consists of all the nodes of graph. For an n-node
4 graph, the number of tree branches is n − 1.
V
2
Number of possible trees = det [[Ar][Ar]T]
a
1
c where A is the reduced incidence matrix and [A]T is
a b transpose of reduced incidence matrix.
2 4
c d
4 b d   1.  Twig: The branch of a tree is known as twig.
Number of twigs (n - 1) is known as tree value of
3
1 3

Figure 1.31 |   Network graphs.


the graph. It is also called the rank of the tree.
  2.  Co-tree: The part of a directed graph that is not
covered by the tree is known as co-tree. A co-tree
1.3.2 Directed and Undirected Graph is represented by dotted line.
  3.  Link or chord: The branch of tree is known as
If every branch of a graph has a direction, then the graph
link or chord. Suppose a graph has n nodes and b
is called as directed (oriented). If the branches of a graph
branches, then the number of branches in a tree is
(n − 1) and number of branches in co-tree (links)
have no direction then it is called an undirected graph as
is l = b − (n − 1).
shown in Figs. 1.32(a) and (b).
2 2

a b a b
4 4 Problem 1.25: Identify which among the following
c d c d
graphs is non-planar.
(a) b b
1 3 1 3

Figure 1.32 |   (a) Directed graph and (b) undirected a c ≅ a c


graph.
Node can be incident to one or more elements. The d d
number of branches incident at a node of a graph indi-
cates the degree of the node. A graph is connected if and (b)
2 3 4
only if there is a path between every pair or nodes. 1

1.3.3 Planar and Non-Planar Graph


(c) 1 2 1 2
A graph is said to be planar if it can be drawn on a plane
surface such that no two branches cross each other 5
[Fig. 1.33(a)]. A non-planar graph cannot be drawn on 5
3 3
a plane surface without a crossing over of two or more
branches. [Fig. 1.133 (b)]. 4
4
(d)
2

1 3
5

(a)    (b)
Figure 1.33 |   (a) Planar graph. (b) Non-planar graph.
4

Chapter 1 Theory 1.indd 22 3/21/2016 11:21:31 AM


1.3  TOPOLOGY     23

Solution: Graphs given in (a), (b) and (c) are planar nodes and b branches, the complete incidence matrix A is
while graph in (d) is non-planar. In graph (d): a rectangular matrix of order n × b, whose branches repre-
sent the column and nodes represents the rows.
Number of branches, b =10
Nodes n = 5 Formation of incidence matrix:
Number of tree branches = n − 1 = 4   1.  Obtain directed graph for the given network.
Links = 6   2.  Assign `+1’ in the matrix if the arrow of a branch
is oriented away from the node.
  3.  Assign `−1’ in the matrix if the arrow of a branch
Problem 1.26: What is the number of equations is oriented towards a node.
required to analyse the circuit given below?   4.  Assign `0’ in the matrix if the branch is not con-
nected to a node.
R R

Problem 1.29: Draw the incidence matrix of the fol-


lowing graph.
R f
C
C a 2 b
C 1 3

d e
c
Solution: (n − 1) = 3 equations, as n = 4.
Ans. (4)
4
Solution: In the incidence matrix column represents
Problem 1.27: If a graph of network has 10 branches branches, while rows represent nodes. Therefore,
and 6 nodes, then what is the number of mesh equa-
tions or KVL equation required to solve the network? Branches
Solution: Nodes a b c d e f
Required KVL equations = Links = b − n + 1 = 10 1 1 0 1 0 0 1
−6+1=5 2 −1 −1 0 1 0 0
Required KCL equations = Twigs = n − 1 = 6 − 1 = 5 3 0 1 0 0 1 −1
Ans. (5, 5) 4 0 0 −1 −1 −1 0

Problem 1.28: How many nodes and branches does Note: In matrix A with n rows and b columns, an entry
the following circuit have? aij in the ith row and jth column has the following values:
2A aij = 1, if jth branch is incident to and oriented
away from the ith node.
aij = −1, if jth branch is incident to and oriented
towards ith node.
4Ω 10 Ω 5Ω
aij = 0, if jth branch is not incident to the ith node.
50 Ω
+ +
30 V − − 20 V
Problem 1.30: Draw the graph of the following network
and write the incidence matrix.
Solution: Five nodes and seven branches. C1
a R1 b c
Ans. (5, 7)

1.3.5 Incidence Matrix +



V R2 R3 R4
A matrix representing the relation between number of
branches and number of nodes in a directed graph is known
as incidence matrix. For a given directed graph with n d

Chapter 1 Theory 1.indd 23 3/21/2016 11:21:37 AM


24     Chapter 1:  ELECTRIC CIRCUITS 

Solution: The graph for the given network is Problem 1.32: If the reduced incidence matrix net-
a 5 b 6 c work is given as
−1 0 0 1 1 0
 
A =  0 1 −1 −1 0 0
2  0 0 0 −1 1
1 3  1
4
then what are possible number of trees?
d Solution:
The incidence matrix A is given as
−1 0 0
 
−1 0  0
1 0 
0 1
0
Branches 0 1
  0 −1
A T =  0 1 −1 −1 0 0 
 0 0 1 −1 0
0 −1 1 
Nodes 1 2 3 4 5 6
−1  1
0 −1
 
a 0 0 0 1 0 1
 0 1
b 0 −1 0 0 −1 1 0
 3 −1 −1
c 0 0 −1 1 0 −1  
= −1 3 −1
−1 0 −1 −1 3
d 1 1 1 0
 
The number of trees is det [ArAT r ] = 3(9 − 1) +
1(−3 − 1) − 1 = 24 − 4 − 4 = 16
1.3.5.1 Properties of Incidence Matrix
Ans. (16)
  1.  The sum of entries in any column is zero.
  2.  The determinant of the incidence matrix of a closed Problem 1.33: For a given network, the reduced inci-
loop is zero. dence matrix is given by
  3.  The rank of the incidence matrix of a connected Branches
graph is (n − 1).
Nodes 1 2 3 4 5 6
1.3.5.2 Reduced Incidence Matrix [Ar] 1 1 0 0 1 −1 0

The reduced incidence matrix is obtained by deleting 2 0 1 0 −1 1 −1


any row in the incidence matrix. The number of possible 3 0 1 0 −1 1 −1
trees of a graph is = det [Ar] [Ar]T
4 0 0 1 0 0 1
where Ar is reduced incidence matrix and A Tr is the
Given that i2 = 2 A, i4 = 4 A, i5 = 2 A, determine i6.
transpose of reduced incidence matrix.
Solution: The network can be drawn as
Problem 1.31: Find the reduced matrix of the matrix 5
in Problem 1.30, and find the number of possible trees. 2 6
1 3
Solution: Reduced incidence matrix is 4
−1 0 0 0 1 0 
 
2
[ A r ] =  0 −1 0 0 −1 1 
1 3
 0 0 −1 1 0 −1 
 
4
The number of possible trees of a graph = i6 + i4 - i5 - i2 = 0
det {[Ar] × [Ar]T}
i6 = −i4 + i5 + i2 = −4 + 2 + 2 = 0 A
−1 0 0  Ans. (0)
 
−1 0 0   0 −1 0 

0 1 0
  0 0 −1 
 
1.3.6 Link Currents: Tie Set Schedules
= det  0 −1 0 0 −1 1   0 0 1 
 0 0 −1 1 0 −1   
  3×6  1 −1 0  In any tree of a graph, addition of a link results in a
 
 0 1 −1  6×3
closed path or loop and a circulatory curve or the link
current. The current in any branch of the graph may
 2 −1 0  then be obtained by noting the
 
= det −1 3 −1  = 2(9 - 1) + (−3) + 0 = 16 - 3 = 13   1.  various link currents flowing through the branch;
 0 −1 3 
  3×3   2.  direction of flow of the link currents through
branch (Fig. 1.34).

Chapter 1 Theory 1.indd 24 3/21/2016 11:21:49 AM


1.3  TOPOLOGY     25

3 j3
Problem 1.35: For the graph shown below, what is
6 7 j7
j6 the tie-matrix corresponding to tree formed branches
i
4,3 5, 6?
4 i2 i4 j4 2
2 j2
5 8 j5 j8
i1
1 5 2 5
1 j1 4 0
Graph Tree
j3 4 6 4 6
1 3
7 j6 j7 3
i3
4 i2 i4 j4 Solution: The tie-set schedule or tie-matrix is
j2
8 j5 i1 j8 Link Branch
Number
1 2 3 4 5 6
j1
1 1 0 0 1 −1 0
Figure 1.34 |   Direction of flow of the link currents.
h Tree
2 0 1 0 0 1 −1
Here j1, j2, …, j8 are branch currents and i1, i2, i3, i4 are 3 0 0 1 −1 0 1
link currents. The set of branches forming the closed
loop on which the link circulates is called a tie set. The
tie set for the tree shown in Fig. 1.34 is 1.3.7 Cut Set and Tree Branch Voltages
j1 = i1, j2 = i2, j3 = i3, j4 = i4 A cut set is the minimum set of branches of a connected
j5 = i 2 − i 1, graph such that the removal of these branches causes the
j6 = i 3 − i 2, graph to be cut into exactly two parts.
j7 = i 4 − i 3,
j8 = i1 − i4 1.3.7.1 Procedure
Tie-set schedule is as follows   1.  Select a tree and co-tree from the directed graph.
Link Branch number   2. Determine the number of tree branches, that is, twigs.
number   3.  The number of cut sets to be assigned equal to the
1 2 3 4 5 6 7 8
number of twigs.
1 +1 0 0 0 −1 0 0 +1   4.  A cut set may contain any number of links, but
−1
should cover only one twig.
2 0 +1 0 0 +1 0 0
  5.  The cut set is oriented in the same direction as the
3 0 0 +1 0 0 +1 −1 0 twig it covers.
  6.  Assign +1 in matrix if the direction of cut sets and
4 0 0 0 +1 0 0 +1 −1 branches are the same.
  7.  Assign −1 in the matrix if the direction of cut sets
and branches are different.
Problem 1.34: If tree is as shown below, what is the   8.  Assign `0’ in the matrix if cut sets do not cover the
tie set for this tree? branch.
j3

i1 j6
Problem 1.36: Formulate the cut-set matrix for the
i3 j4 directed graph shown below.
j2
i2
2
i4
4 b 5
j1 a c
Solution: The tie set is
j3 = i1 j6 = i3 1 6 3
j4 = i2 j8 = i4
j1 = i 2 + i 4 j5 = i1 − i2 − i3 − i4
d
j2 = i1 − i3 j7 = i2 − i1

Chapter 1 Theory 1.indd 25 3/21/2016 11:21:55 AM


26     Chapter 1:  ELECTRIC CIRCUITS 

Solution: The tree and cut set corresponding to the Solution: The tree is as shown below:
graph are shown as follows.
8
Branches 6 7
Links Twigs
Cut set 1 2 3 4
1 2 3 4 5 6
1 −1 1 0 1 0 0 5
2 0 −1 −1 0 1 0 The f-cut sets are: (1, 6, 8)
3 −1 0 −1 0 0 1 (1, 2, 5)
(1, 2, 3, 7, 8)
(1, 2, 3, 4)
2
C1 C2
4 5 Fundamental Cut Set Matrix and KCL
4 b 5 a c
a c b We can obtain KCL equations at any node is terms of
6 the Q-matrix.
6 1 3
QIb = 0
C3
d d With reference to the figure given in Problem 1.36,
Tree   Cut set
 I1 
 
I 
−1 1 0 1 0 0  2   0
Problem 1.37: Valid cut set of the following circuit is   I   
 0 −1 −1 0 1 0  3  =  0
−1 0 −1 0 0 1   4   0
I
3 
I4 
4 I 
 5
2
Expanding the above equations, we get
1 6 5
−I1 + I2 + I4 = 0
−I2 − I3 + I5 = 0
(a) 1, 2, 3, 4     (b) 3, 4, 5, 2
(c) 1, 6, 5, 3     (d) 1, 6, 4, 3 −I1 − I3 + I6 = 0
Ans. (d) These equations are the KCL equation at nodes a, c
and d. The branch voltages and fundamental cut set
voltages are as follows. We can express the branch
1.3.7.2 Fundamental Cut Sets voltages in terms of tree branch voltage or twig
voltages.
A fundamental cut set is a cut set that cuts or contains
one and only one tree branch (twig). Therefore, for a Vb = QTVt
given tree the number of f-cut sets will be equal to the
number of twigs (n − 1).
where Vb is the branch voltage and Vt is the twig voltage.

V1  −1 0 −1 
   
 V 2   1 −1 0 
Problem 1.38: For the tree shown, what are the pos-
    V 
 V 3   0 −1 −1   1 
sible f-cut sets?
V   1 0 V 
0   2 
=
8  4 
V   0 1 V 
 5  0   3
6 7 
V   0 0 1 
 6 
1 2 3 4

5 These are the KVL equations of the cut set matrix.

Chapter 1 Theory 1.indd 26 3/21/2016 11:22:05 AM


1.4  NETWORK THEOREMS     27

1.3.8 Interelation between Matrices 1.4  NETWORK THEOREMS


The inter-relationships between different matrices are as
follows: 1.4.1 Linearity and Superposition
-1 T
Bt = [1At Al]
A linear element is a passive element having a linear
-1 voltage-current relationship. A linear dependent source
Ql = A t A l
is a dependent voltage or current source whose output
voltage or current is proportional only to the first power
BQT = QBT = 0 of a specified voltage or current variable in the circuit
(or the sum of such quantities). For example, the depen-
where A is the incidence matrix, B is the fundamen- dent voltage source VS = 0.3i1 − 2iv2 is linear, but VS =
tal circuit matrix and Q is the fundamental cut-set 0.3i12 and 0.3i1v2 are not.
matrix; At is a square matrix of twigs, Al is the matrix
Linear circuit is a circuit composed entirely of inde-
of links, Bt is a matrix of twigs, Bl is a matrix of links,
pendent sources, linear dependent sources and linear ele-
Qt is an identity matrix of twigs and Ql is the matrix
ments. Linearity is the property of an element describing
of links.
a linear relationship between cause and effect. The prop-
erty is a combination of both the homogeneity (scaling)
1.3.9 Summarised Properties property and additivity property.
Superposition principle or the principle of superposi-
The properties of trees, co-trees, complete incidence
tion states that the response in a linear circuit having
matrix, tie-set schedule, cut-set schedule and link cur-
more than one independent source can be obtained by
rents are summarized as follows:
adding the response caused by the separate independent
  1.  The number of branches of a co-tree is (b − n source acting alone.
+ 1).
  2.  A connected graph has n vertices for number of
branches in its tree. 1.4.2 Superposition Theorem
  3.  Reducing the twig voltages to zero forces the link
voltage also to become zero. Twig voltages are Superposition theorem is applicable for linear networks, that
independent of network. is, R, L, C. It is used to find the solution to network with
Number of independent KCL equation = Number of two or more sources that are not in parallel and not in series.
twigs According to this theorem, in any network the response in
  4.  Algebraic sum of the column entries of an incidence any branch, when all the sources are acting simultaneously,
matrix is zero. is equal to the linear sum of individual responses calculated
  5.  The rank of complete incidence matrix is (n − 1). by taking one independent source at a time.
  6.  The order of incidence matrix is n × b.   1.  Replace ideal voltage source by short circuit.
  7.  Determination of the incidence matrix of a closed   2.  Replace ideal current source by open circuit.
loop is zero.
  8.  If Ar is the reduced incidence matrix and ArT is Note: Power is not a linear quantity as it is found by
its transpose, the number of possible trees will squaring the voltage or current. The superposition theorem
be det [Ar][Ar]T. The number of nodes in an inci- does not apply for power.
dence matrix is n, the number of nodes in incidence
matrix will be (n - 1). Consider the following linear circuits shown in
  9.  If a network contains b branches and n nodes then Figs. 1.35(a) and (b).
the number of mesh currents is b − (n − 1).
V1
10. Tie-set schedule gives the relation between branch N Iy
current and link current. V2
11. Cut-set schedule give the relation between branch (a)
voltages and tree branch.
12. Only link currents are independent in a network. V1 I1 V2 I2
N N
Number of independent KVL equations = Number of (b)
links
Figure 1.35 |   Linear circuits.

Chapter 1 Theory 1.indd 27 3/21/2016 11:22:08 AM


28     Chapter 1:  ELECTRIC CIRCUITS 

Applying superposition theorem on these circuits, we get According to this theorem, any two terminal (com-
plex or simple) networks, containing two or more voltage
I1 + I2 = Iy
sources, current sources and resistors, can be replaced
Now, consider the circuit shown in Fig. 1.36. by an equivalent two-terminal network containing an
equivalent voltage source (VTh) and equivalent resistor
R1 e (RTh) in series.
VTh is the open circuit voltage at terminal. RTh
is the input or equivalent resistance at the terminals
V V R2 I when the independent sources are turned left. By using
the Thevenin theorem, current and voltage across any
branch can be easily calculated in any big/complex
network.
Figure 1.36

Taking only the voltage source, we get the cicuit shown Note: Thevenin’s theorem is not applicable for diode,
in Fig. 1.37. FET, etc.

e For any linear bidirectional circuit (Fig. 1.39) having


more number of active and passive elements, the circuit
R1
would be replaced by single equivalent circuit, consisting
+

V R2 I=0 of equivalent voltage source (VTh) in series with equiva-
lent resistance (RTh).

Figure 1.37 A
Linear
R2 bidirectional
⇒ tV = V network
R1 + R2 B
Taking only current source, we get the cicuit shown in Linear (or)
Fig. 1.38. non-linear

R1 A A
RTh
ZTh
V=0 R2 I
VTh = VOC +
− VTh
Figure 1.38 B B
For DC For AC

Figure 1.39 |   Application of Thevenin’s circuit.


R1R2
tI = I
R1 + R2

Applying superposition principle, we get 1. Finding VTh: Whenever we need to find the response
known as load voltage, we always disconnect the load
R2 R1R2
t = tV +tI =
resistor and find the open circuit voltage across load ter-
V + I
R1 + R2 R1 + R2 minals. Then, we find out the voltage developed due to
active sources across the terminal.

1.4.3 Thevenin’s Theorem Finding RTh: First, open circuit across the terminal.
Then, deactivate all independent voltage and current
Thevenin’s theorem is applicable for any linear, bilateral sources and find the equivalent resistance with respect to
and lumped networks. It is a method for the reduction load terminals. Now, make all active sources zero (i.e., volt-
of a portion of a complex circuit into a simple one. It age source short circuited and current sources open cir-
reduces the need for repeated solution of the same sets cuited). Then, calculate equivalent resistance across the
of equations. terminals.

Chapter 1 Theory 1.indd 28 3/21/2016 11:22:21 AM


1.4  NETWORK THEOREMS     29

Problem 1.39: Find the Thevenin equivalent voltage Problem 1.40: Find the Thevenin equivalent circuit
and resistance for the following circuit. Also determine for the following circuit.
load current and voltage. 6 Ω VA 4Ω
C+
12 kΩ 8 kΩ

10 Ω 10 Ω
I
20 V
4 kΩ R2 = 5 kΩ
D−
48 V
4Ω 6Ω
Solution: From the given circuit
Solution: For determining VTh, VA − 20 VA V
+ + A =0
Step 1: Open 5 kΩ load to obtain the following circuit 10 10 20
2VA − 40 + 2VA + VA =0
12 kΩ 8 kΩ 5VA = 40 ⇒ VA =8V
A 8
+ I= = 0.4 A
  20
48 V 3 mA 4 kΩ
12 V VCD = 10 × 0.4 =4V
The equivalent resistance across the terminals, when
− all active sources are zero is
B
6W 4W
Step 2: Calculate the open circuit voltage across AB,
which is obtained as VTh = 12 V.

For determining RTh,


RTh 10 W 10 W RTh

Step 1: Make all active sources equal to zero.


4W 6W
10 × 10 
12 kΩ 8 kΩ
15 × 10
A  + 10 || 10 = =6 W
 20  25

4 kΩ So, the Thevenin equivalent circuit is


6Ω
C
B +
4V
RTh = 12 kΩ | | 4 kΩ in series with 8 kΩ = 11 kΩ

Now, by the Thevenin theorem, new circuit looks
as follows: D

RTh = 11 kΩ Problem 1.41: Determine the Thevenin equivalent


resistance R Th between terminals a and b.
50 Ω
VTh = 12 V RL = 5 kΩ a
+
V1 100 Ω 200 Ω 0.1V1

b
VTh 12
So, I L = = = 0.75 mA Solution: The problem can be solved using source
R Th + R L 11 + 5 transformation.

VL = IL × RL = 0.75 mA × 5 kΩ = 3.75 V
This network contains only dependent source, there-

fore, its VTh = 0.
Ans. (0.75, 3.75)

Chapter 1 Theory 1.indd 29 3/21/2016 11:22:35 AM


30     Chapter 1:  ELECTRIC CIRCUITS 

For finding RTh, one test voltage V is applied


between terminals a and b. 1600 Ω
250 Ω 100 x
I

+ 16 Ω
+ V1 100 Ω − 20V
V − 1

+

16 Ω
V V + 20V1 80 Ω 20 Ω
−I + + =0 y
100 250
Rx-y = RTh = 32 Ω
50 Ω 200 Ω
a Ans. (120, 32)
+
− 20V
V1 100 Ω + 1 Problem 1.43: Thevenin resistance between termi-
− nals x-y of the circuit is
b
6Ω
Here, V = V1, therefore
V 21V 5Ω 1Ω
I= +
100 250
V
= RTh =
500
= 10.6 W 5A 2Ω 3Ω
I 47 5Ω 4Ω
x
a
+
20 V − 10 Ω 10 Ω
10.6 Ω
b y

Ans. (10.6) Solution: Thevenin resistance can be determined from


the following equivalent circuit when x and y are open.
Problem 1.42: Find the Thevenin equivalent source
6W
voltage and resistance for the following circuit.

5W 1W
80 Ω 20 Ω
+
2W 3W
200 V 5W 4W
− x y x

20 Ω 80 Ω 10 Ω 10 W

y
11 × 1 11
Solution: We have (5 + 6) || 1 = = W
200 12 12
I1 = = 2A
W
100 11 11 + 60 71
So, +5 = =
12 12 12
Vx-y = VTh = 2 × 80 − 20 × 2 = 160 − 140 = 120 V 71
 71  ×4
The RTh voltage source is short circuited.   || 4 = 12 284 12
×
284
12 
= =
71 12 119 119
20 Ω 80 Ω 12
+4

 5 × 10 284 
  || 10
 15 119 
+

5.72 × 10
(3.33 + 2.38) || 10 = = 3.638
80 Ω 20 Ω 15.71  
Ans. (3.638)

Chapter 1 Theory 1.indd 30 3/21/2016 11:22:56 AM


1.4  NETWORK THEOREMS     31

Problem 1.44: Thevenin equivalent resistance RTh 2 kΩ 3 kΩ


for the given network is
+
6Ω +
4V − Vx ISC

8Ω 2Ω
a a
2A
10 kΩ
+
RTh 0.8 mA 10 kΩ 8V −
b b
4Ω 2Ω
4
10 Ω
I sc = = 0.8 mA
5000
+−
V
= 10 kW
8
RTh = Th =
2V I sc 0.8
Solution: From the given circuit, 2 A source is open Ans. (8, 10)
circuit and 2 V source is short circuit.
1.4.4 Norton’s Theorem
6Ω RTh
This theorem is applicable for any linear, bilateral and
12 Ω 4Ω lumped networks. Just like the Thevenin theorem, this
theorem is also used to reduce the complex network into
10 Ω
simpler one. It also reduces the need for repeated solu-
tion of the same sets of equations.
According to this theorem, any two-terminal (com-
12 × 4 64 + 12
RTh = 16 + = = 16 + 3 = 19 W plex or simple) networks containing two or more voltage
16 4 sources, current sources and resistances can be replaced
Ans. (19) by an equivalent two-terminal simpler network contain-
ing only a current source in parallel with a resistor. The
current source is known as Isc, that is, short circuit cur-
Problem 1.45: Determine the Thevenin equivalent rent; and RN is the input or equivalent resistance at the
between terminals a and b. terminal. All independent sources are turned off. By using
Norton’s theorem, current and voltage across any branch
2 kΩ 3 kΩ can be easily calculated in any big complex network.
a
+ 1.4.4.1 Finding Isc
+ Vx Vx
4V −   1.  Short circuit the terminal.
4000
−   2.  Then find the current flowing through the short
b circuit terminal (i.e., Isc).

Solution: (i) For determining VTh: 1.4.4.2 Finding RN


Here, VTh = Vx, so
  1.  Open circuit across the terminal.
  2. Then make all active sources zero (i.e., voltage source
V V
− VTh + V Th = 0 short circuited and current source open circuited).
− 4000
Th
V Th
VTh + 2000Th = 0   3. Calculate the equivalent resistance across the terminals.

−V4000 Th − 8 = 0
4000++2V2000 =0
−VTh + 2V Th − 8 = 0
Th 2000 For a two-terminal network shown in Fig. 1.40(a), the

VTh VTh − 8 = 0
VTh=+82Volts Norton equivalent cicuiot is as shown in Fig. 1.40(b).
VTh = 8 Volts
VTh = 8 Volts A
A
(ii)  For determining RTh:
Isc RN
Consider the following equivalent circuits
B B
Figure 1.40 |   (a) Two-terminal circuit. (b) Norton
equivalent circuit.

Chapter 1 Theory 1.indd 31 3/21/2016 11:23:08 AM


32     Chapter 1:  ELECTRIC CIRCUITS 

Problem 1.46: Find the equivalent Norton circuit of Solution: The resistance is given by
the circuit shown in the following figure. 5×3 15
=2⇒
5Ω 5Ω
=2
VA 3+R 3+R
A
5 + 6 + 2 R ⇒ 2 R = 9 ⇒ R = = 4.5 W
9
2
+
30 V − 5A 10 Ω Ans. (4.5)

B Problem 1.48: Norton’s equivalent of the given


network is
Solution: Replacing 10 Ω resistance by short circuit,
5Ω 4/6 Ω
we have
1
VA − 30 V
+5+ A = 0 +
5 5 4Ω
Isc
VA − 30 + 25 + VA 5V
=0 −
5
2VA = 5 ⇒ VA = 2.5 V 2
2. 5 Solution: Consider the given circuit. The nodal analysis
I sc = = 0.5 A
5 at node 1 gives
For RTh between A and B, we have VA − 5 VA V ×6
+ + A =0
5Ω 5Ω 5 4 4
A
4 VA − 20 + 5 VA + 30 VA
=0
20
10 Ω 20
or 39VA = 20 ⇒ VA = = 0.512 V
39
B 0.512 × 6
10 × 10
I sc = = 0.769 A
RTh = = 5Ω 4
20

Req = 2.88 Ω
The equivalent Norton circuit is
A 0.769 A

Isc = 0.5A RTh = 5 Ω

= 0.66 + 2.22 = 2.88 W


4 20
B Req = +
6 9
Ans. (5) Ans. (2.88)

Problem 1.47: Norton equivalence of the network `X’


Problem 1.49: Determine the Thevenin’s and Norton’s
to the left of AB is a current source IN = 5 A from
B to A, RN = 3 Ω and current through R when it
equivalents across a and b in the given circuit.
is connected across AB = 2 A. What is the value of 100 V
resistance R? −+ a
+
A
N R 0.01 V 20 kΩ V
B

b
Solution: Let V1 = VTh. Now by KVL in first (only)
5A 3Ω R loop,
2A V + 100 − V1 = 0
Applying Nodal analysis, we get

Chapter 1 Theory 1.indd 32 3/21/2016 11:23:33 AM


1.4  NETWORK THEOREMS     33

V RS
−0.01 V1 + +0 = 0
20 × 10 3

−200 V1 + V = 0
+
−200 V1 + V1 − 100 = 0 −
VS RL
I
−100
V1 = = −502.5 mV = VTh
Figure 1.41 |   Variable load conditions.
199
On short circuiting, we have V1 = 0.
100 V VS
I=
−+ R S + RL
Power delivered across the load resistor

VS2R L
20 kV Isc P = I 2R L = Watts
+ (R S + R L)2
        
For maximum power transfer,
Isc
dP
=0
By KVL, 100 − 20 × 103 Isc dR L
So, Isc = 5 mA. Now, Therefore,
 (R + R )2 − R × 2(R + R ) 
=  S L  2
VTh
RTh = = 100.5 W dP L L S
 VS = 0
I sc dR L  2 2

 [(R S + R L) ]
and the equivalent circuit is
Solving we get, 2RL-RS-RL = 0 ⇒ RS = RL.
100.5 Ω 5 mA V VS
a I = Swhen I = VIS=, we get
Hence,
2R S 2R S
2R S
502.5 mV 5 mA 2 2
− VS2 VS2P V 2= VSV 2= VS Watts
⇒ =
4R S= 4R =
Pmax = S
Watts
max S
+ 4R L Pmax 4R L Watts
4R
4R S
L S
5 mA 2 V22
VS2 V 2 PVV2= VS2 2 V 2
Total
PT = power+ PPTS ==
SV V =
T = 2R +4R S =
b TS S+
Watts Watts
4R S 4R 4R S Watts
2R S
S 4R S 4R S 2R S S
2
Useful power V S 4R S
Maximum efficiency = = 2 = 50%
1.4.5 Maximum Power Transfer Theorem Total power V S 2R S
Case 2: Variable ZS and ZL (RL and jXL)
This theorem is applicable only for a linear network, (i)  OnlyRL is variable (Fig. 1.42).
that is, the network with RLC transformer and linear
controlled source as element. The theorem is applicable Z S = RS + jXS
only when load is variable; otherwise, we need to choose
the minimum internal impedance of the source, which
results in maximum current through the fixed load and +
hence a maximum power dissipation across the load. VS
According to this theorem, for any power source, the − ZL = (RL = jXL)
maximum power transferred from the power source to
the load is when the resistance of the load RL is equal to
the equivalent or input resistance of the power sources Figure 1.42
(i.e., Rin = RTh or RN).
VS
The process used to make R L = R in is called imped- I=
R S + R L + j(XS + XL )
ance matching. This means making the output      
impedance of source equal to input impedance of load VS
I =
to maximize the power transfer and minimize load (R S + R L) + (XS + XL )2
2
reflections.
VS2R L
Case 1: Under variable load conditions, RS and RL True power = I2RL =
(Fig. 1.41), we have (R S + R L)2 + (XS + XL )2

Chapter 1 Theory 1.indd 33 3/21/2016 11:23:54 AM


34     Chapter 1:  ELECTRIC CIRCUITS 

RL is variable, so for maximum power


Problem 1.50: In the following circuit the value of R
dP V 2 [(R S + R L)2 + (XS + XL )2 − 2RL (R S + R L) required for the transfer of maximum power through
= S =0
dR L [(R S + R L)2 + (XS + XL )2 ]2 load resistance of 10 Ω is
Rx
dP
For maximum power transfer = 0, so
dR L

R L = R 2S + (X S + XL )2 Load
+ 10 Ω
20 V 5Ω
V s2 R L −
Pmax =
(RS + R L )2 + (XS + XL )2
Hence, h < 50%. (a) 5 Ω    (b) 15 Ω    (c) 10 Ω    (d) 20 Ω
(ii)  OnlyXL is variable. Solution: Let the required resistance be Rx. Then
Using the above expression for maximum power, we have 10 × R x 20
Req =
VS2RL (0) + 2(XS + XL ) − 0 10 + R x Req + 5
dP
When R = 5 Ω, Req = 3.33 Ω
= =0
dXL (R + R )2 + (X + X )2  2
 S L S L  10 × R x
P5 Ω = I25 × 3.33, I5 =
20
VS2RL 10 + R x Req + 5
I is maximum and power is maximum ≅ 50 W
Pmaxx = Watts
(RS + RL )2
(iii) Both RL and XL are varied simultaneously, Ans. (a)

R L = R 2S + (XS + XL )2
Problem 1.51: The value of RL to deliver maximum
Then, RL = RS power is 200 Ω and the value of power is
XL + XS = 0 ⇒ XL = −XS 100 Ω
ZL = RL + jXL = RS − jXS
ZL = ZS* W +
20 V RL
VS2 −
Pmax = 4R P½R L = R S
S
So, % h = 50%, which is the same as in case of vari-
Solution: We have
able load condition.
 20 2
PL =  × 100 = 0.01 × 100 = 1 W
 200 
Case 3: Consider a special case of Case 2 when
XL = 0 (Fig. 1.43).
RS + jXs Ans. (1)
ZS
Problem 1.52: Maximum power delivered to R in Ω
+ by a 20 V DC source with 5 Ω internal resistance is
VS RL
- Solution: We have
R = 5 Ω, I =
20
=2 A
10
Figure 1.43
So, the power delivered to R is
RL = RS2 + (XS + XL )2 = RS2 + XS2 PL = I2R = 4 × 5 = 20 W
Ans. (20)
RL = RS + jXS ⇒ RL = ZS W
Therefore, Problem 1.53: Find the value of R for maximum
VS2RL power transfer.
Pmax = where XL = ZS
(RS + RL )2 + XS2
Hence, efficiency h < 50 %.

Chapter 1 Theory 1.indd 34 3/21/2016 11:24:17 AM


1.4  NETWORK THEOREMS     35

But here, only RL is given


5Ω 10 Ω R RL = |3 + 4j| = 5 Ω
100∠0 100∠0
I= =
+ + 3 + 4j + 5 3 + 4j
10 V − 20 V −
Pmax = I2RL = (|I|2 × 5) = 625 W
Ans. (625)
Solution: For maximum power, the value of resistance
R = RTh 1.4.6 Reciprocity Theorem

In a linear, passive and bilateral networks, the ratio of


response to the excitation is always constant; even if
5Ω 10 Ω RTh the sources are interchanged from input to the output
terminals (Fig. 1.44).

5 × 10 + +
= 3.33 W
50
RTh = = V − N I I N − V
5 + 10 15
Ans. (3.33)
V V
Problem 1.54: Find the maximum power delivered to = K1 = K1
I I
the load in the circuit given below.
(a) (b)
6Ω 8j Ω 8j Ω 6Ω
Figure 1.44 |   Receiprocity condition with sources inter-
A changed for input to output terminals.

100∠0° 90∠0°
RL V
= Constant  
I
B V1 V
= 2
I1 I2
Solution: Now for maximum power transfer, Note:
6Ω 8j Ω 8j Ω 6Ω   1. This theorem is applicable to only linear, passive and
bilateral networks, that is, the network with R, L, C
A so the network with RLC is called reciprocal network.
+   2. The presence of the controlled sources in the network
100∠0° VTh 90∠0° makes it linear, active and bilateral. Hence, reciproc-
− ity theorem is not applicable. So, these networks are
B called non-reciprocal networks.

Problem 1.55: The network shown in the following


figure is a linear passive and bilateral network. Use the
data given in Fig. (a) and apply the principle of reciproc-
Applying Nodal analysis, we get
ity and superposition to determine current ix in Fig. (b).
VTh − 100∠0 VTh − 90∠0 4A
+ =0
6 + 8j 8j + 6
+ 2A
         VTh = 100∠0° 5V − N
So, ZTh = 3 + 4j Ω
The equivalent circuit is
Figure (a)
4j Ω 3Ω
ix

+ +
10 V − N − 10 V
100∠0° V RL

Figure (b)

Chapter 1 Theory 1.indd 35 3/21/2016 11:24:33 AM


36     Chapter 1:  ELECTRIC CIRCUITS 

Solution: By applying the homogeneity property to


Figure (b) = Figure (f) + Figure (d)
Fig. (a), we get circuit given in Fig. (c).
8A ix = −24 − 4 = −28 A
Ans. (28)
+ 2 × 2A = 4A
10 V − N
Problem 1.57: The network shown in following figure
given is reciprocal. Use the data given in Fig. (a) and
Figure (c) the principle of reciprocity and Norton’s theorems to
On applying reciprocity property to Fig. (c), we get determine the current ix in Fig. (b).
circuit in Fig. (d).
8A 5A

+ N 2A
− 10 V
4A N 10 V

Figure (d) Figure (a)

Fig. (b) = Fig. (c) + Fig. (d)


ix
ix1 = ix1 + ix2 = 8 − 4 = 4 A
Ans. (4)
2Ω N 30 V

Problem 1.56: Determine the value of ix in the given


circuits. Figure (b)
4A
Solution: By applying homogenity and reciprocity
+ 2A principles to Fig. (a), we get
5V − N
15 A

Figure (a) Isc N 30 V


ix

− + Isc = 6 A
30 V + N − 10 V
Figure (c)

Figure (b) 5A

N N 2A
Solution: By applying homogenity property to Fig. (c)
in Problem 1.55, we get Fig. (e). 10 V

24 A RTh
RTh = 10 = 2 Ω
5
+ 2 × 6 = 12 A
30 V − N
Figure (d)

Thus the equivalent circuit for Fig. (b) can be repre-


Figure (e) sented as follows
By reversing the polarity of sources in Fig (e), we get
circuit shown in Fig. (f). ix
−24 A
2Ω 2Ω 6A
− −12 A
30 V + N

So, ix = 3 A
Figure (f) Ans. (3)

Chapter 1 Theory 1.indd 36 3/21/2016 11:24:50 AM


1.4  NETWORK THEOREMS     37

1.4.7 Substitution Theorem


Problem 1.58: Using the reciprocity and superposition
theorems simultaneously, determine the current I in
In a linear network, any passive element can be replaced
the network of figure given below if a 5 V source acting
by a voltage source whose magnitude is the drop across
alone can produce a current of 1 A in the resistor R.
that element and polarity of the source is the drop polar-
2Ω R I ity across that element, provided the other branch cur-
rents and voltages are constant (Fig. 1.45).
− I I
+
V1 = 5 V − 2Ω + V2 = 10 V +
(R) +
N IR N − V = IR

Figure (a)
Solution: When current of 1 A is produced, the given Figure 1.45 |   Substitution theorem.
circuit can be represented as shown in Fig. (b).

2Ω R Problem 1.59: For the circuit shown in the following


figure, the 1 Ω resistor can be replaced by
3Ω 3Ω
+
V1 = 5 V − 2Ω I1 = 1 A

+

10 V 4A 1V
Figure (b)
By applying the homogeneity principle to Fig. (a), we
get circuit shown in Fig. (c).
Solution: Using substitution theorem, the equivalent
2Ω R circuit can be represented as follows.
2A 3Ω 3Ω
+
10 V − 2Ω I1 = 2 A + − 1A
6V 1A
+ +
1Ω 1V
+
10 V +
− 4V 4Ω 10 V −
− −
Figure (c)
Applying reciprocity principle to Fig. (c), we get cir-
cuit shown in Fig. (d).2 A 3 Ω 3Ω 3Ω 3Ω
2Ω + R− 1A 1A
6V 1A
+ +
10 V − + 4V 4Ω 1Ω 1V 10 V +
− 4Ω + 1V

2Ω − + 10 V −
2A −

So 1 Ω resistor can be replaced with voltage source of


Figure (d) 1 V with the positive terminal upwards.
Interchanging the polarity, we get circuit shown in
Fig. (e).
1.4.8 Tellengen’s Theorem
2Ω R 4A
In any arbitrary network, the algebraic sum of the
− powers in all the branches at any given instant is zero,
2A 2Ω 2A 10 V that is, the algebraic sum of the power delivered by the
same elements in network is equal to the power absorbed
+
by the remaining elements present in the network.
When the current enters at negative terminal of an
Figure (e)
element, then that element will deliver the power, other-
Figure (a) = Figure (b) + Figure (c) = 1 + 4 ≤ 5A. wise it will absorb the power.

Chapter 1 Theory 1.indd 37 3/21/2016 11:25:01 AM


38     Chapter 1:  ELECTRIC CIRCUITS 

i i Solution: On nodal analysis


Element Element
+ − − + V − 16 V
v v + −8 = 0
2 2
P(absorb) = v × i       P(deliver) = vi
V = 16 V
(a) (b) 2Ω 16 V 2 Ω
Figure 1.46 |   (a) Power absorption. (b) Power + −
8A 0A
delivery.
16 V
+
The source can absorb power [Fig. 1.46(a)] or can deliver 8A 2Ω +
− 16 V
power [Fig. 1.46(b)] but the passive elements always −
absorb power (since the current enters at positive termi-
nal of the respective element). P8 A = 32 × 8 = 256 W (Deliver)
P2 Ω = 16 × 8 = 128 W (Absorb)
b
∑ vk ik = 0
k =1 P2 Ω = 16 × 8 = 128 W (Absorb)
where b is the number of branches. P16 V = 0 × 16 = 0 W
Note: Ans. (0)
  1.  Tellengen’s theorem depends only on the voltage
and current across the element, but not on the type Problem 1.62: Verify Tellengen’s theorem for the cir-
of the element. cuit shown in the following figure.
  2.  Tellengen’s theorem expresses the conservation of
power or energy in the network. 2Ω V 1Ω
ix
+ +
10 V − 3A 2ix
Problem 1.60: For the given circuit, calculate −
3
∑ vx ix
k =1
Solution: The circuit can be represented as
15 V
5Ω 2 Ω 1.4 A V 1Ω


+
+ 4.4 A 4.4 A
3A 3A
+ + +
20 V 5V +
10 V − 2ix
− −
3A

3A

Nodal analysis,
Solution: From the circuit, we have
V − 10 V − 2ix
3 + −3 = 0
∑ vx ix = v1i1 + v2i2 + v3i3 = −20 × 3 + 15 × 3 + 5 × 3 2 1
10 − V
k =1 ix =
= −60 + 45 + 15 = 0 W 2
5 V = 7.2 V
∑ vk ik = 35.6 − 35.6 = 0
Ans. (0)
ix = 1.4 A
k =1
P10 V = 1.4 × 10 = 14 W (Deliver)
Problem 1.61: Determine the power delivered by the
16 V source. P2 Ω = (1.4)2 × 2 = 2 = 3.92 W (Absorb)
2Ω V 2Ω P3 A = 7.2 × 33 = 21.6 W (Delivery)
P = (4.4)2∑
1 Ω × v1x i=
x = v1i1 W
19.36 2 i2 + v3 i3 = −20 × 3 + 15 × 3 + 5 × 3
+ v(Absorb)
k =1
2Ω + 16 V P2i = 2 × 1.4 × 4.4==−12.32 W+ (Absorb)

8A x 60 + 45 15 = 0 W
5
∑ vk ik = 35.6 − 35.6 = 0
k =1

Chapter 1 Theory 1.indd 38 3/21/2016 11:25:19 AM


1.4  NETWORK THEOREMS     39

1.4.9 Compensation Theorem 1.4.10 Millman’s Theorem

When the resistance R of an uncoupled branch, carrying Consider several practical voltage sources connected in
a current I, of a linear time-invariant network is changed parallel as shown in Fig. 1.47.
by ∆R, the currents in all branches would change and
can be obtained by assuming an ideal voltage source of
(VC) connected such that VC = (I∆R) is in series with R1 R2 R3 ..... Rn R′
(R + ∆R). All other sources in the network are replaced
by their internal resistances, and the source voltage (VC)
V′
+ V2 + V3 + ..... Vn + +
− − − − −
V1
should oppose the original current.

Problem 1.63: Find the change in the current in the Figure 1.47 |   Voltage sources.
following circuit when the 5 Ω resistor is changed to
10 Ω by using compensation theorem. According to Millman’s theorem, each branch will have
its own voltage source and series resistance. The equiva-
10 Ω lent voltage and resistance is
V1G1 + V2G2 + V3G3 +  + Vn Gn
V′=
G1 + G2 +  + Gn
+
10 V − 4Ω 5Ω
R′ =
1
G1 + G2 +  + Gn
Solution: For the given circuit 1
where Gn = .
4×5 Rn
= 12.2 W
20 110
Req = 10 + = 10 + =
4+5 9 9
Problem 1.64: Determine the current through the
10
Therefore, I10 W = = 0.81 A 3 Ω resistor.
12.2
When 5 Ω changed to 10 Ω, a voltage source of
IC × ∆R, where ∆R is change in resistance, that is, 5 Ω.
I
1Ω 2Ω 2Ω

10 Ω 3Ω
−12 V
+ + +
2V − − 4V −

10 Ω
4Ω Solution: Using Millman’s theorem
1 1 1
2 × − 12 × + 4 ×
2 = 2 − 6 + 2 = −1 V
+
− V′=
1.8 V 1 2
1 1 2
1+ +
4 2 2
⇒ I5W = 0.81 × = 0.36 A
R′ = = 0.5 W
9 1
1 1
V = 0.36 × 5 = 1.8 V 1+ +
2 2
10 × 4 0.5 Ω
Req = 10 + 10 || 4 = 10 + = 12.85 W
14
1.8 V
3Ω
Total current, I= = 0.14 A
12.85 −1V −
+

4 4
I10 W = 0.14 × = 0.14 × = 0.04 A
10 + 4 14
−1 −2
Therefore, the change in current = 0.81 − 0.04 = 0.77 A I= = A
 −2 
Ans.  
3 + 0.5 7
Ans. (0.77)  7 

Chapter 1 Theory 1.indd 39 3/21/2016 11:25:42 AM


40     Chapter 1:  ELECTRIC CIRCUITS 

Note: If the polarities of V2 source are reversed, then V2


is replaced by −V2 in expression of V ′.
Problem 1.66: Determine the dual of the network.
2Ω 1Ω
1.4.11 Duality Theorem ix
According to this theorem, two linear circuits are said to +
10 V − 1Ω +
2ix
be duals of one another if they are described by the same i1 i2 −
characterizing equations with dual quantities interchanged.
Network and its duals are same only with respect to the
performance, but the elements and connection points of Solution: For the given network
view are not equal. The dual quantities are as follows: i1 = ix
Resistance R ↔ Conductance G 10 = 2i1 + 1 × (ii - i2) (i)
Inductance L ↔ Capacitance C 0 = 1 . i2 + 2ix + 1(i2 - i1) (ii)
Impedance Z ↔ Admittance Y By transferring equation into its dual,
Voltage V ↔ Current i
vx = v1
Voltage source ↔ Current source
10 = 2v1 + 1(v1 - v2) (iii)
Node ↔ Mesh
0 = 1 . v2 + 2vx + 1(v2 - v1) (iv)
Series ↔ Parallel path
Dual circuit is:
Open circuit ↔ Short circuit Ω
Star network ↔ Delta network
v1 1 v2
KCL ↔ KVL
Nortons ↔ Thevenins + Ω Ω
10A vx 2 1 2vx

Problem 1.65: Find the dual of the given network.

W
2 1.4.12 Wye(Star)-Delta Transformations
2Ω 1Ω
Circuit analysis becomes tedious when the circuit ele-
5F ments are neither in parallel nor in series. For example.
consider the bridge circuit in Fig. 1.48.
10∠0° 5H 2F
a
W
1
2H R R

b R
10∠0 A

Dual
R R R
5F

Figure 1.48 |   Circuit elements neither in series nor


W W
10∠0° 2 1 2H parallel.
How do we combine all resistors when the resistors are
neither in series nor in parallel. Many such type of cir-
cuits can be simplified by using three terminal equiva-
lent networks. These are the wye (Y) star or tee (T)
network shown in Fig. 1.49(a) and the delta (Δ) or pi (p)
network shown in Fig. 1.49(b).

Chapter 1 Theory 1.indd 40 3/21/2016 11:25:50 AM


1.4  NETWORK THEOREMS     41

R1 R2 Z1
1 3 1 3 A ZA ZB B
A B

R1 R2

R3 Z3 Z2
R3 ZC

2 4 2 4
Figure 1.51 |   Delta to Wye conversion.
Wye (Y ) Tee (T )
(a)

A B R2 C 1.4.12.2 Wye to Delta Conversion

To convert Y into Δ, the new resistor values (Fig. 1.52)


are calculated as
R0 R1
R0 R1
ZA ZB ZBZC
Z1 = ZA + ZB + Z2 = ZB + ZC +
ZC ZA
B R2 C A ZCZA
Z3 = ZC + ZA +
Delta (pi network) ZB
Delta network
(b) Z1
Figure 1.49 |   (a) Wye (Y) (star) or tee (T) network.
A ZA ZB B
A B
(b) Delta (Δ) or pi (p) network.
Figure 1.50 shows both the interconnections superim-
posed in the same circuit. Z3 Z2
ZC

Z1, Z2, Z3 → ∆ Figure 1.52 |   Wye to Delta conversion.


ZA
Z3 Z1 ZA, ZB, ZC → Y Each resistor in the ∆-network is the sum of all possible
products of Y-resistors taken two at a time, divided by
the opposite Y-resistor.
ZC ZB
Problem 1.67: The Y-network of impedances with equal
magnitude is converted into its delta equivalent, then
Z2 the impedances of each arm of the delta network is

Figure 1.50 |   Wye and Delta interconnections.


Solution: We have  Z1 = 3Z  Z2 = 3Z  Z3 = 3Z
(Delta to Wye conversion makes the impedances 1/3
times.)
1.4.12.1 Delta to Wye Conversion

To convert Δ into Y, the new resistor values (Fig. 1.51) Problem 1.68: In the following circuit, determine Rab.
are calculated as a
Z1Z3 Z1Z2
ZA = ZB =
Z1 + Z2 + Z3 Z1 + Z2 + Z3 R R
Z2Z3
ZC = b R
Z1 + Z2 + Z3

Each resistor in the Y-network is the product of the


resistors in the two adjacent Δ branches, divided by the R
sum of the three Δ resistors. R R

Chapter 1 Theory 1.indd 41 3/21/2016 11:26:00 AM


42     Chapter 1:  ELECTRIC CIRCUITS 

Solution: First convert one delta (∆) to star (Wye) Problem 1.70: Determine the input resistance Rin in
the following circuit.
b
1Ω 1Ω 1Ω

a R R/3
R R/3 1Ω 1Ω
I

R Rin
R/3
b
Solution: The equivalent circuit for the given circuit is

1Ω
R/3
a

2R
Rin 1Ω 2Ω
3

Then equivalent circuit is


1× 2
Rin = 1 + = 1.67 W
R 1+2
Ans. (1.67)
a b

R Problem 1.71: Consider Wye (star) circuit shown in


the following figure.
R
Rab = A
2
 8  RA
 
 3 
RB
Problem 1.69: Determine the equivalent impedance
between a and b. B RC

a C
Here,
2Ω −j4 4Ω
RAB with C open = 6 Ω
RBC with A open = 11 Ω
RCA with B open = 9 Ω
j2 Ω
2Ω 4Ω Then determine RA, RB, RC.

b Solution: RA + RB = 6 (i)

Solution: From the given circuit, we have RB + RC = 11 (ii)


RC + RA = 9 (iii)
2×4=2×4
Adding (i), (ii) and (iii), we get
8=8
2(RA + RB + RC) = 26
So, the bridge is balanced. RA + RB + RC = 13 (iv)
R
4×8 Rab = (iv) - (i) ⇒ RC = 13 - 9 = 7 Ω
= W
8
Zab = Rab =
(iv) - (ii) ⇒ RA = 13 - 11 = 2 Ω
2
4+8 3
8
Ans.   (iv) - (iii) ⇒ RB = 13 - 9 = 4 Ω
 3 
Ans. (2, 4, 7)

Chapter 1 Theory 1.indd 42 3/21/2016 11:26:18 AM


1.5  LAPLACE TRANSFORM     43

1.5  LAPLACE TRANSFORM 1.5.1.2 Integrating Property and Shifting


Theorem

Let f (t) be a continuous signal (periodic or non-periodic). The integrating property of Laplace transform can be
The Laplace transform of the signal f (t) is expressed as
∞  t   0  t 
L[f (t)] = ∫ f (t)e−st dt  (bilateral Laplace transform) L  ∫ f (t)dt = L  ∫ f (t)dt + L  ∫ f (t)
    
−∞ −∞  −∞  0 
 
Since all the real-time signals are causal signals, the t
Laplace transform of causal signal is ∫ f (t)dt
∞ −∞ F (s)
−st = +
L[f (t)] = ∫ f(t)e dt  (unilateral Laplace transform) s s

 f (t) 
0

 t  ∫
where s = s + jw. L  = F (s) ds
s
The Laplace transforms for the various signals, such as The shifting theorem states that
constant, exponential, sine and cosine signals are listed
in Table 1.2. → e−as F (s)
f (t − a) u(t − a) ←

1.5.1 Properties of Laplace Transform


Problem 1.73: Determine
1.5.1.1 Differentiation Property f(t) = −6e−2t [u(t + 3) - u(t - 2)].

d
f (t) ←
→ sF (s) − f (0) Solution: Two-sided
dt
∞ ∞
d2
→ s2 F (s) − sf (0) − f ′(0)
f (t) ← F (s) = −6 ∫ e−2t e−st dt + 6 ∫ e−2t e−st dt
dt2 −3 2
−d
 e−(2 + s)t  ∞  e−(2 + s)  ∞
L[tf (t)] = F (s)
   
ds
= −6   + 6  −(2 + s) 
 −(2 + s)  3   2
Problem 1.72: Find the Laplace transform of t sin 2t.
 e3(2 + s)   −2(2 + s) 
Solution: = +6   + 6 e
  (2 + s) 

−d  2  −4s    
 = 2 + s
ds  s2 + 4  (s2 + 4)2
L[t sin 2t] =

s
Table 1.2 |   Laplace transform of different signalscosh(at)ut ←
→ 2
s − a2
d
d(t) ←→ 1
LT u(t) ←→
LT 1 f (t) ←
→ sF (s) − f (0−)
s dt

∠n
r ←→
1 1
r(t) ←→ tn ←→
LT LT LT
s2 s3 sn +1

−at n ∠n
e−at ←→ where Re[s] > a e−at (−t)← ← →∠n
1 1

LT LT
e− ← 
at en t LT
→ ∠
e−at tn ←→ (s +(as)+
s+a s+a t LT n a)n +1
n +1
n +1
−at
(s + a) w
w w ←  → w 2
LT
−at e
sin(wt)u(t) ← cos(wt)u(t) ← e−at sin w t ← LT → w
s
→ →
sin tLT
sin w t ←→ (s +(as 22+) + a w) 2+ w 2
s +w
2 2
s + w2
2 e
(s + a ) s++wa2
−at
w ←  → s+a 2
LT
−at e
e−at cos w t ←
cos t
LT → s +2 aa ) 2+ w 2
LT
sinh(at)u(t) ←

a
cosh(at)ut ←

s e cos w t ←→ (s +(as )+ +w
s -a
2 2
s − a2
2 (s + a )2 + w 2
d
→ sF (s) − f (0−)
f (t) ←
dt

Chapter 1 Theory 1.indd 43 3/21/2016 11:26:49 AM


44     Chapter 1:  ELECTRIC CIRCUITS 

6  −4 − 2 s Table 1.3 |   Convolution operations


= + e6 + 35 
2 + s 
e

Operations Expression
One-sided Time reversal y(t) = x(t) * h(t)
∞ ∞
F (s) = −6 ∫ e−(2 + s)t dt + 6 ∫ e−(2 + s)t dt x(t) → h(t) → y(t)
Shift (left/right)
0 2 Multiplication L[y(t)] = L{x(t) * h(t)}
6  −(2 + s)  ∞ 6 e−(2 + s)  ∞
Addition Y(s) = H(s) . X(s)
  +−
−(2 + s)   2
= e
2+s 0
Any function convolving with the impulse signal results
6 
−1 + e−4−2s 
in the same function.
2 + s 
=
 f (t) × s(t − t0) → f (t - t0)

1.5.2 Laplace Transform of Periodic Function


1.5.1.3 Initial Value Theorem
Consider a periodic function shown in Fig. 1.53.
The initial value theorem helps to find the initial value
f1(t) f2(t) f3(t)
of f (t) from an s-domain expression. It is applicable only
if f (t) does not contain any impulse function.

lim f (t) = lim [sF (s)] T


t→ 0 s→∞

Figure 1.53 |   Periodic function.


1.5.1.4 Final Value Theorem
The Laplace transform for the periodic function can be
The final value theorem helps to find the final value of found as
f (t) from an s-domain expression. It is applicable only if f(t) = f1(t) + f2(t) + f3(t) + …
f (t) at t = ∞ exists. f(t) = f1(t) u(t) + f1(t − T) u(t − T) + …
lim f (t) = lim[sF (s)] F(s) = F1(s) + F1(s) e−sT + F2(s) e−2sT + …
t →∞ s→ 0
= F1(s) [1 + e−Ts + e−2Ts + …]
1
F (s) = F1(s)
1 − e−Ts
Problem 1.74: Find f(∞) for f (t) = (1 − e−at )ut.

Solution:

f (t) = (1 − e−at )ut


Problem 1.75: Find Laplace transform for the follow-
ing waveform.
f(t)
1 1 a
F (s) = − =
s s+a s(s + a)
A
 a 
lim[sF (s)] = lim   = 1 = f (∞) T 2T
s→ 0 s→ 0  s + a 
Solution: f1(t) = k[u(t) - u(t - a)]
Ans. (1)  1 e−as 
F1(s) = k  − 
 s s 
1.5.1.5 Convolution For periodic function,
f (s) k (1 − e−as )
Convolution involves the following four basic operations f (s) = 1 −Ts =
as shown in Table 1.3. 1−e s (1 − e−Ts )

Chapter 1 Theory 1.indd 44 3/21/2016 11:27:12 AM


1.6  TRANSIENT RESPONSE TO DC AND AC NETWORKS     45

Problem 1.76: Find the Laplace transform of the fol- f1(t)


lowing function.

0 1 2 t
1

f2(t)

t
0 1 2 0 1 t

Solution:
 2p 
f1(t) = 1. sin  t u(t) = sin(pt) u(t)
f(t) = M1G01(t) + M2G02(t)  T 
3 1  f2(t) = sin [p (t - 1)] u(t - 1)
= t [ u(t) − u(t − 1)] +  − t[ u(t − 1) − u(t − 2)]
 2 2 
f(t) = sin pt u(t) + sin (p (t - 1)) u(t - 1)
3 1 −3 p e−sp
= tu(t) − tu(t − 1) + u(t − 1) − t u u(t − 2) f (s) = +
s2 + p 2 s2 + p 2
2 2 2
1
+ + u(t) p
2 = (1 + e−s )
s + p2
2
 3 1  1 −3
= r(t) − u(t − 1)  t − − t + t
 2 2  2 2
1 t
= r(t) − (t − 3)u(t − 1) + u(t − 2) 1.6  TRANSIENT RESPONSE TO DC
2 2 AND AC NETWORKS
1 (t − 3)
= r(t) − (t − 1)u(t − 1) + u(t − 1) +
2 2
The transients in the network are due to the energy stor-
1 (t − 2) 1
= r(t) − (t − 1)u(t − 1) + u(t) = u(t − 2) ing elements of the opposite kind (L and C). If the net-
2 2 2 work consists of only resistors, no transients will occur
e−s e−2s 1 e−2s
at the time of switching. A resistor can accommodate
1
= − + − any amount of current through and any amount of volt-
s2 2s2 s2 2 s age across it. Since the energy in system cannot change
1  e−s  1 e−2s instantaneously, so the energy stored in the inductor and
−2 s 
= 1 − + e  − capacitors cannot change instantaneously, that is, the
s2  2  2 s inductor current and capacitor voltage cannot change
instantaneously. Hence, this behaviour leads to the tran-
sients in the network (Fig. 1.54).
Problem 1.77: Find Laplace transform for the given
waveform. S
f(t)
+ t=0
V − N −∞ 0− 0 0+ ∞

1
0− = −0.000.......1
0+ = +0.000.......1
Figure 1.54 |   Transients in a network.
t(seconds)
0 1
Now, the behaviour of the inductor and capacitor at t = 0+
Solution: For the given circuit: and t = ∞ is as follows:

Chapter 1 Theory 1.indd 45 3/21/2016 11:27:34 AM


46     Chapter 1:  ELECTRIC CIRCUITS 

  1.  Inductive impedance at t = 0+, s → ∞ The inductor current at t = 0− and t = 0+


t 0− t
ZL = sL
∫ ∫ L ∫− L
1 1 1
iL (t) = VL (t)dt = VL (t)dt + V (t)dt
as s → ∞, ZL = ∞. Here, L is open circuit behaviour. L L
−∞ −∞ 0
  2.  Capacitive impedance at t = 0+, s → ∞ t
= iL (0− ) +
L ∫− L
1
1 V (t)dt
ZC = 0
sC
+
1 We want i only at t = 0 , so
as s → ∞, ZC = = 0. Here, C is short circuit 0+
∞ iL(0+ ) = iL(0− ) + ∫ VL (t)dt
1
behaviour.
L
  3.  Inductive impedance at t = ∞, s → ∞   0−
+ −
ZL = sL    iL(0 ) = iL (0 ) + 0
as s → 0, ZL = 0. Here, L is short circuit behaviour. + −
   iL (0 ) = iL(0 )
  4.  Capacitive impedance at t = ∞, s → 0 So, the inductor current just after the switching is equal
1 to the current before switching, so the inductor current
ZC = cannot change instantaneously.
sC
as s → 0, ZC = ∞. Here, C is open circuit behaviour.
For all excitations except impulse,
E L(0+ ) = E L(0− )
In circuits, voltage and current conditions are continu-
ally changing. We therefore need to include the effects 0+
If VL(t) = d (t), then iL(t) = iL (0− ) + ∫ d (t)dt
1
of time and transient events on circuit conditions. The
L
transients are of two kinds based on the type of voltage: 0−

iL(t) = iL(0− ) +
1
  1.  DC Transients
  2.  AC Transients L
So, the inductor current can change instantaneously for
1.6.1 Initial and Steady State impulse voltage across it,

E L(0− ) = L iL (0− )2 Joules


1
Whenever the impedance source is connected to the net- 2
work for a long time, the network is said to be in steady 1
state. In steady state, energy stored in the network is E L(0+ ) = L iL (0+ )2 Joules
2
The capacitor voltage at t = 0− and t = 0+
maximum and constant, that is, the energy stored in
the inductor and capacitor is maximum and constant.
t


Hence, 1
VC (t) = i(t)dt
1 2 C
Li = maximum and constant and i L = maximum −∞
2 and constant. 0− t

∫ ∫ iC(t)dt
1 1
1 = iC(t)dt +
CV 2 = maximum and constant and VC = maximum C
−∞
C
0−
2 and constant. t
= VC (0− ) + ∫ iC(t)dt
1
when iL = maximum and constant
C
0−
di
VL = L L ⇒ VL = 0 We want voltage only at t = 0 +
dt
0+


+ 1
The inductor short circuits in steady state. In steady VC (0 ) = VC (0 ) + iC(t)dt
C
state condition, inductor acts as a constant current 0−
source. Since the current is maximum and constant, VC VC(0+) = VC(0−)
= maximum and constant EL(0+) = EC(0−)
dVC
iC = C ⇒ iC = 0 The voltage across the capacitor cannot change instan-
dt taneously for all excitations except impulse for impulse
The capacitor is open in steady state. In steady state, current.
VC (0+ ) = VC (0− ) +
the capacitor acts as constant voltage source, since the 1
voltage across capacitor is maximum and constant. C

Chapter 1 Theory 1.indd 46 3/21/2016 11:28:13 AM


1.6  TRANSIENT RESPONSE TO DC AND AC NETWORKS     47

R ←→ R W
The equivalent circuits in transient for current free inductor LT
and voltage free capacitor are shown in Figs. 1.55 and 1.56,
respectivley.
→ sL W
L ←
iL(t) iL(t)
+ +
W
1
C ←

sC
VL(t) L I0 VL(t) L I0
The transformations are depicted as follows in Fig. 1.57
− −
with characteristic equations.

Figure 1.55 |   Current free inductor.


iR(t) IR(s)
iC(t) iC(t) + +
+ +
+ +
− V0
VR(t) R VR(s) R
VC(t) C V0 VC(t)

+

C − −
− −
VR (t) = RiR (t) (a)    VR (s) = I (s)R
Figure 1.56 |   Voltage free capacitor.
      

1.6.2 DC Transients iL(t) IL(s) IL(s)


+ + +
Capacitors and inductors, when wired in series with
I0 LI0 +
resistors, react over a period of time to sudden changes −
in DC voltage (transient), as they absorb and release VL(s) I0
VL(t) L sL s VL(s)
energy. This response of capacitors and inductors to DC
sL
transient voltage can be determined for the following
cases.
− − −
Case 1: Source-free circuits
(b)
These are circuits without any independent source.             
di (t) di
There are three types of source-free circuits: VL (t) = L L VL = L L = L(sI L (s) − i L(0+ )]
dt    dt
  1.  Source-free RL circuit: Here L consists of initial
condition I0. 1 I
I L (s) = VL (s) + 0
  2.  Source-free RC circuit: Here C consists of initial sL s
condition V0.
  3.  Source-free RLC circuit: Here L consists of initial
condition I0 and C consists of initial condition V0. iC iC IC(s)
+ + +
Case 2: Circuits with sources
+ + −V0
With sources, initial condition is (t = 0) and final condi- + − V0 − s
tion is (t = ∞), so VC C V0 VC VC(s)
− 1
iL(0 ), VC(0 ), iL(∞), VC (∞)
+ + C
sC
Since the inductor and capacitor effects are eliminated − − −
from the network at t = 0+ and at t = ∞, the nature of (c)
the circuit at these two instances is resistive.
dVC (t)
Case 3: Laplace transform iC = C IC (s) = C[sVC (s) − VC (0+ )]
dt
Figure 1.57 |   Laplace transformations and characteristic
The Laplace transform approach is used for solving
problems in the presence of sources for t > 0. It can be
equations.
used for RLC circuits but not required for RL and RC
circuits. The Laplace transform approach involves

Chapter 1 Theory 1.indd 47 3/21/2016 11:28:40 AM


48     Chapter 1:  ELECTRIC CIRCUITS 

1.6.2.1 Source Free RL, RC and RLC For resistive circuit shown in Fig. 1.59, at t = 0+, we have
Circuits i(0+) = ke−0, iL(0+) = I0, k = I0.
So,
i(t) = I 0 e−(R / L)t for t ≥ 0
Source Free RL Circuit
In a source-free RL circuit, the source is suddenly discon-
i(t) = I 0 e−t /t where t =
nected and we select the inductor current as the response L
s
since inductor current cannot change instantaneously. R
i(t) = 0.0067 I0 < 0.1% of I0 ≈ 0
Figure 1.58 (a) shows source free RL cicuit and the cor-
At t = 5t,
responding equivalent circuit is shown in Fig. 1.58(b).
This can be represented as shown in Fig. 1.60.

i
i
0.368 I0
I0
− + R L I0 0.00657 I0
t=t t = 5t
R VR VL L
+ −
t ≥0 Figure 1.60 |   Plot of time vs. current in source free RL

circuit.
(a) (b)
Figure 1.58 |   Source free RL (a) circuit and
(b) equivalent circuit. Problem 1.78: Determine the time constant of the
circuit shown in the figure given below.
Based on current direction, separating the current with
a current source, we get 2Ω 1Ω
−VL − VR = 0
di
−L = iR = 0 +
1V − 2Ω 1H
dt
di
L + iR = 0
dt
di  R 
+  i = 0 Solution: We have
dt  L 
 L 
  s
d R t = 
Let
dt
= D , so Di + i = 0
L  Req 

  Now, Req = 2||2 + 1 = 2 Ω


D + R  i = 0
 L 
or

So, t =
1
R s
Characteristic equation is D + =0 2
L Ans. (0.5)
R
D =−
L
Problem 1.79: Determine the time constant in the
Solution of first order equation, circuit given in the following figure.

i(t) = ke−(R / L)t (for t ≥ 0) 20 Ω 1H 2Ω

i(t)
5A 10 Ω 2H 2H
R I0

Solution: Current passing through 1 H inductor and


Figure 1.59 2 Ω resistor is the same.

Chapter 1 Theory 1.indd 48 3/21/2016 11:29:09 AM


1.6  TRANSIENT RESPONSE TO DC AND AC NETWORKS     49

20 Ω 2Ω 1H t =
L
=
5
=
1
Req 50 10

iL (t) = 2.7e−10 t A
5A 10 Ω 2H 2H
(iii) At t = 0+
10 Ω

+
20 Ω 2Ω 40 Ω V(0+) VL(0+) 2.4 A

5A 10 Ω 2H V(0+) = 40 (−2.4) = −96 V


−V40 + V10 + VL (0+) = 0
− 96 + 10 × 2.4 + VL(0+) = 0
Leq VL(0+) = 72 V
t =
2 1
= = s
Req 12 6
Ans. (1/6)
Source Free RC Circuit
A source-free RC circuit is a circuit in which DC source
Problem 1.80: For the circuit shown in the follow- is suddenly disconnected and the energy which is already
ing figure, the switch is closed for long time and it is stored in the capacitor is dissipated through the resistor
opened at t = 0. Determine iL(0+), V(0+), iL(t) for (Fig. 1.65).
t ≥ 0. v iC
iR
10 Ω +
R C v0
+ t=0 −
40 Ω V 5H
− 24V
iL (0V)
Figure 1.61 |   Source free RC circuit.
Solution:  (i) At t = 0−
Here,
10 Ω iR + iC = 0
v dv
+ +C⋅ =0
40 Ω 24 V R dt
− iL (0−) dv 1
+ ⋅v = 0
dt RC
 
D + 1  v = 0
iL (0− ) =  RC 
24
= 2.4 A = iL (0+ )
10
⇒ v(t) = ke−t /RC , t ≥ 0
i40 W (0− ) =
24
= 0.6 A
40 At t = 0, the circuit is as shown in Fig. 1.62
VL(0−) = 0 V v

(ii) At t ≥ 0 v0
+
10 Ω −
R
+ +
C
40 Ω 5H VL 2.4 A
− −
t ≥0
−t /t
iL (t) = 2.4e A Figure 1.62

Chapter 1 Theory 1.indd 49 3/21/2016 11:29:36 AM


50     Chapter 1:  ELECTRIC CIRCUITS 

v(0+ ) = v0 Solution: First we calculate Req across capacitor


−t / RC
(for t ≥ 0) 5 × (8 + 12) 5 × 20 5 × 20
=4W
V (t) = ke
Req = = =
V (0 ) = ke ⇒ k = V0
+ 0 5 + (8 + 12) 5 + 20 25

V (t) = V0 e−t /t (for t ≥ 0) t = ReqC = 4 × 0.1 = 0.4 s

where, t = RC seconds v = v(0)e−t /t = 10e−t / 0.4 V


This equation shows that voltage response of circuit
is an exponential decay of its initial value, that is, initial ⇒ VC = 10e−2.5t V
voltage V0 ( Fig. 1.63). When there is no external source Using voltage division rule, we get
of excitation present in the circuit, then the response
× V = 0.6 × (10e−2.5t ) = 6e−2.5t
12
obtained is called natural response of the circuit. Here, vx =
the time constant is t = RC. 12 + 8
6e−2.5t
= 0.5e−2.5t
vx
ix = =
V0
12 12

0.368 V0

Problem 1.82: The switch in the circuit has been


0.00657 V0
t closed for a long time, and is opened at t = 0. Find
0 T 5T v(t) for t ≥ 0. Calculate the initial energy stored in
Figure 1.63 |   Plot of voltage vs. time for source free the capacitor.
RC cicuit. t=0
3Ω 1Ω
  1.  When the network consists of several independent
sources, multiple resistors and multiple inductors
(separable) or single inductor, then inductor cur- +
rent for t ≥ 0 is 24 V + 9Ω

v 20 mF

i L(t) = i L(∞) + [i L(0) − ie(∞)]e−t /t
Leq
t =
L L
R
=
Req
=
Req Solution: For t < 0, since capacitor is open circuit to
DC, we calculate voltage across 9 Ω resistor which is the
diL (t)
VL (t) = L ,(for t ≥ 0) initial voltage of capacitor. By using voltage division rule,
dt
9 9
vC (t) = (24) = × 24 × 18 V  (for t < 0)
  2.  When the network consists of several independent 9+3 12
sources, multiple resistors, and multiple capacitors ⇒ vC(0) = V0 = 18 V
Now, for t > 0
(separable) or single capacitor, then the capacitor
voltage is
1Ω
VC (t) = VC (∞) + [VC (0) − VL (∞)]e−t /t (for t ≥ 0)
t = RC = Req C = Req Ceq
+
dV (t) 9Ω 15 V 20 mF
iL(t) = C C −
dt

Problem 1.81: In the figure shown, let VC (0) = 10 V.


Find v,VC, vx and ix for t > 0. Req = 1 + 9 = 10 Ω
8Ω t = ReqC = 10 × 20 × 10−3 = 0.2 s
i01 V(t) = V0 e−t/t = 18e−t/0.2 ⇒ V(t) = 18e−5t V
The initial energy stored in the capacitor is
+
+
5Ω − C
V 4 Ω Vx
CV02 = × 20 × 10−3 × 18 × 18 = 3.24 J
0.1 F 1 1
− WC (0) =
2 2
Ans. (3.24)

Chapter 1 Theory 1.indd 50 3/21/2016 11:30:07 AM


1.6  TRANSIENT RESPONSE TO DC AND AC NETWORKS     51

v
Problem 1.83: For the circuit shown in the figure
below, the switch is closed for a long time and it is i
opened at t = 0. Determine VC (0+), i(0+) and VL
(2 ms). R L C

500 Ω t = 0 500 Ω

Figure 1.64 |   Source free parallel RLC circuit.

1 µF
2 kΩ + The nodal equation for the circuit is
50 V

t
+ ∫ v dt − i(t0 ) + C
v 1 dv
i =0
R L dt
t0
Solution: At t = 0−
Initial conditions are as follows:
500 Ω 500 Ω
i(0+) = I0
v(0+) = V0
+ Differentiating both sides w.r.t. t, we get
50 V 2 kΩ
− d2v 1 dv 1
C 2
+ + v=0
dt R dt L

VC(0-) =
50
× 2 × 103= 40 V = VC(0+) Solution of above differential equation is as follows.
2.5 × 103 Assuming v = Aest,
At t = 0+ 1 1
500 Ω CAs2est + Asest + Aest = 0
R L
− +
 1
Aest Cs2 + s +  = 0
1

+
+ R L
2 kΩ

i(0+) − 40 V
For this equation to be satisfied for all time, one of its
three factors must be zero. So,
1 1
40 Cs2 +
s+ = 0
i(0+ ) = = 16 mA R L
2.5 × 103
This is an auxiliary/characteristic equation:
At t ≥ 0
 1 2
+ 
1 1
s1 = − −
500 Ω 2RC  2RC  LC

 1 2
− 
1 1
s2 = − −
 2RC 
+
2 kΩ i VC 1 µF

2RC LC

v1 = A1es1t and v2 = A2es2 t


−6 −3
VC (t) = 40e−t /2.5×10 ×1×10 = 40e−t /2.5×10 V d 2 v1
3
1 dv1 1
C 2
+ + v1 = 0
VC(0−) = VC(0+) = 40 V
dt R dt L
dv2 1 dv2 1
t = 2.0 ms C 2
+ + v2 = 0
R dt L
VC (2 ms) = 40e−2/2.5 = 17.97 V
dt
From the above two equations, we have
Ans. (40, 16, 17.97)
d 2 (v1 + v2 ) 1 d(v1 + v2 ) 1
C 2
+ + (v1 + v2 ) = 0
dt R dt L
Source Free RLC Circuits (Parallel and
Series) v(t) = A1es1t + A2es2 t

s1 = −a + a 2 − w 02
Consider the circuit given in Fig. 1.64 for source free
parallel RLC circuit.

Chapter 1 Theory 1.indd 51 3/21/2016 12:28:47 PM


52     Chapter 1:  ELECTRIC CIRCUITS 

s2 = −a − a 2 − w 02 C
d2v
+
1 dv 1
+ v=0
2 R dt L
dt
where a = ,w =
1 1
. Using i = Aest, we have
2RC 0 LC
Consider the source free RLC series circuit shown in
Fig. 1.69(a) along with its dual circuit shown in Fig. 1.65(b) 1
LAs2est + RASest + Aest = 0
− vC +
C

 1
Aest Ls2 + Rs +  = 0
i iL iL
C

+ + or
R L C v C
R L vL
− − 1
Auxiliary equation is Ls2 + Rs + =0
C
       (a)         (b)
−R  R 2
Figure 1.65 |   (a) Series RLC circuit. (b) Dual of series ±   −
1
 2L 
s1,2 =
RLC circuit. 2L LC
The fundamental integral-differential equations for s1,2 = −a ± a 2 − w 02
Fig. 1.65 are
−R
t

∫ i dt − vC (t0 ) = 0   a =
di 1
For Fig. 1.65(a) L + Ri + Therefore,
dt C 2L
t0
t
w0 =
1
+ v + ∫ v dt − iL(t0 ) = 0
di 1 1
For Fig. 1.65(b) C and
dt R L LC
t0

The respective second order equation obtained by dif- Important Relations for Source Free RLC
ferentiating these equation w.r.t. t is The important relations for source free RLC circuits are
2
d i di 1 listed in Table 1.4.
L 2
+R + i=0
dt dt C

Table 1.4 |   Summarised realtions for source free RLC


Type Condition Criteria a w0 Response

a > w0 A1es1t + A2es2 t s1,2 = −– + – − w 0


1 R 1 2 2
Parallel series Overdamped and
2RC 2L LC

Parallel series Critically a = w0 a < w0 1 and R 1


damped 2RC 2L LC
Parallel series Undamped 1 R 1 e−at (A1t + A2 )
e−a t (B1 cos w d t + B2 sin w d t) where
and
2RC 2L LC
w d = w 02 − a 2

Problem 1.84: S is in position `a’ for a long time. S a


moved to position `b’ at t = 0. At t = 0+, the values
of IC and VL are b
IC 1Ω
IS = 5 A 1F 4Ω +
VL

Chapter 1 Theory 1.indd 52 3/21/2016 12:29:33 PM


1.6  TRANSIENT RESPONSE TO DC AND AC NETWORKS     53

Solution: At t = 0-
Applying KVL equations to the equivalent circuit, we
have
di
1Ω
Ri + L = V 0 u(t)
dt
VC(0−) 4 Ω
IS = 5 A Now, i(t) = 0 for t < 0. For positive time, u(t) is unity.
I2(0−)
For t > 0
di Ldi
Ri + L = V 0 = dt
VC × 5 dt V 0 − Ri
= 5 ⇒ VC = 4 Volts
4
Integrating both sides, we get
4
IL = 5 × =4A L
4 +1 − ln(V0 − Ri) = t + k
R

1Ω value of `k’ can be found by using initial conditions of the


circuit. We have i (0−) = 0 and i(0+) = 0.
IC
4Ω +
4V VC 4A Setting i = 0 at t = 0, we get

L
By superposition theorem − ln V0 = k
        R
1Ω L
I1 − [ln(V0 − Ri) − ln V0 ] = t
4V 4Ω R
V0 − Ri
= e−Rt / L
V0
I1 = 1 A         
i = 0 − 0 e−Rt / L (where t > 0)
V V
1Ω R R
4Ω
I2
V 
i =  0 − 0 e−Rt / L  u(t)
4A V
 R R 
I2 = 4 A
(1 − e−tR / L )
V0
IC = I1 + I2 = 5 A (increasing) So,  i(t) =
R
Therefore, —5 A (deceasing)
VL = 0 For charging inductor (with DC source), current response
Ans. (5, 0) rises exponentially, and voltage across inductor response
decays exponentially.
1.6.2.2 Source Driven RL, RC and RLC Note: The general formula for calculating the current
Circuits through inductor is

Source Driven RL Circuit i(t) = [i(0+ ) − i(∞)]e−tR / L + i(∞)


When a DC source is applied to RL circuit, the voltage
or current source can be modelled as a step function and Problem 1.85: In the network shown in the figure
the response is known as step response. Consider the below, a steady state is reached with the switch K
driven RL circuit show in Fig. 1.66(a) and the equivalent open. At t = 0, the switch K is closed. For the ele-
circuit in Fig. 1.66(b). ment values given, determine the values of VA(0—)
i(t) and VA(0+).
t=o i(t)
R 10 Ω
R
10 Ω VA 20 Ω
+ VB

V0 + V0u(t) L

L
K
2H iL
5V
(a) (b) 10 Ω
Figure 1.66 |   (a) Driven RL circuit with switch
and (b) equivalent circuit.

Chapter 1 Theory 1.indd 53 3/21/2016 12:29:58 PM


54     Chapter 1:  ELECTRIC CIRCUITS 

Solution: The steady state is reached with switch K Source Driven RC Circuit
open. When a DC source is applied to an RC circuit, the
i1 10 Ω ­voltage or current source can be modelled as a step
­function and the response is known as step function.
i2 VA Figure 1. 67 (a) shows the circuit for a driven RC­
5V 10 Ω 20 Ω circuit and the ­equivalent circuit is shown in Fig.
1.67(b).
iL t=0 i(t) i(t)
R R

+ +
5 2
iL = ×4 = A V0 + V(t)+ V0u(t) V (t)
− − −
C C
30 3 −
Therefore,  

VA (0− ) = i2 × 20 =
10 2 10 (a) (b)
× × 20 =
Figure 1.67 |   (a) Driven RC circuit with switch
V
40 3 3
iL(0—) = iL(0+) (b) equivalent circuit.
At t = 0, K is closed Since the voltage of a capacitor cannot change instanta-
neously, so
10 Ω V(0−) = V(0+) = 0
VA The KVL equations are as follows:
VB
10 Ω 20 Ω V0 = Ri(t) + ∫ i(t) dt    (for t > 0)
1
5V C
10 Ω 2/3 A   
di i
0=R +
dt C
di i
V A − 5 V A V A − VB 0= +
dt RC
+ + =0
10 10 20
i(t) = k e−t /RC
VB − 5 VB − V A
10
+
20
+ iL = 0 At V(0−) = V(0+) = 0 V. Therefore,
2(VA − 5 +VA) + VA −VB = 0 V0
i= = ke0
5VA −VB = 10 R
2
2(VB − 5) +VB − VA = − × 20 or k=
V0
3 R
40 −10
3VB − VA = 10 − = V0 −t /RC
3 3 and i(t) = e
−50
R
15VB − 5V A = For charging capacitor (with DC source), current
3
response decays exponentially. Voltage across the capac-
5VA − VB = 10 itor response rises exponentially.
−20 The general formula for calculation of capacitor
14VB =
3 voltage is
20 10 VC (t) = [VC (0+ ) − VC (∞)]e−t /RC + VC (∞)
VB = − =−
3 × 14 21
10 200 40
⇒ VA (0+ ) = 5VA = 10 − ⇒ V1.86:
10 200 40 +
5VA = 10 − = =Problem A (0 ) For
= the network shown in the figure
21 21 21 21 21 21
given below, the switch is in position 1 for a long time
10 40  10 40 
 ,  Ans.  ,  and is moved to 2 at t = 0. Determine i(0+).
 3 21   3 21 

Chapter 1 Theory 1.indd 54 3/21/2016 12:30:32 PM


1.6  TRANSIENT RESPONSE TO DC AND AC NETWORKS     55

1
C1 Solution: At t = 0−,
2Ω
2
R
V R L 1Ω 1Ω
i(t)
C2

−V −V
60 V
VC(0−)
V
(a) 0   (b)     (c)     (d) iL(0−)
R 2R 2R
Solution: The equivalent circuit is
VC1
At t = 0−, steady state is reached
+
R
iL
iL (0− ) =
60
V + = 20A = iL (0+ )
VC2 3
− VC(0−) = 20 V = VC (0+)
At t ≥ 0
At t = 0−, −V + VC 1(0− ) + 0 = 0
2Ω
VC 1(0− ) = V = VC 1(0+ )
iL(0− ) = 0 A
At t = 0 , + 1Ω 1Ω
V
+−
60 V +
R L − 20 V
R i1 (0+) 20 A C

Ri1(0+ ) + V + Ri1(0+ ) = 0
At t = 0+
i1(0+ ) . 2 R = −V
V 2Ω
i1(0+ ) = −
2R + −
+
Ans. (c) 0.5 Ω

N

Problem 1.87: For the circuit shown in the figure 60 V + + 20 V


VL(0+) −
given below, the switch is open for a long time.
Determine iC(0+) and VL(0+). − 20 A iC(0+)

2Ω
Applying nodal analysis at N, we get
1Ω 1Ω
t=0 20 − 60
+ 20 + iC (0+ ) = 0
2.5
iC(t)
−16 + 20 + iC(0+) = 0
60 V

iC(0+) = −4 A
L C

VL(0+) = 20 V
Ans. (4, 20)

Chapter 1 Theory 1.indd 55 3/21/2016 12:30:52 PM


56     Chapter 1:  ELECTRIC CIRCUITS 

Source Driven RLC Circuit (Parallel and


Solution:
­Series)
(i) At t = 0, the switch is closed. So,
Figure 1.68 shows the source driven RLC parallel circuit.

i(t) VC 1 (0− ) = VC 1 (0+ ) = 0


+
I0 R L V(t) C
t=0 − VC 2 (0− ) = VC2 (0+ ) = 0

(ii) At t → ∞, the circuit is


Figure 1.68 |   Source driven parallel RLC circuit.
iL1(∞) +
Here, 5V 1Ω
V (t) dV (t) −
+ i(t) + C = I0
L dt +
+ iL2(∞) VC1(∞)
di(t) 10 V − −
Inductor voltage, V (t) = L
dt
+
2 5V 1Ω +

d i(t) 1 di(t) i(t) I VC2(∞)
+ + = 0 −
dt2 RC dt LC LC

Figure 1.69 depicts the source driven RLC series circuit. At t → ∞, the steady state is reached, so
10
R L i L1(∞) = = 5 A = iL 2 (∞)
2
VC 1 (∞) = 0 V and VC2 (∞) = 5 V
t=0 +
V0 + Ans. (0, 5)

C V (t)
i(t) −

Figure 1.69 |   Source driven RLC circuit in series. Problem 1.89: In a series RLC circuit with DC exci-
tation, the entire steady state voltage will be dropped
Here, across
di(t) S R
L + Ri(t) + V (t) = V0 (for t > 0)
dt t=0
dV (t) L
Capacitor current i(t) = C V
dt
i(t)
2
d V (t) R dV (t) 1 V
2
+ + V (t) = 0
dt L dt LC LC C
(a) R only      (b) R and L only
Problem 1.88: Determine the steady state voltages (c) R and C only (d) C only
across the capacitors.
Solution: We have t → ∞, the circuit is in steady
1H
state:
R
t=0 1Ω + −
0V
V
+
VC1

1F
+
10 V − 1H − +
VC(∞)
+ VL (∞) + 0 − V = 0
VC2 1F
− ⇒ VC (∞) = V volts

Ans. (d)

Chapter 1 Theory 1.indd 56 3/21/2016 12:31:27 PM


1.6  TRANSIENT RESPONSE TO DC AND AC NETWORKS     57

1.6.3 AC TRANSIENTS 1.6.3.2 RL Circuit with AC Excitation

In AC circuits with respect to selection of operating fre- The RL circuit with AC excitation is shown in Fig. 1.70.
quency, circuit elements and switching operation, it is S R
possible to obtain transient free response. In DC circuits,
it is not possible to obtain transient free response. The t=0
network with AC excitation is analysed in steady state V(t) = Vm sin(ωt+f) L
using only by “phases”.
i(t)

1.6.3.1 Phase Notations Figure 1.70 |   RL circuit with AC voltage excitation.

The phase notation is defined for the cosinusoidal signal. The current response can be determine as
The sinusoidal signals are converted into cosinusoidal i(t) = itr(t) + iss(t)
signal by subtracting 90° from the phase. −
R
t
j(w t + f ) = ke L + iss (t)
V (t) = Vm cos(w t + f ) = RP ⋅ [Vm e ]
By using Laplace transformation application, we get
= RP ⋅ [Vm e j −f ⋅ e jw t ] = RP ⋅ [V ⋅ e jw t ] I (s) 1
= H(s) =
V (s) R + sL
where V is the phasor of the given V(t), V = Vm e jf is the
H( jw ) =
1
R + jwL
exponential form used for multiplication and division,
V = Vm ∠f is polar form, and V = Vm (cos f + j sin f )is
H( jw ) =
rectangular form used for addition and subtraction. Here 1
RP denotes the real part of the function. R + (wL)2
2

For current excitation, I (t) = I m cos(wt + q )  wL 


H( jw ) = ∠ − tan−1  
1
 R 
Current phasor is R + (wL)
2 2

I = I m e jq  = I m ∠q  = I m (cos q + j sin q ) −
R
  wL 
sin wt + f − tan−1  
t Vm
L
  R 
i(t) = ke +
V R 2 + (wL)2 
Impedance phasor is Z =
I
I The current response can also be determined by using
Admittance phasor is Y = phasor method in steady state as shown in Fig. 1.71.
V
If the voltage excitation is R

V (t) = Vm sin(wt + f ) = Vm cos(w t + f − 90°) + + −



V∠f -90°
= RP ⋅ [Vm e j(wt +f −90°) ] = RP ⋅ [Vm e j(f −90°) ⋅ e jwt ]
jwL
  I
Therefore, Vm = Vm e j(f −90°)
= V ∠f − 90° −
  m
= Vm [cos(f − 90°) + j sin(f − 90°)] Figure 1.71 |   Phasor method for determining current
response.

Problem 1.90: Write the phasor form of Vm ∠f − 90° − I ⋅ R − jwL ⋅ I = 0


(i) V(t) = 5 cos(2t + 30°) Vm ∠f − 90°
I=
R + jwL
(ii) V(t) = 5 sin(2t + 30°

Solution: Phasor forms are given by Vm ∠f − 90°


=
 wL 
(i)  V = 5e j30° = 5∠30° = 5[cos 30° + j sin 30°] R 2 + (wL)2 . tan−1  
R
 3 j  wL 
= 5  +  ∠f − tan−1   − 90°
Vm
 R 
=
 2 2 
     
R + (wL)
2 2

1 3 
(ii)  V = 5.e j(−60°) = 5∠ − 60° = 5  − j Therefore,
 2 2 
I = a∠b = ae jb

Chapter 1 Theory 1.indd 57 3/21/2016 12:32:17 PM


58     Chapter 1:  ELECTRIC CIRCUITS 

Thus,  wL 
wt0 = tan−1  
i(t) = RP [I ⋅ e jwt
]  R 

= RP[ae jb ⋅ e jwt ]
If the excitation is
V (t) = Vm cos(wt + f )
= RP [ae j(wt + b ) ]
Then the complete response through the inductor is
= a cos(wt + b )
i(t) = itr(t) + iss(t)
  wL  
= a cos w − f − tan−1   − 90° 
  R   wL 
R
 −
cos wt + f − tan−1  
t Vm
L
  R 
i(t) = ke +
  w  R 2 + (wL)2
= a sin wt + f − tan−1  
L
  R 
where,
  wL 
. sin w t + f − tan−1   −Vm   wL 
Vm
iss (t) =
  R  cos f − tan−1  
  R 
k=
R + (wL)
2 2
R 2 + (wL)2
R
− t
Therefore, i(t) = ke L + iss (t) In this, the condition for the transient-free response,
 wL  p
By substituting t = 0 f − tan−1   =
 R  2
  wL 
i(0) = ke−0 + sin f − tan−1  
Vm
  R  Therefore, at t = 0
R 2 + (wL)2  wL  p
f = tan−1   +
  wL   R  2
sin f − tan−1  
Vm
  R 
0=k+
R 2 + (wL)2 Then the condition at t = t0
−Vm   wL   wL  p
sin f − tan−1   wt0 + f = tan−1   +
k=
  R   R  2
R 2 + (wL)2 
If f = 0°
−1    wL  p
Suppose f − tan  wL 
  = 0 wt0 = tan−1   +
R  R  2
⇒ k = 0 ⇒ itr(t) = 0

Then the transient free response Problem 1.91: For the circuit shown in the figure
i(t) = iss(t) given below, the source frequency is 50 Hz.
(This perfect circuit is a hightly desirable state.) Determine the value of t0 which results in a transient
free response.
So, the condition for transient free response at t = 0 is
5Ω
−1  
 wL 
f = tan
 
R t=0
There is no transient at the time of switching. So f sin(ωt)
+
0.01H
depends upon circuit elements and applied frequency.
f = 50 Hz −
Note: From the above condition, if the total
phase of the excitation at the time of switching is
 wL 
equal to tan−1   , then no transients will result in
 R  Solution: At t = t0 only
 wL 
the system for the sinusoidal excitation at the time of wt0 = tan−1  
 R 
switching.
 2pf ⋅ L 
2pf ⋅ t0 = tan−1 
Now if the switch is closed at t = t0 instead of t = 0  R 
then the condition for the transient free response is
tan−1(2p × 0.1)
1
 wL  t0 =
wt0 + f = tan−1   2p ⋅ 50
 R 
1 .
If f = 0° = 32.14°
100p

Chapter 1 Theory 1.indd 58 3/21/2016 12:33:01 PM


1.6  TRANSIENT RESPONSE TO DC AND AC NETWORKS     59

1 32.14 × p 25 Ω
= . = 1.78 ms V
100p 180
(a) In the same problem, if the excitation cosωt is + j30 Ω
used, then 5∠0° 100 Ω
p 3.14 −
t0 = 1.78 + = 1.78 + = 6.78 ms IL
2 2
(b) In the same problem, if the excitation sin(wt − 10°) V − 5∠0° V V
is used, + + =0
25 j30 100
wt0 − 10° = 32.14°
V
wt0 = 32.14° + 10° IL =
j30
1 Therefore,
t0 = 1.78 + = 2.34 ms
180 × 100
Ans. (1.78) i(t) = RP [I L e j3t ] A

Problem 1.92: Determine the steady state current


i(t) in the circuit shown in the following figure. Problem 1.93: In the following circuit IS = 0 for
25 Ω  p
t < 0 and system is at rest. IS = cos 3t +  for t > 0.
 4
Obtain i(t).
+ 30 mH
5 cos(103 t) 100 Ω 3Ω 1Ω
− i(t)
iL
IS 2Ω 1H
Solution: i
(i) In the first step we determine equivalent Thevenin
voltage and resistance.
25 Ω Solution: KVL for loop containing the inductor in
terms of i is
di
I × i + 1 × = (IS − i) × 2
VTh 100 Ω dt
di
+ 3i = 2IS
dt
 p
+ 3i = 2 cos 3t + 
5 cos(103 t) di
VTh = × 100 = 4 cos103 t  4
125 W dt
RTh = 25 || 100 = 20 W
Now, we have

(ii)  The steady state current is determined as follows: i = ke−pt + e−pt ∫ Q ⋅ e pt dt


20 Ω  p
= ke−3t + e−3t ∫ 2 cos 3t +  e3t dt
 4
30 mH CF PI
4 cos(103 t)
where CF is complementary function and PI is par-
iL
ticular integration. Thus
 p
  30  PI = 2e−3t ∫ e3t cos 3t +  dt

cos 103 t − tan−1   4
4
  20 
iss (t) =
Also, since cos (A + B) = cos A cos B − sin A sin B
2 2
(20) + (30)

  3  So,
cos 103 t − tan−1    p
4
  2  cos 3t +  =
= 1
(cos 3t − sin 3t)
2
(20) + (30)2
 4 2
Steady state current can also be determined by phasor 2e−3t
∫e (cos 3t − sin 3t) dt
3t
method, =
2

Chapter 1 Theory 1.indd 59 3/21/2016 12:33:29 PM


60     Chapter 1:  ELECTRIC CIRCUITS 

1 Therefore,
Putting 3t = x, we get dt = dx
3
sin (w t + f − tan−1(wCR))
2 −x Vm
PI = e ∫ ex (cos x − sin x) dx VC (t) = VC (tr)(t) +
3 1 + (wCR) 2

2 −x 
e  ∫ ex cos x dx − ∫ ex sin dx
At t = 0
=
 
0 − ke−0 + sin (f − tan−1(wCR))
3 Vm
1 + (wCR)
{ }
 cos x ⋅ ex − (− sin x)ex − ex sin dx
2 −x 2
=
3
e 
 ∫ ∫ 
 where
−Vm
=
2 −x  x
e e cos x + ∫ ex sin x ∫ ex sin dx k= sin (f − tan−1(wCR))
3   1 + (wCR) 2

2 2 Suppose,
Therefore, PI = cos x = cos 3t
f − tan−1(wCR) = 0
3 3
General solution GS = CF + PI
⇒ k = 0 ⇒ VC(tr)(t) = 0
i(t) = ke−3t +
2
So, cos 3t
3 VC(t) = Vss(t)
But i(t) = 0 at t = 0 therefore,
Thus, it is a transient free response.
2
0 = ke0 + ×1 So, the condition for the transient-free response is,
3
− 2 f = tan−1(wCR) at t = 0
k=
3
If the switch is closed at t = t0
[cos 3t − e−3t ]
2
or     i(t) =
3 wt0 + f = tan−1(wCR)

If f = 0°
1.6.3.3 RC Circuits with AC Excitations
w 0 t0 = tan−1(wCR)
Figure 1.72 shows the RC circuit with exciatation with
AC voltage. If the voltage excitation V (t) = Vmcos (wt + f ) is used, then
S R
t

. cos (wt + f − tan−1(wCR))
RC
Vm
+ t=0 VC (t) = ke +
1 + (wCR) 2
Vm sin(wt+f) C VC (t)
where
− i(t)
−Vm
k= . cos(wt + f − tan−1(wCR))
Figure 1.72 |   RC circuit wth AC voltage excitation. 1 + (wCR) 2

The capacitor voltage is given by The condition for the transient-free response at t = 0
VC(t) = VC(tr)(t) + VC(ss)(t) p
f = tan−1(wCR) +
VC (s) 1 2
= H(s) = At t = t0
V (s) 1 + sCR
  p
  f + w 0 t0 = tan−1(wCR) +
 V (s)  1
.
2
VC (s) = 
 1  sC
 R +  If f = 0°
sC 
p
H( jw ) = w 0 t0 = tan−1(wCR) +
1
1 + jwCR 2
H( jw ) = ∠ − tan−1(wCR)
1
1 + (wCR) 2

Chapter 1 Theory 1.indd 60 3/21/2016 12:34:11 PM


1.6  TRANSIENT RESPONSE TO DC AND AC NETWORKS     61

Problem 1.94: If a 1 W resistance is in series frequency (w), the initial phase (f ), the circuit
with a 0.5 F capacitor and a voltage source constants (R, L, C) and on the nature of the exci-
 p tations (sin or cos).
V (t) = 10 cos 2t +  , determine the steady state
 4
Problem 1.95: System is at rest for t < 0 in the figure
current in the network and value of t0 which results
in transient free response.
given below.
Solution: The steady state current is given by 1Ω A

 p 
cos 2t + − tan−1(1)
10

iss (t) = +
1 + (2 × 0.5 × 1)  VS +
− 1Ω −
2 4 1F V

 p p  10
cos 2t + −  =
10

= cos(2t)
2  4 4 2
(i) VS = sin2t
1Ω (ii) VS = u(t)
Determine V for t > 0.
10cos(2t+p/4) i(t) 0.5 F Solution: Applying KCL at node A, we get
dV V V − VS
C + + =0
I (s) 1 dt 1 1
= H(s) =
V (s) 1 dV
R+ + V = (VS − V )
sC dt
1 dV
H( j2) = ∠ − 45° + 2V = VS
2 dt

Transient-free condition dV
(i)  + 2V = sin 2t
dt
p p
2t0 + = tan−1(wCR) + GS = CF + PI
= ke−2t + e2t ∫ sin 2t ⋅ e2t dt
4 2
p p
2t0 = tan−1(1) + −

2 4 = ke−2t + e−2t ∫ e2t sin 2t dt
p    
Putting 2t = x, we get
t0 = = 0.785 s
e−x
4
∫e
x
If the exciataion voltage used in the above problem PI = sin x dx
 p
2
is V = 10 sin 2t + , then eax
 4 I= ∫ eax sin bx dx = (a sin bx − b cos bx)
p −1 p (a2 + b2 )
2t0 + = tan (1) = ⇒ 2t0 = 0 ⇒ t0 = 0 s
4 4 ex
∫e (sin x − cos x)
x
= sin x dx =
2
Note: e−x ex 1
PI = (sin x − cos x) = (sin 2t − cos 2t)
  1. Transition-free condition is not possible if circuit has 2 2 4
GS = CF + PI = ke−2t + (sin 2t − cos 2t)
both L and C elements. 1
  2.  From the following important relations, we observe 4
that: Now, V = 0 at t < 0, so
 wL p 
wt0 + f = tan−1  + 
 R 2 1 1
0 = ke0 + (0 − 1) ⇒ k =
p 4 4
wt0 + f = tan−1(wCR) +
2 1 −2 t 1
So, V = e + (sin 2t − cos 2t)
The transient-free time (t0) in RL and RC net- 4 4
work with AC excitations depends on the source

Chapter 1 Theory 1.indd 61 3/21/2016 12:34:50 PM


62        Chapter 1:  ELECTRIC CIRCUITS 

Problem 1.97: Determine the steady state voltage


(ii)  dV + 2V = u(t)
dt across the capacitor and inductor in the circuit given

Here CF = ke−2t and PI can be determined as


in Problem 1.96.

PI = e−2t ∫ u(t)⋅ e2t dt = e−2t ∫ 1 ⋅ e2t dt =


1 Solution: To determine VC(t), we have
2
VC (s) 1 1
−2t = H(s) = ×
1   sC
Therefore, GS = V = ke + V (s) R + sL + 1

 sC 
2
1 1
0 = ke0 + ⇒k=−
2 2 1
H(s) = 2
(1 − e−2t )
1 s + s+1
or  V =
2
⇒ H( jw ) =
1
−w 2 + jw + 1

2
∠ − p + tan−1  
1.6.3.4 RLC Circuits with AC Excitations 1 1
 3 
H( j2) = =
1 − w + jw
2
13
The transient-free response condition is not possible for
the networks having both the types of the elements, that  2 
sin 2t + tan−1   − p 
10
is, inductor and capacitor. Whenever the LC elements ⇒ VC (ss)(t) =
13   3  
are present in the circuit, always complex roots will
result in the transient response. So,
To determine steady state voltage across the induc-
S1S2 = a ± jb tor, we have
i tr(t) = eat⋅(k1 cos bt + k 2 sin bt)
s2 −w 2
The constant k1 and k2 are functions of sin and cos func- H(s) = and H( jw ) =
s2 + s + 1 1 − w 2 + jw
tions. So, it is not possible to suppress both k1 and k2
simultaneously to zero. Hence, the exponential term is
always present in the transient response. −4 4∠ + p
Therefore, H( j2) = =
1 − 4 + 2j 2j − 3

∠ − p + tan−1(2/3) + p
Problem 1.96: Determine the steady state current in 4
=
the circuit shown in the given figure below. 13
1Ω 1Η ∠ tan−1(2/3)
4
=
13
  2 
sin 2t + tan−1  
40
+
  3 
VL (s) =
10 sin 2t 1F 3
− i(t)

Solution: We have 1.7  SINUSOIDAL STEADY STATE


I (s) 1
ANALYSES USING PHASORS
= H(s) =
V (s) 1
R + sL + Steady state indicates the absence of transients and it is
sC
achieved after the five times of time constant (5t) of the
H( jw ) =
1
switching action (Fig. 1.73).
R + jwL +
1
jw C R1 C R2
3 − + − + −
∠ − tan−1  
1 2 +
 2 
H( j2) = =
1 13 + + − +
Vm cos(wt + f)
1 + j2 +
j2 L1 L2
Therefore, steady state current is
− i1(t) − + i2(t) −
  3 
10 × sin 2t − tan−1   A
2
  2  Figure 1.73 |   Sinusoidal steady state analyses.
iss (t) =
13 

Chapter 1 Theory 1.indd 62 3/21/2016 12:39:33 PM


1.7  SINUSOIDAL STEADY STATE ANALYSES USING PHASORS        63

Transform the network into the phasor domain, Frequency of sinusoidal does not appear in its phasor
L1 = jwL1 , i2(t) = I2 representation.

L2 = jwL2, i1(t) = I1 z = x + jy is the rectangular form

Vm ∠f °
1
C= Z = r∠f is the polar form
jw C
Vm ∠f = R1I1 + jwL1(I1 − I2 ) 
Z = re jf is the exponential form
(1.13)

I + R2 I2 + jwL2 I2 + jwL1(I2 − I1 )  (1.14)


1 .
0=
jwC 2 where r = x2 + y 2 , f = tan−1 y /x , x = r cos f and
∆ ∆ y = r sin f.
I1 = 1 , I2 = 2
∆ ∆ Addition and subtraction of complex numbers are better
Therefore, i1(t) = RP[I1e jwt ] A performed in the rectangular form. Multiplication and
i2 (t) = RP [I2e jwt ] A
division are better done in polar form (Table 1.5).

Table 1.5 |   Operations and their phasor representa-


Problem 1.98: For the circuit shown in figure given
tions
below, determine the phasors I, I1, I2, E2, E0.
I j1Ω E2 I2 0.5 Ω j 2 Ω Operation Expression

I1 Addition z1 + z2 = (x1 + x2) + j(y1 + y2)


z1 − z2 = (x1 − x2) + j(y1 − y2)
+ +
E1 = 10∠10° 1Ω 0.5 Ω Subtraction
− − E0
−j1 Multiplication z1z2 = r1r2∠f1 + f2
z1 r
V Division = 1 ∠f 1− f 2
Solution: Z= z2 r2
I
Impedance phasors,
= ∠ −f
1 1
Reciprocal
Z = j1 + (1 - j1) || (1 + j2) = 1.61∠29.74° z r
E
I= 1 =
10
∠ − 19.74° = 6.21∠ − 19.74° Square root z = r∠f/2
  Z 1.6
z* = x − jy = r∠ − f = re−jf
Complex
I (1 + j2) conjugate
I1 =
1 − j1 + 1 + j2
6.21∠ − 9.74°(1 + j2)
= = 6.21∠17.12° The relation between time domain and phasor domain
1 − j1 + 1 + 2 j are listed as follows in Table 1.6:
E2 = (1 - j1) . I1
= 2∠ − 45°I1 = 8.78∠ − 27.88° Table 1.6 |   Relation between time and phasor
domain
I (1 − j1)
I2 = = 3.927∠ − 91.30 Time Domain Phasor Domain
1 − j1 + 1 + j2

Therefore, E0 = -0.5I2= +1.963Ð-89.7 Vm cos(w t + f ) Vm ∠f

1.7.1 Phasor Vm sin(wt + f ) Vm ∠f − 90

A phasor is a complex number that represents the ampli- I m cos(wt + f ) I m ∠f


tude and phase of a sinusoid. Phasor is a mathematical
representative of an AC quantity in polar form. I m sin(wt + f ) I m ∠f − 90°

Sinusoidal form Phasor notation dv jwV


dt
v(t) = 10sin(300t + 30°) 10∠30° V
v(t) = 15sinwt 15∠0° V ∫ vdt V
jw
v(t) = 0.05sin(105t − 90°) 0.05∠ − 90° V

Chapter 1 Theory 1.indd 63 3/21/2016 12:40:21 PM


64        Chapter 1:  ELECTRIC CIRCUITS 

Problem 1.99: Transform the following sinusoids to Solution: We have


phasors.
− 3 jwI = 50∠75°
8I
(a)  i(t) = 6 cos (50t − 40°)
4I +
jw
I = 6∠ − 40° As w = 2,  I (4 − j4 − j6) = 50∠75°
(b)  v = −4 sin (30t + 50°) 50∠75° 50∠75°
sinv -sinA = cos (A + 90°) I= = = 4.642∠143.2° A
4 − j10 10.77∠ − 68.2°
v = 4 cos (30t + 50° + 90°)
So, i(t) = 4.642 cos (2t + 143.2°) A
= 4 cos (30t + 140°)
v = 4∠140° 1.7.2 Phasor Relationship for Circuit Elements
(c)  v = −7 cos (2t + 40°) The phasor relationship for circuit elements are listed as
−cosA = cos (A + 180°) follows:
v = 7 cos (2t + 40° + 180°)
  1.  When current and voltage are in the same phase
= 7 cos (2t + 220°)
(Fig. 1.74)
v = 7∠220°
i = I m cos(wt + f )
(d)  i = 4 sin (10t + 10°)
sinA = cos (90 - A) v = iR = RI m cos(wt + f )
= cos (A − 90°)
i = 4 cos (10t + 10° − 90°) V = RI m ∠f and I = I m ∠f
= 4 cos (10t − 80°) V = RI
Im
i = 4∠ − 80° V

I
Problem 1.100: Find the sinusoids represented by the
following phasors.
f
(a)  I = −3 + j4 Re
= 5∠126.87° 0
i(t) = 5 cos (wt + 126.87°) A Figure 1.74 |   Current and voltage in same phase.

(b)  v = j8e−j20°   2.  When current lags the voltage by 90° (Fig. 1.75):
j = 1∠90°
= −wLI m sin(wt + f )
Ldi
v=
v = j8∠ − 20° = (1 < 90°)(8 < −20°) dt
= 8∠90 − 20 = 8∠70° V v = wI m cos(wt + f + 90°)

v(t) = 8 cos(wt + 70°) V V = wLI m e j(f + 90°) = wLI m e jf e j 90°



j 90°
(c)  v = −10∠30° = wLI m ∠fe = jwLI
= ( j2 )10∠30° Im
w
= (1∠180°)(10∠30°) V
I
= 10∠210°
v(t) = 10 cos(wt + 210°)
f
(d)  I = j (5 - j12) 0
Re

= 12 + j5 = 13∠22.62° Figure 1.75 |   Current lags voltage by 90°.


i(t) = 13 cos(wt + 22.62°)   3. When the current leads the voltage by 90° (Fig. 1.76).
V = Vm cos(wt + f )
dv
Problem 1.101: Using phasor approach, determine i(t). i=C
dt
4i + 8 ∫ idt −
3di
= 50 cos(2t + 75°) I = jwCV and V =
I
dt jwC

Chapter 1 Theory 1.indd 64 3/21/2016 12:41:08 PM


1.7  SINUSOIDAL STEADY STATE ANALYSES USING PHASORS        65

Im
Problem 1.104: Determine the current I in the circuit
shown in the figure given below.
I
V 1Ω I IC = 4 ∠90°

IL
f 1 V 3 ∠0°
Re I H
ω = 3 rad/s 3
Figure 1.76 |   Current leads the voltage by 90°.

(a) 1∠90°   (b) 3∠60°   (c) 5∠ − 45°   (d) 2∠45°


Problem 1.102: In the circuit shown in the figure
given below, current I1 leads I2 by angle Solution:
V V I2
IC = 4∠90° = 4e j 90° = j4
2Ω
I1
V 3∠0°
C IL = = = 3∠ − 90° = −j3
V jwL j3 ×
1
w = 2 rad/s j2
3
I = I L + IC = j4 − j3 = j = 1∠90° A
(a) 30°   (b) 60°   (c) 90°   (d) 135°
Ans. (a)
Solution:
V V 1.7.2.1 Phasor Diagrams
I = I1 + I2 = +
ZC ZL
V V V The pictorial representation of all the phase currents and
I1 = =j = ∠90°
wC
phase voltages in a circuit is called the phasor diagram.
ZC XC
The phasor diagrams along with the characteristic equa-
V V
I2 = = ∠ − 45° tions for different circuits are discussed as follows.
2 + j2 2 2
I1 Series RL Circuit
In the RL series circuit shown in Fig. 1.77(a), we have
VR = IR and VL = IZL = IjwL = IjXL= IXL∠90°
135°
90°
V In any RL circuit due to the inductances, the cur-
45° rent lags the voltage, therefore, the phasor diagram is as
shown in Fig. 1.77(b).
I2
Ans. (d) R

VR −
+
+ +
Problem 1.103: The currents i1(t), −i2(t) and i3(t) are VL
V L
meeting at a junction
− −
i1
i2
(a)
i3
VL = IXL
i1(t) = −6sinwt mA, i2(t) = 9coswt mA, i3(t) = ?
V
Solution: Given that:
i1 = 6∠90°, i2 = 8∠0°
f
Applying KCL, we get −i1 − i2 − i3 = 0 I
−1
i3 = −(i1 + i2) = −6j − 8 = 10∠180° + tan (6/8) (b)
= 10∠180° + 36.86° = 10e j(p + 36.86°)
Figure 1.77 |   RL series (a) circuit and (b) phasor
i3(t) = 10 cos (wt + 36.86° + p) diagram.
= 10 cos (wt + 36.86°) mA

Chapter 1 Theory 1.indd 65 3/21/2016 12:41:37 PM


66        Chapter 1:  ELECTRIC CIRCUITS 

From Fig. 1.77(b), we have I R L


Voltage V = VR2 + VL2 +V − + V −
+ R R +
V 
Impedance angle is f = tan−1  L 
V VC C
 VR  − −
Power factor = cosf (lagging)
Figure 1.79 |   Series RLC circuit.
Series RC Circuit Three cases are possible depending on the capacitor and
In the series RC circuit shown in Fig. 1.78(a), we have inductor voltages, that is, VL > VC; VL < VC; and VL =
VC. The corresponding phasor diagrams are illustrated
VR = IR, so in Fig. 1.80 (a), (b) and (c), respectively and the expres-
= -IjwC = IXC∠ - 90°
1 sions for voltage, impedance angle and power factor
VC = IZC = I
jwC given in Table 1.7.
As in any RC circuit due to the capacitive nature, the VL
current leads the voltage, so the phasor diagram is as
shown in Fig. 1.78(b). (VL − VC)
R V
f 90°
VR −
+ I
+ + VR 90°
V VC C
− − VC
(a)
(a)
VL
VR = IR
I
f 90˚
VR 90°
f
I
V 90°
V
(VC − VL)
VC = IXC
(b) VC
Figure 1.78 |   RC series (a) circuit and (b) phasor
(b)
diagram.
VL
From Fig. 1.78(b), we have

V = VC2 + VR2

V 
f = tan−1  C 
I
VR
 VR 

Power factor = cos f (leading) VC


VR
Series RLC Circuit I
In the series RLC circuit shown in Fig. 1.79, we have (c)
VR = IR Figure 1.80 |   Phasor diagrams for RLC series circuit.
VL = IXL∠90° (a) VL     > VC; (b) VL < VC and
(c) VL = VC.
VC = IXC∠ − 90°

Chapter 1 Theory 1.indd 66 3/21/2016 12:41:58 PM


1.7  SINUSOIDAL STEADY STATE ANALYSES USING PHASORS        67

Table 1.7 |   Parameters for series RLC circuit conditions


Condition Voltage Impedance Angle Power Factor

 V − V 
VL > VC V = VR2 + (VL − VC )2 f = tan−1  L C
cos f (lagging)
 VR 

VL < VC  V − V 
V = VR2 + (VC − VL )2 f = tan−1  C L
cos f (leading)
 VR 

VL = VC V = VR f=0 cos f = 1 or power factor is unity. Hence the


circuit is resistive in nature and the j terms in
y or z is 0

Parallel RL Circuit Parallel RC Circuit


Consider the parallel RL circuit given in Fig. 1.81(a). From the parallel RC circuit shown in Fig. 1.82(a), we have
Here, the urrent through resistor and inductor are: V
IR =
VR V R
IR = = VC V
R R IC = = VC jwL = C ∠90°
ZC XC
VL V V
IL = = L = ∠ − 90°
ZL jwL XL IC =
VC
∠90°
XC
So, the corresponding phasor diagram is as shown in
Fig. 1.81(b). Therefore, the phasor diagram is as shown in Fig. 1.82(b)
I
I
IR IC
IR IL + +
+ + + +
V VR R VL C V
V VR R VL L V − − −
− − −
(a)
(a)
IC
V
IR = I
R
V
f f
V
IR
I
V (b)
IL =
XL Figure 1.82 |   RL parallel (a) circuit and (b) phasor
diagram.
(b)
Figure 1.81 |   RL parallel (a) circuit and (b) phasor From Fig. 1.82(b), we have
diagram.
I = IR2 + IC2
I 
From Fig. 1.81(b), we have
f = tan−1  C 
 IR 
I = IR2 + I L2
cos f = Power factor (leading)
 
−1  I L
f = tan  
 IR  Parallel RLC Circuit
cos f = Power factor (lagging) From the RLC parallel cicuits as shown in Fig. 1.83, we have

Chapter 1 Theory 1.indd 67 3/21/2016 12:42:25 PM


68        Chapter 1:  ELECTRIC CIRCUITS 

I IC
IR IL IC
+ + +
VR R VL VC V IR
− − − 90°
f
V

Figure 1.83 |   Parallel RLC circuit.


I
(IC − IL)
V V V
IR = IL = ∠ − 90°    IC = ∠90°
R    XL XC IL
There are three phasor diagrams possible, depending on (b)
the relation between capacitor and inductor current, that
isIC > IL; IC < IL and IC = IL, as shown in Fig. 1.84 (a),
(b) and (c), respectively. The expressions for character- IC
istic parameters are listed in Table 1.8.
IC

(IC − IL) V
I I = IR
f 90°
V
IR 90°
IL
(c)
IL Figure 1.84 |   Phasor diagrams for RLC parallel
(a) circuits. (a) IC > IL (b) IC < IL (c) IC = IL.

Table 1.8|   Parameters for parallel RLC circuit conditions


Condition Voltage Impedance Angle Power Factor

 I − I 
IC > IL I = IR2 + (IC − I L )2 f = tan−1  C L
cos f (leading)
 IR 

 I − I  cos f (lagging)
IC < IL I = IR2 + (I L − IC )2 f = tan−1  L C
 IR 

IC = IL I = IR f=0 cos f = 1 or power factor is unity. Hence the


circuit is resistive in nature.

Problem 1.105: In the circuit shown in the figure 8 8


(a) I1 will be lead by tan−1  , I2 will lag by tan−1  
given below, if I1 = I2 = 10 A then
6 6

I1 = 10 A I2 = 10 A 6 6
(b) I1 will lead by tan−1  , I2 will lag by tan−1  
8A
 8   8 
IL IR= 6
8 8
(c) I1 will lag by tan−1  , I2 will lead by tan−1  
120∠0° L R C  6   6 

6 6
(d) I1 will lag by tan−1  , I2 will lead by tan−1  
 8   8 

Chapter 1 Theory 1.indd 68 3/21/2016 12:42:59 PM


1.7  SINUSOIDAL STEADY STATE ANALYSES USING PHASORS        69

Solution: From the given circuity diagram |IC| = wCV = 300 × 2p × 50 × 159.23 × 10−6 = 15 A

I = IR2 + (IC − I L )2 IR = I 2 − IC2 = (25)2 − (15)2 = 20 A


Given that IC = 8 A, I = 10 A and IR = 6 A. Then VR
= 15 W
300
current through the inductor is R= =
IR 20
10 = 36 + (I L − 8)2 ⇒ I = 16 A With 360 Volts main, the circuit is
The phasor diagram is 25 A
IC = 8A

10A 360 C R = 15 Ω
I2 =
f1
f1
90° 120∠0°
f2 IR= 6A
I1 360
= IR = = 24 A
10A
IL − IC = 8 A
15

IC = 252 − 242 = 7 A
IL = 16 A
IC = Vw C
8
I2 leads by tan−1  . 7 = 360 × 2p × f1 × 159.23 × 10−6
 6 
f1 = 19.4 Hz
−1  8 
 .
 6 
I1 lags by tan Ans. (19.4)

I
Power factor cos f =
6
= R = 0.6 (lags) Problem 1.107: In the circuit shown in figure
10 I given below, the reading of the ammeter (A1) is
The power delivered by source, 6 A, and ammeter (A2) is 8 A. The reading of the
ammeter (A) is
P = Vrms I rms cos f
120 10 A
= . .(0.6) = 360
      2 2
Ans. (c)
+ R L
V

Problem 1.106: A 159.23 μF capacitor is in parallel
with a resistance R, draws current of 25 A, from a A1 A2
300 V, 50 Hz mains. Using phasor relations, find the
frequency (f  ) at which this combination draws the Solution: For the given circuit, the phasor diagram is
same current from a 360 Volts mains.
25 A 10
8
IC IR f
300 V C R 6
50 HZ
I = IR2 + I L2 = 36 + 64 = 10 A

8
f = tan−1   = 25.32°
Solution: From the circuit, we have  6 

cos f =
V 6
IC = = 0.6 (lags)
ZC 10
Ans. (10)

Chapter 1 Theory 1.indd 69 3/21/2016 12:43:27 PM


70        Chapter 1:  ELECTRIC CIRCUITS 

1.8  MAGNETICALLY COUPLED The dot convetion and equations for voltage Vo for
CIRCUITS different circuits are shown in Fig. 1.86.
i
+
M
When two coils are placed close to each other, a change
di
in current in the first coil produces a change in magnetic L1 L2 Vo    Vo = M
flux in the second coil. The change in the flux induces dt
voltage in the second coil. This voltage is called induced −
voltage and the two coils are said to have a mutual induc- (a)
tance (M). Figure 1.85 shows two coils placed together,
where a current i1 through L1 produces an open circuit i
+
voltage V2 across L2 and a current i2 through L2 pro- M
duces an open circuit voltage V1 across L1. di
L1 L2 Vo    Vo = −M
dt
M + + M

L1 L2 V2 V1 L1 L2 L1 (b)
i1 i2 i
− − M
+

Figure 1.85 |   Mutual inductance. di


L1 L2 Vo    Vo = −M
dt
Then

V2 (t) = M 21
di1(t) −
dt (c)
di2 (t) i
V1(t) = M12 +
dt M
di
Here M21 is mutual inductance of coil 2 with respect to L1 L2 Vo    Vo = M
coil 1 and M12 is mutual inductance of coil 1 with respect dt
to coil 2. Also,

M12 = M21 = M
(d)
Figure 1.86 |   Circuits with dot convention and voltage
1.8.1 DOT Convention equations.
The inductor is two terminal element and we are able 1.8.2 Series Connection of Coupled Inductors
to use the passive sign convention in order to select the
correct sign for the voltage L or jwLI .
di
Figures 1.87(a) and (b) show two circuits for series -
dt connected coupled inductors.
If the current enters the terminal at which the positive
voltage reference is located, then the positive sign is
used. Mutual inductance however, cannot be treated
in exactly the same way because four terminals are V1 i1 L1 V1 i1 L1
involved. The choice of correct sign is established by
use of one of several possibilities that include the dot
convention. M M
The dot convention makes use of a large dot placed
at one end of each of the two coils which are mutually
coupled. A current entering the dotted terminal of one V2 i2 L2 V2 i2 L2
coil produces an open circuit voltage with a positive volt-
age reference at the dotted terminal of the second coil. A    
current entering the undotted terminal of one coil pro- (a) (b)
vides a voltage that is positively sensed at the undotted
terminal of the second coil. Figure 1.87 |   Series connection of coupled inductors.

Chapter 1 Theory 1.indd 70 3/21/2016 12:43:50 PM


1.8  MAGNETICALLY COUPLED CIRCUITS        71

For the circuit shown in Fig. 1.87(a), the equations for di1 di (L − M )
= 2 2
dt (L1 − M )
magnetic flux and inductance are:
dt
f = Li
di di2 (L2 − M ) di2
f = f1 + f 2
= +
dt dt (L1 − M ) dt
f1 = L1i1 + Mi2 di  L − M  di2
= 1 + 2 
f 2 = L2 i2 + Mi1 dt  L1 − M  dt
di
f = L1i1 + Mi2 + L2 i2 + Mi1 V = Leq
dt
i1 = i2 = i di di di
Leq = L1 + M 2
L = L1 + L2 + 2M dt dt dt
For the circuit shown in Fig. 1.87(b), the equations for  di1 di 
 L1 +M 2
di 1
=
magnetic flux and inductance are: dt Leq  dt dt 

f = f1+ f2  di (L − M ) di2 
L 2 2
1
=  1 dt (L − M ) + M dt 
f1 = L1i1 − Mi2 
Leq 1 

1  (L2 − M ) 
f 2 = L2 i2 − Mi1 + M  2
di
=  L1
Leq  (L1 − M )  dt
f = L1i1 − Mi2 + L2 i2 − Mi1
L2 − M  (L − M ) 
i1 = i2 = i 1 L 2 
1+
L1 − M
=  (L − M )
1 + M 
L = L1 + L2 - 2M Leq  1 
(L2 − M )
L1 +M
1.8.3 Parallel Connection of Coupled Coils (L1 − M )
Leq =
L −M
1+ 2
Figure 1.88 shows the circuit for two inductors con- L1 − M
nected in parallel.
L1(L2 − M ) + M (L1 − M )
M Leq =
i L1 − M + L2 − M
i1 i2 L1L2 − M 2
=
V L1 + L2 − 2M
L1 L2
In the second circuit for parallel connection of coupled
coil shown in Fig. 1.89, the equivalent inductance is
Figure 1.88 |   Parallel-connected inductor coils. given by

The equations for current, voltage and equivalent induc- L1L2 − M 2


Leq =
tance are: L1 + L2 + 2M
i = i1 + i2 i
di di1 di2 M
= +
dt dt dt
di1 di V
V = L1 +M 2 L1 L2
dt dt
di2 di
= L2 +M 1
Figure 1.89 |   Parallel connected inductor-coils.
dt dt
di di di di
L1 1 + M 2 = L2 2 + M 1 1.8.4 Analysis of Multiwinding Coupled Circuits
dt dt dt dt
di1 di
(L1 − M ) = 2 (L2 − M )
Consider the multiwinding coupled circuit shown in
dt dt Fig. 1.90.

Chapter 1 Theory 1.indd 71 3/21/2016 12:44:25 PM


72        Chapter 1:  ELECTRIC CIRCUITS 


Problem 1.109: Calculate the effective inductance of
+ V2 the circuit shown in the following figure.
i2
i 8H

+
V1 a
i1 4H
− v 10 H
5H

i3 b
6H
+ V3 −

Figure 1.90 |   Multiwinding coupled inductors.


Solution: From the given circuit
di di di di di di di
The inductance matrix for the given circuit is v=8 − 4 + 10 − 4 + 5 + 6 + 5
dt dt dt dt dt dt dt
 L11 L13 
 
L12 di
L =  L21 L22 L23  v = 26
  dt
 L31 L32 L33 
di
Since, v = Leq ⇒ Leq = 26 H
Where L12 = L21, L32 = L23, L31 = L13 is the mutual dt
Ans. (26)
inductance.
 di 
[V ] = [ L ]   Problem 1.110: Write the mesh equations for the
 dt  given network.
V1   L11 L13  di1 /dt  - j2
     5Ω
L12
V2  =  L21 L22 L23  di2 /dt
    
V3   L31 L32 L33  di3 /dt i1 i2 i3

v1 j3 j4 j5 3Ω

Problem 1.108: For the multiwinding coupled circuit


shown in the following figure, write the inductance
matrix.
Solution:
+ −
V2 5i1 + j3 (i1 - i2) + j4 (i3 - i2) = v1
i2
−j2i2 + j3 (i2 - i1) + j4 (i2 - i3) + j5(i2 - i3) +
i1 j4(i2 - i1) = 0
3i3 + j5 (i3 - i2) + j4(i1 - i2) = 0
+ L2 +

V1 V3 Problem 1.111: The inductance matrix for the circuit


L1 of three series connected coupled coils is given. Find
the inductances and indicate the dots.
− −
i3

Solution:
 L1 L12 −L13 
 
[L] =  L21 L2 −L23   4 −4 1
   
−L31 −L32 L3  L =  −4 2 −3 
 1 −3 6 


Chapter 1 Theory 1.indd 72 3/21/2016 12:44:44 PM


1.9  RESONANCE     73

I1 N1 : N2 I2
Solution: The inductances and circuit with dot + +
­convention is

1H V1 L1 L2 V2
4H
− −
Figure 1.91 |   Ideal transformer.
4H 2H 6H

3H
N2 V I
n= = 2 = 1
N1 V1 I2
Problem 1.112: Find the voltage across the 10 W Here n is the turn ratio.
resistor for the following network.

j3
1.9  RESONANCE

j4 j2 Resonance is the phenomenon that exists because of


i1 i2 the energy storing element of the opposite kinds called
inductor and capacitor. A network is in resonance when
10∠0° -j15 10 Ω v
2 the voltage and current at the network input terminals
v1 are in phase. At resonance, there exists the energy trans-
fer between the inductor and capacitor at a constant
frequency as a function of time. So, the sinusoidal oscil-
lations are produced in the circuit.
Solution:
v2 = i2 10
1.9.1 Series Resonance
j4i1 − j15(i1 − i2 ) + j3i2 = 10∠0°
Consider series RLC circuit in Fig. 1.92.
−j11i1 + j18i2 = 0∠0°
j2i2 + 10i2 − j15 (i2 − i1 ) + j3i1 = 0 I R L
j18i1 + i2 (10 − j13) = 0 +V − + V −
+ R L
+
V VC C

1.8.5 Ideal Transformer −

In coupled coils, the degree to which mutual inductance


M approaches its maximum value is described by the Z
coupling coefficient defined as Figure 1.92 |   RLC circuit in series.
M We have
k=
L1L2
Z = ZR + ZL + ZC
L1L2 ≥ M
Z( jw ) = R + jwL +
1
jwC
M ≤ L1L2
 1 
Z( jw ) = R + j wL − 
 wC 
M
k=
L1L2
As w = 0 ⇒ Z( j0) = ∞ ⇒ I = 0
0 ≤k ≤1 As w = ∞ ⇒ Z( j∞) = ∞ ⇒ I = 0

At w = w0,  w 0 L =
An ideal transformer (Fig. 1.91) is one with perfect cou- 1
pling (k = 1). It consists of two (or more) coils with a w 0C
large number of turns wound on a common core of high Hence, we obtain Fig. 1.93 which plots the change of
permeability frequency as a function of impedance Z.

Chapter 1 Theory 1.indd 73 3/21/2016 12:50:17 PM


74     Chapter 1:  ELECTRIC CIRCUITS 

1 V
∠ − 90°
V 1
wC VC = IZC = =
wL R jw 0 C R w 0C

1
= V∠ − 90° = QV∠ − 90°
R w 0 RC

Phasor diagram for RLC series circuit resonance is shown


in Fig. 1.94.
0 w0 w VL = QV

|Z|
VR 90° VR = V
I
90°

|Z| = R
VC = QV

Figure 1.94 |   Phasor diagram for RLC series resonance.

w0 w Now, f = 0° , so cos f = 1 which means power factor is


0
unity. This condition is called “maximum power transfer
Capacitive Inductive
condition”.
impedence impedance
w < 1 w > 1
  1.9.1.1 Frequency Response for Series
 w0  w0
 
Resonance
Leading pf Logging pf
The frequency response is given by
Unity power factor (upf)
V V
Figure 1.93 |   Frequency as a function of impedance.
I = =
Z  1 
2
R 2 + wL − 
 wC 
w0 =
1
So, we have
LC Figure 1.95 depicts the frequency response for series
1 resonance.
or Z = R and  f =
2p LC
Voltage phasor Im
Now, Current phasor =
Impedance phasor Im
V V √2
I= and I =
Z R
fL f0 fH f
V
VR = IR = ⋅R = V
R
Bandwidth I = Im = V
At resonance, VR = V and R
w L
VL = IZL = jw 0 L = 0 V∠90°
V Figure 1.95 |   Frequency response for series resonance.
R R
At resonance, w = w0. The resonance circuits are band-pass
Therefore, filters (discussed in next section). The mean value of fH, fL is
VL = QV∠90°
fH fL = f0
where Q is the quality factor.
f0
In series, fH − fL =
w0L
Q
1 1 L
Q= = = f
R w 0 RC R C Bandwidth = 0
Q

Chapter 1 Theory 1.indd 74 3/21/2016 12:50:51 PM


1.9  RESONANCE     75

Therefore, Q > 1 for series circuit (Q ≈ 10). The quality |Y |


factor of a resonant circuit is the ratio of its resonant
frequency to its bandwidth.
Some important points regarding series resonance are
|Y | =
as follows: 1
R
  1.  At resonance, the voltages across the inductor and
capacitor are magnified by Q times (greater than 1)
so these circuits are called voltage magnification
circuits at resonance. 0 w0 w
  2.  Since the circuit accepts maximum current at reso-
nance compared to any other operating frequency. Inductive Capacitive
So, the series RLC circuits at resonance is called nature nature
acceptor circuits. (Lagging pf) (Leading pf)
  3.  A resonant circuit is designed to operate at or near Figure 1.97 |   Frequency as a function of admittance.
its resonant frequency. For a high Q circuit (Q ≥10)
the half-power frequencies are symmetrical around
At w = w0, w 0 C =
1
the resonant frequency.
w0L
1.9.2 Parallel Resonance
So, w 0 =
1 1
and Y =
LC R
Consider the parallel RLC circuit shown in Fig. 1.96.
I
I Now, V = = IR
Y
IR IL IC V IR
+ + + + IR = = =I
V VR R VL L VC C V R R
− − − −
V V IR R
IL = = = = I∠ − 90°
ZL jw 0 L jw 0 L w 0 L

Figure 1.96 |   Parallel RLC circuit.


R
Q=
w0L
We have
We have I L = QI∠ − 90°
1 1 1 1
Y = ⇒Y = + +
= IRjw 0 C = w 0 RC I∠90° = QI∠90°
R R ZL ZC V
IC =
ZC
Y ( jw ) = + jwC
1 1
+
R jwL So, the phasor diagram for RLC parallel circuit reso-
 1  nance is shown in Fig. 1.98.
Y ( jw ) = + j wC −
1

R  wL  Q = w 0 CR
As w = 0 ⇒ Y ( j0) = ∞ ⇒ V = 0 Here f = 0°, power factor = 1. So, the maximum power
As w = ∞ ⇒ Y ( j∞) = ∞ ⇒ V = 0 is transferred at resonance.

Hence we obtain Fig. 1.97, which depicts the change in IC = QI


frequency as a function of admittance Y.
1
wL
wC V
IR

R IL = QI

Figure 1.98 |   Phasor diagram for RLC parallel circuit


w0 w
resonance.
0

Chapter 1 Theory 1.indd 75 3/21/2016 12:51:24 PM


76     Chapter 1:  ELECTRIC CIRCUITS 

1.9.2.1 Frequency Response for Parallel


Solution: IR = I = 1 mA
Resonance
I L + IC = QI∠ − 90° + QI∠90°
The frequency response is as follows:
= −jQI + jQI = 0
I
V = IR + IC = 1 mA + jQI
 2
1w C = 1 
R 2  wL 
+ = (1 + jQ) mA

As Vm = IR at w = w0 as shown in Fig. 1.99. IR + IC = 1 + Q2 mA ≥ 1 mA (∵ Q > 1)


|V |
IR + I L = 1mA - jQI

IR + I L = 1 + Q2 mA ≥ 1 mA
Vm
Vm
√2 Ans. (c)

fL f0 fH
Problem 1.114: Determine resonance frequency of
BW the circuit shown in the figure given below.

Figure 1.99 |   Frequency response for parallel


4H 1F
resonance.
f0 10 Ω
We have f0 = fL fH so bandwidth =
Q
Vin
1F
= w 0 CR
R
The quality factor is Q =
w0L
Some important points regarding parallel resonance are Solution: From the circuit, we have
as follows:
 1   1 
Z = 10 +  j4w +
  1.  At resonance, the inductor and capacitor are mag-
 ||  
nified by Q-times. So, the parallel RLC circuit at  jw   jw 
resonance is called current magnification circuit.
  2.  At resonance, the circuit rejects part of the current 4 − 1/w 2
compared to any other operating frequency. So, the = 10 +
j4w +
1 1
parallel RLC circuits at resonance are called rejec- +
jw jw
tor circuits.
 
 2 
 4 − 1/w 
Problem 1.113: The circuit shown is at resonance. = 10 − j  
 2 
 4w − 
Which one of the following relation is true?
w

IR IL IC At resonance circuit is purely resistive, therefore


imaginary part = 0.
1 mA R L C
1
4− =0
w2

(a) IR < 1 mA w=
1
= 0.5 rad/s
(b) |IR + IC| > 1 mA 2
(c) |IR + IL| > 1 mA Ans. (0.5)
(d) |IL + IC| > 1 mA

Chapter 1 Theory 1.indd 76 3/21/2016 12:51:49 PM


1.10  FILTERS     77

Problem 1.115: For the circuit shown in the following Problem 1.116: A coil with a quality factor of 10 is
figure put in series with a capacitor 10 μF and combination
a 2Ω is drawing maximum current at a frequency of 50 Hz.
A second capacitor C2 is connected in parallel with the
I circuit. What should be the value of C2 such that the
Vi(t) 4H 4F 2Ω V combined circuit to act purely as a resistance at a fre-
quency of 100 Hz, if the applied voltage is 100 V rms?

Solution: The circuit at resonance is given as follows:


b
C = 10 µF
Q = 10
(a) Determine the frequency (w0) which results in a
maximum impedance across the terminals a, b.
(b) Determine the impedance at w = w0.
(c) If Vi(t) = Vmsin(w0t) then find iL(t) and iC(t).
V1, f = 50 Hz
Solution: From the given circuit:
1 L
(a) Z = 2 + ( jXL ) || (−jXC ) || 2 Q= = 10
R C
XC XL
= 2+ || 2 1
j(XL − XC ) f0 = 50 =
2 p LC1
2XL XC / j(XL − XC ) Q
= 2+ C1
XL XC
2+
j(XL − XC )
2XL XC [XL XC − j2(XL − XC ]
= 2+
(XL XC )2 + 4(XL − XL )2
C2
At resonance imaginary part = 0
f0 = 100
−2 (XL - XC) = 0
XL = XC
1 1
Y = +
w0 =
1 1
 1 
R + j wL −
= rad/s 1

jwC2
LC 4
 C1 
(b) Z w = 2+2 = 4 W w = 2p ×100
=1/ 4 rad/s
Y = jwC2 + 0.0001 - j0.0021
0

t
(c) Vi (t) = Vm sin   = 0.0001 + j(wC2 - 0.00021)
 4 
Z=4Ω C2 =
0.00021
= 3.34 mF
t 2 p × 100
Vm sin  
V (t) 4 Y = 10−4 , Z = 10 kΩ
i(t) = i = = iR (t)
Z 4
Vrms 100
t
VR = 2i (t) = m sin  
V I rms = = = 10 mA
 4  Z 10 × 103
2
V = VL(t) = VC(t)  Ans. (3.34)

dvC (t)
iC(t) = C
dt 1.10  FILTERS
t
. cos  
Vm 1
 4 
= 4.
2 4 A filter is basically a circuit which is designed to pass
t 
iC(t) = m cos = Vm sin  + 90° A
V t signals with desired frequencies and reject or alternate
2 4 
4  others; it is also called a frequency selective device.
 t  t 
iL (t) = ∫ sin   dt = sin  − 90° A
1 Vm Vm With respect to components present in networks,
4 2 
4 2 
4  ­
filters are classified as follows:

Chapter 1 Theory 1.indd 77 3/21/2016 12:52:17 PM


78     Chapter 1:  ELECTRIC CIRCUITS 

  1. Active filters: Filters which consist of active devices 2. High-pass filter (HPF): This filter passes high
such as operational amplifiers that have tendency to frequencies and rejects low frequencies (Fig. 1.102).
amplify signals are called active filters. So, in active
filters, it is possible to increase the gain of the system. 1/Cs
In the design of active filters, inductors are not pre-
ferred since their size is bulky and cost is high.
  2. Passive filters: These filters only contain passive
components in the circuit, that is, series and parallel Vi (s) R Vo(s)
connections of inductor and capacitors.
With respect to operating frequencies, filters are classi-
fied as follows: (a)

1. Low-pass filter (LPF): A filter that passes low


|H(w)|
frequencies and stops high frequencies is called low-pass
filter (Fig. 1.100).
R
|H(w)|
1 1 wC w
Vi(s) Vo(s)
Cs
(b)
wC w
Figure 1.102 |   High-pass filter. (a) Circuit.
   0
(a) (b) (b) Frequency response.

Figure 1.100 |   Low-pass filter. (a) Circuit. R RCs


Vo (s) = V i (s) = V (s)
(b) Frequency response. R + 1/Cs 1 + RCs i
V i (s) 1/s 1
Vo (s) − =
Vo (s) RCs
V (s) =
1/Cs + R 1 + RCs i Vi (s) 1 + RCs
Vo (s) 1 This expression is for first order HPF.
=
V i (s) 1 + RCs
The circuit for second order HPF is shown in Fig. 1.103.
This is the expression for first order filter. Therefore,
there is only one energy storage element for the first 2C 2C
order filter.
In second order LPF, usually symmetrical networks
are designed (Fig. 1.101). L

L/2 L/2

(a)
C
C

(a)
2L 2L
L

2C 2C (b)
Figure 1.103 |   Second order high-pass filters.
(a) Symmetrical T. (b) Symmetrical p.
(b)   3.  Band-pass filters: These types of filters pass
Figure 1.101 |   Second order low-pass filter. frequencies within a frequency band and block
(a) Symmetrical T. (b) Symmetrical p. or attenuate frequencies outside the band.

Chapter 1 Theory 1.indd 78 3/21/2016 12:52:39 PM


1.11  THREE-PHASE CIRCUITS     79

From Fig. 1.104(b), it can be seen that only the fre- These filters are also obtained by the combination of
quencies lying between w1 and w2 are passed, while both low pass and high pass filters. The necessary condi-
the other frequencies are blocked. tion is
fc (HPF) > fc (LPF)
L1/2 2 C1 L1/2 2 C1
5. All-pass filters: In these filters, all frequencies
are passed and also for these types of filters, zeros
are present in the right half plane and poles are
C2 C2
present in left half of plane, that is, poles and zeros
are symmetric about jw axis.
The transfer functions of first order filters are listed
(a) as follows:
|H(w)|
1 Low-pass filter
1 + ts

ts
w1 w2 w High-pass filter
1 + ts
(b)
Figure 1.104 |   Band-pass filter. (a) Circuit. 1 − ts
All-pass filters
(b) Frequency response. 1 + ts
These filters are also obtained by the combination of both
low-pass and high-pass filters. The necessary condition The transfer functions of second order filters are listed
is that the cut-off frequency of low-pass filter should be as follows:
greater than cut-off frequency of high-pass filter.
fc (LPF) > fc (HPF) p
LPF (No zeros only poles)
  4.  Band elimination filters/band-stop filters: 2
s + as + b
In these types of filters, the frequencies outside
a frequency band are passed, while the frequen-
cies within the band are blocked. Figures 1.105(a) ps2 HPF (Two zeros at origin)
2
and (b) show the circuit and frequency response of s + as + b
band stop filters.
ps2 (Band elimination filter)
L1/2 L1/2
s2 + as + b

s2 − ps + q (All-pass filters)
L2 s2 + as + b
2 C1 2 C1

C2
1.11  THREE-PHASE CIRCUITS
(a)

|H(w)| Three-phase circuits are used for generation, transmis-


sion and distribution of bulk electrical power. These
circuits are generally employed in larger machines.
In the case when larger power is required, the use of
w1 w2 w three-phase systems is more economical and convenient
than three single phase systems. There are two types of
(b) three-phase circuits:
Figure 1.105 |   Band elimination filter. (a) Circuit.   1.  Balanced three-phase circuits
(b) Frequnecy response.   2.  Unbalanced three-phase circuits

Chapter 1 Theory 1.indd 79 3/21/2016 12:52:53 PM


80     Chapter 1:  ELECTRIC CIRCUITS 

1.11.1 Balanced Three-Phase Circuits The line and phase voltage-current relations (Fig. 1.108)
are as follows:
In the balanced three-phase circuits, the phasors of volt- VBN −VYN
ages VRN, VBN and VYN are exactly equal in magnitude VRY
and each has a progressive phase displacement of 120°, (Line voltage)
as shown in Fig. 1.106.
VBN
120° 60°
30°

120° N VRN(Phase voltage)


120° VRN
120°

VYN
VYN
Figure 1.106 |   Balanced three-phase supply (phasor
diagram). (a)

−IYB
The instantaneous values of these voltages can be rep- IBR
resented as IR
vRN = 2 VRN sin wt (Line current)

vYN = 2 V YN sin(wt − 120°)


120° 60°
vBN = 2 VBN sin(wt − 240°) 30°
Hence, vYN lags vRN by 120°; vBN lags vYN by 120°. The N IRY(Phase current)
phase sequence is represented by RYB.
Now, if the three-phase load is such that the magni-
tudes and angles of each of the three phases are equal,
then the three phase load is balanced. Three-phase loads
connected in star and delta are shown in Figs. 1.107(a)
IYB
and (b), respectively.
ILine (b)

VLine Iph Figure 1.108 |   Actual phase and line voltage-


current relation in a (a) star
connection and (b) delta connection.
Vph VRY = VRN + V NY
VRY = VRN − VNY

VRY = VRN
2
+ (−V YN )2 − 2 × VRN × (−V YN ) × cos 60°
(a)  
ILine = 3 × VRN | VRN | = | V YN |

| IR | =  IRY + (−I YB )2 − 2 × IRY × (−I YB ) × cos 60°


Iph
2
Vph  
VLine = 3 × IRY
The following voltage and current relationships are
established in balanced three-phase system as:
V
  1.  In Y connection, I ph = I L and Vph = L
(b) 3
IL
Figure 1.107 |   (a) Star connection. (b) Delta connection.   2.  In ∆ connection, I ph = and Vph = VL
3

Chapter 1 Theory 1.indd 80 3/21/2016 12:53:15 PM


1.12  AC POWER ANALYSIS     81

1.11.2 Unbalanced Three-Phase Circuits T T /2

∫ ∫
1 1
Xavg = x(t) dt = x(t) dt
The unbalanced three-phase circuit can result from any T T
0 −T / 2
of the following conditions:
Area over all period
  1.  Unbalanced three-phase sources acting on balanced =
loads. Period
  2.  Balanced sources acting on unbalanced loads. The average value of sine or cosine function of any phase
  3.  Unbalanced three-phase sources acting on unbal- and frequency is zero.
anced loads.
It can be resolved into three balanced phasor systems; 1.12.2 Average Power
and this method involves resolving symmetrical compo-
nents of the original phasors or simply symmetrical com- The average power in watts is the average of its
ponents. The unbalanced components Y- or -connected instantaneous power over one period. For periodic
loads, transformers, generators and transmission lines can waveform,
be resolved into symmetrical components. The three bal-
t1 + T
anced voltage and current phasors can be resolved into

1
three symmetrical components (a) positive sequence with P = p(t)dt
T
phase sequence same as original; (b) negative sequence t1
with phase sequence opposite to that of ­original and The average power in the sinusoidal steady state is
(c) three phasors equal in magnitude (Fig. 1.109).
V (t) = Vm cos(wt + q )
Vc1 Vb2
Vc i(t) = I m cos(wt + f )

V I cos(q − f )
1
Va Va1 Va2 P =
2 m m

Vb1 Vc2 1.12.2.1 Average Power Absorbed by an


Vb Ideal Resistor
(a) (b) (c)
Figure 1.109 |   Symmetrical components.
1 1
PR = V I cos 0° = Vm I m
2 m m 2
1 2
or PR = I m R
2
1.12  AC POWER ANALYSIS
Vm2
PR =
2R
Instantaneous power (in Watts) is the power at any
instant of time. 1.12.2.2 Average Power Absorbed by Purely
p(t) = v(t) i(t) Reactive Element

The average power (in Watts) is the average of the The average power delivered to any device which is
instantaneous power over all period. purely reactive (i.e., contains no resistor) must be zero.
The phase difference between voltage and current is 90°.
T

∫ p(t)dt cos(q − f ) = cos 90° = 0


1
P =
T
0
So,     Px = 0.
A resistive load (R) absorbs power at all times, while a
reactive load (L or C) absorbs zero average power. 1.12.2.3 Average Power for Non-Periodic
Functions
1.12.1 Sinusoidal Signals
The power for non-periodic functions can be obtained as
The average value of any periodic signal x(t) with period i(t) = sint + sinpt
T  is

Chapter 1 Theory 1.indd 81 3/21/2016 12:53:30 PM


82     Chapter 1:  ELECTRIC CIRCUITS 

i(t) = I m1 cosw 2 t + I m2 cosw 2 t + … P =


1 2
I R
2 m
1 2 2 Im
P = (I + I m 2 + ...)R Since I eff = , the average power is
2 m1 2
2
P = I eff R
1.12.3 Effective (RMS) Values of Current and       
Voltage 2
Veff
P = Veff I eff cos(q − f ) =
R
The effective value, also called rms value, of any function
is given by
Problem 1.117: Calculate the average power absorbed
T by an impedance Z = 30 - j 70 Ω when a voltage
∫ x (t) dt
1 2
Xrms = V = 120∠0° is applied across it.
T
0
V 120∠0°
Solution: I = =
The effective value of any periodic current is equal to Z 30 − j70
the value of the direct current, which flowing through an
R-ohm resistor delivers the same average power to the 120∠0°
= = 1.576∠66.8° A
resistor as does the periodic current. 76.16∠ − 66.8°
T T

∫ i Rdt = ∫ i2dt V I cos(q v − q i )


1 2 R 1
P = Pavg =
T T 2 m m
0 0
2 1
P = I eff R = × 120 × 1.576 cos(0 − 66.8°)
T
2

∫ i dt
1 2 = 37.24 W
I eff =
T
0 Ans. (37.24)
The rms values of sine or cosine function of any phase
and frequency are maximum value/ 2. Problem 1.118: Given the time-domain voltage
 pt 
v = 4 cos   V.
1.12.3.1 Effective (RMS) Value of a  6 
Sinusoidal Wave Find both the average power and an expression for
the instantaneous power that results when the corre-
i(t) = I m cos(wt + f ) sponding phasor voltage V = 4∠0° V is applied across
2p
T = an impedance Z = 2∠60° W.
w
The effective RMS value of current is: Solution: Given that V = 4∠0°
T
Z = 2∠60°
∫ Im cos (wt + f ) dt
1 2 2
I eff =
T V 4∠0°
0
I= = = 2∠ − 60°A
2p /w 2 2∠60°
w 1 1 
= Im
2p ∫  + cos(2wt + 2f ) dt
 2 2 
1
Pavg = 4 × 2 cos(60°) = 2 Watts
0 2
w 2p /w I m pt
= Im [t ] = v(t) = 4 cos
4p 0 2 6
 pt 
i(t) = 2 cos  − 60°
1.12.3.2 Use of RMS Values to Compute  6 
Average Power pt  pt 
cos  − 60°

p(t) = v(t)i(t) = 8 cos
The average power delivered to an R-ohm resistor by a 6  6 
 pt 
= 2 + 4 cos  − 60°
sinusoidal current is
 3 

Chapter 1 Theory 1.indd 82 3/21/2016 12:54:17 PM


1.13  TWO-PORT NETWORK     83

Problem 1.119: Determine the average power dissi-


1  t3 
2
pated by the voltage sources if 100 4 1  200 
=  25  + (t)2 =  + 200
v(t) = 2 + 3 2 cos(10t + 45° − 3 cos(10t) V to 1 Ω 4  3  4 
4 3 
0
resistor is
= 8.165 A
(a) 8.5 W (b)  17.5 W Power absorbed by a 2 Ω resistor
(c) 17 W (d)  35 W
2
P = I rms R = (8.165)2(2) = 133.3 W
V2
Solution: Pavg = rms W Ans. (133.3)
R
T
1.12.3.3 Effective Value with Multiple
∫V
2 1 2
Vrms = (t)dt
T Frequency Circuits
0

 3 2
In order to determine the effective value of a periodic or
≠ 2 + 3 +  
2 2 non-periodic waveform which is composed of sum of a
 2  number of sinusoids of different frequency,
Note: Same frequency cosine or sine are added for
the simplification first.
2
(
P = I eff + I22 eff +  R)
 1 
v(t) = 2 + 3 2  cos 10t ⋅
1
− sin 10t ⋅  − 3 cos(10t) I eff = I1 2eff + I22 eff + 
 2 2 
= 2 + 3 cos 10t - 3 sin 10t - 3 cos 10t Apparent power = Veff × Ieff
= 2 - 3 sin 10t Average power
= cos(q − f )
P
Power factor = =
Vrms = 22 + (−3 / 2 ) = 8.5
2 Apparant power Veff I eff

Pavg = 8.5 W
1.13  TWO-PORT NETWORK
Ans. (a)

A pair of terminals at which a signal may enter or leave


Problem 1.120: Determine the rms value of the
a network is called a port, and a network having only
­current waveform given in the following figure. If the
current is passed through a 2 Ω resistor, find the average
one such pair of terminals is called a one-port network.
When more than one pair of terminals is present, the
power absorbed by the resistor.
network is known as a multiport network. A two-port
i(t) network having two pair of terminals, one is labelled as
input port and other as output port (Fig. 1.110).
10 I1 I2
1 + +2

0 V1 N V2
2 4 6 8 10 t 11 − − 12
Figure 1.110 |   A two port network.
−10
The given network N is can be any of the following types:

Solution: The period of the waveform is T = 4.   1.  Symmetrical network: A network is said to be
symmetrical if it looks exactly the same whether
 5t 0 < t < 2 viewed from input or output.
i(t) = 
−10 2 < t < 4
  2.  Asymmetrical network: An asymmetrical net-
work differs when viewed from input and output.
T 2 4    3.  Reciprocal network: If the network obeys the
1 2 
∫  ∫ (5t) dt + ∫ (−10) dt
1
I rms = i2dt = 2
reciprocity theorem, then it is called reciprocal net-
T 4 
0 0 2  work. All the passive (R, L, C and T/F) networks
are always reciprocal.

Chapter 1 Theory 1.indd 83 3/21/2016 12:54:36 PM


84        Chapter 1:  ELECTRIC CIRCUITS 

  4.  Non-reciprocal network: If the network does Thus, six types of parameters exist, which are shown
not obey the reciprocity theorem, then it is called in Table 1.9.
non-reciprocal network. All the active networks are
said to be non-reciprocal, such as sources, diodes Table 1.9 |   Types of two-port network parameters
and transistors.
Z-parameters A′B′C′D′ parameters
The examples of different types of networks are illus-
(Impedance parameter) (Inverse transmission
trated in Fig. 1.111.
parameters)
2Ω 2Ω Y-parameters h-parameters (Hybrid
(Admittance parameters) parameters)
1Ω ABCD parameters g-parameters (Inverse
(Transmission parameters) hybrid parameters)

The characteristic equations and conditions for these


(a) parameters are listed as follows.
2Ω 2Ω   1.  Z-parameters or open circuit impedance
parameters:
2Ω 2Ω  V1   Z11 Z12   I1 
 =  
V2  Z21 Z22   I2 
V1 = Z11 I1 + Z12 I2
(b)
V2 = Z21 I1 + Z22 I2
2Ω 1Ω
V1 V1
Z11 =    Z12 =
I1 I2 = 0
I2 I1 = 0
1Ω
V2 V2
Z21 =    Z22 =
I1 I2 = 0
I2 I1 = 0
(c)
(a)  Symmetric condition Z11 = Z22
2Ω 1Ω (b)  Reciprocal condition Z12 = Z21
  2.  Y-parameters or short circuit admittance
2Ω 2Ω parameters:
 I1   Y11 Y12   V1 
 =  
 I2   Y21 Y22  V2 
(d)
Figure 1.111 |   Examples of two-port networks.
I1 = Y11V1 + Y12V2
(a) Symmetrical, (b) symmetrical and I2 = Y21V1 + Y22V2
reciprocal, (c) asymmetrical and (d)
(a)  Symmetric condition Y11 = Y22
asymmetrical and reciprocal networks.
(b)  Reciprocal condition Y12 = Y21
From the above observations, we have
  3.  T or Transmission (A, B, C, D) parameters
  1.  All the symmetrical networks are always reciprocal
V1   A B   V2 
but the converse need not be true.  =  
  2.  An asymmetrical network may be a reciprocal.  I1  C D  −I2 

1.13.1 Types of Network Parameters V1 = AV2 − BI2

Consider the input (V1, I1) and output (V2, I2)vari- I1 = CV2 − DI2
ables. From these four variables, we can select two
V1 V1
variables as independent variables and two as depen- A=    B = −
V2 I2
dent at a time, with which we can have 4 C2 = 6 ways. I2 = 0 V2 = 0

Chapter 1 Theory 1.indd 84 3/21/2016 1:01:00 PM


1.13 TWO-PORT NETWORK        85

I1 I1 (a)  Symmetric condition |h| = 1 ⇒ |h11h22 − h21h12|


C= D =− =1
V2 I2
  I2 = 0 V2 = 0 (b)  Reciprocal condition h12 = −h21
(a)  Symmetric condition A = D   6.  g-Parameters
(b)  Reciprocal condition |AD − BC | = 1
 I1   g11 g12   V1 
 =  
  4.  Inverse transmission (A′B′C ′D′) parameters V2   g21 g22  V2 

V2   A ′ B ′  V1 
I1 = g11V1 + g12I2
 =  
 I2  C ′ D ′ −I1  V2 = g21V1 + g22I2
(a)  Symmetrical condition |g| = 1 ⇒
V2 = A′V1 − B′I1
   |g11 g22 − g12 g21| = 1
I2 = C′V1 − D′I1 (b)  Reciprocal condition g12 = −g21
(a)  Symmetric condition A′ = D′
(b)  Reciprocal condition |A′D′ − B′C′| = 1 1.13.1.1 Relations among Network
Parameters
  5.  h or Hybrid parameters
V1   h11 h12   I1  The relation among the two-port network parameters
 =   are as follows:
 I2  h21 h22  V2 
  1.  Y = Z −1
V1 = h11I1 + h12V2   2.  T′ = T −1
I2 = h21I1 + h22V2   3.  g = h−1
Some more relations are summarised in Table 1.10.

Table 1.10 |   Inter-relation between network parameters


[Z ] [Y ] [T ] [T ¢] [h ] [g ]

Y22 − Y12 A ∆T D′ 1 ∆h h12 1 g


[Z  ] Z11 Z12 − 12
∆Y ∆Y C C C′ C′ h22 h22 g11 g11
DT ′ A′
1 D h21 1 g21 ∆g

Y Y
Z21 Z22 − 21 11 C′ C′
∆Y ∆Y C C h22 h22 g11 g11

[Y  ] Z Z D ∆T A′ 1 1 h ∆g g12
22
− 12 Y11 Y12 − − − 12
∆z ∆z B B B ′ B′ h11 h11 g22 g22
1 A ∆T ′ D ′ h21 ∆h
Z21 Z11 − − −
g21 1
− Y21 Y22 B′ B′
∆z ∆z B B h11 h11 g22 g22

[T  ]
Z11 ∆z Y22 1 D′ B′ ∆h h11 1 g22
− − AB − −
Z21 Z21 Y21 Y21 ∆T ′ ∆T ′ h21 h21 g21 g21
1 Z22 ∆Y Y C ′ A′ g11 ∆g
− − 11 CD −
h22

1
Z21 Z21 Y21 Y21 ∆T ′ ∆T ′ h21 h21 g21 g21

[T ′] Z ∆ ∆g
A′ B ′
Y11 1 D B 1 h11
− − g22
− −
22 z
Z12 Z12 Y12 Y12 ∆T ∆T h12 h12 g12 g12
∆ C A h22 ∆h
1 Z11 − Y − Y22 C ′ D′ g11 1
Z12 Z12 Y12 Y ∆T ∆T h12 h12 − −
12 g12 g12
(Continued)

Chapter 1 Theory 1.indd 85 3/21/2016 1:02:45 PM


86        Chapter 1:  ELECTRIC CIRCUITS 

Table 1.10 |   (Continued )

[Z ] [Y] [T ] [T ′] [h] [g]


[h]
∆z Z12 1 Y B ∆T B′ 1 g22 g
− 12 h11 h12 − 12
Z22 Z22 Y11 Y11 D D A′ A′ ∆g ∆g

Y21 ∆Y 1 C ∆T ′ C ′

Z21 1 g21 g11
− − h21 h22 −
Z22 Z22 Y11 Y11 D D A′ A′ ∆g ∆g
[g]
1 Z ∆Y Y12 C ∆T C ′ 1 −1 h22 h 12
− 12 − − g11 g12
Z11 Z11 Y22 Y22 A A D ′A D¢ ∆h ∆ h

Z21 ∆z Y21 1 ∆T ′ B ′
− 1 B
− −
h 21 h11
g21 g22
Z11 Z11 Y22 Y22 A A D′ D′ ∆h ∆h

Problem 1.121: Find Y- parameter of network shown


V1
in the figure given below. A=
V2 I2 = 0
I1 V1 V2 I2
+ YC +
V1 = (ZA + ZB)I1
YA YB V2 V2 = ZBI1
ZA + ZB
− −
V1
=
V2 I2 = 0
ZB
Solution: From the given network
−I1 + V1YA + (V1 - V2)YC = 0 V2 = ZBI1
−I2 + (V2 - V1)YC + V2YB = 0 I1 1
C= = 
I1 = (YA + YC) V1 - YCV2 V2 I2 = 0
ZB
I2 = −YCV1 + (YB + YC) V2 V1 = I1ZA = −I2ZA
 I1   Y A + YC −YC   V 
 =   1
V1
B=− = ZA W
I   − Y  V 
 2  C Y B + YC   2  I2 V2 = 0
If YA = YB, the network is symmetrical. −I1
If Y12 = Y21, the network is reciprocal. D= =1
I2 V2 = 0

ZA + ZB
Problem 1.122: Find the transmission parameters for ZA
ZB
the given network. T =
I1 I2 1
1
+ ZA + ZB
For transmission parameters:
V1 ZB V2
If AD - BC = 1 network is passive and reciprocal.
− − If A ≠ D, the network is asymmetrical.

Solution: For the given network, the transmission (T)


parameters are given by Problem 1.123: The Z-parameter of the network is
V1   A B   V   s 10s 
 = Z= 
 2  10s 100s
 I  C D  −I2 
 1 Then determine the T-parameters of the network.
V1 = AV2 - BI2
Solution:
I1 = CV2 - DI2
V1 = sI1 + 10sI2
V2 = 10sI1 + 100sI2

Chapter 1 Theory 1.indd 86 3/21/2016 1:03:51 PM


1.13 TWO-PORT NETWORK        87

10sI1 = V2 - 100sI2 The Y-parameters are:


1  2 −1
I1 =
1
V − 10I2 Y =  
10s 2 4 − 1 −1 2 
 2 1
 1   − 
V1 = s  V − 10I2  + 10sI2  3 
 10s 2  Y = 3

− 1 2 
1  3 3 
V1 = V − 0I2  
10 2 The T-parameters are determined as
 1 
 0  V1 = 2I1 + I2

T =  10 
 V2 = I1 + 2I2
 1 +10
 10s  V1 = 2(V2 - 2I2) + I2
 
V1 = 2V2 - 3I2
For AD - BC = 1, the network is reciprocal
I1 = V2 - 2I2
For A ≠ D the network is asymmetrical
 2 +3
T = 
1 2 
Problem 1.124: Find Z-parameter, h-parameter and
T-parameters of the given network. For A = D, the network is symmetrical.
I1 1Ω I2 For |AD - BC| = 1, it is a reciprocal network
+ + Inverse T-parameters are:
 2 −3 
3Ω T 1 = T −1 =  
V1 V2 −1 2 
3Ω The h-parameters are:
       V1 = 2I1 + I2
− −
1Ω 1 1
I2 = V2 − I1
2 2
Solution: For the given lattice network, the ­equivalent
1 1
circuit is V1 = 2I1 + V2 − I1
2 2
I1
1 3
V1 = V2 + I1
+
2 2
1 3
1 1
V1 V2
− I2 = V2 − I1
3 I2 + 1 2 2
 3 1 
−  
h= 2 2 

 −1 + 1 
The Z -parameters can be determined as:
 2 2 
V
Z11 = 1    V2 = I2 (4||4) 
I1 I = 0
2
For, h12 = −h21 reciprocal network
V1 = I1 (4||4)   V2 = 2I2 I1 = 2 For, |h| = 1 symmetrical network

V2
 V1 = 2I1   = 2 W = Z22
I2 I1 = 0
1.13.2 Interconnections of Two-Port Network
Z11 = 2 Ω   Z22 = 2 Ω
The are three possible interconnections in two-port
3I I 2I
V2 = 1 − 1 , ⇒ V2 = 1 = I1 network:
  2 2 2
  1.  Series connection: [Z ] = [ZA ] + [ZB ]
V2
= Z21 = 1 W = Z12   2.  Parallel connection: [Y ] = [Y A ] + [YB ]
I
  1   3.  Cascade connection: [ T ] = [ TA ].[ TB ]
 2 1
Z= 
  1 2 These are discussed as follows.

Chapter 1 Theory 1.indd 87 3/21/2016 1:04:21 PM


88        Chapter 1:  ELECTRIC CIRCUITS 

Parallel Connection of Two-Port Network In matrix form, this can be represented as


The short circuit admittance parameters are used in Z11 Z12  Z11a + Z11b Z12a + Z12b 
characterising for parallel two-port network (Fig. 1.112).  = 
Z21 Z22  Z21a + Z22b Z22a + Z22b 
I1a
+ +
Cascade Connection of Two-Port Network
V1a Na V2a
I1 I2
+ − − The transmission parameters are useful in describing two-
+ port network which is connected in cascade (Fig. 1.114).
V1 V2
− I1b I2b −
I1 + + I2 I1a Na I2a I1b Nb I2b
I1 I2
V1b Nb V2b +
+ Aa Ba Ab Bb +
− − V1 V1a V2a V1b V2b V2
− Ca Da Cb Db − −
Figure 1.112 |   Parallel connection.
For network Na,
I1a = y11aV1a + y12aV2a Figure 1.114 |   Cascade connection.

I2a = y21aV1a + y22aV2a V1 a   Aa Ba   V2 a 


 =  
For network Nb,  I1 a  Ca Da  −I2 a 

V1 b   Ab Bb   V 2 b 
I1b = y11bV11a + y12bV2b
 =  
I2b = y21bV1b + y22bV2b  I1 b  Cb Db  −I2 b 
V1 = V1a = V1b and V2 = V2a = V2b
V1   A B   V2 
I1 = I1a + I1b and I2 = I2a + I2b  =  
 I1  C D  −I2 
I1 = (y11a + y11b) V1 + (y12a + y12b) V2
I2 = (y21a + y21b) V1 + (y22a + y22b) V2 We have V1 = V1a, I1 = I1a, V2a = V1b, I2a = −I1b,
I2b = I2, I2b = V2
Series Connection of Two-Port Network
 A B   Aa Ba   Ab Bb 
When two, networks A and B are connected in series  =  
having Z-parameters [ZA] and [ZR], then the Z-parameters C D  Ca Da  Cb Db 
of overall network is the sum of the Z-parameters for the
individual networks (Fig. 1.113). This result may be generalised for any number of two-
port connected in cascade.
I1 = I1a I2 = I2a
+ +
+ +
V1a V

Na
− 2a Problem 1.125: The impedance parameters of the
two-port network are
V1 I1 V2
 2 3 1 1
  and  
+ + 1 2 1 1
V1b V
− − 2b
Nb
− − If these two networks are connected in series then
I1 = I1b
admittance parameter of the overall two-port network
Figure 1.113 |   Series connection. will be

Thus, we have Solution: Z = [ZA] + [ZA]


     I1a = Ib = I1
 3 4
Z= 
     I2a = I2b = I2 2 3
     V1 = V1a + V1b
1  3 −4 
     V1 = Z11a I1a + Z12aI2a + Z11b I1b + Z12bI2b Y = Z −1 =  
(9 − 8) −2 3 
       V1 = (Z11a + Z11b) I1 + (Z12a + Z12b) I2
     V2 = Z21aI1a + Z22aI2a + Z21bI1b + Z22bI2b  3 −4 
= 
     = (Z21a + Z21b) I1 + (Z22a + Z22b) I2 −2 3 

Chapter 1 Theory 1.indd 88 3/21/2016 1:04:42 PM


IMPORTANT FORMULAS        89

Problem 1.126: Find the Z-parameter for the given


−1 1  2 −1 1  4 −2 
Y A = ZA =   YB =  
parallel two-port network.
3− 1 2 
    12 −2 4 
2Ω 2Ω

 2 −1   1 1
   − 
1Ω 1Ω =  3 3     =  3

6
 −1 1  
 −1 2   
 3 3   6 3 
 
1 2

 −1 
Solution: The given circuit can be represented as fol-  1 
lowing twin T- networks Y = Y A + YB =  2 

1Ω 1Ω 2Ω  −1 1 
2  2
 
 1 4 2
1Ω 2 1   
−1 4 2 = 3 3 W
Z=Y =   2 4
3 1 1  
 2 1  4 2 2  3 3
   
ZA =      ZB =  
1 2 2 4

IMPORTANT FORMULAS

I. Circuit Concepts and Laws 8. Inductor


(a) Total flux

1. Current density
 i y = Nf

J = A/m2
A where N is the number of turns of coil and f is flux
2. Power dissipation per turn
W y = Li Webers
J ⋅ E = (sE ) ⋅ E = sE 2  3 
 m  (b) Voltage across the inductor

dy
3. Ohms’s law
  = L⋅
di
(a) J = sE (in network theory)
VL = Volts
dt dt
  where s = conductivity of material. (c) Current across the inductor is
dq
(b) V = Ri  Volts ⇒ V = R ⋅
t


(in circuit theory) 1
dt iL = V ⋅ dt Amperes
(c) i = G ⋅ V Ampere
L
−∞
where G is the conductance (expressed in Siemens (d) Power
d 1 
=  Li2  Watts
1
or  mho) = P = Li ⋅
di
R 

dt dt 2 
4. Resistance
(e) Energy in the inductor
W
l
R=
s ⋅A 1
WL = L ⋅ i2 Joules
2
5. Resistivity
9. Capacitor
r = W-m
1
s (a) Current through capacitor
6. Power dV
iC = C .
V2 dt
P =Watts
R (b) Voltage across the capacitor

7. Energy associated with resistor t


1
VC = i dt Volts

ER = ∫ i2R ⋅ dt Joules

C
−∞

Chapter 1 Theory 1.indd 89 3/21/2016 1:05:17 PM


90        Chapter 1:  ELECTRIC CIRCUITS 

(c) Power III. Topology


dV
P = CVWatts 18. For an n-node graph, the number of tree branches,
dt is n − 1.
(d) Energy of the capacitor
Number of possible trees = det[Ar][Ar]T
1
W = CV 2 Joules where A is the reduced incidence matrix and [A]T
2 is its transpose.
10. Voltage and current relation in an inductor
19. Order of incidence matrix is n × b
di
V = L⋅ 20. Number of branches of a co-tree is (b − n + 1)
dt
11. Voltage and current relationship in a capacitor 21. Number of independent KCL equation = Number
dV of twigs.
i=C⋅
dt 22. The rank of complete incidence matrix is (n − 1).
12. In practical voltage sources 23. If a network contains b branches and n nodes then
V = VS − IR S the number of mesh currents is b − (n − 1).
13. In practical current sources 24. Number of independent KVL equation = Number
of links.
V
I = IS − 25. Inter-relationships between different matrices are
RS
as follows:
Bt = [1At-1Al]T
II. Circuit Analysis Techniques

Ql = At-1Al
14. Parallel impedance and current division
I1R2 I2R1
Z = R ⇒ I1 = , I2 = BQT = QBT = 0
R1 + R2 R1 + R2
IV. Network Theorems
1 I ⋅ C1 I ⋅ C2
Z= ⇒ I1 = , I2 =
jwC C1 + C2 C1 + C2 26. Superposition theorem
R2 R1R2
I ⋅ L2 I ⋅ L1 t = tV +tI =
Z = jwL ⇒I1=
V + I
, I2 = R1 + R2 R 1 + R2
L1 + L2 L1 + L2
15. Series impedance and voltage division VTh
27. Thevenin’s theorem I L =
R Th + R L
VR1 VR2
R = Z : V1 = , V2 = 28. Maximum power theorem
R1 + R2 R1 + R2
(a) Under variable load conditions, RS and RL
VL1 VL2
Z = jwL ⇒ V1 = ,V = VS2
L1 + L2 2 L1 + L2 Pmax = Watts
4R L
1 VC2 VC1
Z= ⇒ V1 = , V2 = (b) For XL variable
jwC C1 + C2 C1 + C2 VSRL
Pmax=
16. Mesh analysis: Steps to follow (R S S+ R
V R L )2 + X 2
L S
SV+2RL) +
2
(c) Both RL and XL(Rare varied XS2
simultaneously
  (i) Identify the meshes. S
V S2 4R
(ii) Assign the mesh current. Pmax = S
4R S
(iii) Write the mesh equation using KVL + Ohm's 29. Reciprocity theorem
law.
V1 V
= 2 or
17. Nodal analysis: Steps to follow I1 I2

  (i) Identify the nodes. 30. Tellengens theorem


b
(ii) Assign the node voltage and ground node. ∑ vk ik = 0
k =1
(iii) Write nodal equations using KCL + Ohm's
law. where b is the number of branches.

Chapter 1 Theory 1.indd 90 3/21/2016 1:05:55 PM


IMPORTANT FORMULAS        91

31. Millman’s theorem 42. Laplace transform for the periodic function
V1G1 + V2G2 + V3G3 +  + Vn Gn 1
V′= F (s) = F1(s)
G1 + G2 +  + Gn 1 − e−Ts
VI. Transients
R′ =
1
G1 + G2 +  + Gn 43. Initial and steady state
1 (a)  The inductor current at t = 0− and t = 0+
where Gn =
Rn iL(0+ ) = iL (0− )
32. Delta to Wye Conversion (b)  For all excitations except impulse,
Z1Z3 Z1Z2 E L(0+ ) = E L(0− )
ZA = ZB =
Z1 + Z2 + Z3 Z1 + Z2 + Z3 (c)  If VL(t) = d(t), then
iL(t) = iL(0− ) +
Z1Z3 1
ZC = L
Z1 + Z2 + Z3
(d)  The capacitor voltage at t = 0− and t = 0+
33. Wye to Delta Conversion VC(0+) = VC(0−)
EL(0+) = EC(0−)
Z Z Z Z
Z1 = ZA + ZB + A B Z2 = ZB + ZC + B C
ZC ZA
(e)  For all excitations except impulse,
Z Z
Z3 = ZC + ZA + C A VC (0+ ) = VC (0− ) +
1
ZB C
44. DC transients
V. Laplace transform
(a) Source free RL circuit
34. Laplace transform of a signal (bilateral Laplace
transform) i(t) = I 0 e−(R / L)t for t ≥ 0

∫ f (t)e−st dt
i(t) = I 0 e−t /t where t =
L[f (t)] = L
s
−∞ R
35. Laplace transform of causal signal (unilateral (b)  Source free RC circuit
Laplace transform):
∞ V (t) = V0 e−t /t (for t ≥ 0)
−st
L[f (t)] = ∫ f(t)e dt
where, t = RC seconds
0
VC (t) = VC (∞) + [VC (0) − VL (∞)]e−t /t
36. Expressions in Table 1.2.
(for t ≥ 0)
37. Differentiating property
diL(t)
−d VL (t) = L , (for t ≥ 0)
L[tf (t)] = F (s) dt
ds
38. Integrating property dVC (t)
iL(t) = C

 f (t)  dt
L
 t 
= ∫ F (s) ds (c)  Source free parallel RLC circuit
s
1 1
39. Shifting theorem Cs2 + s+ = 0
R L
f (t − a) u (t − a) ↔ e−as F (s)
(d)  Source free series RLC circuit
40. (a) Initial value theorem 1
Ls2 + Rs + = 0
lim f (t) = lim [sF (s)] C
t→ 0 s→∞
(e)  Source driven RL circuit
(b) Final value theorem
i(t) = 0 (1 − e−tR / L )
V
lim f (t) = lim[sF (s)]
t →∞ s→ 0 R
41. Any function convolving with the impulse signal i(t) = [i(0+ ) − i(∞) e−tR / L + i(∞)]
results in the same function.
(General formula for calculating current through
f (t) × s(t − t0) ® f (t − t0) inductor.)

Chapter 1 Theory 1.indd 91 3/21/2016 1:06:28 PM


92        Chapter 1:  ELECTRIC CIRCUITS 

(iii)  Conditions for transient-free response


(f)  Source driven RC circuits
 wL  p
f − tan−1   =
i(t) = 0 e−t /RC
V
 R  2
R
 wL  p
VC (t) = [VC (0+ ) − VC (∞)]e−t /RC + VC (∞) f = tan−1   +
 R  2
(at t = 0)
(General formula for calculating voltage
 wL  p
through capacitor.) wt0 + f = tan−1   +
 R  2
(at t = t0)
(g)  Source driven RLC circuit in parallel (c)  RC circuits with AC excitations
di(t) (i) Capacitor voltage
V (t) = L (inductor voltage)
dt VC(t) = VC(tr)(t) + VC(ss)(t)
d 2 i(t) 1 di(t) i(t) I
= 0
sin(wt + f − tan−1(wCR))
2
+ + Vm
dt RC dt LC LC VC (t) = VC (tr)(t) +
(h)  Source driven RLC circuit in series 1 + (wCR) 2

dV (t) (ii)  For voltage excitation V (t) = Vm cos(wt + f )


i(t) = C (capacitor current)
dt −
t
. cos(wt + f − tan−1(wCR))
RC
Vm
2
d V (t) R dV (t) 1 V VC (t) = ke +
+ + V (t) = 0 1 + (wCR) 2
dt2 L dt LC LC
(iii) Conditions for transient-free response
p
f = tan−1(wCR) +
45. AC Transients
(a)  Phase notations 2
−1 p
  (i) Voltage excitation f = tan (wCR) + ( at t = 0)
2
V (t) = I m cos(wt + q ) −1 p
f + w 0 t0 = tan (wCR) + (at t = t0)
  (ii) Current phasor 2
I = I m e jf = I m ∠f = I m (cos q + j sin q ) VII. Sinusoidal Steady-State Analysis using Phasors
  (iii) Impedance phasor 46. Expressions in Table 1.5 and 1.6
V 47. Phasor relationship for circuit elements
Z=
I (a)  When current and voltage are in the same phase
 (iv) Admittance phasor
V = RI
I (b)  When current lags the voltage by 90°
Y =
V = jwLI
V
(b) RL circuit with AC excitation: (c)  When the current leads the voltage by 90°
  (i) Current response
I = jwCV and V =
I

R
t jwC
i(t) = itr(t) + iss(t) = ke L + iss (t) 48. Phasor diagrams

  wL 
R
− (a)  Series RL circuit
sin w t + f − tan−1  
t Vm
L
  R 
i(t) = ke +
R + (wL)
2 2    Voltage V = VR2 + VL2
V 
(by Laplace transform)    Impedance angle f = tan−1  L 
 VR 
  wL 
. sin wt + f − tan−1  
Vm    Power factor = cosf (lags)
  R 
iss (t) =
R 2 (wL)2 
(b)  Series RC circuit
(by phasor method)    Voltage V = VC2 + VR2
(ii)  For voltage excitation V (t) = Vm cos(w t + f ) V 
   Impedance angle f = tan−1  C 
 V2 
  L 
R

cos wt + f − tan−1   Power factor = cos f (leading)
t Vm
i(t) = ke + 
 R 
L  

R (wL)
2 2
(c)  Series RLC circuit: Expressions in Table 1.7.

Chapter 1 Theory 1.indd 92 3/21/2016 1:07:12 PM


IMPORTANT FORMULAS        93

(d)  Parallel RL Circuit (c)  Frequency response


V V
I = IR2 + I L2 I = =
Z  1 
2
I  R 2 + wL − 
f = tan−1  L   wC 
 IR  f0
fH − fL =
  Power factor = cos f (lagging) Q
(e)  Parallel RC circuit 53. Parallel resonance (Parallel RLC circuit)
(a)  Quality factor
I = IR2 + IC2
= w 0 CR
R
Q=
I  w0L
f = tan−1  C 
 IR  (b)  Capacitor current

cos f = Power factor (leading) = IRjw 0 C = w 0 RC I∠90° = QI∠90°


V
IC =
ZC
(f)  Parallel RLC circuit: Expressions in Table 1.8. (c)  Frequency response
VIII. Magnetically Coupled Circuits I
V =
 2
w C = 1 
49. Mutual inductance 1
R 2  wL 
di1(t) +
V2 (t) = M 21
dt f
Bandwidth = 0
di2 (t) Q
V1(t) = M12
dt IX. Filters
   M12 = M21 = M 54. Low-pass filter
50. For two inductors connected in series Vo (s) 1
= (first-order)
(a) For current entering dotted terminals in both V i (s) 1 + RCs
the inductors 55. High-pass filter
f = L1i1 + Mi2 + L2 i2 + Mi1 Vo (s) RCs
= (first-order)
i1 = i2 = i V i (s) 1 + RCs
L = L1 + L2 + 2 M 56. Band-pass filters: Necessary condition to obtain
(b) For current entering on dotted and second from combination of LPF and HPF
fc (LPF) > fc (HPF)
undotted terminal
f = L1i1 − Mi2 + L2 i2 − Mi1
57. Band elimination (band stop) filters: Necessary con-
i1 = i2 = i dition to obtain from combination of HPF and LPF
L = L1 + L2 - 2 M
fc (HPF) > fc (LPF)
51. For two inductors connected in parallel
XI. Three Phase Circuits
L1L2 − M 2
(a)  Leq = 58. Balanced three phase circuits
L1 + L2 − 2 M
VL
In Y connection, I ph = I L and Vph =
L1L2 − M 2
3
(b)  Leq =
IL
In ∆ connection, I ph =
L1 + L2 + 2 M
and Vph = VL
IX. Resonance 3

52. Series resonance (Series RLC circuit) XII. AC Power Analysis

(a)  Quality factor 59. Average power


w L 1 1 L (a) Average of the instantaneous power over all
Q= 0 = =
R w 0 RC R C period
T
P = ∫ p(t)dt
(b)  Capacitor voltage 1
VC = QV∠ − 90° T
0

Chapter 1 Theory 1.indd 93 3/21/2016 1:07:48 PM


94        Chapter 1:  ELECTRIC CIRCUITS 

(b)  Average power for periodic waveform V2 = Z21I1 + Z22I2


t1 + T Symmetric condition Z11 = Z22

1
P = p(t)dt Reciprocity condition Z12 = Z21
T
t1
62. Y-parameters or short circuit admittance parameters
(c)  Average power in the sinusoidal steady state
I1 = Y11V1 + Y12V2
P = V m I m cos(w − f )
1
2 I2 = Y21V1 + Y22V2
(d)  Average power absorbed by an ideal resistor Symmetric condition Y11 = Y22
Vm2 Reciprocal condition Y12 = Y21
PR =
2R 63. T or Transmission (A, B, C, D) parameters
V1 = AV2 − BI2
(e) Average power absorbed by purely reactive
element
Px = 0 I1 = CV2 − DI2
(f)  Average power for non-periodic functions Symmetric condition A = D
1 2 2 Reciprocal condition |AD − BC| = 1
P = (I m 1 + I m 2 + ...)R
2 64. Inverse transmission (A′B ′C ′D ′) parameters
60. Effective or rms values
V2 = A′V1 − B ′I1
  (a)  For any function
I2 = C ′V1 − D ′I1
T
Symmetric condition A′ = D ′
∫ x (t) dt
1 2
Xrms =
Reciprocal condition |A′D ′ − B ′C ′| = 1
T
0
(b)  Power
2 65. h or Hybrid parameters
P = I eff R
V1 = h11I1 + h12V2
T

∫ i dt
1 2 I2 = h21I1 + h22V2
         where I eff =
T
0 Symmetric condition |h| = 1 ⇒ |h11h22 − h21h12|
=1
(c) Effective (rms) value of current in sinusoidal
wave Reciprocal condition h12 = −h21
T
66. g-Parameters
∫ Im cos
Im
(wt + f ) dt =
1 2 2
I eff =
T 2 I1 = g11V1 + g12I2
0
(d) Average power from effective values of current V2 = g21V1 + g22I2
and voltage Symmetrical condition |g| = 1 ⇒ |g11g22 − g12g21|
V2
P = Veff I eff cos(q − f ) = eff
=1
R Reciprocal condition g12 = −g21
(v) Effective values with multiple frequency circuits
67. Relation among the two-port network parameters:
2
P = I eff(+ I22 eff +  R ) Y = Z  −1
I eff = I1 2eff + I22 eff +  T ′ = T  −1
g = h−1
Average power
Power factor =
Apparant power 68. Expressions in Table 1.10.
69. Interconnections of two-port network
= cos(q − f )
P
=
Veff I eff (a) Series connection: [Z ] = [ZA ] + [ZB ]

XIII. Two-Port Network (b) Parallel connection: [Y ] = [Y A ] + [YB ]


61. Z-parameters or open circuit impedance parameters
(c) Cascade connection: [ T ] = [ TA ] . [ TB ]
V1 = Z11I1 + Z12I2

Chapter 1 Theory 1.indd 94 3/21/2016 1:08:17 PM


SOLVED EXAMPLES     95

SOLVED EXAMPLES

1. Determine V1 and V2 in the circuit shown in the (a) 2 Ω (b) 5 Ω


following figure. (c) 8 Ω (d) 9.5 Ω
V1 10 Ω V2
−+ Solution:  Current through RL is I3 which needs
to be zero so I3 = 0. Applying current loop method
0.6 V1
20 Ω
and KVL equation for loop current I1, we get
20 Ω
10 Ω 10 A −10I1 − 20 − 8I1 = 0
6A
+ + −18I1 = 20 ⇒ I1 = −1.11 A (i)
30 V 50 V Changing the direction of I1 current and making it
− − positive, we get I1 = 1.11 A.
Applying KVL equation for loop current I2, we get
20 − 6I2 − RI2 = 0
Solution:  Converting 10 A source to voltage source
(ii)
and applying nodal analysis at nodes 1 and 2., we
have To make I3 = 0, we have the following equation
8I1 = RI2 (iii)
KCL equation at node 1,
Putting value of RI2 from Eq. (iii) into Eq. (ii) we
V1 − 30 V1 + 0.6 V∠ − V2
−6 + + =0 have
20 10 20 − 6I2 − 8I1 = 0
KCL equation at node 2, Put positive value of I1 in the above equation, we get
V2 − 0.6 V∠ − V1 V2 − 100 V2 − 50 20 − 6I2 − 8(1.11) = 0
10
+
10
+
20
=0 20 − 6I2 − 8.88 = 0
−6I2 = −11.12
−120 + V − 30 + 2 V + 12 V − 2 V = 0 (i)
1 1 1 2 Putting the values of I1 and I2 in Eq. (iii), we get
−2 V2 + 2 V2 −12 V1 + 2 V2 −200 + V2 − 50 = 0 (ii) 8 × 1.11 = R × 1.85
4.2 V1 − 2V2 = 150 (iii) R = 4.8  5 W
−3.2 V1 + 5 V2 = 250 (iv) Ans. (b)
From Eqs. (i) to (iv), we get 3. In the circuit shown in the following figure, if
 4. 2 − 2  VS = 10 + 20 u(t), determine
∆=  ⇒ det ∆ = 21 − 6.4 = 14.6
−3.2 5
diL (0+ ) dVC (0+ )
150 −2 ,
V 1 ∆1 =   ⇒ det D1 = 750 + 500 = 1250 dt dt
250 5
1 mH
det ∆1 1250 iL
So, V1 = = = 85.616 V
det ∆ 14.6
+
Hence, from (iv) V2 = 104.79V VS + 1 µF 50 Ω

VC
Ans. (85.616, 104.79) −

2. In the following circuit, find the value of R so that


current through RL is zero.
Solution:
(i)  At t = 0−, VS = 10 V
10 Ω 6Ω
+
I1 20 V I2 iL(0−)

8Ω
+
R +
10 V − VC(0−) 50 Ω
I3 = 0 −

RL

Ch 1_Solved Examples & Practise exercises.indd 95 3/19/2016 3:49:52 PM


96     CHAPTER 1:  ELECTRIC CIRCUITS

iL (0− ) = + R1 −
10 V2 V
= 0.2 A = iL (0+ ) +
50
i1
VC (0− ) = 10 V = VC (0+ )
+
a i1
+ R1
V1 − V2

(ii)  At t ≥ 0

0.2 A −

iL Solution:  We have
V2
1 mH iC(t) −i1 + − ai1 = 0
R1
+
+
30 V − − 10 V 50 Ω V2 − V1 V2
+ − ai1 = 0
1 µF R1 R1

V2 V
(iii)  At t = 0+, −i1 + = ai1 ⇒ 2 = (a + 1)i1
R1 R1
0.2 A
V1 − V2 V − V2
⇒ 2 = 1
V
i1 =
N R1 a +1 1
+ V (0+) − iC(0+)  1 
V2  + 1 = V1
L
+  a + 1 
50 Ω
+ +
30 V − 10 V −

V2 a +1 V 1+a
⇒ = ⇒ 2 =
V1 a +2 V1 2+a

Applying KVL at node N Alternately:


−30 + VL (0+) + 10 = 0
= ai1 = 0 ⇒ 2 = (1 + a )i1
V2 V
−i1 +
VL (0+) = 20 V R1 R1

diL (0+ )  V − V 
= (1 + a )  1
V2 2
V (2 + a ) = (1 + a )V1
 R1  2
L = 20
dt R1
diL(0+ ) 20
= = 2 × 104 A/s V2 1+a
dt L ⇒ =
V1 2+a
Applying KCL at node N
10 5. Draw the graph corresponding to the given incidence
−0.2 + iC (0+ ) + =0
50 matrix.

iC(0+ ) = 0 A
Branches
Nodes
dVC (0+ ) dVC (0+ ) a b c d e f g h
C =0 ⇒ = 0 V/s 1 −1 0 0 0 1 0 1 0
dt dt
2 0 −1 0 0 0 0 −1 1
Ans. (2 × 104, 0)
3 0 0 −1 −1 0 −1 0 −1
4. In the circuit given in the following figure determine
4 0 0 0 0 −1 1 0 0
V
the ratio of 2 .
V1 5 1 1 1 1 0 0 0 0

Ch 1_Solved Examples & Practise exercises.indd 96 3/19/2016 3:50:24 PM


SOLVED EXAMPLES     97

Solution: Independent j3 = i1 + i3
j5 = i1 − i2 − i3
4 j6 = −i1 + i2

e f
7. Draw the incidence matrix of the following graph.
g 2 h
1 3 2
b c 6
a 1
d 2
5
1 3
5
6. For the given network, draw the graph showing the 4 3
link currents. Write the set schedule.

1Ω
4
1
5 6 5 6
Solution:  The incidence matrix is

4 4 Branches
3 2 3 2 Nodes
1 2 3 4 5 6
1 +1 0 0 −1 +1 0
2 −1 +1 0 0 0 +1
3 0 −1 +1 0 −1 0
4 0 0 −1 +1 0 −1
Solution:  We have n = 4, b = 6. Now,
twigs = 4 − 1 = 3 and l = 6 − 3 = 3.
8. In the following graph, what is the incidence
matrix A?
j1

j5 i1 B 2
j6
1 2
1
i3 A 4 C 3
j3 i2
j4 j2
5 3

D 4
Branch number
Links
1 2 3 4 5 6
Solution:  The incidence matrix is
1 +1 0 0 0 +1 −1
2 0 +1 +1 0 −1 +1 Nodes
Branches

3 0 0 +1 +1 −1 0 1 2 3 4 5
1 1 0 0 0 −1
Now, j1 = i1 j4 = i3 2 −1 1 0 +1 0
j2 = i2 j5 = i1 − i2 3 0 −1 −1 0 0
j 3 = i 2 + i 3 j6 = −i1 + i2
4 0 0 +1 −1 +1
Identities j1 = i1, j2 = i2, j

Ch 1_Solved Examples & Practise exercises.indd 97 3/19/2016 3:50:32 PM


98     CHAPTER 1:  ELECTRIC CIRCUITS

9. In the circuit given below, the incidence matrix is Solution:  The incidence matrix for the circuit is

Branches
I6 Nodes
C I4 1 2 3 4 5 6 7 8 9 10
I5 R 1 1 −1 −1 −1 0 0 0 0 0 0
R
R 2 0 0 +1 0 +1 −1 −1 0 0 0
R I7
I3 3 0 0 0 0 0 +1 0 +1 +1 1
I1 L 4 0 0 0 +1 −1 0 0 0 −1 +1
I2
R 5 −1 1 0 0 0 0 +1 −1 0 0

E Reduced matrix may be found by deleting any one


row, (third row here).
Solution:  The equivalent circuit is
Branches
Nodes
2 1 2 3 4 5 6 7 8 9 10
I6
1 1 −1 −1 −1 0 0 0 0 0 0
I4
I5 2 0 0 +1 0 +1 −1 −1 0 0 0
3
1 I7 4 0 0 0 +1 −1 0 0 0 −1 +1

I3 5 −1 1 0 0 0 0 +1 −1 0 0
I1 I2
4 11. The graph of the incidence matrix is given by
Branches
The incidence matrix is Nodes
1 2 3 4 5 6 7 8
1 1 0 +1 -1 +1 1 0 0
Branches
Nodes 2 0 −1 0 0 0 −1 +1 0
1 2 3 4 5 6 7
1 −1 1 0 0 1 −1 0 3 −1 0 −1 0 0 0 −1 +1

2 0 0 0 +1 −1 1 −1 4 0 +1 0 +1 −1 0 0 −1

3 +1 0 −1 −1 0 0 0 Solution:
4 0 −1 +1 0 0 0 1
1 4

1 4
10. The reduced incidence matrix of the given network 6 2 7 3 8
below is
3 2
R 4 5
12. In the circuit given below, draw the incidence matrix.
I9 R
R R
R I4 10 A
I10 I5 r1
2 R r3
3 1 r2
+ V
− S
R I6 I7 I3
I8 r4
R I2 R
R
+ R
− I1 r5
E
r6
5 R

Ch 1_Solved Examples & Practise exercises.indd 98 3/19/2016 3:50:39 PM


SOLVED EXAMPLES     99

Solution:  The equivalent circuit is The tie-set matrix is

1 Branches
6 Nodes
3 1 2 3 4 5 6
5 I1 +1 1 0 0 0 0
4 2 I2 0 +1 +1 -1 0 0
2
I3 0 +1 0 −1 -1 0
4 1
I4 0 +1 0 -1 0 +1
3
The incidence matrix is
14. The network shown in the figure given below con-
Branches sists of only two elements. The response for a
unit step excitation is i(t) = e−3t A, then the
Nodes
1 2 3 4 5 6
1 0 0 -1 0 −1 +1 elements are

2 −1 −1 +1 0 0 0
i(t)
3 +1 0 0 −1 0 0
0 +1 0 +1 +1 −1
+
V(t) = u(t) − N
4

13. What is the tie-set matrix of the circuit given


below?
Solution:
10 Ω 2H 10 Ω 5H
1
5Ω
10 Ω
I (s) s + 3 = s = Y (s)
H(s) = =
V (s) 1 s+3
10 H + 2H 5A
s
50∠0° 10 µF

1 3 1 1
Z(s) = = 1+ = 1+ =R+
 
s  
Y (s) s 1 Cs
Solution  Graph of the given circuit is 3

1 4 2
i(t)
3 1Ω
+
V(t) −
5 2 1
6 F
1 3

3
15. Find Thevenin and Norton equivalent circuit
The arbitrary graph is between A and B for the circuit given below.

1 4 2 20 Ω 10 Ω

3 + −20j Ω
5 I2 2 10∠0° 4j Ω
6 I1 1 −

Ch 1_Solved Examples & Practise exercises.indd 99 3/19/2016 3:50:51 PM


100     CHAPTER 1:  ELECTRIC CIRCUITS

Solution:  Thevenin equivalent is 1 2Ω


10∠0° 10∠0°
I= = = 0.33∠ − 7.59 A t=0
30 + 4 j 30.2∠7.59 2
5A 8Ω + 3H
VAB = −6.62∠ − 7.59 + 10 = 3.4 + 0.872 j = 3.51∠14.41 V V 12 Ω

iC
( j4 + 10)20 200 + 80 j
R Th = −20 j + = −20 j +
Solution:  At t = 0−,
30 + 4 j 30 + 4 j
200.03∠1.145 2Ω
= −20 j + = −20 j + 3.68 − 2.24 j
30.26∠7.59
+ −
= 3.68 − 22.4 j
+
R Th = 22.54∠ − 88.60 5A 8Ω

Norton’s equivalent is iC(0−)

When the switch is in position 1 for 4 long time,


20 Ω VA 10 Ω 3 H inductor behaves as a short circuit. Therefore,
5×8
−20j iL (0− ) =
+
= 4 A = iL(0 )
10∠0° 10
4j
Isc At t ≥ 0,
2Ω

VA − 10 VA VA 12 Ω 3H 4A
+ +
20 20∠ − 90° 10 + 4 j

(VA − 10)(10 + 4j) + jVA(10 + 4j) + 20VA = 0


When the switch is moved to position 2,
30VA − 4VA + 14jVA − (100 + 40j) = 0
Rtotal = 2 + 12 = 14 W
26VA + 14jVA = 100 + 40j
t =
L 3
= s
   29.53 ∠ 28.3 VA = 107.7 ∠ 21.8°
Req 14

107.7 ∠ 21.8
14
− t
VA = = 3.64 ∠ 28.3 + 21.8 = 3.64 ∠ − 6.5° iL(t) = 4e 3 A
29.53 ∠ 28.3
At t = 0+
3.64∠ − 6.5
I sc = = 0.182∠ 83.5° 2Ω
     20∠ − 90

A +
V 12 Ω 4A

0.182∠83.5° 22.54∠−80.60°

B V(0+) = (−4) × 12 = −48 V


16. In the circuit shown in the figure given below, the VL(0+) = (−4) × (12 + 2) = −56 V
switch is in position 1 for a long time and it is
moved to position 2 at t = 0. Determine iL(0+), 17. Determine VC(t) and i C(t) for t > 0 in the following
V(0+), iL(t) for t ≥ 0. circuit.

Ch 1_Solved Examples & Practise exercises.indd 100 3/19/2016 3:51:08 PM


SOLVED EXAMPLES     101

2Ω 1Ω t t

ò ò
1 1
iC VS (t) = V1(t) + V2 (t) = i(t) dt + i(t) dt
C1 -¥
C2 -¥
+
+
20u(−t) − 2Ω 1F VC t t

C1 ò0 C2 ò0
1 1
VS (t) = V1(0) + i(t)dt + V2 (0) + i(t)dt

dVS (t) i(t) i(t)


Solution: =0+ +0+
(i)  At t = 0−
dt C1 C2

µF ⋅ (100e−80 t ) = −6.4e−80 t mA
4 d
2Ω 1Ω i(t) =
iC (0−) 5 dt

+ 19. In the circuit given in the following figure, the


+
20 V − 2Ω VC(0−) switch has been closed for a long time and is opened
− at t = 0. Find i and V for all time.

i t=0
VC(0−) = 10V = VC(0+)
10 Ω
(ii)  At t ≥ 0 +
2Ω 1Ω
+
30u(t) − 20 Ω 1 + 10 V

V F
− 4
iC
+
2Ω 1F VC

Solution:  The resistor current i can be discontinuous
at t = 0, while the capacitor voltage V cannot be so.
t ≥ 0 is a source free condition. Hence, it is always better to find V and then obtain i
from V.
VC (t) = 10e−t /2×1 = 10e−t /2 V
30u(t) = 0 t < 0

30u(t) = 30 t > 0
= 5e−t /2 A
dVC (t)
iC (t) = C
dt For t < 0,    V = 10 V

18. In the following circuit, let VS = 100e−80t V and i=−


V
= −1 A
V1(0) = 20 V. Determine the current i(t) for t > 0. 10
i

+ 10 Ω
1 µF V1 +
− 20 Ω + 10 V
+ 2 µF V

VS − i(t) −
+
4 µF V2

20
For t > 0, V∞ = × 30 = 20 V
20 + 10
Solution:  For t > 0, the current is given by
10 Ω i
dVS (t) C1C2 dVS (t)
i(t) = Ceq =
dt C1 + C2 dt
+
+
30 V − 20 Ω 1
F V
Let C1 = 1 mF, C2 = 4 mF. With sources initially at 4 −
(t = 0+) and final at (t → ∞) condition, at t ≥ 0,
we have

Ch 1_Solved Examples & Practise exercises.indd 101 3/19/2016 3:51:28 PM


102     CHAPTER 1:  ELECTRIC CIRCUITS

R Th = 10 || 20 =
20
W t = R Th C = 4 × 103 × 0.5 × 10−3 = 2 s
3
Since the capacitor acts like an open circuit to DC
t =
20 1 5
× = s at steady state V (∞) = 30 V
3 4 3
V (t) = V (∞) + [V (0) − V (∞)]e−t /t
= 30 + [15 − 30 ]e−t /2 = 30 − 15e−t /2
Therefore,

V (t) = V (∞) + [V (0) − V (∞)]e−t /t


Thus, V(15) = 20.902 V
= 20 + (10 − 20)e−t /t = 20 − 10e−3 /5t V
21. Steady state is reached with S open. S is closed at
20 − 10e−3 /5t 1   −3   t = 0. Determine V2(t).
+ −10   e−3 /5t 
V dV
4   5 
i= +C =
20 dt 20 
3Ω S
= 1 − e−3 /5t + e−3 /5t
1 3
2 2
+
20. The switch in the figure shown below has been in 2V 3Ω 1F 3Ω V2

position A for a long time. At t = 0, the switch
moves to B. Determine V(t) for t > 0 and calculate
its value at t = 15 s.
Solution:
3 kΩ A B 4 kΩ At t = 0−,
t=0
3Ω
+ +
+
24 V − 5 kΩ + 30 V
V

0.5 mF −
2V 3Ω V2


Solution:  For t < 0, the switch is at position A.
The capacitor acts like an open circuit to DC.           V2(0−) = 1 V
At t = 0+,
3 kΩ
3Ω
+

+
24 V − 5 kΩ VC (0−) 3Ω 3 Ω V2 (0+)
2V 1V


At t = ∞
VC (0− ) = 24 ×
5
= 15 V = VC (0+ )
5+3
3Ω
For t > 0, the switch is at position B +

4 kΩ 2V 3Ω 3 Ω V2 (∞)

+
VC − + −
− 30 V

1.5 2
Now, V2 (∞) = 2 × = V
We have RTh = 4 kΩ, so 4.5 3

Ch 1_Solved Examples & Practise exercises.indd 102 3/19/2016 3:51:47 PM


SOLVED EXAMPLES     103

2  2 
V (t) = V∞ − (V∞ − V0 )e−at = −  − 1 e−at = + e−t H( jw ) w =1 = ∠ − tan−1 2
1 1
∠ − 45 =
2 1

3 3  3 3 2 5

1 10
22. In the figure given below, determine the steady i(t) = 10 2(t + 10 − 45°) + 5(2 + 11)
state output voltage corresponding to the input 2 5
3 + 4 sin (100 t).
24. Steady state is reached with open S closed at t = 0.
R = 1 kΩ At t = ∞ the voltage marked V is given by.
+ +
1Ω S
Vi = 3 + 4sin(100t) 10 µF Vo
+
3Ω V 2Ω
− −
3A

Solution:  For the given circuit


Vo (s) 1/sC 1 Solution:  At t = 0−
H(s) = = =
Vi (s) R + 1/sC 1 + sCR
1Ω

H( jw ) = and H( jw ) w = 0 = 1∠0°
1
+
1 + jwCR 3A 3Ω V(0−) = 9 V

H( jw ) w =100 =
1
∠ − 45°
2 At t = 0+

Vi (t) = 3 + 4 sin 100t 1Ω

1
Vo (t) = 3 × 1 + × 4 sin (100t − 45°) V
2
3A 3Ω +
9V − 2Ω

23. An input voltage


v+
v(t) = 10 2vcos(t
((tt)10
)=
=+10
10 22 +
5 10)
cos( 2+t10
+ 74
10 ) 22tt +
55.cos(
cos(
43 +74
74..43
43)) V
is applied through as combination of resistor 1 Ω
At t = ∞

and induced steady state current is————————.
1Ω
i(t) R L
+

V(t)
3A 3Ω +
V 2Ω
− −

Solution:
I (s)
=
1 V (∞) = 2 × 1.5 = 3 Volts
V (s) R + sL
or V (∞) = 3 × 1.5 − 1 × 1.5 = 3 V
1
H(s) = Ans. (3)
s+1
25. In the network shown in the figure given below, the
H( jw ) =
1 switch K is changed from position a to b at t = 0
1 + jw (a steady state having been established at position a).

Ch 1_Solved Examples & Practise exercises.indd 103 3/19/2016 3:52:11 PM


104     CHAPTER 1:  ELECTRIC CIRCUITS

C3 R2 L2 Solution:  At t = 0, S is closed, applying KVL is


a
in S-domain,
b K
∫ i(t)dt + 2 × 10−6 ∫ i(t)dt = 0
1 1
i3 1 × 106 i(t) + −6
8 × 10
V R1 L1 R3 C2
i1
C1  106 106  Vc (0) 10
= I (s) 106 +
i2 10
+
2s 
+ .
Show that at t = 0+.
2  8s s s

i1 = i2 = −
V 10 × 10−6 × 8s 10−5
I (s) = =
R1 + R2 + R3  
s(8s + 5) s + 5 

 8 
and i3 = 0 A
10 I (s)
Solution:  When switch is at position a, (t < 0) Hence, V2(s) is V2 (s) = − × 106
s 2s
t<0 R2
10−5 × 106
sV2 (s) = 10 −
 5
2 s + 
VC3
V R3  8
VC2
10
lim V2 (t) = lim sV2 (s) = 10 −
VC1
t →∞ s→ 0  5
2 0 + 
At t = 0 −  8
10 × 4
VC3 = V and iL1 = iC2 = 0 = 10 − =2V
5
VC1 = VC2 = 0 We have V8 (∞) = 2V, so

When the switch is at position b, t = 0+ I (s) 10


× 106 =
V2 (s) =
 5
s s +  × 8
8s
V
R2  8
+− 10
So, we have VS (∞) = lim sV2 (s) = =2 V
s→ 0 5
R3 ×8
R1 i3 8
i1
i2 or V8 (∞) = 2 V

Ans. (2)
−V
i1(0+ ) = i2 (0+ ) = 27. In the circuit given below, find the current i(t)
R1 + R2 + R3 when switch K is closed. The initial voltage across

capacitor V0 = 30 V for t < 0 when the switch is
and i3 (0+) = 0 A open. When switch is closed, the value of current is
26. The switch S is closed at t = 0. If V2(0) = 10 V and
V8(0) = 0 V, respectively, are voltages across the 2 ΜΩ
capacitors, then the voltage at steady state will be. K

V8(t) i(t)
S 2 µF 2 ΜΩ

8 µF +
2 µF

+ 30 V
V2(t) 2 µF 1 ΜΩ
− i(t)
Solution:  From the circuit

Ch 1_Solved Examples & Practise exercises.indd 104 3/19/2016 3:52:34 PM


SOLVED EXAMPLES     105

 3+ 5   3− 5  
 2(1 + 5 ) − 8 −
R  
t 2(1 − 5 )  8  
Impedance of parallel branch, Z(s) =
1 + RCs 30  e  

6  
= e
2 × 10  5 5 
1  
Impedance of series branch, Z1(s) = R +
Cs
  3+ 5   3− 5  
 R RCs + 1   −  −  
Vo
= I (s)  +  30 × 2 (1 + 5 )e  8   8  
 1 + RCs Cs  iC (t) =  + (1 − 5 )e 
2 × 106
s
5 
 
 RCs + 1 + R 2C 2s2 + 2RCs 
= I (s)  
 28. Determine the steady state voltages across the

 Cs (1 + RCs )  capacitors in the following circuit.
R
 1  + +
I (s)R s + + RCs2 + 2s
 RC  t=0
=
[1 + RCs] V1(t) C1 C2 V2(t)
   
i(t)
We have R1 = R2 = 2 MΩ, C1 = C2 = 2 mF, so − −

 1  Solution:  Given that R = 1 Ω, C1 = 1 F, C2 = F,


1
I (s)R s + + 4s2 + 2s
 4  V1(0) = 2 V, V2(0) = 1 V. 2
Vo (s) =
(1 + 4s)
The circuit is source-free. We have
 1 V1(∞) = V2 (∞)
I (s)  4s2 + 3s + 
Vo (s)  4 
= Transform into s-domain as
R (1 + 4s)
1Ω
+ +
V [1 + 4s]
I (s) = + 2 + 1
R [4s2 + 3s + 1 ] − s − s
V1(s) V2(s)
4
1 I(s) 1
s 2s
−3 ± 9 − 4 −3 ± 5 − −
s1, s2 = =
8 8
I (s) 2 1 1
− + − I (s) − − I (s) = 0
 3 + 5   3 − 5 
s 5 s 2s
or s +  s +

 = 0 2 1
 8   8  −
1
I (s) = 5 5 =
3 s+3
1+
  5
 
30 
  i(t) = e−3t   t > 0
1 + 4s 
I (s) =  
2 × 106   3 + 5   3 − 5   t =
1
s
 s +  s +  
  8   8          3
i(t)
  1
 
 
30  2(1 + 5 ) 2(1 − 5 ) 
=  − 
2 × 106   3 + 5  
 3 − 5  
 
5 s + 
 5  s + 

  8   8   0 t

Ch 1_Solved Examples & Practise exercises.indd 105 3/19/2016 3:52:57 PM


106     CHAPTER 1:  ELECTRIC CIRCUITS

1 2 (ii)  At t ≥ 0, it is a source free circuit.


V2 (s) − − I (s) = 0
s s
50 Ω
1 2  1  s + 3 + 2
+  =
s s  s + 3 
V2 (s) = +
200 Ω
s(s + 3) 20 V −
0.1 A
5
2  1  20 µF
= 3 − 
(s + 5)

5 3  s + 3 
=
s(s + 3) i

      Req = 50 Ω
5 2 −3 t
− e for t ≥ 0 −3
VC (t) = 20e−t /1×10 ,
V2 (t) =
3 3
5     t = 1 ms
At t → ∞, V2 (t) = V
3 −3
/1×10−3
2 1 VC (2 ms) = 20e−2×10 = 2.71 V
V1(s) − + I (s) = 0
s s
(iii)  At t = 0+
2 1  1 
V1(s) = −   50 Ω
s s  s + 3 

2.s + 6 − 1 2s + 5
= = i(0+)
      
s(s + 3) s(s + 3) +
20 V − 200 Ω 0.1 A

5 1
V1(s) = 3 + 3 6 t≥0
s s+3 i(0+) = 0A

Since there exists a short circuit across the 200 Ω
5 1 −3 t resistor from t = 0+.
V1(t) = + e t≥0
3 3 30. Find Laplace transform for the given graph.
29. For the circuit shown in the figure given below, the
switch is opened for a long time; it closed at t = 0.
Determine VC(0+), i(0+)and VC(2 ms). 1

50 Ω
0 1 t
+
20 µF 200 Ω t=0 S Solution:  r(t) = t u(t)
VC

0.1 A
f(t) = r(t) − r(t − 1)
i
1 e−s
f (s) = 2

s s2

Solution:  (i) At t = 0
(1 − e−s )
1
=
50 Ω     s2

− 0V +
+ +
VC(0−) 200 Ω 20 V 0.1 A
r=1
− −

At t = 0− steady state is reached.


r = −1
VC(0−) = 20 V = VC(0+)

Ch 1_Solved Examples & Practise exercises.indd 106 3/19/2016 3:53:16 PM


SOLVED EXAMPLES     107

31. Find Laplace transform for the given graph. connected in series. The impedance matrix of the
resulting two-part network will be
Solution:  Two impedance matrix connected in
1 series

 Z A + Z11B Z12 A + Z12 B  14 7 


Zeq =  11 = 
Z12 A + Z21B Z22 A + Z22 B   7 15
0 1 2
Solution:  f(t) = r(t) − 2r (t − 1) + r(t − 2) 34. In the circuit given in the figure below, the energy
absorbed by the 4  Ω resistor in the interval
1 2e−s e−2s (0, ∞) is
= − +
s2 s2 s2

=
1
(1 − 2e−s + e−2s ) 4Ω
2
s

32. In the circuit given below, switch S was at position + +


10 V − V
− C
2F
1 for a long time. At t = 0, switch is moved from
position 1 to 2. Find the particular solution for
i(t) and Vo(t) when the capacitor C2 is having no
charge. VC(0) = 6 V

R2 1 2 R1
+ (a) 36 J (b) 16 J
(c) 256 J (d) None of the above
S
+ Solution:  For the given circuit:
E C1 C2
− Vo
4Ω

+
+
2 F VC, VC(0) = 6 V
Solution:  When the switch is at position 1 for a 10 V

long time, then current through capacitor is zero − iC + 1
and voltage across capacitor at E is as shown in the
following figure.

At t = 0, C behaves as short circuit.


+ Voltage across 4 Ω = 10 — 6 = 4 V
E
− therefore, i(0) = 1 A
At t = ∞, C behave as open circuit
When switch is moved to position 2, the equivalent i(∞) = 0,
capacitance is Time constant t = RC = 4 × 2 = 8 s.
Therefore, i(t) = 1.e−t/8
[from i(t) = i(∞) + i(0) − i(∞)e−t/t ]
C1C2 C1C2
CV = t = R1
C1 + C2

C1 + C2 Energy absorbed by 4 Ω resistor in (0, ∞)
 CC 
−R1 1 2  t ∞ ∞ ∞
 C1 + C2 
t
E − ×2
i (t) = e = ∫ i2Rdt = 4 ∫ e 8 dt = 4 ∫ e−t / 4dt
R
t=0 0 0

C2 ∫
1
Vo (t) = i(t)dt
 −e−t/ 4  ∞
= 4   = 16 J

33. The impedance matries of two-port networks are  1 / 4  0
 4 3 10 4
given by   and   and these two are
 3 7   4 8 Ans. (b)

Ch 1_Solved Examples & Practise exercises.indd 107 3/19/2016 3:53:35 PM


108     CHAPTER 1:  ELECTRIC CIRCUITS

35. For the lattice circuit given in the figure below Given that Za = j5 Ω and Zb = 10 Ω. Therefore,
Za = j5Ω and Zb = 10 Ω. The values of the Z
Z Z12 
parameter Z =  11 
Z21 Z22 
Z Z12 
Z =  11  are  Za + Zb Za − Zb 
Z21 Z22   
 
= 2 2

 Za − Zb Za + Zb 
Zb
 
    
1 3
 2 2 
 j5 + 10 j5 − 10 
Za  
=  2 2 

Za  j5 − 10 j5 + 10 
 2 2 
    
2 4 36. The Z parameters Z11 and Z21 for the 2-port network
given in the figure are ———————.
Zb
I1 4Ω I2
Solution:  The equivalent circuit is
I1
+
Zb 10 Ω
Za E1 E2
20 E1
V1 + V2 −

Za Zb Solution:  From the given network


− E1 = 4I1 + 10(I1 + I2) − 20E1
Network is symmetric, therefore 21E1 = 14I1 + 10I2
Z11 = Z22 E2 = 10(I1 + I2) − 20E1
Z12 = Z21 E2 + 20E1 = 10I1 + 10I2 (as output is open circuit).
The Z parameters are E1 14
V Z11 = =
Z11 = 1 I1 21 I2 = 0
I1 I = 0
2
14
E2 = 10I1 − × 20I1
When I2 = 0 21
V1 (Z + Zb )(Za + Zb ) Za + Zb 210I1 − 280I1 −70
= Z11 = a =
I1 Za + Zb + Za + Zb 2 =
21
= I
21 1 I2 = 0
−70
V2
Z21 = E2
= Z21 =
I1 I2 = 0 I1 21
When I2 = 0 37. Determine the h-parameter of the network given
 Z Zb  below.
V2 = V1  a
− 
      Za + Zb Za + Zb  1Ω I2
I1 V1 2 V1
−+
(Za + Zb )  Z Zb  + +
= I1  a
− 
2  Za + Zb Za + Zb 
V1 1Ω 2 V2 2Ω V2
(Za − Zb )
= I1
    2 − −
Therefore,
V2 Z − Zb
= Z21 = a
I1 2 Solution:  −I1 + V − 2 V2 + V1 + 2 V1 − V2 = 0
1 1

Ch 1_Solved Examples & Practise exercises.indd 108 3/19/2016 3:53:55 PM


SOLVED EXAMPLES     109

I1 = 4V1 − 3V2  1 1  2 1
Solution:  TA =   TB =  
V2 V2 − 2 V1 − V1  0 1 3 2
+ = I2
2 1             T = TATB
3
I2 = −3 V1 + V2  1 1 2 1
=  
2  0 1 3 2

 4 −3   5 3
Y =  = 
−3 3/2 3 2
  
Y12 = Y21 Reciprocal network AD − BC = 1 Reciprocal network
1 3 V1 = AV2 − BI2
V1 =
I + V I1 = CV2 − DI2
4 1 4 2
 1 39. Determine the current through the ideal diode (D).
= V2 − −  I1
3
 4 
   4 4Ω D

3 1  3
I2 = −3  V2 + I1  + V2
id
 4 4  4 +
10 V 4Ω 1Ω
− 2A
3 3
I2 = − V2 − I1
2 4
 1 3 
  Solution:  Since the diode is a non-linear element, the
h =  4 4 
−3 
network is non-linear and hence superposition theorem
 −3
 4 2 
is not applicable (source transformation is applicable).

D
h12 = −h21 Reciprocal network
id
Note: Even though the network comprises two con-
trolled sources in it, it is incidentally obeying the 5
A 2Ω 1Ω 2A
reciprocity property. In general, it is not the case. 2
If instead of 2V2, 3V2 source is used then the Y
parameters are
 4 −4 
  2Ω D 1Ω
Y = 3
 − 3 
 2  id

and the network is non-reciprocal. ⇒ +


5V − +
− 2V
 2 1
38. T-parameters of a two-port network are T =  . If
3 2
a 1 Ω resistor is connected in series with any one
of the input leads then the T-parameters of the For an ideal diode Vf = 0 and Rf = 0
overall two-port network will be So
+ −
1Ω
id
N + −
2Ω 1Ω
1Ω + − id + −
N + +
5V − − 2V
TA
TB

Ch 1_Solved Examples & Practise exercises.indd 109 3/19/2016 3:54:15 PM


110     CHAPTER 1:  ELECTRIC CIRCUITS

5 − 2id− 1id − 2 = 0 Solution:  The period of the voltage waveform is


T = 2p and
⇒ id = 1 A
10 sin t 0 < t < p
Ans. (1) v(t) = 
 0 p < t < 2p
40. In a parallel RLC circuit, the power factor at
The rms value is obtained as
p
f = fH is T

∫ ∫ (10 sin t) dt
2 1 2 1 2
V rms = v (t)dt =
Solution: T 2p
0 0

B
p


Y 1 100
Capacitive = (1 − cos 2t)dt
2p 2
f 0

G p
50  sin 2t 
= t − 
Y = G + jB 2p  2  0
G
2
V rms = 25 V
Inductive
Y Average power absorbed is,
B
Y = G − jB 2
V rms 25
P = = = 2.5 W
R 10
cos f =
G
Y Ans. (25, 2.5)
Vm
At f = fH, V =
2 42. Find the Thevenin equivalent of the circuit between
terminals A and B, and also find the current Ix and
I I power through 10 Ω resistance.
=
4Ω 6Ω 2Ω
Y 1
2.
R A
Ix
2 + 10 Ω
Y = = 2G
R 200∠0° 10 Ω
− 4Ω
Purely resistive,
B
cos f =
1
= 0.707 lead (Capacitive)
2 Solution:  The Thevenin equivalent circuit is
4Ω 6 Ωj 2 Ωj
Note: At f = fL, cos f = pf = 0.707 lag (Inductive) A

Ans. (0.707) 10 Ωj
41. The wave shown in the figure given below is a half 200∠0°
wave rectified sine wave. Find the rms value and the 4Ω
amount of average power dissipated in a 10 Ω resistor.
v(t) B

(4 + 6 j)(10 j + 4) (40 j + 16 − 60 + 24 j)
Req = 2 j + = 2j +
10 8 + 16 j 8 + 16 j
(64 j − 44) 642 + 442 ∠ − 34.5
= 2j + = 2j +
0 p 2p 3p
t 8 + 16 j 17.8∠63.4
= 2 j + 4.32∠ − 97.9 = 2 j − 0.59 − 4.27 j

Ch 1_Solved Examples & Practise exercises.indd 110 3/19/2016 3:54:35 PM


L+=A().L)0152A)+    111

Req = −0.59 − 2.27j − 3   3 


3V1 − 5  + j0.5 − 6 0.5 + j 
VA − 200 VA  2   2 
+ =0
4 + 6j 4 + 10 j
15
(VA − 200)(4 + 10 j) + VA (4 + 6 j) 3V1 + − 1.12 j − 1.22 − j2 − 12 = 0
=0 2
(4 + 6 j)(4 + 10 j)
3V1 + 1.93 − 1.22 − 2.12 j = 0
4VA + j10VA − 800 − 2000j + 4VA + 6VA j = 0
8VA + 16jVA − (800 + 2000j) = 0
3V1 = 0.71 + 2.12 j
8VA + 16j VA = 800 + 2000j
0.71 2.12
VA = 121∠9.76 V V1 = + j = 0.2367 + 0.70 j
3 3
−0.5j−2.27j
V1 = 0.745∠ − 71.48

10 Ω 44. Determine V2/V1 for the given network.


+
121∠9.76 −
5H
R1 = 10 Ω i2
Zeq = 9.41 − 2.27j = 9.676∠ − 13.56 +
i2
121∠9.76 R2
= 12.5∠23.32
V1 = 10 V L1
Ix =
9.676∠ − 13.56
L2
400 Ω V2
w = 50
10 H
100 H

Power = 1.408 kW
43. In the circuit below e1(t) = 5 cos(wt + 60°) and
e2 (t) = 6 sin(wt + 60°). The voltage across 2  Ω Solution:  For the given network
grounded resister is –––.
R1 + jwL1
2Ω 2Ω
1 (i) i1(R1jwL1) − i2 jwM = V1
i1(10 + j500) − i2 j250 = 10
+ +
(ii) i2(R2 + jwL2) − i1 jwM = 0
e1(t) 4Ω e2(t)
i2(400 + j5000) − i1 j250 = 0
− −
(10 + j500) 10
−j250 0
i2 =
(10 + j500) −j250
Solution:
−j250 (400 + j5000)

   e1(t) = 5 cos(w + 60°)


i2 = 0.00102∠ − 84.13°
e1(t) = 5 sin(w + +60° + 90°) V 2 = i 2 × R2

e1(t) = 5 sin(w + 150°) = 5∠150° = 0.001∠ − 84.13° × 400


(i)

e2 (t) = 6 sin(w + +60°) = 6∠60° = 0.408∠ − 84.13°


(ii)

Applying nodal analysis at node 1, we get V2 0.408


= ∠ − 84.13°
V1 10
V1 − 5∠150° V1 V1 − 6∠60°
+ + =0
2 2 2
= 40.8 × 10−3 ∠ − 81.13°
V2
3V1 − 5(cos 150° + j sin 150°) − 6 (cos 60° + j sin 60°) V1

Ch 1_Solved Examples & Practise exercises.indd 111 3/19/2016 3:55:13 PM


112     CHAPTER 1:  ELECTRIC CIRCUITS

PRACTICE EXERCISES

Set 1 (One Mark Questions) 5. The box contains a resistor and independent sources.
For R = 0 Ω and 10 Ω, the value of current is 5 A and
1. The equivalent resistance of four parallel resis- 2.5 A, respectively. The value of I for R = 5 Ω will be
tances is 40 Ω. The currents through them are 0.4,
I
0.5, 0.3 and 0.2 A. The lowest value of resistance is
–––. Box R
2. In the given circuit if the current through the resis-
tance R is zero, what is the value of I?
(a) 3.33 A  (b) 33.3 A  (c) 22.5 A  (d)2.25 A
5Ω 5Ω 6. The circuit shown in Fig. (a) is replaced by Fig. (b).
R = 5Ω
− + The current I remains the same. The value of Rx is
–––.
20 V 10 Ω
5Ω I I
20 Ω

V
4 5
(a) 4 A  (b) 2 A  (c) A  (d) A
5 4 40 Ω
3. The network shown in Fig. (a) can be replaced by (a)
Fig. (b) when IC and RC, respectively, are –––.
20 Ω
Rx
10 Ω 40 Ω
20 Ω
V
+
80 Ω
20 V
− (b)
(a) 2.80 Ω  (b) 3.88 Ω  (c) 0.088 Ω  (d) 4.88 Ω
(a) 7. In the circuit given below, current Ix is
30 Ω

IC RC 10 Ω
Ix 20 Ω
20 Ω 10 A 20 A

(a) 9 A  (b) 8.33 A  (c) 9.33 A  (d) 4 A


(b) 8. In the circuit given below, the instantaneous current
4. Current through the voltage source in the following i1(t) is given by
circuit is –––.
j5 Ω −4j Ω

10 Ω
10∠10° i1 3Ω 20∠60°
20 V 0.5 A
4Ω
1

Ch 1_Solved Examples & Practise exercises.indd 112 3/19/2016 3:55:26 PM


PRACTICE EXERCISES     113

(a) 20∠90° (b) 18∠89.26° 2Ω 1Ω


(c) 17∠89.26° (d) 15.59∠89.26°
2Ω
+
9. In the figure given below, the input impedance is 1V − 2H
given by
j10 j4
1 1 1
(a) s   (b) s  (c) s  (d) 1 s
3 4 2
j10 j10
15. In the circuit given below, the Norton equivalence
j20 current in amperes w.r.t. to the terminals P and
Zin Q is _____.

(a) 6 jΩ  (b) 8 jΩ  (c) 4 jΩ  (d) 10 jΩ


40j Ω
10. In the circuit given in the figure below, determine R. P

+ R − 20∠0° 25 Ω −20j Ω

14 Ω V 1Ω 20j Ω
100 V 40 V Q
+ − + −
10 A +
50 V 2 Ω
16. In the circuit shown in the figure below, the value
of voltage source E is _____.
+
100 V − − + 40 V

15 A V2 0V

10 A −
0V 5A +
3V − 2 +7V
11. In the circuit given below, determine Vab.

+ 2Ω − a + 1Ω −
E
1
+ +
+ − −
1V 2Ω 3A
− −
+ 6V 4V
V1 10 V
b
(a) 14 V  (b) 21 V  (c) 7 V  (d) 0 V
(a) 2.5 V  (b) 3.5 V  (c) 7 V  (d) 5 V 17. Find Vo in the circuit shown in figure below.

13. Find the number of twigs for the given connected


Vo
graph.
+ 3Ω − 6Ω

4Ω 3Ω
12 V

14. Determine the time constant of the circuit shown (a) 2 V (b) −3 V
in the following figure. (c) +3 V (d) None of the above

Ch 1_Solved Examples & Practise exercises.indd 113 3/19/2016 3:55:42 PM


114     CHAPTER 1:  ELECTRIC CIRCUITS

18. Determine the time constant of the circuit shown 23. Current supplied by the AC current source in the
in the figure given below. figure below is

1Ω

1V 1Ω 1F 14 A
R L C
14 A 10 A 3A

19. Find the time constant of the circuit shown in the (a) 20 A (b) 15.6 A
figure given below. (c) 14.6 A (d) None of the above

2Ω 2Ω 2F 24. The circuit given below is initially in steady state.


Switch K is opened at t = 0, the time constant of
the circuit is

10 V 2Ω 1F 1F
K
20 Ω t=0

15 Ω
+ +
20. In the circuit given below, current I2 = 3 A when
the value of R1 is 20 Ω. Find the value of I2 with
0.4 V 4H VR
R1 = 10 Ω. − − +
10 V

I1 R1 I2
N1 N2
(a) 0.55 s  (b) 0.35 s  (c) 0.40 s  (d) 0.50 s
4A 20 Ω
25. The value of current i(t) in the following circuit is
_____.
(a) 5 A   (b) 2 A   (c) 4 A   (d) 3 A
21. The circuit given below is initially in steady state. i(t)
The switch S is closed at t = 0. Find values of V t=0
and dV /dt at t = 0+.
4Ω 2F
2H 20 V
4H 2Ω
S
10 A 5Ω
3Ω V 2F
26. In the circuit given below, the switch K is closed at
t = 0. The time constant and initial value of cur-
rent i(t) are –––.
22. In the figure given below, S is in position `a’ for a i(t) K 20 Ω
time t. At t = 0, S is moved to position `b’. At
t = 0+, the current I in R = 2 W is given by +
4V 4F

a b 20 V +
1Ω
5V 2F
C −
I R = 2Ω 3A
4V
27. In a two-terminal network open circuit, voltage
measured at the same terminals by an electronic
(a) 4 A   (b) 0 A   (c) 1 A   (d) 2 A voltmeter is 200 V. A short circuit current measured

Ch 1_Solved Examples & Practise exercises.indd 114 3/19/2016 3:55:54 PM


PRACTICE EXERCISES     115

at the same terminals by an ammeter of negligi- a b 2000 Ω


ble resistance is 5 A. If load resistance of 100 Ω
is connected at the same terminals, then the cur- S
rent in the load resistor will be _____. 1000 Ω 100 V
28. If the transmission parameters of the network V 10 µF
given below are A = C = 2, B = 4 and D = 5. The i
value of Z input is

(a) 100 V, 110 mA (b) 110 V, 200 mA


(c) 100 V, 100 mA (d) 100 V, 150 mA
Two-port 20 Ω
V1 V2 33. A delta connected load is having impedance Zb = 5Z
N/W
in each branch. So, the corresponding star connection
having impedance Zb in each branch is

Zin 1 1
1

20 44 √5 Z √5 Z Zb
(a) W (b) W
5 45 ⇒ Zb
(c) 40 W (d) 40 W √5 Z 3
45 44 2 3 Zb 2

29. The Y-parameter of the following network is 3

 0 −1/2 5Z
 
5Z
(a) (b)
1/2 1 3 3

(a) non-reciprocal and active (c) 5Z (d) 5Z


(b) non-reciprocal and passive 3 3
(c) reciprocal and active 34. For the circuit given below, determine Rab.
(d) reciprocal and passive
30. Determine Y12 for the network shown in the figure 1Ω 5Ω

2Ω
given below. a b

20 Ω 4Ω 3Ω

YC
35. In the circuit given below when the voltage V is
YA 5Ω YB 10 Ω 20 V, the current i is 2 A. If the supply voltage
across c−d is 200 V, the short circuiting current
flowing through terminal a−b will be _____.
31. Find h12 of the network shown in the figure given
below. a c

Linear
I1 R R I2 E passive i
+ +
N/W

V1 R V2 b d

− − 36. In the circuit shown in the following figure, the


voltage across the 2 Ω resistor is 20 V. The 5 Ω
32. Switch S in moved from `b’ to `a’ of the circuit given resistor between terminals A and B can be replaced
below. What are the values of V(0+) and i(0+)? by an

Ch 1_Solved Examples & Practise exercises.indd 115 3/19/2016 3:56:13 PM


116     CHAPTER 1:  ELECTRIC CIRCUITS

1Ω 10 Ω 20 Ω
R1 2Ω 10 A

+
20 V 5A 5A
A V 5Ω
3Ω
+
E R1 5Ω 5Ω 50 V 3Ω

B

(a) ideal voltage source of 25 V with the positive (a) 1.5 A (b) 1.40 A
terminal upwards. (c) 1.42 A (d) None of above
(b) ideal voltage source of 25 V with negative ter-
minal upwards. 41. Equivalent resistance between A and B terminals is
(c) current source of 2 A upwards.
(d) current source of 2 A downwards.
1Ω 1Ω
37. Find the value of power delivered by dependent 1Ω
current source from the circuit below.

10 Ω 1 1Ω 1Ω

+ I
Vx A B
10 Ω A
− Vx = 20 V 4
W (b) W (c) W (d) W
5 6 4 8
(a)
6    5    3    3
(a) 300 W (b) 400 W 42. The total power absorbed in the given circuit is
(c) 350 W (d) 250 W
1A 0.25 A
38. R, L, C are connected in parallel across a sinusoi-
dal voltage source of 250 V. If the current through 0.75 A
inductor, capacitor and resistance are 3 A, 4 A +
and 3 A, respectively, find the value of XL. 3V 3V 4Ω 0.25 A

10 A
(a) 3 W   (b) 4 W   (c) 5 W   (d) 6 W
3A 3A 4A
43. In the following circuit, current through the induc-
250 V R L C tor is —————————.

5A 5A 4A
(a) 100 Ω (b) 80 Ω 3A
(c) 83.3 Ω (d) none of above +
39. Find value of R so that the current through RL V L R C
is zero. −

4Ω
44. Determine the angle between the VS and VL.

50 V RL = 10 Ω 17.32 Ω j10 Ω
R
+ V −
L

(a) 0 Ω   (b) 5 Ω   (c) 10 Ω   (d) 15 Ω + −


40. Find the value of current through 5 Ω resistance. VS

Ch 1_Solved Examples & Practise exercises.indd 116 3/19/2016 3:56:28 PM


PRACTICE EXERCISES     117

VL VS (a) 214 VAR (b) 0 VAR


(a) VS = ∠90° (b) VL = ∠60° (c) 314 VAR (d) 210 VAR
2 2
V 55. How many 100 W/110 V incandescent lamps con-
(c) VL = VS∠60° (d) VL = S ∠90° nected in series consume total power as a single 50
2
W/110 V incandescent lamp?
45. Determine average and rms values of 56. Power supplied by the following voltage source is
i(t) = 5 + 3 2 cos(100t + 20) + 5 2 cos(200 + 10°) . 2Ω
46. The rms value of a half and full rectified sine wave
of 2 A is 2Ω 5Ω
Ix−6
(a) I m , I m (b) I m , I m +
2 2 2 2 − 10 V
2A
(c) I m , I m (d) Cannot be determined 2Ω
3 3 4A

47. Find the average power being delivered to an imped-


ance ZL = 8 − 11j Ω by a current I = 5∠20°A.
48. Find the average power delivered to a 4 Ω resistor 2 Ω Ix−4 Ix
by the current
i1 = 2cos10t − 3cos20t A (a) —12 V   (b) 10 V   (c) 0 V   (d) 4 V

49. Find the average power delivered to 4 Ω resistor by 57. A voltage source of 240 V having an internal
the current i2 = 2cos10t − 3cos10t. impedance of (4 − 5j ) is supplying power to a com-
plex load impedance Z. What will be the maximum
50. A parallel RLC circuit has w0 = 108 and Q = 20. power transferred to the load?
Given C = 20 pF, find R.
58. In the following circuit, the value of R for maximum
(a) × 104 W     (b) 104 Ω
1 power to be delivered is _____.
2
(c) 2 × 104 Ω (d) 25 Ω 20 Ω 2H

51. The Q-factor of series RLC circuit is 100. If all the


components double, then the Q is
V = Vm cos 20t RL
(a) 0.5Q (b) 4Q
(c) 2Q (d) does not change
52. The power delivered by the 20 V source shown in
the following figure is—————————. 59. Which one of the following theorems is convenient
to determine the power dissipation in 10 Ω resistor?
i 4A

+
+
20 V − 20 V 5 Ω 5Ω 5 mH 10 Ω
2A −
+ +
10 sin(100t) sin(200t) 0.02 µF
53. Power output for DC component of a full wave − −
rectifier with maximum value of 10 A AC current
and RL = 10 Ω is given by
(a) Thevenin’s (b) Maximum power transfer
(a) 405.68 W (b) 134.22 W (c) Millman’s (d) Superposition
(c) 117.38 W (d) 120.40 W
60. The dual of series RL circuit is a
54. In a series RLC circuit R = 30 Ω, XL = 40 Ω,
XC = 40 Ω connected across 220 V, 50 Hz supply. (a) series RC circuit (b) parallel RC circuit
Find the reactive power of the circuit. (c) series RL circuit (d) parallel RL circuit

Ch 1_Solved Examples & Practise exercises.indd 117 3/19/2016 3:56:44 PM


118     CHAPTER 1:  ELECTRIC CIRCUITS

61. Determine the current (I) and average power Set 2 (Two Marks Questions)
(Pavg) in the circuit shown in the following figure.
1. In the circuit given below, voltage across 20 Ω resis-
I 20 Ω 200 V tor is 100 V. What is the total voltage across the
+ − + − combined circuit?
+
100 V∠0° 200 V 5Ω

10 Ω
62. For the series RLC circuit, the phasor diagram at 10 Ω 5Ω
a certain frequency is shown in the figure given VS
below. Then the operating frequency is

R L 20 Ω 100 V
+ V − + V −
+
R L
+ 2. In the circuit given below, the value of current in
the 30 Ω resistance is
V VC C

− 20 Ω
20 Ω
VR = IR

X 2Ω Y 30 Ω Z 10 Ω A
V
20 Ω
VC
(a) f = 0   (b) f = f0   (c) f < f0   (d) f > f0
100 V
63. In the circuit shown in the figure given below,
(a) 2 A   (b) 2.3 A   (c) 1.1 A   (d) 3.1 A
steady state is reached with S open. S is closed at
t = 0. Determine the current I in the 1 Ω resistor 3. In the circuit given below, the values of Iy and Vz are
at t = 0+.
20 Ω 50 V
1Ω S
10 A Vz Iy

1Ω 5Ω 5Ω 10 Ω 15 Ω
4V 1Ω
L
20 V
(a) 50 V, 2 A (b) 44.2 V, 2.5 A
(a) 2 A         (b)1 A   
(c) 60 V, 2.5 A (d) 47.2 V, 2.5 A
(c) 0 A (d) None of the above
4. For the circuit shown in the figure given below,
64. In the circuit given below, find the value of V
that would result in a steady state current of 2 A di (0+ ) dV (0+ )
determine L and C .
through the inductor. dt dt
100 Ω
20 Ω 20 Ω
+
20 Ω 1 mH 1 µF VC u(−t)

V 2H
iL

Ch 1_Solved Examples & Practise exercises.indd 118 3/19/2016 3:56:58 PM


PRACTICE EXERCISES     119

5. Determine the potential difference between P and


W , L = H, C = 4 F
1 1
Q in the following circuit. 9. In the given figure, R =
5 6
2A has input voltage V(t) = 4sin3t. The resulting peak
value of current Im is

2Ω 10 V 4Ω i(t)
VP VQ
+ − + − V(t) R L C
+ 10 V

(a) 50 A (b) 44.72 A


+ − + −
(c) 54.7 A (d) 50.72 A
8Ω 6Ω
0V 10. Determine the time constant in the following circuit.
6. For the circuit shown in the following figure, VS = 0, 20 Ω 1H 2Ω
when I = 4 A. The value I for VS = 16 V is
2Ω 2Ω
5A 10 Ω 2H 2H
I
+
IS 2Ω − VS
11. In the figure below, find voltage V(t).
V(t)
4Ω

(a) 16 A (b) 8 A 1 4H +
(c) 4 A (d) 0 A
7. The circuit shown in the following figure acts as a
e3t 3Ω 3Ω
load resistor of
I V 2Ω e4t I

+ 2
4Ω
+
V − 2I
− (a) 4e − 3e
4t 3t
(b) 4(e4t − 3e3t)
(c) 4(e4t − 3e3t) (d) None of the above
12. Resistance Rx = 10 Ω is connected between A−B
W W
4 8
(a) (b) terminals of the circuit given below. The current
3 3 through A−B is _____.
(c) 6 Ω (d) 2 Ω
8. Twelve 4 Ω resistances are used as edges to form a
A
cube. Resistance between two diagonally opposite 2Ω
corners of cube is
4Ω + 10 Ω
2Ω − 2Ω
4Ω
5A
4Ω 5V

4Ω 4Ω
4Ω 5Ω B
4Ω 4Ω
4Ω
4Ω 13. In the circuit given below, current Ix is given by
4Ω
4Ω 4Ω 3Ω
Ix
W W
20 40
(a) (b)
+ +
6 6 10 V − 2Ω 4Ω − 4V
W W
20 40
(c) (d)
3 3

Ch 1_Solved Examples & Practise exercises.indd 119 3/19/2016 3:57:18 PM


120     CHAPTER 1:  ELECTRIC CIRCUITS

(a) 0.153 A (b) —0.153 A 5Ω 1 V1


(c) 0.316 A (d) Cannot be determind
+
14. For the circuit shown in the following figure deter- 10 V 2Ω V
mine the current iL(t) for t ≥ 0 and time constant. C

Assume iL(0) = 10 A.

10 Ω (a)1 F (b) 0.075 F


(c) 3.5 F (d) 0.150 F

 iL  A 19. Steady state is reached with S open. S is closed at


20 Ω t = 0. At t = 0+, the voltage across the capacitor
4 
0.5 H
iL VC and current iC are given by

15. For the circuit shown in the figure given below, S


switch S is open for a long time and it is closed at
t = 0. Determine current through the battery at
t = 0+ and at t → ∞. 2A
1
Ω 1F
1
H
2 2
S 1Ω
t=0 (a) 1 V, 0 A (b) 0 V, 1 A
10 V 1H 1F (c) 0 V, 0 A (d) 1 V, 1 A
20. Steady state is reached with S open. S is closed at
t = 0. Current I in R = 2 at t = 0+ is given by
(a) 0 A and 10 A (b) 0 A and 0A _____.
(c) 10 A and 0 A (d) 10 A and 10 A
S
16. Find i(t) in the circuit in the figure given below for
t > 0. Assume that the switch has been closed for
1Ω R = 2Ω
a long time.
3A 1F I
t=0

21. For the circuit shown in the figure given below, the
2Ω 3Ω i steady state current is _____.
+ 1
10 V − H
3 1Ω 2H
5V
i(t) 1F
−15t −15t
(a) 2 + 3e (b) 3 + 2e 10sin t
(c) 2 + 3e−5t (d) 3 + 2e−5t
17. Find V(t) for t > 0 in the circuit shown in the figure 22. Determine the voltage across the inductor at t = 0+
given below. Assume the switch has been open for and the total energy stored in steady state.
a long time and is closed at t = 0. Calculate V(t)
at t = 0.5.
S 3Ω

2Ω 6Ω t=0
4F
10 V 4H
t=0
+ 2Ω
+
10 V − V
1
F − 50 V
− 3 +
100 200
(a) 0 V, J (b) 4 V, J
3 9
18. In the given network, V = 5 V, dV /dt = −20 V/s
100 200
(c) 4 V, J (d) 0 V, J
at time t, what is the value of C? 3 9

Ch 1_Solved Examples & Practise exercises.indd 120 3/19/2016 3:57:34 PM


PRACTICE EXERCISES     121

23. In the AC circuit given below, the value of phasor 27. S is open for a long time and steady is reached. S is
voltage VAB is closed at t = 0. Let iL be the current in the induc-
A tor. At t = 0+, diL/dt is –––.

I1 I2 S
6Ω
6Ω ZB
2Ω
10∠60° ZA
−j4 Ω j4 Ω 1A
1
F
2
2H

B
(a) 40∠60° (b) 43.3∠60° 28. In the figure given below RC = 6 ms. The input
(c) 43.3∠90° (d) 43.3∠30° voltage V (t) = 5 sin 203 t. The output voltage
Vo(t) is equal to
24. For the circuit shown in the figure given below, the
switch is closed for a long time and it is opened at R
t = 0. Determine vC(t) and iC(t) for t > 0.

t=0 Vi C Vo(t)
5 kΩ 20 kΩ
iS S i
+ +
10 V + 104iS
− 2 µF VC 5 sin(203t + 45°) 2 sin(20−3t + 45°)
− −
(a) (b)
5
(c) sin(203 t + 45°) (d) None of the above
2
10−6 −25t 20 −25t, 10−3 −25t 29. In the circuit given below, find Vo(0+) as
(a)  e−25t, −
20
e (b) e − e
3 3 3 3 Vi(t) = 20 u(t).
20 −25t 10−3 −25t 20 −25t 10−3 −25t 2 kΩ
(c) e ,− e (d) e ,− e
6 6    6 3
+ +
25. Find VC(0), VC(2 ms) and i(0+) for the circuit,
when the switch is in position 1 for long time and 6 µF
Vi(0+) Vo(t)
it is moved to position 2 at t = 0. 6 kΩ 2 µF
732 Ω − −

(a) 10 e−t/0.04
2 1
(b) 10 u(t)
800 Ω + 50 V
2 µF (d) 20e−t/0.005

V (c) 20

30. Find the voltage output Vx from the circuit given


26. S is closed for a long time and steady state is in the following figure after 5 seconds.
reached. S is opened at t = 0. The voltage marked (a) 10 V (b) 5 V
V is V0 at t = 0+ and Vf at t = ∞. The values of (c) 0 V (d) none of the above
V0 and Vf are, respectively, ––– and –––.
0.5 µF
4Ω
+
10 V
1 + 2 kΩ
Vi
2A 4
F S 4Ω V Vx
− −
5s t

Ch 1_Solved Examples & Practise exercises.indd 121 3/19/2016 3:58:05 PM


122     CHAPTER 1:  ELECTRIC CIRCUITS

31. When switch is opened at t = 0, the values of (a) 1 A (b) 0.5 A


dV + (c) 0.58 A (d) 0.7 A
V(0+) and (0 ), are respectively.
dt 35. The value of impedances Z11 and Z12 in following
V figure is
4Ω 4Ω 3Ω
1 2
200 Ω
1
2A H
S 2 2Ω 2Ω

1¢ 2¢
(a) 400 V, +160000 V/s
36. In a two-port reciprocal network, the output open
(b) 500 V, 160000 V/s
(c) 500 V, −160000 V/s
circuit voltage by the input current is equal to
(d) 400 V, −160000 V/s (a) h12 (b) Z12
(c) Y11 (d) B
32. In the circuit given below, the initial current is I(s)
where s is a Laplace variable. The value of current 37. In the given network, determine Zin.
I(s) is I1 I2
A=C=1
I(s) +
5 mH t = 0.0055 s
V1 +
− B=2 V2 10 Ω

− D=3

5 mV 38. If R1 = R2 = R4 = 2R and R3 = 4R. In the circuit


+ given below, the reading of an ideal voltmeter is

33. In the circuit given below switch was in position R1 R4


`a’ for a long time, and is moved to position `b’ at +
time t = 0. The current i(t) to t > 0 is given by A
+ V −
10 V B

20 kΩ a b R2 R3
i (t)
(a) 2.7 V (b) 3.7 V
+ 10 kΩ (c) 1.7 V (d) 4.7 V
100 V 0.5 µF
39. An ideal ammeter is connected between the terminal
− a and b. The reading of the ammeter is –––.
0.4 µF 0.1 µF 6Ω 3Ω
a

6Ω 6Ω
(a) 10 e−500t mA (b) 10 e−400t mA
9.6 V

(c) 20 e−400t mA (d) 20 e−1000t mA


b
34. In the given circuit below, the current IZ is equal to 40. In the circuit shown below, the voltage across
5 Ω resistor is 30 V. The 10 Ω resistor connected
between terminals a and b can be replaced by an.
5Ω
I −j10 Ω 5Ω
+ 10j Ω R1
+ I2

4Ω
20∠0°V I1 a
− 8Ω +
V R2 6Ω 10 Ω
6Ω 6j Ω −
I3 b

3Ω

Ch 1_Solved Examples & Practise exercises.indd 122 3/19/2016 3:58:19 PM


PRACTICE EXERCISES     123

(a) ideal voltage source 22.5 V with positive termi- 46. A series RLC circuit consists of L = 0.5 H, C = 50
nal upward. µF and R = 40 Ω. Calculate voltage across capac-
(b) ideal voltage source 22.5 with positive terminal itor with 220 V AC rms and 50 Hz frequency.
downward.
(a) 140 V (b) 139.6 V
(c) current source of 22.5 A upward.
(c) 138.02 V (d) 142 V
(d) None of the above.
47. Find the power dissipated across 10 Ω in the circuit
41. Determine the power delivered by the 16 V source.
below.
2Ω V 2Ω
+ − 8Ω

8A 2Ω + 4Ω
− 16 V 5A V 10 Ω 10 A

42. In the circuit shown in the figure given below, the


dependent source is (a) 397.4 W (b) 300 W
(c) 297.5 W (d) None of the above
3Ω
48. Find Vx in the given network.
+ 1Ω
V1 2 Ω +
5Ω

5V −
− +
2A +
2Vx Vx 5 Ω
(a) absorbing 38.4 W   (b) delivery 38.4 W 20 V −
(c) delivery 19.2 W    (d) absorbing 19.2 W
43. In an AC circuit shown in the figure given below,
what value of C will result a unity power factor at
the source? 49. Find the value of dependent current source as
given the following figure.

220 V V1 4Ω 3 Vx
C
50 Hz +
Vx 3 Ω
ZL = 30∠40°
4A −
Y
10 V
44. The power factor of series RLC circuit at f = fL is i1
(a) 0.5 lead (b) 0.707 lead
(c) 0.5 lag (d) 0.707 lag
45. The current i(t) as shown in following figure is (a) 4 V (b) 3.6 V
flowing through a resistor of 10 Ω. The average (c) 5.6 V (d) 4.5 V
power dissipated in the resistor is ––––.
50. Find the value of R in the figure given below
i(t)
6Ω

10 A 10 A
100 V 10 Ω R

0 2 ms 4 6 t(ms)
(a) 1.5 Ω (b) 3.2 Ω
−10 A (c) 2.5 Ω (d) 4 Ω

Ch 1_Solved Examples & Practise exercises.indd 123 3/19/2016 3:58:31 PM


124     CHAPTER 1:  ELECTRIC CIRCUITS

51. Find E0 in the circuit given below. b

5Ω 5Ω
40 Ω 60 Ω
+
5A 5Ω 5Ω E0 2Ω 10 A a I2 10 Ω I3 c


30 Ω 50 Ω
52. In the circuit given in the following figure, power
delivered across 10 Ω is given by d
(a) 100.46 W (b) 9.60 W I1
(c) 90.6 W (d) none of the above f e

ANSWERS TO PRACTICE EXERCISES

Set 1 (One Mark Questions) When R = 10 Ω When R = 5 Ω

1. Total current = 1.4 A. The voltage remaining is 5A 5A


calculated as V = 1.4 × 40 = 56 V 10 Ω 10 Ω
Therefore, lowest value of resistance is 10 Ω 5Ω
56 = 0.5 × R ⇒ R = 112 Ω 2.5 A 2.5 A
Ans. (112)

2. (b) Arrange the circuit into bridge form and as it is 5 × 10


Hence, = 3.33 A
a balanced one, so current source is replaced by 5 Ω. 15
20 = I(5 + 5) 6. (c) The parallel combination 10 || 20 || 40 = 14/30 Ω

20 V V
I= =2A I1 =
14 / 80
=
0.175
10
V
3. When IC = 2 A, RC = 10 Ω, as shown in the fol- I2 =
56
lowing figure. + Rx
640

As I1 = I2, we have
V V
=
2A 10 Ω 10 Ω 0.0875 + Rx 0.175
Rx = 0.175 − 0.0875 = 0.0875 Ω
7. (b) Convert current source to voltage sources
Ans. (2, 10) 10 Ω 30 Ω 20 Ω
4. We have
20 100 V Ix 400
I10 W = =2A
10
I = 2 + 0.5 = 2.5 A
Ans. (2.5) 100 + 400 500
Ix = = = 8.33 A
5. (a) When R = 0, I = 5 A and when R = 10 Ω, I = 2.5 A. 60 60

Ch 1_Solved Examples & Practise exercises.indd 124 3/19/2016 3:58:45 PM


ANSWERS TO PRACTICE EXERCISES     125

8. (d) Applying KCL at node 1, we have 40j P


10∠10° + i1(t) = 20∠60° I1

i1(t) = 20∠60° − 10∠10° = 20(cos60° + j sin60°) 20∠0° 25 Ω


    − 10(cos 10° + j sin 10°) I1
= 20(0.5 + j0.866) − 10(0.98 + j0.173)
= 10 + j17.32 − 9.8 − j1.73 20j Q
20∠0 × 25
= 0.2 + j15.5j
= 15.59∠89.26° I1 =
25 + 60 j
9. (c) The input impedance is = 2.95 − 7.1j
= 7.692∠ − 67.38
Zin = L1 + L2 + L3 − 2M13 + 2M 23 16. (c) Applying KVL on the loop with given potentials
= 24 − 40 + 20 = 4 jW only we get
10 + 4 − E − 7 = 0
10. From the circuit −E = −7 ⇒ E = 7 V
17. (d) Since the diode is a non-linear element, the net-
100 − 40 100 − V
−10 + + =0 work is non-linear and hence superposition theo-
R 14 rem is not applicable.

As V = 30 Volts, so Vo
+ 3Ω −
60 70
−10 + + =0

R 14 36
12 V
13
= 5 ⇒ R = 12 Ω
60
R
Ans. (12) For an ideal diode Vf = 0 and Rf = 0. So

W
36
11. (b) Using nodal at a (6 + 3)  4 =
13
Va − 1 Va Vo = I × 3
+ −3 = 0
2 2 12 12 × 13
I= = A
2Va − 1  
3 + 
36 75
=3  13 
2
So,      2Va = 7 ⇒ Va = 3.5 V = Vab
Therefore,
12 × 13
Vo = × 3 = 6.24 V
13. The number of circuit sets = number of twigs = 75
n − 1 = 3.
18. From the given circuit
 L  Ans. (3)
  s t = Req × C = 1 × 1 = 1 s
14. (d) We have t = 
 Req  Ans. (1)

Now, Req = 2||2 + 1 = 2 Ω 19. From the given circuit


 2 × 2 
So, t = t = Req × Ceq = (2 || 2 + 2) 
2
=3×1=3s
 4 
= 1s
2
15. The Norton equivalent circuit is Ans. (3)

Ch 1_Solved Examples & Practise exercises.indd 125 3/19/2016 3:59:06 PM


126     CHAPTER 1:  ELECTRIC CIRCUITS

20. (c) I2 remains the same, that is, 3A. Therefore, 4A In series connected capacitors,
is the series current. 2×4 8
Ceq = = = 1.33 F
21. In steady state condition, inductor is short circuit 6 6
and capacitor is open circuit and charged to 10 × 5 t = RC = 20 × 1.33 = 26.6 s
= 50 V, that is, V = 50 V. When switch is closed at Ans. (26.6, 0.55)
t = 0, then V at (0+) = 50 V and remains constant. 27. For the two-terminal network, the internal imped-
The capacitance does not allow sudden change in
= 40 W
200
dV + ance of the network =
voltage so (0 ) = 0 as VC is constant. 5
dt Ans. (50, 0)

22. (d) At t = 0−
N/W 200 V N/W 5A

1Ω
VC When load of 100 W is applied, then the current
4V 200
= = 1.428 A
140
VC(0−) = 4 V Ans. (1.428)
At t = 0+
28. (b)For the given two-port network

4V I 2Ω 3A I1 I2

20 W
Two-port
By superposition, I = I1 + I2 = 2 + 0 = 2 A V V2
N/W
23. (b) Current supplied is

I = I1∠0° + I L∠90° + IC∠ − 90° = 14 + 10j — 3j Zin

A = C = 2,  B = 4,  D = 5
  = 14 + 7 j = 142 + 72 = 15.6 A
 A B   2 4
 = 
24. (c) The circuit previously in steady state and induc- C D  2 5
tor is in saturated condition as switch in opened at
t = 0. The inductor will regain its value of 4 H. V2 + 20I2 = 0
V2
Hence, time constant t =
L 4
= = 0. 4 s . V2 = −20I2 ⇒ = −20 W = Z22
R 10 I2

25. Source is 10 V connected across two parallel paths V1


As, Z11 =
iL(t) + iC(t) = i(t) I1
 V1 = 2V2 − 4I2
− t 
R
V 
i(t) = 1 − e L  + e−t /RC = [1 − e−t ] + e−t / 4
V 20 20
I1 = 2V2 − 5I2
R  R 4 2
  Thus,
= 5 [1 − e−t ] + 10e−0.25t A
2V2 − 4I2 2 − 4I 2 / V 2
At t = 0, i(t) = 15 A. Z11 = =
2V2 − 5I2 2 − 5I 2 / V 2
Ans. (15)
2 − 4 / 20 11 / 5
= =
26. Initial value of current, 2 − 5 / 20 9/4
20 − 4 − 5 = 20 I(t) = 0
W
44
Z11 =
45
11
I (t) = = 0.55 A
20 29. (a) As Y11 ≠ Y22 and Y12 ≠ Y21

Ch 1_Solved Examples & Practise exercises.indd 126 3/19/2016 3:59:30 PM


ANSWERS TO PRACTICE EXERCISES     127

1 −1 100
30. Y12 = +Y21 = − = = −0.05  i(0+ ) = = 0.1 A = 100 mA
ZC 20 1000
(As discuessed in Problem 1.119.)
33. (c) Z branch for star connection is
1 1
Y11 = + = 0.25 
5 20
5Z × 5Z 5Z 2 5
1 1 Zb = = = Z
Y22 = + = 0.15  5Z + 5Z + 5Z 3( 5 ) Z 2 3
10 20

Ans. (—0.05) 34. The equivalent circuits corresponding to the given


circuit are:
31. For the given network

V1   h11 h12   I1 


 =   2/7 5
 I2  h21 h22  V2 
4/7
The network is symmetric and reciprocal.
h12 = −h21 8/7 3

I2
h21 = I1 .R
I1 −I2 = 37/7
V2 = 0 R+R 4/7
  
I2 1
h21 = −
1 =− a b
2 I1 2
     29/7
1
h12 = 37 29
2 ×
Rab
4
= + 7 7 = 4 + 37 × 29
Ans. (0.5) 7 37 + 29 7 7(66)
32. (c) At t = 0− (when the switch is at `b’)
7
4 × 66 + 37 × 29
= = 2.89 W
2000 Ω 7(66)
− +
0V Ans. (2.89)
+
VC(O−) 100 V 35. The circuit is

Linear
+

V passive Ix
N/W
VC (0− ) = 100 V = VC (0+ )
(a)
At t = 0+ (when the switch is at `a’) When V = 20 V, Ix = 2 A. When supply voltage
is 200 V,

+
1000 Ω + 1000 Ω − 100 V
+
10 µF VC 10 µF Ix N/W − 200 V

i

For t ≥ 0, it is a source free circuit. (b)


Using reciprocity principle Ix = 20 A.
−6
VC (t) = 100 e−t /1000×10×10 V
Ans. (20)
−2
= 100 e−t /10 V 36. (a) Ideal voltage source of 25 V with positive termi-
nal upwards. (Explanation on bases of substitution
t = 0.1 ms theorem.)

Ch 1_Solved Examples & Practise exercises.indd 127 3/19/2016 4:00:00 PM


128     CHAPTER 1:  ELECTRIC CIRCUITS

20 VS
37. (d) Value of dependent current source = = 5 A. VL = × j10
4 17.3 + j10
As it is connected across 10 Ω resistance, the power
VS
= (5)2 × 10 = 250 W. = .j
3+j
38. (c) Given that IL XL = 250 V, IL = 3 A, therefore,
VS V
250
= 83.3 W = 1∠90° = S ∠60°
XL =
3 2∠30° 2

39. (a) For R = 0, that is, short-circuited. 45. The average current is
40. (c) Voltmeter means open circuit, so current through Iavg = 5 + 0 + 0 = 5
5 Ω is T

∫ i (t)dt
1 2
50 I rms =
I= = 1.42 A T
35 0

41. (a) The equivalent circuit is  3 2 2  5 2 2


= 52 +   +  
 2  = 59
 2   

1
 3 2 2  5 2 2
3 1Ω = 52 +   +  Ans. (5,

1
 2   2  = 59 )
3  
46. (a) We know that

Ω 1Ω
1 Im
3 I rms = = 1A
A B 2

4 4 12 Half wave Full wave


×
1 3 3 1 rectifier rectifier
Req = + = + 9
3 4+4 3 8
3 3 3 Im 2I m
I avg = IDC = I avg =
p p
= + × = + = Ω
1 12 3 1 1 5
3 9 8 3 2 6
Im Im
I rms = I rms =
42. (a) From the given circuit 2 2

 P3V = 3 × 1 = 3 W (delivered)
1 2
  P4Ω = 3 × 0.75 = 2.25 W (absorbed) 47. P = (5) 8 = 100 W
P0.25 = 0.25 × 3 = 0.75 W (absorbed) 2
Ans. (100)
Total power absorbed = 2.25 + 0.75 = 3 W
48. Since the two cosines are of different frequencies,
I
43. Power factor = cos f = R = = 0.6 lag
3 the two average power values may be,
I 5
1 2
P = (2 + 32 ) = 4 = 26 W
2
I = IR2 + (I L − IC )2
Ans. (26)
5 = 3 + (I L − 4)
2 2
49. Here, the two components of current are of same
IL = 8 A frequency and they must therefore be combined
Ans. (8) into a single sinusoidal at that frequency.
i2 = 2cos10t − 3cos10t = −cos10t
44. (b) From the circuit
1 2
VS P = (1) 4 = 2 W
I= 2
17.3 + j10 Ans. (2)

Ch 1_Solved Examples & Practise exercises.indd 128 3/19/2016 4:00:38 PM


ANSWERS TO PRACTICE EXERCISES     129

50. (b) Q = w0CR Total resistance R = 246.9 Ω


= 104 W
20 When two lamps are connected in series
R=
108 × 20 × 10−12 = 0.198 × 246.9 = 49 W = 50 W/110 V single
lamp.
51. (a) We have Ans. (2)
1 L
Q= 56. (c) From the given circuit
R C
Ix − 6 + 6 = 1
On doubling of components
Power delivered by source is zero.
Q′ =
1 2L Q
= = 0.5 Q Ans. (0)
2R 2C 2
57. For maximum power transfer
20
52. I5 = = 4A
5 ZS = (4 − j5) W
Applying KCL VS = 240 V

i−2+4=0⇒i=2A ZL = ZS * = (4 + j5) W
Therefore, power is given by
VS V V
P20V = 20 × 2 = 40 W (delivered) I= = S = SA
P2A = 20 × 2 = 40 W (delivered) (4 + j5) + (4 − j5) 2RS 8
P5Ω = 20 × 4 = 80 W (absorbed) Maximum power
Ans. (40)
VS2 (240)2
Pmax = = = 3.6 kW
4×4
53. (a) The power output for DC component is given by
4RS
 2I 2
2
RL =  m  ⋅ RL
 p 
PDC = IDC Ans. (3.6)

 2 × 10 2 4 × 100 58. Given that:


 ⋅ 10 = 405.68 W
 p 
(10) =
p2  V = Vmcos20t
XL = 2 × 20 = j40 Ω and w = 20 rad/s
54. (b) We have
220∠0° Z = 202 + 402 = 1600 + 4000 = 2000 = 44.72 Ω
I= = 7.33 A
30 Therefore, RL must be = 44.72 Ω for maximum
power transfer.
Reactive power = I2X = (7.33)2 × 0 = 0 VAR
Ans. (44.72)
55. For the 100/110 V lamp
59. (d) The reactive elements depend upon the applied
VI = 100 frequency. Whenever two different frequencies
are operating simultaneously in the network, the
100
I= = 0.9 A superposition theorem is always used.
110
60. (b)
2
Total power consumed I R = 100 61. From the circuit, we have

= 123.4 Ω
100
R= 2 V = (VR )2 + (VL − VC )2 = VR2 + 0
(0.92)
If two lamps are connected in series, the circuit is Now, VR = 100 V, cos f = 1
100 W 100 W V 100
A I= = =5A
20 20

 5 2
2
R =   × 20 = 250 W
 2 
Pavg = I rms

110 V Ans. (5, 250)

Ch 1_Solved Examples & Practise exercises.indd 129 3/19/2016 4:01:00 PM


130     CHAPTER 1:  ELECTRIC CIRCUITS

62. (c) Here f0 is the resonance frequency. Set 2 (Two Marks Questions)
VR leads the voltage V then nature of the circuit is 100
capacitive, therefore f < f0. 1. We have I = = 5 A and
20
Note: If VR lags the voltage V then the operating In parallel combination
frequency f > f0. 55×
×10
33WW
10 1010 116
116..65
65
R eq =
Req = =
= = = 33..33
63. (b) At t = 0−, iL =
4 15
15 3
3 10
10
=2 A     
2 Veq = 5 × 3.33 = 16.65 V
Total voltage = 100 + 16.65 = 116.65 V
I Same voltage across 10 Ω, therefore
5 × 10 10
W
116.65
Req = = Ω=
=I310.33 = 11.665 A
15 3 10
I Itotal = 11.665 + 5 = 16.665 A
4
V5 Ω = 16.665 × 5 = 83.325 V
Vs = 83.325 + 116.65 ≅ 200 V
iL

At t = 0+, we find I using superposition, Ans. (200)


2. (c) The equivalent circuit is
10 Ω
I

4 1Ω I
I3
1Ω 30 Ω
2A 2Ω 10 Ω
I2

4
Due to V = 4, I1 = =2A I1
2
Due to I = 2 A, I2 = −1
So, I=2−1=1A 100 V
Applying current loop method and KVL equation
64. Under steady state conditions, the inductor acts as for loop current I1, we get
a short circuit. −100 − 2(I1 − I3) − 20(I1 − I2) = 0 (i)
20 Ω 20 Ω KVL equation for loop current I2 is

I −30(I2 − I3) − 10I2 − 20(I2 − I1) = 0 (ii)


20 Ω Isc KVL equation for loop current I3 is
V
−10I3 − 30(I3 − I2) − 2(I3 − I4) = 0 (iii)
From Eqs. (i)−(iii), we have
Total impedance is given by −22I1 + 20I2 + 2I3 = 100 (iv)
+20I1 − 60I2 + 30I3 = 0 (v)
R eq = (20  20) + 20 = 10 + 20 = 30 W
+2I1 + 30I2 − 42I3 = 0 (vi)
V Applying Cramer’s rule, we get
Initial = A
30
−22 20 2
V 20 V  
Final = × = A ∆ =  20 −60 30 det ∆ = −16200
30 20 + 20 60  2 30 −42

It is given that steady state current is 2 A. Therefore, −22 100 2
 
For I2, ∆2 =  20 0 30 det ∆2 = 90, 000
 2 0 −42
V
= 2 ⇒ 120 V 
60
−22 20 100
 
Ans. (120) For I3, ∆3 =  20 −60 0 det ∆3 = 72, 000
 2 0
 30

Ch 1_Solved Examples & Practise exercises.indd 130 3/19/2016 4:01:18 PM


ANSWERS TO PRACTICE EXERCISES     131

det ∆2 90000 iL (0− ) = 1 A = iL (0+ )


I2 = = = −5.55 A
det ∆ −16200
VC (0− ) = 100 V = VC (0+ )
det ∆3 72000
I3 = = = −4.44 A (ii) At t ≥ 0,
det ∆ −16200
100 Ω
In 30 Ω resistance, I = (I2 − I3) − +
or |I1| = 5.55 − 4.44 = 1.11 A + 100 V

3. (b) Parallel combination of 5 Ω each on the left
1A L
+
side, then convert into voltage source as shown in C

the following figure.
V1 (iii) At t = 0+,
20 Ω 50 V
1 100 Ω
Vz I1 Iy
25 V 15 Ω iC(0+)
10 Ω
2.5 Ω 1A VL (0+) + 100 V

30 V
100 + VL (0+) − 100 = 0 Þ VL (0+) = 0 V
Applying nodal analysis at at node 1 and KCL at
diL (0+ ) di (0+ )
= 0⇒ L
1, we get
L = 0 A/s
V1 − 50 − 25 V1 V1 − 30 dt dt
+ + =0 dV (0+ )
22.5 10 15 We have iC (0+) = −1 A, C C = −1 A
dt
V1 − 75 V1 V1 − 30 +
dVC (0 )
+ + =0 = −1 × 106 V/s
22.5 10 15 dt
Ans. (0, −1 × 106)
10 V1 − 750 V1 V1 − 30 5. For the given circuit:
+ + =0
225 10 15
At node P:
10 V1 − 750 V1 V1 − 30 V − 10 VP − 0
+ + =0 +2 + P + = 0 ⇒ VP = 4.8 V
45 2 3 2 8
60 V1 − 4500 + 135 V1 + 90 V1 + 2700 = 0 At node Q:
VQ − 10 VQ − 0
7200 −2 + + = 0 ⇒ VQ = 10.8 V
285 V1 − 7200 = 0 ⇒ V1 = = 25.26 V 4 6
285
Therefore, VPQ = VP − VQ = −6 Volts
25.26 − 75
Current I1 = = −2.21 A Ans. (−6)
22.5
6. (b) We have I = 4 A ⇒ VS = 0 and need to find I
The actual correct magnitude of current = 2.21 A for VS = 16 V. In the following equivalent circuit
(+ve)
2Ω V + 2Ω −
Hence, Vz = 2.21 × 20 = 44.2 V. So,
+ − 4A
V 25.26 IS +
Iy = 1 =
10 10
= 2.526 A 2Ω

4. (i) At t = 0−, the circuit is IS = I1 + I2 = 4 + 4 = 8 A
100 Ω In the circuit with IS = 8 A,
+ −
100 V
+ + 2Ω− V + 2Ω −
iL (0−) VC (0−) 1A I
− + + 16 V
8A
2Ω −

Ch 1_Solved Examples & Practise exercises.indd 131 3/19/2016 4:01:47 PM


132     CHAPTER 1:  ELECTRIC CIRCUITS

Applying KVL, we have


= 52 + 102
V V − 16 = 11.8∠63.93
−8 + + =0
2 2
Total input current is,
V = 16 V i(t) = VY = 44.72  sin(3t − 63.4) Therefore,
Im = 44.72 A.
16
I= =8A
2 10. Current passing through 1 H and 2 Ω is the same,
V so the equivalent circuits are
7. (b) As Req = , we have
I 2Ω 2Ω 1H
V V − 2I
−I + + =0
4 2 5A 10 Ω 2H 2H

−4I + V + 2 V − 4I = 0
20 Ω 2Ω
I Volts ⇒ Req = W
8 8
⇒ 3 V = 8I ⇒ V =
3 3
Note: Whenever the dependent sources are pre-
5A 10 Ω 2H
sent in the network, nodal analysis is always used
to calculate the responses.
8. (a) Current Ix is distributed in the cube branches
symmetrically. Leq
t =
2 1
= = s
Ix Req 12 6
Ix A 3 Ix Ans. (1/6)
Ix Ix 6
Ix 3 11. (d) Applying KCL at node 2, we have
3
3 Ix e 4 t = e3 t + I
6 Ix Ix
6
Ix I = e 4 t − e3 t
Ix 6
3 3 Voltage V(t) across 4 H is
Ix
di
= 4(e4t − e3t ) = 4(4e4t − 3e3t )
6
Ix B Ix V (t) = L
3 dt
= 16e − 12e3t
I I 
=  x + x + x  ⋅ 4
I
Then,
 3
VAB 12. Applying mesh analysis
6 3
A
V 5 × 4 20 2Ω
RAB = AB = = Ω 10 Ω
I 6 6
9. (b) We know that 2 Ω I1 I3

∫ L
V (t) dV (t) I = V (t) 2Ω
IR = , IC = C , L
+

5V

R dt 10 V I2 B
Now, VS = 4sin3t
or wt = 3t ⇒ w = 3 rad/s
5Ω
By admittance method, conductance Considering the current loop mesh,

YR =
1
=5S 10 − 2I1 − 2(I1 − I3 ) − 5 = 0
R
1 −j −5I2 + 5 − 2 (I2 − I3) = 0
YL = = = −2 j
j wL 3× 1 −2(I3 − I1) − 2(I3 − I2) − 10I3 = 0
6 −4I1 + 2I3 = −5
YC = jw C = j3 × 4 = 12 j 0 − 7I2 + 2I3 = −5
Total admittance is Y total = 5 − 2 j + 12 j = 5 + 10 j 2I1 + 2I2 − 4I3 = 0

Ch 1_Solved Examples & Practise exercises.indd 132 3/19/2016 4:02:16 PM


ANSWERS TO PRACTICE EXERCISES     133

This can be represented in matrix form as log iL = −50t + k ⇒ iL = ke−50t


−4 0 2 −5
   
k = 10 A = iL(0)
 0 −7 2 =  5
 2
 2 −14  0 iL (t) = 10e−50 t (for t ≥ 0)

∆ = −4[98 − 7] + 2[14] = −376 + 28 = −348 Time constant =
1
s
−4 0 −5  50
 
or ∆3 =  0 −7 −5 = −4[10] − 5[+14]
 2 0 = −40 − 60 = −100
15. (d) (i) At t = 0+, we have
 2
1Ω
−100
So, I3 = = 0.287 A
−348 Ans. (0.287) +
13. (b) The circuit can be represented as 10 V VC (0+) iC (0+)

4Ω VA 3Ω
Ix
iL(0+) = 0 A, VC(0+) = 0V, VL(0+) = 0 V
+
10 V − 2Ω 4Ω +
− 4V
10
iC (0+ ) = = 10 A
1
VA − 10 VA VA − 4
+ + =0 (ii) At t → ∞, we have
4 2 3
3VA − 30 + 6VA + 4VA − 16 = 0 1Ω
46
13VA = 46 ⇒ VA = V
13 + +
Then 10 V VC (∞)
VA − 4 46 / 13 − 4 - iL (∞)
-
Ix = = = −0.153 A
3 3
14. This is a source-free dependent source circuit. 10
VL (∞) = 0 V, iC(∞) = 0 A, iL(∞) = = 10 A
V 10 Ω 1
16. (a) For the given circuit:
iL 20 Ω
i(0− ) =
10
When t < 0,
0.5 H
4 = 5 A = i(0+)
2
iL
10
Nodal equation is When t > 0, i(∞) = =2 A
−iL
2+3
V
+ + iL = 0
t =
L 1 1
4 20 = = s
Req 3 × 5 15
diL
0.5 + 10iL = V
dt i(t) = i(∞) + [i(0) − i(∞)]e−t /t = 2 + [5 − 2]e−15t

−iL iL 1 di = 2 + 3e−15t
+ + + iL = 0
4 2 40 dt
17. At t < 0,
5 1 diL
iL + =0 2Ω
4 40 dt
diL
+ 50iL = 0
dt +
10 V − V (0−) = 10 V = V (0+)

∫ = −∫ 50dt
diL
iL

Ch 1_Solved Examples & Practise exercises.indd 133 3/19/2016 4:02:46 PM


134     CHAPTER 1:  ELECTRIC CIRCUITS

At t > 0, IC = 1 − 2.5 = −1.5 A


2Ω 6Ω IC = C
dV
(given dV /dt)
dt
−1.5 = C(−20) ⇒ C = 0.075 F
+ −
+ 10 V − + 50 V
19. (a) At t = 0−,
10 V −

At steady state, capacitor will open.


2A 1/2 Ω VC = 1 V
2Ω 6Ω
+ − At t = 0+,
+
10 V − V + 50 V
− i
1/2 Ω + 1V Open
2A − circuit
Therefore, 50 + 10 = 8i ⇒ i = 15 / 2 A

15
or V + 2 × = 10, V = −5 V Applying superposition rule, we have
2

t = RC =
3 1 1
× = s 2A 1/2 Ω I1
2 3 2

V (t) = V(∞) + [V (0) − V (∞)]e−t /t

= −5 + (10 + 5) e−t /1/2


where I1 = 2 A (decreasing) and

= −5 + 15e−t2
V (0.5) = 0.5182 V
1/2 Ω
+ 1V

Ans. (0.5812)

18. (b) Applying nodal analysis at 1, we get


where I2 = 2 A (increasing)
5Ω 5V
Therefore, IC = I1 + I2 = 2 − 2 = 0 A
1A + Hence, VC = 1 V, IC = 0 A.
1.5 A
+ 2Ω
10 V − 20. At t = 0−, the circuit is
2.5 A C V = 5V

+
IC = C
dV 3 1Ω VC = 3 V
dt −
Current through 5 W
10 − 5 At t = 0+,
I5 W = A
5
I
Current through 2 W 1Ω 3V 2Ω
3
5
I2 W = = 2.5 A
2

Ch 1_Solved Examples & Practise exercises.indd 134 3/19/2016 4:03:07 PM


ANSWERS TO PRACTICE EXERCISES     135

Using superposition rule, we have 3Ω


+ -
10 V +
I1 +
VC(∞)
1Ω 2Ω
10 V iL(∞)
3 - - +
2Ω 0V
-
I1 = 0A.
10
VC (∞) = 0 V, iL (∞) = A
3
Energy stored in steady state is sum of energies
1Ω 2Ω stored in inductor and capacitor
3V
1 2 200
EL (∞) = Li (∞) = J
2 L 9
3
I2 = A
2 1
EC (∞) = CVC2 (∞) = 0 J
3 2
Therefore, I = I1 + I2 = A
200
Total energy is E = EL (∞) + EC (∞) =
2
Ans. (1.5) J
9
23. (b) According to current divider rule,
21. Applying superposition rule
ZA = 6 − 4 j = 7.21∠ − 33.6°
VV((tt))
II((tt)) =
= Z(t)
Z(t) ZB = 6 + 4 j = 7.21∠ + 33.6°
ZZ((jjww)) =
11
=11+ +22jj +
+j = =11++22jj−
−jj 10∠60° × 7.21∠ + 33.6 72.1∠93.6
j I1 = = = 6∠93.6
ZZ = 6 − 4j + 6 + 4j 12
=11+ + jj
10∠∠00 10 VA − B = I1ZA = 6∠93.6 × 7.21∠33.6° = 43.3∠60° V
10
= 2∠
10 − 45°
II((tt)) ==1+ j = ∠ − 45°
1+ j
24. (b)  (i) At t = 0−
2
10
10 sin(t − 45°) A
22 sin(t − 45°) A
IIss ((tt)) =
ss =
5 kΩ 20 kΩ

Note: No current due to 5 V DC is obtained in + − + −


iS 0V iC
steady state for DC w = 0. +
22. (b) (i) At t = 0+, +
VC(0−)
+
10 V −

3Ω
104 iS −
iC(0+)

10 V 2Ω VC(0+) At t = 0− steady state is reached.


10 − 5 × 103 iS(0−) − 104 iS(0−) = 0

iS (0− ) =
10 2
mA = mA
10 × 2 15 3
VL (0+ ) = = 4 V and iL (0+ ) = 0 A
5
VC (0− ) = 104 iS (0− ) =
20
10
+
V = VC (0+ )
iC (0 ) = = 2 A and VC(0+) = 0 V 3
5
(ii)  At t ≥ 0
   
(ii) At t → ∞, steady state is reached. So

Ch 1_Solved Examples & Practise exercises.indd 135 3/19/2016 4:03:41 PM


136     CHAPTER 1:  ELECTRIC CIRCUITS

20 kΩ 26. (i) At t = 0−
4Ω
iC
+
2 µF VC +
− 2A VC

Since iS = 0, for t ≥ 0 it is a source free circuit.

t
VC(0−) = 8 V

vC (t) =
20
e 20×103 ×2×10−6 (ii) At t = 0+
3
4Ω
20 −25t +
vC (t) = e
3
2A 8V 4 Ω V0
= 2 × 10−6 × e−25t (−25)
dvC (t) 20
iC (t) = C
dt 3

× 10−6 e−25t = − × 10−3 e−25t A
1000 1
=− V0 = 4 V
3 3
(iii) At t = ∞, C will be open circuit.

25. (i) At t = 0 4Ω

732 Ω
2A 4 Ω Vf
+
VC(0−) 50 V

Vf = 2 × 4 = 8 V
Ans. (4, 8)
− + −
VC(0 ) = 50 V = VC (0 ) 27. (i) At t = 0 , circuit will be
(ii) At t ≥ 0
2Ω
1A
+
800 Ω 800 Ω
+ iL = 1 A
2 µF VC − 50 V

2 µF iL(0−) = 1 A = iL(0+)

−6 −3 (ii) At t = 0+
VC(t) = 50 e−t/800 × 2 × 10 = 50 e−t/1.6 × 10
t = 1.6 ms
VC (2 ms) = 50 e−2/1.6 = 14.33 V 2Ω Short
1A
circuit
(iii) At t = 0+ 1A

VR + VL = 0
800 Ω + VR = 2 × 1 = 2 V
− 50 V                 VL = −2 V
i(0+) di
V =L L
dt
50
i(0+ ) = = 62.5 mA −2 diL
800 = = −1 A/s
2 dt
Ans. (50, 14.33, 62.5)   
Ans. (−1)

Ch 1_Solved Examples & Practise exercises.indd 136 3/19/2016 4:04:06 PM


ANSWERS TO PRACTICE EXERCISES     137

28. (c) We have Vi = 5 sin 203 t. So, Therefore,

Z = R − jXC Vx = Vi − VC = 10 − 10 = 0 V
j
=R− 31. (d) Given that: iL (0− ) = iL (0+ ) = 0; at t = 0+ , iL (0+ ) = 0
wC
 j  iL (0− ) = iL (0+ ) = 0; at t = 0+ , iL (0+ ) = 0
= R 1 −
j
=R− 
wC  wRC  IR= 2 A, so V(0+) = 200 × 2 = 400 V
 
= R 1 −
j
−3 
 = R(1 − j) dV (0+ )
 6 × 10 × 6 × 10 
3
For determining , we have
dt
or  Z = R 2 + R 2 ∠ tan−1(−1) = 2R∠ − 45° VL = L L
di
dt
3
V 5 sin 20 t diL V (0+ ) 400
  I = i = A = L = = 800 A/s
Z 2R∠ − 45° dt L 1/2
The output voltage across capacitance is Also,
2 = iR + iL
5 sin 203 t 1
Vo = I XC = × diR diL
2 R∠ − 45° C × 6 × 10+3 0= +
dt dt
diR diL
=
5
sin(203 t + 45°) =− = −800 A/s
2 dt dt

Thus output voltage leads input voltage. We know that V = iR, so

29. (b) We have dVL (0+) diR (0+)


−6 −3
= × R = (−800) × 200 = −160, 000 V
R1C1 = R2C2 = 2 × 10 × 6 × 10 3
= 12 × 10 ms dt dt

Y (s) =
1
+ Cs =
1 + sRC 32. The time constant t = 0.005 s, L = 5 mH. So,
R R
R 5 × 10−3
=1 W
L
Z(s) = = 0.005 ⇒ R =
1 + sRC   R 0.005
V2 (s)  R2   R1
 R2  R
V − t
R2
=  = V (s) V (s)
V1(s) 1 + sRC  1 + sRC 1 + sRC  R + R
+ I (s) = = = e L
1 2 Rs + Ls L[(R/L) + s] L
R2
Therefore, Vo (t) = V (t)
R1 + R2 i In s-domain, put s = 0, so

5 × 10−3
R
6 × 103 −t
× 20 = 10 u(t) I (t) = e L
5 × 10−3
or     Vo (t) =
12 × 103
30. (c) From the given circuit V
I (s) = = 1.0 A
VC (0− ) = VC (0+ ) = 0 Volts
L
  Ans. (1)
VS (0+ ) = 10 Volts
V x = V i − VC
33. (b) Equivalent capacitance can be obtained as

VC (∞) = 10 Volts 20 kΩ a b

VC (t) = 10 − 10e−t /RC


+
At t = 5 seconds 100 V − 10 kΩ
Ceq
VC(5s) = 10 − 10e−5000 = 10 Volts

Ch 1_Solved Examples & Practise exercises.indd 137 3/19/2016 4:04:40 PM


138     CHAPTER 1:  ELECTRIC CIRCUITS

(0.4 + 0.1) × 0.5 35. The output circuit is


Ceq = = 0.25 mF
0.4 + 0.1 + 0.5 4Ω 4Ω 3Ω
−6 −3
t = RC = 0.25 × 10
+ +
× 10 × 10 = 2.5 × 10
3
s Ra
VC (0− ) = VC (0+ ) = 100 V V1 2Ω Rb 2 V2
VC (t) = VR (t) Rc
IR (t) × R = VC (t) − −
−3
e−t /2.5×10 = 10e−400 t mA
100 Converting Δ to Wye
IR (t) = i(t)
10 × 10 3
4Ω Ra Rb 3
34. (c) Applying ∆ to star conversion on the upper + +
delta, we get
V1 Rc V2
10∠ − 90° × 10∠90° 100
ZA = =
8 + 10 j − 10 j 8 − −
  
=1W
8
Ra =
10∠90°× 8
ZB = = 10∠90° = 10 j 8
8
Rb = = 1 W
     8
8
10∠ − 90° × 8
ZC = = 10∠ − 90° = −10 j Rc = = W
4 1
8 8 2
ZB is in series with 6j Ω and ZC in series with 6j Ω
5Ω 4Ω
+ +
ZA
V1 1/2 Ω V2

ZC ZB − −
 11 1 
6Ω   5 + 1 / 2
6 jΩ 5 + 1 / 2 1 / 2   2 2 1/2 
Z= = Z= 
 1 / 2 4 + 1 / 2  1 9   1 / 2 4 + 1 / 2
 2 2
 
Total input impedance
Z11 = W Z12 = W
11 1
Z11 = W Z12 = W
11 1
(ZB + 6 j)(ZC + 6) 2 2 2 2
( 2 , 2) Ans. ( 2 , 2 )
Z = ZA +
(ZB + 6 j + ZC + 6)
11 1 11 1

(16 j)(−10 j + 6)
36. (b)
= 12.5 +
16 j − 10 j + 6 V2
Z21 =
V1
(16∠90°)(11.66∠ − 59.0)
I2 = 0
= 12.5 +
6j + 6
It is a reciprocal network, so
186.56∠31
= 12.5 + 22∠ − 14
Z12 = Z21
= 12.5 +
8.48∠45 Y = Z−1
−1
= 12.5 + 22(0.97 − j0.24)  Y11 Y12   Z11 Z12 
 = 
 Y21 Y22  Z21 Z22 
= 12.5 + 21.34 − 5.32j
1  Z22 −Z12 
= 33.84 − 5.32 j = 34.25∠ − 8.93° =  
[Z11Z22 − Z12Z21 ] −Z21 Z11 
20∠0° −Z21
Current I = = 0.58∠8.93° A
34.25∠ − 8.93° Y21 =
DZ

Ch 1_Solved Examples & Practise exercises.indd 138 3/19/2016 4:05:20 PM


ANSWERS TO PRACTICE EXERCISES     139

37. The transmission parameters for the network are 3Ω 6Ω


a a
 A B  1 2
 =  3Ω 6Ω ⇒ 4.8 V + 6Ω
C D  1 3 −
1.6 A

AD − BC = 1, so it is reciprocal network b b
From the figure, a
V2 + 10I2 = 0

V2 = −10I2 ⇒ 0.8 A 3Ω

I2
= −0.1  b
V2

Isc= 0.8 A
V Ans. (0.8)
Zin = 1
I1 40. (a) As voltage across 5 Ω is 30 V, means current
through it is 5 × I = 30 ⇒ I = 6 A
V1 = V2 − 2I2
This current through immediate parallel branch of
I1 = V2 − 3I2 6 Ω and 4 Ω is given by

V1 V − 2I 2 1 − 2I2 /V2 1 + 2/10 6×4


= 12/13 W
I6 W = = 2.4 A
= 2 = =
V 2 − 3I 2 1 − 3I2 /V2
10
I1 1 + 3/10
6×6
Ans. (12/13) I 4W = = 3.6 A
10
38. (c) Given that R1 = R2 = R4 = 2R, R3 = 4R Then this further goes to parallel branch of 6 Ω
The circuit is and 10 Ω.
6×6
I10 W = = 2.25 A connected between a and b.
2Ω 2Ω 16
So, it can be replaced by voltage source of 22.5 V
with positive terminal downward.
+ A V B
10 V − 41. Applying nodal analysis
I2
V − 16 V
+ −8 = 0
2Ω 4Ω 2 2
I1 V = 16 V
So the circuit is
10 = 4I1 − 4I2
2Ω 2Ω
0 = 10I2 − 4I1
+ −
25 5 16 V 8 A 0A
I1 = and I2 = +
6 3
2Ω + 16 V

32 V 8A
Voltmeter = VA — VB

VA = 10 − 2(I1 − I2 ) = 5 V
VB = 10 − 2I2 = 6.667 V P8 A = 32 × 8 = 256 W (delivered)
VA − VB = 1.6667 V P2 Ω = 16 × 8 = 128 W (absorbed)
P2 Ω = 16 × 8 = 128 W (absorbed)
39. The resistance of an ideal ammeter = 0
The resistance of an ideal voltmeter = ∞ P16 V = 0 × 16 = 0 W
The resistance of an ideal current source = ∞ Ans. (0)
The resistance of an ideal voltage source = 0 42. (b) Applying nodal analysis to the following circuit,
From the equivalent circuits, we have we have

Ch 1_Solved Examples & Practise exercises.indd 139 3/19/2016 4:05:42 PM


140     CHAPTER 1:  ELECTRIC CIRCUITS

1.2 A V1 3Ω V V

=
+ + 3.2 A Z 2R
2V 1Ω

+
V1 2 Ω + Z = 2R
5 V1
− −
pf = cos f =
1
= 0.707 (lead)(Capacitive)
2
Note: At f = fH, pf = cos f = 0.707 (lag) (Inductive)
V1 V − 5V1
−2+ 1 =0 2 4
2 3 ∫ 102 ⋅ 10 dt + ∫ (−10)2 ⋅ 10 dt
V1 = −2.4 V 45. Pavg = 0 2
4 ms
I5V1 = 3.2 A
(1000 × 2 + 1000 × 2)ms
P5V1 = 5 × (−2.4) × 3.2 = −38.4 W =
4 ms
Therefore, 38.4 W power delivered. 1000 × 4 = 1000 W
=
1 1 4
43. Y = + Ans. (1000)
ZC ZL 46. (c) We have:

= jwC + XL = 2 × 3.14 × 50 × 0.5 = 157 W


1

30(cos 40° + j sin 40°)
= 63.65 W
1
XC =
2 × 3.14 × 50 × 50 × 10−6
= jwC +
1
(cos 40 − j sin 40°)

Z = R + j(XL − XC ) = 40 + j (157 − 63.65)
30

 sin 40 
cos 40° + j wC −
1 = 40 + j93.31

30 


=

30 220∠0°
I=
 93.31
402 + 93.312 ∠ tan−1 
Equating j terms = 0

 40 
sin 40°
wC = 220∠0°
30 = = 2.16∠ − 66.79 A
101.52∠66.75
sin 40°
C= = 68.1 mF Voltage (rms) across capacitor = 2.16 × 63.69
30 × 2 × p × 50
Ans. (68.1) = 138.02 V

44. (b) Power factor cos f =


R 47. (c) Convert current sources to voltage sources and
Z the given circuit modifies to the following simple
XC series circuit current
Z 4Ω 8Ω 10 Ω
Capacitive
f
− +
R 20 V 100 V
+ −
Z = R + jX
R
f 120
I=
= 5.45 A
Inductive 22
Z Hence, the power dissipated in 10 Ω resistor is
XL
P10 W = (5.45)2 × 10 = 297.5 W
Z = R + jX
48. Let Ix be the current in the series circuit. Applying
At f = fL KVL in the closed loop, we get
Im
I= 20 − 5Ix − 2Vx − 5Ix = 0 (i)
2

Ch 1_Solved Examples & Practise exercises.indd 140 3/19/2016 4:06:14 PM


ANSWERS TO PRACTICE EXERCISES     141

From the circuit, we have Vx = 5Ix. Putting Vx 5V1 + 15 = 0


into Eq. (i), we get −15
V1 = = −3 V
20 − 5Ix − 10Ix − 5Ix = 0 5
Therefore, Ix= 1 A and hence Vx = 5 × 1 = 5 V. −3
Hence, e0 = × 5 = −3 V
Ans. (5) 5
Ans. (−3)
49. (b) Apply nodal analysis KVL equation at node 1
52. (b) Applying current loop method to:
V1 − 10
−4 + + 3 Vx = 0 Loop 1 fadce with current I1 in clockwise KVL
3
equation, we get
−12 + V1 − 10 + 9Vx = 0 (i)
50 − 30 (I1 − I2) − 50(I1 − I3) = 0 (i)
V − 10
From the circuit, Vx = 3i1 we have i1 = 1 Loop 2 abda with current I2 clockwise, we get
3
Now, Vx = V1 − 10. Putting into Eq. (i), we get −30(I2 − I1) − 40I2 − 10(I2 − I3) = 0 (ii)
−12 + V1 − 10 + 9(V1 − 10) = 0 Loop 3 bcdb with current I3 clockwise, we get
−12 + V1 − 10 + 9V1 − 90 = 0
−10(I3 − I2) − 60I3 − 50(I3 − I1) = 0 (iii)
So, 10 V1 = 112 or V1 = 11.2 V.
From Eqs. (i), (ii) and (ii), we get
112 − 10 
V x =   × 3 = 1.2 V − 80I1 + 30I2 + 50I3 = −50 (iv)
 3 
+ 30I1 − 80I2 + 10I3 = 0
Hence, (v)
So, the value of dependent current source + 50I1 + 10I2 − 120I3 = 0 (vi)
= 3 × 1.2 = 3.6 V.
By dividing by 10 factor, we get
50. (c) Applying current loop method, we get
−8 3 5
100 − 6I1 − 10(I1 − 10)  
= 0 ∆ =  3 −8 1 , where det ∆ = −422
−6I1 − 10I1 + 100 −100  5 1 −2

=
16I1 = 200
For I2,
I1 = 12.5
−8 −5 5
Due to parallel branch, 10 Ω and R  
∆2 =  3 0 1 , where det ∆2 = 205
10 × (I1 − I2) = I2 × R  5 0 −12
    
10(12.5 − 10) = 10R For I3,
R = 2.5 W −8 3 −5 
 
51. Convert current sources into voltage sources as ∆3 =  3 −8 0 , where det ∆3 = −215
 5 0
shown in the following figure.      1

5Ω V1 1 5Ω det ∆2 205 det ∆3


So, I2 = = = 0.48 A and I3 = = 0.5 A
det ∆ −422 det ∆
5Ω det ∆3
5Ω I3 = = 0.5 A
20 V det ∆
25 V

Hence, I3 > I2 in magnitude. So, I3 − I2 flowing


Applying nodal analysis and KCL at node 1, we get through 10 Ω branch
V1 − 25 V1 V1 + 20
+ + =0 I10 Ω = 0.5 + 0.48 = 0.98 A
10 5 5

V1 − 25 + 2 V1 + 2 V1 + 40 Power delivered across 10 Ω resistance


=0
10 = (0.98)2 × 10 = 9.60 W

Ch 1_Solved Examples & Practise exercises.indd 141 3/19/2016 4:06:31 PM


142        CHAPTER 1:  ELECTRIC CIRCUITS

SOLVED GATE PREVIOUS YEARS’ QUESTIONS

1. Figure given below shows the waveform of the 2. A segment of a circuit is shown in the figure given
­current passing through an inductor of resistance below. vR = 5 V, vC = 4sin2t. The voltage v L is
1 Ω and inductance 2 H. The energy absorbed by given by
the inductor in the first four seconds is
Q
+
6A 5 Ω vR

1A 1F
+ −
P R
t
0 2s 4s 2A + vC
2 H vL
(a)  144 J (b)  98 J −
(c)  132 J (d)  168 J S
(GATE 2003: 1 Mark) (a)  3 - 8cos2t (b)  32sin2t
(c)  16sin2t (d)  16cos2t
Solution:  Power is the rate of change of energy. (GATE 2003: 1 Mark)
Therefore,
t Solution:  Applying Kirchhoff’s current law, sum of
E = ∫ P dt incoming currents is equal to sum of outgoing ­currents.
0
We know that Q

æ dI ö æ dI ö +
P = VI = I × ç L ÷ ç as VL = L ÷ 5 Ω vR
è dt ø è dt ø −
1 A iC 1 F
+ −
P iL R
 dI 
t 2A + vC
E = ∫ LI   ⋅ dt
 dt  2 H vL

0

ìdI /dt = 3 0 £ t < 2 S


Given that í
î = 0 2<t<4 i L = iC + 1 + 2 Þ iL = i C + 3
On ­substituting the limits, we have We know that
dv
iC = −C C = −1 × (4 sin 2t) = −8 cos 2t
4 2 4 2 d
E = ∫ 2I ⋅ dt = 2∫ 3I ⋅ dt +2∫ 0 ⋅ I ⋅ dt = 6 ⋅ ∫ I ⋅ dt
dI
dt dt
0
dt 0 0 0
= 6 × area under the current curve Therefore, iL = −8 cos 2t + 3
1
= 6 × × 2 × 6 = 36 J vL = L
di L d
= 2 × (−8 cos 2t + 3)
2 dt dt
= 2[8 × sin 2t × 2 + 0 ] = 32 sin 2t
ìI = 3t 0£t£2
Given that í
î =6 2<t<4 Ans. (b)
For resistor, 3. In the figure given below, Z1 = 10∠ − 60°, Z2 = 10∠60°, Z
t Z1 = 10∠ − 60°, Z2 = 10∠60°, Z3 = 50∠53.13°. Thevenin imped-
E = ∫ I 2 ⋅ R dt [as P = VI = I ⋅ IR = I 2R ] ance seen from X-Y is
0
Z1 Z3 X
 t3 
2 4 2 +
= ∫ (3t)2 ⋅ dt + ∫ 62 ⋅ dt = 9 ⋅   + 36[t ]24 100∠0° Z2
0 2  3  0
Y
8
= 9 × + 36 × 2 = 24 + 72 = 96 J
3 (a)  56.6Ð45° (b)  60Ð30°
Therefore, total energy = 36 + 96 = 132 J. (c)  70Ð30° (d)  34.4Ð65°
Ans. (c) (GATE 2003: 1 Mark)

Chapter 1 Solved Question Paper.indd 142 3/19/2016 4:26:55 PM


SOLVED GATE PREVIOUS YEARS’ QUESTIONS        143

Solution:  Given that 2A


Z1 = 10∠ − 60°, Z2 = 10∠60°, Z3 = 50∠53.13°

2Ω 4Ω
Z1 Z3
P Q
Source ZTh +
sc Z2
10 V

To obtain Thevenin impedance, short circuit the 8Ω 6Ω


source. Therefore,
ZTh = Z3 + (Z1 Z2 ) (a)  12 V (b)  10 V
(c)  -6 V (d)  8 V
Convert polar co-ordinates into rectangular (GATE 2003: 2 Marks)
co-ordinates.

Z1 = 5(1 − 3 j) Z2 = 5(1 + 3 j) Z3 = 10(3 + 4 j). Solution:  At node P,


 5(1 − 3 j) × 5(1 + 3 j)  VP − 10 VP
Zth = 10(3 + 4 j) +   2+ + =0
 5(1 − 3 j) + 5(1 + 3 j)  2 8
16 + 4VP − 40 + VP = 0
= 40 + 40j
5VP − 24 = 0 Þ VP =
24
ZTh in polar co-ordinates = 40 2 × Ð45° W 5
Ans. (a)
4. In the circuit of figure given below, the magnitudes At node Q,
of VL and VC are twice that of VR. The inductance VQ − 10 VQ − 0
of the coil is 2= +
4 6
VR 6 VQ − 60 + 4VQ
2=
5Ω
24
VC
48 = 10 VQ − 60 Þ 24 = 5 VQ − 30 Þ VQ =
C 54
5
5∠0°
Therefore, the potential difference between P and
L VL Q is VP - VQ
24 54
VP − VQ = − = −6V
(a)  2.14 mH (b)  5.30 H 5 5
Ans. (c)
(c)  31.8 mH (d)  1.32 H
6. Two AC sources feed a common variable resistive
(GATE 2003: 2 Marks)
load as shown in the figure given below. Under
the maximum power transfer condition, the power
Solution:  Given that for resonance
absorbed by the load resistance RL is
VVLL =
= VVCC =
=22VVRR
Voltage across inductor VL = IR × jw L.
6Ω j8 Ω 6Ω j8 Ω
At resonance, IR = 5Ð 0° = = 1 A.
5
5
RL
VL = wL = 2VR where w L = 2 × 5 = 10
110∠0° 90∠0°
2p f × L = 10 (where w = 2p f)
2 × p × 50 × L = 10 Þ L = 31.8 mH
Ans. (c) (a)  2200 W (b)  1250 W
(c)  1000 W (d)  625 W
5. In the figure given below, the potential difference
between points P and Q is (GATE 2003: 2 Marks)

Chapter 1 Solved Question Paper.indd 143 3/19/2016 4:27:14 PM


144        CHAPTER 1:  ELECTRIC CIRCUITS

Solution:  For maximum power transfer, Solution:  The equivalent circuit is


RL = ZTh and P = I 2RL RΩ
To find the Thevenin impedance, short circuit the
5A
14 Ω 1Ω
voltage source
V
6Ω 8j Ω 6Ω 8j Ω
5A 10 A
10 A 5A

+
100 V − 2Ω +
− 40 V

ZTh = (6 + 8 jW) (6 + 8 jW) Applying nodal analysis at V:


(6 + 8 jW) × (6 + 8 jW) V − 100 V V − 40
= = + + =0
(6 + 8 jW) + (6 + 8 jW) 14 2 1
= 3 + 4 jW V − 100 + 7V + 14V − 560 = 0
660
Therefore, RL = 32 + 42 = 9 + 16 = 25 = 5 W. V = = 30 V
22
Current through 14 Ω and 1 Ω resistors
To find the Thevenin’s voltage, open the load:
6Ω 8j Ω 6Ω 8j Ω 100 − 30 70
I2 = = =5A
14 14
40 − 30
+ I1 = = 10 A
1
VTh
− Therefore, current through R is 5 A (as 10 =5+ 5)
110∠0° 90∠0°
100 − 40
So,
=5
R
By nodal analysis method,
VTh − 110Ð 0° V Th − 90Ð 0°
= 12 Ω
60
+ =0 R=
6 + 8j 6 + 8j 5
VTh − 100Ð 0° V Ans. (d)
Therefore, 8. In the circuit shown in the figure given below, the
2
switch S is closed at time t = 0. The voltage across
VTh the inductance at t = 0+, is
× RL =
100
P = ×5
RTh + RL (3 + 4 j) + 5 3Ω
(100) × 5
2 S
= = 625 W
80 4F
Ans. (d) +
10 V − 4Ω 4H
7. In the given figure, the value of R is 4Ω

RΩ
14 Ω 1Ω (a)  2 V (b)  4 V
(c)  -6 V (d)  8 V
10 A 5A
(GATE 2003: 2 Marks)
+
100 V − 2Ω +
− 40 V
Solution:
At t < 0; VC (0− ) = 0; iL (0− ) = 0.
At t = 0; VC (0) = 0; iL (0) = 0.
(a)  10 Ω (b)  18 Ω
(c)  24 Ω (d)  12 Ω At t > 0; VC (0+ ) = 0; iL (0+ ) = 0.
(GATE 2003: 2 Marks) Therefore, to find voltage across inductor,

Chapter 1 Solved Question Paper.indd 144 3/19/2016 4:27:31 PM


SOLVED GATE PREVIOUS YEARS’ QUESTIONS        145

3Ω 5EA − 2E1 − 2E2 = 0  (i)

Similarly at node 1
E1 − EA E1
+ = 0 2E1 − EA = 0 Þ EA = 2E1
10 V 4Ω VL (0+) 2 2
4Ω E1 − EA E1
+ = 0 2E1 − EA = 0 Þ EA = 2E1 (ii)
2 2
Solving Eqs. (ii) and (i), we get
10 æ 4×4 ö 10
VL (0+ ) = ×ç ÷ = ×2 = 4 V 5(2E1 ) − 2E1 − 2E2 = 0
æ 4×4 ö è 4 + 4 ø 3 + 2
3+ç ÷
è4+4ø
E1
8E1 = 2E2 Þ
2 1
= =
Ans. (b) E2 8 4
9. The h-parameters for a two-port network are
Alternately:
E1   h h 12  I1 
defined by   =  11
h 22  E 
 I2   21  2
h Equivalent resistance as seen from E2
For the two-port network shown in the figure given (2 Ω + 2 Ω) || 4 Ω + 2 Ω = 4 Ω
below, the value of h12 is given by E
Since, I2 = 2
4Ω 2Ω 2Ω 4
I E E
I1 I2 Thus, E1 = 2 × 2= 2 × 2 = 2
2 8 4
1
Hence, h12 = = 0.25
E1 2Ω 4Ω E2 4
Ans. (d)
10. The value of Z in the figure given below which
is most appropriate to cause parallel resonance at
500 Hz is
(a)  0.125 (b)  0.167
(c)  0.625 (d)  0.25
(GATE 2003: 2 Marks) 5Ω

2H Z
Solution:  Given
E1  h11 h12   I1  E = I1h11 + E2h12
 I  = h ⇒ 1
 2   21 h22  E2  I2 = I1h21 + E2h22
(a)  125.00 mH (b)  304.20 mF
h12
E
= 1 (c)  2.0 mF (d)  0.05 mF
E2
(GATE 2004: 1 Mark)
as for an open circuit I1 = 0.
E1 2Ω EA E2 Solution:  When Z is capacitive, resonance occurs.
+ + In case of parallel resonance, susceptance of circuit
1 A
should be zero.

E1 2Ω 4Ω E2
+ jwC = 0
1
jwL
1 + j2w 2 LC
= 0 Þ 1 − w 2 LC = 0 Þ C = 2
1
− − jwL w L
Applying KVL at node A,
EA − E1 EA − E2 EA 1
+ + =0 We know that resonant frequency, w =
2 2 4 LC

Chapter 1 Solved Question Paper.indd 145 3/19/2016 4:27:46 PM


146        CHAPTER 1:  ELECTRIC CIRCUITS

Therefore, (a)  12 V (b)  24 V


1 1 (c)  30 V (d)  44 V
C= = = 0.05 µF
(2p × 500) × 2 2 4p × 2500 × 2 (GATE 2004: 2 Marks)

Ans. (d) Solution:  The equivalent circuit is


11. The rms value of the periodic waveform given in
1 A 10 Ω 1 6Ω P
the following figure is
2A
6Ω
6A
1A E
t
T/2 T

−6 A Q
At node 1, V1 = (2 + 1) × 6 = 18 V
(a)  2 6A (b)  6 2A
By nodal analysis,
(c)  4/3A (d)  1.5 A
E − V1
(GATE 2004: 2 Marks) =2
6
Solution:  The rms value of periodic waveform is E − 18 = 12 ⇒ E = 30 V
Ans. (c)
∫0
1 T
I rms = I 2 (t) dt
13. In the figure given below, Ra, Rb and Rc are 20 Ω,
T

From the graph, 10 Ω and 10 Ω, respectively. The resistances R1, R2


and R3 in Ω of an equivalent star-connection are
ì æ 12 ö
0£t<
T
ï−
I (t) = í çè T ÷ø
t,
2 a
ïî6, T/2 < t £ T
So,
a
R1
 T /2  12t  2 
∫  dt + ∫T /2 (6) dt 
1 T
 −
2
I rms =
T  0 T  Rb Rc
R3 R2
 
1 144  t3 
T /2
=    + 36[ t ]T /2 
T
T  T 2  3  
 0
c Ra b c b

1  144  T 3   T 
(a)  2.5, 5, 5 (b)  5, 2.5, 5
=  2   + 36   
 2 
(c)  5, 5, 2.5 (d)  2.5, 5, 2.5
T  T  24  
(GATE 2004: 2 Marks)
1
= [6 T + 18T ] = 24 = 2 6 A
Solution:  Given that Ra = 20 W, Rb = Rc = 10 W.
T Ans. (a)
12. In the figure given below, the value of the source Using Δ to wye conversion,
voltage is
Rb Rc 10 × 10
= 2.5 W
P 2A 100
R1 = = =
Ra + Rb + Rc 20 + 10 + 10 40
10 Ω 6Ω
Ra Rc 20 × 10
=5 W
200
R2 = = =
6Ω
+ Ra + Rb + Rc 40 40
1A − E
Ra Rb 20 × 10
R3 = = =5 W
Ra + Rb + Rc 40
Q Ans. (a)

Chapter 1 Solved Question Paper.indd 146 3/19/2016 4:28:02 PM


SOLVED GATE PREVIOUS YEARS’ QUESTIONS        147

14. In the figure given below, the admittance values At node V1,
of the elements in Siemens are YR = 0.5 + 0j,
YL = 0 - 1.5 j, YC = 0 + 0.3 j, respectively. The value
100
× (1011R)
of I as a phasor when the voltage E across the ele- 10 + (1011R)
ments is 10∠10° V is 100 æ 10R ö 50R
V1 = ×ç ÷ =  (ii)
æ 10R ö è 10 + R ø 5 + R
10 + ç ÷
è 10 + R ø
I YA YL YC E = 10 0° V Therefore, from Eq. (i)
50R
R = 5+R Þ
50
=2
(a)  1.5 + 0.5 j (b)  5 - 18 j 2 5+R
(c)  0.5 + 1.8 j (d)  5 - 12j  50 = 10 + 2R
(GATE 2004: 2 Marks) Þ R = 20 W
Alternately:
Solution:  We have At node V1:
V1 V1 − 100
Þ I = E × Y eq
E 1
I= , Zeq = + + 2 = 0 ⇒ V1 = 80 V
Zeq Y eq 10 10
40 V
Y eq = YR + Y L + YC R= = 20 Ω
2A
= (0.5 + 0 j) + (0 − 1.5 j) + (0 + 0.3 j)
= 0. 5 − 1. 2 j Ans. (b)
Þ I = 10(0.5 − 1.2 j) = 5 − 12 j 16. In the figure given below, the capacitor initially has
a charge of 10 Coulombs. The current in the circuit
Ans. (d) one second after the switch S is closed will be
15. In the figure given below, the value of resistance R
in Ω is S
2Ω
V1 +
+
10 Ω 2A
100 V − −
0.5 F

+ 10 Ω R
100 V −
(a)  14.7 A (b)  18.5 A
(c)  40.0 A (d)  50.0 A
(a) 10 (b) 20 (c) 30 (d) 40 (GATE 2004: 2 Marks)
(GATE 2004: 2 Marks)
Solution:  Let VC(0) is the initial voltage across
Solution:  Current through R Ω resistor = 2 A. capacitor, Q0 is the corresponding charge at VC and
VC (∞) is the voltage when switch is closed.
V1 V
I= ⇒ 2 = 1 (i) Q0 = CVC(0)
R R
10 = 0.5VC(0) ⇒ VC(0) = 20 V
10 V1 2
VC(∞) = 100 V
The transient response,

100 V 10 R VC (t) = VC (¥) + [VC (0) − VC (¥)]e−t/RC


= 100 + (20 − 100)e−t/2×0.5
= 100 − 80 × e−t/1

= C × [100 − 80e−t ]
dVC (t)
iC (t) = C ×
d
dt dt
= 0.5[+80e−t ] = 40e−t

Chapter 1 Solved Question Paper.indd 147 3/19/2016 4:28:15 PM


VC (t) = VC (¥) + [VC (0) − VC (¥)]e−t/RC
−t/2×0.5
(20 − 100)e
= 100 +CIRCUITS
148        CHAPTER 1:  ELECTRIC
= 100 − 80 × e−t/1
(a)  2.5 Ω (b)  5.0 Ω
= C × [100 − 80e−t ]
dVC (t)
iC (t) = C ×
d
(c)  7.5 Ω (d)  10.0 Ω
dt dt
= 0.5[+80e−t ] = 40e−t (GATE 2005: 1 Mark)

At t = 1 s, i (t) = 40e−1 = 14.71 A Solution:  Current I =


V
Ans. (a) Sum of resistances (Req )
17. The rms value of the resultant current in a wire 100
I=
which carries a DC current of 10 A and a sinusoi-  10 × 10  
R+
 10 + 10 
dal alternating current of peak value 20 A is (i)

(a)  14.1 A (b)  17.3 A


(c)  22.4 A (d)  30.0 A Given that I = 8. Substituting in Eq. (i), we get

(GATE 2004: 2 Marks) 100
8=
20A + 100
Solution:  Given that IDC = 10 A, IAC = 20sinwt, 20
I = 10 + 20sinwt
Þ 100 = 8R + 40 ÞR= = 7.5 W
100 60
2 8=
(20) R+5 8
I rms = (10)2 + = 100 + 200 = 300 = 17.32 A
2 Ans. (c)
Ans. (b) 20. RMS value of the voltage u(t) = 3 + 4cos(3t) is
18. The Z matrix of a 2-port network as given by (a)  17 V (b)  5 V
0.9 0.2 (c)  7 V (d)  (3 + 2 2 )V
0.2 0.6 (GATE 2005: 1 Mark)
The element Y22 of the corresponding Y matrix of
the same network is given by Solution:  We have u(t) = 3 + 4cos3t
(a)  1.2 (b)  0.4
42
(c)  -0.4 (d)  1.8 urms = 32 + = 9 + 8 = 17 V
2
(GATE 2004: 2 Marks) Ans. (a)
21. For the two-port network shown in the figure given
Solution:  Give that below, the Z-matrix is given by
0.9 0.2
Z=
0.2 0.6 i1 Z2 i2

Y-parameter conversion,

 Y11 Y12  0.9 0.2−1 v1 Z1 v2


Y Y22  0.2 0.6
=
 21
1  0. 6 − 0. 2 
0.5 −0.2 0.9 
=
 Z1 Z1 + Z2 
(a)  
Z2 
1
Y22 = 0.9 × = 1.8  1
Z + Z2
0.5
Ans. (d)  Z1 Z1 
(b)  
19. In the figure given below, the value of R is Z1 + Z2 Z2 

Z Z2 
8A R
(c)   1
Z2 Z1 + Z2 

Z Z1 
10 Ω 10 Ω (d)   1
Z1 + Z2 
100 V
Z2

(GATE 2005: 1 Mark)

Chapter 1 Solved Question Paper.indd 148 3/19/2016 4:28:35 PM


SOLVED GATE PREVIOUS YEARS’ QUESTIONS        149

Solution:  Apply Kirchhoff’s voltage law, we have, 23. For the three-phase circuit shown in the figure
Loop 1: v1 − (i1 + i2)Z1 = 0 given below ratio of the currents IR : IY : IB is
given by
v1 = Z1i1 + Z1i2  (i) IR
R
Loop 2: v2 − i2Z2 − (i 1+ i 2)Z1 = 0
R1
v2 = Z1i 1+ (Z1 + Z2 )i 2  (ii) Balanced IB
Z Z1  three-phase voltage B
Therefore, Z-matrix, Z =  1
Z1 Z1 + Z2  source R1

IY
Ans. (d) Y
22. The RL circuit of the figure is fed from a constant
magnitude, variable frequency sinusoidal voltage (a)  1 : 1 : 3 (b)  1:1:2
source vin . At 100 Hz, the R and L elements each
3
have a voltage drop urms. If the frequency of the (c)  1:1:0 (d)  1 : 1 :
source is changed to 50 Hz, then new voltage drop 2
across R is (GATE 2005: 2 Marks)
R Solution:  We have
IB = IR Ð0° + I Y Ð20°

æ 120° ö
L
IB2 = IR2 + I Y
2
+ 2IR I Y cos ç ÷
è 2 ø
+ 2 × IR I Y ×
1
= IR2 + I Y
2
5 2 2
(a)  u (b)  u
8 rms 3 rms IB2 = IR2 + IR2 + IR2 = 3IR2 [as IR = I Y ]

(c) 
8
u (d) 
3
u IB = 3IR2 = 3 × IR = 3 × I Y
5 rms 2 rms
IR : I Y : IB = 1 : 1 : 1 3
(GATE 2005: 2 Marks)
Ans. (a)
Solution:  At f = 100 Hz, 24. For the triangular wave form shown in the figure
given below, rms value of the ­voltage is equal to
Vin × R V ( jw 1L)
urms = = in V(t)
R + jw 1L R + jw 1L
1
(drop across R and L).
At f = 50 Hz, R = w1L.

Vin × R T/2 T 3T/2 2T t


¢ =
urms
R + jw 2 L (a) 
1
(b) 
1
6 3
urms R + jw 2 L R 2 + w 22 L2 w12 L2 + w 22 L2
= = =
¢ R + jw1L
1 2
urms R 2 + w12 L2 w12 L2 + w12 L2 (c)  (d) 
3 3
L2 (w12 + w 22 ) f12 + f22 1002 + 502
= = =
L2 (w12 + w12 )
(GATE 2005: 2 Marks)
2f12 2(100)2
5
=  2  T
8   T  t, 0 ≤ t ≤ 2
¢ = 8/5 urms
urms Solution:  Given that V (t) = 
0, T
<t≤ T
Ans. (c)  2

Chapter 1 Solved Question Paper.indd 149 3/19/2016 4:28:58 PM


150        CHAPTER 1:  ELECTRIC CIRCUITS

So, By nodal analysis,


VTh − 4 VTh
= 0 ⇒ 2 VTh − 4 = 0 ⇒ VTh = 2 V
∫0
1 T +
Vrms = V 2 (t) dt 10 10
T
To find the Thevenin resistance, short circuit the
T /2  2t 
2
∫0   dt + ∫T /2 (0) dt
1 T
source.
=
T T
10 × 10
=5 W
4  t3 
T /2 RTh =
10 × 10

1 4 T /2 2
= × 2 t dt =  
T T 0 T 3  3  0 Ans. (a)
3 Linked Answer Questions 27 and 28: A coil of
4 T 1
= 3
× = inductance 10 H and resistance 40 Ω is connected
T 24 6
as shown in the figure. After the switch S has been
Ans. (a) in contact with point 1 for a very long time, it is
moved to point 2 at t = 0.
25. The circuit shown in the figure is in steady state, 27. If, at t = 0+, the voltage across the coil is 120 V,
when the switch is closed at t = 0. Assuming that the value of resistance R is
the inductance is ideal, the current through the
inductor at t = 0+ equals 20 Ω
1
10 Ω S 2 10 H
10 V 10 mH t=0 120 V
R
40 Ω

(a)  0 A (b)  0.5 A (a)  0 Ω (b)  20 Ω


(c)  1 A (d)  2 A (c)  40 Ω (d)  60 Ω

(GATE 2005: 2 Marks) (GATE 2005: 2 Marks)


Solution:
Solution:  For t < 0; switch is open,
10 1 20 Ω
so iL = = 1 A.
10 2
When switch is closed, iL = iL (0+) = 1 A (inductor
current does not change simultaneously). 120 V
Ans. (c) 40 Ω
26. In the given figure, the Thevenin’s equivalent pair
(voltage and impedance), as seen at the terminals
iL (0 −) =
Total voltage
P-Q, is given by Sum of resistance
10 Ω P =
120
=2 A  [Position 1]
20 + 40
In the inductor, the current does not change simul-
20 Ω 4V 10 Ω Unknown
taneously. Therefore,
iL (0+) = iL (0-) = 2 A
network
[Position 2].
+
Voltage across the inductor at t = 0
Q VL (0+) = 120 V
(a)  2 V, 5 Ω (b)  2 V, 7.5 Ω By applying KVL,
(c)  4 V, 5 Ω (d)  4 V, 7.5 Ω 120 = 2(40 + R + 20) Þ R = 0 W
 Ans. (a)
(GATE 2005: 2 Marks)
28. For the value of obtained in the above question,
Solution:  To find the Thevenin voltage open the time taken for 95% of the stored energy to be
­circuit the terminals P and Q. dissipated is close to

Chapter 1 Solved Question Paper.indd 150 3/19/2016 4:29:10 PM


SOLVED GATE PREVIOUS YEARS’ QUESTIONS        151

(a)  0.10 s (b)  0.15 s R Y B


(c)  0.50 s (d)  1.0 s
(GATE 2005: 2 Marks)

Solution:  E1 is the stored energy and E2 is the


dissipated energy. So,
i22 E2
= = 0.95 (GATE 2006: 1 Mark)
i12 E1

Therefore, i22 = 0.95i 12 Þ i 2 = 0.9746i 1


Solution:  Since the core is non-ideal, leakage flux
will be large, leading to a small value of equivalent
When the switch is in position 2, inductance. Thus, the inductance of the winding
i(t) = i 1e−(R / L)t = 2e−(60/10)t = 2e−6 t  (i) will be very low.
Ans. (a)
After 95% of energy dissipation,
i(t) = 2 - 2 ´ 0.97 = 0.05 A
31. The parameters of the circuit shown in the
(ii)
figure given below are Ri = 1 MΩ, Ro = 10 Ω,
Substituting Eq. (ii) in Eq. (i), we get
A = 106 V/V.
0.05 = 2e−6t ⇒ t = 0.5 s
If Vi = 1 mV, then output voltage, input impedance
Ans. (c) and output impedance, respectively, are
29. In the figure given below, the current source is
Ro
I < 0 A, R = 1 Ω, the impedances are ZC = -j Ω Ri
and ZL = 2 j Ω. The Thevenin equivalent looking +
into the circuit across X-Y is +
Vi AVi
X −

(a)  1 V, ¥, 10 Ω (b)  l V, 0, 10 Ω
(c)  l V, 0, ¥ (d)  10 V, ¥, 10 Ω

Y (GATE 2006: 2 Marks)

(a)  2Ð0 V, (1 + 2 j)W (b)  2Ð45°V, (1 + 2 j)W


Vo
Solution:  Voltage gain (A) =
(c)  2Ð45°V, (1 + j)W (d)  2Ð45°V, (1 + j)W
Vi
Vo = AVi = 106 × 1 mV = 1 V
(GATE 2006: 1 Mark)
Vi V
Input impedance Zin = Zi = = i =¥
Solution:  To find the Thevenin voltage: ii 0

VTh = I (R + ZL + ZC ) = 1Ð0°(1 + 2 j − j) = 1(1 + j) Output impedance Zo =


Vo
= Ro = 10 Ω
io
VTh = 2∠45° V Ans. (a)
To find the Thevenin impedance, open circuit the 32. In the circuit shown in the figure given below,
source ­current source I = 1 A, voltage source V = 5 V,
R1 = R2 = R3 = 1 Ω, L1 = L2 = L3 = 1 H,
ZTh = R + ZL + ZC = 1 + 2 j − j = 1 + jW C1 = C2 = 1 F. The currents (in A) through R3
and the voltage source V, respectively, will be
Ans. (d)
30. The three-limbed non-ideal core shown in the
figure given below has three windings with nomi- R1 R2
nal inductances L each when measured individu- L1 L3
ally with a single phase AC source. The inductance
+
of the windings as connected will be C1 −
C2
(a)  very low (b)  L/3 L2 V
R3
(c)  3L (d)  very high I

Chapter 1 Solved Question Paper.indd 151 3/19/2016 4:29:29 PM


152        CHAPTER 1:  ELECTRIC CIRCUITS

(a)  1, 4 (b)  5, 1 I
(c)  5, 2 (d)  5, 4
(GATE 2006: 2 Marks)
V2
Solution:  Given circuit includes only DC sources.
Therefore, inductors are short circuited, while
V1
capacitors are open circuited. VC
1 A R1 R2 I2
The phasor diagram which is applicable to this
I1 ­circuit is
L1 C1 L3
C2 (a) 
V I V2 V1
R3
L2
I VC

Therefore, VR 3 = I1R3 Þ 5 = 1 × I1 Þ I1 = 5 A (b) 

Applying Kirchhoff’s current law, we have V2 I


I1 = 1 + I2 ⇒ I2 = 4 A
V1
Ans. (d)
33. The parameter type and the matrix representation VC
of the relevant two-port parameters that describe
the circuit shown are
(c)  VC
I1 I2
+ +
V1
V1 V2
V2 I
− −

0 0
(a)  z parameters, 
0 0 (d)  V1
1 0
(b)  h parameters, 
0 1 V2
0 0
(c)  h parameters, 
0 0
 1 0
(d)  z parameters, 
0 1
I

(GATE 2006: 2 Marks) VC


(GATE 2006: 2 Marks)
Solution:  Clearly, Z11 is infinite: Thus, two port
networks are defined in terms of h-parameters.
0 0 
Solution:  For resonance, VL = VC.
Therefore, from the options we have 
0 0
. Impedance of the given circuit ZR = R1 + R2.

Ans. (c) V1
IR =
34. The circuit shown in the following figure is R 1+ R 2
energised by a sinusoidal voltage source V1 at
V1
a frequency which causes resonance with a current V2 = IR R 2 + j(VL − VC ) = × R 2 + j(VL − VC )
of I. R1R 2

Chapter 1 Solved Question Paper.indd 152 3/19/2016 4:29:42 PM


SOLVED GATE PREVIOUS YEARS’ QUESTIONS        153

1 1 V Ð 0° VT = V0 cos(w 0 t)
VC = × IR = × R
jw C jw C R1 + R 2
VR Ð − 90° Ans. (b)
= [as 1/j = −90°].
w CX(R 1+ R 2 ) 36. An energy meter connected to an immersion heater
(resistive) operating on an AC 230 V, 50 Hz, AC
Therefore, the phasor diagram is single phase source reads 2.3 units (kWh) in 1 hour.
The heater is removed from the supply and now
connected to a 400 V peak-to-peak square wave
I I2 V1 source of 150 Hz. The power in kW dissipated by
the heater will be
VC (a)  3.478 (b)  1.739
(c)  1.540 (d)  0.870
Ans. (a)
35. An ideal capacitor is charged to a voltage V0 and (GATE 2006: 2 Marks)
connected at t = 0 across an ideal inductor L. (The Solution:  Given that V = 230 V, P = 2.3 kWh =
circuit now consists of a capacitor and inductor 2.3 ´ 103 Wh, f = 50 Hz. Therefore,
1
alone). If we let w 0 = V2 (230)2
= 2.3 × 103 Þ = 2.3 × 103 Þ R = 23 W
, the voltage across
LC
the capacitor at time t > 0 is given by
R R
It is connected to a square wave source of 400 Vp—p.
(a)  V0 (b)  V0cos(w0t) Therefore,

(c)  V0sin (w0t) (d)  V0 e−w 0 t cos (w 0 t) 2


Vrms 2002
P = = = 1.739 kW
(GATE 2006: 2 Marks) R 23
(as Vp − p = 400 and Vp = 200)
Solution:  We know that Ans. (b)
dV (t) 37. The state equation for the current I1 shown in the
i C(t) = C C network given below in terms of the voltage Vx and
dt
Laplace transform, IC (s) = Cs [V (s) − V (0)] where
the independent source V, is given by

V(0) is the initial voltage. 3 Ω 0.2 H 5Ω


+ −
Similarly, iL (t) = ò V × (t)dt
1
L + VxI1 −
1 V (s)
V − +
I L (s) = 0.5 H 0.2 Vx
L S
dI1 5
Therefore, applying Kirchhoff’s current law, we get (a)  = −1.4 Vx − 3.75I1 + V
dt 4
 1 
dI1 5
= −1.4 Vx − 3.75I1 − V
V (s) ⋅ Cs +  = CV (0) (b) 
 Ls  dt 4
dI1 5
 = −1.4 Vx + 3.75I1 + V
1 
(c) 
V (s) ⋅ C s +  = V (0) dt 4
 LCs  dI1 5
(d)  = −1.4 Vx − 3.75I1 − V
 w2 
dt 4
V (s) s + 0  = V (0) (GATE 2007: 2 Marks)
 s 

 s2 + w 2  Solution:  Appling KVL,
V (s)  0
 = V (0)
 s  dI1
Loop 1: V − 3(I1 + I2 ) − Vx − 0.5 = 0 (i)
dt
 
V (s) = V (0)  2 
s
2 dI1
 s + w 0  Loop 2: −5I2 + 0.2Vx + 0.5 =0
dt

Chapter 1 Solved Question Paper.indd 153 3/19/2016 4:30:06 PM


154        CHAPTER 1:  ELECTRIC CIRCUITS

dI1 SW2 R2
−5I2 = −0.2 Vx − 0.5
dt
SW1 10 Ω
dI R1 +
I2 = 0.04 Vx + 0.1 1 10 A
10 Ω
 (ii) L C 10 V
dt -
Substituting Eq. (ii) in Eq. (i), we get
 dI  dI
V − 3I1 − 3 0.04 Vx + 0.1 1  − V2 − 0.5 1 = 0 (a)  55 A, 4.5 V (b)  5.5 A, 45 V
 dt  dt (c)  45 A, 5.5 V (d)  4.5 A, 55 V
dI dI
V − 3I1 − 0.12 Vx − 0.3 1 − Vx − 0.5 1 = 0 (GATE 2007: 2 Marks)
dt dt
dI1 Solution:  We know that
−0.8 = 1.12 Vx + 3I1 + 3I1 − V
VC (0+ ) = VC (0− ) = 10 V
dt
dI1 5
= −1.4 Vx − 3.75I1 + V iL (0+ ) = iL (0− ) = 10 A
dt 4
Ans. (a)
38. In the following figure, transformer T1 has two VL(0+) 10 Ω
secondaries, all three windings having the same iC(0+)
number of turns and with polarities as indicated. +
One secondary is shorted by a 10 Ω resistor R, and 10 Ω VC(0+)

the other by a 15 mF capacitor. The switch SW is
opened (t = 0) when the capacitor is charged to iL(0+)
5 V with the left plate as positive. At t = 0+, the
Applying nodal analysis,
voltage Vp and current IR are

SW VL (0+ ) VL (0+ ) − VC (0+ )


T1
IR + = i L(0+ )
R 10 10
2VL (0+ ) VC (0+ )
+ − = 10
25 V C 10 10
2VL (0+ ) − 10 = 100
Vp
2VL (0+ ) = 190 Þ VL (0+ ) =
110
= 55 V
(a)  -25 V, 0.0 A 2
(b)  Very large voltage, very large current
(c)  5.0 V, 0.5 A Current through the capacitor,
(d)  -5.0 V, -0.5 A V L (0+ ) − VC (0+ ) 55 − 10
iC (0+ ) = = = 4.5 A
(GATE 2007: 2 Marks) 10 10
Ans. (d)
Solution:  Since the secondary winding is at 5 V with 40. In the figure given below, all phasors are with
opposite polarity, primary shows -5 V (due to the ­reference to the potential at point “O”. The locus
same number of turns). The same -5 V will appear of voltage phasor VYX as R is varied from zero to
across the secondary which is shorted by a 10 Ω resis- infinity is shown by
−5
tor. Current through the resistance = = 0.5 A
10
R
Ans. (d) V∠0°
VYX
39. In the circuit shown in the following figure switch
SW1 is initially closed and SW2 is open. The induc- X Y
tor L carries a current of 10 A and the capacitor is
charged to 10 V with polarities as indicated. SW2 V∠0°
is initially closed at t = 0- and SW1 is opened at
t = 0. The current through C and the voltage
across L at t = 0+ is O

Chapter 1 Solved Question Paper.indd 154 3/19/2016 4:30:16 PM


SOLVED GATE PREVIOUS YEARS’ QUESTIONS        155

(a)  O 2 V (b)  Power dissipated, P0 = V0 I0 = 1 W


VYX
Ans. (a)
VYX
42. The matrix A given below is the node incidence matrix
O 2V
of a network. The columns correspond to branches of
Locus of VYX Locus of VYX the network while the rows correspond to nodes.
(c)  O 2 V (d)   1 1 1 0 0 0
 0 −1 0 − 1 1 0
A=
0 −1 −1
VYX VYX
−1 0 0
O 2V  0 0 −1 1 0 1
Locus of VYX Locus of VYX 

Let V = [v1v2  v6 ]T denote the vector of branch

(GATE 2007: 2 Marks) voltages while I = [i1 i2 … i6]T that of branch currents.

Solution:  From the given circuit, The vector E = [e1 e2 e3 e4]T denotes the vector of
V X = VÐ 0° (i) node voltages relative to a common ground.
Which of the following statements is true?
So, (a) The equations v1 − v2 + v3 = 0, v3 + v4 − v5 = 0
V Y − 2VÐ 0° VY 2VvÐ 0+° v = 0, v + v − v = 0 are KVL equations for the network

+ V Y ( jw c) = 0 Þ + V Y ( jw c) −
v1 2 =0
3 3 4 5
R R R for some loops.
V Y + V Y ( jw cR) − 2VÐ 0° (b) The equations v1 − v3 − v6 = 0, v4 + v5 − v6 = 0
= 0 Þ V Y (1 + jw cR) = 2VÐ 0°
R v1 − v3 − v6 = 0, v4 + v5 − v6 = 0 are KVL equations for the
2VÐ 0° 
network  for
 some loops.
Þ V YX = V X − V Y = V −
2V
VY =
1 + jw cR 1 + jw cR (c)  E =  A V E
(d)  A V E= 0 are KVL equations for the network
As R ® 0, V XY = V −
2V
= −V
1 (GATE 2007: 2 Marks)

As R ® ¥, V YX = V −
2V
= V −0 = V Solution:  Given the incidence matrix
¥
é 1 1 1 0 0 0ù
Ans. (b) ê 0 −1 0 −1 1 0ú
A=ê ú
41. A 3 V DC supply with an internal ­resistance of
ê − 1 0 0 0 − 1 1ú
2 Ω supplies a passive non-linear ­resistance char- êë 0 0 −1 1 0 1 úû
acterised by the relation VNL = I2NL. The power    
dissipated in the non-linear resistance is From the options, for V E = A V .E

(a)  1.0 W (b)  1.5 W é e1 ù é 1 1 1 0 0 0ù


ê e ú ê − − 0ú
ê 2ú = ê
0 1 0 1 1
ú [v v v v v v ]
(c)  2.5 W (d)  3.0 W T
ê e3ú ê− 1 0 0 0 − 1 − 1ú 1 2 3 4 5 6
êëe4 úû êë 0 0 −1 1 úû
(GATE 2007: 2 Marks)
1 0
Solution:  Circuit based on the given data is as below.
é v1 +v2 +v3 ù
2Ω ê−v −v +v ú
+ =ê 2 4 5
ú
ê−v1 −v5 −v6 ú
+ êë−v3 −v4 +v6 úû
V0 = I 02
3V
− Ans. (c)
I0
Linked Answer Questions 43 and 44: An
− inductor designed with 400 turns coil wound on an
From the circuit, we have iron core of 16 cm2 cross sectional area and with a
cut of an air gap length of 1 mm. The coil is con-
2+I0022I−0 3−
2 2
I 02 + 2I 02 −I 032 VI+0 = V 3=V 0= 0 nected to a 230 V, 50 Hz AC supply. Neglect coil
2I 2 + 3I − I − 3 = 0
I 0 + 3I 0 −
2
I 0I 0+ 0−33I 0=−00I 0 −0 3 = 0
resistance, core loss, iron reluctance and leakage
inductance (mo = 4p ´ 10-7 H/m).
I 0 (I 0 + 3)I-0 1(I(I0I0(0I++ 3))-3=
0 3+ )1(-0I1(I+0 3+) =
3) 0= 0
0 43. The current in the inductor is
Þ I0 = 1 A Þ or Þ =A1-A
I 0 II=00 1= 3orAorI 0 I=0 -=3A-3A
(a) 18.08 A (b) 9.04 A
= 1Using
Using I 0 Using A, Iwe I 0= 1=A1, we
0 have
A, we
havehave
(c) 4.56 A (d) 2.28 A
2 I2 Þ V = 1 V
V0 = I 20 VÞ 0
V=V0 I==
0 0 10 ÞV V 0 =0 1 V
(GATE 2007: 2 Marks)

Chapter 1 Solved Question Paper.indd 155 3/19/2016 4:30:27 PM


156        CHAPTER 1:  ELECTRIC CIRCUITS

Solution:  We know that inductance is given by 46. The Thevenin’s equivalent of a circuit operating at
w = 5 rad/s, has V¥ = 3.71Ð -15.9° V and Z0 = 2.38
m0 N 2 A
L= - j 0.667 Ω. At this frequency, the minimal realiza-
l tion of the Thevenin’s impedance will have a
4p × 10−7 × 400 × 400 × (16 × 10−4 )
= (a)  resistor and a capacitor and an inductor
(1 × 10−3 ) (b)  resistor and a capacitor
= 321.6 mH (c)  resistor and an inductor
(d)  capacitor and an inductor
V = I ×L
(GATE 2008: 1 Mark)
V
I =
2pfL
Solution:  Given that w = 5 rad/s. So,
230
= = 2.28 A
2 × 3.14 × 50 × 321.6 × 10−3 V¥ = 3.71 Ð − 15.9°
 (i)
Z0 = 2.38 − j0.667 W
Ans. (d)
44. The average force on the core to reduce the air gap Let us assume that there is only resistance and
will be capacitance in the circuit. Then,

(a) 832.29 N (b) 1666.22 N q = − tan−1(w CR)


(c) 3332.47 N (d) 6664.84 N
j
Z0 = R −  (ii)
(GATE 2007: 2 Marks) wC
From Eqs. (i) and (ii), R = 2.38 Ω; w C = 1/0.667.
Solution:  Energy (E) stored in the inductor is Therefore,
given by
 2.38 
q = − tan−1  = −73.34°
1  0.667 
E = LI 2
2 But in the given data, q = -15.9°. Therefore, there
= × 321.6 × 10−3 × (2.28)2
1 is an inductor is also connected in the circuit.
2 Ans. (a)
= 0.835 47. The time constant for the given circuit will be

Then average force is 1F 3W

E 0.835
F = = = 835 N
l 1 × 10−3 1F 1F 3W 3A
Ans. (a)
45. The number of chords in the graph of the given
circuit will be
1 1
(a)  s (b)  s
9 4
(c)  4 s (d)  9 s
(GATE 2008: 2 Marks)
+

Solution:  Overall time constant T = CeqReq.
Equivalent resistance is
(a)  3 (b)  4 3Ω
(c)  5 (d)  6
(GATE 2008: 1 Mark)
3Ω
Req
Solution:  Let l is the number of chords, n is the o
of nodes (4), b is the number of branches (6).
We have l = b - n + 1 = 6 - 4 +1 = 3
Req = 3 + 3 = 6 (series connection)
Ans. (a)

Chapter 1 Solved Question Paper.indd 156 3/19/2016 4:30:36 PM


SOLVED GATE PREVIOUS YEARS’ QUESTIONS        157

Equivalent capacitance is obtained by eliminating 49. Assuming ideal elements in the circuit shown
two parallel capacitances, each of 1 F. below, the voltage Vab will be
Ceq = 1 + 1 = 2 F
a 2Ω
Elimination a series connection of 2 F with 1 F +
2 ×1 2 −
Ceq = = F 1A
2+1 3 Vab I + 5V
2
So, the time constant is T = 6 × = 4 s −
3
Ans. (c) b
(a)  -3 V (b)  0 V
48. The resonant frequency for the given circuit will be (c)  3 V (d)  5 V

0.1 H (GATE 2008: 2 Marks)

Solution:  Apply Kirchhoff’s voltage law to the


1F 1W given circuit,
Vab − 2I + 5 = 0
Va = 2I − 5 = 2(1) − 5 = −3 V
(a)  1 rad/s (b)  2 rad/s
Ans. (a)
(c)  3 rad/s (d)  4 rad/s
50. A capacitor consists of two metal plates each
(GATE 2008: 2 Marks) 500 × 500 mm2 and spaced 6 mm apart. The space
between the metal plates is filled with a glass plate
Solution:  To find the resonant frequency, it is of 4 mm thickness and a layer of paper of 2 mm
necessary to find the overall impedance of circuit, thickness. The relative permittivities of the glass
1 and paper are 8 and 2, respectively. Neglecting the
R
jw C R fringing effect, the capacitance will be (Given that
Z = jw L + = jw L + e0 = 8.85 × 10-12 F/m)
1
+R 1 + jw CR
jw C (a)  983.33 pF (b)  1475 pF
Multiplying and dividing by the complex conjugate. (c)  6637.5 pF (d)  9956.25 pF
R (1 − jw CR) (GATE 2008: 2 Marks)
Z = jw L + ×
1 + jw CR (1 − jw CR)
R(1 − jw CR) Solution:  The equivalent capacitance is given by
= jw L +
(1 + w 2C 2R2 ) C1 × C2
Ceq =
jwL(1 + w C R ) + R(1 − jwCR)
2 2 2 C1 + C2
=
(1 + w 2C 2R2 ) Given that:
d1 = 4 mm, e r = 8 mm, d2 = 2 mm, e r = 2 mm.
Separating real and imaginary parts we have 1 2

R j w L(1 + w 2C 2R 2 ) − w 2CR e 0 × er × A 8.85 × 10−12 × 8 × 500 × 500 × 10−6


1
C1 = =
4 × 10−3
Z= +
1 + w 2C 2R2 1 + w 2C 2R2 d1

Imaginary term = 0, so = 442.5 × 10−11 F


8.85 × 10−12 × 2 × 500 × 500 × 10−6
w L(1 + w 2C 2R 2 ) − w CR 2 = 0 C2 =
2 × 10−3
w × (0.1)[1 + w (1)(1)] = w (1)(1)
2 2
= 221.25 × 10 −11
F
442.5 × 10−11 × 221.25 × 10−11
1
0.1w (1 + w 2 ) = w ⇒ 1 + w 2 =
0.1 Ceq = −11 −11
= 147.65 × 10−11
442.5 × 10 + 221.25 × 10
1 + w 2 = 10 ⇒ w 2 = 9 ⇒ w = 3 rad/s
= 1475 pF
Ans. (c) Ans. (b)

Chapter 1 Solved Question Paper.indd 157 3/19/2016 4:30:43 PM


158        CHAPTER 1:  ELECTRIC CIRCUITS

51. A coil of 300 turns is wound on a non-magnetic


core having a mean circumference of 300 mm and a 6
I(t) mA
cross-sectional area of 300 mm2. The inductance of 5
the coil corresponding to a magnetising current of 4
3A will be 3
(Given that m0 = 4m × 10-7 H/m) 2
(a)  37.68 mH (b)  113.04 mH 1
(c)  37.68 mH (d)  113.04 mH
0 1 2 3 4 5 6 7 8 9
(GATE 2008: 2 Marks) t(ms)
53. The charge stored in the capacitor at t = 5 ms,
Solution:  Given that l = 300 mm, n = 300, will be
A = 300 mm2. Now,
(a)  8 nC (b)  10 nC
−7 −6
m0 n A 4p × 10 × (300) × (300 × 10
2 2 (c)  13 nC (d)  16 nC
)
L= = −3
l (300 × 10 ) (GATE 2008: 2 Marks)
= 113.04 µH
Ans. (b) Solution:

52. In the circuit shown in the figure given below, the i(t)
value of the current i will be given by
A
4
1W 3W
a+ V
ab −b
4Vab B
+
2 E
+
5V − 1W 1W

I
D C
O 2 5
(a)  0.31 A (b)  1.25A
t
We have Q = ∫ i(t) dt
(c)  1.75A (d)  2.5A
(GATE 2008: 2 Marks) 0
where Q is the charge stored at t = 5 ms and
Solution:  Node equation at a, i(t)dt is the area under the curve.

Va − 5 Va − 0 Q = Area under {(OAD) + EBCD + AEB}


+ = 0 Þ Va = 2.5 V
1 1
1 1 = ×2× 4 + 3×2 + ×2×3
Node equation at b, 2 2
= 4 + 6 + 3 = 13 nC
Vb − 4 Vab Vb − 0
+ =0 Ans. (c)
3 1
54. The capacitor charged up to 5 ms, as per the ­current
Vb − 4 (Va − Vb ) + 3Vb = 0
profile given in the figure, is connected across an
Vb − 4 (2.5 − Vb ) + 3Vb = 0 inductor of 0.6 mE. Then value of voltage across
8Vb = 10 Þ Vb = 1.25 V the capacitor after 1 ms will approximately be
(a)  18.8 V (b)   23.5 V
V (c)  -23.5 V (d)  -30.6 V
I = b = 1.25 A
1 (GATE 2008: 1 Mark)
Ans. (b)
Solution:  We know that
Linked Answer Questions 53 and 54: The Q0
­current I(t) sketched in the figure flows through an C=
initially uncharged 0.3 nF capacitor. V0

Chapter 1 Solved Question Paper.indd 158 3/19/2016 4:30:49 PM


SOLVED GATE PREVIOUS YEARS’ QUESTIONS        159

Q0 13 Solution:  To find the resistance of incandescent


⇒ V0 = = = 43.3 V lamp rated at 200 W/220 V.
C 0.3
V2
VC(t) = V0cosw0t P1 =
R1
(as capacitor connected across an inductor)
V2 2202
w0 = = 242 W.
1 1
= ⇒ R1 = =
LC 0.3 × 10−9 ×0.6 × 10−3 P1 200
= 2.35 × 106 rad/s Similarly for 100 W/220 V,
Therefore, at t = 1 ms
VC (t) = 43.33 cos (2.3 × 106 × 1 × 106 ) V2 2202
R2 = = = 484 W.
100
= 43.3(−0.70) = −30.31 V P2
Ans. (d) For the series connection,
55. The current through the 2 kΩ resistance in the cir-
cuit shown below is R2 = n × R1
1 kΩ C 1 kΩ 484 = n × 242 Þ n = 2
Ans. (d)
A B
2 kΩ 57. In the figure shown below, all elements used are
ideal. For time t < 0, S1 remained closed and S2
1 kΩ D 1 kΩ open. At t = 0, S1 is opened and S2 is closed. If the
voltage VC2 across the capacitor C2 at t = 0 is zero,
6V the voltage across the capacitor combination at
t = 0+ will be
(a)  0 mA (b)  1 mA
(c)  2 mA (d)  6 mA S1 S2
(GATE 2009: 1 Mark)

Solution:  The equivalent bridge circuit is 3V C1 1F C2 2F


R1 R2

(a)  1 V (b)  2 V
A B
2 kΩ (c)  1.5 V (d)  3 V
R3 R4
(GATE 2009: 1 Mark)

Solution:  We have
8V For state 1: t < 0 S1: closed and S2: opened
In the bridge circuit, So, C1 is charged up to 3 V ⇒ VC (0) = 3 V.
For state 2: S1: opened and S2: closed. So,
R1R4 = R2R3 = 1 kΩ
Ceq VC1(0+ ) = C1VC1(0+ ) + C2VC2 (0+ )
and I = 0 mA for a balanced bridge.
Ans. (a) (2 + 1) × 3 = 1 × 3 + 2 × VC2 (0+ )
56. How many 200 W/220 V incandescent lamps con- 9 = 3 + 2VC2 (0+ )
nected in series would consume the same total
power as a single 100 W/220 V incandescent lamp? or VC2 (0+ ) = 3 V
(a)  Not possible (b)  4 Ans. (d)
(c)  3 (d)  2 58. The equivalent capacitance of the input loop of the
(GATE 2009: 1 Mark) circuit shown below is

Chapter 1 Solved Question Paper.indd 159 3/19/2016 4:30:57 PM


160        CHAPTER 1:  ELECTRIC CIRCUITS

i1 1 kΩ 1 kΩ VR 2 = 2 V
VR 2 4
IR 2 = = =2 A
1 kΩ 2 2
Input 49i1
loop 100 µF
To make the current double we need to take
100 µF VR 2 = 8 V.
Ans. (b)
(a)  2 mF (b)  100 mF Linked Answer Questions 60 and 61:
(c)  200 mF (d)  4 mF
2 kΩ 3 VAB
(GATE 2009: 1 Mark) +− A

5V +
Solution:  By applying Kirchhoff’s voltage law, − 2 kΩ 1 kΩ
1
V1 − i 1(1 + 1) × 103 − (i + 49i 1) = 0
jw C 1 B
1 1 60. For the circuit given above, Thevenin’s resistance
V1 − 2i 1× 103 − i − 49i 1= 0
jw C 1 jw C across the terminals A and B is
1 (a)  0.5 kΩ (b)  0.2 kΩ
V1 = 2 × 103 i 1+ 50i 1
jw C (c)  1 kΩ (d)  0.11 kΩ
(GATE 2009: 2 Marks)
Input impedance,
Solution:  To obtain Thevenin’s circuit,
2 × 103 i1 +
1
50i 1
V1 jw C 3 VAB = 3 VS
Z1 = =
i1 i1 2 kΩ VP
+− A
1 +
= 2 × 10 + 3
jw (C /50)
5V 2 kΩ 1 kΩ VS, IS
100 m F
= 2 mF
C
Ceq = =
50 50 −
Ans. (a) B
59. For the circuit shown below, find out the current Applying Kirchhoff’s current law,
flowing through the 2 Ω resistance. Also, identify
the changes to be made to double the current V P − 5 V P VS
+ + = IS
through the 2 Ω resistance. 2 2 1
VP − 5 + VP + 2VS = 2IS
2VP + 2VS = 2IS + 5
IS = 5 A V P + V S = I S + 2. 5  (i)
VS = 4 V + 2Ω
− Also given,

VP − VS = 3VS
(a)  (5 A; Put VS = 20 V) VP = 4VS (ii)

(b)  (2 A; Put VS = 8 V)
(c)  (5 A; Put VS = 10 A) Substituting Eq. (ii) in Eq. (i),
(d)  (7 A; Put VS = 12 A)
5VS = IS + 2.5
(GATE 2009: 2 Marks)
V S = 0. 2 I S + 0 . 5  (iii)
Solution:  From the given figure, we have Equivalent Thevenin circuit is

Chapter 1 Solved Question Paper.indd 160 3/19/2016 4:31:04 PM


SOLVED GATE PREVIOUS YEARS’ QUESTIONS        161

RTh When the switch is opened, the equivalent circuit is


+

VTh IS, VS VC(0+) 4Ω

VS = ISRTh + VTh  (iv) For t = 0+, VC (0+ ) = VC (0) = 4 V

Comparing Eqs. (iii) and (iv), we get RTh = 0.2 k W and VThCurrent through 4 Ω at t = 0+ ,
= 0.5 V.
RTh = 0.2 k W and VTh = 0.5 V. V (0+ ) 4
i1 = C = =1 A
Ans. (b) 4 4
61. For the circuit given above, Thevenin’s voltage Current in capacitor at t = 0+ , i C(0+ ) = i1 = 1 A
across the terminals A and B is
Ans. (b)
(a)  1.25 V (b)  0.25 V 63. As shown in the figure given below, a 1 Ω ­resistance
(c)  1 V (d)  0.5 V is connected across a source that has a load line
(GATE 2009: 2 Marks) v + i = 100. The current through the resistance is
i
+
Solution:  From the solution of previous question,
VTh = 0.5 V. Source v 1W
Ans. (d) −
62. The switch in the circuit has been closed for a long
time. It is opened at t = 0. At t = 0+ the current (a)  25 A (b)  50 A
through the 1 mF capacitor is (c)  100 A (d)  200 A
(GATE 2010: 1 Mark)
1Ω
Solution: Given that
t=0 Open circuit voltage VTh = 100 V.
5V 1 µF 4Ω
Short circuit current i sc = 100 A.

So,

VTh
=1 W
100
(a)  0 A (b)  1 A RTh = =
isc 100
(c)  1.25 A (d)  5 A
Equivalent circuit is
(GATE 2010: 1 Mark)
1W
Solution: When the switch is closed, the equivalent
circuit is
100 V 1W
1Ω

+
5V VC(0) 4Ω i=
100
= 50 A
− 1+1
Ans. (b)
64. If 12 Ω resistor draws a current of 1 A as shown in
4 ×1 the figure given below, the value of resistance R is
VC (0) = ×5 = 4 V
4 +1 (a)  4 Ω (b)  6 Ω (c)  8 Ω (d)  18 Ω

Chapter 1 Solved Question Paper.indd 161 3/19/2016 4:31:13 PM


162        CHAPTER 1:  ELECTRIC CIRCUITS

1Ω R
For Port 2: Output
¢ I 1¢ + Z12
V1¢ = Z11 ¢ I2¢

V2¢ = Z21
¢ I1¢ + Z22
¢ I2¢
2A 1A 12 Ω 6V
Now, I1 = I1′ and I2 = I2′ . When the resistance of
1 Ω is connected,
(GATE 2010: 2 Marks) V1′ = V1 + I1′ × 1 = V1I1′ = V1 + I1
____________ |
Solution: The circuit can be represented as ↓
1Ω
V1′ = Z11I1 + Z12 I2 + I1
2A VA i R
 
1A
2A 12 Ω 6V V1′ = I1 {Z11 + 1} + Z12 I2

′ = Z11 + 1 
Z11 (i)

Therefore, current across resistor R is


′ = Z12 .
Z12 (ii)
i = 2 −1
For output port,
On applying nodal analysis, voltage across resistor
R is V2′ = Z21
′ + I1′ + Z22
′ I2′ = Z21
′ + I1 + Z22
′ I2
VA = 1 × 12 = 12 V. So,
′ = Z21 
Z21 (iii)
V −6 12 − 6
i= A ⇒R= =6 Ω
R 1 ′ = Z22 
Z22 (iv)
Ans. (b)
Therefore, from Eqs. (i)-(iv), we have
65. The two-port network P shown in the figure has
Z11 + 1 Z12 
Z=
Z22 
ports 1 and 2, denoted by terminals (a, b) and
(c, d), respectively. It has an impedance matrix Z21
Z with parameters denoted by Zij. A 1 Ω resistor Ans. (c)
is connected in series with the network at port 1 66. The rms value of the current i(t) in the circuit
as shown in the figure. The impedance matrix of shown below is
the modified two-port network (shown as a dashed
box) is 1F
1H

e 1Ω a c 1Ω
P
i(t)
b d 1Ω
+
(1.0sin t) V
 Z + 1 Z12 + 1 Z + 1 Z12 
(a)   11
Z21 Z22 + 1
(b)   11
  Z21 Z22 + 1 (a) 
1
A (b) 
1
A
2 2
 Z + 1 Z12   Z + 1 Z12 
(c)   11
Z21 Z22 
(d)   11
  Z21 + 1 Z22  (c)  1 A (d)  2A

(GATE 2010: 2 Marks) (GATE 2011: 1 Mark)

Solution:  Solution: From the given circuit, 1/VS = sint. In


For Port 1: Input general source voltage is expressed as,
V1 = Z11I1 + Z12 I2 VS = sinw t

Therefore by comparison, w = 1 rad/s.


V2 = Z21I1 + Z22 I2

Chapter 1 Solved Question Paper.indd 162 3/19/2016 4:31:26 PM


SOLVED GATE PREVIOUS YEARS’ QUESTIONS        163

XL = w L = 1 W The high-pass filter is


XC = w C = 1 W

So, net impedance is 0.


Ohm’s law, V = IR, so current is given by
V sint
I (t) : S = = sint
R 1 20 Hz
I 1
I rms = m = A
2 2 The resultant system can be given by the following
Ans. (b) band-pass filter
67. In the circuit given below, the value of R required
for the transfer of maximum power to the load
having resistance of 3 Ω is

R
20 Hz 30 Hz
6Ω
Ans. (d)
+
10 V 3Ω Load
Common Data Questions 69 and 70: The
input voltage given to a converter is
(a)  zero (b)  3 Ω v i = 100 2 sin(100 p t) V
(c)  6 Ω (d)  infinity
The current drawn by the converter is
(GATE 2011: 1 Mark) i i = (10 2(100 p t − p /3) + 5 2 sin(300 p t + p /4)
+2 2 sin(500 p t − p /6) A
Solution:  Power transferred to the load,
æ ö
2
10 69. The input power factor of the converter is
P = I 2RL = ç ÷ RL
è RTh + RL ø (a)  0.31 (b)  0.44
For maximum power transfer RTh = 0, so (c)  0.5 (d)  0.71

=0ÞR=0
6R (GATE 2011: 2 Marks)
RTh =
6+R
Ans. (a) Solution:  Given that:
68. A low-pass filter with a cut-off frequency of 30 Hz vi = 100 2 sin(100 p t)
i i = 10 2 sin(100 p t − p /3) + 5 2 sin(300 p t + p /4)
is cascaded with a high-pass filter with a cut-off
frequency of 20 Hz. The resultant system of filters
will function as +2 2 sin(500 p t − p /6)
(a) an all-pass filter. Now, i i = i 1 + i 3 + i 5. As i3 = i5 = 0 , input power
(b) an all-stop filter.
(c) a band stop (band-reject) filter. factor depends only on the fundamental component,
(d) a band-pass filter. I (t) = 10 2 sin(100 π t − π 3 )
(GATE 2011: 1 Mark) V (t) = 100 2 sin(100 p t)

p
Therefore, power factor = cosf1 = cos
Solution:  The low-pass filter is
= 0.5
3
Ans. (c)
70. The active power drawn by the converter is
(a)  181 W (b)  500 W
(c)  707 W (d)  887 W
30 Hz (GATE 2011: 2 Marks)

Chapter 1 Solved Question Paper.indd 163 3/19/2016 4:31:34 PM


164        CHAPTER 1:  ELECTRIC CIRCUITS

Solution:  For three harmonic components, we 73. In the circuit shown below, the current through the
have, inductor is
P = Vrms × I rms 1 cos q1 + Vrms × I rms  3 cos q3
+ Vrms × I rms  5 cos q5 j1 Ω
1Ω
Now, Vrms3 = Vrms5 = 0, so 1∠0A
P = Vrms1 × I rms1 × cos θ = 100 × 10 × cos p 3
1∠0V 1∠0V
− −
= 100 × 10 × 0.5 = 500 W
+ +
1∠0A
1Ω
Ans. (b)
−j1 Ω
Common Data Questions 70 and 71: An RLC
­circuit with relevant data is given below.

IS IRL IC 2 −1
(a)  A (b)  A
1+ j 1+ j
+ R
1
VS C (c)  A (d)  0 A
1+ j
L (GATE 2012: 1 Mark)

We have VS = 1Ð 0 V, IS = 2Ðp /4 A, Solution:  Applying the nodal method,


IRL = 2Ð − p /4 A. V1 + 1∠ 0° V1 + 1∠ 0°
+ = 1∠ 0°
71. The power dissipated in the resistor R is 1 j1
V1( j1 + 1) + j1 + 1∠ 0° = j1
(a)  0.5 W (b)  1 W
−1
(c)  2 W (d)  2 W V1 =
1 + j1
(GATE 2011: 2 Marks)
1
− +1
V1 + 1∠ 0° 1+ j
Solution:  We have Current I1 = =
j1 j1
VS = 1Ð 0 V and IRL = 2Ð −p /4 A
j 1
As the power delivered by the source is equal to the = = A
(1 + j)j 1 + j
power dissipated across the resistance,

P = VI cos f = 1 × 2 × cos p/4 = 2 ×


1 Ans. (c)
=1W
2 74. The impedance looking into nodes 1 and 2 in the
Ans. (b) given circuit is
72. The current IC in the figure above is
ib
1
(a)  -2A j (b)  − A
2j
1 1 kΩ 99ib
(c)  + A (d)  + 2A j
2j
9 kΩ 1
(GATE 2011: 2 Marks) +
100 Ω 1V
Solution:  From the circuit, we have −
2
IC = IS − IRL = 2Ð p /4 − 2Ð p /4
= 2 [(cos p /4 + j sin p /4 ) − (cos p /4 − j sin p /4 ) ] (a)  50 Ω (b)  100 Ω
= +2j A (c)  5 kΩ (d)  10.1 kΩ
Ans. (d) (GATE 2012: 1 Mark)

Chapter 1 Solved Question Paper.indd 164 3/19/2016 4:31:44 PM


SOLVED GATE PREVIOUS YEARS’ QUESTIONS        165

Solution:  Adding a test source across the termi- Solution:  Laplace equivalent for the given circuit is
nals 1 and 2, we have

1 i(s) 1
ib 1 kΩ
sC1 Cs2
IT
VC(0)
100 Ω + +
9 kΩ − VT
99ib
s −

VT VC (0)/s VC (0)
I (s) = =
é C + C2 ù
ZTh = 1 1
IT + sê 1 ú
ë C1C2s û
sC1 sC2
By applying KCL,
CC
I (s) = 12 × 1 2 = 12Ceq
VT VT
+ − 99I b = I T  (i)
9 × 10 + 1 × 10
3 3 100 C1 + C2
VT V
+ T − 99ib = I T (i) Taking inverse Laplace transform on Eq. (i), we
10 × 10 3 100  have
Also, i(t) = 12Ceq d (t)
VT −VT
ib = − =  (ii)
9 × 10 + 1 × 10
3 3
10 × 103 Thus, the solution is an impulse function.
Ans. (d)
Substituting Eq. (ii) in Eq. (i), we get 76. If VA-VB = 6 V; then VC  -VD is

VT VT 99VT R VA 2Ω VB R
+ + = IT
10 × 103 100 10 × 103
R R −
100VT V
+ T = IT
R R 1Ω R 10 V
+
10 × 10 3 100
R
200VT 2V
= IT Þ T = IT + − VC VD
10 × 10 3 100 5V 2A
VT
= 50 W
100
ZTh = = (a)  -5 V (b)  2 V
IT 2 (c)  3 V (d)  6 V
Ans. (a) (GATE 2012: 2 Marks)
75. In the following figure, C1 and C2 are ideal capaci-
tors. C1 has been charged to 12 V before the ideal Solution:  Given that VA − VB = 6 V. Current
switch S is closed at t = 0. The current i(t) for all t is through 2 Ω resistor is
V A − VB 6
S t=0 I AB = = =3 A
R 2
With reference to the terminals B and D, the ­circuit is

C1 i(t) C2 IAB
R
B

R +
(a)  Zero R 10 V
(b)  A step function −
(c)  An exponentially decaying function
(d)  An impulse function A
IDC R
(GATE 2012: 1 Mark)

Chapter 1 Solved Question Paper.indd 165 3/19/2016 4:31:53 PM


166        CHAPTER 1:  ELECTRIC CIRCUITS

Here, I AB = I DC = 3 A, as the output current from 10 − 3


I1 =
A to B is equal to the input current from D to C; 2+R
the circuit being one-port network.
So, the total current flowing through 1 Ω resistor Power delivered from circuit A → circuit B,

 10 − 3   10 − 3 
by applying KCL at D is, 2
PAB = (I12 )R + 3I1 = 
 2 + R 
R + 3
I1 W = 2 + IDC = 2 + 3 = 5 A  2 + R 
VCD = 1 × (−I1 ) = −5 V 49R 21 42 + 70R
PAB = + = (i)
Ans. (a) (2 + R)2 (2 + R) (2 + R)2
77. Assuming both the voltage sources are in phase, To find the maximum power transferred, differenti-
the value of R for which maximum power is trans- ate Eq. (i) w.r.t. R and equate to zero,
(2 + R)2 70 − (42 + 70R) × 2(2 + R)
ferred from circuit A to circuit B is
dPAB
= =0
2Ω R dR (2 + R)4
(2 + R) [(2 + R)70 − (42 + 70R) × 2] = 0
+ +
10 V −j1 Ω 3V 140 + 70R − 84 − 140R = 0
− − 70R = 56 ⇒ R = 0.8 Ω
Ans. (a)
Circuit A Circuit B Common Data Questions 78 and 79: With
10 V DC connected at port A in the linear non-
(a)  0.8 Ω (b)  1.4 Ω reciprocal two-port network shown below, the fol-
(c)  2 Ω (d)  2.8 Ω lowing were observed:
 (i) 1 Ω connected at port B draws a current of 3 A.
(ii) 2.5 Ω, connected at port B draws a current of
(GATE 2012: 2 Marks)
2 A.
Solution:  For circuit B, Thevenin equivalent ­circuit +
is
R A B

+ 78. For the same network, with 6 V DC connected at



VTh, RTh 3∠0° V
−j1 Ω port A, 1 Ω connected at port B draws 7/3 A. If 8 V
DC is connected to port A, the open circuit voltage
at port B is
Thevenin impedance is (a)  6 V (b)  7 V
R (c)  8 V (d)  9 V
(GATE 2012: 2 Marks)

ZTh ZTh = R
−j1 Ω Solution:  Let 6 V be connected to Port A. Then
Thevenin voltage is represented as, VTh = 6 V,

R L = 1 W and I L = A . So,
7
VTh = 3∠ 0° V 3

Therefore, combining circuit A ® circuit B, æ 7ö æ 7ö


VTh(6 V) = ç RTh × ÷ + ç 1 × ÷
we have è 3ø è 3ø
7 7 14 7 21
I1
2W R = 2× + = + = =7 V
3 3 3 3 3
At Port B,
VTh = V1a + b
10∠0° 3∠0°
where a1+is bthe [input
9 = 10V at V1 voltage
= 10 V applied
(see next
atsolution)]
port A. For
V1 = 10 V and 6 V, the relation can be writen as

Chapter 1 Solved Question Paper.indd 166 3/19/2016 4:32:03 PM


SOLVED GATE PREVIOUS YEARS’ QUESTIONS        167
VTh = V1a + b
9 = 10a + b [at V1 = 10 V (see next solution)] (i) Thevenin resistance remains same for both the
ports, so for RL = 7 Ω, we have
7 = 6a + b [at V1 = 6 V ]  (ii)
Solving Eqs. (i) and (ii), we get a = 0.5, b = 4 . VTh(10 V) −9
Therefore, IL = = =1 A
RTh + RL 2+7
VTh (V 1) = 0.5V1 + 4 
Ans. (c)
for any voltage applied to the port.
Linked Answer Questions 80 and 81: In the
From the given options, for V1 = 8 V circuit shown, the three voltmeter readings are V1
VTh(8 V) = (0.5 × 8) + 4 = 4 + 4 = 8 = 220 V, V2 = 122 V, V3 = 136 V.
R I
Now, as VTh(8 V) = Voc , we have Voc = 8 V
Ans. (c) V2
RL
79. With 10 V DC connected at port A, the current Load
drawn by 7 Ω connected at port B is
V1 V3
3 5 X
(a)  A (b)  A
7 7
9
(c)  1 A (d)  A 80. The power factor of the load is
7
(a)  0.45 (b)  0.50
(GATE 2012: 2 Marks) (c)  0.55 (d)  0.60
(GATE 2012: 2 Marks)
Solution:  For 10 V connected to Port A, the
Thevenin equivalent circuit is
Solution:  Given that V1 = 220 V, V2 = 122 V, V3 = 136 V.
RTh Port BV = 220 V, V = 122 V, V = 136 V. From the circuit,
1 2 3
I Sum of applied voltage = Sum other voltage drops
L

RL V1 = V2 + V3
VTh (10 V)
In polar coordinates,

V1 = V2 ∠ 0° + V3 ∠ q °
Port A
V1 = V2 + V3 cos q + jV3 sin q
VTh(10 V)
IL = (i)
RTh + RL Separating real and imaginary parts,

When RL = 1 W, I L = 3A , we have V1 = (V2 + V3 cos q ) + j(V3 sin q )
VTh(10 V)
3=  (ii) To find magnitude of V1,
RTh + 1
When RL = 2.5 W, I L = 2 A , we have V1 = (V2 + V3 cos q )2 + (V2 sin q )2
VTh(10 V)
2=  (iii) 220 = (122 + 136 cos q )2 + (136 sin q )2
RTh + 2.5
Dividing Eq. (ii) by Eq. (iii), we get Therefore, cos q = 0.45
3 RTh + 2.5 Ans. (a)
=
2 RTh + 1 81. If RL = 5 Ω, the approximate power consumption
in the load is
3RTh + 3 = 2RTh + 5 ⇒ RTh = 2 Ω
(a)  700 W (b)  750 W
Substituting in Eq. (i), we get (c)  800 W (d)  850 W
VTh(10 V)
Þ VTh(10 V) = 9 V
(GATE 2012: 2 Marks)
3=
2 +1 Solution:  Voltage at load resistance,

Chapter 1 Solved Question Paper.indd 167 3/19/2016 4:32:17 PM


168        CHAPTER 1:  ELECTRIC CIRCUITS

VR = V3 cos q = 136 × 0.45 = 61.2 V Solution:  We have VS = 100Ð 53.13°


From the figure, VS = (3 + j4)I1 + j40I2
Power absorbed by load,
100 ∠ 53.13 = (3 + 4 j)I1 + 0 (as I2 = 0 and RL = ∞)
VR2 2
(61.2)
100 ∠ 53.13
PL = = = 750 W
RL 5 I1 = = 20 and VL1 = j4 × 20 = j80
3 + 4j
= 10 × VL1 = 10 × j80 = 800 j = 800 ∠ 90°
Ans. (b)
VTh
82. Consider a delta connection of resistors and its
equivalent star connection as shown below. If all Ans. (c)
elements of the delta connection are scaled by a 84. Three capacitors C1, C2 and C3, whose values are
factor k, k > 0, the elements of the corresponding 10 mF, 5 mF and 2 mF, respectively, have break-
star equivalent will be scaled by a factor of down voltages of 10 V, 5 V and 2 V, respectively.
For the interconnection shown, the maximum safe
voltage in Volts that can be applied across the
Ra RC RB combination and the corresponding total charge in
mC stored in the effective capacitance across the
Rb Rc RA terminals are, respectively,

C2 C3

(a)  k2 (b)  k
(c)  1/k (d)  k
(GATE 2013: 1 Mark) C1

Solution:  We have (a)  2.8 and 36 (b)  7 and 119


(c)  2.8 and 32 (d)  7 and 80
Ra Rb (GATE 2013: 2 Marks)
RC =
Ra + Rb + Rc
Solution:  Given that C1 = 10 µF, C2 = 5 µF, C3 = 2 µF,
µ µ C = 2 µF,
C13 = 10 µF, C2 = 5 µF, C3V1==2 10
µF,V, V2 = 5 V, V3 = 2 V.
Let k be a scalar factor, then C 1 = 10 F, C2 = 5 F,
V1 = 10 V, V2 = 5 V, V3 = 2 V.
(Ra k)(Rbk) V1 = 10 V, V2 = 5 V, V3 = 2 V.
RC¢ = = RCk
Ra k + Rbk + Rc k Charge stored in C2 , Q2 = C2V2 = 5 × 5 = 25 µ C
Ans. (b) Charge stored in C , Q = C V = 2 × 2 = 4 µC
3 3 3 3
83. In the circuit shown below, if the source voltage
VS = 100Ð53.13° V then the Thevenin’s equiva- Charge stored in C2 and C3 will be the same.
lent voltage in Volts as seen by the load resistance Therefore, Q2 = Q3 = 4 mC
RL is
Q2 4 µC 4 µC
V2 = = = 0.8 V and V3 = =2 V
C2 5 µF 2 µF
3Ω j4 Ω j6 Ω 5Ω
+ −
Total voltage VT = V2 + V3 = 0.8 + 2 = 2.8 V.
VL1
+ +
Therefore,
RL = 10 Ω
− − 10 V
j40I2
L1
C1 = 2.8 × 10 = 28 µC
I1 I2

(a)  100Ð90° (b)  800Ð0° QEff = 28 + 4 = 32 m C


(c)  800Ð90° (d)  100Ð60°
Ans. (c)
(GATE 2013: 2 Marks)

Chapter 1 Solved Question Paper.indd 168 3/19/2016 4:32:29 PM


CHAPTER 2

ELECTROMAGNETIC FIELDS

This chapter covers topics related electromagnetic fields, point charges is directly proportional to the product of
which is concerned with electric and magnetic phenom- the magnitude of the charges and inversely proportional
ena. These include important laws and their applications, to the square of the distance between them, that is,
such as Coulomb’s law, Gauss’s law, Biot-Savart’s law,
Q1Q2
Ampere’s law, Faraday’s law, etc., concepts of electric field F =k
intensity, electric flux density, electric field and potential r2
due to point, line, plane and spherical charge distribu- where proportionality constant k is called electrostatic
tions, effect of dielectric medium, capacitance of simple constant and has the value: 9.0 × 109 Nm2C−2 in free
configurations, divergence and curl, Lorentz force, self space; Q1 and Q2 are charges in Coulomb and r is the
and mutual inductance, magnetomotive force, reluctance, distance between the two charges in meter.
magnetic circuits along with the relevant calculations.

2.2  ELECTRIC FIELD INTENSITY


2.1  COULOMB’S LAW
The electric field intensity is the force on a unit positive
Coulomb’s law, or Coulomb’s inverse-square law is a law charge placed at the point in the field. The strength of
of physics describing the force of attraction or repulsion the electric field is measured using a quantity called elec-
between electrically charged particles. According to this tric field intensity. The greater the electric field inten-
law, the magnitude of the electrostatic force between two sity, the stronger the field. In a uniform field, the electric

Chapter 2.indd 169 3/19/2016 5:11:16 PM


170     Chapter 2:  ELECTROMAGNETIC FIELDS 

field intensity is constant (the same at any point in the D


D = e0erE and E = (in presence of dielectric)
field); while in a radial field, the electric field inten- e 0e r
sity decreases as the distance from the central charge
increases. Thus, So, it is apparent that a formulas derived for E from
F Coulomb’s law can be used in calculating D, except that
E= we have to multiply those formulas by e0,
Q
For Q > 0, the electric field intensity is measured in 2.4  DIVERGENCE AND CURL
Newton per Coulomb or volt per meter. The electric field
intensity at point R due to Q point charge located at
distance r is readily obtained from: 2.4.1 Divergence

Q(r − r) 
a r =
Q
3
E= (2.1) In vector calculus, divergence is a vector operator that
4pe 0 r 2 4pe 0 r − r measures the magnitude of a vector field’s source or sink
at a given point, in terms of a signed scalar. More tech-
where e0 is permittivity of vacuum, in presence of dielec- nically, the divergence represents the volume density of
tric e = e0er    is used where er is relative permittivity. the outward flux of a vector field from an infinitesimal
Electric field strength is a vector quantity. It has both volume around a given point.
magnitude and direction. The magnitude of the electric
field intensity is defined in terms of how it is measured. 2.4.1.1 Definition
The electric field does not depend on the quantity of
charge on the test signal. A divergence of A at a given point P is the outward flux
per unit volume at the volume shrink about P. Hence,

2.3  ELECTRIC FLUX DENSITY div A = ∇ ⋅ A = lim


∫ S A ⋅ ds
∆V → 0 ∆V
where ΔV is the volume enclosed by the closed surface
Electric flux density is the measure of the intensity of an S in which P is located. In Cartesian coordinate system,
electric field generated by a free electric charge, correspond- the divergence is represented as:
ing to the number of electric field lines passing through a
given area. It is also called electric displacement. Electric ∂A x ∂A y ∂A z
∇⋅A = + +  (2.2)
flux is the normal (perpendicular) flux per unit area. ∂x ∂y ∂z
If a flux f is passed through an area A m2 normal to
Similarly, for the cylindrical system:
the area, then the flux density (denoted by D) is:
f ¶Ar A r 1 ¶Aq ¶A z
D= Ñ×A = + + +
A ¶r r r ¶q ¶z
If an electric charge is placed in the centre of a sphere
or virtual sphere then the electric flux on the surface of For spherical system,

1 ¶ 2 1 ¶Af ¶
the sphere is:
Ñ×A = (A sin q )
1
f
q q
Q (r Ar ) + +
D= = r 2 ¶r r sin q ¶ f r sin q ¶
A 4p r2
where r is the radius of the sphere.
2.4.1.2 Properties
The SI unit of electric flux is coulomb per meter square.
  1.  It produces a scalar field (i.e., because scalar
product is involved).
2.3.1 Relation between Electric Flux Density   2.  ∇ ⋅ (A + B) = ∇ ⋅ A + ∇ ⋅ B
and Electric Field Intensity
  3.  ∇ ⋅ (VA) = V ∇ ⋅ A + A ⋅ ∇V
If we compare the following formulas,
Q Q 2.4.2 Curl
D= and E =
4pr 2
4pe 0 r 2
In vector calculus, curl is a vector operator that mea-
we have, sures the amount of rotation or angular momentum of
D
D = e0E or E = (in vacuum) a vector about a given point. Hence, if curl of a vector
e0 exists, then the vector field is said to be rotational.

Chapter 2.indd 170 3/19/2016 5:11:29 PM


2.7  ELECTRIC FIELD AND POTENTIAL     171

2.4.2.1 Definition Gauss’s law follows Coulomb’s laws and the superposition
theorem. It constitutes one of the fundamental laws of elec-
If V(x, y, z) is a differentiable vector field, then, curl V or tromagnetism. Gauss’s law states that the total electric flux
rotation of V is written as ∇ × V and is given by f through any closed surface is equal to the total charge

  ∂ ∂  ∂  enclosed by that surface. Thus,
∇ × V =  i + j + k  × (V1 i + V2 j + V3 k ) f = Qenc
 ∂x ∂y ∂z 
i j k
 = ∂ / ∂x ∂ / ∂y ∂ / ∂z (2.3)
that is, f = ∫ df = ∫ D ⋅ ds = total charge enclosed
S S
V1 V2 V3 Q = ∫ r ⋅ dV
 ∂V ∂V   ∂V ∂V   ∂V ∂V 
=  3 − 2  i +  1 − 3  j +  2 − 1  k
 ∂y ∂z   ∂z ∂x   ∂x ∂y 
Q= ∫ D ⋅ ds = ∫ r V dV  (2.4)
S V
Using divergence theorem in Eq. (2.4), we have
2.4.2.2 Properties

  1.  ∇ × (∇f) = 0, that is, the curl of gradient of a


∫ D ⋅ ds = ∫ ∇ ⋅ r V dV  (2.5)
V

scalar field is zero.
Comparing the two volume integrals, we get
  2.  Ñ × (Ñ × A) = 0, that is, the divergence of curl of a
vector field is zero. rV = ∇ ⋅ D  (2.6)
  
  3.  ∇ × (∇ × A) = ∇(∇ ⋅ A) − ∇ A
2
Equation (2.6) is the first of the four Maxwell’s equa-
tions to be derived. It states that the volume charge
density is the same as the divergence of the electric flux
2.5  DIVERGENCE AND STOKES’ density. Equations (2.4) and (2.6) are basically stating
THEOREMS Gauss’s law in different ways. Equation (2.4) is the inte-
gral form whereas Eq. (2.6) is the differential or point
form of the Gauss’s law. Gauss’s law is an alternative
The divergence theorem (also known as Gauss statement of Coulomb’s law.
divergence theorem or Green’s theorem), states that if
V is the volume bounded by a closed surface S, and if A Proper application of the divergence theorem to
is a vector function of position with continuous deriva- Coulomb’s law results in Gauss law. Gauss’s law pro-
vides an easy means of finding E or D for symmetrical
tives, then    
òòò Ñ × AdV = òò A × nds = òò A × ds
charges distribution such as point charges on infinite
line, etc.
where n is the positive normal to S.
Just like divergence theorem, we have Stokes’
2.7  ELECTRIC FIELD AND POTENTIAL
t­ heorem which states that if S is an open two-sided
surface, bounded by a closed curve C, then
     
∫ A ⋅ dr = ∫∫ (∇ × A) ⋅ nds = ∫∫ (∇ × A) ⋅ ds 2.7.1 Due to Point Charge
where integral dr gives the perimeter of the total surface
S and A has continuous derivatives. It can be noticed Suppose a point charge Q is located at the origin
while comparing the divergence and Stokes’ theorems (Fig. 2.1). To determine E, we can use the Gauss’s law
that Stokes’ theorem converts a line integral into a sur- and the formula given by:
face integral whereas divergence theorem converts a sur- D = e 0E  (2.7)
face integral into a volume integral.
Note: Using Stokes’ theorem, it can be proven that z
divergence of a curl of a vector is zero. P D
r
2.6  GAUSS’ LAW Q
y

The Gauss’ law is used to find electric field when the charge
x
is continuously distributed within an object (virtual) with
symmetrical geometry, such as sphere, cylinder, or plane. Figure 2.1 |   Point charge located at the origin.

Chapter 2.indd 171 3/19/2016 5:11:43 PM


172     Chapter 2:  ELECTROMAGNETIC FIELDS 

It is easy to see that choosing a spherical surface 


 

xy plane or z = 0 plane. To evaluate D /E at point P, we
c­ ontaining P will satisfy symmetry conditions. So, by
choose rectangular box and cut it symmetrically (Fig 2.3)
applying Gauss’s law:
Q= ∫ D ⋅ ds = Dr ∫ ds z rS C/m2
S S

Q = Dr 4pr 2

a r 
Q y
D= (2.8)
4p r 2
From Eqs. (2.7) and (2.8), we have Gaussian
x surface
a r
Q
e0E =
4p r 2 Figure 2.3 |   Electric field due to infinite sheet of
charge.
Q
a r 

Since D is normal to the sheet, D = Dz a z . Applying
E=
4pe 0 r 2
Gauss theorem, we have
2.7.2 Due to Line Charge  
r S ∫ ds = q = ∫ D ⋅ ds = Dz  ∫ ds + ∫ ds

Suppose the infinite line of uniform charge with density S S  top bottom 
r C/m lies along z-axis (Fig. 2.2). To determine D at 
 
L Note that D × ds evaluated on the sides of the box is
point P, we can choose a cylindrical surface containing P
zero. So, we have
to satisfy the symmetry condition.
rS A = Dz (A + A)
rLL = Q = ∫ D ⋅ ds = DP ∫ ds r
S S   E = D = S a z
e0 2e 0
Q = DP2pr L
where ∫
 ds = 2pr L is the surface area of the Gaussian
2.7.4 Due to Uniformly Charged Sphere
surface. Note that ∫ D ⋅ ds evaluated on the top and Now consider a sphere of radius a with uniform charge
bottom surfaces of the cylinder is zero. Since D has no density r0 C/m2 (Fig. 2.4).
z-component, it means that D is tangential to those
­surfaces. Thus,
rL
D= a r r a
2pr r0 r
rL
a r
D a r
E= =
e0 2pre 0
(a) (b)
Figure 2.4 |   Electric field due to uniformly charged
Line
charge
sphere.

Let a be the radius and r be the share containing


P
D uniform charge r0. Then Fig. 2.4(a) depicts the condition
L a > r and for r ≥ a, the Gaussian surface is as shown in
the Fig. 2.3(b). So, the charge enclosed by the surface is
the entire charge in this case.
2p p a

Figure 2.2 |   Electric field due to infinite line charge.


Qenc = ∫ r dV = r0 ∫ dV ≡
V ∫ = r0 ∫ ∫ ∫ r 2 sin q dr dq df
V V 0 0 r =0
4
2.7.3 Due to a Plane (or Infinite Sheet) of Charge Qenc = r0 pa3  (2.9)
3
The electric flux is
Field due to infinite sheet of charge, that is, a plane can
f= ∫ D ⋅ ds = Dr 4p r
2
be determined as follows. Consider an infinite sheet of
charge with surface charge density rS (C/m2) placed on S

Chapter 2.indd 172 3/19/2016 5:12:07 PM


2.8  CAPACITANCE     173

From Gauss's law, φ = Qenc, so The conductors are maintained at a potential differ-
ence V given by:
4
Dr × 4p r2 = p a3 r0 1
3
V = V1 − V2 = −∫ E ⋅ dl
a3
D = 2 r0a r (where r ≥ a) 2
3r
r where E is the electric field existing between the con-
and D = r0 a r   (where 0 < r ≤ a) ductors, and conductor 1 is assumed to carry a positive
3
charge. Now, we define that the capacitance C of the
2.7.5 Due to Different Distributions capacitor is the ratio of the magnitude of the charge

 on one of the plates to the potential difference between
To obtain E, that is, field due to the different distribu- them, that is,
tion we use Gauss law given by,
 Q e0 ∫ E ⋅ ds
∫ D ⋅ ds = Qenc  (2.10) C= =
S
V ∫ E ⋅ dl
For Eq. (2.10), E is calculated by using
D = e 0E 
Now, we can obtain C for any given two-conductor
(2.11)

  capacitance by the following two methods:
as ò D × ds = Qenc
f=   1.  Assuming Q and determining V in terms of Q.
S

 
  2.  Assuming V and determining Q in terms of V.
ò E × ds = Q
e0 
D 2.8.1 Expression for a Parallel Plate Capacitor
2.7.6 Effect of Dielectric Medium on Electric Field For a parallel plate capacitor,
Consider the electric field due to point charge given by

 rS =
Q
a r
Q
E=
4pe 0 r 2
S

Due to presence of the dielectric, the permittivity of free 1 d


 −Q 
space (i.e., air) e0 is replaced by e0er where er is the nor- V = −∫ E ⋅ dl = −∫  a x dx
malised dielectric constant of the dielectric. 2 0 0
e 

a r
Q
E= Qd 
4pe 0 e r r2 V = (2.12)
eA

2.8  CAPACITANCE Q eA
C= =
V d
Generally speaking, to have a capacitance we must
have two (or more) conductors carrying equal but oppo- where A is the cross-sectional area of capacitor plates.
site charges. This implies that all the flux lines leaving
one conductor must necessarily terminate at the surface
of the other conductor. Now, consider two conductor 2.8.2 Series and Parallel Combination of
capacitors shown in Fig. 2.5. Capacitors
E
−− + When capacitors are connected in series as shown in
− −
− Q− + + Fig. 2.6(a), the equivalent capacitance is given by
− − Q
− − + +
−− + + Ceq =
1
+
1
+
1
C1 C2 C3

− + When capacitors are connected in parallel as shown in


Fig. 2.6(b), the equivalent capacitance is given by
V
Figure 2.5 |   Two conductor capacitors.
Ceq = C1 + C2 + C3

Chapter 2.indd 173 3/19/2016 5:12:27 PM


174     Chapter 2:  ELECTROMAGNETIC FIELDS 

Terminal at any point due to a steady current in an infinitely long


straight wire is directly proportional to the current and
+q C1 inversely proportional to the distance from point to wire.
V1 
−q 
 I dL × a r
dH =
4p r 2
+ +q
B V2 Note that the direction of r must be from the current
− V
−q C2 element to the point at which dH is to be determined.

+q
2.11  AMPERE’S LAW
V3 C3
−q
The Ampere circuit law states that the line integral of H
Terminal around a closed path is the same as the net current Ienc
(a) enclosed by the path. In other words, the circulation of
H equals Ienc, that is,
Terminal

+ +q3 +q2 +q1 ∫ H ⋅ dl = Ienc


V V3 V2 V1

B
−q3 −q2 −q1
C3 C2 C1
2.12  FARADAY’S LAW AND LENZ’S
Terminal LAW OF ELECTROMAGNETIC
(b) INDUCTION
Figure 2.6 |   (a) Series and (b) parallel combina-
tion of capacitors. Faraday’s laws explain the relationship between electric
circuit and magnetic field. This law is the basic working
principle of the most of the electric motors, generators,
2.9  MAGNETIC FLUX AND transformers, inductors, etc.
MAGNETIC FIELD
2.12.1 Faraday’s First Law
The total number of magnetic field lines crossing through
the surface is known as the magnetic flux of the any sur- It states that whenever a conductor is placed in a vary-

 ing magnetic field, an emf gets induced across the con-
face held in magnetic field H . It is expressed as

 
ductors (called an induced emf) and if the conductor is
f B = ∫ H ⋅ dA a closed circuit then induced current flows through it.


The magnetic field H is uniform and perpendicular 2.12.2 Faraday’s Second Law
to the area. The SI units of magnetic flux is T m2 or
weber (Wb). According to this law, the magnitude of induced elec-
The number of magnetic lines of force per unit area tromagnetic field (emf) is equal to the rate of change of

 flux linked with the coil. The flux linkage is the product
is called the magnetic flux density ( B ), and this
of number of turns and the flux associated with the
represents the strength of the field. coil. The emf induced in a closed circuit is proportional
to the time rate of change of magnetic flux linking the
2.10  BIOT—SAVART LAW circuit.
df
e∝−
dt
The Biot—Savart law relates magnetic fields to the cur-
rents, which are their sources. In a manner similar to where e is the induced emf, f is the magnetic flux. The
how Coulomb’s law relates electric fields to the point negative sign indicates that the direction of the induced
charges, which are their sources. The magnetic intensity emf is opposite to the cause that produces it.

Chapter 2.indd 174 3/19/2016 5:12:36 PM


2.16  INDUCTANCE     175

2.12.3 Derivation of Faraday’s Law Table 2.1 |   Maxwell's equations

Consider the conductor is moving in magnetic field. Differential Integral Form


Then, flux linked with the coil at initial position of the Form
conductor = Nf1 Wb where N is number of turns of 
 


 ∂D  
   ∂D   
∇×H = ∫ H ⋅ dl = ∫  dt + J  ds
the coil.
+J
Flux linked with the coil at final position = Nf2 Wb dt
Change in flux = N(f1 − f2) 
 


 ∂B 
  ∂B 
∇×E = −
Let f1 = f2 = f , then change in flux = Nf
dt ∫ E ⋅ dl = −∫ dt ds
Nf
and the rate of change =
t 
 
 
So, the derivative of rate of change of flux linkage ∇⋅D = r ∫ D ⋅ ds = ∫ r V dV
df
=N and by the Faraday’s law, this is equal to the
dt 
 
 
induced emf. So, ∇⋅B = 0 ò B × ds = 0
 df 
E = N   volt The differential and integral forms of the equations
 dt  can be equated using Stokes’ theorem for the first two
equations and using divergence theorem for the last two
2.12.4 Lenz’s Law equations.

It states that when an emf is induced according to


Faraday’s law, the polarity (direction) of that induced 2.14.1 Magnetic Boundary Conditions
emf is such that it opposes the cause of its production.


Thus, by considering Lenz’s law, The magnetic boundary conditions for the behaviour of
df
magnetic field strength (H ) and magnetic flux density


E = −N volts (B ) are as follows:
dt
The negative sign shows that the direction of the induced   1.  The normal component of B is continuous across
emf and the direction of change in magnetic field have the interface

 

opposite sign. B n1 = B n 2

  2.  The tangential component of H is continuous


2.13  LORENTZ FORCE across the interface.

 

H t1 = H t2
The force on a charged particle moving in electric and
magnetic field is equal to the particle’s charge times
2.15  POYNTING VECTOR
the sum of the electric field and the cross product
of the particle’s velocity with the magnitude of flux
density. Poynting vector is the instantaneous rate of electromag-

 
  
F = q[ E + (v × H )] netic energy flow per unit area at a point. It is given by
    
P = E ×H
where F is force, E is electric field, v is velocity, H is 
 

magnetic field. where E and H are the electric and magnetic fields,
respectively. As the direction of energy flow is the direc-
tion of propagation, the Poynting vector specifies this
2.14  MAXWELL’S EQUATIONS direction of propagation.

Maxwell’s equations are four equations relating time 2.16  INDUCTANCE


varying electric and magnetic fields. These equations
form the basis of electromagnetic theory and the propa-
A circuit (or conducting path) carrying current I ­produces
gation of electromagnetic waves. They are mathemati-
cally represented as listed in Table 2.1. a magnetic field B that causes a flux f = ∫ B ⋅ ds to

Chapter 2.indd 175 3/19/2016 5:13:02 PM


176     Chapter 2:  ELECTROMAGNETIC FIELDS 

pass through each turn of the circuit as shown. If it has So, the emf introduced is
N identical turns, then the flux linkage y is DI
D = −L
y = Nf  (2.13) dt
and also, if the medium surrounding the circuit is where L is constant and known as self-inductance. L depends
linear, the flux linkage y is proportional to the current on the physical characteristics of the coil, and is given by
I producing it, that is: m0 AN 2
y µI L=
l
or y = LI where m0 is magnetic permeability, A is the area, N is the
where L is the proportionality constant called the induc- number of turns and L is the length. The unit of L is Henry.
tance of the circuit. The inductance L is the property of the
physical arrangement of the circuit. A circuit or part of the 2.16.3 Mutual Inductance
circuit that has inductor is called an inductor circuit.
Mutual inductance occurs when the change in the current
The energy stored by an inductor is
in one inductor induces a voltage in the other nearby induc-
1 2 tor. Consider instead of having a single circuit, we have two
WL = LI  (2.14)
2 circuits carrying currents I1 and I2 as shown in Fig. 2.8.

2.16.1 Series and Parallel Combination of W12


Inductors ×
f22
W21
When in inductors are connected in series, the equiva- f11
lent inductance is given by
Leq = L1 + L2 + L3 + M +
When in inductors are connected in parallel, the equiva-
lent inductance is given by I2
L
1 1 1
Leq = + +
L1 L2 L3 − −
Figure 2.8 |   Two circuits carrying currents I1 and I2.
2.16.2 Self Inductance A magnetic interaction exists between the circuits. Four
component fluxes f11, f12, f21, and f22 are produced and
Self inductance is defined as the magnetic induction of a
voltage in a current carrying wire when the current in a f12 = ò B × ds
s1
wire changes (Fig. 2.7).
We define the mutual inductance M21 as the ratio of
Magnetic line the flux linkage l12 = N1f12 on circuit I to current I21,
of force that is,
l12 Nf
M12 = = 1 12
I2 I2

Mutual inductance M21 is given by

N 2f 21
Coil loop M 21 =
I1

It is also given by
M21 = N1 N2 P21
where P21 is the permeance of the space occupied by the
flux and N1 and N2 are number of turns in coil 1 and 2,
respectively.
Figure 2.7 |   Self inductance. Mutual inductance between two coils can also be rep-
Inductance is given by voltage induced as resented as
m mNN A
df M = 0 r 1 2
VL = −N L
dt

Chapter 2.indd 176 3/19/2016 5:13:18 PM


2.17  SELF AND MUTUAL INDUCTANCE FOR SIMPLE CONFIGURATIONS     177

where m0 is the permeability of free space, mr is the rela- neglecting the spreading of the magnetic field lines near
tive permeability of the soft iron core, N is the number the ends of the solenoid.
of coil turns, A is the cross-sectional area in m2 and L is As n is a number per unit length, inductance can be
the length of coil is meter. written as a product of the permeability constant m0
Note: The emf induced in either coil is proportional and length. This means that m0 can be expressed in the
to the rate of change of current in the other coil, it can unit henry per meter:
m0 = 4p × 10−7 T ⋅ m/A
be concluded that M21 = M12 = M, however, this conclu-
sion is in no way obvious.
The mutual inductance is related to self inductance = 4p × 10−7 H/m.
by the following expression For a given current, the magnetic flux is now much
M = K L1L2 greater due to the increase in the field originating from
the magnetization of the ferromagnetic material. For
where K is the coupling coefficient. example, if the material has a magnetic permeability of
500 m0 , the inductance would increase by a factor of 500.
2.17  SELF AND MUTUAL INDUCTANCE
FOR SIMPLE CONFIGURATIONS 2.17.1.2 Mutual Inductance

Consider two solenoids C1 and C2 wound over a soft iron


2.17.1 Inductance of Solenoid core and having N1 and N2 turns respectively, as shown
in Fig. 2.9.
2.17.1.1 Self Inductance
I1 C1 (coil 1)
e
Consider a long solenoid of cross-sectional area A. We
need to find the inductance per unit length near its Iron core
middle. For this we must first calculate the flux link- C2 (coil 2)
age set up by a given current in the solenoid windings.
Suppose we have to find this at length l near the middle Figure 2.9 |   Two closely wound solenoids.
of this solenoid, then the flux linkage for this section of
Current I1 flows through solenoid C1 and due to this a
the solenoid is
magnetic field is created around solenoid C2. Due to the
Nf B = (nl)(BA), net change in magnetic flux inside solenoid C2, an emf is
in which n is the number of turns per unit length of the induced in the coil. The magnetic field B1 at any point
solenoid and B is the magnitude of the magnetic field inside solenoid C1 due to current I1 is
within the solenoid. The magnitude B is given by m0 N1I1
B1 =  (2.17)
B = m0 In l
The magnetic flux linked with each turn of solenoid C2
is = B1 × area of each turn = B1 A.
and so we have

Nf B (nl)(BA) (nl)( m0 In)(A) Total magnetic flux linked with solenoid C2 having
L= = =  (2.15)
I I I N2 turns is
= m0 n2lA f2 = B1A × N2  (2.18)

Thus, the inductance per unit length near the centre of Substituting Eq. (2.18) in Eq. (2.17), we get
a long solenoid is m0 N1I1
f2 = AN2  (2.19)
L
= m0 n2 A  (2.16) l
l Let M12 be the coefficient of mutual inductance for the
Inductance, like capacitance, depends only on the geom- solenoids C1 and C2. The magnetic flux linked with sole-
etry of the device, that is, the square of the number of noid C2 should be directly proportional to the current
turns per unit length in this case. If we triple the number flowing through C1 and so we get
of turns (N), then the flux (f B = BA = m0 InA) is also f2 = M12 I1  (2.20)
tripled through each turn, multiplying the flux linkage
Nf B and thus the inductance L by a factor of 9. On comparing Eqs. (2.18), (2.19), and (2.20) we get
If the solenoid is very much longer than its radius, then m0 N1N2 A
Eq. (2.15) gives its inductance to a good approximation, M12 =
l

Chapter 2.indd 177 3/19/2016 5:13:35 PM


178     Chapter 2:  ELECTROMAGNETIC FIELDS 

Thus the above expression gives the value of coefficient m0l  b 


L= ln  
2p  a 
of mutual inductance between two solenoids.

2.17.2 Inductance of a Coaxial Cable 2.17.2.2 Mutual Inductance


2.17.2.1 Self Inductance Figure 2.11 shows two circular close-packed coils, the
smaller (radius R2, with N2 turns) being coaxial with the
Consider a long coaxial cable (Fig. 2.10) consisting of larger (radius R1, with N1 turns) and in the same plane.
two thin-walled concentric conducting cylinders with
radii a and b. The inner cylinder carries a steady cur-
rent I, and the outer cylinder provides the return path
for that current. The current sets up a magnetic field R1
between the two cylinders. R2
We need to calculate the energy stored in the magnetic
field for a length l of the cable.

Outer
cylinder
dr
r b
Thin I1
a
+ −
cylindrical
shell
Figure 2.11 |   Small coil is located at the centre of a
Inner large coil.
cylinder
The mutual inductance of the coils can be determined
Figure 2.10 |   Cross-section of a long coaxial cable. by flowing current I1 through the large coil. First we
derive an expression for the mutual inductance M for this
We apply Ampere’s law, using a circular path of inte-
arrangement of the two coils, assuming that R1 < R2. The
gration with radius r such that a < r < b. The only
mutual inductance M for these coils is the ratio of the flux
current enclosed by this path is current i on the inner
linkage (Nf) through one coil to the current I in the other
cylinder. Thus, we can write Ampere’s law as
coil, which produces that flux linkage. For this we assume

 
ò × ds = m0 I 
B (2.21) that currents exist in the coils and then calculate the flux
linkage in one of the coils.


Because of the circular symmetry, at all points along The magnetic field through the larger coil due to the
the circular path, B is tangent to the path and has the smaller coil is non-uniform in both magnitude and direc-
same magnitude B. Let us take the direction of inte- tion; so the flux is difficult to calculate. However, the

gration along the path as the direction of the magnetic
 smaller coil is small enough for us to assume that the
field around the path. Then we can replace B × ds with magnetic field through it due to the larger coil is approx-
B dscos0° = B ds and then move magnitude B in front imately uniform. Thus, the flux through the smaller coil
of the integration symbol. The integral that remains is due to the larger coil is also approximately uniform.
ds, which just gives the circumference 2p/r of the path. Hence, M is found as
Thus, Eq. (2.21) simplifies to N 2f21
M =  (2.22)
B(2p r) = m0 I I1
The flux f21 through each turn of the smaller coil is,
Now, the the stored energy per unit length of the cable is from fB = BA,
ò dWB = òB 2 m0 m0 I 2 m0 I 2 æ b ö f21 = B1A2
2 b

ò r 4p ln çè a ÷ø
WB dr
= = =
l l l 4p a where B1 is the magnitude of the magnetic field at
points within the smaller coil due to the larger coil and
This suggests a very simple way to calculate the A2 (= pR 22 ) is the area enclosed by the turns. Thus, the
self-inductance of the cable. The energy can also be flux linkage in the smaller coil (with its N2 turns) is
written as (1/2)LI2. Comparing the two expressions,
we get N2f21 = N2 B1A2  (2.23)

Chapter 2.indd 178 3/19/2016 5:13:48 PM


IMPORTANT FORMULAS     179

To find B1 at points within the smaller coil, we can use 2.18  MAGNETIC CIRCUIT AND
Biot−Savart’s law, MAGNETOMOTIVE FORCE
m0 IR2
B(z) =
2(R2 + z 2 )3/2 Magnetic circuit is the combination of two or more
closed-loop paths containing the magnetic flux. The
with z set to 0 because the smaller coil is in the plane of magnetomotive force (mmf) is given by
the larger coil. Thus, the larger coil (with its N1 turns)
produces a total magnetic field of magnitude F = ∫ H ⋅ dl
m0 I1
Its unit is given by ampere-turn. The magnetomotive
B1 = N1  (2.24) force is also defined as
F = fR
2R1

at points within the smaller coil. Substituting Eq. (2.24) where f is the magnetic flux and R is the reluctance of
for B1 and pR22 for A2 in Eq. (2.23) yields the circuit. Magnetomotive force is analogous to electro-
motive force since it is the cause of magnetic flux in a
pm0 N1N 2R22 I1
N 2f21 =
magnetic circuit.
2R1
2.18.1 Reluctance
Substituting this result into Eq. (2.22), we find
In a DC field, the reluctance is the ratio of the magneto-
Nf pm N N R 2 motive force (mmf) in a magnetic circuit to the magnetic
M = 2 21 = 0 1 2 2  (2.25)
I1 2R1 flux in the circuit. It is expressed as
F
Consider the situation where we reverse the roles of R=
the two coils, that is, if we produce a current I2 in the f
smaller coil, then M can be calculated as
where R is reluctance in ampere per turn, F is the mag-
netomotive force (mmf) and f is the magnetic flux in
N1f12 Weber.
M=
I2 On the basis of the physical dimensions of the object,
reluctance is given by
The calculation of f12 (the non-uniform flux of the
1 l
smaller coil’s magnetic field encompassed by the larger R= (l/A) =
coil) is not simple. If we were to do the calculation m mA
numerically using a computer, we would find M to be
2.3 mH, as above. This emphasises that the equation where l is the length, A is the cross-sectional area and m
(M21 = M12 = M) is not obvious. is magnetic permeability.

IMPORTANT FORMULAS

Q1Q2
1. Coulomb’s law: F = k (a) Cartesian coordinate system:
r2
F ∂Ax ∂Ay ∂Az
2. Electric field intensity: E = ;   ∇⋅A = + +
Q ∂x ∂y ∂z

QQ Q((rr −
Q − rr))
EE= = aarr =
=  (b) Cylindrical system:

pe00rr22
44pe pe00½½rr −
44pe − rr½½33
11 ∂∂ ∂∂AA AArr 11 ∂∂AAqq ∂∂AAzz
∇∇⋅⋅AA= ((rrAArr)) +
+ rr +
f
  = + + + +
+
3. Electric flux density: D = =
Q
; D = e0erE rr ∂∂yy ∂∂rr rr rr ∂∂qq ∂∂zz
A 4pr2
ò A × ds
(c) Spherical system:
1 ¶ 2 1 ¶Aq ¶
Ñ × A = (A sin q )
1
4. Divergence: div A = Ñ × A = lim S
r sin q ¶q q
(r Ar) + +
DV ® 0 ∆V r 2 ¶r r sin q ¶q

Chapter 2.indd 179 3/19/2016 5:14:07 PM


180     Chapter 2:  ELECTROMAGNETIC FIELDS 

1 ¶ 2 1 ¶Aq ¶ (b)  Series and parallel combination of capacitors


(A sin q )
1
r sin q ¶q q
A= (r Ar) + +
r 2 ¶r r sin q ¶q Ceq =
1
+
1
+
1
(series)
  C1 C2 C3
(d)  Properties: Ceq = C1 + C2 + C3 (parallel)
∇ ⋅ (A + B) = ∇ ⋅ A + ∇ ⋅ B
•  
 
•  Ñ × (VA) = V Ñ × A + A × ÑV 11. Magnetic flux: f B = ò H × dA


 æ ¶V 3 ¶V 2 ö 
 IdL × aR
 æ ¶ ¶ ö æ ¶ ¶ ö
- 3 ÷ j + ç 2 − 1 ÷ k
V V V V
5. Curl: Ñ × V = ç − 12. Biot—Savart’s law: dH =
÷i +ç
1
4p R 2
è ¶y ¶z ø è ¶z ¶x ø è ¶x ¶y ø
ö  æ ¶V1 ¶V3 ö  æ ¶V2 ¶V1 ö 
- −
13. Ampere’s law: ∫ H ⋅ dl = Ienc
÷i +ç ÷j + ç ÷k
ø è ¶z ¶x ø è ¶x ¶y ø df  df 
14. Faraday’s law: E = − or E = N   volts
Properties: dt  dt 

(a) ∇ × (∇f) = 0, that is, the curl of gradient of a df


15. Lenz’s law: E = −N volts
scalar field is zero. dt
 
 
  
(b) Ñ × (Ñ × A) = 0 that is, the divergence of curl of 16. Lorentz force: F = q[E + (v × H )]
a vector field is zero. 17. Maxwell’s equations:
  
(c) Ñ
 × (Ñ × A ) = Ñ(Ñ × A ) − Ñ2 A
Differential form Integral form

 

     ¶D  
   ∂D  
6. Divergence theorem: òòò Ñ × AdV = òò A × nds = òò A × ds Ñ × H = +J ∫ H ⋅ dl = ∫  dt + ∂J  d s
    dt  
Ñ × AdV = òò A × nds = òò A × ds 
 

→ → → → →   ¶B 
  ¶B 
7. Stokes’ theorem: ∫ A⋅ d r = ∫∫ (∇ × A) ⋅ nds = ∫∫ (∇ × A) ⋅ d sÑ × E = − ò E × dl= −ò dt ds

 
dt
→ → → → 

r = ∫∫ (∇ × A) ⋅ nds = ∫∫ (∇ × A) ⋅ d s Ñ×D = r ∫ D ⋅ ds = ∫ r V dV

 
 
ò df = ò D × ds = total charge
8. Gauss’s law: f =  Ñ×B = 0
∫ B ⋅ ds = 0
S S 
 

enclosed Q = ∫ r ⋅ dV 18. Magnetic boundary conditions: Bn1 = B n 2 and
Ht1 = Ht2
9. Electric field:
 
 
Q
a r 19. Poynting vector: P = E × H
(a) Due to a point charge: E =
4pe 0 r 2
20. Inductance
rL
a r
D
(b) Due to line charge: E = = (a) Flux linkage y = LI
e 0 2p re 0

1 2
r (b) Energy stored by an inductor: WL = LI
= s a z
D
(c) Due to infinite sheet of charge: E = 2
e0 2e 0
(c) Series and parallel combination of inductors
a3
(d) Due to uniformly charged sphere: D = r0 a r Leq = L1 + L2 + L3 (series)
r 3r 2
(where r ≥ a) and D = r0 a r (where 0 < r ≤ a)
3 Leq =
1
+
1
+
1
(parallel)

  L1 L2 L3
ò E × ds = Q
(e) Due to different distributions: e 0 
m0 AN 2
D (d) Self inductance: L =
l
a r
Q
(f) Effect of dielectric medium: E = m mNN A
4pe 0 e r r2 (e) Mutual inductance: M = 0 r 1 2
∫ E ⋅ ds
L
Q e0 
10. Capacitance: C = = (f) Relation between L and M: M = K L1L2
V ∫ E ⋅ dl 21. Self and mutual inductance for simple
(a) Expression for a parallel plate capacitor:
configurations:
Q eA
C= =
V d

Chapter 2.indd 180 3/19/2016 5:14:54 PM


SOLVED EXAMPLES     181

L m NN A 22. Magnetomotive force (mmf):


(a) Solenoid: = m0 n2 A M12 = 0 1 2
∫ H ⋅ dl and F = fR
l l
F =
m l  b
(b) Coaxial cable: L = 0 ln  
 a

2p F l
23. Reluctance : R = ; R=
N 2f 21 pm0 N1N 2R22 f mA
M = =
I1 2R1

SOLVED EXAMPLES

1. Two point charges of magnitude 3 × 10−9 C and Solution:  As emf = v ˙ l ˙ B. Therefore,
−5 × 10−9 C are 2 m apart. Determine the magni-
vlB 8 × 0.5 × 10−3 × 10 40
tude of the force between them and state whether I= = = µA = 20 µA
it is attractive or repulsive. R 100 20
Solution:  We use Coulomb’s law to calculate the Ans. (b)
magnitude of the force.
QQ 3. In Question 2, what is the force exerted on the rod
F = k 12 2 due to interaction of current and magnetic field?
r
(a) +1 nN a x (b) −1 nN a x
It is given that Q1 = 3 × 10−9 C, Q2 = −5 × 10−9 C
and r = 2 m. (c) 10 nN a x (d) −10 nN a x
We know that: 
 

k = 9 × 109 W m2 C −2 Solution:  F = I (l × B) = 20 mA × m × 0.5 × 10−3 (a z ) Wb/m2
10

 

100
F = I (l × B) = 20 mA × m × 0.5 × 10−3 (a z ) Wb/m2
10
3 × 10−9 −5 × 10−9 100
= 1 nN a x
2m Ans. (a)
4. Determine the flux density D at (4, 0, 3) if there
3 × 10−9 × (−5) × 10−9 is a point charge −5p mC at (4, 0, 0) and a line
F = 9 × 109 ×
(2)2 charge 8p mC/m along the y-axis.
= −3.375 × 10−8 N

Solution:  D = D0 + DL
Q(r − r )
Hence, the magnitude of force is −3.375 × 10−8 N. 
Q
D0 = e 0 E = a r =  3
Negative sign indicates an attractive force. 4p r2 4pr − r
2. Consider the following setup. Now, r − r ′ = (4, 0, 3) − (4, 0, 0) = (0, 0, 3). Hence,

−5p × 10−3 (0, 0, 3)


D0 = = −0.138 a z mC
⊗ ⊗ 4p (0, 0, 3)
3
ˆx m/s
R = 20 Ω
v=8a
⊗ l = 10 cm
⊗ ⊗
PL
⊗ B = 0.5 aˆz m Wb / m2 Also, DL = a r
2pr
z
x (4, 0, 3) − (0, 0, 0) (4, 0, 3)
Substituting ar = = and
(4, 0, 3) − (0, 0, 0)
y
5
The rod can freely slide on the conducting rails for r = 5, we get
the given values. 3p
DL = (4a x + 3a z )
What is the induced current I in the resistor? 2p + 25
(a) −20 μA (b) 20 μA DL = 0.24 a x + 0.18 a z mC/m2
(c) 10 μA (d) No current flow D = D0 + DL = 240 a x + 42 a z μC/m2

Chapter 2.indd 181 3/19/2016 5:15:18 PM


182     Chapter 2:  ELECTROMAGNETIC FIELDS 

5. The correct expression for electric flux density is Solution:  The correct expressions for D are
Q f Q
(1) D =   (2) D =   (3) D = Q f
4p r2 A 4pe 0 r2 D= 2; D= ; D = e0erE
(a) 1, 2, 3 (b) 1 only 4p r A
Ans. (c)
(c) 1 and 2 (d) 2 only

PRACTICE EXERCISES

Set 1 (One Mark Questions) →


B = 4 m Wb/m2
A
⊗ ⊗
1. Which among the following is the modified form of
Ampere’s law?



  
 

ò H × ds = I  (b) ∇ ×H = s E + e dt
(a) 
dD 6 cm

 
 20 m/s
⊗ ⊗
dE
(c) ∇ × H = s E + e (d) None of these
dt

 
∫ H ⋅ dl for the following current distribution
B
2. Find 
and contour. Find the emf generated with the end A taken as
reference.
(a) −4.8 mV (b) 19.6 mV
(c) −19.6 mV (d) +4.8 mV

⊗ ⊗ 30 A 5A 19 A 2. A magnetic field H = (2a x + 4a y − 9a z ) Wb/m2
23 A

exists at a point. If a charge moving with a velocity of
v = (a x − 3a y + 5a z ) m/s experiences no net force,
then the electric field must be
where  denotes coming outwards, ⊗ denotes going
inwards. (a) (−7 a x − 19a y − 10 a z ) V/m
(a) −25 A (b) +25 A (b) (7 a x + 19a y + 10 a z ) V/m
(c) −29 A (d) +29 A (c) (−7 a x + 19a y − 10 a z ) V/m
(d) (7 a x − 19a y + 10 a z ) V/m
3. Calculate the electric field at a distance of 2 m
from a Van der Graaff generator carrying a charge
of 1 × 10−6 C. 3. A rectangular coil is surrounded by a uniform field
of magnitude B = 20 Wb/m2. For mr = 1, the mag-
4. Line integral can be transformed into a surface
netostatic energy density Wm in J/m3 is equal to
integral by using
(a) 5 × 108 (b) 15.9 × 107
−7
(a) Divergence theorem (c) 50 × 10 (d) 50 × 10−8
(b) Gauss theorem
4. The total energy associated in a system of four
(c) Stokes’ theorem
identical charges of Q = 3 nC at the corners of 1 m
(d) None of these
on a side, is approximately equal to
5. Four fundamental equations of electromagnetics
(a) 292 nJ (b) 486 nJ
are known as
(c) 438 nJ (d) 1876 nJ
(a) Fleming’s laws (b) Faraday’s laws
5. The incoming solar radiation at a place on the sur-
(c) Lorentz equations (d) Maxwell’s equations
face of the Earth is 1.5 kN/m2. The amplitude of
electric field corresponding to this incident power
Set 2 (Two Marks Questions) is nearly equal to
1. A bar of length 6 cm was moved at 20 m/s in the (a) 958 V/m (b) 1063 V/m
field as shown: (c) 1105 V/m (d) 1240 V/m

Chapter 2.indd 182 3/19/2016 5:15:36 PM


ANSWERS TO PRACTICE EXERCISES     183

6. The volume charge density of a spherical body 7. The value of constant `b ’ so that the vector
 2
of radius b centered at the origin is rV (r,q , f ) = (k / r) C/m
A = (2x − 3y) u x + (y − 2x) u y + (x + bz) u z to be
rV (r,q , f ) = (k / r) C/m (k is constant). The total charge in
2
a solenoid vector field is equal to
Coulomb in the spherical body will be equal to
(a) b = −1 (b) b = −2
(a) kb2/ 2p (b) 2 pkb2
(c) b = −3 (d) b = −4
(c) pkb2 (d) 4 pkb2

ANSWERS TO PRACTICE EXERCISES

Set 1 (One Mark Questions) B2 B2


 1 1
mH 2 = BH =

 
 de
Wm =
2m
=
2 m0 mr
1. (c) ∇ × H = sE + e 2 2
dt
   20 × 20
=
2. (a) The net current enclosed along contour 2 × 4p × 10−7 × 1
= 5 − 30 = −25 A.
50 × 107
1 Q = = 15.9 × 107 J/m3
3. For E =
4 pe 0 d 2
, p

1 × 10−6
4. (c) The total energy associated in a system
= 2.25 × 1015 V m−1
1
E= × equals to
4 pe 0 (2)2
1 n
Wt = W1 + W2 + W3 + W4 = ∑ Qi Vi
Ans. (2.25 × 1015) 2 i =1
4. (c)
⇒ 2Wt = Q1V1 + Q2V2 + Q3V3 + Q4V4  (i)
5. (d)
Because of the symmetry of the system,
Set 2 (Two Marks Questions) 2Wt = 4Q1V1  (ii)

1. (a) We know that emf = vlB and force on electron Q Q Q


   V1 = + +
4 p e 0 r12 4 p e 0 r13 4 p e 0 r14
= qv × B . This force is from end A to end B.
3 × 10−9 1 1 1 
6
Therefore, emf = 20 × × 4 = 4.8 mV .
1000 =
4 p e0  1 + 1 + 2  = 72.998 V
Therefore, more electrons accumulate the end B,  
making it more negative.
Now, from Eq. (ii),
So, emf = −4.8 mV
Wt = 2QV1 = 2 × 3 × 10−9 × 72.98

 
   
  
 
 
2. (a) F = qE + q v × H = 0 ⇒ E = -v ´ H = H ´ v = 437.9 nJ ≈ 438 nJ


  B = (2a x + 4a y − 9a z ) Wb/m2
5. (b) The incoming power density is equal to

 2
  v = (a x − 3a y + 5a z ) m/s P =
E
2h
a x a y a z


E= 2 4 −9 where E is the peak amplitude of the electric field
1 −3 5 and h is the impedance in free space = 120p  Ω.
2
E = (20 − 27)a x − (10 + 9)a y + (−6 − 4)a z V/m E
As given, = 1.5 × 103 W/m2
2h

= (−7a x − 19a y − 10a z ) V/m
So, E = 1.5 × 2 × 120p × 103 W/m2 × Ω
2
3. (b) The magnetostatic energy density (Wm) in
J/m3 is defined by ⇒ E = 1130973.3 = 1063.4 V/m

Chapter 2.indd 183 3/19/2016 5:15:58 PM


184     Chapter 2:  ELECTROMAGNETIC FIELDS 

6. (b) Here we are given spherical coordinates system 7. (c) For any vector field to be a solenoidal field,
(r, q, f). As we know the range of variables is divergence of that vector is zero, that is

0 ≤ r ≤ ∞; 0 ≤ q ≤ p ; 0 ≤ f ≤ 2p for a sphere. ∇ ⋅ A = 0, then
The differential volume dV = r2 sinq dr dq df
æ ¶ ¶ ¶ ö
ç u x + u y + u z ÷ × [(2x − 3y)u x +
è ¶x ¶y ¶z ø
So, total charge,

Q= ∫ rV dV (y − 2x)u y + (x + bz)u z ] = 0
V ¶ ¶ ¶
p 2p Þ (2x − 3y) + (y − 2x) + (x + b z) = 0
¶x ¶y ¶z
b
= ∫ ∫ ∫
k 2
r sin q dr dq df
r =0 q =0 f =0
r (2 + 1 + b ) = 0 Þ b = −3

        = 2pkb2 Coulombs

SOLVED GATE PREVIOUS YEARS’ QUESTIONS

1. Two conductors are carrying forward and return configuration is measured to be L H/m. If distance
current of +I and −I as shown in the figure given of separation between the strips is now reduced to


below. The magnetic field intensity H at point P is: x/2 m, the inductance per unit length of the con-
figuration is:
+I −I


→ P
z z +I
→ → x
x d d x

y y
→ −I
w
I  I 
(a) y (b) x
pd pd L
I  I  (a) 2L H/m (b)
4
H/m
(c) y (d) x
2pd 2pd L
(c) H/m (d) 4L H/m
(GATE 2003: 1 Mark) 2
(GATE 2003: 1 Mark)
Solution:
(i) For the conductor carrying the current 
 + I
I  Solution:  Let width = w m, current in the for-
amperes, magnetic field intensity is H = ⋅y ward direction = +I, current in the reverse direc-


+I

2p d
magnetic field intensity H =
I
⋅ y (Right hand rule) tion = −I, distance between two strips = x m, then

+I 2p d For distance x, inductance is L H/m.
−I −I
(ii) For the conductor carrying the current L
 
x
For
I distance , inductance is H/m
amperes, magnetic field intensity is H = (− y ) = ⋅y 2 2
− I 2p d 2p d

 −I  I  Ans. (c)

magnetic field intensity is H = (− ) =
y ⋅ y
−I 2p d 2p d 3. A point charge of +1 nC is placed in a space with
a permittivity of 8.85 × 10−12 F/m as shown in
Therefore, the total magnetic field intensity the following figure. The potential difference VPQ
I  I  I  between two points P and Q at distances of 40 mm
= y + y = y
2p d 2p d pd and 20 mm, respectively, from the point charge is:
 Ans. (a)
20 mm Q
2. Two infinite strips of width w m in x-direction, P
as shown in the following figure, are carrying for- 1nC
ward and return currents of +I and −I in the
z-direction. The strips are separated by a distance 40 mm
of x m. The inductance per unit length of the

Chapter 2.indd 184 3/19/2016 5:16:09 PM


SOLVED GATE PREVIOUS YEARS’ QUESTIONS     185

(a) 0.22 kV (b) −225 V Solution:  Given that e r1 = 3, e r2 = 4, t1 = 0.5 mm, t = 1 mm.
(c) −2.24 kV e r1 = 3, e r2 = 4, t1 = 0.5 mm, t = 1 mm.
(d) 15 V
(GATE 2003: 2 Marks) e e A e e A
C1 = 0 r1 and C2 = 0 r2
t1 t2
Solution:  Given that q = 1nC, e = 8.85 × 10−12 F/m When the capacitors are connected in series, charge
will be the same for both C1 and C2. Therefore,
VPQ = VP − VQ Q1 = Q2, and
9 × 109 × 1 × 10−9 9 × 109 × 1 × 10−9 C1[100 − V ] = C2V
= −
40 × 10 −3
20 × 10 −3 e 0e r1A e e A
[100 − V ] = 0 r 2 × V
 1 1  t1 t2
= 9 × 103  −  = −225 V
 40 20 
(100 − V ) = V Þ 300 − 3V = 2V
3 4
Ans. (b)  0.5 1
300 = 5 V Þ V = 60 V
4. A parallel plate capacitor has an electrode area
Ans. (b)
of 100 mm2, with a spacing of 0.1 mm between
the electrodes. The dielectric between the plates is 6. A parallel plate capacitor is shown in the figure
air with a permittivity of 8.85 × 10−12 F/m. The given below. It is made of two square metal plates
charge on the capacitor is 100 V. The stored energy of 400 mm side. The 14 mm space between the
in the capacitor is: plates is filled with two layers of dielectrics of er1 = 4,
6 mm thick and er2 = 2, 8 mm thick. Neglecting
(a) 8.85 pJ (b) 440 pJ fringing of fields at the edges, the capacitance is:
(c) 22.1 nJ (d) 44.3 nJ
e0 = 8.85 × 10−12 F/m
(GATE 2003: 2 Marks)
er1 =
Solution:  Given that A = 100 mm2 , d = 0.1 mm, e 0 = 8.85 × 10−12 F/m, q 4;
= d100
= 6Vmm
= V.
−12
0 mm , d = 0.1 mm, e 0 = 8.85 × 10
2
F/m, q = 100 V = V . Now, we er2 = 2; a = 8 mm
have

E = 1/2 CV 2 (a) 1,298 pF (b) 944 pF


8.85 × 10−12 × 100 × 10−6
(c) 354 pF (d) 257 pF
e0A
C= =
d 0.1 × 10−3 (GATE 2004: 1 Mark)

= 8.85 × 10−12 F Solution:  For the given diagram, capacitance


C1C2
E = × 8.85 × 10−12 × 1002 = 44.3 nJ
1
=
2 C1 + C2

Ans. (d) e 0 e r 1A 8.85 × 10−12 × 4 × (400 × 10−3 )2


C1 = =
5. A composite parallel plate capacitor is made up d1 6 × 10 −3
= 94.4 × 10−11 F
of two different dielectric materials with different
thicknesses (t1 and t2) as shown in the figure given
below. The two different dielectric materials are e0er 2A 8.85 × 10−12 × 2 × (400 × 10−3 )2
C2 = =
separated by a conducting foil F. The voltage of d2 8 × 10−3
the conducting foil is:
= 35.4 × 10−11 F
100 V 94.4 × 10−11 × 35.4 × 10−11 3341.76 × 10−11
er1 = 3: t1 = 0.5 mm Ceq = =
94.4 × 10−11 + 35.4 × 10−11 129.8
er2 = 4: t2 = 1 mm
0V = 25.74 × 10−11 F = 257 × 10−12 or 257 pF

(a) 52 V (b) 60 V Ans. (d)


(c) 67 V (d) 33 V 7. The inductance of a long solenoid of length 1000 mm
wound uniformly with 3000 turns on a cylindrical
(GATE 2003: 2 Marks) paper tube of 60 mm diameter is:

Chapter 2.indd 185 3/19/2016 5:16:16 PM


186     Chapter 2:  ELECTROMAGNETIC FIELDS 

(a) 3.2 μH Solution:  The divergence of the curl of any vector




(b) 3.2 mH
(c) 32.0 mH (d) 3.2 H field is zero. Therefore, Ñ(Ñ × E ) = 0 .
(GATE 2004: 1 Mark) Ans. (d)
Solution:  For solenoid, 11. The charge distribution in a metal-dielectric-

4p × 10−7 × (3000)2 × p (30 × 10−3 )2


semiconductor specimen is shown in the following
m0 N A
2
figure. The negative charge density decreases linearly
L= =
l (1000 × 10−3 ) in the semiconductor as shown. The electric field
= 31.94 × 10−3 H = 32 mH
distribution is as shown in:

Ans. (c) Metal Dielectric Semiconductor


+ −
8. For a linear electromagnetic circuit, which of the + −
+ −

following statements is true?
+ x
(a) Field energy is equal to the co-energy. X1 X2 X3
(b) Field energy is greater than the co-energy.
(c) Field energy is lesser than the co-energy. (a) E
x
(d) Co-energy is zero.
(GATE 2004: 1 Mark) (b) E
x

l i (c) E
Solution:  Field energy = Wf = ∫ id l = Co - energy = Wf ′ = ∫ l di
x
0 0 (d) E
x
l i X1 X2 X3
Wf = ∫ id l = Co- energy = Wf ′ = ∫ l di (GATE 2005: 2 Marks)
0 0

These are equal for a linear electromagnetic field. Solution:  We know that:
(i) Electric field inside a conductor is zero.
Ans. (a)
(ii) Dielectric charge distribution is constant, which
9. In the figure given below, the initial capacitor volt- implies that electric field remains constant.
age is zero. The switch is closed at t = 0. The final (iii) In case of semiconductor, electric field varies
steady-state voltage across the capacitor is linearly with charge density. Hence the distri-
bution is as shown below.
t=0 10 Ω (ii)
E (iii)
20 V 10 µF 10 Ω x
(i)
This corresponds to the plot given in option (a)
(a) 20 V (b) 10 V Ans. (a)
(c) 5 V (d) 0 V
12. Which of the following statements holds for the
(GATE 2005: 1 Mark) divergence of electric and magnetic flux densities?

Solution:  Given that VC (0) = 0 (a) Both are zero.


(b) These are zero for static densities but non-zero
Capacitor will be fully charged when t → ∞. Therefore, for time varying densities.
20 20
VC (∞) = (10) = × 10 = 10 V (c) It is zero for the electric flux density.
10 + 10 20 (d) It is zero for the magnetic flux density.
 Ans. (b) (GATE 2006: 1 Mark)

 

10. If E is the electric field intensity, Ñ(Ñ ×E ) is Solution:  From Maxwell’s equation,
equal to: rV

 
 ∇ ⋅ D = r V and ∇ ⋅ E = → Non-zero
(a) E (b) ½E ½ e
(c) Null vector (d) Zero ∇ ⋅ B = 0 → Zero
Therefore, it is zero for magnetic flux density.
(GATE 2005: 1 Mark)
 Ans. (d)

Chapter 2.indd 186 3/19/2016 5:16:22 PM


SOLVED GATE PREVIOUS YEARS’ QUESTIONS     187

13. Consider the following statements with reference 1 Qr 3 Qr


dp (a) (b)
to the equation . 4pe 0 R 3 4pe 0 R 3
dt
(1) This is a point form of the continuity equation. 1 Q 1 QR
(c) (d)
(2) Divergence of current density is equal to the 4pe 0 r2 4pe 0 r 3
decrease of charge per unit volume per unit at
every point. (GATE 2007: 1 Mark)
(3) This is Maxwell’s divergence equation.
(4) This represents the conservation of charge. Solution:  From the Gauss’s law,
Select the correct answer:
(a) Only 2 and 4 are true.
(b) 1, 2 and 3 are true.
 (c) 2, 3 and 4 are true. r
(d) 1, 2 and 4 are true.
(GATE 2006: 2 Marks)
R
Solution:  For the given statements:

−dpV (r, t)
(i) Point form of continuity equation =
dt
−∂p
f = Qenc = ∫ D ⋅ ds
(ii) Maxwell’s continuity equation ∇ ⋅ J =
∂t Qenc =
Q 4
× p r3
(iii) ∂p/∂t represents the conservation of charge. 4
p R3 3
Ans. (d) 3
Qr3
14. Divergence of the vector field
∫ D ⋅ ds =
Therefore, 
R3
V (x, y, z) = −(x cos xy + y)iˆ + (y cos xy)jˆ +
(sin z2 + x2 + y2)kˆ is Qr3 Qr
D ⋅ 4p r 2 = =
(a) 2z cosz2 R 3
4pe 0R 3
(b) sin xy + 2z cosz2
Qr3
(c) x sinxy − cosz [as D = e 0 E ]
Qr
⋅ 4p r2 = E3 =
D Therefore, =
(d) None of these R 4pe 0R 3
(GATE 2007: 1 Mark) Ans. (a)
16. Two point charges Q1 = 10 μC and Q2 = 20 μC
Solution:  Divergence = ∇˙V
are placed at coordinates (1, 1, 0) and (−1, −1, 0),
In Cartesian co-ordinates,
respectively. The total electric flux passing through
a plane z = 20 will be:
∂  ∂  ∂ 
∇= i+ j+ k (a) 7.5 μC (b) 13.5 μC
∂x ∂y ∂z
(c) 15.0 μC (d) 22.5 μC
∂ ∂
∇= [−(x cos xy + y)] + [(y cos xy)]
∂x ∂y (GATE 2008: 2 Marks)
∂ Solution:   Gauss’s law states that the total elec-
+ [(sin z 2 + x2 + y 2 )]
∂z tric flux f through any closed surface is equal to
= −x(− sin xy)y + y(− sin xy)x + 2z cos z 2 the total charge enclosed by that surface.

= xy sin xy − xy sin xy + 2z cos z 2 f = Qenc = ∫ D . ds


S
= 2z cos z 2
In this case, half of the flux will pass through the
 Ans. (a) plane at z = 20 , and other half through the plane
at z = −20. Thus, total electric flux passing through
15. A solid sphere made of insulating material has a plane at z = 20 will be half of the total flux.
radius R and has a total charge Q distributed uni-
Qenc 20 + 10
formly in its volume. What is the magnitude of the Thus, f¢ = = = 15 µC
electric field intensity, E, at a distance r (0 < r < R) 2 2
inside the sphere? Ans. (c)

Chapter 2.indd 187 3/19/2016 5:16:33 PM


188     Chapter 2:  ELECTROMAGNETIC FIELDS 


17. Divergence of the three-dimensional radial vector (a) 2 µC (b) 4 µC
field r is: (c) 8 µC (d) 10 µC
(a) 3 (b) 1/r (GATE 2011: 2 Marks)
(c) i + j + k (d) 3(i + j + k )
Solution:  Given that er = 2.26 and e0 = 8.85
pF/m = 8.85 × 10−2 pF/cm
(GATE 2010: 1 Mark)

Solution: Dielectric break down strength (V/d) = 50 kV/cm



Suppose r = xi + yj + zk.  Then
l = 40 cm w = 20 cm
eA
⋅ V = e0er A
V
Charge q = CV =

div (r ) = div (xi + yj + zk ) = 3
d d

eA
⋅ V = e 0 e r A = 2.26 × 8.85 × 10−14 × 20 × 40 × 50
V
Ans.
q =(a)
CV =
d d
18. A capacitor is made with a polymeric dielectric having
an er of 2.26 and a dielectric breakdown strength of 50 × 103 = 8 µC
kV/cm. The permittivity of free space is 8.85 pF/m. Ans. (c)
If the rectangular plates of the capacitor have a width
of 20 cm and a length of 40 cm, then the maximum
electric charge in the capacitor is

Chapter 2.indd 188 3/19/2016 5:16:36 PM


SECTION III: SIGNALS AND SYSTEMS

MARKS DISTRIBUTION FOR GATE QUESTIONS

9
8
Number of questions

7
6
5 Marks 1
4 Marks 2
3 Total number of questions
2
1
0
2009 2010 2011 2012 2013 2014 2015

Chapter 3.indd 189 3/23/2016 4:36:00 PM


190        CHAPTER 3:  SIGNALS AND SYSTEMS

TOPIC DISTRIBUTION FOR GATE QUESTIONS

Year Concepts
2015 Introduction to signals and systems, Linear time invariant (LTI) system, Z-transform
2014 Fourier transform, Z-transform
2013 Impulse response, LTI system, Z-transform
2012 Z-transform, Laplace transform, Fourier analysis, Signal analysis
2011 Fourier series, Z-transformation, Laplace transform, Signal analysis
2010 Signal analysis, Fourier transform
2009 Continuous and discrete time signal, LTI system, Fourier analysis, Z-transform

Chapter 3.indd 190 3/23/2016 4:36:00 PM


CHAPTER 3

SIGNALS AND SYSTEMS

A signal is a variation in value of a parameter, for   3.  Digital signals: Discretized in time and quan-
example, temperature, speech, video, etc. Since we are tized in amplitude. If the amplitude of a signal can
in ­electrical domain; we can define signals as a variation assume only a value from a finite set of ­numbers,
in electrical parameter (usually a current or a voltage) then the amplitude is said to be discretized or
with respect to time. quantized.
Mathematically signals are modeled as function of one Engineers must model two distinct physical
or more independent variables. Examples of independent ­ henomena. First physical systems are modeled
p
variables used to represent signals are time, frequency or by ­mathematical equations and in the second,
spatial coordinates. Some signal-types are: physical signals are ­modeled by mathematical
  1. Continuous-time or analog signals: Continuous functions.
in time and continuous in amplitude. In contin-
uous-time signals, the amplitude can assume 3.1  ELEMENTARY SIGNALS
any value in the continuous range from
−∞ to +∞.
  2. Discrete-time signals: Discretized in time and The basic continuous-time and discrete-time signals
continuous in amplitude. Most of the discrete-time along with their properties and characteristic plots are
signals arise from sampling of continuous-time signal. given in Table 3.1.

Chapter 3.indd 191 3/23/2016 4:36:03 PM


192        CHAPTER 3:  SIGNALS AND SYSTEMS

Table 3.1 |   Basic continuous-time and discrete-time signals and their properties
Signal   Expression                      Plot
Continuous-Time Signals

δ(t)
Unit impulse function or
Dirac delta function d(t) = ∞ t=0
d (t) = 
= 0 t≠0
¥

∫ d (t)dt = 1
−∞

d (−t) = d (t)
t
0
Unit step function u(t)
1 t ≥ 0 u(t)
u(t) = 
0 t < 0

t
0

Unit ramp function r(t)


t t ≥ 0 r(t)
r(t) = 
0 t < 0
r(t) = tu(t)

t
0

Rectangular (or Gate)


 t
pulse A rect   A
 2a 

t
−a a

Triangular pulse
 t x(t)
1 − t ≤a
x(t) =  a 1
0 t >a

−a 0 a

(Continued)

Chapter 3.indd 192 3/23/2016 4:36:18 PM


3.1  ELEMENTARY SIGNALS        193

Table 3.1 |   (Continued)  


Signal   Expression                      Plot
Signum function sgn(t)
1 t>0

sgn(t) = 0 t=0
−1 t < 0
1

t
0

−1

Sampling function Sa(x) 1


sin x
Sa(x) =
x

sin px
sin c(x) =
px
−2p −p p 2p

1 For x = 0 1

sin c(x) ≡  sin px
 x otherwise

−2 −1 0 1 2

Exponential signal x(t) x(t)


x(t) = Aebt
Here A and b are real

b = Positive
A

0
x(t)

b = Negative
A

0
(Continued)

Chapter 3.indd 193 3/23/2016 4:36:30 PM


194        CHAPTER 3:  SIGNALS AND SYSTEMS

Table 3.1 |   (Continued) 

Signal   Expression                      Plot

x(t)
A

x(t) = A sin(wt + f )
Sinusoidal function x(t)
where w = 2pf = 2p / T
0

x(t) = ga(t) = e−a t


2 2

Gaussian function
−∞ < t < ∞

Discrete-Time Signal
d(n)

1
1 n = 0
Unit sample sequence d(n) d (n) = 
0 n ≠ 0

n
0

u(n)

1 n ≥ 0 1
Unit step sequence u(n) u(n) = 
0 n < 0 ...

n
0 1 2 3

r(n)

n n ≥ 0
Unit ramp sequence r(n) r(n) = 
0 n < 0 ...

n
0 1 2 3

(Continued)

Chapter 3.indd 194 3/23/2016 4:36:42 PM


3.1  ELEMENTARY SIGNALS        195

Table 3.1 |   (Continued)  

Signal   Expression                      Plot

x(n) = A sin(wn + f )
Sinusoidal sequence
−∞ < n < ∞

g(n)

a >1
an n≥0
Exponential sequence g(n) = 
0 n< 0
n

3.1.1  Basic Operations of Independent Variables Then the required operations can be carried out as follows:
(a) Time shifting (time advance) operation, x(t + 2):
The basic operations on independent variables, such as
11 −1−≤1 t≤+t +
2 ≤2 3≤ 3
00 otherwise
time shifting and time scaling expansion and compres- x(tx(+t +
2)2=) =
sion) delay can be illustrated by the following example.  otherwise
11 −3−≤3 t≤ ≤t 1≤ 1
==00 otherwise
  otherwise e e
Problem 3.1: An independent variable, x(t), given by x(t+2)
1 − 1 ≤ t ≤ 3
x(t) = 
0 otherwise
Determine the outcome of the following operations:

(a) time shifting (advancement)


(b) time shifting (delay) t
(c) time scaling (compression) −3 0 1
(d) time scaling (expansion) (b) Time delay operation, x(t - 2):
1 −1 ≤ t − 2 ≤ 3
Solution: The given variable x(t) is given by and rep- x(t − 2) = 
resented as 0 otherwise
1 1 ≤ t ≤ 5
1 − 1 ≤ t ≤ 3 =
x(t) =  0 otherwise
0 otherwise
x(t−2)
x(t)

t
t 1 5
−1 0 3 It can be seen that the width of the signal is not
affected due to time shifting operation.

Chapter 3.indd 195 3/23/2016 4:36:56 PM


196        CHAPTER 3:  SIGNALS AND SYSTEMS

  For example: Black and white TV picture


(c) Time scaling (compression): operation with may be ­represented as I(x, y, t) in which bright-
respect to t, x(2t): ness is function of time. Hence TV picture is
1 −1 ≤ 2t ≤ 3
three-dimensional signal.
x(2t) =    2.  Continuous-time and discrete-time ­signals:
0 otherwise Continuous-time signals (analog signal) are
 −1 defined for every value of time, that is, it takes
1
3
≤t≤
= 2 2 any value in the interval (a, b) where a and b may
0 otherwise be −∞ and ∞, respectively. For example: x(t) =
sin 2pt −∞ < t < ∞
x(2t)   Discrete-time signals are defined only at cer-
tain ­specific value of time. These time instants
need not be equidistant For example:
 0. 2 n ∀ n = 0, 1, 2, 3...
x(n) = 
0 otherwise
Discrete-time signals can also be repesented as x[n].
t
−1/2 0 3/2 Continuous and discrete signals are shown in
Fig. 3.1
(d) Time scaling operation (expansion) with respect
x(t)
 t
to t , x  
 2
 t  1 −2 ≤ t ≤ 6
x  = 
 2  0 otherwise

x(t/2)

1 (t)
(a)

t x(n)
−2 0 6
It can be seen that the width of the signal
changes in time scaling operation.

3.2  CLASSIFICATION OF SIGNALS

A signal is a function representing a variable or a ­physical n


quantity that conveys some information. (b)

   Figure 3.1 |   (a) Continuous-time signal;


  1.  Multichannel and multidimensional signals:
If different signals are recorded from the same
source, they are known as multichannel signal. (b) Discrete-time signal.
  For example: In electrocardiography, ECG   3.  Deterministic and random signals: The
signals recorded from 3-lead or 12-lead which signals which can be described uniquely by a
result in 3-channel and 12-channel signal. ­mathematical expression, graph or well-defined
  If signal is a function of a single independent rule are known as deterministic signals.
variable, it is called one-dimensional signal.   For example:
­Similarly, if the function depends on m inde-
1 0 ≤ n ≤ 2
pendent variables, it is called m-dimensional or x(n) = u(n) − u(n − 3) = 
­multidimensional signal. 0 otherwise

Chapter 3.indd 196 3/23/2016 4:37:10 PM


3.2  CLASSIFICATION OF SIGNALS        197

There are signals that cannot be described to Similarly total energy in discrete-time signal
any ­reasonable accuracy by mathematical equa- x(n) over time interval n1 ≤ n ≤ n2 is defined as
tion known as ­random signals. These signals n2

take random value at a given time. For example, 2
E= x(n)
picture signal in TV, sound signals in radio and n = n1
output of noise generator.
  4.  Periodic and aperiodic signals: A signal x(t) is Similarly, we can define normalized average
periodic if and only if power P of x(t) and x(n) as
T
x(t + T0) = x(t) −∞ < t < ∞

1 2
P = lim x(t) dt
T →∞ 2T
where T0 is called constant period. −T
  The smallest value of T0 is known as a fun- and
N

damental period. Any deterministic signal not 1 2
P = lim x(n)
satisfying above property is called an aperiodic N →∞ 2N + 1
n =−N
signal.
  For example, consider the signal Based on above definitions we can summarize that
x(t) = A cos[wt + f ] •  x(t) is an energy signal if and only if 0 < E < ∞
so that P = 0.
where A is amplitude f is phase and w = 2pf is •  x(t) is a power signal if and only if 0 < P < ∞
frequency in rad/s. Time period of the signal is thus implying that E = ∞.
2p 1 •  Signals that do not satisfy these properties are
T0 = = . Then
w f neither energy nor power signals.

 2p    2p  
x ( t + T0 ) = x  t +  = A cos  w  t +  + f 
 w   w  Problem 3.2: Determine if x(n) = (0.8)n u(n) is an
= A cos(wt + 2p + f )= A cos(wt + f ) = x(t)
energy signal or a power signal.

Solution: For the given signal, the total energy is


Hence, the signal is periodic.
∞ ∞ ∞
∑ x(n) = ∑ (0.8)2n = ∑ (0.64)n
  5.  Energy and power signal: If v(t) and i(t) are, 2
E=
respectively, the voltage and current across a n =−∞ n =0 n=0
resistor with resistance R, then the instantaneous
1
power is ⇒E= = 2.78 < ∞
1 − 0.64
1
p(t) = v(t)i(t) = v 2 (t) Therefore, x(n) is an energy signal.
R
The total energy expended over time interval t1
≤ t ≤ t2 is
Problem 3.3: Determine if x(n) = A u(n) is energy
t2 t2
or power signal.
∫ p(t)dt = ∫ R v
1 2
(t)dt
t1 t1 Solution: For the given signal, the total energy is
given by
and average power over this interval is ∞ ∞
∑ ∑ A2 = ∞
2
t2 t2 E= x(n) =
n =−∞
∫ ∫
1 1 1 2 n =0
p(t) = v (t)dt
t2 − t1 t1
t2 − t1 t1
R So, it is not an energy signal.
The normalized average power is given by
Similarly, we can define power and energy for any
N
∑ x(n)
continuous-time signal x(t) or any discrete-time 1 2
P = lim
signal x(n). N →∞ 2N + 1
n =−N
  Total energy over time interval t1 ≤ t ≤ t2 in
N
A2

continuous-time signal x(t) is 1 N +1
= lim A2 = A2 lim =
t2 N →∞ 2N + 1 N →∞ 2N + 1 2
n =0
∫ x(t)
2
E= dt
t1 Hence, it is a power signal.

Chapter 3.indd 197 3/23/2016 4:37:41 PM


198        CHAPTER 3:  SIGNALS AND SYSTEMS

  6.  Even and odd signals: A continuous-time signal I/P O/P


x(t) and discrete-time signal x(n) are said to be System
signal signal
even if they satisfy the following condition
x(-t) = x(t) for all t Figure 3.2 |   System.

and x(-n) = x(n) for all n


3.3.1  Continuous-Time and Discrete-Time Systems
that is, even signals are symmetric about the verti-
cal axis or time origin. A continuous-time system is one in which continuous-time
  Similarly, x(t) and x(n) are said to be odd signals input signals are applied and result in ­continuous-time
if they satisfy the following condition output signals.
x(-t) = -x(t) We can represent input-output relationship for con-
x(-n) = -x(n) tinuous-time system as shown in Fig. 3.3.

that is, odd signals are anti-symmetric about the y(t) = T [x(t)]
vertical axis or time origin,
So we can show that x(t) → y(t)
1
xe (t) = [x(t) + x(−t)]
2 Continuous-
x(t) y(t)
1 time system
xo (t) = [x(t) − x(−t)]
Figure 3.3 |   Continuous-time system.
2
The above definitions are true if the signals are
real-valued signals. Similarly, a discrete-time system is one that trans-
  A complex-valued signal x(t) is said to be conju- forms discrete-time inputs into discrete-time outputs as
gated symmetric if it satisfies the condition shown in Fig. 3.4.
x(-t) = x*(t)
x(n) Discrete-time y(n)
system

Figure 3.4 |   Discrete-time system.


Problem 3.4: Determine if x(t) = A cos (wt) and x(t)
= A sin (wt) are odd or even signals.

Solution: For the signal x(t) = A cos (wt), y(n) = T [x(n)]

x(−t) = Acos (w ×−t) x(n) → y(n)


= Acos (−wt)
In a communication system, the input signal could
= A cos wt be speech signal or computer data, the system itself is
So x(−t) = x(t) made up of the combination of a transmitter, channel
and receiver and output signal is an estimate of original
Hence, it is an even signal. message signal.
Similarly, for the signal x(t) = A sin (wt) Examples of continuous-time system are electric
­ etwork composed of resistor, capacitor and inductor
n
x(−t) = Asin( − wt) = −A sin wt that are driven by continuous-time source.
So,   x(−t) = −x(t) Example of discrete-time system is optical wand
­sensing system of a department store cash register.
Hence, it is an odd signal.

3.3.2 Time Invariant and Time Variant Systems


3.3  SYSTEM
A system is time invariant , if its input-output relation-
ship does not change with time. Otherwise it is said to
A system is defined as an entity that manipulates one or be time variant.
more signals to accomplish a function, thereby yielding a
y(n) = T [x(n)]
new signal as shown in Fig. 3.2.

Chapter 3.indd 198 3/23/2016 4:37:57 PM


3.3  SYSTEM         199

Now suppose that the same input signal is delayed by Problem 3.6: Determine whether the following ­systems
k units of time to yield x(n - k) and again applied to are linear or non-linear:
the same system. If the characteristics of system do not
change with time, the output of relaxed system will be (a) y(n) = x(n4)
y(n -k), that is, (b) y(n) = x3(n)

x(n) 
→ y(n)
T Solution: (a) For the given system y(n) = x(n4), if we
apply two inputs x1(n) and x2(n) to the system, it will
produce two outputs y1(n) and y2(n),
x(n − k) 
→ y(n − k)
T
y1(n) = x1(n4)

Problem 3.5:  Determine whether the following sig- y2(n) = x2(n4)


nals are time invariant:
y3 (n) = T [a1x1(n) + a2 x2 (n)]
(a)  y(n) = x(n) - x(n - 2)
(b)  y(n) = nx(n) = a1T [x1(n)] + a2T [x2 (n)]

Solution: (a) The given signal is = a1x1(n 4 ) + a2 x2 (n 4 )= a1y1(n) + a2 y2 (n)


y(n) = x(n) − x(n − 2) = T [x(n)] Let y3 (n) be the linear combination of two ­outputs,
If input is delayed by k units of time then the then
output will be y3 (n) = a1y1(n) + a2 y2 (n)
y(n, k) = T [x(n − k)] = x(n − k) − x(n − k − 2) (1) Therefore,
y 3 (n) = y3 (n)
If we delay y(n) by k units, then
Hence, the system is linear.
y(n − k) = x(n − k) − x(n − k − 2) (2)

(b) For the given system y(n) = x3(n), if we apply
From Eqs. (1) and (2)
two inputs x1(n) and x2(n) to the system, it will
y(n, k) = y(n − k) produce two outputs y1(n) and y2(n),
Thus, the system is time invariant. y1(n) = x13 (n) and y2 (n) = x23 (n)
(b) The given signal is y(n) = nx(n) = T [x(n)]
y3 (n) = T [a1x1(n) + a2 x2 (n)]
If input is delayed by k units of time then the
output will be
= [a1x1(n) + a2 x2 (n)]3
y(n, k) = T [x(n − k)] = nx(n − k) (3)

where, multiplier n is not the part of the input. = a13 x13 (n) + a23 x23 (n) + 3a1a2 x1(n)x2 (n)[a1x1(n) + a2 x2 (n)]

If we delay y(n) by k units, then = a13 x13 (n) + a23 x23 (n) + 3a1a2 x1(n)x2 (n)[a1x1(n) + a2 x2 (n)]
y(n − k) = (n − k)x(n − k) (4)
And the linear combination of two outputs is
Therefore, from Eqs, (3) and (4), y3 (n) = a1x13 (n) + a2 x23 (n)
y(n, k) ≠ y(n − k) Therefore,
So, the system is time variant. y3 (n) ≠ y3 (n) .

3.3.3  Linear and Non-Linear Systems So the system is non-linear.

System is known as linear if it satisfies the principle of


3.3.4  Causal and Non-Causal Systems
homogeneity and additivity
T [a1x1(n) + a2 x2 (n)] = a1 T [x1(n)] + a2 T [x2 (n)] A system is causal or non-anticipatory if its response to
an input does not depend on future values of that input.
where a1 and a2 are arbitrary constants. A system not It depends only on the input at the present time and in
satisfying this condition is said to be non-linear. the past.

Chapter 3.indd 199 3/23/2016 4:38:26 PM


200        CHAPTER 3:  SIGNALS AND SYSTEMS

y(t) =F[x(t), x(t - 1), x(t - 2)] 3.3.6  Invertible and Non-Invertible Systems
y(n) = F[x(n), x(n - 1), x(n - 2) ….] If a system has a unique relationship between its input
where F[.] is any arbitrary function. and output, the system is known as invertible, as shown
in Fig. 3.5. If the relationship between input and output
If the system does not satisfy this condition, it is is not unique, that is distinct input does not lead to
called non-causal. Such a system has an output that distinct output, the system is said to be non-invertible.
depends not only on present and past inputs but also on
future inputs. A system whose present response depends y(t)
on future values of the inputs is called as a non-causal x(t) T [x(t)] T -1 x(t)
system.
Figure 3.5 |   Invertible system.

TT-1 = I (Identity operator)


Problem 3.7: Determine if the following relations are
causal or non-causal:
(a) y(n) = x(n) - x(n - 2) Square law systems violate a necessary condition for
(b) y(n) = x(3n). invertibility, which is that distinct inputs produce dis-
tinct ­outputs. The distinct inputs x(t) and -x(t) pro-
Solution: (a) The given system y(n) = x(n) - x(n - 2) duce same output y(t). Therefore, square law system is
is causal because output depends on present and past non-invertible.
input values. For example, y(t) = Ax(t) means that the system is
(b) The system y(n) = x(3n) is non-causal because invertible but y(t) = Ax2(t) means that the system is
output depends on future input value, that is, said to be non-invertible.
for n = 1
y(1) depends on x(3) which is future value. 3.3.7 Static and Dynamic Systems

A discrete-time system or continuous-time system is


3.3.5 Stable and Unstable Systems called static or memory-less if its output at any instant
n depends on input sample at the same time instant but
A system is said to be bounded-input, bounded-output not on past or future samples of the input. There cannot
(BIBO) stable if and only if every bounded input results be any energy storage elements like a capacitor and an
in a bounded output, that is, if x(t) ≤ M x < ∞ then inductor in static system.
y(t) ≤ M y < ∞
If output of a system depends on present as well as
on past values of input, the system is called a dynamic
Problem 3.8: Determine if the following system is system. For example
stable or unstable. y(n) = x(n) + px2(n) is memory-less system whereas
y(n) = x(n) + 3x(n - 1) is dynamic in nature. In the
y(n) = x2 (n) + y(n − 1)
second system, for n = 0, y(0) = x(0) + 3x(-1), that is,
Solution: Let us select input sequence y(n) at time n = 0 depends on input x(n) at time n = 0
x(n) = c d(n) as well as at n = -1 thus showing dynamic characteristic.
where c is a constant.
Then 3.4  TIME DOMAIN REPRESENTATION
y(n) = c2d2(n) + y(n - 1) OF LINEAR TIME INVARIANT SYSTEM
Let
y(-1) = 0 Our objective here is on system descriptions that relate
y(0) = c 2 the output signal to the input signal when both signals
are represented as a function of time. These descriptions
y(1) = c2 are useful for analysing and predicting the behaviour of
y(2) = c2 Linear Time Invariant (LTI) systems and for implement-
y(3) = c2 ing discrete-time systems on a computer.

Therefore, the system is BIBO stable if c < ∞. In LTI system, we can determine the output due to
an arbitrary input by expressing the input as a weighted

Chapter 3.indd 200 3/23/2016 4:38:30 PM


3.4  TIME DOMAIN REPRESENTATION OF LINEAR TIME INVARIANT SYSTEM        201

∞ ∞
∑ ∑
superposition of time-shifted impulses. By linearity and
time invariance, the output must be a weighted superpo- = x(k) h(n − k) = x(k) hk (n)
k =−∞ n =−∞
sition of time-shifted impulse responses. Convolution is
used for determining the output from the input and the where hk (n) = ℑd
[ (n − k)] is the response of the system
impulse response. to an impulse, that is, impulse response of the system.
Another method to characterise the behaviour of LTI Thus, output of LTI system is given by weighted sum
system is linear constant coefficient differential or differ- of time shifted impulse response called convolution sum.
ence equation. Differential equation is used to represent Therefore,
continuous-time system, whereas difference equation ∞ ∞
represents discrete-time system. x(n) * h(n) = ∑ x(k) h(n − k)= ∑ x(n − k)h(k)
k =−∞ k =−∞

3.4.1  Convolution Problem 3.9: Assume that a LTI system has impulse
response h(n) = {2 21} and determine the output of
The impulse response is the output of a LTI system due ↑
to an impulse input applied at time t = 0 or n = 0. The this system in response to input
impulse response completely characterises the behaviour
x(n) = {2 3 − 3}
of any LTI system. ↑
If the input to a linear system is expressed as a Solution: The given system can be represented as
weighted superposition of time-shifted impulses, then ∞
the output is a weighted superposition of the system y(n) = ∑ x(n − k)h(k)
response to each time shifted impulse. k =−∞
This weighted superposition is termed as the con- For n = 0


volution sum for discrete-time system and convolution
y(0) = x(−k)h(k) = 10
­integral for continuous-time system.
k =−∞
For n = 1
3.4.1.1 Convolution Sum ∞
y(1) = ∑ x(1 − k)h(k) = 2
Consider the product of a signal x(n) and the impulse k =−∞
sequence d(n), written as For n =2
x(n)d (n) = x(0)d (n) ∞
y(2) = ∑ x(2 − k)h(k) = -3
Generalize this relationship to the product of x(n) and k =−∞
time shifted impulse sequence For n = 3
x(n)d (n − k) = x(k)d (n − k) ∞

where x(n) denotes a signal, while x(k) represents


y(3) = ∑ x(3 − k)h(k) = -3
k =−∞
the value of signal x(n) at time k. So, multiplication
Now, for n = -1
of signal by a time-shifted impulse characterises the


signal behaviour. So, signal x(n) can be represented as
y(−1) = x(−1 − k)h(k) = 4
x (n) = …+ x(-2) d(n + 2) + x(-1) d(n + 1) + x(0) k =−∞
    d(n) + x(1) d(n - 1) + x(2) d(n - 2) + ….

{ }
Therefore,
or ∞

y(n) = —4, 10 , 2, —3, —3
x(n) = x(k)d (n − k) ↑
k =−∞ In linear convolution, length of convolved sequence is
Consider again the system shown in Fig. 3.4. Let ℑ (N1 + N2 - 1)
denote the system. When x(n) signal pass through the
(3 + 3 - 1) = 5
system, then output response y(n) is given as
 ∞ 
Following are some important operations performed
y(n) = ℑ  ∑ x(k) d (n − k) to find convolution and these are depicted in Fig. 3.6.
k =−∞    1.  Folding: x(k) → x(−k)
 ∞    2.  Shifting: x(−k) → x(n − k)
=  ∑ x(k) ℑ d (n − k)
k =−∞    3.  Multiplication: x(n − k) → h(k)x(n − k)

Chapter 3.indd 201 3/23/2016 4:38:56 PM


202        CHAPTER 3:  SIGNALS AND SYSTEMS


  4.  Addition: ∑ h(k)x(n − k) 3.4.1.2 Convolution Integral
−∞
(a) h(n) = {2, 2, 1} In continuous-time system, integration replaces the sum
due to continuous nature of input.
Convolution integral can be represented as
h(k)
2 ∞

∫ x(t )d (t − t )dt
2
x(t) =
1
−∞
Therefore,
−1 0 1 ∞  ∞
y(t) = ℑ  ∫ x(t )d (t − t )dt  = ∫ x(t )ℑ[d (t − t )]dt

x(n) = {2, 3, −3}

(b) −∞  −∞

3 ∞

∫ x(t )h(t − t )dt


x(k) 2 =
−∞
where h(t − t ) is the system in impulse response. Therefore,
0 1 2 ∞
y(t) = x(t) ∗ h(t) = ∫ x(t ) h(t − t )dt
−3 −∞
(c)  x(n) = {2, 3, −3}
3.4.2  Differential Equation Representation of
3 LTI System
x(k) 2
The general form for linear constant coefficient ­differential
equation is
0 1 2 d n y(t) d n−1y(t) dy(t) d m x(t) dm
+ a n −1 + … + a1 + a0 y(t) = bm + bm −1
dtn dtn−1 dt dtm d
−3
d n y(t) d n−1yx(1−k)
(t) dy(t) d m x(t) d m−1x(t) dx(t)
(d) + a n −1 n −1
+ … + a1 + a0 y(t) = b m + bm −1 m−1
+ … + b1 + b0 x(t)
dtn dt dt dt m
dt dt

2 where x(t) and y(t) are the system input and output,
3
respectively. Also, dkx/dtk, dky/dtk are the kth time
derivatives of x(t), y(t) and ak and bk are the constants.

−1 0 1 Advantage of differential equation is that in finite


impulse response (FIR), system realization is possible
using convolution integral but if it is infinite impulse
  −3 response (IIR) system then convolution is clearly impos-
(e) h(k)x(1−k) sible because of lot of mathematical calculation and
hence memory is required.

6 2 3.4.3 Difference Equation Representation of


LTI System

−1 0 1 Input-output relationship of LTI system can be given as


follows:
−6 N M
∑ ak y(n − k) = ∑ bk x(n − k)

Figure 3.6 |   Representation of signals after operations. k =0 k =0
(a) Sequence h(k). (b) Sequence x (k).
(c) Folded sequence x (- k). (d) Shifted Here N is the order of difference equation.
sequence x (1- k). (e) Multiplied sequence By using difference equation we can obtain the system
h(k) x (1- k).
output as

Chapter 3.indd 202 3/23/2016 4:39:12 PM


3.4  TIME DOMAIN REPRESENTATION OF LINEAR TIME INVARIANT SYSTEM        203

N M Now if we consider x(t) as strictly or white sense


a0 y(n) + ∑ ak y(n − k) = ∑ bk x(n − k) stationary
k =1 k =0 ∞

If a0 = 1, then Ryy (t1 , t2 ) = ∫ h(a )Rxx (t1 , t2 − a )da


−∞

Using t = t1 - t2 in the above equation, we have


N M
y(n) = − ∑ ak y(n − k) + ∑ bk x(n − k) ∞


k =1 k =0
Ryy (t1 , t2 ) = h(a )Rxx (t − a )da
The above equation clearly indicates that the present −∞
output value can be obtained by present and past value
of input and past output value. Ryy (t1 , t2 ) = (h * Rxx )(t )

3.4.4  Linear Time Invariant System with 3.4.5  Types of LTI Systems
Random Input
3.4.5.1 Continuous-Time LTI System
Consider an LTI system h(t) as shown in Fig. 3.7. Let
a random input x(t) be applied to it to find the output The LTI systems which obey the two properties, linear-
y(t) due to this random input. ity and time variance, are known as continuous time
LTI systems. In a continuous-time LTI system, if the
applied input is continuous them the output also will be
x(t) h(t) y(t)
continuous. The behaviour of a linear, continuous-time,
LTI system can be described as follows:
Figure 3.7 |   Linear time invariant system.   1.  If x(t) is the input applied to the system shown in
Fig. 3.8, then output y(t) will be given by
To find the output, we need to find the mean and auto-
correlation of y(t) as follows: h (t)
  1.  Determining mean: Here, by convolution CT- LTI
x(t) y(t)
∞ system
y(t) = ∫ x(t )h(t − t )dt
Figure 3.8 |   Continuous-time LTI system.
−∞

Mean y(t) is given by


y(t) = x(t) * h(t)
∞ 
E[y(t)] = E  ∫ x(t )h(t − t )dt  where h(t) is the impulse response of the system.
−∞  Then

Now if the mean of the random signal is given y(t) = ∫ x(t − t )h(t )dt
by hx then −∞

∞  ∞
E[y(t)] =  ∫ h(t − t )E(x(t )dt)
−∞ 
= ∫ x(t ) ⋅ h(t − t )dt
−∞

E[y(t)] = (h * hx ) (t)
Thus, for a continuous-time LTI system, the
transfer function is defined as the output
  2.  To determine auto-correlation Ryy (t1, t2): obtained when we apply impulse as an input.

R(t1 , t2 ) = E[y(t1 )y(t2 )] 3.4.5.2 Discrete-Time LTI system

∞ ∞ 
Discrete-time LTI system refers to the LTI system or
= E  ∫ ∫ h(t1 − b )h(t2 − a )(x( b )x(a ))(da ⋅ db ) circuit that takes one discrete-time input signal and
−∞ −∞  produces one discrete time output signal. In these LTI
systems, the system function is denoted by h(n), and


y(n) is the output obtained from the impulse at time
Ryy (t1t2 ) = h(t2 − a )h(t1 − b )Rxx (a , b )da ⋅ db t = 0.
−∞

Chapter 3.indd 203 3/23/2016 4:39:32 PM


204        CHAPTER 3:  SIGNALS AND SYSTEMS

For any discrete-time LTI system, as shown in By using convolution property


Fig. 3.9, we consider x(n) as the linear combination of
y(n) = u(n) * h(n)
impulses, that is,
For continuous time
x(n) h(n) y(n) y(t) = u(t) * h(t)

We know that output y(t) is given by


Figure 3.9 |   Discrete-time LTI system.
y(t) = u(t) * h(t)

x(n) = ∑ ak < d k (n) Differentiating with respect to time, we get
k =−∞
dy(t) du(t)
So the output y(n), using discrete convolution is given by:- = * h(t)
dt dt
∞ ∞
y(n) = ∑ ak h(n − k) = ∑ h(n − k)x(k) du(t)
k =−∞ k =−∞ = d (t)
dt
y(n) = h(n) * x(n)
dy(t)
So by using the above convolution formula, we can obtain = d (t) * h(t)
dt
the output y(n) when the applied input x(n) is given.
The output is given by
t
3.4.5.3 Static and Dynamic LTI Systems y(t) = ∫ h(t )dt
−∞
Static systems are those systems whose output only So unit step response of any LTI system is the integra-
depends on the present input, that is, time t. These tion with respect to time.
­systems are represented as

y(t) = h[u(t)] = h[u1(t), u2 (t) …] 3.5  CONTINUOUS-TIME FOURIER


SERIES
On the other hand dynamic systems are those systems
whose output depends on the past values of the time.
These systems are represented as We can represent a signal as a weighted superposition

y(t) = h ( t, u(t)) = h(tu1(t), u(t), t2 u(t)…)


of complex sinusoids. If such signal is applied to a linear
system, then the system output is a weighted superposi-
tion of the system response to each complex sinusoid.
3.4.5.4 Invertible LTI Systems Here we consider procedure for resolving signals into
superposition of sines and cosines or equivalently complex
An LTI system is said to be invertible if there exists an exponential signals of the form ejwt. For periodic power
inverse system which when connected in series with the signal, this resolution results in Fourier series coefficient of
original system produces an output identical to input. the signal and the resulting representation of such signals
The system is represented as is known as Fourier series of the signal. For energy signals
x(n) * d (n) = x(n) of finite or infinite duration, the Fourier integral provides
x(n) * h(n) * h−1(n) = x(n)
the desired resolution with signal representation for such
signal given by inverse Fourier transform integral.
h(n) * h−1(n) = d (n)

3.4.5.5 Unit Step Response LTI System 3.5.1  Coefficient Values of Fourier Series

The response of any LTI system for an unit step input is If any periodic function x(t) satisfies Dirichlet condition,
known as unit step response of LTI system and is repre- it is possible to express it in an infinite series as
x(t) = a0 + a1 cos wt + a2 cos 2wt +  + an cos nwt + b1 sin wt + b2 sin 2
sented as shown in Fig. 3.10.

x(t) = a0 + a1 cos wt + a2 cos 2wt +  + an cos nwxt= wta(+ wt 2+wat2+cos
 2+wbtn+
wst) n+watn cos nwt + b1 sin wt + b2 s
u(n) h(n) y(n) (t+
)a=
0
sin

a0 +
b1 +
n 2cos nwt + bn sin n
1acos
b sin in
∞ n =1 ∞
∑ (an cos nwt + bn sin nwt)
Figure 3.10 |   Unit step response
= a0 + = a0 + ∑ (an cos nwt + bn sin nwt)
n =1 of LTI system. n =1

Chapter 3.indd 204 3/23/2016 4:39:50 PM


3.5  CONTINUOUS-TIME FOURIER SERIES        205

where coefficient values are given as Therefore,



+ ∑
T 1 2
sin nwt , when n is odd
∫ x(t)dt
1 x(t) =
a0 =
T 2 n =1 np
0
T 1

1 x(t) = , when n is even
an = x(t) cosnwt dt 2
T 0
T 3.5.1.1 Symmetry of Coefficient Values

1
bn = x(t) sinnwt dt
T 0 If x(t) is symmetric, coefficient an and bn can be written as
T /2


Dirichlet conditions are listed as follows 2
an = [x(t) + x(−t)]cos nwt dt
  1.  The function has a finite number of discontinuities T 0
in each period. bn = 0
  2.  The function has a finite number of maxima and
minima in each period. Otherwise if x(t) is anti-symmetric, then
T an = 0
  3.  The integral ∫ x(t) dt is finite. T /2


2
0 bn = [x(t) − x(−t)]sin nwt dt
T 0
Problem 3.10: Find the Fourier series representation
for the following function. 3.5.2  Exponential Form of Fourier Series
f(t)
We can write with the help of Euler’s identity

1 e jnwt − e−jnwt
sinnwt =  (3.1)
2j
t and
T/2 T 3T/2 2T
e jnwt + e−jnwt
Solution: The coefficient values can be determined as cosnwt =  (3.2)
2
T T /2

∫ x(t)dt = ∫
1 1 1 We know that
a0 = dt =
T T 2 ∞
0
T
0
T /2
x(t) = a0 + ∑ (an cosnwt + bn sinnwt)  (3.3)
n =1
∫ x(t) cos nwt dt = T ∫
2 2
an = cos nwt dt
T 0 0 Putting Eqs. (3.1) and (3.2) in Eq. (3.3), we get
2 1
[sin nwt]0 = [sin nwT /2 − sin 0 ]
T /2
= ∞  e jnwt + e−jnwt   e jnwt − e−jnwt 
nwT nwT x(t) = a0 + ∑ an   + bn  
=
1
[sin np − sin 0 ] = 0 n =1  2   2j 
nwT

1 
∑  2 (an − jbn )ejnwt + 2 (an + jbn )e−jnwt  (3.4)
T T /2 1
∫ sin nwt ⋅ x(t)dt = T ∫
2 2 = a0 +
bn = sin nwt dt n =1
T 0 0

−2 [cos nwt ]0
T /2 Let
−2
= = [cos nwT /2 − cos 0 ] D0 = a0 (3.5)
T nw nwT
−1 −1 Dn = 1/2 (an - jbn) (3.6)
= [cos np − 1] = [(−1)n − 1]
np np D-n = 1/2 (an + jbn)  (3.7)
Thus, Therefore,
2
bn = when n is odd
np ∞
=0 when n is even x(t) = D0 + ∑ Dn ejnwt + D−n e−jnwt 
n =1

Chapter 3.indd 205 3/23/2016 4:40:16 PM


206        CHAPTER 3:  SIGNALS AND SYSTEMS


x(t) = ∑ Dn e jnwt  (3.8) Solution: Here
n =−∞ t
x(t) =
Now as T
1
Dn = (an − jbn )
T T
−jnwt −jnwt
∫ x(t)e dt = ∫ t⋅e
1 1
2 Dn = dt
T2
12 
T T T 0 0
=  ∫ x(t) cos nwt dt − j ∫ x(t) sin nwt dt
2
1  e−jnwt e−jnwt 
T
2  T 0 T 
0
= 2 −t − 
T T  jnw ( jnw )2  0
∫ x(t)[cos nwt − j sin nwt]dt
1
=
e−jnwT e−jnwT
T 0 1  1 
T = 2 
−T + − 
−jwt T  jnw (nw ) (nw )2 
∫ x(t)e
1 2
⇒ Dn = T dt (3.9)
0
1  2 e−j2pn T 2e−j2pn T2 
Coefficient Dn has only a discrete spectrum because = −T + − 
values Dn exist only for discrete values of n. T 2  j2pn (2pn)2 (2pn)2 
From Eqs. (3.6) and (3.7) we can conclude that
T2  1 1 1 
an = Dn + D−n = −
2  j2pn
+ − 
bn = j(Dn − D−n ) T  (2pn)2
(2pn)2 
1
Problem 3.11: Find the exponential Fourier series for =−
j2pn
the following function.
Therefore,


1 1 jnwt
x(t) = − e
n =−∞ j2pn

 he properties of continuous-time Fourier series are


T
0 T 2T listed in Table 3.2.

Table 3.2 |   Properties of continuous-time Fourier series

Property Periodic Signal Fourier Series Coefficients

x(t) Periodic with period T and ak



y(t) fundamental frequency w 0 = 2p /T
bk

Linearity Ax(t) + By(t) Aak + Bbk

Time-shifting x(t − t0 ) ak e−jkw 0 t0 = ak e−jk(2p / T )t0

Frequency-shifting e jMw 0 t = e jM (2p / T )t x(t) ak −M

x * (t ) x*(t)
*
Conjugation a− k

Time reversal x (−t ) a−k

Time scaling x(at), a > 0 (periodic with period T/a) ak

Periodic convolution ∫T x(t )y(t − t )dt Tak bk

(Continued)

Chapter 3.indd 206 3/23/2016 4:40:53 PM


3.6  DISCRETE-TIME FOURIER SERIES        207

Table 3.2 |   (Continued)  


Property Periodic Signal Fourier Series Coefficients

+∞
Multiplication x(t)y(t) ∑ al bk −l
l =−∞

dx(t) 2p
Differentiation jkw 0ak = jk a
dt T k
t

Integration ∫ x(t)dt  1   1 
 jkw  ak =  jk(2p / T ) ak
−∞ 
(finite-valued and periodic only if a0 = 0) 0

a = a*
−k
 k
Re{ak } = Re{a−k }
Conjugate symmetry for 
x(t) real Im{ak } = − Im{a−k }

real signals
 ak = a−k
∠a = −∠a
 k −k

Real and even signals x(t) real and even ak real and even
Real and odd signals x(t) real and odd ak purely imaginary and odd

Even-odd decomposition of xe (t) = Ev{x(t)} [x(t) real] Re{ak }



real signals xo (t) = Od{x(t)} [x(t) real] j Im{ak }

+∞
∫T ∑
Parseval’s relation for periodic 1 2 2
x(t) dt = ak
signals T k =−∞

3.6  DISCRETE-TIME FOURIER SERIES Problem 3.12: Determine the spectra of the signal
x(n) = cos p n/3.

Fourier series representation for x(n) consist of N Solution: The given equation for the signal can be
­harmonically related exponential function written as
2pn
x(n) = cos
e j2πkn/N where N is period of sequence x(n) 6
that is, N = 6
x(n) = x(n + N)
Therefore,
A Fourier series represented as
1 5
N −1
ak = ∑ x(n)e−j2pkn /6 k = 0, 1, 2, 3, 4, 5
6 n =0
x(n) = ∑ ak ej2pkn /N for n = 0, 1, ……, N - 1
1 5 2pn
∑ cos 6 e− j2pkn /6
k =0
=
6 n =0
and Fourier series coefficient ak can be written as
1 5  e j2pn /6 + e−j2pn /6  −j2pkn /6
1 N −1
∑ x(n)e−j2pnk / N for k = 0, 1, …., N - 1
= ∑
6 n = 0 
e
ak =
N
2 
n =0

Chapter 3.indd 207 3/23/2016 4:41:11 PM


208        CHAPTER 3:  SIGNALS AND SYSTEMS

N −1  N −1 * −j2pkn / N 
∑  ∑ ak e
1
5 = x(n) 
∑ ej2pn /6(1−k) + e−j2pn /6(1+k)   k =0 
1 N
12 n = 0 
= n =0
k = 0, 1...5.
On substituting values of k in ak , we find N −1 1 N −1 
a0 = a2 = a3 =a4 = 0 = ∑ ak*  N ∑ x(n)e−j2pkn /N 
k =0  n =0 
and a1 = 1/2 and a5 = 1/2.
N −1
3.6.1  Power Density Spectrum of Periodic Signal = ∑ | ak | 2
k =0
The power density is given by
The properties of discrete-time Fourier series are listed
N −1 N −1
∑ ∑ x(n)x* (n)
1 2 1 in Table 3.3.
P = x(n) =
N n =0 N n =0

Table 3.3 |   Properties of discrete-time Fourier series


Property Periodic Signal Fourier Series Coefficients

x(n) Periodic with period N and ak 


  Periodic with period N
y(n) fundamental frequency w 0 = 2p /N bk 

Linearity Ax(n) + By(n) Aak + Bbk

x(n − n0 )
ak e−jk(2p / N )n0
Time shift

Frequency shift e jM (2p / N )n x(n) ak−M

Conjugation x * (n) *
a− k

Time reversal x(−n) a−k

x(n/m) if n is a multiple of m
x(m)(n) =  1
Time scaling 0 if n is not a multiple of m a
m k
(periodic with period mN) (viewed as periodic with period mN)

Periodic convolution ∑ x(n)y(n − r)


Nak bk
r= N

Multiplication x(n)y(n) ∑ al bk −l
l= N

x(n) − x(n − 1)
(1 − e−jk(2p / N ) )ak
First difference

n
∑ x(k) 

1 
−jk(2p / N )  k
Running sum a
k =−∞
 (1 − e )
(finite-valued and periodic only if a0 = 0)
(Continued)

Chapter 3.indd 208 3/23/2016 4:41:42 PM


3.7  CONTINUOUS-TIME FOURIER TRANSFORM        209

Table 3.3 |   (Continued)  


Property Periodic Signal Fourier Series Coefficients

a = a*
−k
 k
Re{ak } = Re{a−k }
Conjugate symmetry for 
x(n) Real Im{ak } = − Im{a−k }

real signals
 ak = a−k
∠a = −∠a
 k −k

Real and even Signals x(n) real and even ak real and even
Real and odd Signals x(n) real and odd ak purely imaginary and odd

Even-odd decomposition of xe (n) = Ev{x(n)} [x(n) real ] Re{ak }


x (n) = Od{x(n)} [x(n) real ]
real signals  o j Im{ak }

∑ ∑
Parseval’s relation for peri- 1 2 2
x(n) = ak
odic signals N n= N k= N

3.7  CONTINUOUS-TIME FOURIER When


T /2 2p
TRANSFORM −j

1 nt
Dn = xp (t)e T
T −T / 2
If the function is aperiodic, it cannot be analysed by
Let us define Fourier transform of x(t) as
Fourier series, but analysis of such function is possible
using Fourier transform (FT) if the following condition ∞
is ­satisfied by function as signal x(t) X( jw ) = ∫ x(t)e−j2pft dt
∞ −∞

∫ x(t)dt < ∞
Therefore,
−∞ 1
Dn = X(nf0 )
that is, it must have finite value. T
where f = 1/T.
To derive the expression for Fourier transform we
allow the period to increase without limit. When the  n
TDn = X(nf0 ) = X  
period becomes infinite, the signal becomes aperiodic T
and its spectrum becomes continuous, that is, Therefore,

 n
∑ X   e j2pnf0 t
x(t) = lim xp (t) 1
T →∞ xp (t) =
T n =−∞
T
where xp denotes aperiodic signal. So the above equation
shows that we can obtain spectrum of x(t), that is, ape- As, T → ∞, we can write, ∆f = 1/T.
riodic signal from the spectrum of xp(t) by taking limit Therefore,
as Tp → ∞ ∞
∞ 2p
∑ X(n∆f )e j2pn∆ft ∆f

j nt xp (t) =
T
xp (t) = Dn e n =−∞
n =−∞
Where ∞
1
Dn = (an − jbn )
lim xp (t) = lim
T →∞ ∆f → 0
∑ X(n∆f )e j2pn∆ft ∆f
n =−∞
2

2p
and w = ⇒ x(t) = ∫ X( jw )e j2pft dt
T −∞

Chapter 3.indd 209 3/23/2016 4:42:05 PM


210        CHAPTER 3:  SIGNALS AND SYSTEMS

The conditions that guarantee the existence of Fourier


transform are called Dirichlet conditions. These are X(F )

  1. The signal x(t) has a finite number of finite


discontinuities.
  2.  Signal x(t) has finite number of maxima and
minima.
  3.  The signal x(t) is absolutely integrable that is F
∞ −2/b −1/b 1/b 2/b
∫ | x(t) | dt < ∞
−∞
To evaluate the energy density spectrum of an aperi-
odic signal, let us consider x(t) as a finite energy signal. Problem 3.14: Find Fourier transform of impulse
Therefore, function.


Solution: The impulse function is given by
Ex = | x(t)| 2 dt
1 t = 0
−∞ d (t) = 
0 otherwise
¥

ò
Therefore,
Ex = x(t)x * (t)dt

−¥ −jwt
X( jw ) = ∫ d (t)e dt = e−jwt =1
¥ é¥ ù −∞
t=0
= ò x(t)dt ê ò x * (f )e−j2pft df ú where    w = 2pf
−¥ êë−¥ úû

¥ é¥ ù ∞
ò êò −j 2pft
ú = ∫ x(f ) df
2
X=* (f )df x(t)e dt Problem 3.15: Find the Fourier transform of Signum
−¥ êë−¥ úû −∞ function
This is known as Parseval’s relation for an aperiodic
Solution: The Signum function can be written and rep-
signal.
resented as:
1 t>o

Problem 3.13: Determine the Fourier transform and sgn(t) = 0 t=0
energy density spectrum of rectangular pulse signal −1 t<0
defined as
1 t ≤ b /2 x(t)
x(t) = 
0 t > b /2

Solution: The Fourier transform (FT) and energy +1


­density spectrum are represented as follows:

b /2 b /2
e−j2pft t
X( jw ) = ∫ e−j2pft dt =
−b /2
− j2pf
−b /2

−1
sin pfb
= Ab
pfb

x(t)
Since ∫ f(t) dt → ∞ , that is, area under curve of f(t)
−∞
is infinite, so to get the Fourier transform of sgn(t), let
1
sgn(t) = lim FT e−a(t) sgn(t)
a→0

 2 jw 
t
−b/2 b/2 = lim  − 2
2
 = jw
a→0  a + w 2 

Chapter 3.indd 210 3/23/2016 4:42:28 PM


3.7  CONTINUOUS-TIME FOURIER TRANSFORM     211

To prove this, consider the function


Problem 3.16: Find the Fourier transform of unit ∞
step function. FT[x(t)] = ∫ x(t)e-jwt dt
-∞
Solution: Unit step function can be written and rep-
Therefore,
resented as ∞
1 for t ≥ 0 ∫ x(at)e-jwt dt
x(t) =  FT[x(at)] =
0 for t < 0 -∞
 et P = at, therefore, dt = dp/a. Substituting in
L
x+1 the above equation, we get

dp 1  w 
FT[x(at)] = ∫ x( p)e-jw /ap = X 
-∞
a a  a
1
 his relation is satisfied for, a > 0 or a < 0,
T
so equation can be written as
1 w
FT[x(at)] = X 
a  a
t

  3.  Time differentiation:


Again the area under x(t) is infinite so condition

If x(t) ←
→ X( jw )
∫ f(t)dt < ∞ is not satisfied. then
-∞ dx(t)
1 ←
→ ( jw )X( jw )
u(t) can be implemented using u(t) = [1 + sgn(t)] dt
2
Therefore,
d 2 x(t)
←
→ ( jw )2 X( jw )
1  1  dt
FT [u(t)] = FT   + FT  sgn(t)
2  2  
2pd (w ) 1 2 
= + +
2 2 jw d n x(t)
1 ←
→ ( jw )n X( jw )
= pd (w ) + dt
jw
The relation can be proven as follows:

 dx(t)  dx(t) -jwt
FT  = ∫ ⋅e
 dt  -∞ dt
dt
3.7.1  Properties of Continuous-Time Fourier
Transform ∞

= e-jwt x(t) -∞ - -jwt
∫ (-jw ) e x(t)dt
  1.  Linearity: -∞
If x1(t) ←→ X1( jw )
FT ∞
= jw ∫ x(t)e-jwt dt
and -∞
x2 (t) ←→ X2 ( jw )
FT Hence,
 dx(t) 
FT  = jwX( jw )
 dt 
then

FT[a1x1(t) + a2 x2 (t)] ←→ a1FT[x1(t)] + a2 FT[x2 (t)]


FT
  4.  Time shifting property:
If x(t) ←
→ X( jw )
or a1X1 (jw ) + a2X2(jw )
then
→ e-jwt0 X( jw )
  2.  Time scaling: This is given by
x(t - t0 ) ←
1 w
FT[x(at)] = X 
| a |  a → e jwt0 X( jw )
x(t + t0 ) ←

Chapter 3.indd 211 3/24/2016 12:49:26 PM


212     CHAPTER 3:  SIGNALS AND SYSTEMS

  5.  Frequency shifting: The relation can be proven as follows:



x(t) ←
→ X( jw ) (x1(t) * x2 (t))e-jwt dt
If FT[x1(t) * x2 (t)] = ∫
then -∞
-jw 0 t ∞
∞ 
e x(t) ←
→ X[ j(w - w 0 )] = ∫  ∫ x1(l )x2 (t - l )dl  e-jwt dt
This can be proven as follows: -∞ 
-∞ 
∞ ∞
∞ 
FT[x(t)] = ∫ x(t)e-jwt dt = ∫  ∫ x2 (t - l )e-jwt dt  x1(l )dl
-∞ -∞ 
-∞ 
Therefore, 
∞ ∞
jw 0 t jw 0 t
FT[e x(t)] = ∫ e -jwt
x(t)e dt = X2 ( jw ) ∫ x1(l )e-jwl dl
-∞ -∞

∞ = X2(jw)⋅X1(jw)
= ∫ x(t)e-j(w -w 0 )t dt
  8.  Frequency convolution: This property states
-∞
that the multiplication of two time function x1(t)
= X[ j(w - w 0 )] and x2(t) in time domain is equivalent to con-
volving their Fourier transform in frequency
  6.  Symmetry or duality property: domain divided by 2
 If  x(t) ←
→ X( jw )  If x1(t) ←
→ X1( jw )
then
X(t) ←
→ 2px(-w )   and x2 (t) ←
→ X2 ( jw )

This can be proven as follows:   then


∞ X1( jw ) * X2 ( jw )
X( jw )e jwt dw x1(t) ⋅ x2 (t) ←


1
x(t) =
2p 2p
-∞
Therefore, The relation can be proven as follows:


∫ X( jw )e-jwt dw  X ( jw ) * X2 ( jw ) 
1
x(-t)w jwt
FT-1  1 ∫ [X1(jw ) * X2 (jw )]e
1
2p = dw
-∞  2p  (2p )2 -∞
∞ ∞
X ( jw ) * X2 ( jw ) 
∫ X( jw )e − jwt dw FT-1  1 jwt
2px(-t) =
∫ [X1(jw ) * X2 (jw )]e
1
  = dw
-∞  2p  (2p )2 -∞
Interchanging jw and t we get Again


2px(-jw ) = ∫ X(t)e-jtw dt
X1( jw ) * X2 ( jw ) = ∫ X1(u)X2 (w - u)du
-∞
-∞
Hence,   Therefore,
∞∞
X(t) ←
→ 2px(-jw )  X ( jw ) * X2 ( jw ) 
FT-1  1 ∫  ∫ X1(u)X2 (w - u)du
1
2p  =
  (2p )2 -∞ 
-∞
  7.  Convolution property:
∞ ∞  jwt
→ X1( jw ) FT-1  X1( jw ) * X2 ( jw )  = 1
  x1(t) ←
 If

 2p 
 (2p )2
∫  ∫ X1(u)X2 (w - u)du e dw
and -∞ -∞ 
x2 (t) ←
→ X2 ( jw ),  Let us put w = u + l, that is, dw = dl. Therefore,
∞ ∞
 X ( jw ) * X2 (w ) 
then
-j( l +
FT-1  1 ∫ 1  ∫ X2 (l)e
1
x1(t) * x2 (t) ←
→ X1( jw )X2 ( jw ) 2p  = X (u)
  (2p )2 -∞ -∞

Chapter 3.indd 212 3/24/2016 12:49:36 PM


3.7  CONTINUOUS-TIME FOURIER TRANSFORM     213

∞ ∞   The relations can be proven as follows:


 X1( jw ) * X2 (w )  -j( l + u)t
∫ 1  ∫ X2 (l)e
1
  = X (u) dl  du  Let us consider
 2p  (2p )2 -∞ -∞  t

∞ ∞
g(t) = ∫ x(l )dl
 1 -jlt
 -∞
∫ X1(u)e . 2p ∫ X2(l )e dl 
-jut 1
=  such that
 2p -∞ -∞  dg(t)
= x(t)
= x1(t) ⋅ x2 (t) dt
 Therefore,
  9.   Frequency differentiation:
 dg(t) 
FT[x(t)] = FT  = X( jw )
If x1(t) ←
→ X( jw )  dt 
  then  or,    jw G( jw ) = X( jw )
dX(w )
(-jt)x(t) ←
→ 1
dw G( jw ) = X( jw )
jw
  that is,
 ∞ 
.
FT  ∫ x(l )dl  =
1
X( jw )
d n X( jw ) -∞  jw
(-jt)n x(t) ←

dw n  G(jw) must exist to have a Fourier transform
 The relation can be proven as follows: of g(t) and hence there should not be any DC

term. Its presence will not allow the satisfac-
X( jw ) = ∫ x(t)e-jwt dt tion of integrability condition.

-∞
  Therefore, ∫ x(t)dt must tend to zero.
∞ -∞
dX( jw )
= ∫ x(t) ⋅ (-jw )e-jwt dt When t → ∞ , we can write
dw -∞
  ∞

∞ ∫ x(t)dt = 0 at lim g(t) = 0


t →∞
-jwt
∫ (-jw )x(t)e
-∞
= dt
-∞  This signal will not be an energy signal if the
 Hence, above condition is not satisfied and hence, its
dX( jw )
transform will have an impulse at w = 0.
= FT [(-jw )x(t)]
dw  Function u(t - l ) = 1 ∀ t = l , therefore,
t t
 Therefore,
dX( jw )
g(t) = ∫ x(l )dl = ∫ x(l )u(t - l )dl
-jw x(t) ←
→ -∞ -∞
dw

  10.  Time integration: = ∫ x(l )u(t - l )dl = x(t) * u(t)
-∞
 If x(t) ←
→ X( jw )  So by time convolution property.
 then G( jw ) = X( jw ) ⋅ FT[u(t)]

∫ x(t)dt ←  1 
1
→ X( jw )
= X( jw ) pd (w ) +
(provided X(0) = 0)
jw
∞ - ∞  jw 
∫ x(t)dt ←
1
→ X( jw ) (provided X(0) = 0)  But
-∞
jw X( jw )
 and X( jw )d (w ) = pX(0)d (w ) +
t jw

1
x(t)dt ←
→ X( jw ) + pX(0)d (w ) ≠ 0) X(0) = 0, then
 Hence
(provided X(0)
-∞
jw X( jw )
t G( jw ) =
∫ x(t)dt ← jw
1
→ X( jw ) + pX(0)d (w ) (provided X(0) ≠ 0)
-∞
jw

Chapter 3.indd 213 3/24/2016 12:49:46 PM


214     CHAPTER 3:  SIGNALS AND SYSTEMS

  11.  Fourier transform of x(—t) [Time reversal]:


Solution: Let us decompose x(t) into two signals
If x(t) ←
→ X( jw ) shown as follows.
x1(t)
then
x(-t) ←
→ X(-jw )

The above relation can be proven as follows: F


Since

FT[x(t)] = X( jw ) = ∫ x(t)e-jwt dt
t
-∞ −1/2 1/2
Therefore,
∞ x2(t)
FT[x(-t)] = ∫ x(-t)e jwt dt = X(-jw )
-∞

F
Problem 3.17: If a system is having impulse response of
h(t) = e-at u(t) and input to the system is x(t) = e-at u(t),
then find its system function. t
−3/2 3/2
Solution: Given that h(t) = e-atu(t)
 herefore, x(t) can be written in form of x1(t) and
T
Therefore, x2(t), so,
1
H( jw ) = 1
x(t) = x1(t - 2.5) + x2 (t - 2.5)
( jw + a) 2
and
2 sin(w /2) 2 sin(3w /2)
x(t) = e-bt u(t) X1( jw ) = and X2 ( jw ) =
w w
Therefore, 1 -j(5/2)w
X( jw ) = e [X1( jw ) + X2 ( jw )]
1
X( jw ) =
2
( jw + b) Therefore, from shifting property
Therefore,  sin (w /2) + 2 sin(3w /2) 
X( jw ) = e-j5w /2  
Y ( jw ) = H( jw ) × X( jw ) w
 
1
=
( jw + a)( jw + b)
1 -a + b 1 a - b Problem 3.19: Find the Fourier transform of signal
x(t) = e-a t for a > 0
= +
( jw + a) jw + b
1  -bt
e u(t) - e-at u(t) Solution: As we know
a-b 
y(t) =
e-at u(t) ←→
FT 1
a + jw
Signal x(t) = e-a t can be decomposed into two terms
e-at t>0
x(t) = e-a t =  at
Problem 3.18: Find the Fourier transform of signal
shown below. e t<0
x(t) Therefore,

x(t) = e-a t = e-at u(t) + eat u(-t)


1.5 Therefore, from time reversal property
1
eat u(-t) ←→
1 a - jw
Hence,
1 1
X( jw ) =
t 2a
+ = 2
1 2 3 4 a + jw a - jw a +w2

Chapter 3.indd 214 3/24/2016 12:49:55 PM


3.7  CONTINUOUS-TIME FOURIER TRANSFORM     215

Problem 3.20: Using duality property find the Fourier Problem 3.21: Determine Fourier transform of the
transform H(jw) of the signal signal
2 sin(t)sin(t/2)
xt =
pt2
h(t) =
1 + t2
Solution: From duality Solution: The given signal can be written as

X(t) ←
→ 2px(-jw )  sin(t)  sin(t/2)
x(t) = p 
 pt   pt 
As we know
Applying multiplication property, we get
e -t ←→ X( jw ) =
FT 2
1+w 2
1  sin(t)   sin(t/2) 
X( jw ) = FT   × FT 
 pt 
Therefore,

2  pt 
 2  jwt
e- t = ∫
1
 2
e dw
2p 
 ourier transfer of each sine function is a rectangular
F
-∞ 1 + w pulse, so we can proceed to convolve those pulses to
 ultiplying the above equation by 2p and ­replacing
M obtain X(jw) as
t by -t, we get
X(jw)

 2  -jwt
2pe- t = ∫
 2
e dw
-∞ 1 + w

On interchanging the variables t and w, we get 1/2

 2  -jwt
2pe- w = ∫ 2
e dt
-∞ 1 + t
 jw
Therefore,
−3/2 −1 −1/2 1/2 1 3/2

 2 
FT  2
= 2pe- w
 1+ t  The properties of continuous-time Fourier transform are
listed in Table 3.4.

Table 3.4 |   Properties of continuous-time Fourier transform


Property Periodic Signal Fourier Series Coefficients

x(t) X(jw)
y(t) Y(jw)
Linearity Ax(t) + by(t) aX( jw ) + bY ( jw )

Time shifting x(t - t0 ) e-jwt0 X( jw )

Frequency shifting e jw 0 t x(t) X( j(w - w 0 )

Conjugation x * (t) X * (− jw )

Time-reversal x(-t) X(-jw )

1  jw 
Time and frequency-scaling X 
a  a 
x(at)

Convolution x(t) * y(t) X( jw )Y ( jw )

(Continued)

Chapter 3.indd 215 3/24/2016 12:50:05 PM


216     CHAPTER 3:  SIGNALS AND SYSTEMS

Table 3.4 |   (Continued)  

Property Periodic Signal Fourier Series Coefficients

1
Multiplication x(t)y(t) X( jw ) * Y ( jw )
2p
d
Differentiation in time x(t) jwX( jw )
dt


1
Integration x(t)dt X( jw ) + pX(0)d (w )
-∞
jw

d
Differentiation in frequency tx(t) j X( jw )
dw

X( jw ) = X * (-jw )

Re{X( jw )} = Re{X(-jw )}

Conjugate symmetry for real signals x(t) real Im{X( jw )} = - Im{X(-jw )}
 X( jw ) = X(-jw )

∠X( jw ) = -∠X(-jw )

Symmetry for real and even signals x(t) real and even X(jw) real and even
Symmetry for real and odd signals x(t) real and odd X(jw) purely imaginary and odd

Even-odd decomposition for real xe (t) = Ev{x(t)} [x(t) real ] Re{X( jw )}
x (t) = Od{x(t)} [x(t) real ]
signals  o j Im{X( jw )}

+∞ +∞

∫ ∫
Parseval’s relation for aperiodic 1
X( jw ) dw
2 2
x(t) dt =
signals
-∞
2p -∞

3.8  DISCRETE-TIME FOURIER and for inverse discrete-time Fourier transform (synthe-
TRANSFORM sis equation):
1 p
x(n) ≡ DTFT −1[X(e jw )] = X(e jw )e jwn dw ; (for - ∞ < n < ∞
2p ∫-p
−1 p
[X(e jwwe)] = ∫-p X(e
jw
)e jwn dw ; (for - ∞ < n < ∞)
1
(n) ≡ DTFT
To obtain discrete-time Fourier xtransform (DTFT),
apply the same procedure for discrete-time signal as 2p
we have used to obtain Fourier transform for continuous- Notation we can use discrete-time Fourier transform is
time signal.
x(n) ← X(e jw )
DTFT
Consider a sequence x(n), that for finite duration is
-N1 ≤ n ≤ N2 and zero otherwise. We can construct a
periodic sequence x(n) for which x(n) is one period. As 3.8.1  Properties of Discrete-Time Fourier
we choose the period N to be large, x(n) is identical to Transform
x(n) over a longer interval, and as N → ∞ x(n) = x(n)   1.  Linearity:
for any finite value of n. x1(n) ←→ X1(e jw )
DTFT
If
For direct transform (analysis equation):
x2 (n) ←→ X2 (e jw )
DTFT

X(e jw ) ≡ DTFT[x(n)] = ∑ x(n)e-jwn
then
n =-∞ a1x1(n) + a2 x2 (n) ←→ a1X1(e jw ) + a2 X2 (e jw )
DTFT

Chapter 3.indd 216 3/24/2016 12:50:14 PM


3.8  DISCRETE-TIME FOURIER TRANSFORM     217

then
→ X(e jw ) = X1(e jw )X2 (e jw )
x(n) = x1(n) * x2 (n) ←
Problem 3.22: Determine the Fourier transform of
the signal.
x(n) = a| n | -1 < a < 1 → X(e jw ) = X1(e jw )X2 (e jw )
x(n) = x1(n) * x2 (n) ←
Solution: x(n) = x1(n) + x2(n) The relation can be proven as follows:

Where
an
x(n) = x1(n) * x2 (n) = ∑ x1(k) x2 (n - k)
n≥0 k =-∞
x1(n) = 
0 n<0 Therefore,

a-n
x2 (n) = 
n<0 X(e jw ) = ∑ x(n)e-jwn
n =-∞
0 n≥0
∞ ∞  -jwn
∑  ∑ x1(k)x2 (n - k) e
Therefore,
∞ ∞ =
n =-∞  
X1(e jw ) = ∑ x(n)e-jnw = ∑ an e-jwn k =-∞
n =-∞
∞  ∞ 
n =0

∞ = ∑ x1(k)  ∑ x2 (n - k)e-jwn 
∑ (ae-jw )n = 1 - ae-jw  n =-∞ 
1
= k =-∞
n =0 ∞
∞ -1 = ∑ x1(k)e-jwk ⋅ X2 (e jw )
X2 (e jw ) = ∑ x2 (n)e-jwn = ∑ a-n e-jwn k =-∞
n =-∞ n =-∞
= X1(e jw )X2 (e jw )

-1
ae jw
= ∑ (ae jw )-n = ∑ (aejw )k = 1 - aejw
n =-∞ k =1 Problem 3.23: Determine the convolution of sequence.
x1(n) = x2 (n) = {1, 2, 1}
Therefore, ↑
X(e jw ) = X1(e jw ) + X2 (e jw ) Solution: X1(e jw
) = X2 (e jw
) = 2 + 2 cos w

1 ae jw 1 - a2 Therefore,
= -jw
+ jw
=
1 - ae 1 - ae 1 - 2a cosw + a 2
X(e jw ) = X1(e jw )X2 (e jw ) = (2 + 2 cos w )2

= 4 + 4 cos2 w + 8 cos w
  2.  Time shifting:
= 4 + 2[cos 2w + 1] + 8 cos w
jw
x(n) ←→ X(e
DTFT
If )
= 6 + 2 cos 2w + 8 cos w
then
= 6 + e2 jw + e-2 jw + 4e jw + 4e-jw
x(n - k) ←→ e-jwk X(e jw )
DTFT

  3.  Time reversal: = e2 jw + 4e jw + 6 + 4e-jw + e-2 jw

x(n) ←→ X(e jw ) Taking inverse transform we obtain


DTFT
If
then x(n) = {1, 4, 6, 4, 1}

-jw
x(-n) ←→ X(e
DTFT
)

 his can be proven, similarly as in case of


T
­continuous-time signal.   5.  Periodicity property:
  4.  Convolution: → X1(e jw )
x1(n) ←

x1(n) ←→ X1(e jw ) X(e j(w +2pm) ) = X(e jw )


DTFT

x2 (n) ←→ X2 (e jw ) with period 2p, where m is an integer.


DTFT

Chapter 3.indd 217 3/24/2016 12:50:28 PM


218     CHAPTER 3:  SIGNALS AND SYSTEMS

  6.  Correlation: The RHS of the above equation can be written as


jw p  ∞  * jw
x1(n) ←
→ X1(e
 ∑ x1(n)e
-jwn

If ) 1
 X2 (e )dw
2p -p  n =-∞ 
→ X2 (e jw )
x2 (n) ←
∞ p
∑ X *(e jw )e-jwn dw
2p -∫p 2
then 1
= x1(n)
n =-∞
→ Sx x (w ) = X1(e jw )X2 (e-jw )
g x x (m) ←

1 2 1 2

 he relation can be proven similarly as in case


T = ∑ x1(n)x2*(n)
of convolution of continuous-time signal. Here n =-∞
we have If   x1(n) = x2(n) = x(n)

g x x (n) =
1 2
∑ x1(k)x2 (k - n) then
k =-∞ p

∑ | x(n) | = 2p ∫ | X(e jw )| 2 dw
1 2
  7.  Frequency shifting:
n =-∞ -p
If       → X(e jw )
x(n) ←
  10. Multiplication of two sequences (Windowing
then theorem):
e jw 0 n x(n) ←
→ Xe j(w -w 0 ) If
x1(n) ←→ X1(e jw )
FT

 his property can be proved by direct substitu-


T
x2 (n) ←→ X2 (e jw )
tion into analysis equation. FT

  8.  Modulation theorem:


then
p
→ X(e jw )
x(n) ←
x3 (n) ≡ x1(n)x2 (n) ←→ X3 (e jw ) = ∫ X1(l )X2 (e j(w -
FT 1
then 2p -p
p
x(n) cos w 0 n ←→ X(e j(w +w 0 ) ) +xX((ne)j(w≡-xw 0()n))x (n) ←
1
→ X3 (e jw ) = ∫ X1(l )X2 (e j(w - l ) )dl
FT FT 1
2 3 1 2
2p -p
The relationship can be proven as follows:
The relation can be proven as follows:
 e jw 0 n + e-jw 0 n  -jwn


FT[x(n) cos w 0 (n)] = ∑ x(n)  e
n =-∞  2 
X3 (e jw ) = ∑ x3 (n)e-jwn
n =-∞
∞ ∞

∑ x(n)e-j(w -w 0 )n + 2 ∑ x(n)e-j(w +w 0 )
1 1
=
2 n =-∞ n =-∞
= ∑ x1(n)x2 (n)e-jwn
n =-∞

Xe j(w -w 0 ) + Xe j(w +w 0 )
1 1 p
= ∞  1 
2 2 = ∑  2p ∫ X1(l )e jln dl  x2 (n)e-jwn

n =-∞  -p
= [Xe j(w -w 0 ) + Xe j(w +w 0 ) ]
1
2 p  ∞ 
∫ X1(l )dl  ∑ x2 (n)e-j(w - l )n 
1
=
  9.  Parseval’s theorem: 2p -p  n =-∞ 
x1(n) ←→ X1(e jw )
FT
If p

∫ X1(l )X2e j(w - l )dl


1
=
x2 (n) ←→ X2 (e
FT jw
) 2p -p

then Note: C
 onvolution integral is known as periodic con-
∞ p volution of X1(ejw) and X2(ejw) because it is
∑ ∫ X1(e jw )X2*(e jw )dw
1
x1(n)x2*(n) = the convolution of two periodic functions hav-
n =-∞ 2p -p ing the same period.

Chapter 3.indd 218 3/24/2016 12:50:40 PM


3.8  DISCRETE-TIME FOURIER TRANSFORM     219

  11.  Differentiation in frequency domain:


Therefore,
x(n) ←→ X(e jw )
FT
 2 -j26.6  jpn /2
y(n) = A  e  e
then  5
dX(e jw )  pn 
nx(n) ←→ j
FT
 -26.6
dw Ae j 
2
= 2
for - ∞ < n < ∞
   The relation can be proven as follows: 5

X(e jw ) = ∑ x(n)e-jwn
n =-∞
   Therefore, Problem 3.25: Consider a causal LTI system
jw ∞ ∞ that is characterised by the difference equation.
= ∑ x(n)(-jn)e-jwn = -j ∑ nx(n)e-jwn 3
dX(e )
1
dw y(n) = y(n - 1) + y(n - 2) = 2x(n) . Determine the
n =-∞ n =-∞ 4 8
dX(e jw ) ∞ ∞ frequency response of the system.
= ∑ x(n)(-jn)e-jwn = -j ∑ nx(n)e-jwn
dw n =-∞ n =-∞ Solution: Taking Fourier transform on both sides of

Then the given difference equation, we get
dX(e jw )
Y (e jw ) - e-jw Y (e jw ) + e-2 jw Y (e jw ) = 2X(e jw )
FT[nx(n)] = j 3 1
dw 4 8
 
Y (e jw ) 1 - e-jw + e-2 jw  = 2X(e jw )
Problem 3.24: Determine the output sequence of the 3 1
system with impulse response  4 8 
 1 Y (e jw )
n
= H(e jw )
2
h(n) =   u(n)
 2 jw
=
 3 -jw 1 -2 jw 
 1 - e
X(e )
when the input is complex exponential sequence
+ e 
4 8

x(n) = Ae jpn /2 for −∞ < n < ∞ H(e jw ) =


4 2
-
1 -jw 1 jw
1- e 1- e
Solution: The discrete-time Fourier transform of the 2 4
impulse response Taking inverse transform

H(e jw ) = ∑ -h(n)e-jwn =
1
 1  1
n n
n =-∞
1 -jw
1- e h(n) = 4   u(n) - 2   u(n)
 2  4
At w = p/2 2

p 
e-j26.6
1 2
H  = =
 2 1 5 The properties of discrete-time Fourier transform are
1+ j
2 listed in Table 3.5.

Table 3.5 |   Properties of discrete-time Fourier transform


Property Aperiodic Signal Fourier Transform

X(e jw )
x(n)
y(n)  Periodic with period 2p
Y (e jw ) 
Linearity ax(n) + by(n) aX(e jw ) + bY (e jw )

Time-shifting x(n - n0 ) e-jwn0 X(ejw)

Frequency-shifting e jw 0 n x(n) X(e j(w -w 0 ) )

(Continued)

Chapter 3.indd 219 3/24/2016 12:50:48 PM


220     CHAPTER 3:  SIGNALS AND SYSTEMS

Table 3.5 |   (Continued)  

Property Aperiodic Signal Fourier Transform

Conjugation x * (n) X * (e-jw )

Time reversal x(-n) X(e-jw )

x(n/k), if n = multiple of k
Time expansion x(k)(n) = 
if n ≠ multiple of k X(e jkw )
0,

Convolution x(n) * y(n) X(e jw )Y (e jw )

jq
∫2p X(e )Y (e j(w -q ) )dq
Multiplication 1
x(n)y(n)
2p

x(n) - x(n - 1)
(1 - e-jw )X(e jw )
Differencing in time

n
∑ X(e jw )
1
Accumulation x(k) -jw
k =-∞ 1-e

+∞
+pX(e j0 ) ∑ d (w - 2pk)
k =-∞

Differentiation in frequency dX(e jw )


j
ux(n) dw

X(e jw ) = X * (e-jw )

Re{X(e jw )} = Re{X(e-jw )}

Conjugate symmetry for real jw − jw
x(n) real Im{X(e )} = − Im{X(e )}

signals
jw
 X(e = X(e-jw )
 jw -jw
∠X(e ) = −∠X(e )

X(e jw ) real and even


Symmetry for real, even signals x(n) real and even

Symmetry for real, odd signals x(n) real and odd X(e jw ) purely imaginary and
odd

Even-odd decomposition of real xe (n) = Ev{x n } [x(n) real ] Re {X(ejw)}



xo (n) = Od{x n } [x(n) real ] jIm {X(ejw)}
signals

+∞
∑ ∫2p X(e jw ) dw
Parseval’s relation for aperiodic 2 1 2
x(n) =
signals
n =-∞ 2p

Chapter 3.indd 220 3/24/2016 12:50:55 PM


3.9  DISCRETE FOURIER ­TRANSFORM (DFT) AND FAST FOURIER ­TRANSFORM (FFT)     221

3.9  DISCRETE FOURIER ­TRANSFORM Length of both the sequences is N. x(k) is called N-point
(DFT) AND FAST FOURIER DFT of x(n) and x(n) is the inverse discrete Fourier
transform of X(k).
­TRANSFORM (FFT)
N -1 2p

∑ x(k)e
1 j kn
x(n) = N
3.9.1  Discrete Fourier Transform (DFT) N k =0
n = 0, 1, 2,… N - 1
Discrete Fourier transform (DFT) can be formulated by
extending the Fourier transform of continuous time sig- We can use twiddle factor to denote
nals to a finite length discrete time sequence through 2p
-j
sampling and windowing. WN = e N
The DFT transforms a finite length sequence x(n),
0 ≤ n ≤ (N-1) into another sequence x(k), 0 ≤ k ≤ (N-1) Therefore, Eq. (3.9) reduces to
of the same length, where
N -1
N -1 -j
2p
∑ x(n)WN nk
∑ x(n)e
kn x(k) =
x(k) = N
 (3.9) n =0
n =0
k = 0, 1, 2,… N − 1 The properties of DFT are listed in Table 3.6.

Table 3.6 |   Properties of discrete Fourier transform

N -1 2p N -1 2p
-j
∑ ∑
1 j kn kn
Property x(n ) = x(k )e N x(k ) = x(n )e N
N k =0 n =0

Linearity ax(n) + by(n) aX(k) + bY (k)


Conjugation x * (n) X * (-k) = X * (N - k)
Real x(n) = x * (n) X(k) = X * (-k)
Real and even x(N - n) = x(n) X(k) = X(-k)
Real and odd x(N - n) = -x(n) X(k) = -X(-k)
Time reversal x(-n) = x(N - n) X(-k) = X(N - k)
2pmk
Circular shift x(n - m) -j
X(k)e N

 2pnp  X(k - p) + X(k + p)


x(n)cos 
 N 
Modulation
2
2pnp
Frequency shift j
N X(k - p)
x(n)e
N -1
Circular convolution x(n) ⊗ y(n) = ∑ x(m)y(n - m) X(k)Y (k)
m=0
N -1

1
Product property x(n)y(n) X(k) ⊗ Y (k) = X(m)Y (k - n)
N m=0
N −1
1 N -1
∑ x(n)y(n) = ∑ X(k)Y (-k) N -1

n =0 N k =0 1
Parseval’s theorem x(n)y * (n) = X(k)Y * (k)
N -1 N -1
∑ ∑
2 1 2 n =0 N
x(n) = X(k)
n =0 N k =0

Chapter 3.indd 221 3/24/2016 12:51:06 PM


222     CHAPTER 3:  SIGNALS AND SYSTEMS

3.9.2  Fast Fourier Transform (FFT) Sampling rate ≥ Nyquist rate

We have seen that DFT of sequence x(n) is Let fmax is maximum frequency in message signal then,
N -1 2p Nyquist rate = 2fmax
-j
∑ x(n)e
kn
x(k) = N k = 0, 1, 2,… N − 1
n =0
Thus,
Sampling rate ≥ Nyquist rate (2fmax )
For finding a single DFT sample, that is, X(0) or
X(1)…, X(N-1), we have to perform N multiplications
and (N-1) additions. We have N DFT samples, so for
finding X(0) to X(N-1), have to perform N2 multiplica- 3.10.1  Impulse Train Sampling of a Continuous-
tions and N (N-1) additions. For a 1000 point DFT, the Time Signal
number of multiplications and additions would thus be,
N2 = (1000)2 and N (N-1) = (1000)(999), respectively. For impulse train sampling, the message signal is multi-
This would require huge circuitry and memory space. plied with the impulse train to get samples.
Fast Fourier transform (FFT) is a method for effi- Let x*(t) be sampled data
ciently computing DFT. We can reduce the number of x(t) original continuous-time signal
computations of DFT using the periodicity and sym- Pd(t) train of impulse function.
metry property of DFT. Because of the symmetry of
the twiddle factor W kn, the unnecessary complex com- These signals are represented as shown in Fig. 3.11 (a),
N
(b) and (c).
putations using its higher power terms, like W 4 n, W 5n,
N N

etc., can be replaced with algorithms based on symme- (a) x(t)


try called the FFT algorithms. Wkn possesses circular
symmetry for N point DFT, so

W k(n + N ) = W kn
t
FFT algorithms reduce the time of finding the DFT from
several minutes to less than a second. The two FFT
algorithms used are:  
  1.  Decimation in time (DIT) (b) Pd(t)
  2.  Decimation in frequency (DIF)

3.9.2.1 Radix or Base Algorithm


t
By a radix or R-algorithm, we mean a FFT algorithm
with radix R, where R is an integer. The number of T
samples N and R are related as  
N = Rm N = Rm (c) x*(t)
where m = 1, 2, 3 …where m = 1, 2, 3 …
Consider the following cases:
t
  1.  If R = 2, then N = 2m. So,
T
For m = 2, we have a 4-point, Radix-2 FFT.
For m = 3, we have a 8-point, Radix-3 FFT  
  2.  If R = 3, then N = 3m. So, for m = 2, we have Figure 3.11 |   Impulse train sampling of continuous-time
9-point, Radix-1 FFT. signal.

3.10  SAMPLING THEOREM FOR Pd (t) = ∑ d (t - nT )
-∞
CONTINUOUS-TIME SIGNAL
x * (t) = x(t)Pd (t)
Sampling theorem states that if sampling frequency of a ∞
message signal is greater than or equal to the Nyquist rate = x(t)∑ d (t - nT )
then perfect reconstruction of message signal is possible, -∞

Chapter 3.indd 222 3/24/2016 12:51:11 PM


3.11  LAPLACE TRANSFORM     223

So, the only values of x(t) having significance are at t = nT And rectangular pulse used to analyze zero order hold
∞ reconstruction is represented as shown in Fig 3.14.
x * (t) = ∑ x(nT )d (t - nT )
h0(t)
-∞
Now, by taking Fourier series expansion of Pd(t), we get

Pd (t) = ∑
1 jmw S t
e
−∞ T 1

∑ x(t)ejmw t
1
⇒ x * (t) = S

T -∞
Ts
Now, taking Fourier transform to both the sides, we get
∞ Figure 3.14 |   Rectangular pulse used to analyze zero
∑ X(f - mfs )
1 order hold reconstruction.
X * (f ) =
T -∞
The output of zero order hold is expressed in terms of
The spectrum of sampled-data signal using ideal impulse h0(t) as
sampling is shown in Fig. 3.12. ∞
x0 (t) = ∑ x(n)h0 (t - nTs )
n =-∞
x(f )
This equation is the convolution of the impulse sampled
signal xd(t) with h0(t)

−fn fn
x0 (t) = h0 (t) * ∑ x(n)d (t - nTs )
n =-∞
x∗(f) = h0 (t) * xd (t)
Now we take the Fourier transform of x0(t) using convo-
−fn fn fs−fn fs fs+fn 2fs−fn 2fs 2fs+fn lution multiplication property of FT, to get

Figure 3.12 |   Spectrum of sampled-data signal using X0 ( jw ) = H0 ( jw )Xd ( jw )


ideal impulse sampling.
Using results of Problem 3.13 and time shift property,
we have
sin(wTs /2)
h0 (t) ←→ H0 ( jw ) = 2e-jwTS /2
3.10.2  Zero-Order Hold Sampling FT
w
In this type of sampling the zero-order hold system sam-
ples a continuous-time signal x(t) at a given instant and
holds this value until the next instant at which another
3.11  LAPLACE TRANSFORM
sample is taken. The process of zero-order hold sampling
is depicted in Fig. 3.13.
Fourier analysis is extremely useful in the study of many
x(t) problems of practical importance involving signals and
LTI systems. System analysis in time domain involves the
x(n) Zero-order solution of differential equation as the evaluation of the
x(n) x0(t)
hold superposition integral. The Fourier transform provided an
alternative approach wherein a differential equation relat-
ing the input and output of a system was transformed
n t to an algebraic equation. Unfortunately, there are many
−1 1 3 −Ts Ts signals of interest that arise in system analysis for which
Figure 3.13 |   Zero-order hold sampling.
Fourier transform does not exist. However, we can find
a transform that applies to a wider class of signals than
Consider that h0(t) is used for zero-order hold circuit Fourier transform does. This can be accomplished by mul-
impulse response, where tiplying a signal x(t) by an exponential convergence factor

1, 0 ≤ t ≤ Ts
h0 (t) =  X(q + jw ) = ∫ x(t)e-qt e-jwt dt
0, otherwise -∞

Chapter 3.indd 223 3/24/2016 12:51:16 PM


224     CHAPTER 3:  SIGNALS AND SYSTEMS

Signals that are zero for t < 0, terme-qt where q is posi-   2.  Laplace transform of derivatives:

∫ udv = uv -∫ vdu
tive result in Fourier transform
Using s = q + jw, the above equation can be written as
u = x(t)

LT[x(t)] = X(s) = ∫ x(t)e -st
dt dv = e-st dt
-∞
v = − e −st
1
du = x ′(t),
The above expression shows the Laplace transfer, of s
function x(t) in time domain t, where s = q + jw. ∞
X(s) = ∫ x(t)e-st dt
Notation LT indicates Laplace transform 0

Function x(t) is multiplied by e-qt. The new function  x(t)e - st  ∞ 1 
=  - ∫  - e-st  x ′(t)dt
 -s 0  s 
x(t)e-qt shows integral convergence which needs proper 0
chosen value of q and the region of q for which the inte- x(0) 1 ∞ -st
gral converges is called region of convergence of = + ∫ e dt
s s 0
function x(t)

sX(s) = x(0) + ∫ x’(t) e-st dt
0

∫0 x ′(t) e-st dt = sX(s) - x(0)
3.11.1  Inverse Laplace Transform

Inverse Laplace transform is represented by  dx(t) 


LT  = sX(s) - x(0)
q + jw  dt 
LT-1[X(s)] = x(t) = ∫
1
X(s)est ds
2p
Therefore,
q - jw
 dxn (t) 
-1 LT  n -1 -
 = s X(s) - s x(0 ) - s
n n -2
x ′(0- ) - … x(n -1)′ (0- )
Here LT indicates inverse Laplace transform.  dt 
 dxn (t) 
LT  n -1 -
 = s X(s) - s x(0 ) - s
n n -2
x ′(0- ) - … x(n -1)′ (0- )
 dt 
3.11.2  Properties of Laplace Transform
Problem 3.26: The differential equation characteris-
  1.  Linearity:
ing a continuous-time invariant, linear system is
LT[a1x1(t) + a2x2(t)] = a1X1(s) + a2X2(s) given by
d2 y(t) dy(t)
where 2 2
+5 + 3y(t) = u(t)
dt dt
X1(s) = LT[x1(t)]
dy(0)
with initial condition y(0) = -2 and = -2
X2(s) = LT[x2(t)] dt

 onsider the example of Laplace transform of


C With the help of given initial condition, find Y(s)
cos w0t. It is given by of the LTI system.

1 
Solution: Given the differential equation
LT[cos w 0 t] = LT  e jw 0 t + e-jw 0 t 
1
 2 2  d2 y(t) dy(t)
2 2
+5 + 3y(t) = u(t)
dt dt

 1 jw 0 t 1 -jw 0 t  -st

From transformation of the derivatives, we get
=  e + e  e dt
-∞
2 2 1
2[s2 Y (s) - sy(0) - y(0)] + 5 [5Y (s) - y(0)] + 3Y (s) =
s
1 1 1 1 s 1
= + = 2 (2s2 + 5s + 3) Y (s) = + 2sy(0) + 2y(0) + 5y(0)
2 s - jw 0 2 s + jw 0 s + w 02 s
1
(2s2 + 5s + 3) Y (s) = - 4s - 4s - 10s
Similarly, we can find s
w0 -18s2 + 1
LT[sin w 0 t] = Y (s) =
s2 + w 02 s(2s2 + 5s + 3)

Chapter 3.indd 224 3/24/2016 12:51:23 PM


3.11  LAPLACE TRANSFORM     225

  3.  Laplace transform of an integral:


t Problem 3.28: Find the Laplace transform of the
y(t) = ∫ x(l )dl ­following waveform.
-∞
f(t)
 t  X(s) y(0- )
LT  ∫ x(l )dl  = +
-∞  s s
A
where t
y(0- ) = ∫ x(l )dl t = 0
T 2T 3T 4T
t
-∞

−A
Problem 3.27: Find the Laplace transform of cur-
rent in the following circuit.
t=0
Solution: Here the period is 2T. Therefore,

L C 2T 
 ∫ f (t)e-st dt 
+ 1
LT[f (t) =
x(t) 1 - e-2sT  0 
i(t) R
− T 2T 
 ∫ Ae-st dt + ∫ (-A)e-st dt 
1
=
1 - e-2sT  0 T 
Solution: For the given circuit 1  A -st T A -st
2T 
= - e dt + e dt 
t 1 - e-2sT  s s T  
∫ i(l)dl = x(t)
di(t) 1 0
L + Ri(t) +
 A -sT 
- 1) + (e-2sT - e-sT )
dt C -∞ 1 A
= - s (e
I (s) U (0 ) - 1 - e-2sT s 
⇒ LsI (s) = RI(s) +
A 
+ = X(s)
(1 - e-sT )2 
sC s 1
-2sT  s
=
where i(0-) = 0 because switch is open for t < 0 1-e  
0- 
A 1-e -sT 
= 
s  1 + e-sT 
C -∫∞
- 1
vc (0 ) = i(l )dl is the voltage across capaci-

tor at t = 0-. Therefore,


sX(s) - vc (0- )   5.  Complex frequency shift:
I (s) =
  R 1 
L s2 +   s + If    X(t) ←→ X(s)
LT
  
 L LC 

x(t)e-at ←→ x(s + a)


LT
then
  4.  Laplace transform of periodic functions:
f(t) = f(t + T )
T 2T (n +1)T Problem 3.29: Find the Laplace transform of
∫e ∫e ∫
-st -st -st
LT[f (t)] = f (t)dt + f (t)dt + e f (t)dt x(t) = e-at cos w 0 t.
0 T nT
T T Solution: We know that
∫e f (t)dt + ∫ e-st e-st f (t)dt +…
-st
=
s
0 0 LT[cos w 0 t] =
s + w 02
2

( )∫e
T
-sT -2sT -nsT -st
= 1+e +e +…e +… f (t)dt
Therefore, from shifting property
0

LT[x(t)] = LT e-at cos w 0 t  =


T s+a
∫ e f(t)dt
1 -st
= (s + a)2 + w 02
1 - e-sT 0

Chapter 3.indd 225 3/24/2016 12:51:30 PM


226     CHAPTER 3:  SIGNALS AND SYSTEMS

  6.  Delay theorem:


dX(s)
If    x(t) ←→ X(s)
LT or, LT[-tx(t)] =
ds
then dX(s)
-st0 or LT[tx(t)] = -
x(t - t0 ) ←→ e
LT
X(s) ds
For example, let us determine the Laplace trans-
form of x(t) = te-at u(t)
 onsider the example, for finding the Laplace
C
transform of x(t) = u(t) - 2u(t - 2). We know
that the Laplace transform is Since
e-at u(t) ←→
LT 1

LT[u(t)] = ∫ e-st dt =
1 s+a
s therefore,
0
d  1 
LT te-at u(t) = -
1
ds  s + a  (s + a)2
Therefore, =
LT[x(t)] = LT[u(t) - 2u(t - 2)]
  9.  Initial value theorem: Let us assume that
x(t) = 0 and t < 0 and x(t) contains no impulses as
= LT[u(t)] - 2LT[u(t - 2)]
higher order singularities at the origin. Initial value
of x(t), that is, x(0+) is defined as
- 2e-2s = [1 - 2e-2s ]
1 1 1
= x(0+ ) = lim sX(s)
s s s s →∞
  7.  Laplace transform of convolution of two   10.  Final value theorem:
sequences:
lim x(t) = lim sX(s)
t →∞ s→ 0
If    y(t) = x1(t) × x2(t)
then Problem 3.30: Find the initial and final value of
Y(s) = X1(s) X2(s)
2s3 + 5s2 + 12
X(s) =
The above relation can be proven as follows: s3 + 4s2 + 14s + 20
∞ ∞ 
∫ ∫ 1 2
 l - l l  e-st dt
Y (s) = LT[y(t)] = x ( )x (t )d Solution:
0 0 
x(0+ ) = lim X(s) = 2
s →∞
∞ ∞ 
= ∫ x1(l )  ∫ x2 (n)e-sn dn  e-sl dl
and
0  0  x(∞) = lim x(t) = lim sX(s)
t →∞ s→ 0

= X1(s)X2 (s) 2s3 + 5s2 + 12


= lim s ⋅ =0
s→ 0 s3 + 4s2 + 14s + 20

  8.  Differentiation in s-domain:   11.  Time scaling:


x(t) ←→ X(s)
LT
X(s) = ∫ x(t)e-st dt
-∞
1  s
x(at) ←→
then LT
X 

| a |  a
= ∫ (-t)x(t)e-st dt
dX(s)
ds -∞ The properties of the Laplace transform are listed
in Table 3.7.

Chapter 3.indd 226 3/24/2016 12:51:38 PM


3.11  LAPLACE TRANSFORM     227

Table 3.7 |   Properties of Laplace transform

Property Signal Transform ROC

x(t) X(s) R
x1(t) X1(s) R1
x2(t) X2(s) R2

Linearity ax1(t) + bx2 (t) aX1(s) + bX2 (s) At least R1 ∩ R2

Time shifting x(t - t0 ) e-st0 X(s) 

Shifted version of  [i.e., s


Shifting in the s-domain es0 t x(t) X(s - s0 ) is in the ROC if (s - s0) is
in ]

1  s Scaled ROC (i.e., s is in


Time scaling x(at) X 
a  a the ROC if (s/a) is in )

Conjugation x * (t) X * (s* ) 

Convolution x1(t) * x2 (t) X1(s)X2 (s) At least R1 ∩ R2

Differentiation in the d
time domain x(t) sX(s) At least 
dt

Differentiation in the
-tx(t)
d
s-domain X(s) 
ds


Integration in the time
x(t )d(t )
1
domain X(s) At least R ∩ {Re{s} > 0}
-∞
s

Initial value theorem If x(t) = 0 for t < 0 and x(t) contains no impulses or higher-order singularities at
t = 0, then

x(0+ ) = lim sX(s)


s →∞

Final value theorem If x(t) = 0 for t < 0 and x(t) has a finite limit as t → ∞, then

lim x(t) = lim sX(s)


t →∞ s→ 0

The signals and their Laplace transforms are listed in Table 3.8.

Chapter 3.indd 227 3/24/2016 12:51:48 PM


228     CHAPTER 3:  SIGNALS AND SYSTEMS

Table 3.8 |   Laplace transform of elementary functions

Signals Laplace Transform ROC

d (t)
1 All s

Re{s} > 0
u(t) 1
s

-u(-t) Re{s} < 0


1
s

t n -1 1
- u(t) Re{s} > 0
(n - 1)! sn

tn-1 1
− u(-t) Re{s} < 0
(n - 1)! sn

e-at u(t)
1
Re{s} > -a
s+a

e-at u(-t)
1
Re{s} < -a
s+a

tn-1 -at 1
- e u(t) Re{s} > -a
(n - 1)! (s + a )n

tn-1 -at 1
- e u(-t) Re{s} < -a
(n - 1)! (s + a )n

d(t − T ) e-sT All s

s
cos w 0 t u(t) Re{s} > 0
s + w 02
2

w0
sin w 0 t u(t) Re{s} > 0
s + w 02
2

s+a
e-at cos w 0 t u(t) Re{s} > -a
(s + a )2 + w 02

w0
e-at sin w 0 t u(t) Re{s} > -a
(s + a )2 + w 02

d nd (t)
un (t) = sn All s
dtn

u-n (t) = u(t) * … * u(t)


  1
Re{s} > 0
n times sn

Chapter 3.indd 228 3/24/2016 12:52:00 PM


3.12  Z-TRANSFORM AND ITS APPLICATION TO THE ANALYSIS OF LTI SYSTEM     229

3.12  Z-TRANSFORM AND ITS 3.12.2  Properties of Z-Transform


APPLICATION TO THE ANALYSIS
OF LTI SYSTEM   1.  Linearity:
x1(n) ←
→ X1(z)
Z
If

x2 (n) ←
→ X2 (z)
Transform techniques are used for analysis of signals and Z
linear time invariant system. The Z-transform is used to then
take discrete-time domain signals into a complex vari-
x(n) = a1x1(n) + a2 x2 (n) ←
→ X(z) = a1X1(z) + a2 x2 (z)
Z
able frequency domain.
Z-transform of a discrete-time signal x(n) is defined as
x(n) = a1x1(n) + a2 x2 (n) ←→ X(z) = a1X1(z) + a2 x2 (z)
Z

X(z) = ∑ x(n)z-n The relation can be proven as follows:
n =-∞
x(n) = a1x1(n) + a2x2(n)
where z is a complex variable.
Z-transform of x(n) is given by
Z-transform of x(n) is also represented by the operator
∞ ∞

X(z) = Z[x(n)]
X(z) = ∑ x(n)z-n = ∑ [a1x1(n) + a2 x2 (n)]z-n
n =-∞ n =-∞
whereas relationship between x(n) and X(z) is indicated by ∞ ∞
= a1 ∑ x1(n)z-n + a2 ∑ x2 (n)z-n
x(n) ←
→ X(z)
Z
n =-∞ n =-∞

Since Z-transform is an infinite power series it exists = a1X1(z) + a2 X2 (z)


only for those values of z for which these series converge.
 OC of X(z) is the overlap or intersection of indi-
R
vidual regions of convergence of X1(z) and X2(z).
3.12.1  Region of Convergence

The region-of-convergence (ROC) are the values Problem 3.32: Determine the Z-transform and ROC
of z for which the Z-transform converges, that is, of the signal.
all value of z for which X(z) attains a finite value. x(n) = [3(2n) - 4(3)n]u(n)
Therefore, ROC should be mentioned along with the Solution: Let us define
Z-transformation.
x1(n) = 2nu(n)
x2(n) = 3nu(n)
Problem 3.31: Find the Z-transform of the following   x(n) = 3x1(n) - 4x2(n)
sequence.
So according to linearity property
x(n) = {1, 2, 4, 6, 9}

X(z) = 3X1(z) - 4X2(z)
Solution: The Z-transform is given by and

∑ x(n)z-n
1
a n u(n)←
→ ROC |z| > |a|
Z
X(z) =
n =-∞ 1 - az-1
4 Therefore,
= ∑ x(n)z-n 1
X1(z) = ROC |z| > 2
1 - 2z-1
n =0

=1 ⋅ z 0 + 2 ⋅ z-1 + 4 ⋅ z-2 + 6 ⋅ z-3 + 9 ⋅ z-4


1
-1 -2 -3 -4
X2 (z) = ROC |z| > 3
= 1 + 2z + 4z + 6z + 9z 1 - 3z-1
X(Z) has finite value for all values of z except at z = 0. 1 1
X(z) = 3 × -1
-4
Therefore, ROC is entire z-plane except z = 0. 1 - 2z 1 - 3z-1

Chapter 3.indd 229 3/24/2016 12:52:05 PM


230     CHAPTER 3:  SIGNALS AND SYSTEMS

ROC ROC
where a is a constant.
The above relation can be proven as follows:
x1(z) x2(z) ∞
Z[an x(n)] = ∑ an x(n)z-n
n =-∞

∑ x(n)(a-1z)-n = X(a-1z)
2 3
=
n =-∞
If ROC of X(z) = r1 < |z| < r2, then ROC of
ROC of X(z) = R1 ∩ R2 (Common region or
X(a-1z) = r1 <
z
overlap region) < r2 = |a| r1 < |z| < |a| r2
a
Therefore, ROC of X(z) = z > 3
  2.  Time shifting: Problem 3.34: Determine the Z-transform of the
signal x(n) = an (coswn) ⋅ u(n).
x(n) ←
→ X(z)
Z

Solution: Given that x(n) = an (coswn) ⋅ u(n)


→ z-k X(z)
x(n - k) ←
Z

By using Euler’s identity, we have


The above relation can be proven as follows:
∞ e jwn + e-jwn
∑ -n coswn =
Z[x(n - k)] = x(n - k)z 2
n =-∞ Therefore,
Let n - k = p, therefore, x(n) =
1 n jwn
a e u(n) + an e-jwn u(n)
1
Therefore, 2 2
∞ ∞
∑ ∑
1  n jwn
x( p)z-( p +k) = z-k x( p)z-p Z a e u(n) + Z an e-jwn u(n)
1
Z[x(n - k)] = X(z) =
p =-∞ p =-∞
2 2
1 ∞ n jwn -n 1 ∞ n -jwn -n
Hence,    Z[x(n - k)] = z -k
X(z)
= ∑ a e z +2 ∑ a e z
2 n =0 n =0
 OC of z-kX(z) is the same as that of X(z) 1 ∞ 1 ∞
(
∑ aejw z-1 ) (
∑ ae-jw z-1 )
R n n
except for z = 0 if k = 0 and z = -∞ if k < 0. =
2 n =0
+
2 n =0
1 1 1 1
= w
+
Problem 3.33: Find Z-transform of x(n) = u(n) 2 1 - ae z
j - 1 2 1 - ae jw z-1
-

- u(n - 8). (1 - ae jw z-1 ) + (1 - ae-jw z-1 )


=
Solution: From the time shifting property, we have 2(1 - ae jw z-1 )(1 - ae-jw z-1 )

u(n)← →
1
z >1  e jw + e-jw 
1 - az-1 
Z
1 - z -1 
 2 
=
u(n - 8) ←
→z
Z -8 1
z >1 1 - az-1(e jw + e-jw ) + a2z-2
1 - z -1 1 - az-1 cos w
Therefore, = z >a
1 - 2az-1 cos w + a2z-2
1 -8 1
X(z) = -z
1 - z -1 1 - z -1
  4.  Time reversal:
  3.  Scaling in z-domain: If
x(n) ←
→ x(z) ROC r1 < |z| < r2
Z
If
x(n) ←
→ X(z) ROC r1 < |z| < r2
Z
→ X(z-1 ) ROC
1 1
x(-n) ← < z <
Z
r2 r1
then ∞

a x(n)←
n  z
→ X   ROC |a|r1 < |z| < |a| r2
Z Z[x(-n)] = ∑ x(-n)z-n
 a n =-∞

Chapter 3.indd 230 3/24/2016 12:52:14 PM


3.12  Z-TRANSFORM AND ITS APPLICATION TO THE ANALYSIS OF LTI SYSTEM     231

Put n = p in RHS
-∞ Problem 3.36: Determine Z-transform of the signal
Z[x(-n)] = ∑ x( p)z p
x(n) = nanu(n)
p =∞
∞ Solution: We have
= ∑ x( p)(z-1 )-p = x(z-1)
x1(n) = nan u(n) ←
→ X1(z) =
Z 1
p =-∞ 1 - az-1
ROC of x(n) is the inverse of that of x(-n) this ROC |z| > |a|
means that if z0 belongs to the ROC of x(n), then
1/z0 is in ROC of x(-n) From differentiation property
dX[z ]
Z[nan u(n)] = -z
dz
az-1
Problem 3.35: Determine the Z-transform of signal
d  1 
x(n) = A u(-n) = -z   =
dz  1 - az-1  (1 - az-1 )2
Solution: From the time reversal property, we have
1  ROC |z| > |a|
Z[u(n)] = ROC |z| > 1
1 - z -1   6.  Convolution in time domain:
Then
x1(n) ←
→ X1(z)
Z
1 If
Z[u(-n)] = ROC |z| < 1
1-z x2 (n) ←
→ X2 (z)
Z

then
x1(n) * x2 (n) ←
→ X1(Z )X2 (Z )
z
  5.  Differentiation in z -domain:

x(n) ←
→ X(z)
Z The above relation can be proven as follows:
If


then
x1(n) * x2 (n) = x1(k)x2 (n - k)
d[X(z)]
nx(n) ←
→ -z k =-∞
Z
dz

∑ x1(n) * x2 (n)z-n
The above relation can be proven as follows:
X(z) =

X(z) = ∑ x(n)z-n
n =-∞

n =-∞ ∞ ∞
Differentiating both sides with respect to z, we get = ∑ x1(k) ∑ x2 (n - k)z-n
k =-∞ n =-∞

[X(z)] = ∑ [x(n)z-n ]
d d
 ∞ 
=  ∑ x1(k)z-k  X2 (z)
dz n =-∞ dz
∞ k =-∞ 

d -n
= x(n) [z ]
n =-∞ dz (By time shifting property)

= ∑ x(n)(-n)z-(n +1) = X1(z) ⋅ X2 (z)
n =-∞
∞  OC of the product of X1(z) and X2(z) is the over-
R
= -z-1 ∑ nx(n)z-n lap as intersection of two individual sequences.
n =-∞

∑ nx(n)z-n = -z
d
Therefore, X(z)
n =-∞ dz Problem 3.37: Find the convolution of
d x1(n) = {{1, 3, 2}
Thus, Z[nx(n)] = -z X(z)
dz x2(n) = {{1, 2, 1}
Similarly, Solution:
 
k 2
∑ x1(n)z-n = -1 + 3z-1 + 2z-2
d
nk x(n) ←→ -z
Z
X(z) X1(z) =
 dz  n =0

Chapter 3.indd 231 3/24/2016 12:52:22 PM


232     CHAPTER 3:  SIGNALS AND SYSTEMS

  8.  Multiplication of two sequences:


2
X2 (z) = ∑ x2 (n)z -n
= 1 + 2z -1
+z -2
If    x1(n) ←
→ X1(z)
Z

  n =0
x2 (n) ←
→ X2 (z)
Z
-1 -2 -1 -2
X(z) = X1(z)X2 (z) = (-1 + 3z + 2z ) (1 + 2z +z )
then
-1 -2 -1 -2 -3
= -1 - 2z -z + 3z + 6z + 3z + 2z + 4z-3 + 2z-1 Z
-2
 z  -1
∫ X1(n )X2  n  z dn
1
x1(n)x2 (n) ←

= -1 - 2z-1 - z-2 + 3z-1 + 6z-2 + 3z-3 + 2z-2 + 4z-3 + 2z-1 2pj C

= -1 + z-1 + 7z-2 + 7z-3 + 2z-4


 here C is the closed contour which encloses the
w
origin and lies in the ROC that is common to
Let us take inverse Z-transform of X(z) both x1(n) and x2(n) ROC.

x(n) = Z -1 [x(z)]
  The above relation can be proven as follows:
= {−1, 1, 7, 7, 2}
Z-transform of x(n) = x1(n)x2(n) is

X(z) = ∑ x(n)z-n
  7.  Correlation of two sequences: n =-∞

If    x1(n) ←
→ X1(z) ∑ x1(n)x2 (n)z-n
Z
=
n =-∞
x2 (n) ←
→ X2 (z)
Z
Inverse Z-transform of x1(n) is given by
then

∫ X1(n )n dn
n -1
2pj 
1
g x1x2 (l) = ∑ → Rx1x2 (z) = x1 (z ) x2 (z-1 )
x1(n)x2 (n - l) ←
Z x1(n) =
n =-∞ C
Therefore,
→ Rx1x2 (z) = x1 (z ) x2 (z-1 )
x1(n)x2 (n - l) ←
Z

∫ X1(n )n dn ⋅ x2(n)
1 n -1
∞    x(n) = x1(n)x2 (n) =
2pj C
The above relation can be proven as follows:
∞ Therefore,
g x1x2 (l) = ∑ x1(n)x2 (n - l)
X(z) = ∑

x(n)z-n
n =-∞
n =-∞

= ∑ x1(n)x2 (n - l) ∞  1 
n =-∞ = ∑  ∫
 X1(n )n n -1dn x2 (n)z-n
∞  2pj C
n =-∞  
= ∑ x1(n)x2[-(l - n)]
 ∞  z   -1
-n
n =-∞
∫ 1  ∑ 2  n   n dn
1
= X (n )  x (n)
2pj C
Hence,
 n =-∞ 
g x1x2 (l) = x1(l) * x2 (-l)
 z
The term in the bracket is equal to X2   .
Taking Z-transform on both sides, we get n 
So
Z[g x1x2 (l)] = Z[x1(l) * x2 (−l)].
 z

 X1(n )X2   n -1dn
1
X(Z ) =
We know from time reversal property that if 2pj C n 
Z[x1(l)] = X1(z) then
If ROC of X1(n) and X2(n) be
Z[x2 ( −l )] = X2 (z −1 ) a1 < |v| < a2 and b1 < |z| < b2
Therefore,  z
then ROC of X2   will be
n 
Z[g x1x2 (l)] = X1(z)X2 (z-1 )
z
 OC of the Z-transform of correlation is the
R b1 < < b2 or, b1 |n| < |z| < b2 |n|
intersection or overlap of ROC of two individual n
sequences. Hence, a1b1 < |z| < a2b2

Chapter 3.indd 232 3/24/2016 12:52:32 PM


3.12  Z-TRANSFORM AND ITS APPLICATION TO THE ANALYSIS OF LTI SYSTEM     233

For causal signal


Problem 3.38: Determine the auto-correlation sequence ∞
of the signal Z[x(n)] = ∑ x(n)Z -n
x(n) = anu(n) for -1 < a < 1 n =0
Therefore,
Solution: As we know
1 Z[x(n)] = x(0) + x(1)z-1 + x(2)z-2 + ....
X(z) = -1
  ROC |z| > |a|
1 - az
We can write
and
  Z[x(n + 1)] = Zx(n) - Zx(0)
-1 1 1
X(z )=   ROC | z |<
1 - az |a| ⇒ Z[x(n + 1)] - Z[x(n)] = (z - 1)X(z) - Zx(0)
Therefore,
that is,
1 1
Rxx (z) = -1
.
1 - az 1 - az [x(1) - x(0)] + [x(2) - x(1)] + ... + [x(∞) - x(∞ - 1) = (z - 1)X(z) - Zx

=
1 [x(1)| a-| <
  ROC + [x1(2) - x(1)] + ... + [x(∞) - x(∞ - 1) = (z - 1)X(z) - Zx(0)
x(|0z)]| <
-1 |a|
1 - a(z + z )+ a 2
Taking z → 1
[x(1) - x(0)] + [x(2) - x(1)] + ... + [x(∞) - x(∞ - 1)]

  9. Initial value theorem: If x(n) is causal, that is, = -x(0) + lim(z - 1)X(z)
x(n) = 0 for n < 0), then x(0) = lim X(z) z →1
z →∞
x(∞) = lim(z - 1)X(z)
The relation can be proven as follows: z →1

From definition of Z-transform, we have Hence, the final value theorem.



X(z) = ∑ x(n)Z -n
n =-∞
Problem 3.39: Find the initial and final value at x(n)
Since x(n) is causal, so if its Z-transform X(z) is given by
¥
X(z) = å x(n)z-n X(z) =
0.6z 3
n =0 (z - 1)(z - 0.2z 2 - 0.85z + 0.6)
3
-1 -2
= x(0) + x(1)z + x(2)z +
Solution: Initial value x(0) is given by
Taking the limit z → ∞, we have
x(0) = lim X(z) = 0
-1 -2 z →∞
lim X(z) = lim [x(0) + x(1)z + x(2)z + .…]
z →∞ Z →∞ Final value is given by

or, x(0) = lim X(z) x(∞) = lim(z - 1)X(z)


z →∞ z →1

Hence, initial value theorem. 0.6z 3


= lim(z - 1)
  10.  Final value theorem: If sequence x(n) has
z →1 (z - 1)(z 3 - 0.2z 2 - 0.85z + 0.6)
finite value as n → ∞, called as x(∞), then this
0.6
value can be determined by the knowledge of one = = 1.0909
sided Z-transform, that is, 1 - 0.2 - 0.85 + 0.6

lim x(n) = lim[(z - 1)X(z)]


n →∞ z →1

The relation can be proven as follows: The properties of Z-transform are tabulated in Table 3.9.

Chapter 3.indd 233 3/24/2016 12:52:41 PM


234     CHAPTER 3:  SIGNALS AND SYSTEMS

Table 3.9 |   Properties of Z-transform

Signal x[n] —∞ <n <∞ Z-Transform X(z ) Region for Convergence

Rx, R1, R2
x(n), x1(n) and x2 (n) X(z), X1(z) and X2 (z)

x1(n) + bx2 (n) aX1(z) + bX2 (z) contains R1 ∩ R2


Rx except for the possible addition
(n - n0 ) z-n0 X(z) or deletion of z = 0 or z = ∞

|z0| Rx
z0n ⋅ x(n) X(z/z0 )

dX(z) Rx except for the possible addition


nx(n) -z or deletion of z = 0 or z = ∞
dz

X*(z*) Rx
x *[n]

1
X(z) + X * (z * )
Re {x(n)}
contains Rx
2

1
X(z) - X * (z * ) contains Rx
Im{x(n)} 2j

x(-n) X(1 / z) 1/Rx = {z : z-1 ∈ Rx }

X1(z) ⋅ X2 (z)
x1(n) * x2 (n) contains R1 ∩ R2

dn


1
X1(n )X2 (z /n )
x1(n) ⋅ x2 (n)
contains R1 R2
2pj n


* dn
∑ ∫ X1(n )X2 (1/n ) n
2pj 
1
Parseval’s theorem: x1(n) ⋅ x2* (n) = *

n =-∞

Initial value theorem: x(n) = 0, for n < 0  ⇒   lim X(z) = x[0]
z →∞

Table 3.10 lists the common Z-transform pairs along with their region of convergence.

Table 3.10 |   Common Z-transform pairs along with their region of convergence

Signal x(n) —∞ <n <∞ Z-Transform X(z) ROC

d (n)
1 All z

z > 0, if n0 > 0
d (n - n0 ) z-n0
z < ∞, if n0 < 0

1
u(n) |z| > 1
1 - z -1

(Continued)

Chapter 3.indd 234 3/24/2016 12:52:51 PM


3.12  Z-TRANSFORM AND ITS APPLICATION TO THE ANALYSIS OF LTI SYSTEM     235

Table 3.10 |   (Continued)  


Signal x(n) —∞ <n <∞ Z-Transform X(z) ROC

1
-u(-n - 1) |z| < 1
1 - z -1

1
an u(n) |z| > |a|
1 - az-1

a z -1
nan u(n) |z| > |a|
(1 - az-1 )2

1
-an u(-n - 1) |z| < |a|
1 - az-1

a z -1
-nan u(-n - 1) |z| < |a|
(1 - az-1 )2

a z -1
(n + 1)an u(n) |z| > |a|
(1 - az-1 )2

1 - z-1 cos w 0
( cos w 0 n)u(n) |z| > 1
1 - 2z-1 cos w 0 + z-2

z-1 sin w 0
(sin w 0 n)u(n) |z| > 1
1 - 2z-1 cos w 0 + z-2

1 - az-1 cos w 0
(an cos w 0 n)u(n) |z| > |a|
1 - 2az-1 cos w 0 + a2z-2

az-1 sin w 0
(an sin w 0 n)u(n) |z| > |a|
1 - 2az-1 cos w 0 + a2z-2



3.12.3  Inverse Z-Transform
∫ X(z)z dz = ∫z
n -1 n -1-k
x(k) dz
If X(z) is the Z-transform of signal x(n), then the proce- C k =-∞ C
dure for transforming from z- domain to the time domain From Cauchy integral theorem
is called inverse Z-transform.
1 k = n

 z n -1-k dz = 
1

2pj C 0 k ≠ n
X(z) = ∑ x(k)z-k
k =-∞
∫ X(z)z dz
n -1
2pj 
1
Multiplying both sides of the above equation with zn-1 x(n) =
and integrating both sides over a closed contour within C
the ROC of X(z) which encloses the origin There are following three methods to find inverse
∞ Z-transform:
∫ X(z)z ∫ ∑
n -1
dz = x(k)z n -1-k dz   1.  Direct evaluation by contour integration.
C C k =-∞   2.  Expansion into a series of terms, in the variables
where C denotes the closed contour in the ROC of X(z) z and z-1.
in counter clockwise direction. Therefore,   3.  Partial fraction expansion and table look up.

Chapter 3.indd 235 3/24/2016 12:52:59 PM


236     CHAPTER 3:  SIGNALS AND SYSTEMS

3.12.3.1 Inverse Z-Transform using Partial


Fraction Expansion Solution: Given that
1
X(z) = -1
Consider the following function 1 - 1.8z + 0.8z-2

X(z) = a 1X1(z) + a 2 X2 (z) + ... + a k Xk (z) X(z) z


or,      = 2
where a1, a2 …. are constant and X1, X2 … Xk(z) are the z z - 1.8z + 0.8
Z-transform of x1(n), x2(n) … xk(n)
z
=
and X(z) is the linear combination of individual (z - 1)(z - 0.8)
Z-transform of x1(n), x2(n) …. xk(n).
So inverse Z-transform of X(z) is A B
= +
(z - 1) (z - 0.8)
x(n) = a 1x1(n) + a 2 x2 (n) + ... + a k xk (n)  

Let us assume that Z-transform X(z), is rational in 5 -4


= +
nature, that is, (z - 1) (z - 0.8)
N (z) a + a1z-1 + a2z-2 + aM z-M
X(z) = = 0 -4
D(z) b0 + b1z-1 + b2z-2 + ... + bN z-N
5
= -1
+
1-z 1 - 0.8z-1
Let b0 = 1, then
(a) For ROC |z| > 1, the time sequence is causal,
N N -1 N -2 ( N -M ) therefore,
N (z) az + a1z + a2z + ... + aM z
X(z) = = N -1 N -2
D(z) z N
+ b1z + b2z + ... + bN x(n) = 5(1)n u(n) - 4(0.8)n u(n) = [5 - 4(0.8)n ]u(n)
(b) For ROC |z| < 0.8, both the signals are anti-causal,
N -1
X(z) a0z + a1z N -2 + a2z N -3 + ... + aM z N -M -1 therefore
=
z (z - g 1 )(z - g 2 )...(z - g N ) x(n) = 5[(1)n u(-n - 1)] - 4[-(0.8)n u(-n - 1)]

A1 A2 AN = [-5(1)n + 4(0.8)n ]u(-n - 1)


= + + ... +
(z - g 1 ) (z - g 2 ) (z - g N ) (c) For ROC 0.8 < |z| < 1, the first signal is anti-
where causal and second is causal, therefore,
(z -g i )X(z)   1    
x(n) = 5 Z −1  − 4 Z −1 
1
−1   −1  
Ai =
z z =g i
 1 − z    1 − 0.8z  
Ai = − 5 (1)n u(−n − 1) − 4(0.8)n u(n)
Inverse Z-transform of is given by
1 - g i z -1

  (g )n u(n) z > a


Z -1 
Ai if
-1
= i n 3.12.3.2 Inverse Z-Transform using Power
 1 - g iz  -( b i ) u(-n - 1) if z < a Series Expansion

The relation

Problem 3.40: Find the inverse Z-transform of the X(z) = ∑ x(n)z-n
following function: n =-∞
1
X(z) = = ... + x(-3)z 3 + x(-2)z 2 + x(-1)z + x(0) + x(1)z-1 + x(2)z-2 + ..
1 - 1.8z + 0.8z-2
-1

for following ROC: = ... + x(-3)z 3 + x(-2)z 2 + x(-1)z + x(0) + x(1)z-1 + x(2)z-2 + ...
(a) |z| > 1 gives power series expansion in the form
(b) |z| < 0.8
(c) 0.8 < |z| < 1 x(n) = {...x(-2), x(-1), x(0), x(1), x(2),...}

Chapter 3.indd 236 3/24/2016 12:53:06 PM


3.13  REALISABLE LTI SYSTEMS     237

3.13  REALISABLE LTI SYSTEMS


Problem 3.41: Determine the inverse Z-transform of
1
x(z) = by long division method if
2 - 4z + 2z-2
-1
A system is said to be realisable if it is causal and stable
(a) ROC |z| > 1
both. Stability and causality impose the same constraints
on the system function of a LTI system.
(b) ROC |z| < 1/2
For a stable system, the impulse response must be
 (a)    For ROC |z| > 1, that is, ROC is exterior of absolutely summable, that is
the circle, it is causal sequence. So power series ∞
∑ h(n) < ∞

)
expansion should be in negative power of z.
-1 -2 n =-∞
1/2-z + 3 / 2z
1 This is equivalent to the condition

1 ∓ 2z-1 ± z-2
∑ h(n) z-n < ∞
2 - 4z −1 + 2z −2 -
n =-∞
-1
2z - z -2
For z = 1, the ROC of the system junction must include
2 z-1 ∓ 4z-2 ± 2z-3 the unit circle if the system is stable.
-
For a causal system
3z-2 - 2z-3
h(n) = 0, for n < 0
-3 z-2 ∓ 6z-3 ± 3z-2
- The ROC of H(z) is exterior of the circle of radius R,
4z -3
- 3z -2 that is, z > R . Because no poles lie within the ROC,
Therefore, all the poles of H(z) must lie inside the unit circle for
­realizable system, that is,
+ z-1 + z-2 + ...
1 1 3
z >R 0 ≤ R <1
X(z) = -1 -2
=
2 - 4z + 2z 2 2

Then the inverse Z-transform of X(z)

⇒ x(n) = IZT[X(z)] = {1/2, 1, 3/2,...} Problem 3.42: Determine the step response of the

causal system described by the difference equation
(b)    If ROC |z| < 1/2, that is., ROC is interior of
the ­circle, then the integral x(n) is non-causal y(n) = y(n - 1) + x(n)
and requires a power series expansion in positive
Solution: The system function is
power of Z.
1
1 2 z 2 + 2z 3 + 7 2 z 4 + ... H(z) =
2z-2 - 4z-1 + 2 ) 1 So, pole at z =1.
1 - z -1

1 ∓ 4z ± z 2
− x(n) = u(n)
4z − z 2
Therefore,
4z ∓ 8z 2 ± 4z 3 X(z) =
1
- 1 - z -1
7z - 4z
2 3
Y (z) = X(z)H(z)
7z 2 ∓ 14z 3 ± 724 =
1
- (1 - z-1 )2
10z - 7z
3 4
So, double pole at z = 1. Therefore,
1 7
X(z) = z 2 + 2z 3 + z 4 + ...
2 2 y(n) = (n + 1)u(n)
Therefore, Z-transform form n ≤ 0 is
which is a ramp sequence. Thus y(n) is unbounded
X(z) = x(0)z + x(-1)z + x(-2)z + ...
0 1 2 even, when the input is bounded. Therefore, the
system is unstable.
x(n) = {...7/2, 2, 1/2, 0, 0}

Chapter 3.indd 237 3/24/2016 12:53:12 PM


238     CHAPTER 3:  SIGNALS AND SYSTEMS

Problem 3.43: Determine the response of the system Problem 3.45: The step response of an LTI system is
n-2
 1
d (n) =  
5 1
y(n - 1) - y(n - 2) + x(n) u(n + 2)
 3
y(n) =
6 6
To the input signal (a) Determine the impulse response h(n).
1
x(n) = d (n) - d (n - 1)
(b) Find the system function H(z) and sketch the
3 pole zero plot.
(c) Check is the system is causal and stable.
Solution: The system function is given by
1 1 Solution:
H(z) = =
1 - z -1 + z -2
5 1  1 -1   1 -1 
 1 - z   1 - z 
(a) The impulse response is given by
6 6 2 3
h(n) = d (n) - d (n - 1)
So poles at z = 1/2 and 1/3 n -2 n -3
 1  1
1 -1 =  u(n + 2) -   u(n + 1)
X(z) = 1 − z  3  3
3
-4 -4
 1  1
Y (z) = H(z)X(z) =  d (n + 2) -   d (n + 1)
 3  3
z −1
1
= -3
1−1 z  1  1
n
+  d (n + 1) - 18   u(n)
 3  3
Therefore,
 1
2
 1
n
y(n) =   u(n)
 2 = 81d (n + 2) - 54d (n + 1) - 18   u(n)
 3

(b) The system function is given by


18
Problem 3.44: Compute the response of the system H(z) = 81z 2 - 54z -
1 - z -1
1
y(n) = 0.7 y(n - 1) - 0.12y(n - 2) + x(n - 1) + x(n - 2) 3
to the input x(n) = nu(n). Is the system stable? 81z(z-1 )
=
1 - z -1
1
Solution: Given x(n) = nu(n).
3
Y (z) z -1 + z -2 So, zero at z = 0 and z = 1; pole at z = 1/3
=
X(z) 1 - 0.7z-1 + 0.12z-2  he is not causal but stable as the pole is inside
T
z -1 the unit circle.
X(z) =
(1 - z-1 )2
z -2+z -3
Y (z) =
  
(1 - z −1 )2  1 - z-1   1 - z-2 
3 2 Problem 3.46: A system is described by difference
 10  5  equation
All the poles lie in the unit circle, so the system is
stable. y(n) + y(n - 1) = x(n)
n<0
4.76z-1
y(n) = 0
-12.36 -26.5
Y (z) = -1 2
+ -1
+
(1 - z (1 - z æ 3 -1 ö (a) Determine the transfer function and stability.
ç1 - z ÷
) )
è 10 ø (b) Determine the impulse response h(n) and show
38.9 that it behaves in accordance with (a).
+
æ 2 -2 ö
ç1 - z ÷
Solution:
è 5 ø (a) The transfer function is given by
é æ 3 ö
n
æ2ö ù
n
y(n) = ê4.76n - 12.36 - 26.5 ç ÷ + 38.9 ç ÷ ú u(n) Y (z) + Z -1Y (z) = X(z)
êë è 10 ø è 5 ø úû

Chapter 3.indd 238 3/24/2016 12:53:16 PM


IMPORTANT FORMULAS     239

Therefore, Problem 3.48: Consider the system shown, with


Y (z) 1 impulse response h (n ) = an u (n ) - 1 < a < 1
H(z) = =
X(z) 1 + z-1 Determine output y(n) of the system to the input
signal x(n) = u(n + 5)-u(n - 10).
Pole is at z = -1, therefore system is stable.
(b) The impulse response is given by u(n)
h(n) = (-1) u(n)n +
x(n) y(n)

H(z) = ∑ (-1)n z-n −
n =0 z−2 u(n)
ROC

|(-1)z-1 < 1, |z > 1 Solution: S(n) = u(n) * h(n)



 ince ROC is contained in the unit circle, the
S
system is stable, as shown in part (a).
= ∑ u(k)h(n - k)
k =0
n
= ∑ h(n - k)
k =0
n
∑ an-k
Problem 3.47: Determine the range of values of the
parameter a for which the LTI system with impulse =
k =0
response
an n ≥ 0, n is even a n +1
-1
h(n) =  = ,n≥0
a -1
0 otherwise
is stable For  x(n) = u(n + 5) - u(n - 10)
a n +6 - 1 an -9 - 1
Solution: The impulse response is given by S(n + 5) - S(n - 10) = u(n + 5) - u(n - 10)
9 -1 9 -1
∞ ∞ n

∑ ∑ aS(n + 5) - S(n - 10) = a


n +6
-1 an -9 - 1
h(n) = u(n + 5) - u(n - 10)
n =-∞ n = 0, 9 -1 9 -1
n even

∞ 2n
y(n) = x(n) * h(n) - x(n) * h(n - 2)
= ∑ a
n =0 a n +6 - 1 an -9 - 1 an + 4 - 1
y(n) = u(n + 5) - u(n - 10) - u(n + 3) +
=
1 a -1 a -1 a -1
1- a
2

a -1 n +6
an -9 - 1 an + 4 - 1 an -11 - 1
The system is stable if a < 1
y(n) = u(n + 5) - u(n - 10) - u(n + 3) + u(n - 12)
a -1 a -1 a -1 a -1

IMPORTANT FORMULAS

1. Formulas given in Table 3.1. 3. Power and energy signals


2. Periodic and aperiodic signals (a)  Instantaneous power
1 2
p(t) = v(t)i(t) =
v (t)
x(t + T0 ) = x(t)   -∞ < t < ∞ R
(b) Total energy expended over time interval t1
≤ t ≤ t2
where T0 is constant period. Any deterministic t2 t2

∫ ∫ R v (t)dt
signal not satisfying above property is called an 1 2
p(t)dt =
aperiodic signal. t 1
t 1

Chapter 3.indd 239 3/24/2016 12:53:21 PM


240     CHAPTER 3:  SIGNALS AND SYSTEMS

(c)  Average power over the interval t1 ≤ t ≤ t2 8.   Time invariant and time varying systems
x(n) 
→ y(n)
t2 t2 T

∫ ∫
1 1 1 2
p(t) = v (t)dt
t2 - t1 t1
t2 - t1 t1
R x(n - k) 
→ y(n - k)
T

(d)  Normalized average power P of x(t)   The system not obeying this condition is said to be
T time varying.

1 2
P = lim x(t) dt
T →∞ 2T 9.  Linear and non-linear systems
-T

(e)  Normalized average power P of x(n) T [a1x1(n) + a2 x2 (n)] = a1 T [x1(n)] + a2 T [x2 (n)]
N   A system not satisfying this condition is said to be
∑ x(n)
1 2
P = lim non-linear.
N →∞ 2N + 1
n =-N
10. Causal and non-causal systems
4. Energy signal   t[x1(t)] = t[x2(t)] for t ≤ t0
(a) Total energy in continuous-time signal x(t)   y(n) = F[x(n), x(n - 1), x(n - 2) ….]
over time interval t1 ≤ t ≤ t2
t2   where F[.] is any arbitrary function.

∫ x(t)
2   If the system does not satisfy this condition, it is
E= dt
t1 called non-causal
(b) Total energy in discrete-time signal x(n) over 11. Stable and unstable systems
time interval n1 ≤ n ≤ n2
n2 A system is bounded-input, bounded-output (BIBO)

2 stable
E= x(n)
n = n1 if x(t) ≤ M x < ∞ then y(t) ≤ M y < ∞
5. Even and odd signals
12. Output of LTI system is given by weighted sum of
(a) A continuous-time signal x(t) and discrete- time shifted impulse response called convolution sum
time signal x(n) are said to be even if ∞ ∞
x(-t) = x(t) for all t x(n) * h(n) = ∑ x(k) h(n - k) = ∑ x(n - k)h(k)
k =-∞ k =-∞
and x(-n) = x(n) for all n
13. Operations performed to find convolution:
1
xe (t) = [x(t) + x(-t)]
2 •  Folding: x(k) → x(-k)

(b) A continuous-time signal x(t) and discrete-


•  Shifting: x(-k) → x(n - k)
time signal x(n) are said to be odd if •  Multiplication: x(n - k) → h(k)x(n - k)
x(-t) = -x(t) ∞

x(-n) = -x(n)
•  Addition: ∑
h(k)x(n - k)
-∞
1 14. Convolution integral
xo (t) = [x(t) - x(-t)] ∞
2
6. Input-output relationship for continuous-time system
x(t) = ∫ x(t )d (t - t )dt
-∞
y(t) = T [x(t)] Output for continuous-time signal

x(t) → y(t) y(t) = x(t) ∗ h(t) = ∫ x(t ) h(t - t )dt


-∞
7. Input-output relationship for discrete-time system 15. Differential equation representation of LTI system
dny d n -1 y dmx d m-1x
+ an -1
dy
y(n) = T [x(n)] n n -1
+ … + a1 + a0 y = b m m + bm-1 m-1 + … +
dt dt dt dt dt
x(n) → y(n) dny d n -1 y dmx m -1
+ an -1
dy d x dx
+ … + a1 + a0 y = b m m + bm-1 + … + b1 + b0 x
dtn dtn-1 dt dt dtm-1 dt

Chapter 3.indd 240 3/24/2016 12:53:28 PM


IMPORTANT FORMULAS     241

16. Input-output relationship of LTI system by differ- T

∫ x(t)sin nwt dt
1
ence equation bn =
T 0
N M
∑ ak y(n − k) = ∑ bk x(n - k) (c) All the properties of continuous-time Fourier
k =0 k =0
series given in Table 3.2.
17. Continuous-time and discrete-time LTI systems
22. Discrete-time Fourier series
(a)  Continuous-time LTI system output (a)  Expression
y(t) = x(t) * h(t) N -1

  where h(t) is the impulse response. Then


x(n) = ∑ Ck ej2pkn /N for n = 0, 1, …….N - 1
k =0
∞ ∞

∫ ∫
(b)  Coefficients
y(t) = x(t - t )h(t )dt = x(t ) ⋅ h(t - t )dt
-∞ -∞ N -1
∑ x(n)e-j2pnk / N for k = 0, 1, …., N - 1
1
Ck =
(b)  Discrete-time LTI system input and output N n =0

x(n) = ∑ ak < d k (n) (c) All the properties of discrete-time Fourier se-
k =-∞ ries ­given in Table 3.3.
∞ ∞ 23. Power density spectrum of periodic signal
y(n) = ∑ ak h(n - k) = ∑ h(n - k)x(k)
k =-∞ k =-∞ N -1 N -1 N -1
∑ ∑ ∑ | Ck |2
1 2 1
P = x(n) = x(n)x+ (n)=
y(n) = (h * x) [n] N n =0 N n =0 k =0

18. Static and dynamic LTI systems 24. Continuous-Time Fourier Transform
(a)  Output of static system
(a)  Expression
y(t) = h[u(t)] = h[u1(t), u2 (t) …] ∞
X( jw ) = ∫ x(t)e-j2pft dt
(b)  Output of dynamic system
-∞
y(t) = h ( t, u(t)) = h(tu1(t), u(t), t u(t)…) 2 (b) Energy density spectrum for aperiodic signal
(Parseval’s relation)
19. Invertible LTI system ∞

[x * d (n)] = x(n)
Ex = ∫ x(t)x * (t)dt
-∞
(x * h * h-1 ) [n] = x(n)
∞ ∞  ∞
(n * h-1 ) = d (n) = ∫ X * (f )df  ∫ x(t)e-j2pft
dt  = ∫ x(f ) df
2

-∞ -∞  -∞
20. Unit step response of LTI system (continuous time)
t (c) All the properties of continuous-time Fourier
y(t) = ∫ h(t )dt ­transform listed in Table 3.4.
-∞ (d) Continuous-time Fourier transform pairs in
21. Continuous-Time Fourier series Table 3.5.
(a)  Expression 25. Discrete-Time Fourier transform

x(t) = a = a0 + ∑ (an cos nwt + bn sin nwt) (a)  Expression
n =1 ∞
(b)  Coefficients X(e jw ) ≡ DTFT[x(n)] = ∑ x(n)e-jwn
T n =-∞

∫ x(t)dt
1 (b)  Inverse transform
a0 =
T p
x(n) ≡ DTFT-1[X(e jw )] = ∫-p X(e
jw
)e jwn dw ; (for - ∞ <
0 1
T 2p
∫ x(t)cos nwt dt
1
an = p
x(n) ≡ DTFT [X(e jw )] =
-1
∫-p X(e
jw
)e jwn dw ; (for - ∞ < n < ∞)
1
T 0
2p

Chapter 3.indd 241 3/24/2016 12:53:36 PM


242     CHAPTER 3:  SIGNALS AND SYSTEMS

(c) All the properties of discrete-time Fourier 30. Laplace transform


transform listed in Table 3.6. (a)  For function x(t) in time domain t
(d) Basic pairs for discrete-time Fourier transform ∞
listed in Table 3.7. LT[x(t)] = X(s) = ∫ x(t)e-st dt
-∞
26. Discrete Fourier transform (DFT) where s = q + jw
(a)  Expression (b)  Inverse Laplace transform is represented by
N -1 2p
-j q + jw
∑ x(n)e
kn


x(k) = N -1 1
LT [X(s)] = x(t) = X(s)est ds
n =0 2p q - jw
k = 0, 1, 2,… N - 1
(c) All the properties of Laplace transform listed
2p in Table 3.9.
-j
In terms of twiddle factor WN = e N (d)  Laplace transform pairs listed in Table 3.10.
31. Z-transform
N -1
∑ x(n)WN nk
(a)  Expression for discrete-time signal
x(k) =

∑ x(n)z-n
n =0
X(z) =
(b)  Inverse DFT
n =-∞

N -1 2p (b) All the properties of Z-transform listed in


∑ x(k)e
1 j kn
x(n) = N Table 3.11.
N k =0
32. Inverse Z-transform
n = 0, 1, 2,… N - 1
(a)  Expression

(c)  All the properties of DFT listed in Table 3.8. X(z) = ∑ x(k)z-k
27. Sampling theorem k =-∞

∫ X(z)z dz
n -1
2pj 
1
Sampling rate ≥ Nyquist rate (2fmax )
x(n) =
C

(b) Inverse Z-transform of X(z)by partial fraction


28. Impulse train sampling of a continuous-time signal
method
∞  ∞ 
x * (t) = x(t)Pd (t) = x(t)∑ d (t - nT ) as Pd (t) = ∑ d (t - nT )x(n) = a 1x1(n) + a 2 x2 (n) + ... + a k xk (n)
-∞  -∞ 
Ai
(c)  Inverse Z-transform of
∞  ∞  1 - g i z -1
= x(t)Pd (t) = x(t)∑ d (t - nT ) as Pd (t) = ∑ d (t - nT )
-∞  -∞    (g )n u(n) z > a
Z -1 
Ai if
-1
 = i n
 1 - g iz  - (bi ) u(-n - 1) if
where x*(t) is sampled data; x(t) is original
z < a
­continuous-time signal and Pd(t) is train of impulse
function. (d)  Inverse Z-transform by power series expansion
29. Zero-order hold sampling ∞

Output in terms of h0(t)


X(z) = ∑ x(n)z-n
n =-∞

= ... + x(-3)z 3 + x(-2)z 2 + x(-1)z + x(0) + x(1)z-1 + x(2)z-
x0 (t) = ∑ x(n)h0 (t - nTs )
n =-∞
= ... + x(-3)z 3 + x(-2)z 2 + x(-1)z + x(0) + x(1)z-1 + x(2)z-2 + ...
sin(wTs / 2)
h0 (t) ←→ H0 ( jw ) = 2e-jwTS /2
FT
(e)  Common Z-transform pairs given in Table 3.12.
w

Chapter 3.indd 242 3/24/2016 12:53:41 PM


SOLVED EXAMPLES     243

SOLVED EXAMPLES

1. Which of the following is true? Solution:


(a) d (at) = d (t)
Final value theorem is applicable only when the
system is stable. Here, inverse Laplace transform of
(b) d (at) = ad (t) F(s) is cos(t) which is only marginally stable.
1
(c) d (at) = d (t) Ans. (a)
a 0
(d) None of the above 5. ROC of ∑ d (n − p) is
p =−∞
Solution:  Time scaling property of d (t) (a) |z| > 1 (b) |z| < 1
1 (c) |z| = 1 (d) 0 < |z| < 1
d (t) = d (t) a > 0
a
Ans. (c) Solution:  The given signal is
0
2. ROC of x1(n) and x1(−n) is the x(n) = ∑ d (n − p)
(a) same p =−∞
(b) reciprocal of each other x(n) = d (n) + d (n + 1) + d (n + 2)d (n + p)d (n + ∞)
(c) negative of each other
x(n) = u(−n)
(d) complement of each other
Therefore, |z| < 1
Ans. (b)
Solution: 
x1(n) ↔ X1(z)     ROC Rx 6. A causal system is always
By time reversal property (a) stable
x1 (−n) ↔ X1(1/z)     ROC 1/Rx (b) unstable
Hence, these are reciprocal of each other. (c) may or may not be stable
Ans. (b) (d) cannot be determined
3. Discrete time signal x(k) = (−1)k is periodic with Solution:  A causal system may or may not be
fundamental period stable. For stability its ROC must include unit
(a) 1 circle otherwise it is unstable.
(b) 3 Ans. (c)
(c) 2 7. A finite impulse response (FIR) system has the
(d) None of above ROC

Solution:  The given signal can be plotted as (a) inside the circle
follows. (b) outside the circle
(c) entire z-plane except z = 0
x(k) (d) entire z-plane except z = ∞ and z = 0
1
−1 1 3 Solution:  Consider the following FIR system
−2 0
K
2 x(n) = {2, 4, 5, 7, 0, 1}

−1 Then on Z-transform

The function repeats itself in 2 seconds so the X(z) = 2z 2 + 4z + 5 + 7z−1 + z−3


­fundamental period is 2 seconds.
Ans. (c) Thus ROC is entire z-plane except z = 0 and z = ∞.
s Ans. (d)
4. Final value of F (s) = 2 is
s +1 8. System 5y + 3y − 4 = x(t) is
(a) cannot be determined (a) linear and unstable
(b) 0 (b) linear stable
(c) 1 (c) non-linear and unstable
(d) ∞ (d) non-linear and stable

Chapter 3_Solved Examples and Practice Exercise.indd 243 3/23/2016 10:42:37 AM


244     CHAPTER 3:  SIGNALS AND SYSTEMS

Solution:  Given the system 5y + 3y − 4y = x(t) 10. The continuous time system described by
can be represented as y(t) = x2(t) is

d2 y dy (a) causal, linear and time variant


5 +3 − 4y = x(t) (b) causal, linear and time invariant
dt2 dt (c) causal, non-linear and time invariant
So the system is linear. (d) non-causal, non-linear and time invariant
Now taking Laplace transform on both sides, we get
Solution:  In the given system, y(t) depends on x(t),
5s Y (s) + 3sY (s) − 4Y (s) = X(s)
2 that is, present value. Therefore, system is causal.
Also,
Y (s)
=
1
=1 a 1x1(t) → y1(t) = a 1x12 (t)
X(s) 5s2 + 3s − 4
a 2 x2 (t) → y2 (t) = a 2 x22 (t)
Now,
Now  [a 1x1(t) + a 2 x2 (t)]2 = y3 (t) = a 12 x12 (t) + a 22 x22 (t) + a 1a 2 x1(t)x2
−3 ± 32 − 4 × 5 ×−4
s1 , s2 =
2×5 [a 1x1(t) + a 2 x2 (t)]2 = y3 (t) = a 12 x12 (t) + a 22 x22 (t) + a 1a 2 x1(t)x2 (t)
−3 ± 89 −3 ± 9.4339
= =
10 10 Therefore, y3 (t) ≠ y1(t) + y2 (t)
Hence, the system is non-linear.
Therefore,
For the given system, we have
s1 = 0.64339
y(t, k) = x2(t - k)
s2 = −1.24339
Because of pole at s = 0.64339, system is unstable. and y(t - k) = x2(t - k)
Ans. (a) Therefore, system is time invariant.
Hence, the system is causal, non-linear and time
9. Inverse Fourier transform of u(w) is
invariant.
j 1 j Ans. (c)
(a) d (t) + (b)  d (t) +
pt 2 pt 11. x(n) = k2|n| for k < 1 is
1 j 1
(c)  d (t) + (d)  d (t) + sgn(t) (a) an energy signal
2 2pt 2 (b) a power signal
(c) neither energy nor a power signal
Solution: (d) an energy as well as power signal

x(t) X(w) Solution:  x(n) = k2n


For k < 1
X(t) 2px(-w)

u(t) pd(w) + 1
jw

Using the property of duality of Fourier transformer


0 n →∞
1 + pd(t) 2pu(-w)
jt As the amplitude is bounded at infinity therefore
the system is energy for n ≥ 0.
- 1 + pd(-t) 2pu(w) For −∞ < n < ∞ the signal is neither energy
jt
nor-power.
- 1 + pd(t) 2pu(w) Ans. (c)
jt
12. ROC of causal LTI system
u(w) - 1 + 1 d(t)
j2pt 2 (a) is entire s-plane
u(w) j + 1 d(t) (b) is right-half of s-plane
2pt 2 (c) is left half of s-plane
Ans. (c) (d) none of the above

Chapter 3_Solved Examples and Practice Exercise.indd 244 3/23/2016 10:42:55 AM


SOLVED EXAMPLES     245

Solution:  An LTI system is causal if its output 15. The signal x(n) = (0.6)nu(n) is
depends on the current and past input x(t). If the
(a) stable (b) unstable
system is initially relaxed with y(t) y <0 = 0 then its
response y(t) = h(t) to an impulse x(t) = d (t), is,
(c) data sufficient (d) None of above
h(t) = h(t) u(t). Its response to general input x(t) is
Solution:  Taking Z-transform of the given signal,
∞ ∞ we have
y(t) = h(t) * x(t) = ∫ h(t )x(t − t )dt = ∫ h(t )x(t − t )dt

−∞ 0
X(z) = ∑ (0.6)n z−n
Ans. (b) n =0

= 1 + 0.6z−1 + (0.6z−1 )2 + ...


13. The Fourier transform of rectangular pulse is
(a) impulse (b) sinc function
(c) rectangular pulse (d) triangular pulse 1
=
1 − 0.6z−1
1, t ≤ t /2
Solution: x(t) = 
0, t > t /2 ROC |0.6z−1| < 1 = 0.6 < |z| = |z| > 0.6, that is, ROC
Therefore, contains unit circle. Hence given signal is stable.
Ans. (a)

FT[x(t)] = ∫ x(t)e−jwt dt 1
16. At t = 0, inverse Laplace transform of
−∞ (s + 1)(s2 − 1)
is equal to
t /2
1 −jw t t /2 (a) 1/4 (b) 1
= ∫ e−jw tdt = − e
jw
(c) 1/2 (d) 0
−t /2
−t /2

Solution: Using partial fraction expansion method, 
etwt /2 − e−jwt /2 we have
=
jw 1 −1 1 1
= − +
(s + 1)(s − 1)
2 4(s + 1) 2(s + 1)2 4(s − 1)
2 j sin wt /2 sin wt /2
= =t
jw wt /2 Therefore,
  −1 −t 1
= t sin c(wt /2) LT−1  e − + e−t + et = x(t)
1 1
=
 (s + 1)(s − 1) 
2 4 2 4
Ans. (b)
At t = 0
14. The period of signal y(t) = 5 sin 20pt + 7 cos 24pt
−1 1 1
(a) 40 (b) 30 x(0) = − ×0 + = 0
4 2 4
(c) 60 (d) 10
By using initial value theorem
Solution:
y(t) = 5 sin 20pt + 7 cos 24pt x(t) = sX(s)
lim t → 0 lim s →∞
w 20p
T1 = = = 10 1
2p 2p sX(s) = s ⋅
lim s →∞ (s + 1)(s2 − 1)
w 24p
T2 = = = 12 s 1
2p 2p = ⋅
s  1  1
 1 +   s − 
T1 10 5 s s
= =
T2 12 6 x(t) = 0
lim t →0
T = 6 × 10 = 60 seconds
Ans. (c) Ans. (d)

Chapter 3_Solved Examples and Practice Exercise.indd 245 3/23/2016 10:43:23 AM


246     CHAPTER 3:  SIGNALS AND SYSTEMS

17. If X(z) = log (1 + az−1) then signal x(n) will be  


= az−1 
dX(z) 1
−z −  , |z| > |r|
an an  1 − (−a)z 
1
(a) (−1)n +1 u(n − 1) (b) (−1)n +1
dz
u(n)
n n
Now from differentiation property
an
(c) (−1)n +1 u(n − 1) (d) None of above nx(n) = a(−a)n−1 u(n - 1)
n
Therefore,
Solution:  Taking derivative of X(z), we have
an
x(n) = (−1)n +1 u(n − 1)
dX(z) −az−2 n
=
dz 1 + az−1 Ans. (a)

PRACTICE EXERCISES

Set 1 (One Mark Questions) 5. What is the period of the sinusoidal signal x(n) =
5 cos [0.2 pn]?
1. The Fourier series of a real periodic function has
(a) 10 (b) 5
only
(c) 1 (d) 0
I. cosine terms if it is even
6. When y(t) ↔Y(jw); x(t) ↔ X(jw); h(t) ↔ H(jw).
II. sine terms if it is even
What is Y(jw)?
III. cosine terms if it is odd
IV. sine terms if it is odd (a) X(jw)/H(jw)
(b) X(jw) H(jw)
Which of the above statements are correct?
(c) X(jw) + H(jw)
(a) I and IV
(d) X(jw) - H(jw)
(b) I and III
(c) II and IV(d) II and III 7. If the unit step response of a network is (1 − e−at),
then its unit impulse response is
2. If the Fourier transform of x(t) is 2/w sin (pw),
then what is the Fourier transform of e  j5tx(t)? (a) ae−a t (b) a −1e−at
(a) [2/(w−5)] sin(pw) (c) (1 − a −1 )e−a t (d) (1 − a )e−at
(b) [2/w sin] {p(w−5)} 8. An analog signal has the spectrum as shown in
(c) [2/(w+5)] sin [p(w+5)]} the following figure. The minimum sampling rate
(d) [2/(w−5)] sin [p(w−5)] needed to completely represent this signal is
3. A real signal x(t) has Fourier transform x(f ). Which
Amplitude
one of the following is correct?
(a) Magnitude of x(f ) has even symmetry, while 1
phase of x(f ) has odd symmetry
(b) Magnitude of x(f ) has odd symmetry, while f(kHz)
phase of x(f ) has even symmetry −1.5 −1 0 1 1.5
(c) Both magnitude and phase of x(f ) have even (a) 3 kHz
symmetry (b) 2 kHz
(d) Both magnitude and phase of x(f ) have odd (c) 1 kHz
symmetry (d) 0.5 kHz
4. Which of the following is the correct relation? 9. The system characterised by the equation
(a) FT(at) ←
→ aFT(w /a) y(t) = ax(t) + b is
(b) FT(at) ←
→ aFT(aw ) (a) linear for any value of b
(b) linear if b > 0
(c) FT(t/a) ←
→ aFT(w /a)
(c) linear if b < 0
(d) FT(at) ←
→ (1/a)FT(w /a) (d) non-linear

Chapter 3_Solved Examples and Practice Exercise.indd 246 3/23/2016 10:43:41 AM


PRACTICE EXERCISES     247

10. A continuous time periodic signal x(t), having a 16. Pick the correct relation
period T, is convolved with itself. The resulting
N
signal is k+
(a) WN 2 = −WNk
(a) not periodic.
(b) periodic with a period T. k+
N
(c) periodic having a period 2T. (b) WN 2 = −WNk
(d) periodic having a period T/2.
N
11. The energy of the following signal is k+
= −WN
2 N /2
(c) WN
A
N
k+
= −WN
2 N /2
(d) WN
t
−T1/2 0 T1/2 17. Discrete time system is stable if the poles are
(a) within unit circle
A2 2
(a)  (b) A (b) outside unit circle
2 (c) on the unit circle
(c) AT12 (d) A2 T1 (d) None of the above

12. What is the Nyquist frequency for the following 18. Fourier transform of exponential signal ejwt is
signal? (a) constant
x(t) = 2 cos50p t + 10 sin200p t − cos100p t
(b) rectangular gate
(c) an impulse
(a) 50 Hz (b) 100 Hz (d) a series of impulse
(c) 200 Hz (d) 300 Hz
13. The final value of x(t) = [2 + e−2t ] u(t) is Set 2 (Two Marks Questions)

1. Let x(t) ↔ X( jw ) be Fourier transform of the


(a) 2 (b) 3
(c) e−3t (d) 0
signal. X(5t - 3) in terms of X(jw) is given as
14. Match List-I with List-II and select the correct
j 3w j 3w
answer using the code given in the list below. 1 − 5 X
jw  1 5 X
jw 
(a)  e   (b)  e  
List-I List-II 5 5  5 5 
 1
n
1. x(n) =   u(n)  jw   jw 
(c)  e−j3w X  (d)  e j3w X 
(a) Even signal
 4
1 1
 5   5 

5 5
(b) Causal signal 2. x(−n) = x(n)
(c) Periodic signal 3. x(t) u(t) 2. For a random variable x having the probability
(d) Energy signal 4. x(n) = x(n + N) density function (PDF) as shown in the following
figure, what are the values of the mean and the
(a) A B C D
variance, respectively?
2 3 4 1
(b) A B C D P (x)
1 3 4 2
(c) A B C D
1/4
2 4 3 1
(d) A B C D
1 4 3 2
15. What is the 10-point DFT of x(n) in the following
function?
x(n) = d (n) + 2d (n − 5)
(a) 1/2 and 2/3
(b) 1 and 4/3
(a) 1 (b) 2(−1)k (c) 1 and 2/3
(c) 1 + 2(−1)k (d) None (d) 2 and 4/3

Chapter 3_Solved Examples and Practice Exercise.indd 247 3/23/2016 10:44:02 AM


248     CHAPTER 3:  SIGNALS AND SYSTEMS

3. The Nyquist sampling interval for the signal 9. The unit impulse response of LTI system is u(t) for
sinc (700t) + sinc (500t) is t > 0 the response of the system for an excitation
e−atu(t), a > 0, will be
(a)  1 s (b)  p s
1 − e−at
350 350 (a) ae−at (b) 
a
1 1
(c) a(1 − e−at ) (d) 1 − e−at
(c)  s (d)  s
700 700
4. A discrete-time signal x(n) has Fourier transform 10. The Dirac delta function δ (t ) is defines as
x(ejw). Match the signal given in List-I with the
(a)  d (t) = 
Fourier transform in List-II and select the correct 1 t=0
answer using the code given below the lists: 0 otherwise
∞ t=0
(b)  d (t) = 
List-I List-II
(a) x(−n) 1. X*(e−jw) 0 otherwise
(b) nx(n) 2. X(e−jw)
e−jw X(e jw) (c)  d (t) = 

and ∫ d (t)dt = 1
(c) x*(n) 3. 1 t = 0
jd/dw X(e jw) −∞
(d) x(n−1) 4. 0 otherwise
∞ t = 0
(d)  d (t) = 
Codes: ∞
A B C D and ∫ d (t)dt = 1
−∞
(a) 1 3 2 4 0 otherwise
(b) 2 4 1 3 11. The signal x(n) = 3nu(−n − 1) is
(c) 1 4 2 3
(d) 2 3 1 4 (a) stable
(b) unstable
5. Which of the following function is a periodic one?
(c) data insufficient
(a) sin (10pt) + sin (20pt) (d) None of the above
(b) sin (10t) + sin (20pt)
(c) sin (10pt) + sin (20t) 12. Let P define linearity, Q → time invariance,
(d) sin (10t) + sin (25pt) R → causality and S → stability.

6. The convolution of f(t) with itself is given to be Consider a discrete time system defined by input
t output relationship
∫ F (t )dt . Then what is f(t)?
0 ìx(n) n > 0
ï
(a) The unit ramp function y(n) = í0 n = 0
ïx(n + 1) n < 0
(b) Equal to 1 î
(c) The unit step function
(d) The unit impulse function where x(n) and y(n) are input and output of the
7. A system is defined by its impulse response system. The system is
h(n) = 2nu(n−2). The system is (a) P, Q, R, S
(b) P, Q, S but not R
(a) stable and causal (c) P, S but not Q, R
(b) causal but not stable
(d) P but not Q, R, S
(c) stable but not causal
(d) unstable and non-causal 13. How many inverse Laplace transforms exist for
1
8. The inverse Fourier transform of X(w ) = 5
(4 + jw )2 F (s) =
is (s + 1) (s − 2)(s + 3)3
2
−8t
(a) e u(t)
(b) e−4t u(t) (a) 3
(c) te−4t u(t) (b) 4
(d) te−8t u(t) (c) 5
(d) 6

Chapter 3_Solved Examples and Practice Exercise.indd 248 3/23/2016 10:44:21 AM


ANSWERS TO PRACTICE EXERCISES     249

ANSWERS TO PRACTICE EXERCISES

SET 1 (One Mark Questions) 6. (b) Output Y(jw) is the product of input transfer
function multiplied with the system transfer func-
1. (a) For any periodic function, Fourier series is tion, that is Y(jw) = X(jw) ⋅ H(jw)
given by odd and even functions as
d
T /2
7. (a) d (t) = u(t)
dt

1
xo (f ) = x(t)cos n wt dt  (i)
T So for unit impulse response we can differentiate
−T /2
the unit step response to get

(1 − e−a t ) = e−a t
T /2 d

1
x(t)sin(n w t) dt
y(t) =
xe (f ) = (ii) dt
T −T /2
8. (c) For a band pass signal, the minimum sampling
If x(n) is even then Eq. (i) is even and Eq. (ii) is rate is twice the bandwidth which is 0.5 kHz here.
odd so we get only cosine term and if signal is odd 9. (d) The system, is non-linear because x(t) = 0 does
we get only sine term similarity, not lead to y(t) = 0, which is a violation of the
2. (d) The given Fourier transform is principle of homogeneity.
10. (b) Convolution of a periodic signal (time period T)
2
x(t) ←→ sin(pw )
FT
with itself will give the same period T.
w
11. (d) Energy is given by
For ej5t x(t), by shifting in frequency property, we T1 /2


have
E= x(t)2 dt = A2 T1
x(t) ←→ x(f ) −T1 /2
FT

It is energy signal, its normalized power is zero


at
x(t)e ←→ x(w − a )
FT
12. (c) Here the highest frequency present in the signal
is wmax= 200 p or fmax = 100 Hz.
2
e j5t x(t) ←
→ sin p (w − 5)
(w − 5)
Therefore, the Nyquist frequency fs = 2fmax= 200 Hz.
13. (a) The final value is given by
3. (a) For any real signal
lim x(t) = lim sX(s)
t →∞ s→ 0
x(t) ←
→ x(f )
2 1 
= lim s  +
s + 2 
= 2
the magnitude graph always follows even symme- s→ 0  s
try and phase plot follows odd symmetry.
14. (a)
4. (d) By scaling property
15. (c) With the help of time shifting property, we have
x(t) ←
→ x(w )
2p
−j
(−1)k
5k
10
1 w 
X(k) = 1 + 2e = 1+2
x(at) ←
→ x 
a a  16. (a)
5. (a) For any discrete signal to be periodic w 2p we 17. (d) Discrete time system is stable if poles are
must be rational. within the unit circle in case of causal system. If
For the given signal x(n) = 5 cos (0.2 pn) the system is anti-causal then for stability pole
should lie outside the unit circle.
w 0.2p 1
= =
2p 2p
18. (c) Since exponential signal contains only single
10
frequency term w0, its Fourier transform is an
Therefore, time period is 10 seconds. impulse at w = w0.

Chapter 3_Solved Examples and Practice Exercise.indd 249 3/23/2016 10:44:41 AM


250     CHAPTER 3:  SIGNALS AND SYSTEMS

SET 2 (Two Marks Questions) The Nyquist rate, fs = 2 × fmax = 2 × 350 = 700 Hz

1. (a) Given that Nyquist sampling interval = 1 = 1 = 1 s


fs 2fmax 700
x(t) ←→ X( jw )
FT

4. (b) For A: By scaling property


By applying first shifting, we have
x(−n) ←→ X(e−jw )
FT
x(t) ←→ X(w )
FT

For B: Multiplication in time results differentiation


→ e−at
x(t − a) ← in frequency

X(e jw )
d
By using this property nx(n) ←
→j
dw
→ X( jw )e−3jw
x(t − 3) ←
For C: By conjugate property
Now by scaling property → X * (e−jw )
x * (n) ←
x ( t ) ←
→ X(w) For D: By shifting property

x(at) ←

1  w
X  → e−jw X(e jw )
x(n − 1) ←
a  a
5. (a) For a function be periodic.
For x(5t - 3),
−j 3w x(t) = sin at + sin bt
1  jw 
x(5t − 3) ←
→ X  e 5
5  5  T1
And for periodicity must be rational number.
T2
2. (b) For the given probability density function
For option (a), that is, sin(10pt) + sin(20pt)
P (x) w 10p
1/4 T1 = = =5
2p 2p
w 20p
−1 3 T2 =
2p
=
2p
= 10

3 3
T1 5 1
= =  (Rational number)
Mean E(x) = ∫ xf(x) dt = ∫ x dx
1 T2 10 2
−1 −1
4
Time period = 2 × T1 = 10 seconds
1 x 2 3
3 Therefore, first signal is periodic with time

1
=   = [32 − 1] = 1 period 10 s.
4 −1  2  8
−1 Consider the signal in option (b), that is
3 3
sin(10t) + sin(20pt)

∫ 4 −∫1
w
1
E(x2 ) = x2f(x)dx = x2dx 10 5
T1 = = =
−1 10 2p p

1  x3 
3
1  28  7 w 20p
  =
=   T2 = = = 10
4 3 
=
4 3 3 2p 2p
−1
T1 5 1
= =
p ´ 10 2p
For variance  (Irrational number)
T2
V = E[x] = E[x]2
7 4 Hence, the signal is non-periodic.
−1 =
3 3 6. (c) Given that the convolution of F(t) is given by
t
x = sin c (700t) + sin c (500t)
∫ f(t )dt
3. (c)
sin c (700p t) sin c (500p t)
= + 0
700p t 500p t
We know that convolution with u(t) is similar as
The maximum frequency component is fmax = 350 Hz integration. Therefore

Chapter 3_Solved Examples and Practice Exercise.indd 250 3/23/2016 10:45:14 AM


ANSWERS TO PRACTICE EXERCISES     251

u(t) * u(t) ←


→ y(w ) 1
H(s) = , X(s) =
1
s     s+a
d d
u(t) * u(t) ←
→ [y(w )] 1 1
dt dt Y (s) = H(s) ⋅ X(s) = .
s (s + a)
t

∫ d (t) * u(t)dt ←


d
→ [y(w )] 1 1 
0
dt y(t) = LT−1  . 
 s (s + a) 
t
y(w ) = ∫ u(t )dt 1 1 1  1
= LT−1 − = (1 − e−at )
0 a  s s + a  a

Hence, it is unit step function.
10. (d) The Dirac delta function has infinite (unde-
7. (b) The given impulse response h(n) = 2n u(n - 2), fined) value at t = 0 and 0 for t ≠ 0. The area
can be expressed as h(n) = 2n - 2 22 u(n - 2) under this function is unity.
d(t)
z −2
Z-transform = 4
1 − 2z−1
ROC |z*| > 2
t
As the system ROC is right sided it is causal and as Hence,

∞ t = 0
ROC lies outside unit circle it is unstable.
δ(t) =  and ∫ δ(t)dt = 1
8. (c) We have
0 otherwise −∞

X(w ) =
1
11. (a) Taking Z-transform of the given signal
(4 + jw )2
∞ −1
=
1
×
1 X(z) = ∑ 3n u(−n − 1)z−n = ∑ 3n z−n
(4 + jw ) (4 + jw ) n =−∞ n =−∞
= X1(w ) × X2 (w ) Put n = −p
∞ ∞
∑ (1/3)p z p = ∑ (1/3z)p
where
X(z) =
1 1
X1(w ) = , X2 (w ) = p =1 p =1
(4 + jw ) (4 + jw )
1
z
1  1 
2 3
1
Therefore, = z +  z  +  z  + = 3
3  3 
x1(t) = e−4 t u(t), x2 (t) = e−4 t u(t)
1
3 1− z
3
By time convolution property 1
∞ ROC z < 1 or |z| < 3
∫ x1(t )x2 (t − t )dt
x(t) = x1(t) * x2 (t) = 3
−∞ Since ROC contains a unit circle, so the signal is
t t stable.
= ∫ e−4t e−4(t−t )dt = ∫ e−4 t e−4t e4t dt 12. (c) The given system can be defined as
0 0
y(n) = u(n − 1)x(n) + u(−n − 1)x(n + 1)
t t
= e−4 t ∫ e−4t + 4t dt = e−4t ∫ dt So it is linear, stable, time invariant and not causal.
0 0
13. (b) Number of inverse Laplace transform is equal
−4 t −4 t
=e [t − 0] = te to number of valid region of convergence
Re(s) > 2
= te−4 t u(t) for t ≥ 0
Re(s) < −3
9. (b) Given that −1 < Re(s) < 2
h(t) = u(t) and x(t) − e−at u(t), a > 0 −3 < Re(s) < −1

Chapter 3_Solved Examples and Practice Exercise.indd 251 3/23/2016 10:45:52 AM


252        CHAPTER 3:  SIGNALS AND SYSTEMS

SOLVED GATE PREVIOUS YEARS’ QUESTIONS

1. If u(t) is the unit step and d(t) is the unit impulse 2. Which of the following is true?
1 (a) A finite signal is always bounded.
function, the inverse z-transform of F (z) =
for k > 0 is z +1 (b) Abounded signal always possesses finite
energy.
(a) (−1)k d (k) (b) d (k) − (−1)k (c) A bounded signal is always zero outside the

(c) (−1)k u(k) (d) u(k) − (−1)k interval, [-t0, t0] for some t0.

(d) A bounded signal is always finite.
(GATE 2005: 2 Marks)
(GATE 2006: 1 Mark)
Solution:  Given that
Unit step function u(t) Solution:  A bounded signal always possesses
finite energy. So
u(t)
t0

∫ g(t) dt < ∞
2
1 E=
−t0
t
0 Ans. (b)
3. x (t) is a real-valued function of a real variable with
Impulse function d(t) period T. Its trigonometric Fourier series expan-
d(t) sion contains no terms of frequency, that is w = 2p
(2k)/T; k = l, 2,.... Also, no sine terms are present.
1 Then x (t) satisfies the equation
t (a) x(t) = -x(t - T)
0 (b) x(t) = x(T - t) = -x(-t)
(c) x(t) = x(T - t) = -x(t - T/2)
(d) x(t) = x(t - T)= x(t - T/2)
Given that
1
F (z) = (GATE 2006: 1 Mark)
z +1
This can be rewritten as, Solution:  Trigonometric Fourier series is given by,
1 z ∞
= 1−
F (z) =
z +1 z +1 x(t) = A0 + ∑ (an cos nw 0 t + bn sin nw 0 t)
n =1
é z z +1−z 1 ù
êë∵ 1 − z + 1 = z + 1 = z + 1 úû As there are no sine terms, so
Therefore,
ò0 x(t) sin nw 0 t × dt
2 T0
z bn =
F (z) = 1 − T0
z +1
é ù
2 ê 0 ú
z
= 1−
T T
−1
z(z + 1) = ê ò x(t ) sin n w 0t dt + ò x(t) sin n w 0 t dt ú
T0 ê 0 ú
êë úû
T0
1
= 1 − −1 2
(z + 1)
Z -transform (1) = d (k) where t = T − t and dt = −dt
 1 
Z -transform  = (−1)k é T0 2 ù
 1 + z−1  ê ò x(T − t) sin n w 0 (T − t)(−dt)ú
Therefore, 2 êê T0 ú
ú
bn =
T0 ê T ú
f (k) = d (k) − (−1)k ê+ ò x(t) sin n w 0 t dt ú
ê T 2 ú
Ans. (b) ë 0 û

Chapter 3 Solved Question Paper .indd 252 3/22/2016 2:31:19 PM


SOLVED GATE PREVIOUS YEARS’ QUESTIONS        253

é T0 ù
ê ò x(T − t) sin (2np − nw 0 )dt ú {
When a = −1, h (n ) = 1, − 1, − 1

}
2 êê T0 2 ú
ú Ans. (a)
=
T0 ê T ú
ê+ ò x(t) sin n w 0 t dt
5. The discrete-time signal
ú 2p 
ê T 2 ú ∴ w 0 =
ë 0 û  T  ∞ 3n 2 n
x[n ] ↔ X(z) = ∑ n = 0 z
2+n
é T0 ù
ê− ò x(T − t) sin (nw 0 t)dt ú where ↔ denotes a transform-pair relationship, is
2 êê T0 2 ú
ú
orthogonal to the signal
=
T0 ê T ú ∞  2
n
ê+ ò x(t) sin n w 0 t dt ú (a) y1[n ] ↔ Y1(z) = ∑ n = 0   z−n
 3
ê T 2 ú
ë 0 û
This implies that bn = 0, when x(t) = x(T − t)

(
(b) y2 [n ] ↔ Y2 (z) = ∑ n = 0 5n − n z−(2n +1) )

For half-wave symmetries (c) y3 [n ] ↔ Y3 (z) = ∑ n =−∞ 2− n z−n
 T
x(t) = −x  t −  (d) y4 [n ] ↔ Y 4 (z) = 2z−4 + 3z−2 + 1
 2
Ans. (c) (GATE 2006: 2 Marks)

4. x[n] = 0; n - 1, n>0, x[- 1] = - 1, x[0] = 2 is the


input and y[n] = 0; n<- 1, n>2, y[-1]  = -1 = Solution: 
y[1], y[0] = 3, y[2] = -2 is the output of a discrete-
x[n ] =  0, 0, 0, −1, 2, 0, 0, 0
time LTI system. The system impulse response h[n]  ↑ 

y[n ] =  −1, 3, −1, −2


will be
(a) h[n] = 0; n<0, n>2, h[0] = l, h[l] = h[2] = -l  ↑ 
(b) h[n] = 0; n<-l, n>l, h[-l] = l, h[0] = h[l] = 2 Left hand limit of y[n] is -1,
(c) h[n] = 0; n<0, n>3, h[0] = -l, h[l] = 2, h[2] = l By using y[n] = x[n] + h[n], we get,
(d) h[n] = 0; n<-2, n>l, h[-2] = h[1] = h[-l] =
  -h[0] = 3 −1 = −1 + h[n ]

(GATE 2006: 2 Marks) So, h[n] is define for n ≥ 0


Right hand limit of y[n] is 2,
Solution:  Given that Here we get,

{
h[n] = 0 n > 2, n < 0
x(n) = −1, 2 ; for − 1 ≤ n ≤ 0 ∞

y(n) = {−1, 3, − 1, − 2 ; for − 1 ≤ n ≤ 2
y(n) = ∑ x[k ]h[n − k ]
k =−∞

If, impulse response is h(n), then y(n) = h(n) * x(n) y[0 ] = ∑ x[k ]h[−k ]
k =−∞
Length of h(n) = 0 to 2
y[0 ] = x[−1]h[1] + x[0 ]h[0 ] = 3
{ }
Let h(n) = a, b, c

−h[1] + 2h[0 ] = 3
This is possible, when h[-1] = -1, h[0] = 1.
By convolution
Therefore, discrete time signal is orthogonal to

 2
a b c n

−1 −a −b −c y (n ) ↔ Y1 (z ) = ∑  3  z−n
n =0
2a 2b 2c
Ans. (a)
2
6. A continuous-time system is described by
y (n ) = {−a, (2a − b ) , (2b − c ) , 2c} − x(t)
y(t) = e ,
a = 1 ; 2a − b = 3 b = −1
where y(t) is the output and x(t) is the input. y(t)
2a − c = −1 Þ c = −1 is bounded

Chapter 3 Solved Question Paper .indd 253 3/22/2016 2:31:48 PM


254        CHAPTER 3:  SIGNALS AND SYSTEMS

(a) only when x(t) is bounded.


(b) only when x(t) is non-negative. (b)
(c) only for t ≥ 0 if x(t) is bounded for t ≥ 0 w
(d) even when x(t) is not bounded.
(c)
(GATE 2006: 2 Marks)
− x(t)
Solution:  Given that y(t) = e w
Therefore, for x(t) → ∞, y(t) → 0 and for x(t) = 0,
y(t) = 1. Thus, y(t) will always be bounded irre- (d)
spective of the nature of input x(t).
Ans. (d) w
7. y[n] denotes output and x [n] denotes input of a dis- (GATE 2007: 1 Mark)
crete-time system given by the difference equation
y[n] - 0.8y[n - 1] = x[n] + 1.25x[n + 1].
Solution:  Given that frequency of the signal
w = 1 kHz. The spectrum of the sampled signal
Its right-sided impulse response is s(jw) will contain signals that replicate the given
(a) causal. (b) unbounded. signal for U(jw) at frequencies ± nfs where n = 0,
(c) periodic. (d) non-negative. 1, 2,¼. The sampling frequency
(GATE 2006: 2 Marks) 1 1
fs = = = 1 kHz
1.25 Z Ts 1 × 10−3
Solution:  H(z) = 1 +
1 − 0.8Z −1 The signal sampled at intervals of 1 ms is given by
So, the system is casual. S(f)
Ans. (a)
8. Let a signal a1sin(w1t + f1) be applied to a stable
linear time-invariant system. Let the corresponding
steady state output be represented as a2F (w2t +
f2). Then which of the following statements is true?
−3 −2 −1 0 1 2 3 f(kHz)
(a) F is not necessarily a “sine” or “cosine” func-
tion but must be periodic with w1 = w2. The frequency spectrum of the sampled signal is
(b) F must be a “sine” or “cosine” function with given by
a1 = a2 S(f)
(c) F must be a “sine” function with w1 = w2 and
f1 = f2.
(d) F must be a “sine” or “cosine” function with
w1 = w2
(GATE 2007: 1 Mark)
Ans. (d) f(kHz)
9. The frequency spectrum of a signal is shown in the
Ans. (d)
following figure. If this signal is ideally sampled at
10. A signal x(t) is given by
intervals of 1 ms, then frequency spectrum of the
sampled signal will be 1, − T 4 < t ≤ 3T 4

U(jw) x(t) = −1, 3T 4 < t ≤ 7 T 4
−x(t + T )
Which among the following gives the fundamental
Fourier term of x(t)?
w 4  pt p  4  pt p
(a) cos  −  (b) cos  + 
T 4   2T 4
1 kHz
p p
U ∗(jw)
 pt p   pt p
(a)
4 4
(c) sin  −  (d) sin  + 
p T 4  p  2T 4

w (GATE 2007: 2 Marks)

Chapter 3 Solved Question Paper .indd 254 3/22/2016 2:32:07 PM


SOLVED GATE PREVIOUS YEARS’ QUESTIONS        255

Solution:  Function x(t) is periodic with a time time invariant system has the impulse response
period 2T and also given that x(t) = −x(t + T ), so h[n] defined by these two signals as
half-wave symmetry
h[n] = x[n - 1]*y[n]
x(t) where * denotes discrete time convolution. Then
+1 the output of the system for the input d [n - 1]
(a) has Z-transform z-1 X(z) Y(z)
3T/4 7T/4
t (b) equals d [n - 2] - 3d [n - 3] + 2d[n - 4] - 6d
−T/4 0 [n - 5]
(c) has Z-transform 1 - 3z -1 + 2z -2 - 6z-3
−1 (d) does not satisfy any of the above three.
(GATE 2007: 2 Marks)
Hence, the fundamental fourier term is given by
4  pt p 
sin  − 
Solution:  Impulse response
p T 4
h(n) = x(n − 1) * y(n)
Ans. (c)
Taking Z-transform, we get
11. If u(t), r(t) denote the unit step and unit ramp
H(z) = z−1x(z)y(z)
functions respectively and u(t)*r(t) their convolu-
tion, then the function u(t + l)*r(t - 2) is given
Given that x(z) = 1 − 3z−1 , y(z) = 1 + 2z −2, so
by

1 1
(a) (t − 1)(t − 2) (b) (t − 1)(t − 2) H(z) = z−1(1 − 3z−1 )(1 + 2z−2 )
2 2
1 The output of the system, for u(n) = d (n − 1) is,
(c) (t − 1)2 u(t − 1) (d) None of these
2
u (n ) ←
→ u ( z ) = z −1
Z
(GATE 2007: 2 Marks)
y(z) = H(z)u(z)
Solution:  Convolution sum is given by,
y(z) = z−1(1 − 3z−1 )(1 + 2z−2 )z−1
h (t ) = u (t + 1) * r (t − 2)
= z−2 (1 − 3z−1 + 2z−2 − 6z−3 )
Taking Laplace transform y(z) = z−2 − 3z−3 + 2z−4 − 6z−5
H(s) = L[n(t)] = L[u(t + 1)] * L[r(t − 2)]
Taking inverse Z-transform;
1
LT[u(t)] = u(s) =
s y(n) = d (n − 2) − 3d (n − 3) + 2d (n − 4) − 6d (n − 5)

 1 Ans. (c)
LT[u(t + 1)] = es  2  (by time shifting property) -at
s  13. A signal e sin(wt) is the input to a linear-time
1 invariant system. Given K and f are constants,
LT[r(t)] = R(s) = the output of the system will be of the form Ke-bt
s2
sin(nt + f), where
 1
LT[r(t − 2)] = e−2s  2  (by time shifting property) (a) b need not be equal to a but n equal to w
s  (b) n need not be equal to w but b equal to a
 1  1  1 (c) b equal to a and n equal to w
H (s) = es   e−2s  2  = e−s  3  (d) b need not be equal to a and n need not to be
 s s  s 
equal to w
h(t) = L−1[H(s)] =
1
(t − 1)2 u (tt − 1)
2 (GATE 2008: 1 Mark)
Ans. (c)
Solution:  The signal input is
12. X(z); = 1 - 3z-, Y(z) = l + 2z-2 are Z-transforms
of two signals x[n] and y[n], respectively. A linear g(t) = e−at sin(wt)

Chapter 3 Solved Question Paper .indd 255 3/22/2016 2:32:29 PM


256        CHAPTER 3:  SIGNALS AND SYSTEMS

and the output is given by As per option (a)

y(t) = Ke−bt sin(vt + f ) z1 ( n ) = x ( n − 3 )

For linear time invariant systems, the frequency of z2 ( n ) = z1 ( 4n ) = x ( 4n − 3 )


the output must be equal to the input frequency, y ( n ) = z2 (−n ) = x (−4n − 3 ) ≠ x ( 3 − 4n )
so v is equal to w. The degree of attenuation will b
As per option (b)
will depend on the system parameters, so need not
be equal to a. z1 ( n ) = x ( n + 3 )
Ans. (a) z2 ( n ) = z1 ( 4n ) = x ( 4n + 3 )
14. The impulse response of a causal linear time-invari- y ( n ) = z2 (−n ) = x (−4n + 3 )
ant system is given as h(t).
Now consider the following two statements: As per option (c)
Statement I: Principle of superposition holds n1(n) = x(4n)
Statement II: h(t) = 0 for t < 0
n2 (n) = n1 (−n) = x(−4n)
Which one of the following statements is correct?
y(n) = n2 (n + 3) = x[−4(x + 3)] ≠ x(3 − 4n)
(a) Statement (I) is correct and Statement (II) is
wrong. as per option (d)
(b) Statement (II) is correct and Statement (I) is n1(n) = x(4n)
n2 (n) = n1 (−n) = x(−4n)
wrong.
(c) Both Statement (I) and Statement (II) are wrong.
(d) Both Statement (I) and Statement (II) are y(n) = n2 (n − 3) = x[−4(x − 3)] ≠ x(3 − 4n)
correct. Hence, correct option is (b).
(GATE 2008: 1 Mark) Ans. (b)
16. A system with input x(t) and output y(t) is defined
Solution:  For a linear time invariant system, by the input-output relation:
principle of superposition holds good. Therefore,
−2 t
for causal system, h(t) = 0 for t < 0
Ans. (d)
y(t) = ∫ x(t )dt
−∞
15. Given a sequence x[n], to generate the sequence
y[n] = x[3 - 4n]. Which one of the following proce- The system will be
dures would be correct? (a) causal, time-invariant and unstable
(b) causal, time-invariant and stable
(a) First delay x[n] by 3 sample to generate z1[n], (c) non-causal, time-invariant and unstable
then pick every fourth sample of z1[n] to gener- (d) non-causal, time-variant and unstable
ate z2[n], and then finally time reverse z2 [n] to
obtain y[n]. (GATE 2008: 2 Marks)
(b) First advance x[n] by 3 samples to generate
z[n], then pick every fourth sample of z1[n] to Solution:  The input-output relation is given by
generate z2 [n], and then finally time reverse z2 2t
[n] to obtain y[n]. y(t) = ∫ x(t) dz
(c) First pick every fourth sample of x[n] to gen- −∞
erate ν11[n], time-reverse ν1 [n] to obtain ν2
Causality: y (t ) depends on x(−2t), therefore system
[n] and finally advance ν2 [n] by 3 samples to
is non-causal.
obtain y[n].
Non-linear time-variance:
(d) First pick every fourth sample of x[n] to gener-
ate ν1 [n], time-reverse ν1 [n] to obtain ν2 [n], 2t
and finally delay ν2 [n] by 3 samples to obtain y(t) = ∫ x (t − t 0 ) dt ≠ y (t − t 0 )
y[n]. ∞
Therefore, the system is time variant.
(GATE 2008: 2 Marks)
Stability: y(t) is unbounded for bounded input,
that is
Solution:  Let us consider the procedures given in
the options x(t ) = e−t (bounded)

Chapter 3 Solved Question Paper .indd 256 3/22/2016 2:32:44 PM


SOLVED GATE PREVIOUS YEARS’ QUESTIONS        257

−2 t −2 t
 e−t 
T T
−t
(a) (b)
y(t) = ∫ e dt =  
 −1  ∞
(unbounded) 8 4
−∞ T
Ans. (d) (c) (d) 2T
2
17. A signal x(t) = sinc (at) where a is a real constant.
 sin(p x)
(GATE 2008: 2 Marks)
 sin c(x) = p x  is the input to a linear-time
Solution:  Given
invariant system whose impulse response h(t) =
sinc (bt) where b is a real constant. If min (a, b) y (t ) = x ( t − t0 ) + x ( t + t0 ) .
denotes the minimum of a and b, and similarly
max (a, b) denotes the maximum of a and b, and Let ak be Fourier series coefficient of x(t)
K is a constant, which one of the following state- Then Fourier coefficients of y(t) are
ments is true about the output of the system?
bk = e−jkwt0 ak + e jkwt0 ak
(a) It will be of the form Ksin(gt) where g = min
(a, b) bk = 2ak cos kwt0
(b) It will be of the form Ksin(gt) where g = max
(a, b) For all odd values of k, bk = 0, so for odd k,
(c) It will be of the form Ksinc(at) p
(d) It cannot be a since type of signal kwt0 =
2
2p p

(GATE 2008: 2 Marks)
t =
T 0 2
Solution:  The output is given by
T
y (t) = x (t) ∗ h (t) For k = 1, t0 =
4
 Ans. (b)
Y ( jw ) = X ( jw ) H ( jw )
19. H(z) is a transfer function of a real system. When a
Given that signal x[n] = (t + j)n is the input to such a system,
the output is zero. Further, the region of conver-
x(t) = sin c (a t) (i)
 
gence (ROC) of  1 − z−1  H(z) is the entire
 1
h(t) = sin c ( b t) (ii)  2 

Taking Fourier transform of x (t ) and h (t ) in
z-plane (except z = 0). It can then be inferred that
H(z) can have a minimum of
Eqs. (i) and (ii), we get
(a) one pole and one zero.
p æw ö
X( jw ) = F [x(t)] = rect ç ÷ ; for − a < w < −a
(b) one pole and two zeros.
a è 2a ø (c) two poles and one zero.
p æw ö (d) two poles and two zeros.
H( jw ) = F [h(t)] = rect ç ÷ ; for − b < w < −b
b è 2b ø (GATE 2008: 2 Marks)
p2 æw ö æw ö
Y ( jw ) = rect ç ÷ rect ç ÷ Solution:  Given that x[n ] = (t + j)n and
ab è 2a ø è 2b ø
æw ö  
Y ( jw ) = K rect ç ÷ ROC =  1 − z−1  H(z)
1
è 2g ø  2 
 2z − 1
where g is min(a, b). So =
 2z 
H(z)

y ( t ) = K rect (g t)
So H(z) is of the form
 zn 
Ans. (a)
18. Let x(t) be a periodic signal with time period T.  (2z − 1)
Let y(t) = x(t - t0) + x(t + t0) for some t0.  

The Fourier series coefficients of y(t) are denoted So the correct answer is one pole and two zeros.
by bk. If bk = 0 for all odd k, then t0 can be equal to Ans. (b)

Chapter 3 Solved Question Paper .indd 257 3/22/2016 2:33:06 PM


258        CHAPTER 3:  SIGNALS AND SYSTEMS

z
with |z|>a, then residue of X[z] 1 0
= ∫ (1) e jwt dt + ∫ (1) e − jwt dt
20. If X(z) =
(z − a)2
zn-1 at z = a for n ≥ 0 will be 0 −1

e −jwt 1
 e−jwt 
0
(a) an - 1    (b) an   (c) nan    (d) nan - 1 =  + 

(GATE 2008: 2 Marks)  −jw  0  −jw  −1

1 − e−jw  +
1  1  jw
e − 1
jw  jw 
=
Solution:  Given that
z
; with z > a e− jw 2
e jw
jw 2
− e− jw 2  + e jw − e− jw 2 
x(z) = 2 e 2
(z − a)2 =
jw  jw 
Residue of x (z ) z n −1 at z = a,
=
(e jw 2
− e− jw 2
)(e − jw 2
+ e jw 2
)
jw
(z − a)2 × (z)z n−1
d
dz w  w 
z =a 2 sin   2 cos  
 2  2
z n−1
d z =
= (z − a)2 × w
dz (z − a)2
z =a w  w 
= 2 cos   sin c  
 2  2p 
= nz n−1 |z =a
d n
= z
dz z =a
Ans. (c)
Ans. (d) 22. A linear time invariant system with an impulse
 1
21. Let x(t) = rect  t −  (where rect (x) = 1 for response h(t) produces output y(t) when input x(t) is
 2
1 1 applied. When the input x(t - t) is applied to a system
− ≤ x ≤ and zero otherwise). with impulse response h(t - t), then output will be
2 2
sin(px) (a) y(t) (b) y(2(t - t))
Then if sinc (x) =
px
, then Fourier transform (c) y(t - t) (d) y(t - 2t)
of x(t) + x(-t) will be given by (GATE 2009: 1 Mark)
w  w 
(a) sinc   (b) 2 sinc   Solution:  Let
 2p   2p 
LT[x(t)] = X(s);
w  w  w  w  LT[y(t)] = Y (s);
(c) 2 sinc   cos   (d) sinc   sin  
 2p   2  2p   2 LT[h(t)] = H(s)

The output is
(GATE 2008: 2 Marks)
Y (s) = X(s)H(s)
Solution:  Given that
The input is
 1
x (t ) = rect  t −  LT[x(t − t )] = e−st X(s)
 2
LT[h(t − t )] = e−st H(s)
1, −0.5 ≤ (t − 0.5) ≤ 0.5 or 0 ≤ t ≤ 1
⇒ x (t ) =  Y ′(s) = e−st X(s) ⋅ e−st H(s)
0, otherwise
= e−2st X(s)H(s) = e−2st Y (s)
Also
y ′ (t ) = y (t − 2t )
 1
x (−t ) = rect  −t − 
 2
Ans. (d)

1, −0.5 ≤ (−t − 0.5) ≤ 0.5 (or) − 1 ≤ t ≤ 0


23. A cascade of three linear-time invariant systems is
x (−t ) =  causal and unstable. From this, we conclude that
0, otherwise
(a) each system in the cascade is individually causal
∞ ∞
F [x (t ) + x (−t )] = ∫ x (t ) e
−jwt
∫ x (−t) e
−jwt and unstable.
dt + dt (b) at least one system is unstable and at least one
−∞ −∞
system is causal.

Chapter 3 Solved Question Paper .indd 258 3/22/2016 2:33:27 PM


SOLVED GATE PREVIOUS YEARS’ QUESTIONS        259

x ( t ) = (a−2 )e−j2w 0 t + (a−1 )e−jw 0 t


(c) at least one system is causal and all systems
are unstable.
(d) the majority are unstable and the majority are + a0 + a1e jw 0 t + a2e j2w 0 t
causal.
On substituting the constant values given, we get
(GATE 2009: 2 Marks)
x(t) = (2 − j)e2 jw 0 t + (0.5 + 0.2 j)e−jw 0 t
Solution:  For a cascaded system
+ 2 j + (0.5 − 0.2)e jw 0 t + (2 + j)e j2w 0 t
= 2[e−j2w 0 t + e j2w 0 t ] + j[e j2w 0 t − e−j2w 0 t ]
Input H1(z) H2(z) H3(z) H(z)

For non-causal system, let + 0.5[e jw 0 t + e−jw 0 t ] − 0.2 j[e jw 0 t − e−jw 0 t ] + 2 j


= 2(2 cos 2w 0 t) + j(2 j sin 2w 0 t)
H1(z) = z 2 + z1 + 1
+ 0.5(2 cos 2w 0 t) − 0.2 j(2 j sin w 0 t) + 2 j
H2 (z) = z 3 + z 2 + 1
= 4 cos 2w 0 t − 2 sin 2w 0 t + cos w 0 t
+ 0.4 sin w 0 t + 2 j
Therefore,
H(z) = H1(z)H2 (z)H3 (z) =2
= (z 2 + z + 1)(z 3 + z 2 + 1)H3 (z)  (i) Thus imaginary part is constant.
As, H(z) is causal and H3(z) is causal, so Ans. (c)

H3 (z) = z−6 + z−4 + 1


25. The Z-transform of a signal x[n] is given by
4z−3 + 3z−1 + 2 − 6z 2 + 2z 3
Substituting in Eq. (1), we have H(z) is causal It is applied to a system, with a transfer function
H(z) = (z 2 + z + 1) (z 3 + z 2 + 1) (z−6 + z−4 + 1) H(z) = 3z−1 − 2
Let the output be y(n). Which of the following is
Hence, H(z)will be unstable, when any one of the
true?
system is unstable.
Ans. (b) (a) y(n) is non-causal with finite support
24. The Fourier series coefficients of a periodic signal (b) y(n) is causal with infinite support
x(t), expressed as (c) y(n) = 0; |n| > 3

∞ (d) Re[Y (z)]z−ejq = −Re[Y (z)]z = ejq ; Im [Y (z)]z = ejq = Im[Y (z)]z = e jq ;
x(t) = ∑ k =−∞ ak e j2pkt / T
Re[Y (z)]z−ejq = −Re[Y (z)]z = ejq ; Im [Y (z)]z = e jq = Im[Y (z)]z = e jq ; p £ q < p
are given by
(GATE 2009: 2 Marks)
a-2 = -j1; a-1 = 0.5 + j 0.2; a0 = j2;
a1 = 0.5 - j 0.2;
Solution:  Given that
a2 = 2+ j1; and ak = 0; for |k| > 2.
Which of the following is true? x[n ] = 4z−3 + 3z−1 + 2 − 6z 2 + 2z 3
(a) x(t) has finite energy because only finitely many
coefficients are non-zero. H(z) = 3z−1 − 2
(b) x(t) has zero average value because it is periodic. Y (z) = H(z)X(z)
= (3z−1 − 2)(4z−3 + 3z−1 + 2 − 6z 2 + 2z 3 )
(c) The imaginary part of x(t) is constant.
(d) The real part of x(t) is even.
= 12z−4 + 9z−2 + 6z−1 − 18z + 6z 2 − 8z−3
(GATE 2009: 2 Marks)
−6z−1 − 4 + 12z 2 − 4z 3
Solution:  Given that: = 12z−4 − 8z−3 + 9z−2 − 4 − 18z + 18z 2 − 4z 3
∞ j 2pkt
x (t ) = ∑ ak e T
The output is given by
k =−∞
y ( n ) = 12d ( n − 4 ) − 8d ( n − 3 ) + 9d ( n − 2 ) − 4d ( n )
Fundamental frequency of signal w 0 = 2p T .
Therefore, −18d ( n + 1) + 18d ( n + 2 ) − 4d ( n + 3 )

Chapter 3 Solved Question Paper .indd 259 3/22/2016 2:33:45 PM


260        CHAPTER 3:  SIGNALS AND SYSTEMS

Therefore, y (n ) ≠ 0; n < 0 , so y(n) is non-causal Solution:  Fourier series expression is given as,
with finite support. ∞
Ans. (a) x(t) = A0 + ∑ an cos nw 0 t + bn sin nw 0 t
n =1
 p
26. The period of the signal x(t) = 8 sin  0.8p t+  is For odd function, x(t) = −x(t). Also, A0 = 0 and an = 0
 4
A0 = 0 and an = 0
(a) 0.4 p s (b) 0.8 p s
(c) 1.25 s (d) 2.5 s T
bn = ∫ x (t ) sin nw 0 t ⋅ dt
2
(GATE 2010: 1 Mark) T 0

p 2  
T 2

T

T  ∫0 ∫
Solution:  Given that x(t) = 8 sin  0.8p t +  = (1) L sin n w t dt + (− 1) sin n w t dt 
 4 0 0

 T 2 
2p 2p  cos nw t  T 2  cos nw t  T 
 
Also, T = = = 2.5 s 2
w 0.8p 0 0
 −nw 0  0
= + 
T  nw 0  T 2 
Ans. (d)  
[(1 − cos np ) + (cos 2np − cos np )]
27. The system represented by the input-output rela- 2
=
5t nw 0 T
tionship y(t) = ∫ x(t )dt , t > 0, t > 0 is
[1 − (−1) ]
2
[∵ w 0 = p T ]
n
−∞ =
np
(a) linear and causal
Therefore,
(b) linear but not causal
(c) causal but not linear  4
 , n odd
(d) neither linear nor causal bn =  np
(GATE 2010: 1 Mark) 0, n even

Solution:  Given that Thus, only odd harmonics will be present in x(t)
5t for second harmonic component, and the amplitude
y(t) = ∫ x(t ) dt , t>0 is zero.
−∞ Ans. (a)
Causality: y (t ) depends on x(5t), t > 0 So the 29. At t = 0, the function f (t) =
sin t
has
system is non-causal, as it depends on future value t
of input. (a) a minimum. (b) a discontinuity.
Linearity: The given system is linear as output is (c) a point of inflection. (d) a maximum.
the integration of the input (linear function)
Ans. (b) (GATE 2010: 2 Marks)
28. The second harmonic component of the periodic Solution:  Given that at
waveform given in the figure has an amplitude of
t = 0 ; f (t ) =
sin t
t

+1 sin x
By L’hospital rule, lim =1
x=0 x
1 As r(t) is sinc function, so it is minimum at t = 0.
0 T/2 T
Ans. (d)
−1 30. x(t) is a positive rectangular pulse from t = -1 to
t = +1 with unit height as shown in the following
2 ∞

∫ X(w ) dw {where X(w) is


(a) 0     (b) 1     (c)      (d) 5 2
p
figure. The value of
−∞
(GATE 2010: 1 Mark) the Fourier transform of x(t)} is

Chapter 3 Solved Question Paper .indd 260 3/22/2016 2:34:09 PM


SOLVED GATE PREVIOUS YEARS’ QUESTIONS        261

x(t) On comparing with output given in Eq. (ii), we get


1 a =1
−a + b = 0 ; b = a = 1
t −b + c = 0 ; c = b = 1
−1
−c + d = 0 ; d = c = 1
0 1
(a) 2     (b) 2p     (c) 4     (d) 4p Therefore,
(GATE 2010: 2 Marks) h (n ) = {1, 1, 1, 1}

Solution:  By Parseval’s identity, Ans. (c)
∞ ∞ Common Data for Questions 32 and 33:
∫ X (w ) dw = ∫ x (t ) dt
1 2 2 Given f(t) and g(t) as shown in the following figure:
2p −∞ −∞ f(t) g(t)

∫ X (w ) dw = 2p × 2 = 4p
2

−∞ 1 1
Ans. (d)
31. Given the finite length input x[n] and the corre- 0 1 t 0 3 5 t
sponding finite length output y[n] of an LTI system
as shown in the following figure. The impulse 32. g(t) can be expressed as
response h[n] of the system is t 
(a) g(t) = f(2t - 3) (b) g(t) = f  − 3
3 
 3  t 3
(c) g(t) = f  2t −  (d) g(t) = f  − 
h[n]
[n] = {1, -1} y[n] = {1, 0, 0 , -1}  2  2 2
↑ ↑

(GATE 2010: 2 Marks)
(a) h[n ] = {1, 0, 0, 1} (b) h[n ] = {1, 0, 1}
↑ ↑ Solution:  First scale F(t) by a factor of 1/2. Then
(c) h[n ] = {1, 1, 1, 1} (d) h[n ] = {1, 1, 1}  t
↑ ↑ g1 (t ) = F    (i)
 2

(GATE 2010: 2 Marks)

Solution:  Given that, finite length input is g1(t)

x[n ] = {1 − 1} 0 ≤ n ≤ 1 (i)
1
And output is
y[n ] = {−1, 0, 0, 0, −1} 0 ≤ n ≤ 4 (ii) 2 t
Impulse response, Next shift g1(t) by 3, then
y[n ] = h[n ] ∗ x[n ]  t − 3
g(t) = g1(t − 3) = f 
 2 
 (ii)
We have, length of y[n ] = 0 → 4; length of
x[n ] = 0 → 1; length of h[n ] = 0 → 3 g1(t)
Let h[n ] = {a, b, c, d}
By convolution
1
a b c d
1
a b c d 3 5 t

From Eqs. (i) and (ii)


−a −b −c −d
 t 3
−1 g(t) = f  − 
 2 2
y[n ] = {a, − a + b, − b + c, − c + d, −d}  Ans. (d)

Chapter 3 Solved Question Paper .indd 261 3/22/2016 2:34:40 PM


262        CHAPTER 3:  SIGNALS AND SYSTEMS

33. The Laplace transform, of g(t) is Solution:  Given two continuous time signals that
exist for t >0
(b) (e−5s − e−3s )
1 1
(a) (e3s − e5s )
s s x ( t ) = e−t
e−3s y ( x ) = e−2t
(1 − e−2s )
1
(c) (d) (e5s − e3s )
Convolution is z (t ) = x (t ) ∗ y (t )
s s
(GATE 2010: 2 Marks)
On taking Z-transform
Solution:  g(t) can be expressed as,
1 1
z(s) = x(s). y(s) = ⋅
g(t) = u(t − 3) − u(t − 5) s+1 s+ 2
By partial fraction method,
On taking Laplace transform (shifting properties)
1 A B
e−3s ( s + 1) ( s + 2 )
= +
G(s) = e−3s − e−5s = [1 − e−2s ]
1 1 s +1 s + 2
s s s 1 = A ( s + 2 ) + B ( s + 1)
Ans. (c) s = −1; s = −2;
A =1 B =1
34. The Fourier series expansion
∞ 1 1
∑ an cosnwt + bn sinnwt
z(s) = +
f (t) = a0 + s +1 s + 2
LT−1(s) = z ( t ) = e−t − e−2t
n =1
of the periodic signal shown below will contain the
following non-zero terms Ans. (a)
36. A zero mean random signal is uniformly distrib-
f(t) uted between limits -a and +a and its mean
square value is equal to its variance. Then the rms
value of the signal is
a a
t (a)     (b)     (c) a 2     (d) a 3
0 3 2
(GATE 2011: 2 Marks)
(a) a0 and bn, n = l, 3, 5, ……. ∞ 1/ 2
(b) a0 and an, n = 1, 2, 3, …. ∞ Solution:  We know that rms value = [variance]
(c) a0, an and bn, n = 1, 2, 3, … ∞
(d) a0 and an, n = 1, 3, 5, … ∞ [a − (−a )]2 4a 2 a2
Variance = = =
12 12 3
Solution:  The given waveform satisfies the condi-
Therefore, rms value is
tion of half wave symmetry. For half wave symme-
try, the signal contains only odd harmonics, there
will not be any even harmonics. a2 a
=
As it satisfies the condition for even symmetry, 3 3
so bn = 0. Therefore, two possible options are (b) Ans. (a)
and (d).
However, from half wave symmetry (odd har- 37. Let the Laplace transform of a function f(t) which
monic), we get n = 1, 3, 5, 7 ¥, therefore a0 and exists for t >0 be F1(s) and the Laplace transform
an, n = 1, 3, 5, …∞ is the correct answer. of its delayed version f(t − t) be F2(s). Let F1*(s)
be the complex conjugate of F1(s) with the Laplace
variable set as s = s + jw. If
Ans. (d)
35. Given two continuous time signals x(t) = e-t and
y(t) = e-2t F2 (s) ⋅ F1* (s)
which exist for t > 0, the convolution G(s) = 2
z(t) = x(t) * y(t) is F1(s)

(a) e-t - e-2t (b) e-3t then the inverse Laplace transform of G(s) is
(c) e+t (d) e-t + e-2t (a) an ideal impulse d(t).
(GATE 2011: 1 Mark) (b) an ideal delayed impulse d(t −t).

Chapter 3 Solved Question Paper .indd 262 3/22/2016 2:35:06 PM


SOLVED GATE PREVIOUS YEARS’ QUESTIONS        263

(c) an ideal step function u(t). 1 1 1


(a) < z <3 (b) < z <
(d) an ideal delayed step function impulse u(t − t). 3 3 2
1 1
(GATE 2011: 2 Marks) (c) < z < 3 < z
(d)
2 3
Solution:  Given that
LT [f (t)] = F1 (s) for t > 0 (GATE 2012: 1 Mark)
       F2 (s) = LT {F (t − t)} = e −st F1 (s)
(Delayed version of F1 (s)). Solution:  Given that
Given,
 1  1
n n
x(n) =   −   u(n)
F2 (s) ⋅ F1 * (s)  3  3
G(s) = −n
 1  1  1
2 n n
F1(s) =   u(n) +   u(−n − 1) −   u(n)
 3  2  2
e−st F1(s) ⋅ F1 * (s)
= 2 Talking Z-transform,
F1(s)

 1
n
e−st F1(s) ∑  3  z−n u(n)
2
= e−st
X(z) =
= 2 n =−∞
F1(s)
∞ −n
 1
The inverse Laplace transform is given by + ∑  3  z−n u(−n − 1)
n =−∞
LT−1[G(s)] = g(t) = d (t − t ) ∞
 1  −n
n
− ∑  z u(n)
Ans. (b) n =−∞ 2
∞ −1 −n
 1  −n  1
n
∑  3  ∑  3  z−n
1
38. The unilateral Laplace transform of f(t) is . = z +
s2 + s + 1 n =∞ n =−∞
The unilateral Laplace transform of t f(t) is ∞
 1
n

2s + 1 − ∑   z−n
(a) −
s
(b) −  
n =0 2
2
(s + s + 1) 2
(s + s + 1)2
2
Let m = -n, then
s 2s + 1
(c)  2 (d)  2
∞ ∞ ∞
 1 1   1
n m n
(s + s + 1)2 (s + s + 1)2
X(z) = ∑  3z  + ∑  z
 
− ∑  2z 
(GATE 2012: 1 Mark) n =0 m =1 3 n =0


 1
n
∑  3z 
1 1 1
Solution:  Given f (t) = 2 ; converges if < 1 or z >
s + s+1 n =0 3z 3


1 
1 m

1
 z  ; converges if z < 1 or z < 3
LT [f (t)] = F (s) = 2
s + s+1
m =1 3 3
dF (s)
LT [tf (t)] = (−1) ∞
 1
n
∑  2z  ; converges if 2z < 1 or z > 2
ds 1 1
−(2s + 1)
= (−1) 2 n −0
s + s+1
1
= 2
2s + 1 Therefore, region of convergence = < z <3
2
s + s+1
Ans. (c)
Ans. (d)
|n|
40. Let y[n] denote the convolution of h[n] and g[n],
39. If x[n ] = (1 3) − (1 2) u[n ], then the region of
n
where h[n] =(1/2)n u[n] and g[n] is a causal
convergence (ROC) of its Z-transform in the sequence. If y[0] = 1 and y[1] =1/2, then g[1]
z-plane will be equals

Chapter 3 Solved Question Paper .indd 263 3/22/2016 2:35:30 PM


264        CHAPTER 3:  SIGNALS AND SYSTEMS

1 3 Inverse Fourier transform of sinc function is a rect-


(a) 0     (b)      (c) 1     (d) angular function
2 2 h1(t)
h1(t)
(GATE 2012: 2 Marks)
0.5
Solution:  Given that 0.5 FT sin 3w
−3 0
t FT sinw3w
 1
n 3
h[n ] =   u(n) y[0 ] = 1 y[1] =
1
−3 0 3
t w
 2 2
Convolution sum, h2(t)

h2(t)
y[n ] = h[n ] ∗ g[n ] = ∑ h(n) g(n − k) 1/2
k =−∞
sin w
1/2
FT
t sinw w
−1
For causal sequence, FT
w
1 t
∞ −1
∑ h(n) g(n − k)
1
y[n ] =
k =0 F −1[H( jw )] = h(t) = h1(t) + h2 (t)
y[n ] = h(n)g(n) + h(n)g(n − 1) + h(n)g(n − 2) + ..... 1 1
h(0) = h1(0) + h2 (0) = + =1
2 2
When n = 0;
Ans. (c)
y[0 ] = h(0)g(0) + h(1)g(−1) + .....
(∵ for casual system g(−1) = g(−2) + ...... = 0)
42. The input x(t) and output y(t) of a system are
t
= h(0)g(0) related as y(t) = ∫ x(t ) cos(3t )dt . The system is
When n = 1, −∞

y[1] = h(1)g(1) + h(1)g(0) + h(2)g(−1) + ......


(a) time-invariant and stable
(b) stable and not time-invariant
= h(1)g(1) + h(1)g(0) (c) time-invariant and not stable
Since (d) not time-invariant and not stable

 1
1 (GATE 2012: 2 Marks)
1 1
h[1] =   = and y[1] =
 2 2 2
Solution:  Given the relation between input and
We have output as
t
1 1
= [ g(1) + g(0)] ⇒ g(1) = 1 − g(0)
2 2
y(t) = ∫ x(t ) cos(3t )dt
−∞
Already we have,
To check for time invariance, let x(t) = d (t). Therefore,
g(0) 1
y(0) = h(0)g(0); g(0) = = =1 t

∫ d (t) cos(3t )dt


h(0) 1
y(t) =
Therefore, g (1) = 1 − 1 = 0 −∞
Ans. (a) = u(t) cos(0) = u(t)
41. The Fourier transform of a signal h(t) is H(jw) = For delayed input,
(2 cosw) (sin2w)/w. The value of h(0) is t

(a)
1
(b)
1
(c) 1 (d) 2
y(t, t0 ) = ∫ d (t − t0 ) cos(3t )dt
4      2      −∞
    
= u(t) cos(3t0 )
(GATE 2012: 2 Marks)
Delayed output,
Solution:  Given that y(t − t0 ) = u(t − t0 )
(2 cos w )(sin 2w ) y(t, t0 ) ≠ y(t − t0 )
F [h(t)] = H ( jw ) =
w Hence, it is time variant.

Chapter 3 Solved Question Paper .indd 264 3/22/2016 2:35:59 PM


SOLVED GATE PREVIOUS YEARS’ QUESTIONS        265

To check for stability, consider a bounded input Solution:  Given that


v(t) = 30 sin 100t + 10 cos 300t + 6 sin (500t + p 4)
x(t) = cos3t

1 − cos 6t
t t

∫ ∫
2
y(t) = cos 3t = So the fundamental component is 30 sin 100t.
2
−∞ −∞ Third harmonic component is 10 cos 300t.
t t

∫ ∫
1 1
= dt − cos 6t ⋅ dt Magnitude = 10 =
30
2 2
−∞ −∞ 3
Frequency = 300 = 100 × 3 rad/s
As t → ∞, y(t) → ∞ (unbounded), hence the
system is not stable.  p
Fifth harmonic component = 6 sin  500t + 
Ans. (d)  4
43. The impulse response of a system is h(t) = t u(t). 30
For an input u(t − 1), the output is Magnitude = 6 =
5
t2 t(t − 1)
(a) u(t) (b) u(t − 1) Frequency = 500t = 100 × 5 rad/s
2 2
Therefore, fundamental frequency = 100
(t − 1)2 t2 − 1
(c) u(t − 1) (d) u(t − 1) Ans. (a)
2 2
46. A band-limited signal with a maximum frequency
(GATE 2013: 1 Mark) of 5 kHz is to be sampled. According to the sam-
pling theorem, the sampling frequency in kHz
Solution:  Given that impulse response is h(t) = which is not valid is
tu(t). For input response u(t-1), (a) 5     (b) 12     (c) 15     (d) 20
d(t) → ta(t)
(GATE 2013: 1 Mark)
u(t) → tu(t)dt = u
t Solution:  Given that fmax = 5 kHz
t2
∫ tdt = 2
u(t) According to sampling theorem, fs ≥ 2fmax
0
Since fmax = 5 kHz
(t − 1)2
u(t − 1) → u(t − 1) Therefore, fs min = 10 kHz
2
Ans. (c) This implies that fs should be ≥10 kHz. So option
(a) is invalid option.
44. Two systems with impulse responses h1 and h2(t)
Ans. (a)
are connected in cascade. Then the overall impulse
response of the cascaded system is given by 47. Assuming zero initial condition, the response y(t) of
the system given below to a unit step input u(t) is
(a) product of h1(t) and h2(t)
(b) sum of h1(t) and h2(t) u(s) 1 y(s)
(c) convolution of h1(t) and h2(t) s
(d) subtraction of h2(t) from h(t)
t2
(GATE 2013: 1 Mark) (a)  u(t)   (b)  t u(t)   (c)  u(t)    (d)  e −1u(t)
2
Solution:  The overall impulse response of the (GATE 2013: 1 Mark)
cascaded system is the convolution of individual
impulse responses. Therefore, Solution:  For the given system, the transfer
­function is
h1(t) * h2 (t)
1
Ans. (c) Y /(s) = ⋅ V (s)
s
45. For a periodic signal v(t) = 30 sin 100t + 10 cos 300t
+ 6 sin(500t + p/4), the fundamental frequency in To find y(t), take inverse Laplace transform on
rad/s is both sides
(a) 100    (b) 300    (c) 500    (d) 1500 y(t) = u(t) ⋅ t
(GATE 2013: 1 Mark) Ans. (b)

Chapter 3 Solved Question Paper .indd 265 3/22/2016 2:36:26 PM


266        CHAPTER 3:  SIGNALS AND SYSTEMS

48. Which one of the following statements is NOT Solution:  Given


TRUE for a continuous time causal and stable LTI
system? h(t) = d (t − 1) + d (t − 3)

(a) All the poles of the system must lie on the left We have to find the step response u(t) = at t = 2.
side of the jw axis.
(b) Zeros of the system can lie anywhere in the s-plane. h(t) = d (t − 1) + d (t − 3)
(c) All the poles must lie within |s| = 1.
(d) All the roots of the characteristic equation u(t) h(t) y(t)
must be located on the left side of the jw axis.
(GATE 2013: 1 Mark) Substituting t = 2
h(2) = d (1) + d (−1)
Solution:  For a continuous time causal and stable
LTI system, all poles must lie within unity circle h(2) = u(1) + u(−1) = 1
|s| = 1.
Ans. (b)
Ans. (c)
49. The impulse response of a continuous time system
is given by h(t) = d (t − 1) + d (t − 3). The value of
the step response at t = 2 is
(a) 0      (b) 1      (c) 2      (d) 3
(GATE 2013: 2 Marks)

Chapter 3 Solved Question Paper .indd 266 3/22/2016 2:36:32 PM


SECTION IV: ELECTRICAL MACHINES

MARKS DISTRIBUTION FOR GATE QUESTIONS

12
11
10
Number of questions

9
8
7
6
5 Marks 1
4 Marks 2
3 Total number of questions
2
1
0
2009 2010 2011 2012 2013 2014 2015

TOPIC DISTRIBUTION FOR GATE QUESTIONS

Year Concepts
2015 Transformers, DC machines
2014 Single-phase transformer, DC machines
2013 Single-phase transformer, Induction motor
2012 Induction motor, Synchronous generator, DC motor, Three-phase induction motor, Transformer
2011 DC Machine, Synchronous machine, Transformer, Induction motor
2010 Transformer, Induction motor, DC machines
2009 Synchronous machine, Induction motor, DC machines, Transformer

Chapter 4_part opener.indd 267 3/23/2016 11:21:21 AM


Chapter 4_part opener.indd 268 3/23/2016 11:21:21 AM
CHAPTER 4

ELECTRICAL MACHINES

The electrical machines are energy conversion devices in detail. The double revolving field theory for the opera-
which are widely used either for generating electrical tion of single phase induction machine is provided. Then
energy from mechanical energy or vice versa. These various connections and the characteristics of the single
have a very high degree of importance in the field of phase machine are also covered. In the last section, the
studying electrical engineering. In this unit of the book, three phase synchronous machine is covered. The focus
in the first section, the basic operating philosophy of is on voltage regulation with the help of synchronous
transformer action including transformer operating prin- impedance method, the operating principle is covered.
ciple is described along with different relevant impor-
tant derivations, phasor diagrams and formulae. Then
three phase transformer vector groupings and relevant 4.1  TRANSFORMER
operational features are included. In the next section the
DC machine is described as an energy conversion device.
Important formulae are derived and machine descrip- The transformers are static energy conversion devices
tion is given. The machine characteristics are also pro- which convert electrical power from one voltage level to
vided for various connections. In the next section both another without changing the apparent power. The trans-
the three phase and single phase induction machines former works in the principle of induction between the
are comprehensively covered. This includes the machine two coils and thereby also provides isolation between
characteristics, operating principles, torque production the primary and the secondary windings.

Chapter 4.indd 269 3/23/2016 11:23:19 AM


270     Chapter 4:  Electrical machines 

4.1.1  Ideal Transformer winding is connected to a voltage source of V1 volts and


has T1 turns. The secondary winding has T2 turns and
An ideal single phase transformer has two coils. The can be connected to any load impedance for loading the
basic working principle of a transformer is governed by transformer. Although the primary and secondary are
Faraday’s Law where an electromotive force (emf) is shown on the same limb in Fig 4.1, the two windings are
induced in a coil when it is linked with a time varying made in two different limbs for a practical transformer in
magnetic field. A time-varying field is created by pass- order to reduce weight space, cost weight, etc. Due to the
ing an alternating current through a coil. This coil is impressed voltage V1 in the primary a current I0 amperes
known as the primary coil. Now if the same time varying is passed through the primary winding of T1 turns which
flux is linked by another coil an emf will be induced in sets up an magetomagnetic force (mmf) of I0T1 amperes.
it. This phenomenon is called mutual induction. The This mmf in turn sets up a flux f through the core. Since
second coil is known as the secondary coil. The flux link- the reluctance of the iron path is given by R = l/mA and
age between the two coils can be through any medium the permeability m→∞ as per the earlier assumptions, a
which can even be air. In this case the transformer is vanishingly small value of current I0 will be required to
called air-cored transformer. But the air has got a setup the working flux f for the transformer.
large magnetic reluctance which can result in a large cur-
V1 = V1mcosw t m→•
rent to generate the required magnetic flux for adequate
i ≅0 f
voltage induction in the secondary coil. Moreover, the
linkage of flux generated by the primary winding cannot + +
be ensured to link the secondary coil if the core is made e1 T1
of air. Thus a ferromagnetic core is used between the two
coils to reduce the current required for producing mag- −
netisation flux in the secondary coil and also to ensure
the almost 100% flux linkage between the primary and +
the secondary winding of the transformer. If the second- e2 T2
ary winding is connected to a load, then the load power
has to be supplied from the source through the magnetic −
field as a link. In an ideal case the magnetic medium does
not absorb any energy. But in reality the magnetic core
losses occur in the medium in terms of hysteresis and (a)
eddy current losses although the same can be restricted i1
to within 1% of the total rated output power of the trans-
former. Also the winding losses can be made very small + i2
by proper choice conductor size. Therefore, the ideal effi- +
ciency of a transformer approaches 99% in case of larger
transformers. In order to have better understanding V1 = V1sinw t e1
about the behaviour of the transformer, the approxima- e2
tions made for the analysis of `ideal’ transformer is to be
studied. These idealised conditions are listed as follows:
  1. A linear magnetic circuit with infinite permeability + −
is considered for analysis. This results in a vanish- (b)
Figure 4.1 |   Two-winding ideal transformer:
ingly small current in the primary winding which is
enough to establish the required magnetic flux for
(a) Unloaded machine and (b) circuit.
transfer of power from primary to secondary wind-
ing. Moreover, this assumption leads to a negligible When a sinusoidal voltage V1 is applied to the primary
hysteresis loss in the magnetic core. Also infinite core as shown in Fig. 4.1(a), the instantaneous values of the
permeability results in no leakage of fluxes as all the applied voltage and induced emf e1 can be written as fol-
flux generated confines itself to the iron core ideally. lows based on Kirchhoff’s voltage law as:
  2.  Both the primary and secondary windings have
negligible resistance. This assumption leads to neg- v1 − e1 = 0  (4.1)

ligible copper losses in the windings. In fact, the where v1 is the instantaneous value of the applied voltage
characteristics of practical transformers are very and e1 is the induced emf in the primary winding due to
close to this approximate model and hence no major Faraday’s principle. Equation (4.1) has the assumption
deviation is made in making these assumptions. of zero primary resistance and results in v1  = e1 which
Figure 4.1 shows the schematic representation of a two- implies that the induced emf must be same in magnitude
winding single phase ideal transformer. The primary as the applied voltage at every instance of time. Let

Chapter 4.indd 270 3/23/2016 11:23:23 AM


4.1  TRANSFORMER     271

Since the flux linkage is common to both the coils,


V1 = V1m coswt
f 1m = f 2m = fm
then, as per Faraday’s law the induced emf is given by,
dy 1 The voltage ratio between the primary and secondary
e1 = −
dt windings from the two induced emf Eqs. (4.2) and (4.3) is
where y1 is the flux linkage. The negative sign indicates
E1 N
that the emf is induced in the direction so as to oppose = 1  (4.4)
the cause that produces it (as per Lenz’s law). However, E2 N2

for all further derivations, only the magnitude is taken
Since the coils have a common flux linkage, the auto-
into consideration and minus sign is omitted.
matic transfer switches (ATs) of both the coils are the
For a sinusoidal input voltage V1, it can be easily seen same. Thus, i1T1 = i2T2 provided magnetising current
that the variation of flux linkages can be given by is neglected, which implies that i0 = 0. Thus, the rms
y 1 = y 1m sinw t. value
Thus, the primary induced emf can be obtained from the
above two equations as, I2 N 2 = I1N1

(y sinw t)
d
e1 = or,
dt 1m
Therefore, æN ö
I1 = ç 2 ÷ I2
or, e1 = y 1m × w coswt è N1 ø
Again
Hence, the rms value of induced emf is given by
E1 N
y 1mw = 1
E1 = E2 N2
2
The primary flux linkage y1m is given by N1f1m where Therefore from the above,
f1m is the primary flux and N1 is the number of terms. E1 N I
= 1 = 2
2p fN 1f1m E2 N2 I1
Thus, E1 =
2
or or E1I1 = E2 I2  (4.5)
E1 = 4.44f N1 f1mV  (4.2) Also input voltage is equal to output voltage, so
Here y1m is the peak value of the primary flux link-
V1 I E
age. As per the working principle of transformer, the = 2 = 1
same mutual flux links the secondary winding through V2 I1 E2
the common magnetic core. However, the magnitude of
the flux linkages in the secondary winding will differ and 4.1.1.1 Equivalent Circuit and Phasor
can be given by y2m =N2f2m. The induced emf in the Diagram
secondary winding can be similarly obtained as
dy 2 The equivalent circuit from of a practical trans-
e2 = former obtained from the above discussion is shown
dt
in Fig. 4.2.
The secondary flux linkage can be expressed as y2 =
y2m sinwt. r′1 jx′l1 I1′ r2 jxl2 I2′
I0′
(y sinwt)
d
Thus e2 =
dt 2m Ic′ I′m
V ′1 ZL V2
e2 = y 2m × w coswt
R′c jx′m
or

Therefore, the rms value can be obtained in a similar


manner as,
Figure 4.2 |   Equivalent circuit referred to the
E2 = 4.44f N 2f2m V ­secondary side.

Chapter 4.indd 271 3/23/2016 11:23:48 AM


272     Chapter 4:  Electrical machines 

The element Rc shown in Fig. 4.2 is the equivalent core V1


loss resistance and Xm is the magnetising reactance.
Assuming a linear magnetic circuit the value of Xm can E1
be a constant value. The other terms x1, r1, x2, r2 are I2′ I1
the primary winding leakage reactance, resistance and
secondary winding leakage reactance and resistances I1Z1 f1
respectively.
As per the earlier assumptions the value of the series I1X1 I1R1
f
aI0
elements of the equivalent circuit, that is r1, x1, r2, and I2R2
x2 are very small as compared to the shunt elements Rc I2X2
and Xm. Therefore, the equivalent circuit can be simpli- I2Z2
f2
fied for easier calculations as shown in Fig. 4.3.
I2 V2
I1 I′2 r1 jx11 r′2 jx′12
I0 R = r1 + r′2 E2
Ic Im
V1 Rc Z′L V ′2 X = X11 + X′12 (b)
jxm I1 = I2′ Figure 4.4 |   Transformer phasor diagram under (a) no
load condition and (b) loaded condition.
I1 R jX I′2
4.1.1.2 Impedance Transformation
V1 V ′2 Equivalent circuit of a transformer as shown in Fig. 4.3 can
be represented from either of the primary or the secondary
sides. Thus all the equivalent circuit parameters are to be
Figure 4.3 |   Simplified equivalent circuit. either referred to the primary side when represented from
the primary side or vice versa. The voltages and currents
are also to be transformed to either of the primary or sec-
Phasor Diagram for Transformer under ondary sides for representing the equivalent circuit from
No Load and Loaded Conditions the corresponding side as shown in Figs. 4.5(a) and (b).
In case of a practical transformer, the core will have While referring a secondary impedance to the primary
finite permeability and also there will be core losses. side, the secondary side impedances are multiplied by the
Under no load conditions only primary side currents will square of the turns ratio, a = N1/N2, where N1 and N2 are
flow. These include the current due to core losses and the the primary and secondary side turns. Similarly, the pri-
magnetising current. The phasor diagram under no load mary impedances are divided by the square of turns ratio
condition is shown in Fig. 4.4(a). when they are referred to the secondary side.
The phasor diagram corresponding to a loaded trans- I1 I ′2 a = N1/N2 I2
former can be obtained from the voltage and current
phasors using the ideal equivalent circuit. The same Im I ′m
a2Z
V ′2
should also satisfy the Kirchhoff’s voltage equation 2 V1 Xm
V1 Xm Z2 V2
for both the primary and secondary circuits. The a a2
phasor diagram under loaded condition is shown in
Fig. 4.4(b).
V1 = −E1 (a) (b)

Figure 4.5 |   Equivalent circuits. (a) Primary and


(b) Secondary.
f
0
Im 4.1.1.3 Determination of Equivalent Circuit
Parameters
E2
Open Circuit or No-Load Test
E1
The shunt parameters of the equivalent circuit, that is,
(a) Xm and Rc are calculated from open circuit or no-load

Chapter 4.indd 272 3/23/2016 11:23:53 AM


4.1  TRANSFORMER     273

test. The secondary of the transformer under consid- Short-Circuit Test


eration is kept open circuited with rated input voltage
The series parameters of the equivalent circuit, that is,
applied to the primary terminals. The measuring instru-
resistance and leakage inductance parameters X1, X2 and
ments are connected on the low voltage side and the high
R1, R2 are calculated from short-circuited test (Fig. 4.7).
voltage side is open circuited. The input current and
The measuring instruments are connected on the high
the power are measured. The active component of the
voltage side and the low voltage side is open-circuited.
no load current I0 represents the core loss. The reactive
Voltage is gradually increased from a low value; the
component is the magnetising current Im. The circuit
rated full load current will appear at lower voltages as
diagram and equivalent circuit for the test are shown as
the secondary is short circuited. The values of voltage
follows in Fig. 4.6(a) and (b).
Vsc, current Isc and power Psc are noted.

A
A
V1 V W0 V2
Vsc V Wsc

(a)
(a)
I0
Isc
Im Ic
Re Xe
V1 jXm Rc
Vsc

(b)

Figure 4.6 |   No-load test. (a) Circuit.


(b)
(b) Equivalent circuit. Isc R1 X1 X′2 R′2

From the circuit, the wattmeter reading is the no load


input power. Thus,
Vsc
P0 = V1I 0 cos f0

where, cos f0 is the no-load power factor. (c)

Now, cos f0 =
P0 Figure 4.7 |   Short-circuit test. (a) Circuit.
V1I 0 (b) Equivalent circuit. (c) Equivalent
circuit with primary and secondary
and sin f0 = 1 − cos2 f0 resistances and leakage reactances.

From the equivalent circuit, the wattmeter reads the full


Therefore, utilising the no load equivalent circuit shown
load copper losses. Thus,
in Fig 4.6(b), the equivalent circuit parameters Rc and
Xm can be calculated as, 2
Psc = I sc Re
V1 Psc
Rc =
I 0 cos f0 Re = 2
I sc
V1 And, Re = R1 + R2
and Xm =
I 0 sin f0 Here, the terms Re and Xe refer to equivalent resistance
and reactance, respectively
where, I c = I 0 cos f0 and I m = I 0 sin f0 Now,
Vsc Vsc
Thus both the core loss resistance and magnetising reac- = Re + jXe or = Re2 + Xe2 = Ze
tance can be determined by this test. I sc I sc

Chapter 4.indd 273 3/23/2016 11:24:09 AM


274     Chapter 4:  Electrical machines 

Thus, 4.1.1.4 Voltage Regulation

When a transformer operates on load, its voltage devi-


Xe = Ze2 − Re2
ates from the no-load value. The change in voltage
occurs due the voltage drop occurring in the transformer
It is generally assumed that, X1 = X2 and R1 = R2 when circuit. This drop is termed as the voltage regulation and
referred to any of the sides. Therefore, is expressed as a ratio of the terminal voltage (the abso-
lute value). The voltage regulation can be calculated as

X1 = X2 =
1
X Vnl − Vl
2 e Regulation =
Vnl

R1 = R2 =
1 The terms Vnl and Vl are no-load and load terminal volt-
And R
2 e ages, respectively. The no-load voltage is generally taken
as the reference. Taking rated full load voltage as refer-
Thus, the series parameters can be determined by this
ence the voltage regulation can also be defined in the
test.
other way as,

Vnl − Vl
Sumpner’s or Back to Back Test Regulation =
Vl
The Sumpner’s test is performed to test the tempera-
ture rise of the transformer under full load condition and The equivalent circuit can be redrawn after referring
also to determine its full load efficiency. For this test, from secondary side as shown in Fig. 4.9.
two identical transformers of same rating are required.
The transformer can be loaded fictitiously to full-rated
Re jXe
condition. The primaries are connected in parallel and
secondary windings are connected in phase opposition as I′2
shown in Fig. 4.8.
V1 V ′2
A
T1
x1 x2
Figure 4.9 |   Equivalent circuit drawn from
W2
B A2 T
secondary side.
C
T2 LV The terms Re and Xe are the equivalent resistance and
2I0 I1′ I2′ reactance referred to secondary side. Thus, the correspond-
W1
ing phasor diagram can be drawn as shown in Fig. 4.9.
D
A1 D
V1
Supply
V1

0 A B
Figure 4.8 |   Circuit diagram for Sumpner’s test. f V ′2 I2′Xe C E

Voltage is applied to the primary winding such that full I2′Re


load current is impressed in the secondary loop. The I2′
two sources together meet the core and copper losses
of both the transformers. In this test, iron or core loss Figure 4.10 |   Phasor diagram drawn from the
occurs in the cores and full load copper loss occurs in secondary side.
the windings of both the transformers. Current drawn
by the primaries is twice the no-load current of each From the phasor diagram and equivalent circuit shown
transformer. in Figs. 4.9 and 4.10, it is observed that,

Chapter 4.indd 274 3/23/2016 11:24:17 AM


4.1  TRANSFORMER     275

V1 = I2¢ (Re + jXe ) + V2¢ 4.1.1.5 Transformer Losses

OD = V1 = [OA + AB + BC]2 + [CD]2   1.  Core loss or Iron loss: The transformer core
losses may be divided into two types:
[V2¢ + I2¢ Re cos f + I2¢ Xe sin f ]2
= (i) Hysteresis loss: It is the loss that occurs in
+[I2¢ Xe cos f − I2¢ Re sin f ]2 the iron core, due to the magnetic reversal of
the flux in the transformer core, which results
where f is the load power factor angle and q is internal in heating in the core. This loss is directly
proportional to the supply frequency. This
impedance angle = tan−1 e
X
Re loss can be minimised using core material of
high permeability.
Again,
The hysteresis loss is given by
V1 = V2¢ + I2¢ (Re + jXe )
Ph = khf B xm
= V2¢ + I2¢ (cos f − j sin f )(Re + jXe )  (4.6)
where kh is the hysteresis loss constant and x
Therefore the voltage regulation can be calculated from is Steinmetz's constant.
Eq. (4.6) as, (ii) Eddy current loss: This loss is due to the
alternating flux linking the core, which induces
V1 − V2¢ an emf in the core called the eddy emf, due
Regulation R =  (4.7)
V2¢ to which a current called the eddy current
circulates in the transformer core. This eddy
Both the approximation of Eq. (4.6) and Fig. 4.10 will current circulation converts into heat and is
provide the simplified expression for voltage regulation as, called the eddy current power loss. Eddy cur-
rent loss is proportional to the square of the
Regulation R = I2¢ Re cos f + I2¢ Xe sin f  (4.8) supply frequency.

As can be observed for from Eq. (4.8), the situation for The Eddy current loss is given by
zero voltage zero voltage regulation can occur when, x
Pe = ke f 2B m
I2¢ Re cos f + I2¢ Xe sin f = 0 where ke is the Eddy current loss constant.
X This loss can be minimised by making the core made
Or, tan f = − e  (4.9) of thin sheets of silicon steel stampings, and each
Re
lamination is coated with varnish insulation to con-
Thus giving a load power factor cos f = Xe/Ze, leading tain the pathway of the eddy current. Core losses
as f is negative from Eq. (4.9). are always constant for a particular transformer.
Similarly for maximum voltage regulation,   2.  Copper loss or Ohmic loss: Copper loss (I2R)
occurs in current carrying conductors. In trans-
dR formers, this loss occurs in the primary and the sec-
=0
df ondary windings when the transformer is on load.
The power is wasted in the form of heating due to
which gives, from Eq. (4.8) the coil resistances. This loss is dependent on the
The voltage regulation under leading load, load and hence treated as variable loss.
  3.  Stray losses: Stray losses in a transformer may
  
V2 = E2 − I2 ( re2 + jxe2 ) occur due to stray flux linkages which link with
  
E2 = V2 + I2 ( re2 + jxe2 )
mechanical structures in the transformer or with
winding conductors.
E2 = V 2 + I2 re2 cos q2 − I2 Xe2 sin q2   4.  Dielectric loss: Transformer dielectric loss occurs
in the bushings and oil and the components form-
Thus voltage regulation ing capacitances to earth. But this loss is very less
compared to other losses and generally not taken in
E − V2
= 2 × 100% to account for the transformers rated below 1 MVA.
V2
I2 re2 I x
cos q2 − 2 e2 sin q2
4.1.1.6 Transformer Efficiency
=
V2 V2
Transformer efficiency is the ratio of the output power
= Îr cos q2 − Îx sin q2 to the input power.

Chapter 4.indd 275 3/23/2016 11:24:31 AM


276     Chapter 4:  Electrical machines 

Input = Output + Total losses If V1 voltage is applied to the primary winding AB,
= Output + Iron loss + Copper loss voltage per turn is V1/N1. The voltage across secondary
winding BC is V2 from the Fig. 4.11. Now if the winding
In a transformer, efficiency BC has N2 turns, voltage per turn is for the secondary
winding is given by,V2/N2.
h=
Output power
Output power + Iron loss + Copper loss Thus transformation ratio of autotransformer is given
by,
xV2 I2 cos f
%h = × 100%
xV2 I2 cos f + Pcore + x2Pcu
V2 N
= 2 =a (also represented by K )
V1 N1
where, x = percentage loading of the transformer, V2
and I2 are secondary voltage and currents, respectively, Now, from Fig. 4.11,
cos f = power factor of load and Pcu and Pcore are copper
Transformed power (VA) (V1 − V2 )I1
and core losses, respectively. =
Input power (VA) V1I1
Condition for maximum efficiency
V2
= 1− = 1−a
dh d  xV2 I2 cos f 2  V1
=   =0
dI2 dI2  xV2 I2 cos f 2 + x Pcu + Pcore 
2

( )
Similarly,
xV2 cos q 2 xV2 I2 cos f 2 + x2Pcu + Pcore

( )
Conducted power (VA) V2 I1 V
−xV2 I2 cos q 2 xV2 cos f 2 + x2 2I2 re2 = 0 = = 2 =a
Input power (VA) V1I1 V1

(Using Pcu = x2 I22 re 2 ; where re2 = equivalent resistance Recall that power drawn by an equipment can be
related to secondary side) expressed in watts or volt-ampere (VA), where power in
watts is real power drawn by the equipment (machine)
xV2 I2 cos f 2 + x2Pcu + Pcore − xV2 I2 cos f 2 + 2x2Pcu = 0 and VA is the apparent power obtained as product of
x2Pcu = Pcore voltage applied and current drawn from the equipment.
For the transformer,
Pcore
x=
Pcu N1 V
= 1
N2 V2
All day efficiency

Total output for 24 hours Also N1I1 = N 2 I2


= ×100%
Total output for 24 hour + Total loosses for 24 hours I1 N
or = 2
I2 N1
4.1.1.7 Autotransformer
Secondary side copper loss = I22r2
An autotransformer is a transformer having part of its Now looking from the primary side, this loss will be
winding common for both the primary and secondary
æ N1 ö
2
windings (Fig. 4.11).
ç ÷ × I2 r2
2
A è 2ø
N

Thus, the same loss in terms of primary coil is = I22r′2


I1
So, secondary resistance referred to primary
I2
N 
2
V1 C = r2′ =  1  ⋅ r2
I2 − I1  N2 
V2 Load
Similarly, the secondary leakage reactance referred to
primary
æN ö
I1 B I2 2
x2¢ = ç 1 ÷ = x2
Figure 4.11 |   Autotransformer. è N2 ø

Chapter 4.indd 276 3/23/2016 11:24:48 AM


4.1  TRANSFORMER     277

Advantages: The advantages of autotransformers over Transformer A Transformer A


conventional two-winding transformers for the same VA primary secondary
rating are: A1 A2 a2 a1

  1.  These are usually more efficient than a two-­winding


transformer. Transformer B Transformer B
  2.  Smaller in dimension. primary secondary
  3.  Autotransformers require less copper in their con- B1 B2 b2 b1
struction which leads to lower cost than two-winding
transformers.
  4.  Their core and copper-losses are lower due to less Transformer C Transformer C
resistance and leakage reactance. primary secondary
  5. They give better voltage regulation than two-winding C1 C2 c2 c1
transformer of equivalent ratings.
Disadvantages: The autotransformer has no isolation Figure 4.12 |   Terminal markings for primary and
between the primary and secondary windings. Thus secondary transformers.
they cannot be safely used for stepping down higher
voltages. Also, if secondary becomes open, current will
stop flowing in the primary side, similarly if secondary 4.1.2.1 Different Types of Connections in
is made short, high current will flow on the primary side Three Phase Transformers
than an equivalent two-winding transformer of same
VA rating. A three-phase transformer can be built by connecting
bank of three single-phase transformers or simply by
using one unit of three-phase transformer. The primary
4.1.2  Three-Phase Transformer and secondary windings may be connected in either
star or delta arrangement. The four most common con-
A single-phase transformer generally has only two coils: nections are (i) star-star (Y-Y), (ii) delta-delta (D-D),
primary and secondary although multiple coil single (iii)  star-delta (Y-D) and (iv) delta-star (D-Y). These
phase transformers are also available. For a single phase  connections are shown in Fig. 4.13 and K represents the
transformer, the primary is energised with single-phase transformation ratio = N2/N1.
supply and load can be connected across the secondary.
However, in a three-phase transformer there are three I I/K
primary coils and three secondary coils. These three
primary and secondary coils are required for getting a V KV
three phase supply in the primary and generating a three √3 √3
V KV
phase voltage for the load. These three coils are prop-
erly connected either in star or delta so that the voltage
level of a balanced three-phase supply can be changed
to another three-phase balanced system of different volt-
age level. Three identical single-phase transformers also
can be suitably connected for this purpose to generate
a three-phase voltage at different level after receiving (a)
three phase voltage from any source. Such a system of I/K
I
three-phase transformer is called a three-phase trans-
former bank.
V KV
Figure 4.12 shows the coils of three single phase
transformers with primary and secondary windings. For
transformer A, primary terminals are marked as A1A2
I I
and the secondary terminals are marked as a1a2. The
√3 K√3
markings are done in such a way that A1 and a1 rep-
resent the dot (•) terminals which can be assumed as
start of the windings considering the coils are wound in (b)

Figure 4.13 |   Different connections of three-phase


same direction (i.e. CW or CCW). Similarly, for B and
C transformers the terminals are marked as shown in transformers: (a) star-star, (b) delta-
Fig. 4.12. delta, (c) star-delta and (d) delta-star.

Chapter 4.indd 277 3/23/2016 11:24:51 AM


278     Chapter 4:  Electrical machines 

I √3 I Delta-Delta(D-D) and Open-Delta (V-V)


K Connection
V Valid delta connection (Fig. 4.15) can be implemented
I/K
√3 KV/√3 in the usual way as shown in Fig. 4.13 for star system.
V
The output line to line voltage will be 100 V for an
input line voltage of 200 V considering transformation
ratio of 2:1. From the secondary, one can draw a line
current of 100 3A which means a total of 30 kVA can
be supplied.
When using three units of single-phase transformers in
(c)
a star-star connected system, if one unit fails or is discon-
nected for repair, three-phase power cannot be supplied
I until the unit is replaced by another unit. But in case
I K √3 of delta-delta connection, it is possible to supply three-
I phase power even with two transformers. The resulting

V √3 KV connection is known as open delta or (V-V-connection).


Under no-load conditions, if one transformer is detached,
K √3 V
the other two will continue to maintain the same line
voltages on the secondary side. Under load conditions
however, the secondary line voltages will be somewhat
unbalanced.
In a delta-delta connection, when the third trans-
(d) former C is removed from the circuit, as shown in the
Figure 4.13 |   (Continued).
second part of Fig. 4.15(a), only two transformers are
present to supply the load. This can happen due to
non-availability of one of the three transformers due
to a fault. Assuming a 2:1 ration between the primary
Star-Star (Y-Y) Connection and secondary coils, if the HV side is energised with
When the primary and the secondary sides of the trans- three-phase 200 V supply, 100 V can be obtained in the
former are star connected with A2B2C2 and a2b2c2 secondary side as observed from the phasor diagram
shorted, Fig. 4.12 gets modified to Fig. 4.14. in Fig. 4.15(b). However, as no transformer winding
exists between the primary sides A2 and B1, then for
secondary side no voltage will be induced for the coil
Balanced 3-phase supply

A A1 A2 a2 a1 between a2 and b1. In the diagram, the phasor rep-


resentation is shown by the dotted line assuming a
balanced three-phase supply. If the phase currents
B B1 B2 b2 b1 are 100 A for the three windings without discon-
nection, then the line current that will flow in the
secondary is 100 A and not 100 3A as in normal
C C1 C2 c2 c1 delta−delta connection. Therefore, total maximum
kVA handled by the system of open delta is given by
A1 a1 3 × 100 × 100 VA = 17.32 kVA which is about 57.7%
of the normal delta connected system.
c2 a2 Power delivered to the resistive load by this connec-
A2B2C2
b2 tion can be given as,
c1 b1
C1 B1

Figure 4.14 |   Primary and secondary sides are star PV = 3 × V L × I ph


connected.
For all three transformers in delta, power delivered is
given as,
The terminals of A2B2C2 are connected to primary-side
neutral N and similarly a2b2c2 of the secondary windings
can be connected together to form secondary-side neutral. PD = 3× VL × I ph

Chapter 4.indd 278 3/23/2016 11:25:00 AM


4.1  TRANSFORMER     279

Balanced 3-phase supply


A A1 A2 a2 a1 a
B2
A1
a1 b2
B B1 b2 b1
B1 b1
B2
C2 c2
C C1 c2 c1 c
a2
C2 A2 c1
C1
Delta-delta connection Delta-delta
Balanced 3-phase supply

A A1 A2 a2 a1 a
B2
A1
a1 b2
B B1 b2 b1
B1 b1
B2

C c
a2
A2
Open-delta connetion Open-delta
(a) (b)
Figure 4.15 |   (a) Delta-delta and open-delta connection. (b) Phasor diagrams.

Thus, Teaser
transformer (T)
PV IR I1
= 0.577 or 57.7% R
PD
0.866 N1 N 2 V1
Hence, power handling capacity of open delta connec-
tion reduces to 57.7% of total capacity of a delta-delta IB N1/2 N1/2
transformer. But the advantage is that, if one of the unit B
in delta system develops a fault, it can be removed from C
the circuit and power can be partially supplied to the IY
important loads and the same can be reconnected when Y Main
the fault is removed. transformer (M)
N2
I2
Scott transformer or Scott-T connection V2
A two-phase supply if sometimes required can be pro- (a)
vided using Scott-T connection. It requires two trans-
formers, the main and the teaser transformers. The B
main transformer has the centre tapped at a point C
with primary of N1 turns. The teaser has its primary at VCB VBR
0.866 N1 turns. One end of the teaser primary is con-
nected to the main transformer centre-tap C. The other C VRC
R
end of it is connected to terminal R of the three-phase 30°
supply. The secondary winding turns of both the trans- VYB
VYC
formers are equal. The voltages across the secondaries VRY
of the two transformers are equal in magnitude having
a phase displacement of 90° electrical similar to two- Y
phase connection. Thus Scott connection of two single-
(b)
phase transformers can convert three-phase to two-phase
supply. The connection is depicted in Fig. 4.6(a) along Figure 4.16 |    Scott-T connection. (a) Circuit
with its phasor diagram (b). diagram. (b) Phasor diagram.

Chapter 4.indd 279 3/23/2016 11:25:09 AM


280     Chapter 4:  Electrical machines 

4.1.2.2 Vector Group of Three-Phase 4.1.3  Parallel Operations of Transformers


Transformer
The transformers are generally operated in parallel for
Unlike the single phase transformers, the secondary volt- expansion of capacity or ease of maintenance. Conditions
ages of a three-phase transformer may undergo a phase for satisfactory parallel operation of single-phase trans-
shift of 30° leading or lagging when the connections of the formers are:
primary and the secondary are dissimilar. Dissimilarity   1.  Transformers should be connected properly with
means primary or secondary connections are either in regard to their polarities. This is an essential condi-
star/delta or delta/star. On the other hand, when the tion for paralleling single phase transformers.
connections of the primary and the secondary are simi-   2.  The voltage ratings and voltage ratio of the trans-
lar, that is, either in star/star or delta/delta the phase formers should be equal.
shift between primary and secondary will be rotated 0°,   3.  The per-unit impedance or percentage impedance
or 180° like the single phase transformer. A typical rep- of the transformers should be equal.
resentation of the vector group can be denoted as Dy1 or   4.  The reactance (X)/resistance (R) ratios of the
Yd11. The first capital letter indicates the primary HV transformers should be identical.
winding whereas second lower case latter indicates the For three-phase transformers, in addition to all
connection of the secondary side. The numerical figure above points, the other required conditions are:
indicates the phase shift between the primary and the   5.  Phase sequence of the two transformers to be con-
secondary winding. This phase shift is indicated consider- nected in parallel must be same. This is an essential
ing the HV winding phase as hour hand in the position 12 condition for paralleling three phase transformers.
of a clock and getting the relative position of the LV   6.  Relative phase displacement between the second-
winding after coinciding it with the minute hand. Thus ary line voltages of two transformers to be con-
depending upon phase shift of the minute hand, the posi- nected in parallel must be zero.
tion 1 for 30° lag and position 11 for 30° lead can be
indicated. Similarly the 0° and 180° shift is indicated
by numerals 0 and 6 respectively. The same is shown in 4.1.4  Per Unit System
as shown in Fig. 4.17. This method is known as vector
grouping. In a typical power system, different power equipments
connected together have different voltage and/or power
12 12 levels. Step up or step down transformers are used as
11 1 11 1 connecting units. Hence there is a need for conversion of
10 A 2 10 A 2 all the quantities into normalised values to avoid confu-
a a sion and errors. Per unit (pu) quantity is defined as
9 N n 3 9 n N 3
Actual value of the quantity in any unit
30° lag 30° lead Base value of the qunatity in the same unit
8 4 8 4
7 5 7 5 The base quantities can be specified either from high
6 6 voltage or from low voltage side for which the pu values
will be same for both the sides. The base values can be
12 12 defined as:
11 1 11 1   1.  VAbase or the three-phase apparent power common
10 A 10 A for both the sides.
2 2
a   2.  Base voltage (Vbase) or the line-to-line base volt-
9 9 n N age and it is dependent on the transformer turn
N n 3 3
180° shift ratios.
0° shift a   3.  Base current can be defined as,
8 4 8 4
Pbase
7 5 7 5 I base =
6 6 3 × Vbase
Figure 4.17 |   Clock convention representing vector
groups.   4.  Base impedance can be defined as

The angle between two consecutive numbers on the (Vbase )2


Zbase =
clock is 30°. Pbase

Chapter 4.indd 280 3/23/2016 11:25:13 AM


4.2  DC MACHINES     281

From the above equations, impedance can be laminated. Sometimes pole body and pole shoe are
defined based on old pu values as formed from the same laminations. The pole shoes
are shaped so as to have a somewhat augmented
æ Vbase_old ö Pbase_new
2
air gap at the tips.
Z2 = Z1 × ç ÷
ç Vbase_new ÷ Pbase_old   3.  Field windings: In the case of wound field, the
è ø field winding takes the form of a concentric coil
wound around the main poles. These are absent
4.2  DC MACHINES in permanent magnet DC machines. These carry
the excitation current and produce the main field.
Thus the poles are created electromagnetically.
DC machines are the electromechanical energy conversion The resistance of field winding is larger than the
devices. This machine works after getting input power armature winding resistance.
from a DC source and generates mechanical power or con-   4.  Armature: The armature is where the moving
verts mechanical power from any prime mover into DC conductors are placed and its purpose is to rotate
power. A general view of DC machine is given in Fig. 4.18. conductors in uniform magnetic field. The arma-
ture is constructed by stacking laminations of sheet
Rotation silicon steel. Thickness of these laminations is kept
axis low to reduce the eddy current losses. Open paral-
lel sided equally spaced slots are normally punched
Coil
in the rotor laminations. These slots house the
Magnet
armature windings over them. The armature wind-
ing can be of two types,
S
N (a) Lap winding: In this type of winding, the armature
conductors are so connected that they are divided
Magnet Brush Brush into as many parallel paths as the number of field
poles of the DC machine. Thus, if a machine has
p poles and Z armature conductors, then there will
be a parallel paths, each path having Z/p number
of conductors. The lap winding is used for high
Commutator
current low voltage applications.
(b) Wave winding: In this type of armature wind-
ing, the armature conductors are so connected

Figure 4.18 |   General DC machine.


that they are electrically divided into only two
parallel paths between the positive and nega-
A DC machine consists mainly of two parts: the station- tive brushes, irrespective of the number of field
ary part called stator and the rotating part called rotor. poles of the machine. Thus, Z armature con-
The rotor consists of an armature, a cylindrical metallic ductors are divided into two parallel paths,
core with slots in it to place armature windings or bars, each path having Z/2 conductors. The wave
a commutator and brush assembly. winding is used for high voltage low current
applications.
The stator consists of main field poles used to produce
magnetic flux and commutating poles or interpoles in   5. Commutator: It is the key element which makes
between the main poles to avoid sparking at the com- the DC machines of the present day possible. It is a
mutator. In the case of small machines, sometimes the rotating switch and is placed between armature and
interpoles are avoided. In addition to these, the stator the external circuit such that reverse the connection
has a frame or a yoke which forms the supporting struc- with the external circuit at the instant when current
ture of the machine. is reversed in the armature. It consists of copper seg-
ments tightly fixed with mica insulating separators on
The major parts of a DC machine are described follows: an insulated support. The whole commutator forms
  1.  Stator frame: Frame is the stationary part of a a rigid and solid assembly of insulated copper strips.
machine on which the main poles and commutator   6. Brushes: These are contact on the surface of the
poles are fixed. The portion of the frame which con- commutator as a part of the brush assembly. Usually
stitutes the magnetic path for the magnetic fluxes graphite is used as brush material but the actual
from the main poles and interpoles is called yoke. composition of the brush depends on the peripheral
  2.  Main poles: Poles made of fabricated steel with speed of the commutator and the working voltage.
separate or integral pole shoes are fixed firmly to Their function is to collect current from the commu-
the frame by means of bolts. Pole shoes are usually tator and supply to the external load circuit.

Chapter 4.indd 281 3/23/2016 11:25:15 AM


282     Chapter 4:  Electrical machines 

4.2.1  Commutation Therefore, from Eqs. (4.10) and (4.11) the total induced
emf is,
The induced currents in the armature of a DC generator
E = Bavg × L × v
Z
are alternating in character. The currents flow in one Volts  (4.12)
a
direction when the conductors are under the north pole
and exactly opposite direction when conductors under But v = wD/2 where w is the angular velocity of the
the south pole. machine and D is the diameter.
f
The process by which current in the short-circuited Also Bavg can
= be written
Tesla in terms of pole pitch Y, core
armature coil is reversed while it crosses the brush axis, YLper pole f as
length L and flux
also called magnetic neutral axis, is called commutation.
f
This commutation is achieved by the commutator. If Bavg = Tesla  (4.13)
commutation is improper, then sparking occurs between YL
the brush and commutator which damages both. here Y = pD/p, where p is the number of poles. (4.14)
The self—induced emf in the armature coil undergoing Combining Eqs. (4.12)−(4.14), we get
commutation is called reactance voltage and its value is
given by pfwZ
E=  (4.15)
2pa
2I
1.11L The general form of Eq. (4.15) is
TC
E = Kafw  (4.16)
where TC is the commutation time.

Commutation can be improved by: pZ


where Ka = emf constant = Volts/Wb/rad/s
2pa
  1.  Using interpole, which are small poles fixed to the
yoke in between the main field poles. The number of parallel paths is dependent on the type
  2.  Replacing low resistance copper brushes with higher of the winding chosen. The number of parallel paths are
resistance carbon brushes in the brush assembly. a = 2 for wave winding and a = p for lap winding.
  3.  By an arrangement that neutralises the reactance
voltage by producing a reversing emf in the short-
circuited coil that is under commutation. 4.2.2.1 Torque Production

When armature conductors are carrying currents, they


4.2.2  EMF Induction interact with the main field and experience force on
them. This force effectively results in torque production
The flux density waveform for the DC machine as shown
in the machine. The electrical power generated by an
in Fig. 4.18 is rectangular in nature. A rectangular flux
armature coil having induced emf E and delivering a
density waveform has the advantage that the induced
current of Ia to the load is given by EIa Watts. Now,
emf in armature coils becomes constant and equal to
equating the mechanical and electrical power, we have
each other. The induced emf in one conductor of the
machine can be given by wT = EIa  (4.17)

Ec = Bavg × L × v Volts  (4.10) where  T is the torque (Nm). Substituting for  E from

Eq. (4.16) in Eq. (4.17), we have
where, Bavg is average flux density over a pole in a slot pitch
in Tesla, L is the length of the `active’ conductor in m and v T = KafIa  (4.18)
is the linear velocity of armature conductor in m/s.
If the total number of conductors in the armature coil Equation (4.17) shows that the generated torque is not a
be Z and the number of parallel paths is `a’ between the function of speed but it is proportional to the total flux
brushes by virtue of their connections, then number of and the armature current Ia.
series conductors is Z/a.
Thus the total induced emf between the brushes is 4.2.2.2 Armature reaction
given by
The effect of the armature flux on the main field flux
Z is called armature reaction. Armature reaction in a DC
E = Ec  (4.11)
a machine is illustrated in the Fig. 4.19.

Chapter 4.indd 282 3/23/2016 11:25:27 AM


4.2  DC MACHINES     283

GNA MNA of main field flux as shown in Fig 4.19(c). This effect is
called as armature reaction.
+ The effects of armature reaction can be listed as follows:
  1.  It deteriorates the main flux. In case of generator,
weakening of the main flux reduces the generated
N + S voltage.
  2.  It distorts the main field flux; hence the position
of MNA gets shifted. Brushes should be placed
+
on MNA; otherwise it will lead to sparking on the
brush surface.
(a)
4.2.3  Type of Connections
GNA MNA
According to the connection of the armature and the
+ field windings, the DC generators are divided into differ-
ent types. The first two categories are and.
  1.  Separately excited type: When the field is con-
N + S nected to a DC supply separately from armature
and the armature circuit is also independently con-
nected to a DC source it is known as separately
+ excited generator.
  2.  Self-excited type: When the field is connected in
series or parallel to the armature and the combina-
(b) tion is connected to a DC supply, the machine is
GNA MNA called self-excited DC generator. There are various
sub-divisions of this type:
(a) When the field is connected in series with the
+ armature the machine is called DC series machine.
(b) When the field is connected in parallel with the
armature the machine is called shunt machine.
N + S (c) When both the series and the shunt field is pres-
ent with the shunt field dominating in nature,
it is called compound type DC generator. There
+ are two categories of compound machines:
(i) Long shunt: When the shunt field is con-
nected after the series field at one end.
(c) (ii) Short shunt: When the series field is con-
Figure 4.19 |   Armature reaction in DC machine.
nected directly in parallel with the arma-
(a) Main field flux. (b) Armature flux. ture and then the series field is connected,
(c) Distortion of main field flux due to it is known as short shunt machine.
armature flux. On the other hand according to the magnetic cir-
cuit the compound machines are also divided into
With no current in the armature, the magnetic flux two categories independent of the physical con-
lines due to field poles are uniform and symmetrical nection of long or short shunt type. These are:
to polar axis as shown in the Fig. 4.19(a). Under this
condition, the magnetic neutral axis (MNA) coincides (i) Cumulatively compound: When the series
with the geometric neutral axis (GNA). Magnetic neu- and the shunt field are helping, that is,
tral axis may be defined as the axis along which no emf when the flux produced by them are in the
is produced, as the armature conductors moves parallel same direction.
to the field lines. Figure 4.19(b) shows the armature (ii) Differentially compound: When the two fields
flux lines produced due to armature current. are in opposition, then the compound machine
When the machine is running, both the main field and is called differentially compound machine.
armature fluxes will be present. The armature flux works The different types of DC machines based on types of
upon the main field flux, hence disturbing the flux lines connections are depicted in Fig. 4.20.

Chapter 4.indd 283 3/23/2016 11:25:31 AM


284     Chapter 4:  Electrical machines 

Separately excited
+
+ Self excited



Series wound Shunt wound Compound wound
+
+ +

− −

Long shunt Short shunt

+ +

− −

Figure 4.20 |   Classification of DC machines.

4.2.4  DC Generator Characteristics


Ia = 0
4.2.4.1 Open-Circuit Characteristics
Prime A2
The open circuit characteristics of a DC machine are also mover #
called no-load characteristics. These can be obtained by
running the generator at rated speed after connecting it A1
as separately excited machine and measuring the termi-
nal voltage with open circuited armature at varying field +
current. Figure 4.21 shows a shunt generator connected F 2 If
as separately excited machine with its field connected to VDC
a voltage source through a regulating resistor in poten-
tial divider form to vary the field current. The current
drawn by the field winding can be varied from zero to a − F1

value corresponding to 125% of armature rated voltage. (a)


For each field current the generated armature voltage is
measured. The load current is kept zero as the armature
Saturation zone
is open circuited. The speed of the generator is held at a
constant value which is usually the rated speed. The plot
Induced emf

of the induced emf as a function of the excitation cur- Knee point


rent is known as the open circuit characteristic (occ), or
no-load magnetization curve or no-load saturation char-
acteristic of the machine. Figure 4.21(b) shows a typical
open circuit characteristic of the machine. The charac- Unsaturated
teristic exhibits saturation at larger values of excitation If
current but is linear in the low excitation current zone
Eo = emf due to residual magnetism
due to presence of air-gap in the magnetic circuit. Due to
the residual magnetism exhibited by the iron in the mag- 0
Exciting current
netic structure, the induced emf does not become zero (b)
when the excitation current is reduced to zero, which can
also be observed from Fig. 4.21(b). This residual voltage Figure 4.21 |   DC generator. (a) Circuit diagram.
is about 2−5% of the rated voltage in modern machines. (b) Open circuit characteristics.

Chapter 4.indd 284 3/23/2016 11:25:33 AM


4.2  DC MACHINES     285

4.2.4.2 Load Characteristics 1. There must be some residual magnetism in


the generator field poles.
The load characteristics of a DC machine are known as 2. The direction of field windings should be
the external characteristics. These represent the varia- proper, that is, the field current should assist
tion of armature voltage with its load current at a con- the residual magnetism.
stant speed and constant excitation current. The speed 3. For a shunt generator, the value of resistance
and excitation current are usually kept at rated value. of the field windings should be less than the
The terminal voltage V can be expressed as, critical value of field resistance. The value of
load circuit resistance should be greater than
V = E − Ia Ra − V b  (4.19) the critical value of load resistance.

4. The speed of the machine must be greater
where E is the induced emf, Vb is brush contact drop than critical speed for a shunt generator.
(in Volts) and Ia is the armature current (in Amperes). 5. For a series wound DC generator, for emf to
Here Ra can be assumed to be the sum of armature resis- develop (or current to flow through field wind-
tance, interpole winding resistance, series winding resis- ing) an electrical load must be connected across
tance and compensating winding resistance. But in usual the generator. The external circuit resistance
practice, all other resistances except the armature resis- should be less than the critical resistance.
tance are assumed to be zero. The circuit diagram and
the load characteristics of DC generators are depicted in 4.2.5  Parallel Operations of DC Generators
Figs. 4.22(a) and (b), respectively.
The DC generators are connected in parallel through
IL bus bars. The procedure for paralleling two DC shunt
S2 generators is represented in Fig. 4.23 and the steps are
If listed as follows:
F2
A2 S1   1.  The prime mover of generator 2 is brought up to
Load
Prime rated speed, the field switch S4 is closed.
Vf #   2.  Circuit breaker CB2 is closed and excitation of
mover
second generator is adjusted such that it produces
F1 A1 rated bus bar voltage.
  3.  Now switch S3 is closed thus both generators are
(a)
connected in parallel.
  4.  The second generator to deliver any load will have
Over compounded to generate more voltage than the bus bar volt-
Flat compounded age current supplied by it, that is, I = (E−V)/Ra
where Ra is the armature resistance.
Output voltage

  5.  By adjusting the field excitation, the generated


Under compounded voltage can be changed.

Shunt generator Feeder

Differential compounded +
V Bus-bars
Full load −

CB1 CB2
0 A1 A2
Load current S3
(b) S1

Figure 4.22. |   DC generator. (a) Circuit diagram.


V2
(b) Load characteristics.

Gen.1 Gen.2
4.2.4.3 Conditions for Self-Excitation S2 S4
The essential conditions for voltage build-up in a self-
excited DC generator are: Figure 4.23 |   Shunt generators in parallel.

Chapter 4.indd 285 3/23/2016 11:25:38 AM


286     Chapter 4:  Electrical machines 

Under-compounded generators also operate effectively in The characteristics plots of torque vs. armature cur-
parallel. Over-compounded generators will not operate rent; speed vs. armature current and speed vs. torque for
well unless their series fields are made parallel. This is DC shunt motor are depicted in Fig. 4.24(a), (b) and (c),
attained by connecting two negative brushes together. respectively. The torque varies nearly linearly with the
This is achieved through equaliser bars. armature current and drops slightly due to the effect of
armature reaction as shown in Fig. 4.24(a). The speed
remains nearly constant with increase in current and also
4.2.6  DC Motors
drops down slightly due to armature reaction as shown
in Fig. 4.24(b). Superimposing these two relationships
4.2.6.1 Characteristics of DC Motors
we get, the torque vs. speed characteristic plot shown in
Fig. 4.24(c).
DC Shunt Motor Characteristics
The characteristics for the shunt motors include torque- Te
Tsh
current characteristics, speed-current characteristics
and speed-torque characteristics.
Torque-current relationship for the DC shunt motor Torque (Te)
it is given by

Te = KafIa  (4.20) 0
Ia

where Te is the electromagnetic torque. (a)


Thus the torque-current characteristic curve is a straight
line. But when armature reaction is considered, the flux
falls slightly due to demagnetising effect of armature
reaction which is represented by the dotted line that Speed (N)
should be considered for the characteristics.
The voltage equation for the DC shunt machine is
given by 0
Ia
V = E + Ia Ra  (4.21) (b)

Using E = Kafw in Eq. (4.21), we get V = Kafw + Ia Ra


 (4.22)
Speed (N)
Rearranging Eq. (4.22), we have

(V − Ia Ra )
w =
0
 (4.23)
Ka f Te
(c)
Equation (4.23) provides the speed current characteris- Figure 4.24 |   Shunt motors: (a) Torque versus
tics for the machine. armature current, (b) speed versus
Also combining (4.20) and (4.23), we get armature current and (c) speed versus
torque.

V TR
w = − e a2  DC Series Motor Characteristics
Ka f ( K f ) (4.24)
a The torque-current relationship for the DC series motor
can be given by,
Thus Eq. (4.24) provides the torque-speed characteris-
tics for the machine. Te = KafIa (4.25)

Chapter 4.indd 286 3/23/2016 11:25:49 AM


4.2  DC MACHINES     287

Since f ∝Ia the above Eq. (4.25) can be modified as, Ta Tsh

Te = Ka ′ Ia 2  (4.26)
Torque

Thus the torque-current characteristic curve is parabolic.


But when armature reaction is considered, the flux will 0
not be exactly proportional to armature current making Ia
the characteristic curve slightly straight. (a)
The voltage equation for the DC series machine is
given by

V = E + I a (R a + R f )  (4.27)
Speed (N)

Using E = Kafw in the Eq. (4.26), we have


0
(V − Ia Re )
Ia
w =  (4.28)
Ka f (b)

where Re = R a + R f
Speed (N)
Since f µIa the Eq. (4.28) can be modified as,
0
Ta
V R
w = − e (4.29) (c)
Ka ′ I a K a
Figure 4.25 |   Series motors: (a) Torque versus

armature current, (b) speed versus
Thus the speed-current characteristic curve is a rectan- armature current and (c) speed
gular hyperbola. versus torque.

Substituting for Ia in Eq. (4.29) from Eq. (4.26)


DC Compound Motor Characteristics
V Re
w = −  (4.30) Compound motors have both series field and shunt field
Ka ′ Te Ka on them. They may be classified as,

1. Cumulative-compound motors in which series
Equation (4.30) provides the torque-speed charac- field helps the shunt field.
teristics for the machine which is also a rectangular 2. Differential-compound motors in which series
hyperbola. field opposes the shunt field.
The characteristics plots of torque vs. armature cur- The circuit and characteristics of cumulative compound
rent; speed vs. armature current and speed vs. torque for motors is depicted in Figs. 4.26(a), (b) (c) and (d), respec-
DC series motor are depicted in Fig. 4.25(a), (b) and (c), tively. In the torque vs. armature current characteristics,
respectively. The torque vs. armature current plot is a as the load increases, the total flux increases and Ta pro-
parabola as shown in Fig. 4.25(a). The magnetic satura- portional to fIa as shown in Fig. 4.26(b). In the speed vs.
tion changes this characteristic from parabola to nearly a armature current characteristics, as load increases, flux
straight line. The speed vs. armature current plot is r as per pole also increases. The speed is inversely proportional
shown in Fig. 4.25(b). This characteristics is more or less to flux per pole. So as load increases, flux will increase and
like a rectangular hyperbola. Superimposing these two speed decreases as shown in Fig. 4.26(c). The speed vs.
relationships we get, the torque vs. speed characteristic torque characteristics can be obtained by superimposing
plot shown in Fig. 4.25(c). the above two characteristics as shown in Fig. 4.26(d).

Chapter 4.indd 287 3/23/2016 11:26:01 AM


288     Chapter 4:  Electrical machines 

Ish IL + V − Ia Ra
N =K rpm  (4.31)
Series f
field
Ia In case of series motor Ra is replaced by Ra + Rse
Shunt V From the above Eq. (4.31), the methods of controlling
Ra the speed of a DC motor are,
  1.  By varying the armature voltage, speed can be
controlled
−   2.  By varying the field flux per pole, speed can be
(a) controlled above the base speed and
  3.  By varying the applied voltage to the motor.

nd
Speed Control of Shunt Motor
u
Ta po   1.  Armature voltage control method: By vary-
m
Co Shunt ing the voltage across the armature winding of a
DC motor, the back emf Eb and hence the speed of
the motor N can be changed. This is usually done
0 Ia by inserting a variable resistance in series with the
(b) armature to vary the armature resistance as shown
in Fig. 4.27.

N0 Ish IL
+
Ia
N Rc
V
Ra
Rsh
0 Ia
(c)

Figure 4.27 |   Circuit for armature voltage control.

We have
N
N µ V - Ia (Ra + Rc )

Ta Due to voltage drop in the added resistance Rc, the


0
back emf is decreased and since N ∝ Eb, the speed of
(d) the motor will be reduced. The speed varies widely
Figure 4.26 |   Compound (Cumulative) motor
with load and the output and efficiency of the motor
characteristics. (a) Torque vs. armature are reduced in this method.
current, (b) speed vs. armature current   2.  Field current control method: By varying the
and (c) speed vs. torque characteristics. field flux per pole (f), the motor speed (N ∝ 1/f)
can be changed. A variable resistance is inserted
in series with the shunt field as shown in Fig. 4.28.
4.2.6.2 Speed Control of DC Motors This resistance is varied to vary the field current
and hence the f. Field cannot be strengthened
The speed of a DC motor is given by beyond its limit and it can only be reduced to
increase the speed. This by this method, speed
Eb
N = above the base speeds can be obtained.
f

Chapter 4.indd 288 3/23/2016 11:26:12 AM


4.2  DC MACHINES     289

Ish IL Diverter
+
Ia
+
Rse
Ra V
Ia V
Rsh


− (a)
Field
rheostat Rse
+
Figure 4.28 |   Circuit for field current control. S

Ia V
  3.  Voltage control method: In DC motors, the
voltage source supplying the field is different from
the one which supplies the armature. This method
circumvents the disadvantages of poor speed reg- −
(b)
ulation and low efficiency as in armature control.
However, it is quite costly method. Therefore, Figure 4.30 |   Field flux control method: (a) Field
this method of speed control is employed for diverter resistance flux control and
high rated motors where efficiency is of great (b) tapped field flux control.
significance.
Sometimes, the field coils are paralleled by regrouping to
Speed Control of Series Motor control the speed (Fig. 4.31). By doing this several fixed
speed settings can be achieved.
The DC series motor speed control like the shunt motor
can be controlled by (i) armature-resistance control
method and (ii) field flux control method.
  1.  Armature-resistance control: A variable resis- Ia
tance is connected in series with the supply to
reduce the available voltage and hence to reduce
the speed of the motor as shown in Fig. 4.29. By
changing the value of this resistance, any speed (a)
below the normal speed can be obtained.

Rse
+
Ia
Ia V

(b)

Figure 4.29 |   Circuit for armature resistance control.

  2.  Field flux control method: The flux pro-


Ia Diverter
duced by the field winding in series is varied in
this method using field diverter resistance or by
tapping the series field to vary the field flux as
shown in Fig. 4.30(a) and (b), respectively. These (c)
methods weaken the field and increase the speed
as N ∝ 1/f. Figure 4.31 |   Paralleling the field coils.

Chapter 4.indd 289 3/23/2016 11:26:19 AM


290     Chapter 4:  Electrical machines 

4.2.7  Power Stages in DC Generator and Motor 4.2.8 Testing of DC Machines

4.2.7.1 Power Stages in DC Generator 4.2.8.1 Swinburne’s Test

Figure 4.32 represents the power stages in DC generator. The DC machine is run as a motor at no-load and the
Copper losses are given by (B−C) and friction losses by losses of the machine are determined using this test.
(A−C). Once the losses of the machine are identified, its effi-
ciency at any desired load can be determined. This test
A B C is applicable for shunt and compound DC machines.
The armature resistance and the shunt field resistance
Electric of the DC motor are measured using a battery, a voltme-
Mechanical Iron and power Electric ter and an ammeter.
Cu
power friction developed power For determination of constant losses, the machine is
losses
input losses in arma- output run as a motor on no-load at rated supply voltage. Speed
ture is adjusted to its rated value.
(VIL)
(EgIa) Constant losses, Pc = Input to motor −Armature Cu

Figure 4.32 |   Power stages in DC generator.


loss, that is,
Pc = VI 0 − (I 0 − I sh )2 Ra
The machine efficiency can also be determined using this test.
4.2.7.2 Power Stages in DC Motor

Figure 4.33 represents the power stages in DC generator. 4.2.8.2 Regenerative or Hopkinson’s Test
Copper losses are given by (A−B) and friction losses by
(B−C). This method of determining the efficiency of a DC machine
saves power. This also gives more accurate results. Two
A B C identical DC shunt machines are required for this test.
These are coupled mechanically and connected in paral-
lel across the supply as shown in Fig. 4.34. By adjusting
the field excitation of both the machines, one is run as
Input Mech. power Iron and Output a motor and the other run as a generator. The electric
Cu
developed friction power power from the generator and the supply are fed to the
losses
in armature losses of motor motor. The electric power supplied to the motor is con-
verted into mechanical power and to the motor losses.
VIL watts EbIa watts This mechanical power is supplied back to the generator.
Thus the electrical power taken from the DC supply is
Figure 4.33 |   Power stages of DC motor. the sum of motor and generator losses. This power can
be measured directly by a voltmeter and an ammeter.
Since the power input from the DC supply is equal to the
4.2.7.3 Efficiency of DC Machines power required for supplying the machines’ losses, this
test can be carried out with a small quantity of power.
Efficiency is given by
I2 I1 S I1
Input − losses +
Efficiency = ,
Input
Output (I1 + I2) V1
Efficiency = , I3
Output + losses I4 I1 + I3
For generator, V

hg =
Output VI
=  (4.32)
Output + losses VI + (I + I sh )2 Ra + Pc M G

For motor,

I2 I1
Input − losses VI − (I − I sh )2 Ra + Pc
hm
Figure 4.34 |   Circuit for carrying out Hopkinson’s test.
= =  (4.33)
Input VI

Chapter 4.indd 290 3/23/2016 11:26:28 AM


4.3  THREE PHASE INDUCTION MACHINES     291

4.2.9 Electrical Braking 4.2.9.3 Regenerative Braking

Following three methods of braking are used for DC In this braking mode, the motor is run as generator. This
shunt and series motors, result in the kinetic energy of motor is converted into elec-
  1.  Rheostatic or Dynamic braking trical energy and returned to supply end. The field is dis-
  2.  Plugging connected from the supply and field current is increased
  3.  Regenerative braking from an external source. Induced emf exceeds the supply
voltage (Fig. 4.37). Thus a braking torque is produced.

4.2.9.1 Rheostatic or Dynamic braking +


I
In this method of braking a DC motor, the armature of
the running motor is disconnected from the supply end
and is connected across a variable resistance with the If E V
field winding left connected to the supply (Fig. 4.35).
The armature rotates under the influence of the field
behaving as a generator, sending high current through
the resistor. This causes the energy to dissipate as heat

while stopping the motor. E>V

Figure 4.37 |   Regenerative braking of DC shunt motor.


Ish
+
Ia 4.3  THREE PHASE INDUCTION
MACHINES
R
V
Ra Eb
The induction machines have the maximum share among all
the industrial machines. However, they can be used both as
a motor or a generator. It is the workhorse of modern indus-
− try. Majority of the motors used by industry are squirrel
Figure 4.35 |   Dynamic or Rheostatic braking.
cage rotor type. Both three-phase and single-phase motors
are widely used. The induction generators are seldom used.
Their typical application is in the wind power plants.
4.2.9.2 Plugging The induction motor essentially consists of two parts:
  1.  Stator, which is the stationary part.
The connections to the armature winding are reversed in   2.  Rotor, which is the revolving part.
this method of braking so that the motor tends to rotate
in the opposite direction, thus giving the necessary brak- The overview and cross-section of an induction motor is
ing effect (Fig. 4.36). When the motor slows down even- illustrated in Figs. 4.38(a) and (b), respectively.
tually, the supply must be stopped otherwise the motor
will start rotating in the opposite direction.
Stator

Rotor
+ V −
Ish
+ V −

Ish

Ra Ra Ia
Ia
Eb Eb (a)
R
Figure 4.38 |  Construction of induction machine:
Figure 4.36 |   Motoring and plugging mode. (a) overview and (b) cross-sectional view.

Chapter 4.indd 291 3/23/2016 11:26:33 AM


292     Chapter 4:  Electrical machines 

to three way connected resistances (connected in


Stator
star). Thus the machines are also called as slip ring
type machines.

Three
Wound rotor
slip rings
Shaft

Rotor
Figure 4.40 |   Wound rotor.

4.3.1  Rotating Magnetic Field


(b)
The induction machine operates on the basis of produc-
Figure 4.38 |   (Continued). tion of torque produced by the interaction of the stator
and the rotor magnetic field. The principle of operation
The construction of the stator and rotor are described as of an induction machine is based on the generation of
follows. As per construction of the rotor, the induction a rotating magnetic field produced by the stator wind-
machines are of two types − squirrel cage and wound ings when a balanced three phase voltage is impressed
rotor type. on it.

  1.  Stator construction: The stator of an induc- Considering a cosine waveform of stator current in
tion motor is cylindrical in nature. Laminated iron each phase with frequency f  when a balanced three phase
core with slots form the core with armature coils voltage is applied to three phases of stator winding, the
placed in the slots forming three or single phase three phase currents are given by,
winding.
  2.  Squirrel-cage rotor construction: Rotor has ia = I m cos 2pft  (4.34)

laminated iron core with slots. Metal (copper) bars
are molded in the slots to form cage rotor winding. For the b and c phases, the current equations are
Two copper end rings short circuit the bars. The
bars are skewed to reduce noise and improve start- ib = I m cos(2p ft − 120°)  (4.35)

ing performance. Fig. 4.39 shows the basic features
ic = I m cos(2p ft − 240°) 
of cage rotor.
and (4.36)

Slightly skewed
where Im is the maximum value of the stator current in
rotor bars
each phase.
If Fr is the resultant mmf of the three phases, it
becomes a rotating wave in space and can be derived as
End ring
Fr (t) = Fa cos wt cosq + Fb cos(wt − 2p /3) cos(q − 2p /3)

Figure 4.39 |   Cage rotor. +Fc cos(wt + 2p /3)cos(q + 2p /3)

  3. Wound-rotor construction: Figure 4.40 shows or Fr (t) = 3Fa / 2cos(wt − q )  (4.37)


the rotor of a large wound-rotor motor. In this case
the rotor is wound with winding similar to stator where Fa= Fb = Fc = NIm.
winding. The other three ends of the rotor winding Thus, Fr(t) is a rotating wave with angular velocity of w.
are terminated at three copper end rings known as The graphical representation of rotating field phasor is
slip rings. Three brushes contact the three ­slip-rings depicted in Fig. 4.41.

Chapter 4.indd 292 3/23/2016 11:26:43 AM


4.3  THREE PHASE INDUCTION MACHINES     293

I Therefore the rotor leakage is written as sXlr in the equiv-


Ia Ib Ic alent circuit. Thus rotor current can be expressed as,
sE1
Ir =  (4.38)
Rr2 + (sXlr2 )

The above expression can be modified as,
I1 I2 I3
E1
Ir =  (4.39)
Rr2
2
+ (Xlr2 )
q a F
r
a a s
b′ c′ b′ c′ b′ c′ It can be observed from Eq. (4.39) that the rotor current
q2
Fc Fc
Fb Fa can also be obtained from the stator emf E1 at supply
q3
Fa Fr Fr = frequency f1 when the rotor impedance consists of a resis-
Fb Fa Fb resultant
Fc Fb tance Rr/s and inductive reactance Xlr. This rotor ­current
c b c b c b mmf is also at the frequency of E1 (i.e. stator frequency) and
a′ a′ Fr a′ the term slip no longer gets multiplied with the leakage
reactance. Here it is assumed that the rotor number of
Figure 4.41 |   Graphical view of the rotating field phasors. conductors is same as that of stator. Hence, the trans-
former shown in the equivalent circuit of Fig. 4.42 can
also be removed. Now all the rotor side parameters can
4.3.2  Equivalent Circuit
be referred to the stator side after knowing the stator to
The equivalent circuit of the machine is required to rotor turns ratio. The same can be represented as shown
make quantitative predictions about the behaviour of the in Fig. 4.43(a) which can be further simplified as in
machine, under various operating conditions. It is gener- Fig.  4.43(b) following the same assumptions shown in
ally required to represent the machine under sinusoidal transformer equivalent circuit. The approximate equiv-
steady state operating conditions. For a balanced machine alent circuit neglecting the resistance of magnetising
where all the three phases are identical, a single-phase branch (Rm) is given in Fig. 4.43(c).
or per phase equivalent circuit is sufficient for most pur- Rr′
poses. In order to calculate the machine performance Rs Xls s Xlr′
parameters, this circuit is sufficient where as to calculate
the quantities like output power, torque, etc. the power
or torque calculated in the circuit can be multiplied by Vph Rm Xm
three to obtain the total power or torque of the machine.
The equivalent circuit considering different machine
(a)
parameters can be drawn as shown in Fig. 4.42. (Here
the prime sign indicates that the quantities are referred Rr′
Rs Xls s Xlr′
to the primary side.)

Rs Xls Rr¢ sXlr¢ Vph Rm Xm

Rm Xm E sE1 (b)
1

Rr′
Rs Xls s Xlr′
Figure 4.42 |   Equivalent circuit: Stator and rotor.
Vph Xm
In the equivalent circuit shown in Fig. 4.42, the stator and
the rotor circuits are shown independently. The rotor side
parameters Rr and Xlr are the rotor resistance and leakage
(c)
resistance. The rotor side loop is excited by a voltage sE1,
which is at a frequency slip times the stator side frequency Figure 4.43 |   The equivalent circuit. (a) Exact.
f1 or sf1. The term slip can be defined as the deviation (b) Approximate. (c) Approximate
in rotor speed from synchronous speed in per unit value. equivalent circuit neglecting Rm.

Chapter 4.indd 293 3/23/2016 11:26:51 AM


294     Chapter 4:  Electrical machines 

The phasor diagram for induction motor under load is losses. This is because the no-load current of the induc-
shown in Fig. 4.44. tion machine is less than 40% of full load current and
thus the stator copper loss can be small and since there
V1 is negligible torque. The rotor current is also very small
I1X1
thereby reducing the rotor copper losses to a very small
value and the mechanical rotational losses are always
I1R1 much smaller than these losses. The equivalent circuit of
90°
I1Z1 the machine referring to stator side at no load is shown
in Fig. 4.45. In this circuit the rotor branch is opened
−E1
as the rotor current is negligible at no-load because the
machine speed reaches near synchronous speed due to
r2r I1 negligible developed torque and Rr/s becomes very large.

f1
Rs Xls

Ioc

Ic Voc
0 f Rm Xm
90° Im
I 2rR2
I0
f2r Ioc = No load
90° current

E2r = I2rZ2r Figure 4.45 |   Per phase no-load equivalent condition.

I2r I2rX2
Calculation of Rc and Xm
E1 From no load test neglecting the stator impedance, the
input power Pnl recorded by the wattmeter is only the core
Figure 4.44 |   Induction motor phasor diagram losses. Thus if input voltage Vnl and current Inl is known,
under load.
Pnl = Vnl I nl cosfnl
Pnl
cosfnl =
4.3.3  Determination of Circuit Parameters or
Vnl I nl
In order to determine the various machine performance
parameters, knowledge about the values of the various where cosfnl is the no load power factor of the machine.
elements of the equivalent circuit already developed is Thus from the equivalent circuit,
necessary. The equivalent circuit parameters are divided Vnl
in two categories like in the case of the transformer. Rc = cosfnl
I nl
These are, series parameters and the shunt parameters.
and
The series parameters are the rotor and stator resistances
Vnl
and leakage inductances whereas the shunt parameters Xm = sinfnl
are the core loss resistance and the magnetising induc- I nl
tance. Different tests are required to determine the two With all the readings known for Vnl, Inl and Pnl, Rc and
categories of the machine parameters. Xm can be determined.

4.3.3.1 Determination of the Shunt 4.3.3.2 Determination of Series Parameters


Parameters
In order to determine the series parameters the blocked
In order to determine the shunt parameters the no-load rotor test on the machine is conducted. In this test the
test on the machine is conducted. machine rotor is prevented from rotation input volt-
In this test the machine is run at no load at rated volt- age to the machine is controlled to flow rated current
age and the readings of input power, input current and in it and the readings of input power, input current
voltages are taken for calculation of the machine shunt and voltages are taken for calculation of the machine
parameters. At no-load, the machine torque developed series parameters. In the blocked rotor condition the
by it is very small and the machine losses are mainly core slip remains at unity and the elements representing the

Chapter 4.indd 294 3/23/2016 11:26:56 AM


4.3  THREE PHASE INDUCTION MACHINES     295

magnetising branch Rm and Xm compared to Rr¢ and Xlr¢ Similarly if the stator resistance R1 is measured by an
ohmmeter, the rotor resistance can be determined as,
in the equivalent circuit. Thus, in the equivalent cir-
cuit, the magnetizing branch may be neglected and the R2 = Rbr − R1
reduced machine equivalent circuits is given in Fig. 4.46.
In some cases where stator resistance measurement is
Rs Xls R′ Xlr′ not available, assuming R1 = R2, we have
Isc 1
R1 = R2 = (R )
2 br
Vsc
With all the readings known for Vbr, Ibr and Pbr, R1, R2
Isc = Rated
and X1, X2 can be determined
current
Figure 4.46 |   Per phase blocked rotor equivalent.
4.3.4  Power Flow Diagram of Induction
Machine
Calculation of R1, R2 and X1, X2
The induction machine input power can be divided into
From blocked rotor test equivalent circuit, the input power
different losses and after these the final output power
Pbl recorded by the wattmeter is only the rated copper
is available in the machine shaft and is known as shaft
losses. Thus if input voltage Vbr and current Ibr is known,
power or output power. The losses are:
P br = V br I brcosf br   1.  Stator copper loss (3Is2rs)
  2.  Stator core loss
P br   3.  Rotor core loss
cosf br =
V br I br   4.  Rotor friction and windage loss
or
where cosfbr is the blocked rotor power factor of the The rotor quantities are at slip frequency and hence the
machine. Also the input impedance under blocked rotor rotor core losses are usually neglected. The power flow
condition is given by, diagram can thus be obtained as illustrated in Fig. 4.47.
V br Rotor copper loss is given by sPg, where Pg is air-gap
Z br = power and s is slip. Thus
I br
Thus from the equivalent circuit, Pem = Pg − sPg = Pg (1 − s)

Rbr = Zbr cosf br


4.3.5  Complete Torque−Speed Characteristics
and
Xbr = Zbr sinf br The speed−torque characteristics of the machine can be
obtained from the per phase approximate equivalent cir-
Also, Xbr = X1 + X2 cuit represented in Fig. 4.43(b). For sinusoidal per phase
Assuming X1 = X2, these parameters can be determined voltage Vs impressed on the machine, the per phase rotor
current from the circuit is given by
as, 
 Vs
Ir =
æ R¢ ö
1
Rs + ç r ÷ + j(Xls + Xlr¢ )
X1 = X2 = (X )
2 br
è s ø

Input Pg (Pem) Psh or Po


power (Air-gap Shaft
Electro power or
power)
Stator Stator Rotor magnetic output
Rotor
developed
core copper copper friction power
power
loss loss loss and
(Pcu) windage loss
(PF & W)

Figure 4.47 |   Power flow diagram.

Chapter 4.indd 295 3/23/2016 11:27:11 AM


296     Chapter 4:  Electrical machines 

where Vs is the phase voltage phasor and Ir is the rotor cur- Thus from Eq. (4.40), we have
rent phasor. Since this current is flowing through the rotor
equivalent resistance Rr¢ s , the air-gap power is given by, é −R ¢ ìïæ R¢ ö ü ù
ê r íç Rs + r ÷ + Xe2 ïý ú
2

3Vs2 ê s2 ïîè s ø ïþ ú = 0
2 Rr¢ ê ú (4.41)
Pg = I r w s ê R¢ ì æ ¢ −R ¢ ü ú
R r öæ r ö
ê−
s
í2 ç Rs + ÷ ç 2 ÷ ýú
r 
Vs2 Rr¢ ë s î è s øè s ø þû
=
æ Rr¢ ö
2 s
ç Rs + s ÷ + j(Xls + Xlr¢ )
2 where Xe = Xls + Xlr. Solving Eq. (4.41) and representing
è ø the slip at maximum torque condition as smT, we get
The mechanical power output developed is given by (1−s) (Rr¢ )2 Rs Rr¢ R R¢ (R ¢ )2
Pg. Therefore, the developed torque can be obtained by Rs2 + +2 + Xe2 − 2 s r − 2 2r =0
dividing this mechanical power by the shaft speed wm,
2 smT smT
smT smT
which can be written as,
(Rr ¢)2
Pg (1 − s) Pg (1 − s) 2 Rr¢ Rs2 + Xe2 = 2
= = Ir smT
wm w s (1 − s) sw s
where ws is the synchronous speed of the machine in Rr¢
smT = ±  (4.42)
mechanical radians per second and s is the slip. Since Rs2 + Xe2
the torque produced is calculated on per phase basis, the
overall torque for three phases is given by The positive sign is for motor operation and the negative
Rr¢
sign is for generator operation. Usually the stator imped-
3 Vs2
Te =  (4.40) ance is neglected and the Eq. (4.42) for slip at maximum
ws æ Rr¢ ö
2 s
ç R s + s ÷ + (Xls + Xlr¢ )
2 torque is modified as
è ø Rr¢
smT = ±
Xlr¢
The torque represented by the Eq. 4.40 can be plotted as
a function of `s’ and is called the torque−slip or torque−
speed characteristic since slip also indicates speed of the as Rs = 0 and Xls = 0. Thus the maximum torque can
machine indirectly. A typical torque−speed characteris- be expressed as
tic for a four-pole machine at 50 Hz is shown in Fig. 4.48.
Vs2  ±Rr′ 
2.0 1.0 0.0 −1.0 Tmax =
3
 
80 ws  
2
 R ′ Rs 2 + Xe2 
Rr′ r
60  Rs ±  + Xe 2
40  Rr′ 2
Rs + Xe 
2
Torque (Nm)

20
0 Thus,
−20
Counter
current
−40 3 Vs2 Rs2 + Xe2
Tmax = ±
(R ± ) +X
braking Motoring Generating
−60 ws 2
−80 s Rs2 + Xe2 e
2

−100
Vs2
−120
3

(
ws 2 R ± R 2 + X 2
)
0

0
00

00

00

00

00
0
00

00

50
50

10

15

20

25

30
15

10

s s e

Speed (rpm)
The positive sign is for motor and the negative sign is for
Figure 4.48 |   Speed−torque characteristic curve. the generator. Thus it can be observed that

4.3.5.1 Condition for Maximum Torque Tmax(m) < Tmax(g)

It can be observed from the torque-speed characteristic Now neglecting stator resistance, we have
(Fig. 4.48) that
dTe Rr¢
=0 smT =
ds Xlr¢

Chapter 4.indd 296 3/23/2016 11:27:29 AM


4.3  THREE PHASE INDUCTION MACHINES     297

Therefore, The common methods used to start induction motors


2 are:
3 Vs
Tmax(m) = (4.43) 
w s 2Xlr¢   1.  Direct-on-line starting
  2.  Autotransformer starting
The basic torque given in Eq. (4.40) can be rewritten as   3.  Star-delta starting
Rr¢
  4.  Rotor resistance starting
3 Vs2
Te(fl) = ×
ws æ R¢ ö
2 sfl The first three methods are applicable to both squirrel-
2 ç Rs + r ÷ (Xls + Xlr¢ )2 cage and slip ring motors. However, the fourth method
ç sfl ÷ø
è is applicable only to slip ring motors.
Now neglecting stator leakage impedance parameters Rs
and Xls, we have 4.3.6.1 Direct-On-line Starting
3 Vs2 Rr¢
Te(fl) = × In this method, the motor is started by connecting it
w s æ R¢ ö
2 sfl directly to three-phase supply. The impedance of the
ç r ÷ + (Xlr¢ )2
ç sfl ÷
motor at standstill is relatively low and when it is directly
è ø connected to the supply system. Thus the starting cur-
rent will be high (6 to 9 times the full load current) and
Substituting Rr¢ = smT × Xlr¢ , we get at a low power factor. This method of starting is suitable
for smaller machines with power up to 7.5 kW.
3 Vs2 s × Xlr¢  (4.44)
Te(fl) = × mT
ws æ s × X ö 2 When the motor is started direct-on-line, the starting
sfl
lr¢
ç ÷ + (Xlr¢ )
mT 2 current (Ist) is the short-circuit (blocked rotor) current
è sfl ø (Isc). Thus starting (Tst) and full load (Tf l) torque rela-
tionship at full load slip sf l is given by
From Eqs. (4.43) and (4.44), we have

smT × Xlr¢ I 
2
3 Vs2
×
Tst
=  sc  × sfl
ws æ (s )2 ö sfl Tfl  I fl 
(Xlr¢ )2 ç 1 + mT 2 ÷
ç (sfl ) ÷ø
Te(fl)
= è
Tmax(m) 2 4.3.6.2 Autotransformer Starting
3 Vs
w s 2Xlr¢
This method connects the induction motor to a
Simplifying the above equation, we get reduced voltage during starting and then connects it
to the full voltage at sufficient speed. The tapping on
Te(fl) 2(Xlr¢ )2 sfl the autotransformer is generally set between 65% and
= × smT
Tmax(m) æ (s )2 ö 80% of the line voltage at the time of starting. Thus
(Xlr¢ ) ç 1 + mT 2 ÷
2
ç (sfl ) ÷ø
starting and full load torque relationship at full load
è slip sfl and autotransformer voltage transformation
2 ratio K is given by
=
sfl s
+ mT
I   KI sc 
smT sfl 2 2
Tst
=  st  × sfl =  × sfl
This is a very important relationship between Te(fl) and Tfl  I fl   I fl 
Tmax(m). When Te(fl) is replaced by starting torque Tst,
I 
2
= K  sc  × sfl
sfl is replaced by s which is equal to 1. Thus 2

Tst 2 I  fl
=
Tmax 1
+ smT 4.3.6.3 Star-Delta Starting
smT

4.3.6  Methods of Starting Three-Phase In this method, the stator winding of the motor has to
Induction Motors be designed for delta operation and is connected in star
during the starting period. When the machine reaches
Different starting methods are employed for induction near 70—80% of full speed, the connections are changed
motors depending on the size and the type of the motor. to delta. This method is used for medium-size machines

Chapter 4.indd 297 3/23/2016 11:27:41 AM


298     Chapter 4:  Electrical machines 

with power up to 20 kW. The starting and full load ring rotor, the rotor terminals are to be short circuited
torque relationship at full load slip sfl for this method is through brushes and slip rings in order to have a flow
given by of current through rotor conductors for torque produc-
tion. For the backward field, the torque produced in the
I   I 
2 2
Tst negative (−ve) direction, with respect to the forward
=  st  × sfl =  sc  × sfl
Tfl  I fl   3I fl 
field. Thus, as the two torques are in the opposite direc-
tion, the resultant torque can be obtained after taking
1I 
2
the difference between the two torques produced.
=  sc  × sfl
3  I fl  If the rotor is stationary, the two torque are equal
and opposite, and thus the resultant torque is zero.
Here Ist and Ifl are the starting phase and full load cur- Therefore, there is no starting torque for a single-phase
rent at the time of delta connection. induction motor.
But, if the rotor is started or rotated somehow by
4.4  SINGLE-PHASE INDUCTION giving external force in one direction, the torque in that
direction will be more than the torque in the other direc-
MOTOR tion. Thus the resultant torque in that direction through
which the motor start rotating becomes positive. The
motor will accelerate in the same direction and will come
Single-phase induction motors are widely used in domes-
to a final speed corresponding to the speed of that load.
tic appliances and for a large number of low-power indus-
Analysis of the machine thus involves rotation of two
trial drives. The single-phase induction motor is similar
fields in opposite direction and thus the theory is termed
to three-phase, squirrel-cage motor except that at full
as `double revolving field theory”.
speed only a single winding in the stator is excited. As
the stator winding is fed from a single-phase supply, the
flux in the air gap is alternating only, not a synchro- 4.4.2  Equivalent Circuit of Single Phase
nously rotating one produced by a polyphase winding. Induction Machine
This type of alternating field cannot produce a torque.
Thus, single-phase induction motor needs a starting A single-phase motor receives its power from a single-­
arrangement. If the rotor is initially given some torque in phase source of defined voltage and frequency. As
either direction, then immediately a torque is produced already discussed, the pulsating mmf can be resolved
in the motor and it accelerates to its final speed, which into two revolving mmfs, rotating in opposite direc-
is lower than its synchronous speed. tions. If the rotor is rotating at speed Nr, the compo-
nent mmf rotating in the same direction as the rotor is
called the forward-revolving field, and the oppositely
4.4.1  Double-Field Revolving Theory
rotating mmf is called the backward-rotating field.
When a single distributed stator winding in a machine Each rotating mmf induces a voltage in the rotor wind-
is fed from a single-phase supply, it carries a sinusoi- ing. Therefore, two equivalent circuits are built: one for
dal current A sinusoidal space distribution of the mmf the forward component mmf and one for the backward-
in the air-gap results, whose peak or maximum value rotating component field. Then, the two component
pulsates (alternates) with time. This sinusoidally vary- fields are combined and the two equivalent circuits are
ing flux (f) can be split in to the sum of two rotat- interconnected.
ing fluxes or fields. The magnitudes of these fluxes are The forward rotating component field rotates at syn-
equal to half the value (f/2) of the alternating flux f. chronous speed Ns in the same direction as the rotor.
Both these fluxes rotate synchronously at the speed Therefore, the slip s+ of the rotor with respect to the
Ns = 120 f/p but in opposite directions. Ns is taken as forward-rotating field may be expressed as
rotation per second or rps. The flux or field rotating
at synchronous speed in the same direction as that of Ns − Nr N
s+ = s = = 1+ r
the motor (rotor) is taken as positive or forward field Ns Ns
where as the field rotating in opposite direction to that
of the motor is known as backward field. Both these The amplitude of the forward component mmf is one half
fields induced emf in the rotor conductors. If the rotor of the stator mmf. Hence, one half of the stator current
is squirrel-cage type, with bars short-circuited via end may be associated with the forward mmf. The equivalent
rings, current flows in the rotor conductors, and elec- circuit for this situation is similar to that of three-phase
tromagnetic torque is produced in the same direction of induction motor, with the modification that the core loss
the forward rotating field. On the other hand, for a slip represented by RC is omitted from the equivalent circuit

Chapter 4.indd 298 3/23/2016 11:27:44 AM


4.4  SINGLE-PHASE INDUCTION MOTOR     299

of the single-phase machine. The core loss is treated sep- jX2


arately and is usually lumped with the rotational losses. I1
R1 jX1 2
Thus, the equivalent circuit is as shown in Fig. 4.49. In
the diagram, V1+ is the stator voltage corresponding to +
jXm R2
the forward component 2s
2

I1 jX2
2 R1 V1
jX1 jX2 2
+ R2 jXm R2
V1
+ jXm
s 2 2(2 − s)
− −

Figure 4.49 |   Equivalent circuit corresponding to Figure 4.51 |   Single-phase induction motor
the forward mmf. equivalent circuit.

Next, the backward component field is considered. The 4.4.3  Starting Methods for Single Phase
stator current corresponding to this field is one half of Induction Motor
the stator current, that is (1/2)I1. Because the rotor and
the backward fields are rotating in opposite directions, the From the above discussion it is evident that the normal single
slip s- of the rotor with respect to the backward-rotating phase motors will have no starting torque. Therefore, these
field is expressed as: machines are usually made split phase induction machines
having two stator windings. The two windings are named
N s − (−N r )
s− = s =
as main winding and auxiliary winding. These windings are
Ns placed at a space angle of 90° electrical, in order to produce a
rotating magnetic field when excited from two phase supply.
Nr æ N ö
= 1+ = 2 − ç1 − r ÷ If two phase supply is not available, the currents in the main
Ns è Ns ø and auxiliary stator windings are to be provided at a time
phase angle of 90° for steady torque production.
Therefore,
4.4.3.1 Resistance Split-Phase Motor
s− = 2 − s+

The equivalent circuit corresponding to the backward- R


revolving field is shown in Fig. 4.50 Auxiliary S
winding
I1
I2 Ia Stator Rotor
2 R1 jX1 jX2
+ I Im Main winding
R2
V1− jXm V
2−s
− (a)
φa
Figure 4.50 |   Equivalent circuit corresponding to
the backward mmf. V
φm Ia
Since one half of the current I flowing in an impedance Z
has the same performance effect as the current I flowing
in the impedance (1/2)Z, the equivalent circuits shown
in Figs. 4.49 and 4.50 may be combined to form the over- Im
all equivalent circuit shown in Fig. 4.51. In this equiva- I
lent circuit, the resistances are combined into R1, and (b)
Figure 4.52 |   Resistance split-phase induction
the reactances are also combined into X1. This equiva-
lent circuit can be used to analyse the performance of a
motor: (a) Schematic diagram,
single-­phase induction motor. (b) phasor diagram and (c) torque-
speed characteristics.

Chapter 4.indd 299 3/23/2016 11:27:51 AM


300     Chapter 4:  Electrical machines 

torque the resistance split phase motors have low start-


Main and auxiliary ing torque. Higher phase difference between the two cur-
winding rents can be achieved by putting a capacitor in series
with the auxiliary winding instead of a resistance. These
Torque motors can be further sub-divided into capacitor-start
motor and capacitor-start motor and capacitor start and
capacitor run motors.
Main
winding Capacitor-Start Motor
0 n0 ns Capacitor-start type of single-phase induction motor is
Speed
shown in Fig. 4.53. In this machine, a capacitor and a
Switch opens centrifugal switch is connected in series with the auxil-
(c) iary winding. The capacitor cost will decrease, as it is
used only at the time of starting and that continuous
Figure 4.52 |   (Continued). duty capacitor is not required. The phasor diagram and
the torque−speed characteristics of the motor are shown
A high value of series resistance is connected with auxil- in Figs. 4.53(b) and (c), respectively.
iary winding to make it a resistance split phase machine.
Thus the auxiliary winding circuit has higher resistance Rotor
to reactance (Ra/Xa) ratio as compared with that in the Switch
main winding. When these two windings are connected Auxiliary Start
in parallel and provided with a common supply voltage, winding capacitor
a time phase difference occurs between the two currents (short time
Ia
and a rotating magnetic field is produced, which in turn rated)
results in a starting torque in the machine. The wind- I Im Main
ing connection is shown in Fig. 4.52(a) and the corre- winding
V
sponding phasor diagram of the winding currents along
with the input voltage are shown in Fig. 4.52(b). The (a)
switch, S is a centrifugal switch and connected in series
with the auxiliary winding. Usually the series resistance Ia
results in high amount of copper losses in the winding
θ
to reduce its efficiency and life. Therefore, the switch is 90° V
made to automatically cut out the auxiliary or starting
winding when the motor attains a speed close to full load
speed. It can be seen from Fig. 4.52(b) that the current I
(Ia) in the auxiliary winding lags the voltage (V) by an Im
angle fa, whereas the current (Im) in the main wind-
ing lags the voltage (V) by another angle fm, which is
(b)
close to 90° and obviously greater than fa. Therefore, the
phase angle between the two currents is (90° - fa), which
Torque Main and auxiliary
winding
should be at least greater than 30° for a considerable
amount torque sufficient for starting. These motors are
generally used in applications such as fans, centrifugal
pump, blower etc. Main
winding
4.4.3.2 Capacitor Split-Phase Motor
ns
From the above discussion it is clear that the phase 0 n0 Speed
angle between the two currents of main and auxiliary
(c)
windings is vital for starting torque production. Also this
phase angle cannot be made very high (close to 90°) for Figure 4.53 |   Capacitor-start induction motor:
the resistance split phase motors. To the contrary, this (a) Schematic diagram, (b) phasor
angle is usually less than 60° for such motors. As an diagram and (c) torque−speed
angle close to 90° is ideal to produce maximum starting characteristics.

Chapter 4.indd 300 3/23/2016 11:27:54 AM


4.4  SINGLE-PHASE INDUCTION MOTOR     301

These motors are also used in different applications Since the auxiliary circuit is leading in nature due to
such as compressor, conveyor, refrigerator and air-­ series capacitance, the phase difference between the
conditioning equipment etc. two currents can be obtained from phasor diagram as
fm + fa > 90° during starting and almost equal to 90°
Capacitor-Start and Capacitor-Run Motor during running. Thus during running, the motor acts as
a balanced two-phase machine. Therefore only the for-
For this type of motors, two capacitors are used, one ward rotating field will be present to produce a smooth
for starting purpose and the other for running the running torque. The efficiency of the motor under this
motor. The first capacitor or the starting capacitor condition is higher as no backward field is present. Thus
is rated for intermittent duty. A centrifugal switch using two capacitors during starting and running, the
is also needed to disconnect this capacitor when the performance of the motor improves both at the time of
motor reaches the speed near rated value. The second starting and then running.
capacitor is to be rated for continuous duty and is
used for running the motor. The phasor diagram of
the main and auxiliary currents and the approximate 4.4.4  Shaded-Pole Motors
torque−speed characteristics are shown in Figs. 4.54(b)
and (c), respectively. The shaded pole motors are also single-phase induction
motors having main winding in the stator. A small por-
Start capacitor tion of the stator poles is covered with a short-circuited,
single-turn copper coil which is known as the shading coil.
Rotor
When AC excitation is provided to the main winding a
Run capacitor
sinusoidally varying flux is created which induces emf in
Switch the shading coil. As the coil is a short circuited, induced
currents flow in the shading coil producing its own flux
Ia
in the shaded portion of the pole. Thus the resultant flux
I Im in the shaded pole portion is sum of main flux compo-
V
nent linking the shading coil and the flux produced by
(a) the shading coil. As seen from the phasor diagram given
in Fig. 4.55(b), the shaded pole flux fsp lags the unshaded
pole flux fm by an angle a resulting in a net torque.
Ia

Since the angle between the two fluxes is fixed, this


α = 90° motor has a definite direction of rotation, which cannot
V
I be reversed. In order to achieve the reversal of the direc-
tion of rotation, two shading coils must be provided. One
coil on each end of the stator pole, and the reversal can be
Im achieve by open-circuiting one set of shading coils and by
(b) short-circuiting the other set. The shaded-pole motors are
single-winding self-starting machines, and therefore these
Main and auxiliary winding are less costly. However, the motor has low efficiency. It is
(both capacitors) generally used for domestic fans, record players and tape
Main and auxiliary winding recorders, mixers and grinders, etc.
(run capacitor only)
fm′
Torque Main
Shading winding
coil
Short Squirrel-cage
circuit rotor
0 n0 ns ring
Speed Stator
(c)

Figure 4.54 |   Capacitor-start and capacitor-run


(a)
induction motor: (a) Schematic Figure 4.55 |   (a) Shaded-pole motor (single-phase
diagram, (b) phasor diagram and induction type). (b) Phasor diagram of
(c) torque-speed characteristics. the fluxes.

Chapter 4.indd 301 3/23/2016 11:27:58 AM


302     Chapter 4:  Electrical machines 

f′m core is assembled with Silicon steel laminations of about


0.5 mm thickness to reduce eddy current losses.
fsc(m) The armature coils are made of insulated copper wires.
fsp The conductors are placed in the slots provided in the
armature (Fig. 4.57). The conductors forming the coil
sides are placed parallel to the shaft of the machine and
are connected in series by the end connections. The rotor
a houses the main field winding which is excited by DC
supply to produce stationary magnetic poles which pro-
q sc
0 E sc duces the rotor field to provide locking action with the
armature.
fsc The rotor can be two types, which also classify the
I sc type of the machine. It may be either salient pole con-
(b) struction type or cylindrical type. Saliency is usually
Figure 4.55 |   (Continued).
considered as asymmetry of air-gap along the polar
axis and perpendicular to the pole axis. In the case of
alternator, a small air-gap in the polar axis and larger
4.5  SYNCHRONOUS MACHINES air-gap in the interpolar axis exists for the salient pole
machines. The rotor type selection is dictated actually
by the choice of prime movers used for energy conver-
The basic diagram explaining the construction of an sion purpose. The term high-speed and low-speed rotor
alternator showing all its parts is shown in Fig. 4.56. are sometimes used synonymously with cylindrical and
salient-pole rotors respectively. As the cylindrical rotor
Fi structures exhibit very less friction and windage loss
eld
wi while in rotation, they can be run at higher speed which
nd could be 3000 rpm for a 2-pole machine generating volt-
ing
age at a frequency of 50 Hz. On the other hand the
salient pole structures cannot be run at higher speeds
Rotor due to high friction and windage loss while in rotation
−a and these machines are designed for lower speeds with
N a ing

a higher number of poles.


-tu tu
ar ind
m
rn re
w

Pole facing short-circuited windings known as damper


windings is present in the rotor of the machine. It is simi-
lar to cage windings in the induction machine rotor and
helps in providing starting torque during motoring opera-
Stator tion. It also helps to reduce the oscillating torque during
Flux paths sudden loading or unloading of synchronous generator.

Figure 4.56 |   Schematic diagram of an alternator.

The arrangement for an elementary synchronous genera-


tor is as shown in Fig. 4.56. It consists of an outer stator
housing the three-phase distributed armature winding
over the inner periphery. Mechanical axis of each wind-
ing is displaced by 120 electrical degrees apart to each
other, as in the case of three phase induction machine.
The various parts of the stator include the stator core,
the stator windings which are usually named as the
armature windings and the necessary cooling arrange-
ments. Generally liquid hydrogen is used as the coolant
medium for the larger alternators above 100 MW rating.

Figure 4.57 |   Armature conductors in slots.


The frame may be cast iron type for small sized machines
and of wielded steel type for larger machines. The stator

Chapter 4.indd 302 3/23/2016 11:28:02 AM


4.5  SYNCHRONOUS MACHINES     303

4.5.1  Principle of Operation Z

To provide alternating emf at the terminals of the syn- E Ia V


chronous machine, two methods may be employed.
  1.  Coils may be rotated in the stationary magnetic
field developed by the poles of the electromagnet or, Ia
  2.  The field may be rotated keeping the armature
coils stationary. φ V V
δ IaZ φ δ
For the generation of emf in the armature conductors IaZ
the second technique is generally used for the larger
machines ranging from few MWs to several hundreds of E E
Ia
MWs due to its several advantages. Leading power factor Lagging power factor
Mechanism for production of emf will be same in both (overexcited) (underexcited)
the above cases, thus emf induced will be, (b)

Figure 4.58 |   Phasor diagram for non-salient pole/


E0(rms) = 4.44ffNkp kd Volts cylindrical rotor synchronous machine.
(a) Generator operation. (b) Motor
operation.
where, f = flux per pole
f = frequency of generation Here,
N = number of turns 1. Vt is taken as reference
kp = pitch factor and 2. Vt = E0 + Ia (Ra + jXs )
kd = distribution factor V t = E0 + I a Z s

3. Load angle d is the angle between Vt and E0.


Load angle d is positive for generator mode of
4.5.2  Phasor Diagram
operation and negative for motoring mode. Ra
is armature resistance and Xs is synchronous
The simplified equivalent circuits of synchronous
reactance.
machine operating as synchronous generator and syn-
4. A synchronous machine will be called overex-
chronous motor to determine the steady-state per-
cited if E0 is greater than Vt, normal excited if
formance parameters of the synchronous machine
E0 is equal to Vt or under excited if E0 is less
connected to a power grid are shown in Fig. 4.58(a)
than Vt.
and (b). These parameters include terminal voltage V,
5. VtIacos f is positive meaning motoring mode.
induced emf E, armature current Ia, power factor f, and

load angle d.
If it is negative, it will mean generating mode;
cosf is the power factor.
6. VtIasin f is positive meaning the machine
draws reactive power from bus or underex-
Z cited mode of operation. If it is negative, it
will mean delivering reactive power to bus or
V E−IaZ = V
Ia overexcited mode.
E

4.5.3  Open-Circuit and Short-Circuit Tests


E E The open circuit and short circuit tests are performed to
IaZ
IaZ obtain the synchronous impedance and voltage regula-
δ φ
Ia
tion calculation for the synchronous machine. The open
φ V V circuit characteristics mainly depict the effect of satu-
Ia ration characteristics of synchronous machines and the
Lagging power factor Leading power factor magnetization curve obtained for the machine. The con-
(overexcited) (underexcited) nection in per phase basis to obtain open circuit and
(a) short circuit characteristics of the machine are shown in

Chapter 4.indd 303 3/23/2016 11:28:07 AM


304     Chapter 4:  Electrical machines 

Fig. 4.59(a) and (b), respectively. Usually the unsatu- This method of voltage regulation is based on the sepa-
rated synchronous impedance is calculated by dividing ration of armature leakage reactance and effects of arma-
the open-circuit terminal voltage by the short-circuit ture reaction. The armature leakage reactance is also
current. However, in the saturation zone, the synchro- called Potier reactance. To determine armature leak-
nous impedance becomes a function of the open-circuit age reactance and armature reaction mmf, two tests are
voltage and saturated synchronous impedance can be separately performed on the alternator. These are open
calculated. The open-circuit characteristic (occ) or mag- circuit test and zero power factor test.
netization curve and the short-circuit characteristic (scc) The open circuit test is performed by keeping the
of the synchronous machine is shown in Fig. 4.60. armature open circuited while the field excitation is
Zs varied continuously from zero up to rated value. The
open circuit voltage is noted. The graph of open circuit
+ + voltage and field current is plotted. In zero power factor
test, the armature is connected to a purely inductive load
E0 Eoc = Eaf as shown in the circuit diagram. The inductive load has
zero lagging power factor and hence is the name of the
test. The load and speed are maintained constant. The
curve for terminal voltage versus excitation current is
(a)
plotted. Only two points are required for constructing
Zs a curve called zero power factor saturation curve. The
circuits for open circuit and zero power factor tests are
+ shown in Fig. 4.61
Isc Potential
E0 Alternator divider
Stator +
F1 −A + D.C.
Prime #
− supply
If
(b) mover F2

Figure 4.59 |   Equivalent circuit (a) under open-


V Field
Switch
circuit condition and (b) under Ia
short circuit condition. A

Air-gap
line occ Purely
Rated voltage reactive
Short-circuit line current

c load
Figure 4.61  |  Open circuit test and zero power factor
Open-circuit line to line

scc test setup of alternator.


h
voltage

e The open circuit characteristics and zero power factor (ZPF)


Rated current

characteristics are shown in Fig. 4.62, where the y-axis re-


d presents terminal voltage and induced emf per phase.

Air-gap line
occ
R Potier triangle
O a b g O′
Saturation
Field current
Rated curve (full
R′
Figure 4.60 |   Open-circuit and short-circuit
Vph Q S P load, zero
power factor)
characteristics. P′
B Q′ S′
4.5.4  Zero Power Characteristics and Potier
Triangle
OC A Ifl
In alternators, the voltage regulation is defined as the Field excitation
rise in voltage at the terminals when a load is thrown Figure 4.62 |   Curves for occ and ZPF characteristics
off keeping the excitation and speed at constant values. with Potier triangle.

Chapter 4.indd 304 3/23/2016 11:28:14 AM


4.5  SYNCHRONOUS MACHINES     305

Now, a tangent is drawn on the open-circuit characteristics 4.5.5  Generator Voltage Regulation
(occ) through the origin as shown by the dotted line,
known as air-gap line. The line PQ is drawn parallel and The per unit value of voltage regulation of a synchro-
equal to OA. From point Q, a line parallel to air-gap line nous generator or alternator is defined as the ratio of
is drawn intersecting occ at R. RQ and PR are joined. change in terminal voltage from no load to any load
The triangle PQR is the Potier triangle. From point and the full load voltage. When this ratio is multi-
R, a perpendicular point is dropped at S. plied by 100 the regulation is expressed in percentage
The ZPF full load saturation curve can now be built value. The voltage change can be calculated by taking
by moving the triangle PQR so that R always remains the numerical difference between Et and Vt, where Vt
on occ and line PQ always remains horizontal. This is is the terminal voltage for a given load and Et is the
the dotted triangle P¢Q¢R¢. Triangles OAB and PQR open-circuit or no load voltage. the voltage regulation
are similar triangles. The perpendicular line RS gives is given by
the voltage drop due to the armature leakage reactance. (E0 − Vt )
Line PS represents field current necessary to overcome Voltage regulation = per unit
the demagnetising effect of armature reaction at full Vt
load. Line SQ gives field current required to induce an
The same can be expressed in percentage as
emf for balancing leakage reactance drop RS.
For determining regulation, the rated terminal volt- (E0 − Vt )
Voltage regulation = ×100%
age Vph as a reference phasor is drawn as shown in Vt
Fig. 4.63. The current phasor Iph is drawn lagging or lead-
ing Vph by angle f. IphRaph voltage drop to Vph which is Considering the phasor diagram as shown in Fig. 4.64
in phase with Iph is drawn. The drop IphXLph is made per- for a lagging load, the no-load emf can be calculated as
pendicular to IphRaph vector but leading IphRaph.
E02 = (Vt cos f + IR)2 + (Vt sin f + IXs )2
XLph is Potier reactance obtained by Potier method
(Fig. 4.63). Phasor sum of Vph(rated), IphRaph and Thus knowing the machine impedance parameters and
IphXLph gives the emf Eph. the terminal voltage and armature current, the no load
voltage Et can be calculated using the above equation.
Thus voltage regulation can also be calculated using the
D above formula for regulation.

Armature Et
Eph
reaction drop IZs IXs
FAR
90°+ f
Ff1 C
F E V
90° E 1ph
IphXLph
90° A
FR B
0 f IphRaph
φ
Vph I
Iph G
Figure 4.64 |   Phasor diagram for R-L load.
H
Figure 4.63 |   Phasor diagram corresponding to Potier
triangle. 4.5.6  Two Reaction Theory of Salient-Pole
Machines (Blondel’s Two-Reaction Theory)
Once the value of Eph is known, the regulation can be Since the air-gap for the salient pole machines is not same
given as, throughout the circumference, the effect of armature
Eph - Vph reaction becomes the function of the space orientation
Regulation % = ´ 100% of the armature mmf vector. Thus the synchronous reac-
Vph tance offered due to the armature reaction will also not be

Chapter 4.indd 305 3/23/2016 11:28:22 AM


306     Chapter 4:  Electrical machines 

the same and will depend again on the phasor ­position VT


d
of  the armature mmf vector. To counteract this prob-
lem, the effect of armature reaction in the case of a
f1
Er sind jIaXs
salient pole machine can be shown to be consisting of f1
two components-one acting along the direct axis or the Ia
field axis and the other acting along the quadrature IaXscosfa
axis or the emf axis as the air-gap is not constant for Er
salient pole machine. The mmf components of armature-­
reaction in a salient-pole machine cannot be taken as
acting on the same magnetic circuit. The effect of the
armature reaction thus cannot be considered only by the
single synchronous reactance instead two unique reac- (b)

Figure 4.65 |   Synchronous motor (a) Equivalent


tances will be considered.
The direct-axis component Fd acts on magnetic cir- circuit and (b) Phasor diagram.
cuit identical with that of the main field and produces
a equivalent outcome while the quadrature-axis compo- These are used for examining the behaviour of a synchro-
nent Fq acts along the interpolar space which results in nous motor, due to changes in mechanical load and/or
an altogether smaller effect and a flux distribution dis- changes in field excitation.
similar from that of Fd or the main field mmf. From the phasor diagram,
Blondel’s two-reaction theory considers the effects
of quadrature and direct axis components of armature Ia Xs cos f = −E sin d  (4.45)

reaction. Neglecting saturation, their different effects are
considered by assigning d-axis and q-axis reactances Xd Multiplying Eq. (4.45) by V on both sides and rearrang-
and Xq respectively. Thus the modified equation for ter- ing terms, we get
minal voltage considering the two reaction theory can be
VIa cos f = − sin d 
derived as, VE
(4.46)
Xs

Vt = E0 + Ia Ra + jI d Xd + jI q Xq
The left side of Eq. (4.46) represents active power input.
This power will also represent the electromagnetic power
where, Ia = I d + jI q and Vt = Vd + jVq .
developed per phase. Hence,

P = VIa cos f = − sin d 


VE
4.5.7  Synchronous Motor Power Equation (4.47)
Xs

The armature resistance of a synchronous motor is Considering a three-phase synchronous machine Eq. (4.47)
reasonably small as compared to its synchronous becomes,
reactance, so,
P = 3 ´ VIa cos f = −3 ´ sin d
VE
V = E + jIa Xs Xs

The equivalent circuit neglecting Ra and the correspond- 4.5.8 Power Angle Characteristics
ing phasor diagram is shown in Fig. 4.65(a) and (b)
respectively. From the per-phase power equation, Eq. (4.47), we have

P = VIa cos f = − sin d


VE
Ia jXs
Xs

If the equation is plotted as a function of the load angle d,


To AC the maximum power is obtained at load angle d = 90°.
VT
source
Ef Increase in load angle d will cause a decrease in power
output as shown in Fig. 4.66 and the synchronous
machine will become unstable and will lose synchronism.
Loss of synchronism will result in exchange of current
(a) surges between the machine and network to which it is

Chapter 4.indd 306 3/23/2016 11:28:35 AM


4.5  SYNCHRONOUS MACHINES     307

connected. The limit of stability will occur at load angle Thus, synchronising power,
d = 90° and is known as the steady state stability limit.
P sy = E1I sy cos f1 = E1Isy cos(90 − q ) = E1I sy sin q
The synchronising power coefficient is given by
The synchronising power will supply the machine 2 and
¶P
− W/ deg the system copper losses,
¶d
E1I sy = E2 I sy + copper losses

P Thus,

P0 (180° − a )
Er = 2E cos  (4.48)
2
(Watts)
If, E1 = E2 = E, then Eq. (4.48) becomes
a
Er = 2E sin
2
Considering that a is very small, we have
d
Er = aE
0 90 180
(Electrical degrees)

Figure 4.66 |   Power angle curve.


Synchronising current can thus be expressed as (neglect-
ing Ra),
E aE
4.5.9  Synchronising Power and Synchronising I sy = r =
2X s 2X s
Torque
Synchronising power supplied by machine 1 (per phase)
The possibility of a machine to return to its state of sta- is given as,
bility after a momentary disturbance can be measured by
synchronising power and torque. Consider that two syn- Psy = E1I sy
chronous machines are working in parallel (Fig. 4.67) and
speed of machine 1 decreases. The machine emf E1 will fall aE 2
or, P sy =
back by a certain angle. The resultant emf Er will be the 2X s
phasor difference of E1 and E2. This resultant emf will pro-
duce a synchronising current which will in turn develop For three-phase the above expression can be multiplied
synchronising torque which will accelerate machine 1 and by 3.
retard machine 2. Thus synchronism will be restored.
The power which is associated with this synchronising 4.5.10  Power Factor Control
torque is called synchronising power.
A synchronous machine can be used for improving the
Isy power factor of a system to which it is connected. This
E1 application of synchronous motor is known as synchro-
Isy nous condenser. In this mode, the synchronous motor is
operated in no-load and over-excitation mode. It behaves
Rs Rs φ1
θ
a variable capacitor. In industries, this machine may be
Er used with induction motors and operated with over excita-
tion to draw leading current from the supply. Thus, it can
Xs Xs φ2 compensate for the lagging current drawn by the induction
motor, for improving the overall power factor of the plant.
a
E1 E2
4.5.11  Hunting
E2
Isy
If there is a sudden change in the load torque in a syn-
Figure 4.67 |   Two synchronous machines connected chronous motor, the equilibrium for the machine will be
in parallel. disturbed and the machine speed will change. During

Chapter 4.indd 307 3/23/2016 11:28:48 AM


308     Chapter 4:  Electrical machines 

sudden unloading, the synchronous machine runs at 4.5.13  Cogging and Crawling
super-synchronous speeds. Again during sudden load-
ing, the synchronous machine runs at sub-synchronous Induction and synchronous motors have a series of slots
speeds. The load angle change causes a change in electro- in the stator and in the rotor. If the number of slots
magnetic torque and consequently the speed. All these in the stator (S1) is equal to or an integral multiple of
conditions of change in loads will cause the motor rotor number of slots in rotor (S2), the motor will either not
to oscillate about the final equilibrium position. This accelerate or will not accelerate to full speed, that is stop
oscillation of the rotor about its final equilibrium posi- accelerating at part speed. These phenomena are known
tion is known as hunting. as cogging and crawling, respectively.
Various causes of hunting are listed as follows: When the number of rotor slots is equal to the number
  1.  Sudden change in loads of stator slots, the motor will not start at all, particularly
  2.  Due to faults in the systems with the supply if the supply voltage is low. This happens if magnetic
  3.  Unexpected change in field current locking between the slots and the rotor tends to remain
fixed. So the phenomenon of cogging is also called mag-
The following remedies are used to reduce hunting: netic locking of induction motor. To prevent cogging,
  1.  Damper windings are added in the field pole faces generally there are an unequal number of slots in the
to damp out the oscillations in the rotor. rotor and in the stator. However, torque modulations are
  2.  Flywheels may also be used which increase the still observed under situations where the slot frequencies
inertia of the machine and help in maintaining con- coincide with harmonic frequencies. To reduce this prob-
stant rotor speeds. lem, the slots are skewed to keep an overlap on all slots.
  3.  Designing synchronous machines with appropriate For certain combinations of S1 and S2, the motors
power coefficients. exhibit a tendency to run at very slow speeds, It stops
accelerating at part speed and the acceleration is also
4.5.12  Slip Test limited by the torque output of the motor relative to the
load torque at that speed. This phenomenon of crawling
The unsaturated values of Xd and Xq for a three-phase happens is due to the fact that, flux wave produced by
salient-pole synchronous machine can be found using a stator winding is not purely sine wave. Instead, it is a
the slip test. In this test, low value of balanced three- complex wave consisting a fundamental wave and odd
phase voltage is applied to the machine armature, and harmonics like 3rd, 5th, 7th, etc. The fundamental wave
the rotor is driven at a speed slightly different from the revolves synchronously at synchronous speed Ns whereas
normal synchronous speed while the field circuit is left 3rd, 5th, 7th harmonics may rotate in forward or back-
open (Fig. 4.68). The rotating armature mmf steadily ward direction at Ns/3, Ns/5, Ns/7 speeds respectively.
changes, on account of the slip changing between the Hence, these harmonic fluxes developed in the gap due
polar and interpolar axes. The reluctance of the mag- to the magnetics of the motor create torque fields addi-
netic circuit varies cyclically between an upper and a tional to fundamental torque.
lower limit. The armature current as a result changes in
the opposite sense. The value of Xq and Xd can then be
calculated as,
4.5.14  Comparison between Synchronous
Motors and Induction Motors
Vmin V
Xq = and Xd = max A point-wise comparison of synchronous and induction
I max I min motors is listed as follows:
  1.  A synchronous motor always runs at synchronous
Field Armature speeds which is independent of loads. The induc-
R A tion motor will always run at speeds below the syn-
Three- chronous speed. This is also dictated by loads.
F1 V phase   2.  A synchronous motor is a doubly excited machine
auto Three-
as both the armature and the field coils are ener-
F2 trans- phase
gised separately. The induction motor on the other
Y former supply
B hand needs excitation on the stator field only.
  3.  A synchronous motor can work in wide range of
Open power factors while the induction motor only works
field at lagging power factors.

Figure 4.68 |   Circuit diagram for slip test.


  4.  The synchronous motor is usually not a self-
starting machine and requires special starting

Chapter 4.indd 308 3/23/2016 11:28:51 AM


4.6  STEPPER AND SERVO MOTORS     309

arrangements. The induction machine on the other Permanent Magnet Stepper Motor
hand has self-starting torque.
In the permanent magnet stepper motor, the rotor is
  5. With same output and voltage rating, a synchronous
made of permanent magnet. It is a smooth cylindrical
motor is more efficient than an induction motor.
shaped with fixed stator poles. Supplying current to each
  6.  A synchronous motor is costlier than an induction
stator phase in proper sequence will rotate the rotor
motor of equivalent ratings.
adjusting to the changing electromagnetic field. These
are generally low cost motors having typical step angle
4.6  STEPPER AND SERVO MOTORS range of 7.5° to 15°. Figure 4.70 shows a permanent
magnet stepper motor.

4.6.1 Stepper Motor

Stepper motor is an electromechanical energy conversion Stator 1


device that converts electrical pulse inputs to discrete
mechanical movements. Due to the nature of its working, N
it is primarily used for position control systems. A step in a S S
stepper motor is defined as the angular rotation produced 2 2
N N
by the shaft of the motor each time it receives a pulse. Each S
step causes the shaft to rotate a definite number of degrees. Rotor
Step angle (b) represents the rotation of the shaft caused Windings
by each step and can be expressed mathematically as: 1
360° Permanent
Step angle =
Number of poles magnet poles
Figure 4.70 |   Permanent magnet stepper motor.
4.6.1.1 Types of Stepper Motors

Stepper motors can be classified as: Hybrid Stepper Motor


  1.  Variable reluctance stepper motors Hybrid stepper motors combine the features of both vari-
  2.  Permanent magnet stepper motors able reluctance and permanent magnet stepper motors.
  3.  Hybrid stepper motors. Hybrid stepper motor has rotor made of permanent
magnet with metal teeth. Rotation in a hybrid stepper
Variable Reluctance Stepper Motor motor is produced in the similar way as in a perma-
nent magnet stepper. Hybrid stepper motors are more
In the variable reluctance stepper motor, the rotor is made
expensive than permanent magnet motors, but they use
of soft-steel with teeth. The stator poles have a magnetic
smaller steps. They also have greater torque and maxi-
core assembled with a stack of steel laminations. When
mum speed ratings among the other types. Figure 4.71
the stator coils are sequentially energised from a source,
shows a hybrid stepper motor.
the rotor teeth align with the stator poles. The motor
rotates by minimising the reluctance along the pathway
of the applied magnetic field. Figure 4.69 shows the cross- Stator
section view of a variable reluctance stepper motor. 1

Stator 1 N
S S
2 2
3 2 N N
S
Rotor
Rotor Windings
2 1
3
Permanent
1 Windings magnet poles
Metal teeth and metal teeth

Figure 4.69 |   Variable reluctance stepper motor. Figure 4.71 |   Hybrid stepper motor.

Chapter 4.indd 309 3/23/2016 11:28:56 AM


310     Chapter 4:  Electrical machines 

4.6.1.2 Torque vs. Speed Curves of Stepper A servo motor may be a DC or AC or brushless DC
Motors motor combined with a sensor or position feedback
system. These motors can be used as position control
The torques of stepper motor depends on different motor devices in a closed loop control system. This is done
and driver combinations. Figure 4.72 shows the typical with the aid of a feedback device such as a position
torque vs. speed curves of stepper motor. sensor. The feedback device converts the mechanical
motion into electrical signals which is used as feedback.
This feedback is sent to an error detector. If there is
Holding Pull-in torque
torque an error, it is fed directly to an amplifier, which makes
Pull-out torque necessary corrections. These motors can be used for
Torque (N-m)

Slew speed, torque, and direction control applications. A


range Maximum typical construction of servomotor drive is shown in
response
(pulses the Fig. 4.73.
per second)
Start-stop range
Error
Maximum no detector
Speed (pulses per second) Reference + Servo-

load response Amplifier Sensor
input motor
(pulses per
second)
Feedback loop
Figure 4.72 |   Torque vs. speed curves of stepper motor.
Figure 4.73. |   Block diagram of a typical servomotor
The characteristics features of the speed-torque curve drive.
are described as follows:
  1.  Holding torque: It is the torque that the stepper 4.6.2.1 Types of Servomotors
motor produces when the motor is supplied with
rated current through the stator windings but the DC Servomotor
motor is at standstill.
A DC motor may be used for servomechanism and
  2.  Detent torque: It is the torque that the stepper
consists of gears, position sensing devices and control
motor produces when the stator windings of the
circuitry along with the motor. DC motors are a more
motor are not energised.
common choice in servo-mechanism applications. On the
  3.  Pull-in torque: It is the maximum torque that
basis of motor used, they can be further divided into
the stepper motor can produce at a given speed,
separately-excited servomotor, permanent magnet servo-
so that it can start, stop or reverse its direction of
motor and brushless DC servomotors.
rotation. The stepper motor cannot start at a speed
that is outside this curve.
  4.  Pull-out torque: It is the maximum torque that AC Servomotor
the stepper motor can produce at a given speed Generally, the large industrial servomotors are induc-
while running in synchronism. If the motor is run tion motors. These are equipped with variable frequency
outside of this curve, it will come to standstill. drives for controlling speed. These motors are generally
  5.  Stop-start region: It is the area on and below used in the automation industries. They are used in con-
the pull-in curve in the torque vs. speed character- stant speed and low starting torque applications. These
istic curve. For any load value in this region, the motors require lower maintenance and are less prone to
motor can start, stop, or reverse its direction of failure than the DC servomotor types.
rotation at the corresponding speed.
  6.  Slew range: It is the area between the pull-in
torque curve and the pull-out torque curve. It is 4.6.2.2 Classification of Servomotors based
also the region where the stepper motor is usu- on Operation
ally operated. A stepper motor cannot be directly
started in the slew range. On the basis of operation, servos can be classified into
three types, namely: positional rotation servos, continu-
ous rotation servos and linear servos.
4.6.2 Servo Motors
  1.  Positional rotation servo: This is the most
Servo motors or simply servos are electromechanical common servo motor available. The shaft of this
devices that can produce precise degrees of rotation. type of servo rotates in about 180° or half a cycle.

Chapter 4.indd 310 3/23/2016 11:28:59 AM


IMPORTANT FORMULAS     311

It has gear assembly and position sensors to pre- other devices which rely on precise continuous
vent turning beyond limits to save the sensor. movements.
These servos are commonly used in aircrafts,   3.  Linear servo: These servos are fitted with special
robots, toys and several other applications. gears to change the rotational motion into linear
  2.  Continuous rotation servo: These servos can motion. These servos generally find application in
run continuously in either direction. They usu- automation industries or precise linear drive units
ally employ speed sensors. These servos can be in large industries.
found in radars, robots, unmanned vehicles and

IMPORTANT FORMULAS

I. Ideal Transformer 7. Efficiency (per unit)

1. Instantaneous values of the applied voltage and xV1 cos f


induced emf e1 based on Kirchhoff’s voltage law: xV1 cos f + x2Pcu + Pcore
v1 − e1 = 0
Condition for maximum efficiency
2. RMS value of induced emf E2 = 4.44fT2f 2m V Pcore
x=
3. Voltage ratio between the primary and secondary Pcu
windings
8. Percentage transformer efficiency
E1 T
= 1 xV2 I2 cos f
E2 T2 %h = × 100%
xV2 I2 cos f + Pcore + x2Pcu
RMS value E1I1 = E2 I2
4. Equivalent circuit parameters, 9. All day efficiency:
Total output for 24 hours
V1 ×100%
Rc =
I 0 cos f0
Total output for 24 hour + Total losses for 24 hours

V1 II. Autotransformer
and Xm =
I 0 sin f0 10. Turns or transformation ratio

where, I c = I 0 cos f0 and I m = I 0 sin f0 V2 N


= 2 = a (also represented by K )
V1 N1
5. The equivalent circuit parameters for short circuit
test
Transformed power (VA) (V1 − V2 )I1
11. =
2 Input power (VA) V1I1
Psc = I sc Re
V2
Re =
Psc = 1− = 1−a
2
I sc V1

Xe = Ze2 − Re2 Conducted power (VA) V2 I1 V


12. = = 2 =a
Input power (VA) V1I1 V1
6. Voltage regulation
Vnl − Vl III. Three-Phase Transformer
Vnl
13. Power delivered by all three transformers in open-

R = e r cos q 2 − e x sin q 2
delta connection PV = 3 × V L × I ph

Chapter 4.indd 311 3/23/2016 11:29:21 AM


312     Chapter 4:  Electrical machines 

Power delivered for all three transformers in delta Speed-current characteristics


connection PD = 3× VL × I ph (V − Ia Ra )
w=
Therefore, Ka f
PV Speed-torque characteristics
= 0.577 or 57.7%
PD TR
w=
V
− e a2
Ka f ( K f )
14. Per unit (pu) quantities a

Actual value of the quantity in any unit 21. Characteristics of DC series motors
Base value of the qunatity in the same unit Torque-current relationship Te = Ka fIa
Base current, Voltage equation V = E + Ia ( Ra + Rf )
Pbase
I base = Speed-current characteristic
3 × Vbase
Ra
w=
V

Ka¢Te
Base impedance, Ka
æ Vbase_old ö Pbase_new
2

Z2 = Z1 × ç ÷
22. Speed of DC motors
ç Vbase_new ÷ Pbase_old V − Ia Ra
è ø
N =K rpm
f
IV. DC Machines
23. Efficiency of DC machines
15. Reactance voltage, For generator,

hg =
2I Output VI
1.11L =
TC Output + losses VI + (I + I sh )2 Ra + Pc
where TC is the commutation time. For motor,
16. The total induced emf is, Input − losses VI − (I − I sh )2 Ra + Pc
hm = =
Input VI
E = Bavg × L × v
Z
V
a
24. Testing of DC Machines
pfwZ In Swinburne’s test constant losses,
17. Induced emf in armature E = V
2pa
General form, Pc = VI 0 − (I 0 − I sh )2 Ra
E = Ka fw
V. Three-phase Induction Machine
pZ
where Ka= emf constant = V/Wb/rad/s 25. Three phase currents are given by,
2pa
ia = I m cos 2p ft
18. Electromagnetic torque Te = Ka fIa Nm
ib = I m cos(2p ft − 120°)
19. Characteristics of DC generators
ic = I m cos(2p ft − 240°)
Terminal voltage,
where Im is the maximum value of the stator cur-
V = E − Ia Ra − Vb rent in each phase.
26. Resultant flux of the three phases,
where E is the induced emf, Vb is brush contact
drop (in Volts) and Ia is the armature current Fr (t) = Fa coswtcosq + Fbcos(wt − 2p /3)cos(q − 2p /3)
+Fc cos(wt + 2p /3)cos(q + 2p /3)
(in Amperes)
20. Characteristics of DC shunt motors
Torque-current relationship, Te = Ka fIa or Fr (t) = 3Fa /2cos(wt − q )

Voltage equation, V = E + Ia Ra where Fa= Fb = Fc= NIm.

Chapter 4.indd 312 3/23/2016 11:29:51 AM


IMPORTANT FORMULAS     313

27. Rotor current 35. Methods of starting three-phase motors

E1 Direct-on-line starting
Ir =
I 
2
R r2 Tst
+ (Xlr2 ) =  sc  × sfl
s2 Tfl  I fl 

28. Shunt parameters Pnl = Vnl I nl cosfnl Autotransformer starting

I 
2
V nl
Rc = cosfnl Tst
= K 2  sc  × sfl
I nl Tfl  I fl 
and
V nl Start-delta starting
Xm = sinfnl
I nl
1I 
2
Tst
=  sc  × sfl
29. Series parameters Pbr = Vbr I br cosf br Tfl 3  I fl 

V br
Zbr = VI. Single Phase Induction Machines
I br

R br = Z brcosf br
36. Slip of the rotor with respect to the forward-­
rotating field
and
Ns − Nr
Xbr = Zbr sinf br
N
s+ = s = = 1+ r
Ns Ns
30. Power flow diagram Pem = Pg − sPg = Pg (1 − s)
37. Slip of the rotor with respect to the backward-
31. Per phase rotor current rotating field

 Vs N s − (−N r ) æ N ö
Ir =
æ R¢ ö s− = s = = 2 − ç1 − r ÷
Rs + ç r ÷ + j(Xls + Xlr¢ ) Ns è N s ø
è s ø
32. Air-gap power VII. Synchronous Machines
2 Rr¢
Pg = I r 38. Induced emf
s
E0(rms) = 4.44ffNkpkd Volts
Vs2 Rr¢
=
æ R r¢ ö
2 s No load emf
ç R s + s ÷ + j(Xls + Xlr¢ )
2
è ø E02 = (Vt cos f + IR)2 + (Vt sin f + IXs )2

33. Electromagnetic torque 39. Zero power characteristics


Eph − Vph
3 Vs2 Rr¢ Regulation % = ×100%
Vph
¢
Te =
ws
(Rs + r )2 + (Xls + Xlr¢ )
R s
40. Generator voltage regulation
s
(E0 − Vt )
34. Maximum torque × 100%
Vt
3 Vs2 41. Voltage considering the two reaction theory
Tmax = ±
s(
ws 2 R ± R 2 + X 2
s e ) Vt = E0 + Ia Ra + jI d Xd + jI q Xq

where, Ia = I d + jI q and Vt = Vd + jVq


The positive sign is for motor and the negative sign
42. Synchronous motor power equation
is for the generator. So
P = VIa cos f = − sin d (for single phase motor)
VE
Tmax(m) < Tmax(g) Xs

Chapter 4.indd 313 3/23/2016 11:30:20 AM


314     Chapter 4:  Electrical machines 

44. Synchronising current,


P = 3 × VIa cos f = 3 ×− sin d
VE
aE
Xs Er
(for three phase motor) I sy = =
2Xs 2X s
43. Synchronising power
45. In slip test
Psy = E1I sy cos f1 = E1I sy cos(90° − q ) = E1I sy sin q
Vmin V
aE 2 Xq = and Xd = max
Psy = I max I min
2Xs

SOLVED EXAMPLES

Transformer Load rating of transformer = 600 kVA


Load = 300 kVA at 0.8 point lag
1. If the load power factor is varied from high to low in Thus the percentage voltage drop
case of a transformer, which of the following is true? = x (Îr cos q2 + Îx sin q2 )
(a) Core losses and copper losses will remain Load point = cos q2 = 0.8
unchanged
(b) Core loss increases then sin q2 = 1 − 0.82 = 0.6
(c) Copper loss increases, core loss decreases
Therefore, percentage voltage drop
= (0.5 × 0.8 × 0.35 + 0.5 × 0.6 × 0.08) × 100 %
(d) None of these
Ans. (a)
= 3.8 %
2. A transformer on no load is switched on to a source Ans. (a)
of voltage. It will draw a current, which is
5. A 300 kVA, two winding transformer has iron loss
(a) Same as steady state magnetising current of 2 kW and a copper loss at normal output current
(b) Several times as steady state magnetising cur- of 2.5 kW. The loading at which the efficiency will
rent depending on the initial state of residual be the maximum is
flux in the transformer core
(a) 235.41 kVA (b) 268.3 kVA
(c) Several times as steady state magnetising cur-
(c) 335.41 kVA (d) 160 kVA
rent independent of the initial state of residual
flux in the transformer core
Solution:  Efficiency of the transformer is given by
(d) Twice the steady state magnetising current
Output power
h=
provided the core has no residual flux
Ans. (c) Output power + losses
3. Two transformers of identical voltages but differ- V2 I2 cos q2
=
ent capacities are operating in parallel. For satis- V2 I2 cos q2 + Pcu + Pcore
factory load sharing
Now assuming that the load of x fraction of full
(a) impedances must be equal load gives maximum efficiency, the above equation
(b) per unit impedances must be equal is given by,
(c) per unit impedances and X/R ratios must be
equal dh d  xV2 I2 cos q2 
(d) impedances and X/R ratios must be equal =   =0
dI2 dI2  xV2 I2 cos q2 + x Pcu + Pcore 
2
Ans. (b)
4. For a transformer with a percentage resistance Differentiating, we get

( )
of 3.5% and percentage reactance of 8% of rating
600 kVA, when it is delivering 300 kVA at 0.8 pf xV2 cos q2 xV2 I2 cos q2 + x2Pcu + Pcore

( )
lagging, percentage voltage drop will be
−xV2 I2 cos q2 xV2 cos q2 + x2 2I2 re2 = 0
(a) 3.8%   (b) 8.6%   (c) 9.6%   (d) 11.56%
Solution:  Given that:
(Using Pcu = I22 re2 ; where re2 = equivalent resistance
per unit resistance, Îr = 0.035
per unit reactance, Îx = 6.08 related to secondary side)

Chapter 4.indd 314 3/23/2016 11:30:31 AM


SOLVED EXAMPLES     315

xV2I2 cos q 2 + x2Pcu + Pcore − xV2I2 cos q 2 + 2x2Pcu = 0


And the phasor diagram is
2200 V
x2Pcu = Pcore 72.36°

Pcore
x= I0 = 0.6 A
Pcu
Input power,
Then from the problem, 2200 × 0.6 × cos f0 = 400 W
⇒ cos fnl = 0.303
2
x=
2.5
= 0.8944 Thus no load pf angle fnl = cos-10.303 = 72.36o
From the equivalent circuit
So the load = 0.8944 × 300 = 268.3 KVA at which I0 = Im + Ic
the max efficiency occurs. Thus, from the phasor diagram, iron loss compo-
Ans. (b) nent of current
6. A 6600/440 V, 50 Hz, single-phase transformer has Ic = 0.6 cos 72.36o = 0.6 × 0.303 = 0.182 A
an equivalent resistance of 0.02 pu and equiva- Ans. (a)
lent reactance of 0.05 pu. Find full load voltage 8. For the transformer mentioned in Question 7, find
regulation at 0.8 pf lagging if primary voltage is the magnetising current.
3300 V.
(a) 0.572 A (b) 0.7 A
(a) 0.016 (b) 0.096 (c) 0.8 A (d) 0.9 A
(c) 0.046 (d) 0.009
Solution:  Magnetising current, Im = 0.6 × sin(72.36°)
Solution:  The full load voltage regulation = = 0.572 A
(∈r cos q 2 + ∈x sin q 2) Ans. (a)
Given that Îr = 0.02 and Îx = 0.05. Also, at 0.8 pf 9. A 230/460 V transformer has a primary resistance
lag, cosq2 = 0.8 and sinq2 = 0.6. Therefore, of 0.2 W and a reactance of 0.5 W and corresponding
(0.02 × 0.8 + 0.05 × 0.6) = 0.046 values for secondary are 0.75 W and 1.8 W, respec-
Ans. (c) tively. Find the secondary terminal voltage when
supplying 10 A and 0.8 pf lagging.
7. A 2200/200 V transformer draws a no load primary
current of 0.6 A and absorbs 400 W. Find iron loss (a) 224 V (b) 324 V
current. (c) 424.8 V (d) 567.2 V
(a) 0.182 A (b) 0.2 A Solution:  For the given transformer,
(c) 0.25 A (d) 0.3 A
I2 I2
Solution:  The given transformer can be repre- r1 jx1 r2 jx2
sented as V1 V2
0.6A, 400 W
N1 : N2
2200 V 200 V
N1 V
= 1
N2 V2
2200 V/ 200 V
Also N1I1 = N 2 I2
Neglecting the primary leakage impedance the
I1 N
equivalent circuit is, or = 2
I2 N1
I0 Secondary side copper loss = I22r2
Now looking from the primary side, this loss will be
Ic Im
 N1 
2
Rc jxm
 N  ⋅ I2 r2
2
Vs = 2200 V
2

Chapter 4.indd 315 3/23/2016 11:30:46 AM


316     Chapter 4:  Electrical machines 

Thus, the same loss in terms of primary coil is Solution:  At 0.8 pf lead, voltage regulation =
= I22r′2 Îr cos q2 − Îx sin q2
So, secondary resistance referred to primary =(0.015 × 0.8 - 0.035 × 0.6) × 100% = -0.83%
N 
2
Ans. (d)
= r2′ =  1  ⋅ r2
 N2  13. A 2400/240 V, 200 kVA, single-phase transformer
has a core loss of 1.8 kW at rated voltage. Its equiv-
Similarly, the secondary leakage reactance referred alent resistance is 1.1%. Transformer efficiency at
to primary 0.9 power factor and on full load is
N 
2
(a) 97.57% (b) 98.05%
x2′ =  1  = x2
N 
2
(c) 97.82% (d) 96.56%

So the equivalent impedance of the transformer Solution:  The given transformer can be repre-
referred to secondary sented as

 460 
2
= (0.2 + j0.5) + 0.75 + j1.8 Ω
 230 
= 1.55 + 53.8 Ω = re2 + jxe2
2400/240 V
Now, the secondary voltage is given by 200 kVA
V2 − I2 (re2 + jxe2 ) Given that at full load:
= 460 − 10 ∠ −36.87 o (1.55 + j3.8)
Pcore = 1.8 kW
= 424.8 V
Îr = 0.011
Ans. (c) cos q2 = 0.9
Thus full load copper loss = 0.011 × 200 × 103
10. A transformer has copper loss of 1.5% and reac-
tance 3.5% when tested on load. Calculate its full
= 2200 N
load regulation at unity power factor.
So, efficiency at full load
(a) 1.50   (b) 3.32   (c) 0.83   (d) −0.83 P0
h=
Solution:  Given that Îr = 0.015 and Îx = 0.035
P0 + Pcore + Pcu

Full load regulation = Îr cos q2 + Îx sin q2 200 × 103 × 0.9


= × 100%
At unity power factor, cos q2 = 1, sinq2 = 0 200 × 103 × 0.9 + 2200 + 1800
= 98.05%
So, voltage regulation at unity power factor
Ans. (b)
Îr ×100% = 0.015 × 100% = 1.5%
14. If applied voltage of a certain transformer is
Ans. (a) increased by 50% and frequency is reduced to 50%
11. For the transformer given in Question 10, find full (assuming magnetic circuits remain unsaturated),
load regulation at 0.8 power factor lagging (%). maximum core flux density will
(a) 1.50    (b) 3.32    (c) 0.83    (d) -0.83 (a) change to 3 times its original value
(b) change to 1.5 times its original value
Solution:  At 0.8 pf lag, cos q2 = 0.8, sin q2 = 0.6 (c) change to 0.5 times its original value
At, 0.8 pf lag, voltage regulation = Îr cos q2 + Îx sin q2 (d) remains same as before

= (0.015 × 0.8 + 0.035 × 0.6) × 100% = 3.32% Solution:  The emf induced in a transformer (rms
Ans. (b) value) is E = 4.44 f Bm AN

µ Bm
12. For the transformer given in Question 10, find full E V
Thus, ≈
load regulation at 0.8 power factor leading (%). f f
(a) 1.50    (b) 3.32    (c) 0.83    (d) −0.83 (assuming no leakage impedance drop)

Chapter 4.indd 316 3/23/2016 11:31:01 AM


SOLVED EXAMPLES     317

Given that E2 = 1.5 E1 and f2 = 1.5 f1. So Thus total core loss
x
E1 E Pcore = Ph + Pe = kh fBm + ke f 2 Bm
2
= 2
f1 f2
where kh and ke are the hysteresis loss and eddy
which means no change in case flux density. current loss coefficients and x is the Steinmetz con-
Ans. (d) stant (usually for iron x = 1.6)
15. A single-phase transformer has a rating of 15 kVA, Now under condition 1: input voltage = 1200 V and
600/120 V. It is reconnected as an autotransformer f1 = 50 Hz, Pcore = 800 W, Ph = 550 W and Pe =
to supply at 720 V from a 600 V primary source. 250 W, so
Maximum load it can supply is V1 1200
=
(a) 90 kVA (b) 18 kVA f1 50
(c) 15 kVA (d) 12 kVA
Under condition 2: input voltage = 2400 V and f1
Solution:  The given transformer can be repre- = 100 Hz, so
sented as
V2 2400 1200 V1
= = =
f2 100 50 f1

600 V 120 V So Bm1 = Bm2 (no change in flux density)


Thus,
x
15 kVA Ph1 k 50Bm1 50
= h x
=
When reconnected as an autotransformer the con- Ph2 kh 100Bm1 100
figuration is, 100
Ph2 = × 550 = 1100 W
I2 50

125 A And
I1
2

150 A 720 V Pe2 k 1002 Bm1


25 A = e 2
=4
600 V Pe1 ke 502 Bm1
Pe2 = 4 × 250 = 1000 W
15000 Thus
The current rating of 600 V winding = = 25 A
600
Pcore2 = 1100 + 1000 = 2100 W
And that of the 120 V side is 125 A.
Thus maximum load supplied Ans. (c)
= 125 × 720 = 90 kVA 17. For a 200/400 V, 50 Hz, single-phase transformer:
or, 150 × 600 = 90 kVA
Ans. (a) Open circuit test: 200 V, 0.7 A, 70 W on low volt-
age side
16. When a transformer is connected to 1200 V, 50 Hz
supply, core loss is 800 W of which 550 W is hyster- Short circuit test: 15 V, 10 A, 85 W on high voltage
esis and 250 W is eddy current loss. If the applied side
voltage is raised to 2400 V and frequency to 100 Hz, Calculate secondary voltage when delivering
new core loss is 5 kW at 0.8 pf lagging, primary voltage being
200 V.
(a) 1200 W (b) 3100 W
(c) 2100 W (d) 1700 W (a) 377.8 V (b) 400 V
(c) 440.2 V (d) 482 V
Solution:  The transformer core losses have two
components (i) hysteresis loss (Ph) and (ii) eddy Solution:  The given transformer can be repre-
current loss (Pe) sented as

Chapter 4.indd 317 3/23/2016 11:31:13 AM


318     Chapter 4:  Electrical machines 

(a) 175.53 kVA (b) 137.744 kVA


(c) 150.0 kVA (d) none of these

Solution:
200 V/400 V 0.02 + j0.06 kVA
Neglecting primary and secondary resistance,
0.025 + j0.08 kVA
For open-circuit test: 100 kVA
200 × 0.7 × cos f0 = 70
500 kVA
or, cos f0 =
70
= 0.5
0.7 × 200

sin f0 = 1 − 0.52 = 0.866 Ze 1 = (0.02)2 + (0.06)2 = 0.063

So, Xm =
200
= 329.9 W
Ze 2 = (0.025)2 + (0.08)2 = 0.0838
0.7 × 0.866
Normalising Ze2 to 100 kVA base, we get
= 571.4 W
200
And Rc =
0.7 × 0.5 Ze2 = 0.838 × 5
For short-circuit test: Thus kVA supplied without overloading any one
æZ ö
102 × reff = 85 Þ reff = 0.85 W = (kVA)1 + ç e1 ÷ (kVA)2
è Ze 2 ø
15
Zeff = = 1.5 , kVA1 = kVA with lower leakage impedance
10
So, the kVA supplied
Xeff = 1.52 − 0.852 = 1.236 W 0.063
= 100 + × 500 = 175 kVA
Load = 5 kW at 0.8 pf lag 0.0838 × 5

E2 I2 cos q2 = 5000  (i) Ans. (a)


19. A single-phase transformer with a ratio of 440/110 V
I2 takes a no load current of 5 A at 0.2 pf lagging. If
secondary supplies a current of 12 A at 0.8 pf lag,
0.85 j1.236 find current taken by primary
EH 400 V V2
(a) 30 A   (b) 40 A   (c) 34.45 A   (d) 51.4 A

Solution:  The given transformer can be repre-


V2 = 400 − I2 (0.8 − j0.6 )(0.85 + j1.236 ) sented as
From Eq. (i)
400 × I2 × 0.8 = 5000 Þ I2 = 15.625 A
Then,
V2 = 400 − 15.625 (1.4216 + j0.4788 ) 400/110 V
= (400 − 22.1875)2 + (7.48)2 Given that: I0 = 5.0 A at 0.2 pf lag, cos f0 = 0.2 or
= 377.88 V f0 = 78.46°, I2 = 12 A ∠-36.87o
Ans. (a) I1 12AÐ−36.87°
18. Two single-phase transformers rated 100 kVA and I0 5A I2
500 kVA have pu leakage impedance of (0.02 +
j 0.06) and (0.025 + j 0.08), respectively. Largest
kVA load delivered by parallel combination of these
two transformers without overloading any one is

Chapter 4.indd 318 3/23/2016 11:31:36 AM


SOLVED EXAMPLES     319

Primary current No load emf,


pfNZ 4 × 0.1 × 1000 × 300
N1
I1 = 5Ð −78.46° + 12Ð −36.87° × Þ
440 E= = = 500 V
60 p 60 × 4
N2 110
= 1 − j4.9 + 38.4 − j28.8 Þ 39.4 − j33.7
Thus,
500
I1 = 51.84 A If = = 4A
125
(Assuming energised from lower voltage side.) Now, Vt = E - Iara
Ans. (d) We have, Ia = Ic + If = 90 + 4 = 94 A
20. A three-phase balanced load of 40 kVA is connected So
to V-V bank. The kVA rating of each transformer Vt = 500 - 94 × 0.2 = 481.2 V
in V-V bank should be Ans. (b)

(a) 46     (b) 23     (c) 20     (d) 40 23. A 220 V DC shunt motor runs at 500 rpm when the
armature current is 50 A. Calculate the speed if the
Solution:  KVA ratio of each transformer torque is doubled. Given that armature resistance
2 is 0.2 W.
= 40 × ≈ 23 kVA
3 (a) 476 rpm (b) 432 rpm
Ans. (b) (c) 447 rpm (d) 494 rpm
DC Machines Solution:  For the given 220 V, DC shunt motor
N = 500 rpm, Ia = 50 A, ra = 0.2Ω.
21. In the Hopkinson’s test on two DC machines,
For the first condition,
machine A has a field current of 1.4 A and machine B
has a field current of 1.3 A. Which machine acts as Eb = Ka fN +Ia ra
a generator? 220 = Ka fN + 50 × 0.2
(a) Both machines act as motors 220 − 10 210
(b) Both machines act as generators or, Ka f = =
500 500
(c) Machine A is working as generator and machine
B as motor Now, torque Te = ka fIa
(d) Machine A is working as motor and machine B Since torque ∝ Ia; when torque is doubled, Ia is also
as generator doubled.
Ans. (c) Thus, if the new speed is N1, we have
220 = Ka fN1 + 100 × 0.2
22. A four-pole lap wound DC shunt generator has
300 armature conductors and a flux/pole of 0.1 Wb.
500
It runs at 1000 rpm. The armature and field resis- N1 = (220 − 20) = 476.19 rpm
tances are 0.2 W and 125 W, respectively. Calculate 210
the terminal voltage when it is loaded to take a Ans. (a)
load current of 90 A.
24. Determine the torque developed by the armature
(a) 431.8 V (b) 481.2 V of a 4-pole DC motor having 774 conductors, two
(c) 452.3 V (d) 437.5 V paths in parallel, 24 mWb of pole flux and whose
armature current is 50 A.
Solution:  For the given lap wound DC shunt
(a) 224.6 Nm (b) 243.2 Nm
generator, number of poles p = 4, Z = 300,
f = 0.1 Wb/pole, N = 1000 rpm ra = 0.2 W and
(c) 295.36 Nm (d) 272.34 Nm
rf = 125 W. Solution:  For the given motor, p = 4, Z = 774,
I1 a = 2, f = 24 × 10-3 Wb, Ia = 50 A
pfNZ pfw æ pZ ö
Ia I2 = 90 A Z =ç ÷ fw
I0 Eb = =
a 2pa è 2pa ø
ra The emf constant (in SI units) is given by
rf = 125 W
Ea pZ p × 774
Ka = = = 246
2pa 2p × 2

Chapter 4.indd 319 3/23/2016 11:31:49 AM


320     Chapter 4:  Electrical machines 

So, For motor:


Te = Ka fIa 20 A
= 246 × (10−3 × 24) × 50
ra
1.0 Ω
= 295.6 Nm
Vt = 220 V
Eb(m)
Ans. (c)
25. A 200 V DC shunt motor running at 1000 rpm takes
an armature current of 17.5 A. It is required to
reduce the speed to 600 rpm. What must be the Vt = Eb(m)- Iara = 220 - 1.0 × 20 = 200 V
value of resistance to be inserted in the arma- For generator:
ture circuit if the original armature resistance is
0.4 W? 20 A
(a) 3.6 W (b) 4.4 W
(c) 5.2 W (d) 6.3 W ra
1.0 Ω Vt = 220 V
Solution:  Given that: ra = 0.4 W, V = 200 V,
N = 1000 rpm, Ia = 175 A. Eb(g)
Then,
Eb = Ka fN +Ia r Vt = Eb(g) - Iara = 220 + 20 × 1.0 = 240 V
200 = Ka fN + 17.5 × 0.4 Difference = 240 - 200 = 40 V
In generator,
200 − 17.5 × 0.4 Eb(g) - Eb(m) = (Vt + Iara) - (Vt - Iara)
or, Ka f =
193
= = 2Iara = 2 × 20 × 1 = 40 V
1000 1000
Ans. (c)
Now, let the resistance required to be inserted
27. A DC series motor drives a load, the torque of
be R
which varies as the square of the speed. Assume
motor resistance to be negligible, estimate the per-
R centage reduction in the motor terminal voltage
ra
Vt which will reduce the motor speed to half the value
it has on full voltage.
Eb
(a) 50%   (b) 75%   (c) 25%   (d) 60%

Solution:  Given that


New speed N1 = 600 rpm
So, Tl ∝ w 2 , ra ≈ 0

200 = Ka fN1 + 17.5 × ( 0.4 + R ) Vt


w=
Ka f
 193 
 200 − × 600

− 0.4 = 4.42 Ω For series motor: f ∝ Ia and Te ∝ Ia 2
1000
R=
17.5
Vt
Ans. (b) Then, w =
Ka × K Te
26. A 220 V DC has armature resistance of 1 W. If
w2
the full load current is 20 A, the difference in the
induced voltages when the machine is running as a So, the voltage ratio = = 0.5 = 0.75
motor and a generator is w1

(a) 20 V   (b) 0 V   (c) 40 V   (d) 50 V Ans. (b)
28. A 6-pole, 148 A DC shunt generator has 480 con-
Solution:  Given that: V = 220 V DC, ra = 10 W, ductors and is wave wound. Its field current is 2 A.
Ia = 20 A Find the demagnetising and cross-magnetising

Chapter 4.indd 320 3/23/2016 11:32:06 AM


SOLVED EXAMPLES     321

ampere turns per pole at full load if the brushes Fd


are on geometric neutral axis (GNA). Ff
(a) 0, 3000 ATs/pole (b) 3000, 0 ATs/pole
(c) 0,1500 ATs/pole (d) 1500, 0 ATs/pole 5°

Solution:  For the given shunt generator: number


of poles p = 6, Ia = 148 A, Z = 480 and If = 2 A.
Shunt brush is at MNA, so, demagnetising ampere
turns (AT) = 0

Fa Fc

where Fd is demagnetising ampere turns (AT) and


Fc is cross-magnetising ampere turns.
 pZa   2p − Zap 
Cross-magnetising AT = 2960 −   × 2960 =   296
Fd  2p   2p
 pZa   2p − Zap 
2960 −  × 2960 = 
Ff
 2p   2960 = 2466 AT
GNA
 2p
I a Z 2b
Then, demagnetising AT = = 493 AT
2a × 360°
(as a = 2 for wave winding)
Ans. (b)
30. A 500 V DC shunt motor draws a line current of
Cross magnetising ampere turns 5 A on light-load. If the armature resistance is
0.15 W and field resistance is 200 W, determine the
æ 480 ö Ia 480 148 efficiency of the machine running a generator deliv-
=ç ÷× = × = 2960 AT
è ø 2×6
ering load current of 40 A.
2 ap 2
(a) 72.67% (b) 78.73%
Ans. (a) (c) 87.83% (d) 92.33%
29. For data in Question 29, find the demagnetising
and cross-magnetising ampere turns per pole at full Solution:  The give motor can be represented as
load if the brushes are shifted from GNA by 5° follows
electrical.
(a) 166.67, 2833.33 (b) 493.87, 2500 If 5A
(c) 166.67, 3000 (d) 123.87, 2500 0.15 Ω
rf = 200 Ω 500 V
Solution:  The given condition can be represented
as follows:

rical Given that ra = 0.15 W, rf = 200 W


lect
5° e Brush
b= axis From the diagram,
500
If = A = 2.5 A and Ia = 5 - 2.5 = 2.5 A
200
The input power at high load = 500 × 5 = 2500 W
So, core loss = Input power - Field Cu loss -
Armature Cu loss
= 2500 - (2.5)2 × 200 - (2.5)2 × 0.15
= 2500 - 1250 - 1 ≈ 1249 W
GNA Now, when running on generator, the circuit diagram
is represented as

Chapter 4.indd 321 3/23/2016 11:32:15 AM


322     Chapter 4:  Electrical machines 

So, the speed at which the losses will be halved =


2.5 A 40 A
9.5 rps
Ans. (b)
200 Ω
500 A
32. A 4-pole lap wound DC generator has a developed
power of P watts and voltage of E volts. Two
adjacent brushes of the machine are removed
Power output Po = 500 × 40 = 20 kW as they are worn out. If the machine operates
Ia = 40 + 2.5 = 42.5 A with the remaining brushes, the developed volt-
age and power that can be obtained from the
Po
h= machine are
Po + losses
(a) E and P (b) E/2 and P/2
20 × 103 (c) E and P/4 (d) E and P/2
=
20 × 103 + 1249 + 1250 + 42.5 × 0.15
Solution:  As the number of poles of stator and
= 87.83%
rotor must be the same, two brushes are removed
31. The hysteresis and eddy current losses in a DC such that two parallel paths become open to cur-
machine running at 1000 rpm are 250 W and 100 W, rent. The machine operates on E and I/4. So,
respectively. If the flux remains constant, at what developed voltage is E and power is E × I/4 =
speed will the total losses be halved? P/4.
(a) 8.5 rpm (b) 9.5 rpm Ans. (c)
(c) 10.5 rpm (d) 11.5 rpm
33. Match List I (Type of machine) with List II
Solution:  We know that (Characteristic/application) and select the correct
answer:
Ph = kh nBmx , Pe = ke n2 Bm2 ,
List I
where n is the speed of the machine in rps. A. DC slant generator
1000 50 B. DC series motor
For speed of 1000 rpm, n1 = = rps
60 3 C. Level compounded DC generator
Therefore, D. DC series generator
List II
= kh¢
50 50
250 = kh × Bmx
3 3 1. Electric traction
250 × 3
Þ kh¢ =
2. Has good voltage regulation
= 15
50 3. Must have residual flux
Also, 4. Used as boosters

æ 50 ö æ 50 ö
2 2
100 = ke × ç ÷ Bm2 = ke¢ ç ÷ A  B  C  D
è 3 ø è 3 ø (a) 1   1   2   3
100 × 9 (b) 3   2   1   2
Þ ke¢ =
9
=
50 × 50 25 (c) 4   2   1   3
(d) 3   1   2   4
Let at n2 rpm total losses be halved, then
Ans. (d)
kh¢ n2 + ke¢ n22 = (100 + 250 ) = 175
1
2 Induction Machines
Then
34. A three-phase wound rotor induction motor, when
9 2
n + 15n2 − 175 = 0 stated with load connected to its shaft, was found
25 2 to start but settle down at about half synchronous
9 speed. If the rotor winding as well as the stator
−15 ± 152 + 4 × × 175 winding were star connected, then the cause of the
25
n2 = = 9.5 malfunctioning could be attributed to
9

25

Chapter 4.indd 322 3/23/2016 11:32:25 AM


SOLVED EXAMPLES     323

(a) one of the stator phase windings being short- 38. What will be the torque exerted by 8-pole, 50 Hz, three
circuited phase induction motor operating with a 4% slip which
(b) one of the supply fuses being blown develops a maximum torque of 250 kg m at a speed of
(c) one of the rotor phases being open-circuited 680 rpm. Resistance per phase of rotor is 0.5 W.
(d) two of the rotor phases being open-circuited
(a) 91 kg m (b) 181 kg m
Ans. (c)
(c) 271 kg m (d) 121 kg m
35. A three-phase induction motor fed on a three-
phase voltage controller is suitable for driving Solution:  In a 8 pole machine, the synchronous
loads whose torque is speed is
(a) constant irrespective of speed 120 × 50
= 750 rpm
(b) inversely proportional to the square of speed 8
(c) directly proportional to the square of speed Slip at maximum torque condition
(d) inversely proportional to speed
Ans. (c) smT = 750 − 680 = 0.0933
750
36. A stator of 6-pole, three phase induction motor is
fed from a three phase 50 Hz supply which contains Slip at full load sfl = 0.04 and maximum torque
a pronounced fifth time harmonic. The speed of the Tmax = 250 kg m. Therefore
fifth space harmonic field produced by fifth time Te(fl) 2 2
harmonic in the stator supply will be = = ;
Tmax smT s 0.0933 0.04
(a) 200 rpm (b) 1500 rpm + fl +
sfl smT 0.04 0.0933
(c) 1000 rpm (d) None of these
Te = 181 kg m
Solution:  Ns for fundamental of 50 Hz for 6-pole Ans. (b)
machine is 39. A 10 kW, 50 Hz, three phase, 4-pole induction motor
120 × 50 develops rated torque at 1460 rpm. In case load torque
= 1000 rpm is reduced to half, the power output is (assume linear
6
torque slip characteristics near operating range)
Then for 5th time harmonic, the angular speed of
space harmonic field by fifth time harmonic is (a) 5000 W (b) 1470 W
(c) 5068 W (d) 5034 W
5 × 1000
= 1000 rpm
5 Solution:  Synchronous speed
Ans. (c) 120f 120 × 50
Ns = = = 1500 rpm
37. A starting torque of 80 Nm is developed in an p 4
induction motor by an autotransformer starter 1500 − 1460
with a tapping of 30%. If tapping of autotrans- sfl = = 0.04
1500
former starter is 60%, starting torque will be
Tc
(a) 40 Nm (b) 160 Nm
(c) 240 Nm (d) 320 Nm Trated

Solution:  Given that: x = 0.3 and Te1 = 80 Nm. Trated/2


Since

Te ∝ x2 Te(fl)
1460 1500 N
rpm
80
So, Te(fl) =
0.32 The torque speed (linear) change is given by,
Now for 60% tap, x = 0.6. So, Te − Te (rated) N − 1460
=
Te (rated) − 0 1460 − 1500
æ 0.6 ö
2
Te = ç ÷ 80 = 320 Nm
è 0.3 ø  N 1460 
Te =  − + T + Te (rated)
Ans. (d)  40 40  e (rated)

Chapter 4.indd 323 3/23/2016 11:32:39 AM


324     Chapter 4:  Electrical machines 

Te (rated) PFW = 1% of 10 kW = 100 W


At Te = Pstator = 600 W
2 Slip at maximum torque,
Te (rated)  N 73  1000 − 800
=  − +  Te (rated) + Te (rated)
2  40 2 smT =
1000
= 0. 2
N 73 1 Rotor input power at full load,
or, = +
40 2 2    Pg = PSh + PF&W + Pcu(r)
⇒ N = 1480 rpm Pcu(r) = sPg = 0.07 Pg
Then,
 1460 
P1 =  × 2p  × Te (rated) = 10 kW 10 × 103 + 100 + 0.07 Pg = Pg
 60 
Pg = 10860.2 W
 1480  Te (rated)
P2 =  × 2p  ×
 60 
Ans. (b)
2
42. In the induction motor mentioned in Question 41,
P 1480 with rated voltage at starting, the starting torque is
So, 2 = ⇒ P2 = 5068
P1 1460 × 2 (a) 103.71 Nm (b) 165.9 Nm
Ans. (c) (c) 64.8 Nm (d) 63.3 Nm
40. The rotor power output of a three phase induction
motor is 15 kW and the corresponding slip is 4%. Solution:  We know that
The rotor ohmic losses are Tst 2
= and
(a) 600 W (b) 625 W Tmax 1
+ smT
(c) 650 W (d) 700 W smT
Tfl 2
Solution:  Given that rotor power output is 15 kW. =
Tmax sfl s
Thus, + mT
smT sfl
(Pg - Pcu(r)) = 15 kW
Therefore, substituting values, we get
where Pcu(r) = rotator Cu losses = sPg and Pg is
the air-gap power. 0.07 0.2
Tst +
= 0 . 2 0 .07
Pg − sPg = 15 kW Tmax 1
+ 0.2
0.2
15
Pg = = 15.625 kW 3.2
1 − 0.04 = = 0.677
5.2
So, Pcu(r) = 0.04 × 15.625 kW = 0.625 kW = 625 kW Also,
Ans. (b) Tst = 0.617 Tfl ,
41. A three phase 50 Hz, 6-pole induction motor 10 × 103 × 60
Tfl = Nm = 102.63 Nm
2p × 930
has shaft power output of 10 kW at full load at
930 rpm. Friction and windage loss amount to 1%
of output, total stator losses are 600 W. If maxi- Therefore,
mum torque is developed at 800 rpm, then, rotor Tst = 0.617 × 102.63 = 63.3 Nm
input at full load will be Ans. (d)
(a) 10100 W (b) 10860.2 W 43. A three phase induction motor has 4-pole star con-
(c) 860.2 W (d) 11460.2 W nected stator winding and runs on 400 V, 50 Hz
supply. The rotor resistance is 0.1 W and reactance
Solution:  Given that shaft power, Psh = 10 kW 0.9 W. The stator to rotor turns ratio is 1.75 and the
at 930 rpm full load slip is 5%. The maximum torque will be
120 × 50 (a) 365.7 Nm (b) 184.8 Nm
Synchronous speed Ns = = 1000 rpm
60 (c) 61.56 Nm (d) 99.5 Nm
1000 − 930 Solution:  Given that: rr = 0.1 W, Xlr = 0.9 W,
Slip s = = 0.07
1000 sft = 0.05

Chapter 4.indd 324 3/23/2016 11:32:52 AM


SOLVED EXAMPLES     325

We have (a) 20% (b) 80%


rr (c) 2.5% (d) data Insufficient
smT = =
0.1
Xrl 0.9
Solution:  Let the voltage be V, then the reduced
rf/smT voltage is = 0.9 V.
We know that
I2 jX1r
Te ∝ V 2
Vf
which implies,

Equivalent rotor voltage (per phase) Te ∝ (0.9)2 V 2 = 0.81V 2


400 1
×
So, torque is reduced by 19%
= = 131 V
3 1.75 Ans. (a)
Rotor current at maximum torque and Ns = 1500 rpm; 46. Consider the following statements regarding speed
131 control of induction motors by means of external
Ir = = 145.73 A
rotor resistors:
0.12 + 0.92
1. Reduction in speed is accompanied by reduced
3 2 Rr
Te max = I efficiency
w s r smT 2. With a large resistance in the rotor circuit, the
3 æ 0.1 ö speed would vary considerably with variation in
= × (145.7)2 × ç × 0.9 ÷
2p × 1500 è 0.1 ø torque.
60 3.  The method is very complicated.
= 365 Nm The disadvantages of such a method of speed con-
trol would include
(Resistance and reactance are considered with ref-
(a) 1 and 2 (b) 2 and 3
erence to the rotor side.)
(c) 1 and 3 (d) 1, 2 and 3
Ans. (a)
Ans. (a)
44. A three phase, 1 hp, 50 Hz, 16-pole induction
motor has a rotor impedance of (0.02+j 0.15) W at Synchronous Machines
standstill. Full load torque is obtained at 360 rpm.
Calculate the ratio of maximum to full load torque. 47. An ideal synchronous motor has no starting torque
because the
(a) 1.818    (b) 2.818    (c) 4.238    (d) 1
(a) rotor is made up of salient poles
Solution:  For 16 pole 50 Hz machine, (b) relative velocity between the stator and the
120 × 50 rotor mmf’s is zero
Ns = = 375 rpm (c) relative velocity between stator and rotor mmf ’s
16
is not zero
375 − 360
sfl = = 0.04 (d) rotor winding is highly reactive
375 Ans. (c)
r 0.02
smT = r = = 0.1133 48. The zero power factor characteristic for the Potier’s
Xlr 0.15 diagram can be obtained by loading the alternator
So, using
Te(fl) 2 2 (a) lamp load (b) synchronous motor
= = = 0.55 (c) water load (d) DC motor
Tmax smT s 0.133 0.04
+ fl + Ans. (b)
sfl smT 0.04 0.133
Tmax 49. For the maximum current during slip test on a
= 1.876 synchronous machine, the armature aligns along
Te(fl)
(a) direct-axis
Ans. (a) (b) quadrature-axis
45. If voltage of an induction motor is reduced by 10%, (c) 45° to d-axis
torque is reduced by (d) 45° to q-axis
Ans. (b)

Chapter 4.indd 325 3/23/2016 11:33:02 AM


326     Chapter 4:  Electrical machines 

50. A 6-pole, 50 Hz, three-phase synchronous motor Armature voltage ∝ Nf


and an 8-pole, 50 Hz, three-phase slip ring induc-
tion motor are mechanically coupled and operate ∝ 2N 4f
on the same three-phase, 50 Hz supply system. If ∝ 8Nf
they are left open-circuited, then the frequency
Ans. (d)
of the voltage produced across any two slip rings
would be 52. A synchronous motor with negligible armature
resistance runs at a load angle of 20° at the rated
(a) 16.6 Hz (b) 25.0 Hz frequency. If supply frequency is increased by 10%,
(c) 37.5 Hz (d) 50.0 Hz keeping other parameters constant, the new load
angle will be
Solution:  The 6-pole 50 Hz, three phase synchro-
nous motor and 8-pole 50 Hz, three-phase slip ring (a) 16°    (b) 18°     (c) 20°    (d) 22°

Solution:  Given that d = 20° and if f1 is the ini-


motor can be represented as follows.
tial supply frequency, then frequency after 10%
increase is f2 = 1.1f1. Also,
XS2 = 1.1 XS1 and
E0 Vt E V
P = sin d1 = P2 = 0 t sin d 2
SM IM XS1 XS2
Therefore,
f
XS 2
sin d 2 = sin 20° = 1.1 sin 20°
120 × 50
XS 1
120f
Shaft speed =
p
=
6
= 1000 rpm d 2 = 22°.
Ans. (d)
Rotating field speed of induction motor = 53. The result of a slip test for determining direct-axis
120f 120 × 50 (Xd) and quadrature-axis (Xq) reactances of a star-
= = 750 rpm connected salient-pole alternator are the following
p 8
phase values: Vmax = 108 V,Vmin = 96 V, Imax =
(1000 − 750)8 12 A, Imin = 10 A. Hence, the two reactances will be
Therefore, slip frequency = = 16.6 Hz
(a) Xd = 10.8 Ω and Xq = 8 Ω
120
Ans. (a) (b) Xd = 9 Ω and Xq = 9.6 Ω
(c) Xd = 9.6 Ω and Xq = 9 Ω
51. If the dimensions of all the parts of a synchronous (d) Xd = 8 Ω and Xq = 10.8 Ω
generator, and the number of field and armature
turns are doubled, then the generated voltage will Solution:  We have
change by a factor of V Vmax
Xq = min and Xd =
(a) 1     (b) 2     (c) 4     (d) 8 I max I min

Solution:  Field turn doubled Therefore, substituting values, we have

= 10.8 W, Xq = = 8.0 W
108 96
AT 2 × AT1 AT Xd =
Field flux = = =4 1 10 12
R 2l1 R1 Ans. (a)
4 mA1

PRACTICE EXERCISES

Set 1 (One Mark Questions) each of 0.004 Ω resistance. Calculate armature


resistance.
1. A 4-pole lap wound DC shunt generator has
(a) 0.055   (b) 0.0275   (c) 0.110   (d) 0.22
an armature winding consisting of 220 turns

Chapter 4.indd 326 3/23/2016 11:33:16 AM


PRACTICE EXERCISES     327

2. The Eb/V ratio of a DC motor is an indication of its (1) is independent of value of rotor circuit resistance
(2) is directly proportional to square of supply
(a) efficiency (b) speed regulation
voltage
(c) starting torque (d) running torque
(3) occurs at a slip whose value is independent of
3. The feature of 4-point starter that is the key the value of rotor resistance
advantage over 3-point starters, in starting of a Of these statements, the correct ones are
DC shunt motor is (a) 1, 2 and 3 (b) 1 and 2
(a) provision of protection against over voltages (c) 2 and 3 (d) 1 and 3
in input 9. In a transformer fed from a fundamental frequency
(b) speed control by field weakening method can voltage source, the source of harmonics is the
be done
(c) speed control by armature voltage variation can (a) overload
be done (b) poor insulation
(d) None of the above (c) iron loss
(d) saturation of core
4. Compensating windings are used to rectify the flux
density waveform at 10. If a 230 V DC series motor is connected to a 230 V
AC supply, the
(a) regions under main pole
(b) inter-polar region (a) motor will vibrate violently
(c) throughout the air-gap (b) motor will run with less efficiency and more
(d) None of these sparking
(c) motor will not run
5. A three phase slip ring induction motor is wound (d) fuse will be blown
for 4 poles on stator and 6 poles on rotor. When
three-phase balanced supply at 50 Hz is given to 11. The interpoles in DC machines have a tapering
the motor, it will shape in order to

(a) run at 1500 rpm (a) reduce the overall weight


(b) run at a speed less than 1500 rpm (b) reduce the saturation in the interpole
(c) run at 1000 rpm (c) economise on the material required for inter-
(d) not run poles and their windings
(d) increase the acceleration of commutation
6. The speed of a separately-excited DC motor is
varied by varying the armature voltage in the range 12. If two 8-pole machines of identical armatures are
zero to base speed and by varying the field current wound one with lap winding and the other with
above the base speed. It is suitable for constant wave winding, then

(a) torque drive at all speeds (a) w ave wound machine will have more rated cur-
(b) power drive at all speeds rent and more voltage
(c) torque drive till base speed and constant power (b) lap wound machines will have more rated volt-
drive beyond base speed age and more current
(d) power drive till base speed and constant torque (c) lap wound machine will have more rated volt-
drive beyond base speed age and less current
(d) wave wound machine will have more rated
7. Which of the following test(s) is/are done for pre- voltage and less current
paring circle diagrams of induction motor? 13. Consider the following statements: At starting, the
(1) No load test field excitation of a DC shunt motor is kept at its
(2) Blocked rotor test maximum value to reduce
(3) Voltmeter—ammeter test for DC resistance 1. sparking at brushes
2. acceleration time
(a) 1 only (b) 1 and 2
3. starting current
(c) 2 and 3 (d) All of them
4. voltage dip in supply
8. A three phase slip ring induction motor with neg- of these statements
ligible stator impedance operates on a balanced (a) 1 and 2 are correct
three phase AC supply. Consider the following (b) 2, 3 and 4 are correct
statements in this regard. The maximum torque (c) 1, 3 and 4 are correct
developed by the motor (d) 1, 2, 3 and 4 are correct

Chapter 4.indd 327 3/23/2016 11:33:16 AM


328     Chapter 4:  Electrical machines 

14. If two induction motors A and B are identical (c) s lip at maximum torque is proportional to
except that the air-gap of motor A is 50% greater frequency
than that of motor B then (d) starting torque is proportional to frequency
(a) the no load power factor of A will be better 21. In a DC machine, the armature is always on the
than that of B rotor unlike an AC machine where it could be on
(b) the no load power factor of A will be poorer either the stator or rotor, because
than that of B
(c) the core losses of A will be more than those of B (a) c ommutation action would otherwise not be
(d) the operating flux of A will be smaller than possible
that of B (b) armature reaction demagnetising action would
15. Full-load voltage regulation of a power transformer otherwise be more
is zero, when power factor of the load is near (c) otherwise it would not be possible to place com-
pensating winding in pole shoes
(a) unity and leading (b) zero and leading (d) self-starting of DC motor would not be possible
(c) zero and lagging (d) unity and lagging otherwise
16. If the speed of a DC motor increases with load 22. Full load voltage regulation of a power transformer
torque, then it is a is zero, when power factor of the load is neat
(a) series motor (a) unity and leading
(b) permanent magnet motor (b) zero and leading
(c) differentially compounded motor (c) zero and lagging
(d) cumulatively compound motor (d) unity and lagging
17. The speed of a three-phase induction motor is con-
23. A series motor drawing an armature current of Ia is
trolled from 1 pu to 2 pu using a variable frequency
operating under saturated magnetic conditions. The
inverter. The equivalent circuit parameter most
torque developed in the motor is proportional to
likely to vary is
I I
(a) stator leakage inductance (a)       (b)  2      (c) I a2    (d) Ia
(b) rotor leakage inductance Ia Ia
(c) magnetising inductance
(d) core loss resistance 24. Which one of the following methods would give
a higher than actual value of regulation of an
18. In a three-phase transformer shown in the figure, alternator?
the phase displacement of secondary line voltages
with corresponding primary line voltages will be (a) ZPE method (b) MMF method
(c) EMF method (d) ASA method
(a) Zero (b) 30° lag
(c) 30° lead (d) 180° 25. If a sinusoidal voltage source is connected to a
power transformer, its no-load current would be
19. The three-phase transformers each rated at
75 MVA, 132 KV/1 kV, have the following different (a) sinusoidal and lagging the voltage by 90°
methods of cooling: (b) sinusoidal and lagging the voltage by less than 90°
(c) rich in third harmonic and its fundamental
1. self-oil cooled would lag the voltage by 90°
2. forced oil cooled (d) rich in third harmonic and its fundamental
3. forced air cooled would lag the voltage by less than 90°
The correct sequence in ascending order in terms of
26. Slip test is performed to determine
the weights of these transformers is
(a) 1, 2, 3 (b) 2, 3, 1 (a) slip
(c) 3, 1, 2 (d) 3, 2, 1 (b) direct axis and quadrature axis reactances
20. In the case of a converter—inverter speed con- (c) positive sequence reactance and negative sequence
trol arrangement for an induction motor operat- reactance
ing with v f constant and with negligible stator (d) sub-transient reactance
impedance, the 27. A turbo alternator set feeds power to a three-phase
(a) maximum torque is independent of frequency constant-voltage constant-frequency bus. If the
(b) maximum torque is proportional to frequency steam supply to the set in cut off, then the set will

Chapter 4.indd 328 3/23/2016 11:33:20 AM


PRACTICE EXERCISES     329

(a) continue to run at rated speed in the same 34. Consider the following statements:
direction For a level compounded DC generator to run at
(b) continue to run at a reduced speed in the same constant speed, the series field mmf must effec-
direction tively compensate
(c) run at rated speed in the reverse direction
1. armature reaction mmf
(d) come to a stop
2. armature resistance voltage drop
28. A single-phase induction regulator is a constant 3. brush contact voltage drop
input transformer to obtain smooth variation of
the output voltage by varying the Which of these statements is/are correct?
(a) 2 alone (b) 1 and 2
(a) ratio of turns between primary and secondary (c) 1 and 3 (d) 1, 2 and 3
windings
(b) frequency 35. Zero sequence impedance of a three-phase cage
(c) flux density in the core induction motor is due to
(d) angle between the magnetic axes of the primary (a) stator circuit 3rd time harmonic
and secondary windings (b) stator circuit 3rd space harmonic
29. An auto-transformer having a transformation ratio of (c) rotor circuit 3rd time harmonic
0.8 supplies a load of 10 kW. The power transferred (d) rotor circuit 3rd space harmonic
inductively from the primary to the secondary is 36. A three-phase induction motor is used as an adjust-
(a) 10 kW (b) 8 kW able speed drive from zero to 2.0 per unit speed using
(c) 2 kW (d) Zero a variable frequency inverter. In the speed range of
1.0 to 2.0 per unit, in order to ensure satisfactory
30. The armature mmf of a DC machine has operation which one of the following sets of quanti-
(a) triangular space distribution and rotates at the ties is to be maintained approximately constant?
speed of armature (a) Voltage and power
(b) trapezoidal space distribution and is stationary (b) Flux and torque
in space (c) Voltage and torque
(c) stepped distribution and rotates at the speed of (d) Flux and power
the armature
(d) triangular space distribution and is stationary 37. In the V curve shown in the figure below for a syn-
in space chronous motor, the parameter of x-and y-coordinates
are, respectively,
31. An ideal synchronous motor has no starting torque
because the y
(a) rotor is made up of salient poles
(b) relative velocity between the stator and the
rotor mmf’s is zero
(c) relative velocity between stator and rotor mmf’s
is not zero
(d) rotor winding is highly reactive x
32. As compared to direct-on-line starting, a cage
(a) armature current and field current
induction motor with star-delta starting shall have
(b) power factor and field current
(a) more starting torque (c) armature current and torque
(b) more starting time (d) torque and field current
(c) reduced starting current
38. In an autotransformer of voltage ratio V1/V2 and
V1 > V2, the fraction of power transferred induc-
(d) smoother acceleration
33. The phenomenon of crawling in a three-phase cage tively is
induction motor may be due to
V V
(a) unbalanced supply voltages (a)  1 (b)  2
V1 + V2 V1
(b) 7th space harmonic of air-gap field
(c) 7th time harmonic of voltage wave V − V2 V − V2
(c)  1 (d)  1
(d) 5th space harmonic V1 + V2 V1

Chapter 4.indd 329 3/23/2016 11:33:26 AM


330     Chapter 4:  Electrical machines 

39. A three-phase slip-ring induction motor has chopper- 45. In a shaded-pole induction motor, the rotor runs
­controlled resistance in the rotor circuit. Its power from the
factor and efficiency can be characterised, respec-
(a) s haded portion to the unshaded portion of the
tively, as
pole while the flux in the former leads that of
(a) good and poor the latter
(b) poor and good (b) shaded portion to the unshaded portion of the
(c) good and good pole while the flux in the former lags that in
(d) poor and poor the latter
(c) unshaded portion to the shaded portion while
40. A three-phase slip-ring induction motor when the flux in the former leads that in the latter
started picks up speed but runs stably at about (d) unshaded portion to the shaded portion while
half the normal speed. This is because of the flux in the former lags that in the latter
(a) unbalance in the supply voltages 46. Generally, the no-load of an electrical machine is
(b) non-sinusoidal nature of the supply voltage represented in its equivalent circuit by a
(c) stator circuit asymmetry
(d) rotor circuit asymmetry (a) parallel resistance with a low value
(b) series resistance with a low value
41. Which of the following are the features of a shaded (c) parallel resistance with a high value
pole motor? (d) series resistance with a high value
1. Salient-pole stator 47. When the excitation of normally operating
2. Uniform air-gap unloaded salient-pole synchronous motor suddenly
3. Two stator windings one of which is a short- gets disconnected, it continues to run as a
circuited ring
4. Squirrel-cage rotor (a) schrage motor
(b) spherical motor
Select the correct answer using the codes given:
(c) switched-reluctance motor
(a) 1 and 4 (b) 2 and 4 (d) variable-reluctance motor
(c) 1, 3 and 4 (d) 1, 2 and 4
48. The speed of a DC shunt motor may be varied by
42. A DC series motor is accidentally connected to sin- varying
gle-phase AC supply. The torque produced will be 1. field current
(a) zero average value 2. supply voltage
(b) oscillating 3. armature circuit
(c) steady and unidirectional Select the correct answer using the codes given:
(d) pulsating and unidirectional (a) 1, 2 and 3 (b) 1 and 2
43. The advantage of the double squirrel-cage induc- (c) 1 and 3 (d) 2 and 3
tion motor over single-cage rotor is that its 49. If the rotor power factor of a three-phase induction
(a) efficiency is higher motor is 0.866, the spatial displacement between
(b) power factor is higher the stator magnetic field and the rotor magnetic
(c) slip is larger field will be
(d) starting current is lower (a) 30°    (b) 90°    (c) 120°    (d) 150°
44. The DC shunt generator when driven without con- 50. A three-phase induction machine draws active
necting field winding shows an open circuit termi- power P and reactive power Q from the grid. If it is
nal voltage of 12 V. operated as a generator, P and Q will respectively be
When field winding is connected and excited, the
terminal voltage drops to zero because (a) positive and negative
(b) negative and negative
(a) field resistance is higher than critical resistance (c) positive and positive
(b) there is no residual magnetism in the field (d) negative and positive
circuit
(c) filed winding has got wrongly connected 51. A DC shunt motor having unsaturated magnetic
(d) there is a fault in armature circuit circuit runs at 1000 rpm with rated voltage. If the

Chapter 4.indd 330 3/23/2016 11:33:26 AM


PRACTICE EXERCISES     331

applied voltage is half of the rated voltage, the is terminated at time t0. The short-circuit current
motor will run at Is circulating in the armature will vary till the
machine comes to rest at time ts. Which one of the
(a) 2000 rpm (b) 1000 rpm
curves shown in the following figures best reflects
(c) 750 rpm (d) 500 rpm
this variation?
52. The rotor power output of a three-phase induction
motor is 15 kW and the corresponding slip is 4%. (a) Is
The rotor copper loss will be
(a) 600 W (b) 625 W
(c) 650 W (d) 700 W
53. The current drawn by a 220 V DC motor of arma- t0
t ts
ture resistance 0.5 Ω and back emf 200 V is
(a) 40 A   (b) 44 A    (c) 400 A    (d) 440 A (b) Is

54. Match the test mentioned in List I with the charac-


teristics mentioned in List II that it determines.
List I
t0
A. Voltage drop test t ts
B. Hopkinson’s test
(c) Is
C. Swineburne’s test
D. Retardation test
List II
1. Efficiency
t0
2. Separation of iron and friction losses t ts
3. Open-and short-circuited armature coil
4. Temperature rise (d) Is

A B C D
(a) 3 1 4 2
(b) 3 4 1 2
(c) 3 4 1 2 t0
t ts
(d) 2 1 4 3

3. A synchronous motor is running from an infinite


Set 2 (Two Marks Questions) bus of voltage Vt in steady-state, at about 50%
of its rated load with a power-angle d1, between
1. The windings of a Q kVA, V1/V2 volts, three- Vt and Ef (induced voltage). The load is suddenly
phase delta-delta (Dd) core type transformer are decreased to 25%. Ef attains its new steady-state
reconnected to work as a single-phase transformer, power-angle d2 with Vt by initially
the power rating and the maximum voltage ratio
of the new configuration will be, respectively, (a) falling behind and making a complete rotation
(b) advancing and making a complete rotation
(a) Q/3 kVA and V1/V2 volts
(b) 3Q kVA and 3V1/3V2 volts (c) falling behind, followed by oscillation about d2
(c) 2Q kVA and 3V1 / 3V2  volts (d) advancing, followed by oscillation about d2
(d) 2/3Q kVA and 2V1/2V2 volts 4. As compared to the use of a single series DC motor
for electric traction for a given starting time t, the
2. A three-phase alternator with high rotational iner-
series—parallel control using two similar motors
tia and negligible armature resistance was deliver-
with time t/2 for each series and parallel operation
ing power to an isolated load when the armature
would give a saving in starting energy of
terminals got short-circuited. After a while, the
mechanical power input to the shaft of the machine (a) 100%    (b) 50%    (c) 25%   (d) 75%

Chapter 4.indd 331 3/23/2016 11:33:33 AM


332     Chapter 4:  Electrical machines 

5. The low-voltage winding of a core-type transformer 11. A 10 kVA, 400 V/200 V single-phase transformer
is subdivided into two equal halves, each of half the with a resistance of 3% and a reactance of 6% is
original width of the single winding with the high- supplying a current of 50 A to a resistive load. The
voltage winding in between these two halves of voltage across the load is
low-voltage winding (instead of having usual con-
(a) 194 V (b) 196 V
struction of low-voltage winding adjacent to the
(c) 198 V (d) 390 V
core and surrounded be the high-voltage winding).
Such an interlacing of coils would make the com- 12. A 40 kVA transformer has a core loss of 400 W and
bined primary and secondary leakage reactance a full-load copper loss of 800 W. The proportion of
(in terms of the primary) nearly full-load at maximum efficiency is
(a) twice (b) equal (a) 50%   (b) 62.3%   (c) 70.7%   (d) 100%
(c) half (d) one-fourth
13. A single-phase transformer has a rating of 15 kVA,
6. A three-phase slip-ring induction motor is wound 600/120 V. It is reconnected as an auto-transformer
for 4 poles on stator and 6 poles on rotor. When to supply at 720 V from a 600 V primary source.
three-phase balanced voltage source at 50 Hz is The maximum load it can supply is
applied to the motor, it will run at:
(a) 90 kVA (b) 18 kVA
(a) 1500 rpm (b) 1000 rpm (c) 15 kVA (d) 12 kVA
(c) 750 rpm (d) Zero speed
14. Two single-phase transformers with turn ratios 1
7. If Pcore and Psc represent core and full-load ohmic and 2, respectively, are connected to a three-phase
losses, respectively, the maximum kVA delivered supply on the primary side as shown in the figure.
to load corresponding to maximum efficiency is The voltmeter V2 will read
equal to rated kVA multiplied by
(a) Pcore /Psc (b)  Pc /Psc 1:1
A
(c) (Pcore /Psc ) (d) (Psc /Pcore )
2 2
100 Ð 0°
8. If the applied voltage of a certain transformer is V1
B
increased by 50% and the frequency is reduced to
100 Ð 120°
50% (assuming that the magnetic circuit remains
unsaturated), the maximum core/flux density
will C
1:2
(a) change to three times the original value
(b) change to 1.5 times the original value
(c) change to 0.5 times the original value (a) 100 V   (b) 173 V   (c) 200   (d) 265 V
(d) remain the same as the original value 15. In a single-phase transformer, the magnitude of
9. When short-circuit test on a transformer is per- leakage reactance is twice that of resistance of both
formed at 25 V, 50 Hz, the drawn current I1 is at primary and secondary. With secondary short-­
a lagging power factor angle of f1. If the test is circuited, the input power factor is
performed at 25 V, 25 Hz and the draw current I2 is
at a lagging power factor angle of f2 then (a) 1 / 2    (b) 1/ 5  (c) 2/ 5    (d) 1/3

(a) I2 > I1 and f2 < f1 16. A three-phase induction motor is driving a content
(b) I2 > I1 and f2 > f1 torque load of 1 pu at 5% slip. It has a maximum
torque of 2 pu at 10% slip. Torque—speed ­variation
(c) I2 < I1 and f2 > f1 in stable zone is assumed to be linear. For the
(d) I2 < I1 and f2 < f1 stable operation of the motor, the minimum pu
supply voltage is approximately
10. The starting current of a three phase induction
motor is five times the rated current, while the (a) 0.25    (b) 0.50    (c) 0.707    (d) 0.80
rated slip is 40%. The ratio of starting torque to
17. A 200 V DC shunt motor delivers an output of
full-load torque is
17 kW with an input of 20 kW. The field winding
(a) 0.6    (b) 0.8      (c) 1.0   (d) 1.2 resistance is 50 Ω and armature resistance is 0.04 Ω.

Chapter 4.indd 332 3/23/2016 11:33:44 AM


ANSWERS TO PRACTICE EXERCISES     333

Maximum efficiency will be obtained when the which has a rotor current of frequency 2 Hz. The
total armature copper losses are equal to speed at which the motor operates is
(a) 2632 W (b) 3000 W (a) 1000 rpm (b) 960 rpm
(c) 3680 W (d) 5232 W (c) 750 rpm (d) 720 rpm
18. A three-phase alternator is connected to a delta-delta 20. A single-phase transformer when supplied from
transformer. The hysteresis and eddy current losses 220 V, 50 Hz has eddy current loss of 50 W. If the
of the transformer are, respectively, 300 W and 400 transformer is connected to a voltage of 330 V,
W. If the speed of the alternator is reduced by 10%, 50 Hz, the eddy current loss will be
then the hysteresis and eddy current losses of the
(a) 168.75 W (b) 112.5 W
transformer will be, respectively
(c) 75 W (d) 50 W
(a) 228 W and 262.44 W
21. A three-phase 50 MVA, 10 kV generator has a reac-
(b) 243 W and 324 W
tance of 0.2 ohm per phase. Hence the per-unit
(c) 243 W and 360 W
value of the reactance on a base of 100 MVA 25 kV
(d) 270 W and 400 W
will be
19. A 6-pole, three-phase alternator running at 1000 rpm
(a) 1.25   (b) 0.625    (c) 0.032    (d) 0.32
supplies to an 8-pole, three-phase induction motor

ANSWERS TO PRACTICE EXERCISES

Set 1 (One Marks Questions)

1. (a) The number of conductors = 220 × 2 = 440 29. (c) The given autotransformer can be represented as
(for winding a = 2).
I1 I2
Number of poles p = 4. Thus, armature resistance is

( 440/4 )
× 0.002 = 0.055 W V 0.8 V 10 kW
4
2. (b) 16. (c)
Given that
3. (b) 17. (d) VI1 = 0.8VI2
4. (a) 18. (b) I1
= 0.8 = K
5. (d) I2
19. (c)
6. (c) Inductively transferred power
20. (a)
7. (d) = (1 − K )P = (1 − 0.8)10 = 2 kW
21. (d)
8. (a) 30. (d)
22. (a)
9. (d) 31. (c)
23. (d) 32. (c)
10. (a)
24. (c) 33. (b)
11. (a)
25. (d) 34. (d)
12. (d)
26. (b) 35. (a)
13. (d)
27. (a) 36. (a) In the field weakening to between Ns and 2Ns,
14. (d)
the voltage and power is kept constant to ensure
15. (a) 28. (d) proper operation.

Chapter 4.indd 333 3/23/2016 11:33:49 AM


334     Chapter 4:  Electrical machines 

52. (a) Rotor copper losses are given by


P
P, T Pcu(r) = sPg
4
T = × 15 kW
100
= 600 W

ns 2ns 53. (a) The armature current drawn is given by


V − Eb
37. (a) Ia =
Ra
38. (d) We have
220 − 200
V2 Ia = = 40 A
K= 0.5
V1
54. (b)
Thus inductively transferred power
= (1 − K ) input power Set 2 (Two Marks Questions)
V V − V2
= 1− 2 = 1 1. (a) The given three phase transformer can be rep-
V1 V1
resented as
39. (a) 44. (c) Q kVA V 1/V 2 Dd
40. (a) 45. (c) I1
V1 V2
41. (c) 46. (c) IPh
42. (d) 47. (d)
43. (a) 48. (b)
49. (a) The circuit of the given induction motor can be
represented as For a three-phase transformer:
Xlr
P = 3V1I1
Ir = 3V1I ph = Q
Vr Rr/s
When reconnected to work as a single-phase trans-
former, then anyone of the three transformers can
Given that rotor power factor, be represented as below.
cos fr = 0.866 Iph
or, fr = 30°
V1 V2
30° = fr
60°
Ir P1 = V1IPh = Q/3

Im 2. Since the mechanical power input is disconnected


at t = t0. Thus the speed can fall linearly.
Assuming no stator impedance drop, Vs = Vr.
dw
= Te − TL
Therefore, approximate angle is 30°.
j
50. (d) Negative means delivery power active and posi- dt
tive means absorbing reactive power. TL
Since Te = 0 ⇒ w = − t
j
51. (b) We know that w =
V
, for unsaturated
Ka f Also during initial situation of the fault, the sub-tran-
magnetic circuit. From the relation, we have V µ f sient reactance X will come in to play which is low.
therefore, w2 = w1 = 1000 rpm. Thus, the plot of current given in option (a) is correct.

Chapter 4.indd 334 3/23/2016 11:34:09 AM


ANSWERS TO PRACTICE EXERCISES     335

3. (d) For the given synchronous motor For single machine, the resistance is R, the starting
V1 current is
d1 V 2V
I st = =
R/2 R
Starting power loss
Ef
 2V  Rt 2V
2 2
Initially power is given by =
 R  2
= t
Ef V1 R
P1 = sin d1 Saving in starting energy is
X
When the load is reduced to 25%, power is given by V 2 (2 − 1 − 1 / 4)
= t × 100
P2 = 0.25P1 R V2
t
Ef V2 R
= sin d 2 = 75%
X
5. (c) Mathematical derivations are rigorous, hence
V1
δ2 not provided. In the present case the configura-
δ1 tion of the windings from core are as shown
Ef below.

Ef
Therefore,
sin d 2
= 0.25
sin d1 L
H LH
VVL
So Ef attains its new steady-state power-angle d2
L
V VV V
with Vt by initially advancing followed by oscilla-
tion around d2. 1
4. (d) The given initial connection can be represented as 2

Core
V/2 V/2
Initial connection The leakage fluxes from the HV side will link the
V LV side from both ends so that the leakage fluxes
will reduce to half. Hence, the leakage reactance
For field and starting resistance of R each for each
will reduce to half.
machine for series operation:
V /2 V 6. (d) Stator and rotor poles have to be equal for
I st = = steady average torque production.
R 2R
Starting energy loss 7. (b) We know that efficiency is given by
 V 
2 xPo
h=
t
= 2 ⋅R
 2R  2 xPo + x2Psc + Pcore
V2 t
= ⋅ where x is the fraction of load to rated load.
2R 2 For maximum efficiency,
For the machine in parallel operation, energy loss
x2Psc = Pcore
V 
2
t
= 2  ⋅R
 R 2 Therefore,

V 2t Pcore
= x=
R Psc

Chapter 4.indd 335 3/23/2016 11:34:28 AM


336     Chapter 4:  Electrical machines 

8. (a) Given that V2 = 1.5 V1 and f2 = 0.5 f1. We


x2Pcu = Pcore
know that flux density
Pcore
f∝
V x= × 100%
f Pcu(fl)

400
V2 1.5 V1 V = × 100% = 70.7%
= =3 1 800
f2 0.3 f1 f1
or f2 = 3f1 13. (a) Given that: 15 kVA, 600/120 V
15
9. (b) For short-circuit tests Current ratio of 600 V windings = × 103 = 25 A
Under condition 1: 25 V, 50 Hz and current = I1 at 600
power factor lagging angle f1. Therefore Similarly, current ratio for 120 V winding = 125 A
kVA = 720 × 125 VA = 90 kVA
f1 ∝
25
∝ 0.5
50 125 A
Under condition 2: 25 V, 25 Hz and current = I2 at
power factor lagging angle f2. Therefore, 150 A
25 720 V
flux 2 ∝ ∝1
25 600 V 25 A
Thus, flux2 > flux1
10. (c) Given that Ist = 5 Ifl and sfl = 4% 14. (b) Voltmeter reads VR = 100 ∠0° + 200 ∠120° = 100 − 100 + j866
3 2 3 VR = 100 ∠0° + 200 ∠120° = 100 − 100 + j866 × 200 = 173 V .
Tst = I stR2 = (5I fl )2 R2
ws ws 100Ð120°
3
Tfl = (I )2 s
w s fl
Therefore, VBC
Tst 25I fl2 R2
= sfl = 25 × 0.04 = 1.0
Tfl I fl2 R2 Pr VAB
100Ð0°

11. (a) Given that er = 0.03 and ex = 0.06; and I2 = 50 200Ð120°


A ∠ 0°
Voltage regulation is given by VR

E2 − V2
= e r cos q2 + e x sin q2
V2
Rated current
100Ð0°
10 × 10 3
I2 = = 50 A 15. (b) Given that re1 and xe1, are total resistance and
200 leakage reactance referred to for
Also,
cos q2 = 1 and sin q2 = 0 re1 jxe1

Therefore,
I2 xe2
Regulation = × 1 = 0.03
V2
E2 - V2 = 0.03 × 200 = 6 V xe1 = 2re1
V2 = 200 - 6 = 194 V   ze = re1 + j2re1
12. (c) Given that Pcore = 400 W, Pcu(fl) = 800 W Thus,

For maximum efficiency, q e = tan−1(2) = 63.43°

Chapter 4.indd 336 3/23/2016 11:34:51 AM


ANSWERS TO PRACTICE EXERCISES     337

Thus, input point at short circuit = cos 63.43°


This frequency is generated by alternator,
2
120 ×
1 s = 1000
= 0.44 = Thus,
5 6
240
16. (b) Given that TL = 1 pu; s = 5% = 0.0; Tmax = s= = 0.04
20 pu; smT = 0.1 = 10% 6000
2
20 pu and fs = = 50 Hz
0.04
For an eight pole motor of 50 Hz the speed
120 × 50
T L = 1.0p N= = 750 rpm
8
The speed of the induction motor
= 750 - 0.04 × 750 = 720 rpm
0.1 0.5
We have 20. (b) We have
f1 = k
R2 220
= smT = 0.1 = 4.4 k
X2 50
New f2 = k
Te 330
2 = 6. 6 k
= = 0.8 50
Tmax 0. 5 0.1
+ Eddy current loss is given by
1 0.05
Tst 2 Pe = kef 2f 2
= = 0.198
æ 6.6k ö
Tmax 1 0.1 Pe2 2
+
0.1 1 = kef 2 ç ÷ = 2.25
Pe1 è 4.4k ø
Tst
Te
= 0.247 Pe2 = 2.25 × 50 = 112.5 W

Te ∝ V2
21. (c) Given that
10
VB = kV;
Tst = 0.247 pu 3
Thus, Vst = 0.247 = 0.5 50
VAB = MVA
3
17. (a) Given 200 V DC, shunt motor 17 kW, input = Therefore
50
× 103
20 kW
2
200 IB = 3 = 2886 A;
Field losses = W = 800 W 10
50
3
20 × 103 10 × 103
Full load current = A = 100A ZB = = 2.0 W
3 × 2886
200
Full load armature Cu loss = 368.4 W
Then for a total loss of (20 - 17) = 3 kW Hence 0.2 Ω =
0.2
= 0.1 pu
Core loss = 3 × 103 - 800 - 368.4 = 1831.4 W 2.0
Thus, for maximum efficiency total armature 25
copper loss = 1831.4 + 800 ≈ 2632 W. For 100 MVA, VB =
3
kV

18. (a) Given that Ph = 300 W, f2 = 0.9 f1 and Pe = 400 W 100


100 × 1033
× 10
Now, Valt ∝ f , Bm ∝ f and assuming x = 2, we 3
= 3
IIB = 2309 A
= 2309
= A
have B 25
25
x
Ph2 = kh f2 Bm = 0.9 × 300 = 218 W 33
= 400 × (0.9)4 = 262.44 W 25 × 1033
Pe2 = ke f22 Bm = 25 × 10
2
Z = 66..25
ZBB = × 2309
33 × 2309..44
= 25
19. (d) Given that sfs = 2 Hz, therefore
æ2ö Hence 0.2 Ω =
0.2
fs = ç ÷ t pu = 0.032 pu
èsø 6.25

Chapter 4.indd 337 3/23/2016 11:35:20 AM


338     Chapter 4:  Electrical machines 

SOLVED GATE PREVIOUS YEARS’ QUESTIONS

1. A single-phase transformer has a maximum effi- (c)  3   1   2   4


ciency of 90% at full load and unity power (d)  3   2   1   4
factor. Efficiency at half load at the same power
(GATE 2003: 1 Mark)
factor is
Ans. (c)
(a) 86.7% (b) 88.26%
(c) 88.9% (d) 87.8% 3. A stand-alone engine-driven synchronous generator
is feeding a partly inductive load. A capacitor is
(GATE 2003: 1 Mark) now connected across the load to completely nullify
the inductive current. For this operating condition
Solution:  Efficiency of the transformer is given by,
(a) field current and fuel input have to be reduced
V2 I2 cos f 2
h= (b) field current and fuel input have to be increased
V2 I2 cos f2 + Pi + Pcu (c) field current has to be increased and fuel input
left unaltered
where Pi and Pcu are iron and copper losses, (d) field current has to be reduced and fuel input
respectively. left unaltered
Pi (GATE 2003: 1 Mark)
At full load, the load factor (LF) = = V2 I2 = 1
Pcu
Let output = 1 pu, then we have Solution:  Given that
1×1 (i) Synchronous generator feeds a partly inductive
90% = (as Pi = Pcu )
1 + 2Pi load.
(ii) Capacitor connected across the load nullifies
Pi = 0.0555 mVA
the inductive current.
At half load, LF = 0.5, therefore Therefore, field current has to be reduced and fuel
input should be left unaltered.
0.5 × 1
h= × 100 = 87.8% Ans. (d)
0.5 × 0.0555 × ( 0.5 ) + 0.0555
2
4. Curves X and Y in the figure denote open circuit
Ans. (d) and full-load zero power factor (ZPF) characteristics
of a synchronous generator. Q is a point on the ZPF
2. Group I lists different applications and Group  II characteristics at 1.0 per unit (pu) voltage. The ver-
lists the motors for these applications. Match the tical distance PQ in in the figure gives the voltage
application with the most suitable motor and drop across
choose the right combination among the choices
given thereafter (a) synchronous reactance
(b) magnetising reactance
Group I Group II (c) potier reactance
P. Food mixer 1. P
 ermanent magnet DC (d) leakage reactance
motor
Voltage (pu) X
Q. Cassette tape 2. S
 ingle-phase induction
recorder motor P
Y
R. Domestic 3. Universal motor
1.0 Q
water pump
S. Escalator 4. T
 hree-phase induction
motor
5. DC series motor
6. Stepper motor Field current
Code: (GATE 2003: 1 Mark)
P   Q   R    S
(a)  3   6   4   5 Solution:  Given that, on zero power factor curve,
(b)  1   3   2   4 at point Q the voltage is 1.0 pu. Therefore, voltage

Chapter 4.indd 338 3/23/2016 11:35:27 AM


SOLVED GATE PREVIOUS YEARS’ QUESTIONS     339

drop between points P and Q is across synchronous correct phasors of voltages E1, E2, currents I1, I2
reactance. and core flux f are as shown in the following figure
Ans. (a)
f
5. No-load test on a three-phase induction motor was
conducted at different supply voltages and a plot of I1
input power versus voltage was drawn. This curve I2
was extrapolated to intersect the y-axis. This inter-
section point yields E1 N1 N2 E2 R
(a) core loss
(b) stator copper loss
(c) stray load loss
(d) friction and windage loss
E1 E1
(GATE 2003: 1 Mark)

Solution:  The intersection point gives friction and I2 I2


windage loss.
E2 E2
Po I1 I1

f f
PF & W (A) (B)

E1
Ans. (d) E1
6. The speed-torque regimes in a DC motor and the
control methods suitable for the same are given, I1
I1
respectively, in Group II and Group I
f f
Group I Group II
P. Field control 1. Below base speed E2 E2
I2 I2
Q. Armature 2. Above base speed
control (C) (D)
3. Above base torque
4. Below base torque (a) Figure A (b) Figure B
(c) Figure C (d) Figure D
The match between the control method and the
speed−torque regime is as follows: (GATE 2003: 2 Marks)

Code: Solution:  Let flux be f = fm sin wt


P   Q Therefore, emf induced in secondary
 (a) 1   3
df
(b) 2   1 E2 = − N = −fmw cos wt × N 2
   (c) 2   3 dt 2
(d) 1   4 = fmw sin (wt − 90° ) × N 2
(GATE 2003: 1 Mark) Hence, we can see that f lags behind E1 by 90°
I2 is in phase with E2   (Load is purely resistive)
Solution:  For above base speed we use field con- Now, magnetising current
trol method and for below base speed we use arma-
E1 E
ture control method. I1 = = 1 ∠ − 90°
Ans. (b) jXm Xm
7. The given figure shows an ideal single-phase trans- Hence, I1 lags behind E1 by 90°
former. The primary and secondary coils are wound Since, we know that I1 µ f1, so, f1 also lags behind
on the core as shown. Turns ratio (N1/N2) = 2. The E1 by 90°.

Chapter 4.indd 339 3/23/2016 11:35:39 AM


340     Chapter 4:  Electrical machines 

From above statements we can conclude the phasor


diagram as: I1
E1
R = 10 Ω
V1 1 N1 N2
I2 2
E2
I1 N3

f 3
Ans. (a)
8. To conduct load test on a DC shunt motor, it is
coupled to a generator which is identical to the Xc = 2.5 Ω
motor. The field of the generator is also con-
(a) (—10 + j10) A (b) (—10 — j10) A
nected to the same supply source as the motor.
(c) (10 + j10) A (d) (10 — j10) A
The armature of the generator is connected to a
load resistance. The armature resistance is 0.02 pu. (GATE 2003: 2 Marks)
Armature reaction and mechanical losses can be
neglected. With rated voltage across the motor, the Solution:  Given that
load resistance across the generator is adjusted to R = 10 W; V1 = 100 V and N1 : N 2 : N 2 = 4 : 2 : 1
obtain rated armature current in both motor and
generator. The pu value of this load resistance is We know that

V1 N 4 2 2
(a) 1.0 (b) 0.98 = 1 = Þ V2 = .V1 = × 100
(c) 0.96 (d) 0.94 V2 N2 2 4 4
(GATE 2003: 2 Marks) Also,
V1 N 4
Solution:  Given that, armature resistance is = 1 =
V3 N3 1
Ra = 0.02.
Mechanical losses and armature reaction are 1 1
V3 = ; V1 = 100
neglected, therefore back emf in the motor 4 4
Eb = 1 − 0.02 × 1 = 0.98 pu Therefore,

As the motor is coupled to the generator, emf gen- V2 = 200 V; V3 = 100 V


erated in the motor
Considering secondary winding V2 = I2R
Eg = Eb
V2 200
Therefore, terminal voltage of the generator I2 = = = 20 A
R 10
Vg = 0.98 − 1 × 0.02 = 0.96 pu On connecting the capacitor,
The value of load resistance is V3
= jÐ0
100
I3 = =
Vg −j × C −j × 2.5
0.96
Secondary current I2, referred to primary be I1¢
RL = = = 0.96 pu
Ia 1
Ans. (c)
I1¢ N 2
9. The given figure shows an ideal three-winding = 2 =
transformer. The three windings 1, 2, 3 of the I2 N1 4

I1¢ = 20 ×
transformer are wound on the same core as shown. 2
= 10 A
The turns ratio N1:N2:N3 is 4:2:1. A resistor of 4
10 Ω is connected across winding-2. A capacitor Let third winding current I3 referred to primary
of reactance 2.5 Ω is connected across winding-3. be I1²
I1²
Winding-1 is connected across a 400 V AC supply.
If the supply voltage phasor V1 = 400 Ð0, the
N 1
= 3 = (as I3 flows opposite to I1 )
supply current phasor I1 is given by −I3 N1 4
I1² = −j × 10

Chapter 4.indd 340 3/23/2016 11:36:02 AM


SOLVED GATE PREVIOUS YEARS’ QUESTIONS     341

Therefore, instant as shown in the figure. Thus, half of stator


I1 = I1 + I1¢¢ = 10 − 10j A
and rotor surfaces will form one pole with the other
half forming the second pole. Further, the direction
Ans. (d) of torque acting on the rotor can be clockwise or
10. Following are some of the properties of rotating counter clockwise
electrical machines P.
Stator winding current is dc, rotor-winding current Stator
is ac C
Air gap
Q. Stator winding current is AC, rotor-winding c Rotor
current is DC
b
R. Stator winding current is AC, rotor-winding B D Fs
d
current is ac Stator mmf axis
S. Stator has salient poles and rotor has commutator d Fr
a
T. Rotor has salient poles and slip rings and stator Rotor mmf axis
is cylindrical
A
U. Both stator and rotor have poly-phase windings
DC machines, synchronous machines and induction
machines exhibit some of the above properties as The following table gives four sets of statements
given in the following table. Indicate the correct as regards poles and torque. Select the correct set
combination from this table corresponding to the mmf axes as shown in the
given figure.
DC Synchronous Induction
machines machines machines
(a) P.S. Q.T. R.U. Stator Stator Rotor Rotor Torque
(b) Q.U. P.T. R.S. surface surface surface surface
(c) P.S. R.U. Q.T. ABC CDA abc cda
forms forms forms forms
(d) R.S. Q.U. P.T.
(a) N
 orth South North South Clockwise
(GATE 2003: 2 Marks) pole pole pole pole
Solution:  In DC motor: (b) South North North South Counter
  (i) Stator winding is connected to DC supply and pole pole pole pole clockwise
rotor-winding flow current is AC (P). (c) N
 orth South South North Counter
(ii) Stator is made of salient pole and rotor will pole pole pole pole clockwise
have commutator to carry AC power (S)
(d) South North South North Clockwise
In induction machines: pole pole pole pole
  (i) The AC supply is connected to stator winding (R)
(ii) Rotor and stator are made of poly-phase wind- (GATE 2003: 2 Marks)
ing (U).
In synchronous machines:
Solution:  Given that: Fs is the peak value of
  (i) Stator is fed with ac supply, but rotor will be
stator mmf; Fr is the peak value of rotar mmf and
excited by DC supply (Q).
d is the space angle.
(ii) Stator is cylindrical, rotor is made of both
Direction of rotation of rotor is clockwise or coun-
salient pole and slip rings (T).
ter clockwise.
Hence, the correct combination is represented by Thus, the rotor tends to move when the opposite
option (a). poles are produced in same half of the stator and
Ans. (a) rotor. Therefore, stator surface ABC forms north
11. When stator and rotor windings of a 2-pole rotat- pole; rotor surface abc forms South pole; rotor sur-
ing electrical machine are excited, each would pro- face cda forms North pole and stator surface CDA
duce a sinusoidal mmf distribution in the air gap forms South pole with torque in counter clockwise
with peak values Fs and Fr, respectively. The rotor direction.
mmf lags stator mmf by a space angle d at any Ans. (c)

Chapter 4.indd 341 3/23/2016 11:36:05 AM


342     Chapter 4:  Electrical machines 

12. A 4-pole, three-phase, double-layer winding is 14. A single-phase induction motor with only the main
housed in a 36-slot stator for an AC machine winding excited would exhibit which of the follow-
with 60° phase spread. Coil span is 7 slot pitches. ing response at synchronous speed?
Number of slots in which top and bottom layers
 (a) Rotor current is zero.
belong to different phases is
(b)  Rotor current is non-zero and is at slip frequency.
(a) 24    (b) 18    (c) 12    (d) 0    (c) F
 orward and backward totalling fields are
equal.
(GATE 2003: 2 Marks)
(d) Forward rotating field is more than the back-
Solution:  Given that: number of poles = 4; number ward rotating field.
of phases = 3, number of slots = 36, phase spread = (GATE 2003: 2 Marks)
36° and coil span = 7 (short-pitched).
Solution:  Slip corresponding to the forward field:
Slot 36
Pole pitch = = =9
Pole 4 Ns − Nr
sf =
9 Ns
Slot/pole/phase = = 3
3
And, slip corresponding to the backward field:
Therefore, 3, 2 will have different phases which
implies 36 and 24 will have different phases. Hence N s − (−N r )
number of slots in which top and bottom layers sb =
belong to different phases is 24. Ns
Ans. (a) Now, at rotor speed Nr = Ns, sf = 0 sb = 2
13. A three-phase induction motor is driving a con- Hence, their no relative motion between forward
stant torque load at rated voltage and frequency. field and rotor winding, but there is a relative
If both voltage and frequency are halved, fol- motion between backward field and rotor winding.
lowing statements relate to the new condition if Therefore, forward rotating field is more than the
stator resistance, leakage reactance and core loss backward rotating field.
are ignored Ans. (d)
  P. The difference between synchronous speed and 15. A DC series motor driving an electric train faces a
actual speed remains same constant power load. It is running at rated speed
Q. The air gap flux remains same and rated voltage. If the speed has to be brought
 R. The stator current remains same down to 0.25 pu, the supply voltage has to be
approximately brought down to
  S. The pu slip remains same
Among the above, correct statements are (a) 0.75 pu (b) 0.5 pu
(c) 0.25 pu (d) 0.125 pu
(a) All (b) P, Q and R
(c) Q, R and S (d) P and S (GATE 2003: 2 Marks)

(GATE 2003: 2 Marks) Solution:  In DC motor


Solution:  Given that the induction motor drives Eb = V − Ia Ra
a constant torque load at rated voltage and fre- = V − Ia (Ra + Rse ) [For series motor Ia = I se ]
quency. Rs, XL and core loss are ignored. Also
fZNp
X F Eb =
V = ,F = 60 a
2 2
Eb = KfN [as Z, p, a are constant, so
We know that K is a constant = Zp/60 a]
Eb
N =
120f Kf
Ns = = actual speed
p
In series motor
 (i) When leakage reactance are ignored, air-gap f µ Ise
flux remains the same.
(ii) Rs ignored implies that Is remains the same. V − I (Ra + Rse )
N=
Ans. (b) KI se

Chapter 4.indd 342 3/23/2016 11:36:14 AM


SOLVED GATE PREVIOUS YEARS’ QUESTIONS     343

At constant power, Full load copper losses = 600 W


Eb Ia = T × W = constant
Maximum efficiency occurs at
(i)
T µ f Ia (ii) Wi = X 2Wcu
T µ Ia 2 (For series motor) Wi 300
X= = = 0.707
To maintain constant power, when W is decreased, Wcu 600
torque is to be increased % Efficiency = 0.707 × 100 = 70.7%
1 Ans. (b)
W = , then T = 4
4 18. For a given stepper motor, which of the following
torque has the highest numerical value?
Therefore, Ia is increased two times and voltage is
reduced to 0.5 pu. (a) Detent torque (b) Pull-in torque
Ans. (b) (c) Pull-out torque (d) Holding torque
16. An AC induction motor is used for a speed control (GATE 2004: 1 Mark)
application. It is driven from an inverter with a
constant V/f control. The motor name plate details Solution:
are as follows:   (i) Holding torque is the amount of torque that
V = 415 V  Ph = 3  f = 50 Hz  N = 2850 rpm the motor produces when it has rated current
flowing through the windings but the motor is
The motor is run with the inverter output fre-
at rest.
quency set at 40 Hz, and with half the rated slip.
(ii) Detent torque is the amount of torque that the
The running speed of the motor is
motor produces when it is not energised. No
(a) 2400 rpm (b) 2280 rpm current is flowing through the windings.
(c) 2340 rpm (d) 2790 rpm (iii) Pull-in torque curve shows the maximum
value of torque at given speeds that the motor
(GATE 2003: 2 Marks)
can start, stop or reverse in synchronism with
Solution:  Given that: V = 415 V, Ph = 3, f = 50 Hz the input pulses. The motor cannot start at a
and N = 2850 rpm. We have speed that is beyond this curve.
(iv) Pull-out torque curve shows the maximum value
120f of torque at given speeds that the motor can
N1 =
p generate while running in synchronism. If the
120 × 50 motor is run outside of this curve, it will stall.
p= =2 Therefore, the higher value of torque is pull out
2850
torque and less torque when the torque is pull in
When the output frequency is set at 40 Hz, torque.
Ans. (c)
120
NT = f 19. Which of the following motor definitely has a per-
P 1
manent magnet rotor?
120 × 40
= = 2400 (a) DC commutator motor
2
(b) Brushless DC motor
When the slip is reduced to half, N = 2340 rpm. (c) Stepper motor
Ans. (c) (d) Reluctance motor
17. A 500 kVA, three-phase transformer has iron losses (GATE 2004: 1 Mark)
of 300 W and full load copper losses of 600 W.
The percentage load at which the transformer is Ans. (b)
expected to have maximum efficiency is 20. The type of single-phase induction motor having
the highest power factor at full load is
(a) 50.0% (b) 70.7%
(c) 141.4% (d) 200.0% (a) shaded pole type
(b) split-phase type
(GATE 2004: 1 Mark)
(c) capacitor-start type
(d) capacitor-run type
Solution:  Given rating = 500 kVA
Iron losses = 300 W (GATE 2004: 1 Mark)

Chapter 4.indd 343 3/23/2016 11:36:20 AM


344     Chapter 4:  Electrical machines 

Solution:  Single-phase induction motor is not self- 24. The armature resistance of a permanent magnet
starting. Capacitor-run type motor will have high DC motor is 0.8 Ω. At no load, the motor draws
power factor in which capacitor will be connected 1.5 A from a supply voltage of 25 V and runs at
in running condition. 1500 rpm. The efficiency of the motor while it is
Ans. (d) operating on load at 1500 rpm drawing a current
21. The direction of rotation of a three-phase induc- of 3.5 A from the same source will be
tion motor is clockwise when it is supplied with (a) 48.0% (b) 57.1%
three-phase sinusoidal voltage having phase (c) 59.2% (d) 88.8%
sequence A-B-C. For counter clockwise rotation of
the motor, the phase sequence of the power supply (GATE 2004: 2 Marks)
should be
Solution:  Given that Ra = 0.8 Ω, Vnl = 25 V, N =
(a) B-C-A (b) C-A-B
1500 rpm, Inl = 1.5 A and Ifl = 3.5 A.
(c) A-C-B (d) B-C-A or C-A-B
Under no load condition, we have
(GATE 2004: 1 Mark)
P = V ×I
Solution:  Clockwise rotation is A-B-C = (V − Ia Ra )I
Counter clockwise direction will be A-C-B (any = (25 − 1.5 × 0.8) 1.5
two phases can be reversed). = 35.7 W
Ans. (c)
22. The synchronous speed for the seventh space har- Under load condition:
monic mmf wave of a three-phase, 8 pole, 50 Hz Iron loss = I 2R
induction machine is
=(3.5)2 × 0.8
 (a) 107.14 rpm in forward direction
(b) 107.14 rpm in reverse direction = 9.8 W
   (c) 5250 rpm in forward direction Total loss = No load losses + Iron losses
(d) 5250 rpm in reverse direction
= 35.7 + 9.8 = 45.5 W
(GATE 2004: 2 Marks) Total power, P = VI = 25 × 3.5 = 87.5 W

Pout
× 100
Solution:  Given that number of poles p = 8 and
Efficiency =
frequency f = 50 Hz. Pin
Total power − losses
N
Ns at Y th harmonic = s
7 =
Total power
120f 120 × 50 87.5 − 45.5
Ns = = = 750 rpm = = 48%
p 8 87.5
750 Ans. (a)
Ns at 7th harmonic = = 107.14 rpm
7 25. A 50 kVA, 3300/230 V single-phase transformer
Ans. (a) is connected as an autotransformer shown in the
23. A rotating electrical machine having its self-­ figure. The nominal rating of the autotransformer
inductances of both the stator and the rotor wind- will be
ings independent of the rotor position will definitely
not develop
(a) starting torque (b) synchronising torque
(c) hysteresis torque (d) reluctance torque N2
(GATE 2004: 2 Marks)
Vout
Solution:  Rotating electrical machines having its
N1
self-inductance of stator and rotor windings is inde- Vin = 3300 V
pendent of the rotor position of synchronising torque.
Ans. (b)

Chapter 4.indd 344 3/23/2016 11:36:30 AM


SOLVED GATE PREVIOUS YEARS’ QUESTIONS     345

(a) 50.0 kVA (b) 53.5 kVA For capacitor-start induction motor,


Zm = Rm + Xm = 6 + j4 Ω (Main winding)
(c) 717.4 kVA (d) 767.4 kVA
(GATE 2004: 2 Marks) Za = Ra + Xa = 8 + j6 Ω (Auxiliary winding)
Solution:  Given rating = 50 kVA
∠I m = ∠ − Zm = − ∠6 + 4 j = − 33.7
Vin = 3300 V
∠Ia = ∠8 + j6 +
Vout = 3300 + 230 = 3530 V 1
 (as capacitor is con-
jwc nected in series)
Power
Output current I2 =
Output voltage
= Ð8 + 6j −
j
50 × 10 3 wC
= = 217.4 A
230 a = ∠Ia − ∠ I m
In kVA rating
P2 = Vout × I2 æ ö
6-
1
-1 ç w ÷
= 3530 × 217.4 90 = - tan ç c - (-33.7) ÷
= 767.42 kVA çç 8 ÷÷
è ø
Ans. (d)
1
= 18
26. The resistance and reactance of a 100 kVA wC
11000/400 Δ−Y distribution transformer are 0.02
(∵ w = 2pf )
1
and 0.07 pu, respectively. The phase impedance of C=
2pf ´ 18
the transformer referred to the primary is
= 176.8 mF
1
(a) (0.02 + j 0.07) Ω (b) (0.55 + j 925) Ω =
2 ´ 3.14 ´ 50 ´ 18
(c) (15.125 + j 52.94) Ω (d) (72.6 + j 254.1) Ω
Ans. (a)
(GATE 2004: 2 Marks)
28. Two three-phase, Y-connected alternators are to
Solution:  Given that be paralleled to a set of common bus bars. The
armature has a per phase synchronous reactance
Zpu = 0.02 + j0.07, V2 I2 = 100 kVA of 1.7 Ω and negligible armature resistance. The
line voltage of the first machine is adjusted
(11 × 103 )2
2
Vp
ZBase = = = 3630 W to 3300 V and that of the second machine is
V2 I2 3 100 × 103 adjusted to 3200 V. The machine voltages are in
3 phase at the instant they are paralleled. Under
Phase impedance referred to primary, this condition, the synchronising current per
phase will be
Z1 = Zpu × ZBase = (0.02 + j0.07)(3630)
(a) 16.98 A (b) 29.41 A
= 72.6 + j254.1 (c) 33.96 A (d) 58.82 A
Ans. (d)
(GATE 2004: 2 Marks)
27. A single-phase, 230 V, 50 Hz, 4 pole, capacitor-
start induction motor has the following stand-still Solution:  We have
impedances 3300 3200
Main winding Zm = 6.0 + j 4.0 Ω Ef 1 =
3
Ef 2 =
3
Auxiliary winding Za = 8.0 + j 6.0 Ω
The value of the starting capacitor required to pro- Ec
duce 90° phase difference between the currents in Synchronising current =
Xs1 + Xs2
the main and auxiliary windings will be
(a) 176.84 µF (b) 187.24 µF Reactance of both the alternator will be the same,
(c) 265.26 µF (d) 280.86 µF therefore, synchronising current will be

(GATE 2004: 2 Marks) Ef 1 − Ef 2 1 æ 3300 − 3200 ö


= = ç ÷ = 16.98 A
Xs1 + Xs2 3 è 1. 7 + 1. 7 ø
Solution:  Given that V = 230 V, f = 50 Hz and
number of poles p = 4 Ans. (a)

Chapter 4.indd 345 3/23/2016 11:36:46 AM


346     Chapter 4:  Electrical machines 

29. A 400 V, 15 kW, 4 pole, 50 Hz, Y-connected induc- Solution:  Given that number of poles p = 8,
tion motor has full load slip of 4%. The output N = 250 rpm and flux f = 0.06. The voltage of
torque of the machine at full load is the generator is given by
(a) 1.66 Nm (b) 95.50 Nm fZNp
Eg =
(c) 99.47 Nm (d) 624.73 Nm 60 a
(GATE 2004: 2 Marks) where, a = 2 (wave winding) and Z = 2 × 32 × 6
= 384. Therefore
Solution:  Given that V = 400 V P = 15 kW and
number of poles p = 4 0.06 × 250 × 8 × 384
Eg = = 384 V
60 × 2
120f Ans. (c)
Ns =
p
32. A 400 V, 50 kVA, 0.8 pf leading Δ-connected, 50 Hz
120 × 50 synchronous machine has a synchronous reactance
= = 1500 rpm
4 of 2 Ω and negligible armature resistance. The fric-
tion and windage losses are 2 kW and the core
Actual speed = Synchronised speed − Slip loss is 0.8 kW. The shaft is supplying 9 kW load
 4  at a power factor of 0.8 leading. The line current
= 1500 −  × 1500
 100  drawn is
= 1440 rpm (a) 12.29 A (b) 16.24 A
(c) 21.29 A (d) 36.88 A
The output torque
(GATE 2004: 2 Marks)
æ 2pN ö
ç∵ w s (1 − s) =
P
T= ÷ Solution:  Given that V = 400 V, f = 50 Hz and
w s (1 − s) è 60 ø pf = 0.8.
15 × 103
= = 99.47 Nm Total input power = 9 + 2 + 0.8 kW = 11.8 kW
2p × 1440
Input power
60
Ans. (c) 3 V2I2 = 11.8 kW
30. For a 1.8°, two-phase bipolar stepper motor, the 11.8 kW
I2 = = 21.29 A
stepping rate is 100 steps/s. The rotational speed 3 × 400 × 0.8
of the motor in rpm is
Ans. (c)
(a) 15 (b) 30 33. A 500 MW three-phase Y-connected synchronous
(c) 60 (d) 90 generator has a rated voltage of 21.5 kV at 0.85 pf.
(GATE 2004: 2 Marks) The line current when operating at full load rated
conditions will be
360
Solution:  Number of steps per revolution = = 200 steps(a) 13.43 kA (b) 15.79 kA
360 1.8
(c) 23.25 kA (d) 27.36 kA
= 200 steps
1.8
(GATE 2004: 2 Marks)
Time required for one revolution = 2 seconds
Revolutions/s = 0.5 rps Solution:  Input power
Revolutions/min = 0.5 × 60 = 30 rpm
Ans. (b) 3 VL I L = 500 MW
500 × 106
31. A 8-pole, DC generator has a simplex wave- IL = = 15.79 × 103
wound armature containing 32 coils of 6 turns 3 × 21.5 × 103 × 0.85
each. Its flux per pole is 0.06 Wb. The machine I L = 15.79 kA
is running at 250 rpm. The induced armature
Ans. (b)
voltage is
34. A variable speed drive rated for 1500 rpm, 40 Nm
(a) 96 V (b) 192 V is reversing under no load. The given figure shows
(c) 384 V (d) 768 V the reversing torque and the speed during the tran-
(GATE 2004: 2 Marks) sient. The moment of inertia of the drive is

Chapter 4.indd 346 3/23/2016 11:36:57 AM


SOLVED GATE PREVIOUS YEARS’ QUESTIONS     347

(a) 0.048 kgm2 (b) 0.064 kgm2 X1 ≈ X2′  XM


(c) 0.096 kgm2 (d) 0.128 kgm2
Ans. (d)
36. Which three-phase connections can be used in a trans-
former to introduce a phase difference of 30° between
+20 Nm its output and corresponding input time voltages?
(a) Star−star (b) Star−delta
(c) Delta−delta (d) Delta−zig-zag
t
Torque (GATE 2005: 1 Mark)
0.5 s
Solution:  Only in Y −∆ and ∆−Y , phase shift of
±30° can be introduced.
Therefore, connection is star−delta.
Ans. (b)
+500 rpm 37. On the torque−speed curve of induction motor
shown in the figure, four points of operation are
Speed t marked as W, X, Y and Z. Which one of them
represents the operation at a slip greater than 1?
(a) W    (b) X    (c) Y    (d) Z

−1500 rpm Torque

(GATE 2004: 2 Marks)

Solution:  As we know by formula: X

T = Ia Y
Torque is given as 40 Nm.
Now, a is equal to:
W

500 − (−1500) 2p
0 Speed
a= = 418.67 rad/s2 Z
0. 5 60
Therefore, I will be: (GATE 2005: 1 Mark)
40
I= Solution:  When speed of the motor is in the forward
418.67 direction, slip varies from 0 to 1.
Moment of inertia = 0.096 kgm2 When speed of the motor is in the reverse direction,
Ans. (c) then slip > 1. Therefore, slip > 1 at point W.
35. The equivalent circuit of a transformer has leakage Ans. (a)
reactances X1, X2′ and magnetising reactance XM. 38. For an induction motor, operating at a slip s, the ratio
Their magnitudes satisfy of gross power output to air-gap power is equal to
(a) X1 >> X 2′ >> XM (a) (1 - s)2 (b) (1 - s)
(b) X1 << X 2′ << XM
(c) (1 − s) (d) (1 − s )
(c) X1 ≈ X2 > XM
(d) X1 ≈ X 2′ << XM (GATE 2005: 1 Mark)
(GATE 2005: 1 Mark) Solution:
Gross power
= 1−s
Solution:  As X1 = X2′ Air -gap power
XM > X1 and X2′
Therefore, Ans. (b)

Chapter 4.indd 347 3/23/2016 11:37:12 AM


348     Chapter 4:  Electrical machines 

39. Two magnetic poles revolve around a stationary 41. In relation to DC machines, match the following
armature carrying two coils (c1-c′1, c2-c′2) as and choose the correct combination
shown in the figure. Consider the instant when the
poles are in a position as shown. Identify the correct Group I Group II
statement regarding the polarity of the induced emf Performance variables Proportional to
at this instant in coil sides c1 and c2. P. Armature emf (E) 1. F
 lux (f),
(a)  in c1, no emf in c2 speed (w)
(b) ⊗ in c1, no emf in c2 and armature
(c)  in c2, no emf in c1 current (Ia)
(d) ⊗ in c2, no emf in c1 Q. Developed torque (T) 2. f and w only
R. Developed power (P) 3. f and Ia only
4. Ia and w only
c¢2 5. Ia only
c¢1 (GATE 2005: 2 Marks)
S N   (a) P - 3, Q - 3, R - 1
c1 (b) P - 2, Q - 5, R - 4
   (c) P - 3, Q - 5, R - 4
c2 (d) P - 2, Q - 3, R - 1

Solution:  In a DC motor,
   (i) E = Kfw n
(GATE 2005: 2 Marks)
pfZIa
   (ii) T =
Solution:  At c1 emf will be induced upwards, 2p a
whereas at c2 and c′2 no emf is induced.
Ans. (a) (iii) Thus, from the above expressions power P
depends on f, w and Ia. Therefore, P - 2 Q - 3,
40. A 50 kW DC shunt motor is loaded to draw rated R -1
armature current at any given speed. When driven Ans. (d)
(i) at half the rated speed by armature voltage
42. In relation to the synchronous machines, which one
control, and
of the following statements is false?
(ii) at 1.5 times the rated speed by field control,the
respective output powers delivered by the (a) In salient pole machines, the direct-axis syn-
motor are approximately. chronous reactance is greater than the quadrature-­
(a) 25 kW in (i) and 75 kW in (ii) axis synchronous reactance
(b) 25 kW in (i) and 50 kW in (ii) (b) The damper bars help the synchronous motor
(c)  50 kW in (i) and 75 kW in (ii) self-start
(d) 50 kW in (i) and 60 kW in (ii) (c) Short-circuit ratio is the ratio of the field cur-
rent required to produce the rated voltage on
(GATE 2005: 2 Marks) open circuit to the rated armature current
(d) The V-curve of a synchronous motor represents
Solution:  We know that
the variation in the armature current with field
P ∝N excitation, at a given output power
where N is the rated speed. (GATE 2005: 2 Marks)
At half the rated speed,
Solution:
50   (i) In a synchronous machine, when the armature
Pnew = = 25 kW
2 terminals are shorted, the If should first be
reduced to zero and then started as alternator.
At 1.5 times the rated speed, P = constant.   (ii) In open-circuit, the synchronous machine runs
Therefore Pnew = 50 kW at rated Ns.
Ans. (b) (iii) The field current is gradually increased in
steps.

Chapter 4.indd 348 3/23/2016 11:37:21 AM


SOLVED GATE PREVIOUS YEARS’ QUESTIONS     349

(iv) The short-circuit ratio is the ratio of If required Solution:  The increased rotor resistance will
to produce the rated voltage on open circuit to increase starting torque and hence it will have high
the rated armature current. acceleration.
Ans. (c) Ans. (b)
43. Under no load condition, if the applied voltage to 46. Determine the correctness or otherwise of the fol-
an induction motor is reduced from the rated volt- lowing assertion [A] and the reason [R].
age to half the rated value
Assertion: Under V/f control of induction motor,
(a) the speed decreases and the stator current the maximum value of the developed torque
increases remains constant over a wide range of speed in the
(b) both the speed and the stator current decrease sub-synchronous region.
(c) the speed and the stator current remain practi- Reason: The magnetic flux is maintained
cally constant almost constant at the rated value by keeping
(d) there is negligible change in the speed but the the ratio V/f constant over the considered speed
stator current decreases range.
(GATE 2005: 2 Marks)   (a) Both (A) and (R) are true and (R) is the cor-
rect reason for (A)
Solution:  Under no load condition, when the
applied voltage to an induction motor is reduced (b) Both (A) and (R) are true but (R) is not the
from the rated voltage to half the rated value, both correct reason for (A)
speed and stator current decrease.    (c) Both (A) and (R) are false
Ans. (b) (d) (A) is true but (R) is false
44. A three-phase cage induction motor is started by
direct-on-line (DOL) switching at the rated volt- (GATE 2005: 2 Marks)
age. If the starting current drawn is 6 times the Solution:  In an induction motor
full load current, and the full load slip is 4%, then
ratio of the starting developed torque to the full
load torque is approximately equal to E = 4.44 kw 1 ffT1

(a) 0.24 (b) 1.44


Assertion: In V/f control, the maximum devel-
(c) 2.40 (d) 6.00
oped torque remains the same.
(GATE 2005: 2 Marks) (i) To avoid saturation and to minimise losses,
motor is operated at rated air gap flux by vary-
Solution:  Given that starting current Ist is six ing terminal voltage with frequency.
times the full load current Ifl
(ii) T
 o maintain V/f as constant, the magnetic
I flux should be maintained constant at the
I st = 6I fl ⇒ st = 6 rated value which keeps maximum torque
I fl
constant.
Full-load slip = 4%, therefore, the required ratio is Ans. (a)

Tst æ I st ö
2 Linked Answer Questions 47—48: A 1000 kVA,
=ç ÷ × Sfl 6.6 kV, three-phase star-connected cylindrical pole
Tfl è I fl ø synchronous generator has a synchronous reac-
= 62 × 0.04 = 1.44 tance of 20 Ω. Neglect the armature resistance and
consider operation at full load and unity power
Ans. (b)
factor.
45. In a single-phase induction motor driving a fan
load, the reason for having a high resistance rotor 47. The induced emf (line-to-line) is close to
is to achieve
(a) 5.5 kV (b) 7.2 kV
 (a) low starting torque (c) 9.6 kV (d) 12.5 kV
(b) quick acceleration
 (c) high efficiency
(GATE 2005: 2 Marks)
(d) reduced size
(GATE 2005: 2 Marks)

Chapter 4.indd 349 3/23/2016 11:37:24 AM


350     Chapter 4:  Electrical machines 

Solution:  Given that P = 1000 kVA, V = 6.6 kV (b) The direction of rotation can be changed by
and reactance = 20 Ω. reversing the main winding terminals
(c) The direction of rotation cannot be changed
P = 3VL I L (d) The direction of rotation can be changed by
1000 × 103 interchanging the supply terminals
IL = = 87.47 A
3 × 6.6 × 103 (GATE 2006: 1 Mark)
XI L = 87.47 × 20 = 1.75 kV
Solution:  Main and auxiliary windings are dis-
placed by 90° in space. The direction of rotation can
æ 6.6 ö
2

÷ + (1.75 )
2
Eph = ç be changed by reversing the main winding terminals.
è 3ø Ans. (b)
= 4.2 kV 51. In a DC machine, which of the following state-
EL = 3Eph ments is true?

= 3 × 4. 2 (a) C ompensating winding is used for neutralising


armature reaction, whereas interpole winding is
= 7.26 kV
used for producing residual flux.
Ans. (b) (b) Compensating winding is used for neutralising
48. The power (or torque) angle is close to armature reaction, whereas interpole winding is
used for improving commutation.
(a) 13.9° (b) 18.3° (c) Compensating winding is used for improving
(c) 24.6° (d) 33.0° commutation, whereas interpole winding is
(GATE 2005: 2 Marks) used for neutralising armature reaction.
(d) Compensating winding is used for improving
Solution:  Torque angle is given by commutation, whereas interpole winding is
æX ö used for producing residual flux.
a z = tan−1 ç s ÷
æ X ö è Ra ø
(GATE 2006: 1 Mark)
a z = tan−1 ç s ÷
1 æ 3 × 1.75 ö
Ans. (b)
a z =è tan
a ø−have 
Substituting values, Rwe
çç ÷÷ = 24.6
æ 3 × 1è.75 ö ø
6 . 6 52. A 400 V, 50 Hz, three-phase balanced source sup-
a z = tan−1 çç ÷÷ = 24.6 plies power to a star-connected load whose rating
è 6.6 ø is 12 3 kVA, 0.8 pf (lag). The rating (in kVAR)
Ans. (c) of the delta-connected (capacitive) reactive power
bank necessary to bring the pf to unity is
49. In transformers, which of the following statements
is valid? (a) 28.78 (b) 21.60
(c) 16.60 (d) 12.47
(a) In an open-circuit test, copper losses are obtained
while in short-circuit test, core losses are obtained (GATE 2006: 2 Marks)
(b) In an open-circuit test, current is drawn at high
Solution:
power factor
(c) In a short-circuit test, current is drawn at zero Real power, P = S cos f = 12 × 3 × 0.8 = 16.6 kW
power factor Reactive power, Q = S sin f = 12 × 3 × 0.6 = 12.5 kW
(d) In an open-circuit test, current is drawn at low For unity power factor, we have to set capacitor
power factor bank to Q = 12.5 kW.
(GATE 2006: 1 Mark) Ans. (d)
53. A 220 V DC machine supplies 20 A at 200 V as a gen-
Solution:  In open-circuit test, current is drawn at
erator. The armature resistance is 0.2 ohm. If the
low power factor while in short-circuit test, current
machine is now operated as a motor at same terminal
is drawn at high power factor.
voltage and current but with the flux increased by
Ans. (d)
10%, then ratio of motor speed to generator speed is
50. For a single-phase capacitor start induction motor
which of the following statements is valid? (a) 0.87 (b) 0.95
(c) 0.96 (d) 1.06
(a) T
 he capacitor is used for power factor
improvement (GATE 2006: 2 Marks)

Chapter 4.indd 350 3/23/2016 11:37:31 AM


SOLVED GATE PREVIOUS YEARS’ QUESTIONS     351

Solution:  For the generator, given that: Solution:  Given T1 = 500 kVA, leakage imped-
V = 220 V, Ia = 20 A and Ra = 0.2 Ω ance = 0.05 pu and T2 = 250 kVA. Let pu leakage
Therefore, impedance be x.

Per unit impedance µ


Eg = V + Ia Ra 1
kVA
= 200 + (20)(0.2)
= 204 V 500 kVA × 0.05 = 250 kVA × x

Now, 500
x= × 0.05 = 0.1 pu
Eb = V − Ia Ra 250
Ans. (b)
= 200 − (20)(0.12)
56. The speed of a 4-pole induction motor is controlled
= 196 V
by varying the supply frequency while maintaining
When the flux is increased by 10 %, the ratio of the ratio of supply voltage to supply frequency
motor to generator speed can be calculated as (V/f ) constant. At rated frequency of 50 Hz and
fg
rated voltage of 400 V, its speed is 1440 rpm. What
Eg Ng
= × is the speed at 30 Hz, if the load torque is constant?
Eb Nm fm
(a) 882 rpm (b) 864 rpm
−1
Nm æ 204 ö 1 (c) 840 rpm (d) 828 rpm
=ç ×
Ng è 196 ÷ø 1. 1 (GATE 2006: 2 Marks)
196
= = 0.87
204 × 1.1
Solution:  Given that: number of poles, p = 4,
f1 = 50 Hz, V1 = 400 V, N1 = 1440 rpm and T1 = T2.
Ans. (a) Since V ∝ f , therefore,
54. A synchronous generator is feeding a zero power
V1 f
factor (lagging) load at rated current. The arma- = 1
ture reaction is V2 f2
30
(a) magnetising V2 = 400 × = 240 V
(b) demagnetising 50
Also,
(c) cross-magnetising
(d) ineffective V2 
T ∝
 f 
s
(GATE 2006: 2 Marks)  
Therefore,
Solution:  The distributor of current and fluxes
p
s2 æ V1 ö
2
with phase angle f = + . f T
2 =ç ÷ × 2× 2
s1 è V2 ø f1 T1
For lagging load, the current will be shifted in
space by an angle p/2 from maximum emf which æ 400 ö
2
30
s2 = (0.04) ç ÷ × (as T1 = T2 )
coincides with centre of pole. The field created by è 240 ø 50
the armature reaction mmf will be in direction
= 0.066
opposite to main field flux and will therefore have
complete demagnetising effect. We know that
Ans. (b)
120f
55. Two transformers are to be operated in parallel N r = N s (1 − s); N s =
p
such that they share load in proportion to their
120 × 30
kVA ratings. The rating of the first transformer is Nr = (1 − 0.066)
500 kVA and its pu leakage impedance is 0.05 pu. 4
If the rating of second transformer is 250 kVA, then = 840 rpm
its pu leakage impedance is Ans. (c)
(a) 0.20 (b) 0.10 57. A three-phase, 4-pole, 400 V, 50 Hz, star-con-
(c) 0.05 (d) 0.025 nected induction motor has following circuit
(GATE 2006: 2 Marks) parameters

Chapter 4.indd 351 3/23/2016 11:37:45 AM


352     Chapter 4:  Electrical machines 

r1 = 1.0 Ω, r2′ = 0.5 Ω 59. A three-phase, 400 V, 5 kW, star-connected syn-


chronous motor having an internal reactance of
x1 = x2′ = 1.2 Ω, xm = 35 Ω
10 Ω is operating at 50% load, unity power factor.
The starting torque when the motor is started Now, the excitation is increased by 1%. What will
direct-on-line is (use approximate equivalent cir- be the new load in per cent, if the power factor is to
cuit model): be kept same? Neglect all losses and consider linear
magnetic circuit.
(a) 63.6 Nm (b) 74.3 Nm
(c) 190.8 Nm (d) 222.9 Nm (a) 67.9% (b) 56.9%
(c) 51% (d) 50%
(GATE 2006: 2 Marks)
(GATE 2006: 2 Marks)
Solution:  Given that number of poles, p = 4, V =
400 V, f = 50 Hz, r1 = 1 Ω, r2¢ = 0.5 W, x1 = x2¢ = 1.2 W Solution:  Given that number of phase = 3, V =
x1 = x2¢ = 1.2 W and xm = 35 Ω. 400 V, P = 5 kW, internal reactance = 10 Ω, and
power factor = unity power factor.
Therefore,
Full load current is given by
120f 120 × 50
Ns = = = 1500 rpm
p 4 P 5 × 103
I fl = = = 7.22 A
180 V r2¢
2
3V × 1 3 × 400 × 1
Tst = ×
2pN s (r1 + r2¢ ) + (x1 + x2¢ )2
2 1
E = [(V cos q − Ia Ra )2 + (V sin q − Ia Xs )2 ]2
æ 400 ö
2

ç ÷ × 0. 5
1
éæ 400 ö2 ù2
× è 2ø
180 3
= êç ÷ + (10 × 3.6 ) ú
= 2
2 × 3.14 × 1500 (1.5) + (2.4)2
êëè 3 ø úû
= 63.58 Nm
E = 233.73
Ans. (a)
When excitation is increased by 1%,
58. A three-phase, 10 kW, 400 V, 4-pole, 50 Hz, star-­
connected induction motor draws 20 A on full load. 1
Its no load and blocked-rotor test data are given E2 = 2833.73 × = 236
100
below: 1
No load test 400 V 6 A 1002 W æ æ 400 ö ö 2
2
Ia x = E22 − V 2 = ç 2362 − ç ÷ ÷÷ = 48.9
ç è ø ø
è
Blocked-rotor test 90 V 15 A 762 W 3
Neglecting copper loss in no load test and core loss 48.9 48.9
Ia = = = 4.8
in blocked rotor test, estimate motor’s full load x 10
efficiency. 4.8
Load in % = = 67%
(a) 76% (b) 81% 7.2
(c) 82.4% (d) 85% Ans. (a)
(GATE 2006: 2 Marks)
Common Data for Questions 60 — 62: A 4-pole,
Solution:  Given that P = 10 kW, V = 400 V, p = 4, 50 Hz, synchronous generator has 48 slots in which
f = 50 Hz, Ifl = 20 A a double-layer winding is housed. Each coil has
10 turns and is short pitched by an angle to 36° elec-
 20 
2
Full load copper loss =   × 762 trical. The fundamental flux per pole is 0.025 Wb.
 15 
= 1354.67 60. The line-to-line induced emf (in volts), for a three-
phase star connection is approximately
Total losses = 1354.67 + 1002
= 2356.67 (a) 808 (b) 888
The efficiency of the motor at full load is (c) 1400 (d) 1538

Q/P 10 × 103 (GATE 2006: 2 Marks)


h= = × 100 = 81%
I /P 10 × 103 + 2356.67 Solution:  Given that number of poles, p = 4, f =
Ans. (b) 50 Hz, number of slots = 48, number of turns = 10,

Chapter 4.indd 352 3/23/2016 11:37:57 AM


SOLVED GATE PREVIOUS YEARS’ QUESTIONS     353

a = 36° (electrical) and d = 0.05 Wb. We know 62. The fifth harmonic component of phase emf
that (in volts), for a three-phase star connection is

Eph = 4.44ffNkp kd Volts (i) (a) 0    (b) 269    (c) 281    (d) 808

where, f = flux per pole, f = frequency of genera- (GATE 2006: 2 Marks)


tion, N = number of turns, kp = pitch factor and
Solution:  For the fifth harmonic component of
kd = distribution factor.
phase emf,
The number of slots per pole per phase
Angle = 180/5 = 36°
48
æ na ö
m= = =4
4×3 Maximum value, cos ç ÷ = Em cos(5 × 18) = Em cos(90°) = 0
æ na ö è 2 ø
÷ = Em cos(5 × 18) = Em cos(90°) = 0
48
Slots/pole = = 12 cos ç
4 è 2 ø
Therefore, phase emf of 5th harmonic is zero.
= 15 = b
180 o
Slot angle = Ans. (a)
12
Using these values, we have 63. Consider the transformer connections in a part of
sin(mb /2) sin (4 × 15/2)
a power system shown in the figure. The nature of
Kd = = = 0.957 transformer connections and phase shifts are indi-
m sin b /2 4 sin(15/2) cated for all but one transformer.
a
= cos 180 = 0.951
36
Kp = cos = cos
2 2
48 × 10
Tph = = 160
3 −30° A
Therefore, substituting values in Eq. (i), we have 15 kV
Eph = 4.44 × 50 × 0.957 × 0.951 × 160 × 0.025 q
= 808 V B
30°
EL = 3 × 808 = 1399.5 V
Ans. (c) 0°
61. The line-to-line induced emf (in volts), for a three-
phase connection is approximately
400 kV 220 kV
(a) 1143   (b) 1332   (c) 1617   (d) 1791
Autotransformer
(GATE 2006: 2 Marks)
Which of the following connections, and the cor-
Solution:  For a two-phase winding, responding phase shift q, should be used for the
48 transformer between A and B?
Slots per pole per phase = =6=m
4×2
(a) Star−star (q = 0°)
Tph =
480
= 240 (b) Star−delta (q = -30°)
2 (c) Delta−star (q = 30°)
180 × 4
= 15 = b
(d) Star−zig-zag (q = 30°)
Slot angle =
48 (GATE 2007: 1 Mark)
Using these values, we have
Solution:  There exist equal phase shift at points A
sin (6 × 15/2) and B with respect to source from both the buses.
Kd = = 0.903
6 sin(15/2) Therefore, type of transformer is star—star with
æ 36 ö
phase shift of 0°.
Kp = cos ç ÷ = 0.951
è 2ø
Ans. (a)

Eph = 4.44 × 0.025 × 50 × 240 × 0.951 × 0.903


64. In a transformer, zero voltage regulation at full
load is
= 1143.8 V
(a) not possible
Ans. (a) (b) possible at unity power factor load

Chapter 4.indd 353 3/23/2016 11:38:10 AM


354     Chapter 4:  Electrical machines 

(c) possible at leading power factor load (a) unity


(d) possible at lagging power factor load (b) leading
(c) lagging
(GATE 2007: 1 Mark)
(d) dependent on machine parameters
Solution:  In a transformer, zero voltage regula- (GATE 2007: 2 Marks)
tion at full load gives leading power factor.
Ans. (c) Solution:  The given three-phase synchronous
motor, is running at full load with unity power
65. The DC motor which can provide zero speed regu- factor. On reducing the shaft load by half, If
lation at full load without any controller is remains constant; this implies leading power
(a) series factor.
(b) shunt Ans. (b)
(c) cumulative compound 68. A 100 kVA, 415 V (line), star-connected synchro-
(d) differential compound nous machine generates rated open circuit voltage
(GATE 2007: 1 Mark) of 415 V at a field current of 15 A. The short circuit
armature current at a field current of 10 A is equal
Solution:  Differential motor provides zero speed to the rated armature current. The per unit satu-
regulation at full load without any controller. rated synchronous reactance is
Ans. (d) (a) 1.731 (b) 1.5
66. The electromagnetic torque Te of a drive and its (c) 0.666 (d) 0.577
connected load torque TL are as shown in the (GATE 2007: 2 Marks)
figure. Out of the operating points A, B, C and D,
the stable ones are Solution:  Given that for the star-connected synchro-
(a) A, C, D (b) B, C nous machine P = 100 kVA, Voc = 415 V, If = 15 A
(c) A, D (d) B, C, D Line synchronous impedance
Voc
Te
3 × short circuit phase current
TL Te
T T 415
TL =
A æ 100 × 1000 ö
3 ×ç ÷
è 3 × 415 ø
B
= 1.7722
Speed Speed Ans. (a)
69. A three-phase, three-stack, variable reluctance step
motor has 20 poles on each rotor and stator stack.
C Te Te The step angle of this step motor is
T T
TL D (a) 3°    (b) 6°    (c) 9°    (d) 18°

TL (GATE 2007: 2 Marks)

Solution:  Given, number of poles p = 20 and


Speed Speed number of phases = 3.
Therefore, step angle
(GATE 2007: 1 Mark)
b= = 6
360
Solution:  From the characteristics given in the
options points A and D are stable. 2 × 20
Ans. (c) Ans. (b)
67. A three-phase synchronous motor connected to 70. A single-phase 50 kVA, 250 V/500 V two-­winding
AC mains is running at full load and unity power transformer has an efficiency of 95% at full load,
factor. If its shaft load is reduced by half, with unity power factor. If it is reconfigured as a
field current held constant, its new power factor 500 V/750 V autotransformer, its efficiency at its
will be new rated load at unity power factor will be

Chapter 4.indd 354 3/23/2016 11:38:16 AM


SOLVED GATE PREVIOUS YEARS’ QUESTIONS     355

(a) 95.752% (b) 97.851% Common Data for Questions 72 —74: A three-
(c) 98.276% (d) 99.241% phase squirrel-cage induction motor has a starting
current of seven times the full load current and full
(GATE 2007: 2 Marks)
load slip of 5%.
Solution:  Given that P = 50 kVA, V = 250 V/500 V 72. If an autotransformer is used for reduced voltage
and h = 95 at unity power factor. We know that starting to provide 1.5 per unit starting torque, the
efficiency is given by autotransformer ratio (%) should be
h=
Output power
(a) 57.77% (b) 72.56%
Output power + Iron loss + Copper loss (c) 78.25% (d) 81.33%
Therefore,
(GATE 2007: 2 Marks)
50 × 1 × 1
h= = 95% Solution:  Given that Ist = 7Ifl and sfl = 5 %. Also,
50 + (Wc + Wi )
the ratio of starting torque to full-load torque is
⇒ Wcu + Wi = 2.6
æI ö
2
Tst
When the transformer is reconfigured at 500 V/ = ç st ÷ × x2 × sfl
750 V, we have Tfl è I fl ø

h=
150
= 98.29% 1.5 = 72 × x2 × 0.05
150 + 2.6 ⇒ x = 0.782
Ans. (c)
Therefore ratio is 78.2%.
71. A three-phase squirrel-cage induction motor has a Ans. (c)
starting torque of 150% and a maximum torque of
300% with respect to rated torque at rated voltage 73. If a star−delta starter is used to start this induction
and rated frequency. Neglect the stator resistance motor, the per unit starting torque will be
and rotational losses. The value of slip for maxi- (a) 0.607 (b) 0.816
mum torque is (c) 1.225 (d) 1.616
(a) 13.48% (b) 16.24% (GATE 2007: 2 Marks)
(c) 18.92% (d) 26.79%
Solution:  On using star−delta starter,
(GATE 2007: 2 Marks)
1æI ö
2
Tst
Solution:  Given that Tst = 150% and Tmax = 300% = ç st ÷ × sfl
Tfl 3 è I fl ø
Let Tfl be full load torque. Then,
1
Tst = 1.5Tfl (i) = × 72 × 0.05
3
Tmax = 3Tfl (ii) = 0.816 Ans. (b)
Dividing Eq. (i) by Eq. (ii), we have 74. If a starting torque of 0.5 per unit is required, then
Tst 1.5Tfl the per unit starting current should be
1
= = (iii)
Tmax 3Tfl 2 (a) 4.65 (b) 3.75
(c) 3.16 (d) 2.13
For squirrel cage induction motor,
(GATE 2007: 2 Marks)
Tst 2s
= 2 mT 2 (iv) Equating eqs. (iii) and (iv), we get Given T = 0.5 pu
Solution: 
Tmax smT + 1 st
We have
I 
2
Tst
=  sc  × Sfl
2smT
= (iv) Equating eqs. (iii) and (iv), we get
2
smT + 12 Tfl  I fl 
I 
2
2smT
= 0.5. 0.5 =  sc  × 0.05
s2mT + 12  I fl 
2
+ 0.5 − 2smT = 0 I sc 0.5

0.5smT
= = 3.162
smT = 26.7% I fl 0.05
Ans. (d) Ans. (c)

Chapter 4.indd 355 3/23/2016 11:38:31 AM


356     Chapter 4:  Electrical machines 

75. Distributed winding and short chording employed The wattmeters used in open circuit test and short
in AC machines will result in circuit test of the transformer will, respectively, be
(a) increase in emf and reduction in harmonics (a)  W1 and W2 (b)  W2 and W4
(b) reduction in emf and increase in harmonics (c)  W1 and W4 (d)  W2 and W3
(c) increase in both emf and harmonics GATE 2008: 1 Mark)
(d) reduction in both emf and harmonics
Ans. (d)
(GATE 2008: 1 Mark)
79. A 230 V, 50 Hz, 4-pole, single-phase induction
Solution:  Distributed winding and short chord- motor is rotating in clockwise (forward) direction
ing in AC machines leads to reduction in emf and at a speed of 1425 rpm. If the rotor resistance at
harmonics. standstill is 7.8 Ω, then the effective rotor resis-
Ans. (d) tance in the backward branch of the equivalent
circuit will be
76. Three single-phase transformers are connected to
form a three-phase transformer bank. The trans- (a) 2 Ω (b) 4 Ω
formers are connected in the following manner: (c) 78 Ω (d) 156 Ω
(GATE 2008: 2 Marks)
A1
A2 a2 a1
Solution:  Given that: V = 230 V, p = 4, R2 = 7.8 Ω
B1 (resistance at standstill), f = 50 Hz and Nr =
B2 b2 b1
1425 rpm.
C1 Then the standstill speed can be calculated as
C2 c2 c1
Primary Secondary 120f 120 × 50
Ns = = = 1500 rpm
p 4
The transformer connection will be represented by
Ns − Nr 1500 − 1425
(a) Yd0   (b) Yd1   (c) Yd6   (d) Yd11 s= = = 0.05
Ns 1500
(GATE 2008: 1 Mark)
Ans. (b) Resistance in the background branch
77. In a stepper motor, the detent torque means R2
= 4Ω
7. 8
= =
(a) minimum of the static torque with the phase 2 − s 2 − 0.05
winding excited Ans. (b)
(b) maximum of the static torque with the phase 80. A 400 V, 50 Hz, 30 hp, three-phase induction
winding excited motor is drawing 50 A current at 0.8 power factor
(c) minimum of the static torque with the phase lagging. The stator and rotor copper losses are
winding unexcited 1.5 kW and 900 W, respectively. The friction and
(d) maximum of the static torque with the phase windage losses are 1050 W and the core losses are
winding unexcited 1200 W. The air-gap power of the motor will be
(GATE 2008: 1 Mark) (a) 23.06 kW (b) 24.11 kW
Solution:  Detent torque is defined as the maxi- (c) 25.01 kW (d) 26.21 kW
mum load torque that can be applied to the shaft (GATE 2008: 2 Marks)
of an unexcited motor, without causing continuous
rotation. Solution:  Given that: V = 400 V, stator current loss
Ans. (d) = 1.5 kW, f = 50 Hz, rotor current loss = 900 W,
P = 30 hp, friction and windage loss PF & W = 1050 W
78. It is desired to measure parameters of 230 V/115 V,
I = 50 A, core loss = 1200 W and power factor =
2 kVA, sing1e-phase transformer. The following
0.8 lag.
wattmeters are available in a laboratory:
We have
W1: 250 V, 10 A, Low power factor
W2: 250 V, 5 A, Low power factor Pin = 3VL I L cos f = 3 × 400 × 50 × 0.8
W3:150 V, 10 A, High power factor = 27.7 kW
W4: 150 V, 5 A, High power factor

Chapter 4.indd 356 3/23/2016 11:38:37 AM


SOLVED GATE PREVIOUS YEARS’ QUESTIONS     357

Therefore, air-gap power is Solution:  Given N1 = 100 and N2 = 200 and


Pg = 27.71 − (1.5 + 1.2) = 25 kW df
fm = 0.12 =
dt
Ans. (c)
81. The core of a two-winding transformer is subjected to We know that
a magnetic flux variation as indicated in the figure. E1 ∝ N1

f Therefore,
p r
+ + E2 ∝ 2E1
epq
df
100 200 ers
E = −N
− − dt
q s
In the interval, 0 < t < 1,
df
E1 = −100 = −100 × 0.12 = −12 V
f(Wb) dt
0.12
E2 = 2 E1 = 2 ×−12 = −24 V

(E1 and E2 are in opposition.)


In the interval, 1 < t2 < 2
0 1 2 2.5 t(s)
df
The induced emf (ers) in the secondary winding as = 0, E1 = E2 = 0
dt
a function of time will be of the form
In the region, 2 < t < 2.5,
(a)
df
ers
E1 = −100 = + 24 V
dt
24 V
E2 = +48 V
2 2.5
0 1 t(s)
−48 V Ans. (b)
82. A 400 V, 50 Hz, 4 pole, 1400 rpm, star-connected
(b) ers
squirrel cage induction motor has the following
parameters referred to the stator
48 V
0 1 Rr′ = 1.0 Ω, Xs = Xr′ = 1.5 Ω
2 2.5
−24 V t(s) Neglect stator resistance and core and rotational
losses of the motor.
The motor is controlled from a three-phase volt-
(c) ers age source inverter with constant V/f control. The
stator line-to-line voltage (rms) and frequency to
48 V obtain the maximum torque at starting will be
24 V
(a) 20.6 V, 2.7 Hz (b) 133.3 V, 16.7 Hz
0 1 2 2.5 t(s) (c) 266.6 V, 33.3 Hz (d) 323.3 V, 40.3 Hz
(GATE 2008: 2 Marks)
(d) ers
Solution:  Given that V = 400 V, f = 50 Hz, p = 4,
N = 1400 rpm, R = 100 Ω and Xs = Xr′ = 1.5 Ω
0 1 2 2.5 We have
−24 V t(s) Rr¢
−48 V
Sm =
¢
Xsm + Xrm

(GATE 2008: 2 Marks) At starting, Sm = 1, therefore,

Chapter 4.indd 357 3/23/2016 11:38:56 AM


358     Chapter 4:  Electrical machines 

¢ = R ¢r
Xsm + Xrm The motor is coupled to a 220 V, separately excited
DC generator feeding power to fixed resistance of
2pfm Ls + 0.2pfm Lr′ = 1 10 Ω. Two-wattmeter method is used to measure
the input power to induction motor. The variable
Frequency at maximum torque, resistance is adjusted such that the motor runs at
1410 rpm and the following readings were recorded
1
fm =
2p (Ls + Lr¢ ) W1 = 1800 W, W2 = −200 W
Xs 1.5 83. The speed of rotation of stator magnetic field with
Ls = =
2pf 2p × 50 respect to rotor structure will be

Lr¢ =
1.5 (a) 90 rpm in the direction of rotation
2p × 50 (b) 90 rpm in the opposite direction of rotation
Therefore, (c) 1500 rpm in the direction of rotation
1 (d) 1500 rpm in the opposite direction of rotation
fm =
é 1.5 1.5 ù
2p ê (GATE 2008: 2 Marks)
× 50 úû
+
ë 2p × 50 2p
Solution:  Given that: number of phases = 3,
1
= = 16.5 Hz N = 1410 rpm, V = 440 V, W1 = 1800 W, f = 50 Hz,
3/50 W2 = -200 W p = 4 and the motor coupled is to
220 V. We have
By constant V/f method,
120f 120 × 50
Ns = = = 1500 rpm
V1 400 p 4
= = 8 = constant
f1 500 Relative speed = Ns−Nr = 1500 − 1410 = 90 rpm
(in the direction of rotation)
Therefore, Ans. (a)
V2
=8 84. Neglecting all losses of both the machines, the DC
f2 generator power output and the current through
V2 = 8 × f2 resistance (Rext) will, respectively, be

= 8 × 16.7 (a) 96 W, 3.10 A (b) 120 W, 3.46 A


(c) 1504 W, 12.26 A (d) 1880 W, 13.71 A
= 133.3 V
(GATE 2008: 2 Marks)
Hence stator line-to-line voltage is 133.3 V and
frequency required to obtain maximum torque is Solution:
16.7 Hz.
Ans. (b) Ns − Nr 90
s= = = 0.06
Common Data for Questions 83 and 84: A Ns 1500
three-phase, 440 V, 50 Hz, 4-pole, slip-ring induc- Pin = W1 + W2 = 1800 − 200 = 1600 W
Po = (1 − s)Pin = (1 − 0.06)1600 = 1504 W
tion motor is fed from the rotor side through an
autotransformer and the stator is connected to a
variable resistance as shown in the figure. Pin = Po [ neglecting losses ]

I 2R = 1504
1504
I= = 12.26 A
10

Induction Hence,
Three- Rext
motor
phase, V = 240 V, Ra = 0.5 W
50 Hz, I L = 15 A (line current), Rsh = 80 W
supply N = 80 rad/s
Autotransformer + 220 V −
Ans. (c)

Chapter 4.indd 358 3/23/2016 11:39:15 AM


SOLVED GATE PREVIOUS YEARS’ QUESTIONS     359

Linked Answer Questions 85 and 86: A 240 V, (a) 0.8 pu and 36.86° lag
DC shunt motor draws 15 A while supplying the (b) 0.8 pu and 36.86° lead
rated load at a speed of 80 rad/s. The armature (c) 1.17 pu and 30.96° lead
resistance is 0.5 Ω and the field winding resistance (d) 1.17 pu and 30.96° lag
is 80 Ω.
(GATE 2008: 2 Marks)
85. The net voltage across the armature resistance at
the time of plugging will be Solution:  Giventhat:V = 1Ð0° pu and Ia = 0.6Ð0°pu
We have
(a) 6 V (b) 234 V
(c) 240 V (d) 474 V Zs = Ra + jXs
(GATE 2008: 2 Marks) = 0 + j = 1Ð90° pu
Solution:  Given that: V = 240 V, Ra = 0.5 Ω, Also,
IL = 15 A, Rsh = 80 Ω and N = 80 rad/s. V = E∠d + Ia Zs
We have
E∠d = 1∠0° − (0.6∠0° × 1∠90° )
V = Eb + Ia Ra
= 1.166∠ − 30.96 pu
I L = Ia + I sh
V 240 Therefore, magnitude of excitation voltage =
I sh = = = 3A
Rsh 80 1.17 pu and load angle d = 30.96° (lagging)
Ia = I L − I sh Ans. (d)
= 15 − 3 = 12 A 88. Keeping the excitation voltage same, the load on
the motor is increased such that the motor current
Eb = V − Ia Ra = 240 − (12 × 0.5) = 234 V increases by 20%. The operating power factor will
Vplugging = V + Eb become
= 240 + 234 = 474 V (a) 0.995 lagging (b) 0.995 leading
(c) 0.791 lagging (d) 0.848 leading
Ans. (d)
(GATE 2008: 2 Marks)
86. The external resistance to be added in the arma-
ture circuit to limit the armature current to 125% Solution:

Ia = 1.2 × 0.6 = 0.72 pu


of its rated value is
(a) 31.1 Ω (b) 31.9 Ω E(−d ) = V∠0° − Ia ∠f °Zs
(c) 15 Ω (d) 15.9 Ω
1 − 0.6 j = 1∠0° − 0.72∠f °1∠90°
(GATE 2008: 2 Marks) 1 − 0.6 ∠90° = 1 − 0.72∠f °1∠90°
Solution:  On adding external resistance Rext to 0.6 ∠90° = 0.72∠f °∠90°
limit Ia to 125% 0.6
∠f ° = = 0.833
V 0.72
Ia = 180
Ra + Rext ∠f ° = 0.833 × = 47.74 rad
p
125 474
12 × = Therefore, cosf = 0.6724 leading. The nearest
100 0.5 + Rext
option is 0.848 leading
⇒ Rext = 31.1 Ω Ans. (d)
Ans. (a) 89. A field excitation of 20 A in a certain alternator
Linked Answer Questions 87 and 88: A syn- results in an armature current of 400 A in short-circuit
chronous motor is connected to an infinite bus at and a terminal voltage of 2000 V on open-circuit.
1.0 pu voltage and draws 0.6 pu current at unity The magnitude of the internal voltage drop within
power factor. Its synchronous reactance is 1.0 pu the machine at a load current of 200 A is
and resistance is negligible.
(a) 1 V   (b) 10 V   (c) 100 V   (d) 1000 V
87. The excitation voltage (E) and load angle (d) will,
respectively, be (GATE 2009: 1 Mark)

Chapter 4.indd 359 3/23/2016 11:39:26 AM


360     Chapter 4:  Electrical machines 

Solution:  Given that I f = 20 A , Ia = 400 A, Solution:  At point A: As speed increases, the torque
Vsc = 200 V, I oc = 200 A. increases and as speed decreases, the torque decreases.
At point B: As speed increases, the torque
Therefore, internal resistance decreases.
Vsc Therefore, A is stable and B is unstable.
=5 Ω
2000
Rint = = Ans. (a)
Ia 400
Internal voltage drop 92. A 220 V, 50 Hz, single-phase induction motor has
the following connection diagram and winding ori-
Vint = 5 × 200 = 1000 V entations shown. MM′ is the axis of the main stator
Ans. (d) winding (M1, M2) and AA′ is that of the auxil-
90. The single-phase, 50 Hz, iron core transformer in iary winding (A1, A2). Directions of the winding
the circuit has both the vertical arms of cross-­ axes indicate direction of flux when currents in the
sectional area 20 cm2 and both the horizontal arms windings are in the directions shown. Parameters
of cross-sectional area 10 cm2. If the two windings of each winding are indicated. When switch S is
shown were wound instead on opposite horizontal closed, the motor
arms, the mutual inductance will
M1
rm = 0.1 Ω
Lm = 0.1/p H
ra = 1 Ω
M2
La = 10/p H M
S A1 A2 A A′
(a) double (b) remain same
(c) be halved (d) become one quarter 220 V Rotor
(GATE 2009: 1 Mark) 50 Hz
M′
NBA
Solution:  Inductance, L =
1 (a) rotates clockwise
Inductance is directly proportional to cross-­ (b) rotates anticlockwise
sectional area. Therefore, when area of cross-­section (c) does not rotate
is halved, inductance becomes half.
(d) rotates momentarily and comes to a halt
Ans. (c)
91. A three-phase squirrel cage induction motor sup- (GATE 2009: 2 Marks)
plied from a balanced three-phase source drives a
mechanical load. The torque − speed characteristics Solution:
of the motor (solid curve) and of the load (dotted Zaux = ra + jxLa
curve) are shown in the figure. Of the two equilib-
10
rium points A and B, which of the following options = 1 + j(2p × 50) ×
correctly describes the stability of A and B? p
= 1 + j1000  1000 ∠90°

Zm = rm + fxm
Torque

B
= 0.1 + j(2p × 50) ×
0. 1
A
p
= 0.1 + j10 = Ð90°
0 1.0 N/Nsync
= Ia Ð − 90°
V
Iaux =
Z Aw
0
  (a) A is stable, B is unstable
= I M Ð − 90°
(b) A is unstable, B is stable V
IM =
   (c) Both are stable ZM
(d) Both are unstable
(GATE 2009: 1 Mark) Therefore, starting torque = kIa I m sin(Ia , I m ) = k′ sin(q ) = 0
= kIa I m sin(Ia , I m ) = k ′ sin(q ) = 0

Chapter 4.indd 360 3/23/2016 11:39:41 AM


SOLVED GATE PREVIOUS YEARS’ QUESTIONS     361

So motor does not rotate at starting. Therefore, Solution:  The voltage waveform VA′B′ resembles
single phase motor is not self-starting.
Ans. (a) VA′B′
93. The given figure shows the extended view of a
2-pole DC machine with 10 armature conductors. wt
0
p
Normal brush positions are shown by A and B,
0.2p 0.4p 0.6p 0.8p
placed at the interpolar axis. If the brushes are now
shifted in the direction of rotation to A′ and B′ as Ans. (a)
shown, the voltage waveform VA′B′ will resemble Common Data for Questions 94 and 95:
The star−delta transformer shown below is excited
on the star side with a balanced, 4-wire, three-
N S phase, sinusoidal voltage supply of rated magni-
tude. The transformer is under no-load condition.
B′ A′
B - A + A a
- +
B b
1 2 3 4 5 1′ 2′ 3′ 4′ 5′
C c
Rotation at speed w rad/s
N
S1 S2
(a)
94. With both S1 and S2 open, the core flux waveform
VA′B′ will be
(a) a sinusoid at fundamental frequency
wt (b) flat-topped with third harmonic
0 (c) peaky with third harmonic
0.2p 0.4p 0.6p 0.8p p (d) none of these
(GATE 2009: 2 Marks)
(b)
VA′B′ Solution:  When the switches S1 and S2 are open,
the star connection consists of third harmonics in line
wt current and attains saturation (due to hysteresis).
0 Ans. (b)
0.2p 0.4p 0.6p 0.8p p
95. With S2 closed and S1 open, the current waveform
in the delta winding will be
(c)
(a) a sinusoid at fundamental frequency
VA′B′ (b) flat-topped with third harmonic
(c) only third harmonic
wt (d) none of these
0
0.2p 0.4p 0.6p 0.8p p (GATE 2009: 2 Marks)

Solution:  When S1 is open and S2 is closed, the


(d) connection is open delta. The output will be sinu-
VA′B′ soidal at fundamental frequency.
Ans. (a)
wt Common Data for Questions 96 and 97:
0
0.2p 0.4p 0.6p 0.8p p The circuit diagram shows a two-winding, lossless
transformer with no leakage flux, excited from a
(GATE 2009: 2 Marks) current source, i(t), whose waveform is also shown.

Chapter 4.indd 361 3/23/2016 11:39:50 AM


362     Chapter 4:  Electrical machines 

The transformer has a magnetising inductance of Linked Answer Questions 98 and 99: The figure
400/p mH. given above shows coils 1 and 2, with dot markings as
1:1 A shown, having 4000 and 6000 turns, respectively. Both
the coils have a rated current of 25 A. Coil 1 is excited
S
with single phase, 400 V, 50 Hz supply.

30 Ω
i(t) A C
Coil 1 Coil 2

B B D
98. The coils are to be connected to obtain a single
i(t) phase, 400/1000 V autotransformer to drive a load
of l0 kVA. Which of the options given should be
10 A exercised to realise the required autotransformer?

0 (a) Connect A and D; Common B


5 ms 10 ms 15 ms 20 ms 25 ms 30 ms t (b) Connect B and D; Common C
10 A (c) Connect A and C; Common B
(d) Connect A and C; Common D
(GATE 2009: 2 Marks)
96. The peak voltage across A and B, with S open is
Solution:  Given that N1 = 4000, N 2 = 6000, I = 25 A, f = 5
(a) 400/p V (b) 800 V
N1 = 4000, N 2 = 6000, I = 25 A, f = 50 Hz, V = 400 V .
(c) 4000/p V (d) 800/p V
(GATE 2009: 2 Marks) Coils are to be connected to obtain single phase,
400/1000 V autotransformer to drive a load 10 kVA.
Solution:  When S is open, peak voltage across A ´ 10be
This 10can 3 realised, if we connect A and D;
and B is given by I1 = = 25 A
common to B.
400
10 ´ 103
di Ans. (a)
V =M = M × (slope of I − t curve) I2 = = 10 A
dt
99. In the1000
autotransformer obtained in Question 98,
é 10 ù
× 10−3 × ê
400
= ú the current in each coil is
p ë 5 × 10−3 û
800 (a) Coil-1 is 25 A and Coil-2 is 10 A
= V (b) Coil-1 is 10 A and Coil-2 is 25 A
p
(c) Coil-1 is 10 A and Coil-2 is 15 A
Ans. (d) (d) Coil-1 is 15 A and Coil-2 is 10 A
97. If the waveform of i(t) is changed to i(t)
(GATE 2009: 2 Marks)
= 10 sin (l00pt) A, the peak voltage across A and
Solution:  Given that load = 10 kVA. Therefore,
B with S closed is
primary current400/1000 V V
400/1000
(a) 400 V (b) 240 V 10 ´ 103 3
(c) 320 V (d) 160 V I1 =I = 10 ´ 10
= 25=A25 A
1 400
400
10 ´ 103 3
I2 =I = 10 ´ 10
(GATE 2009: 2 Marks)
Secondary current = 10=A10 A
2 1000
Solution:  When S is closed, peak voltage across A 1000
and B is given by Thus, current through Coil-1 = I1 — I2 = 15 A
Current through Coil-2 = I2 = 10 A
d(10 sin 100pt) C
× 10−3
di 400
V =M =
p
10 A
dt dt
× 10−3 × 10 × 100p cos 100pt
400 25 A
=
p A 15 A 1000 V
= 400 cos 100pt

Therefore, peak voltage is 400 V. B D


Ans. (a) Ans. (d)

Chapter 4.indd 362 3/23/2016 11:39:58 AM


SOLVED GATE PREVIOUS YEARS’ QUESTIONS     363

100. A single-phase transformer has a turns ratio of 1:2 Ns − Nr 1500 − 1600 −1


and is connected to a purely resistive load as shown in s= = =
Ns 1500 15
the figure. The magnetising current drawn is 1 A, and
the secondary current is 1 A. If core losses and leakage = −0.066 (negative)
reactances are neglected, the primary current is
Hence, induction machine acts as induction genera-
(a) 1.41 A (b) 2 A tor, DC machine acts as DC motor.
(c) 2.24 A (d) 3 A Ans. (c)

1:2 1A
Common Data for Questions 102 and 103:
A separately excited DC motor runs at 1500 rpm
under no-load with 200 V applied to the armature.
The field voltage is maintained at its rated value.
The speed of the motor, when it delivers a torque of
(GATE 2010: 1 Mark) 5 Nm, is 1400 rpm as shown in the figure. The rota-
tional losses and armature reaction are neglected.
Solution:  Magnetising current
Speed (rpm)
I0 = 1 A
1500
I2 = 1 A
1400
Secondary current referred to primary
2 ×1
I2¢ = = 2A
1 0 5 Torque (Nm)
Primary current

I p = I 02 + I32 = 12 + 22 = 5 102. The armature resistance of the motor is

= 2.24 A (a) 2 Ω (b) 3.4 Ω


Ans. (c) (c) 4.4 Ω (d) 7.7 Ω

101. A separately excited DC machine is coupled to (GATE 2010: 2 Marks)


a 50 Hz, three-phase, 4-pole induction machine
Solution:  Given N0 = 1500 rpm, Va = 200 V,
as shown in the figure. The DC machine is ener-
T = 5 Nm, N = 1400 rpm
gised first and the machines rotate at 160 rpm.
So, under no load
Subsequently, the induction machine is also con-
nected to a 50 Hz, three-phase source, the phase Eb0 = Va − Ia0 Ra
sequence being consistent with the direction of rota-
Iao = 0
tion. In steady state,
Induction machine Eb 0 = Va = 200 V
DC machine 50 Hz,
We know that N ∝ Eb, therefore
4 pole, 50 Hz
balanced
three-phase N1 E
supply ∝ b1
N2 Eb2
(a) both machines act as generators N2
(b) the DC machine acts as a generator, and the Eb2 = × Eb1
N1
induction machine acts as a motor
1400
(c) the DC machine acts as a motor, and the induc- = × 200 = 186.67 V
tion machine acts as a generator 1500
(d) both machines act as motors
Ia 186.67 × 60
T = Eb = I
(GATE 2010: 2 Marks) w 2p × 1400 a
Solution:  Given that f = 50 Hz, p = 4, N = 1600 rpm Given T = 5  Nm, therefore
We have 5 × 2p × 1400
Ia = = 3.926 A
186.67 × 60
120f 120 × 50
Ns = = = 1500 rpm
p 4 V = Eb + Ia Ra

Chapter 4.indd 363 3/23/2016 11:40:22 AM


364     Chapter 4:  Electrical machines 

Va − Eb 200 − 186.67 Solution:  The phase voltage in the primary side


Ra = = = 3. 4 W 110
Ia 3.926 =
3
Ans. (b) 110
Thus the turns ratio =
103. For the motor to deliver a torque of 2.5 Nm at 1.732 × 220
1400 rpm, the armature voltage to be applied is The delta equivalent load impedance
(a) 125.5 V (b) 193.3 V = (4 × 4 + 4 × 4 + 4 × 4) / 4 = 12 ohm
(c) 200 V (d) 241.7 V Therefore the impedance referred to primary side
 2
= 
1
× 12 = 1 + j0
 2 × 1.732 
(GATE 2010: 2 Marks)
Solution:  Given that T = 2.5 Nm at 1400 rpm.
Ans. (d)
We know that 105.  A 4-point starter is used to start and control the
E I 186.6 × Ia × 60 speed of a
T = b b Þ 2. 5 =
w 2p × 1400  (a) DC shunt motor with armature resistance control
2.5 × 2p × 1400 (b) DC shunt motor with field weakening control
Ia = = 1.963 A
186.6 × 60    (c) DC series motor
(d) DC compound motor
V = Eb + Ia Ra = 186.6 + (1.963)(3.4) = 193.34 V (GATE 2011: 1 Mark)
Ans. (b) Solution:  A DC shunt motor should only rotate
104. A balanced star-connected and purely resistive under rated speed because if the speed is more
load is connected at the secondary of a star-delta than the rated speed, motor will get damaged.
transformer as shown in the following figure. To control the speed of a motor, a rheostat is
The line-to-line voltage rating of the transformer connected to the field side of the motor and by
is 110  V/220V. Neglecting non-idealities of the varying the resistance, we can control the speed
transformer, impedance Z of the equivalent star- of the motor.
connected load, referred to the primary side of the Ans. (a)
transformer, is
106. A three-phase, salient pole synchronous motor
110/220 V r is connected to an infinite bus. It is operated at
R no load at normal excitation. The field excita-
4Ω tion of the motor is first reduced to zero and then
increased in the reverse direction gradually. Then
4Ω
4Ω
the armature current
b
(a) increases continuously
Y (b) first increases and then decreases steeply
y (c) first decreases and then increases steeply
B
(d) remains constant
(GATE 2011: 1 Mark)
R
Solution:  In a salient pole synchronous motor,
Z under normal excitation with no-load condition,
when the field side excitation is reduced, power
Z Z generated will be high because of increase in arma-
ture current. Also, field excitation and armature
Y
current will have inversely proportional relation.
B Therefore, armature current first increases and
then decreases steeply.
(a) (3 + j 0) Ω Ans. (b)
(b) (0.866 − j 0.5) Ω
107. A single-phase air-core transformer, fed from a
(c) (0.866 + j 0.5) Ω rated sinusoidal supply, is operating at no load.
(d) (1 + j 0) Ω The steady state magnetising current drawn by the
(GATE 2010: 2 Marks) transformer from the supply will have the waveform

Chapter 4.indd 364 3/23/2016 11:40:35 AM


SOLVED GATE PREVIOUS YEARS’ QUESTIONS     365

(a) I (b) I 220 − (10)(1)


1= × 0.9
220 − 11.11(Ra + R)
t t 1 + R = 2.79
R = 1.79 W
Ans. (a)
(c) I (d) I 109. A three-phase 440 V, 6-pole, 50 Hz; squirrel cage
induction motor is running at a slip of 5%. The
speed of stator magnetic field with respect to rotor
t t
magnetic field and speed of rotor with respect to
stator magnetic field are

(GATE 2011: 1 Mark) (a) 0, -5 rpm (b) 0, 955 rpm


(c) 1000 rpm, -5 rpm (d) 1000 rpm, 955 rpm
Solution:  The main property of air-core trans-
(GATE 2011: 2 Marks)
former is that there will be no saturation effect.
Hence, output will be pure sinusoidal waveform. Solution:  Given that: Vs = 440 V, p = 6, f = 50 Hz, s = 5% = 0.05
Vs = 440 V, p =(c)
Ans. 6, f = 50 Hz, s = 5% = 0.05
108. A 220 V, DC shunt motor is operating at a speed
of 1440 rpm. The armature resistance is 1.0 Ω and 120f 120 × 50
Ns = = = 1000 rpm
armature current is 10 A. If the excitation of the p 6
machine is reduced by 10%, the extra resistance
to be put in the armature circuit to maintain the N r = N s − sN s
same speed and torque will be 5
= 1000 − (1000)
(a) 1.79 Ω (b) 2.1 Ω 100
(c) 3 Ω (d) 18.9 Ω = 950 rpm
(GATE 2011: 2 Marks)
Stator and rotor magnetic field rotate with the
Solution:  Given that: Va = 220 V, N = 1440 rpm, same speed = 1000 rpm, therefore
Ra =10 W, Ia = 10 A
If the excitation is reduced by 10%, flux will be Relative speed = 1000 − 1000 = 0
reduced by 10%, therefore Nr with respect to stator magnetic field = 950 −
1000 = −50 rpm
 10 
f new = 1 −  f = 0.9f Ans. (No option is correct)
 100 
110. The direct-axis and quadrature axis reactances of
The torque remains constant. a salient pole alternator are 1.2 pu and 1.0 pu,
respectively. The armature resistance is negligible.
Ia f = Ia(new)f (new) If this alternator is delivering rated kVA at unity
power factor and at rated voltage, then its power
Ia f 10f angle is
Ia(new) = = = 11.11 A
fnew 0.9f
(a) 30°   (b) 45°   (c) 60°   (d) 90°
In general,
Eb (GATE 2011: 2 Marks)
N ∝ , therefore
f Solution:  Given that:
N Eb f xd = 1.2 pu, xq = 1 pu , ra = 0, PF angle = 0, Ia = 1 pu
× × new
N new Ebnew f Ia = 1 pu , Vt = 1 pu
Ia (xq cos q + ra sin q )
Using Eb = V − IaRa, we have Power angle, tan d =
Vt + Ia (xq sin q − ra cos q )
220 − Ia Ra 0.9f1
1= × tan d = 1 ⇒ d = tan−1 1 = 45°
220 − Ia(new)(Ra + R) f1
Ans. (b)

Chapter 4.indd 365 3/23/2016 11:41:10 AM


366     Chapter 4:  Electrical machines 

111. The slip of an induction motor normally does not Therefore,


depend on E1 N f
= 1. 1
(a) rotor speed (b) synchronous speed E2 N2 f2
(c) shaft torque (d) core-loss component
203.55 1000 æ f1 ö
= ç ÷
(GATE 2012: 1 Mark) 207.25 1600 è f2 ø
Solution:  We know that slip is given by f2 = 0.637f1
Ns − Nr f1 − f2
s= % reduction in flux = × 100
Ns f1
f1 − 0.637f1
Therefore, slip depends on synchronous speed (Ns) = × 100 = 36.3% (decrease)
and rotor speed (Nr). f1
Torque increases as slip increases up to a maxi- Ans. (d)
mum value and then decreases. 113. The locked rotor current in a three-phase, star-­
Slip does not depend on core-iron component. connected 15 kW, 4-pole, 230 V, 50 Hz induction
Ans. (d) motor at rated conditions is 50 A. Neglecting losses
112. A 220 V, 15 kW, 1000 rpm shunt motor with arma- and magnetising current, the approximate locked
ture resistance of 0.25 Ω, has a rated line current of rotor line current drawn when the motor is con-
68 A and a rated field current of 2.2 A. The change nected to a 236 V, 57 Hz supply is
in field flux required to obtain a speed of 1600 rpm
(a) 58.5 A  (b) 45.0 A  (c) 42.7 A  (d) 55.6 A
while drawing a line current of 52.8 A and a field
current of 1.8 A is (GATE 2012: 2 Marks)
(a) 18.18% increase (b) 18.18% decrease Solution:  Locked rotor line current
(c) 36.36% increase (d) 36.36% decrease
(GATE 2012: 2 Marks) E2 E2
I2 = =
Solution:  Given that Z2 R22 + X22
V = 220 V, Ra = 0.25 W,
Given R2 = 0, therefore,
N1 = 1000 rpm,
I L1 = 68 A, I L2 = 52.8 A, E2 E E
I2 = = 2 ∝ 2
I F1 = 2.2 A, I F2 = 1.8 A X2 wL2 f
50  230   57 
=
 236   50 
Case (i): We have
I L1 = Ia1 + I F1 I2′

Ia1 = 68 − 2.2 = 65.8 A ⇒ I2′ = 45 A


Ans. (b)
Therefore,
E1 = V − Ia1Ra 114. A single-phase 10 kVA, 50 Hz transformer with
1 kV primary winding draws 0.5 A and 55 W, at
= 220 − (65.8)(0.25) rated voltage and frequency on no load. A second
= 203.55 V transformer has a core with all its linear dimen-
Case (ii): We have sions 2 times the corresponding dimensions of the
first transformer. The core material and lamina-
Ia2 = I L2 − I F2 tion thickness are the same in both transformers.
= 52.8 − 1.8 = 51 A The primary windings of both the transformers
E2 = 220 − (51)(0.25) have the same number of turns. If a rated voltage
of 2 kV at 50 Hz is applied to the primary of the
= 207.25 V
second transformer, then the no-load current and
We know that power, respectively, are
N ∝
E
f
(a) 0.7 A, 77.8 W (b) 0.7 A, 155.6 W
(c) 1 A, 110 W (d) 1 A, 220 W
or, E ∝ Nf
or (GATE 2012: 2 Marks)

Chapter 4.indd 366 3/23/2016 11:41:31 AM


SOLVED GATE PREVIOUS YEARS’ QUESTIONS     367

Solution:  Given that area a2 = 2a1, length l2 = 2l1 Copper loss, Wcu = 81 W (at 90% load)
We know that magnetising inductance, Therefore, at x% load,

I 
2
N 2 ma a Wc u (x) = Wc ×  x 
L= ⇒L∝  I fl 
l l
L
L11 = a
a1 ll2 Wc u(x) = Wc × x2
L = a1 .. l2
L22 a22 l11 1
Wcu = Wcu(x)
L
L11 = a
a1 22ll1 x2
= 2a1 .. 1
1
= 81 ×
L
L22 2a11 ll1
1 = 100 W
(0.91)2
⇒ l2 = 2l1
Under maximum efficiency,
To find magnetising current IM2
Wi 64
X = 2X X= = = 0.8 = 80%
XM2 =M2 2XM1 M1 Wcu 100
V Ans. (c)
I M V=
IM = X
XM M 116. The flux density at a point in space is given by
I V XM2 
I M1 M1V1= X1M2 . B = 4xa x + 2ky a y + 8a z Wb/m2 . The value of
I= . V X
I M2 M2V2 X2M1 M1 constant k must be equal to
I æ V ö æ 2 × M1 ö
I M1 M1æ = V1 ç ö æ 1 2÷ çç× M 1 ö ÷÷ (a) -2    (b) -0.5    (c) +0.5    (d) +2
I= ç ÷2çV X ÷÷ ø
I M2 M2è 2V1è ø çè 1 øXè M1 M1 ø
Þ I M2 = 2I M1
(GATE 2013: 1 Mark)
Þ I M2 = 2I M1
I = 2 × 0.5 = 0.707 A Solution:  Given that
I M2 =M2 2 × 0.5 = 0.707 A

V = 2V and I = 2I
V2 = 22 V1 and1 I2 = 2 2I1 1 B = 4xa x + 2ky a y + 8a z

We know that
∂ ∂ ∂
Therefore, power
∇.B = 0 ⇒ (4x) + (2k ) (8) = 0
P2 = 2 2 × 55 = 155.6 W ∂x ∂x y ∂z

Ans. (b) Therefore,

115. A single-phase transformer has no-load loss of 4 + 2k + 0 = 0


64 W, as obtained from an open-circuit test. k = −2
When a short-circuit test is performed on it with Ans. (a)
90% of the rated currents flowing in its both LV
117. Leakage flux in an induction motor is
and HV windings, the measured loss is 81 W. The
transformer has maximum efficiency when oper- (a) flux that leaks through the machine
ated at (b) flux that links both stator and rotor windings
(c) flux that links none of the windings
 (a) 50.0% of the rated current
(d) flux that links the stator winding or the rotor
(b) 64.0% of the rated current winding, but not both
   (c) 80.0% of the rated current
(d) 88.8% of the rated current (GATE 2013: 1 Mark)

(GATE 2013: 1 Mark) Solution:  Leakage flux of induction motor is the


flux that links the stator winding or rotor winding,
Solution:  Given that: but not both.
Core loss, Wcore = 64 W Ans. (d)

Chapter 4.indd 367 3/23/2016 11:41:56 AM


368     Chapter 4:  Electrical machines 

118. A 4-pole induction motor, supplied by a slightly W


unbalanced three-phase 50 Hz source, is rotating at 1:1.25
1440 rpm. The electrical frequency in Hertz of the
induced negative sequence current in the rotor is Y
(a) 100    (b) 98    (c) 52    (d) 48
(GATE 2013: 2 Marks)
X Z
Solution:  Given that: p = 4, f = 50 Hz, N r = 1440 rpm (GATE 2013: 2 Marks)
f = 50 Hz, N r = 1440 rpm
120 × f 120 × 50 Solution:  For the given transformer
Ns = = = 1500 rpm
p 4
N − Nr 1500 − 1440 60 V 1.25
s= s = = = 0.004 =
Ns 1500 1500 V WX 1

Therefore, electrical frequency (Hz) of induced Attenuation factor


negative sequence current in rotor V YZ
= (2 − s)fs = (2 − 0.04) 50 = 98 Hz
= 0.8
V
Ans. (b) Therefore,
119. The following arrangement consists of an ideal trans- V YZ
former and an attenuator which attenuates by a factor = (0.8) × 1.25 = 1
V WX
of 0.8. An AC voltage VWX1 = 100 V is applied across
⇒ V YZ = V WZ
WX to get an open circuit voltage VWX2, across YZ.
Next, an AC voltageVYZ2 = 100 V is applied across V YZ1 100
YZ to get an open circuit voltage VWX2 across WX. When VWX1 = 100 V ⇒ =
V WX1 100
Then, VYZ1VWX1, VWX2 /VYZ2 are, respectively.

(a) 125/100 and 80/100 V WX2 100


When VWX2 = 100 V ⇒ =
(b) 100/100 and 80/100 V YZ2 100
(c) 100/100 and 100/100 Ans. (c)
(d) 80/100 and 80/100

Chapter 4.indd 368 3/23/2016 11:42:13 AM


SECTION V: POWER SYSTEMS

MARKS DISTRIBUTION FOR GATE QUESTIONS

12
11
10
9
Number of questions

8
7
6
5 Marks 1
4 Marks 2
3 Total number of questions

2
1
0
2009 2010 2011 2012 2013 2014 2015

Chapter 5.indd 369 3/23/2016 11:57:01 AM


TOPIC DISTRIBUTION FOR GATE QUESTIONS

Year Concepts
2015 Transmission and distribution, Economics and power generation, Symmetrical
components and fault calculation, Power system and stability, Generating station,
Transient and over voltage
2014 Fault analysis, Economic operation
2013 Swing curve and equal area criteria, Power generation and load flow, System stability
2012 Load flow analysis, Symmetrical component, Economic operation, Fault analysis
2011 Transmission line, Relay, Load flow, Economic operation, Fault analysis
2010 Surge impedance, Circuit breaker, Fault analysis, Transmission line, Conductors
2009 Power station practice, Transmission line, Load flow analysis, Relay, Stability analysis

Chapter 5.indd 370 3/23/2016 11:57:01 AM


CHAPTER 5

POWER SYSTEMS

This chapter covers topics related to power systems. The main component of power system is the generator
These include transmission lines and their parameter which converts mechanical energy to electrical energy in
calculations, regulations, different insulators and their general. The synchronous machines known as alterna-
descriptions and different compensators for transmission tors are used to produce high power with high efficiency.
lines and both symmetrical and asymmetrical faults of Therefore, these machines are widely used as generators
transmission lines along with the relevant calculations. in power systems. A generator serves two basic func-
Different protection schemes with use of relays and circuit tions. The first one is to produce active power (MW) and
breakers and system stability concepts are also covered. the second function is to produce reactive power (MVar)
as demanded by load.
5.1  POWER GENERATION CONCEPTS Transmission system comprises of transformers,
transmission towers and lines. The transmission of power
needs to be carried out at high voltages, so power trans-
The power systems are designed with the purpose of gen- formers are used to set up voltage levels. The power is
erating electric power and supplying it to various types of transmitted by means of transmission towers and lines
loads spread over large areas by means of transmission and and voltage levels get reduced at each stage. The elec-
distribution network. The main components are therefore: trical power is distributed for local usage by means of
  1.  Generation system distribution systems which are designed based on AC
  2.  Transmission system or DC inputs and the level of voltages required by the
  3.  Distribution system loads, that is, residential, commercial and industrial
  4.  Loads consumption.

Chapter 5.indd 371 3/23/2016 11:57:01 AM


372     Chapter 5:  POWER SYSTEMS 

Based on the nature of the fuel that is used for genera-   7.  Diversity factor: It is ratio of maximum demand
tion of power, the power plants are classified as: from different consumer types to the maximum
  1.  Thermal power plants: In these, the prime demand of the system.
mover for driving the alternator can be steam tur- Sum of individual maximum demands
bine (steam generated from coal or gas) or diesel Maximum demand of the systtem
engine.
Its value is greater than or equal to 1.
  2.  Hydroelectric power plants: In these, the
mechanical (potential) energy of water at a high   8.  Plant capacity: It is capacity for which the plant
level is used for moving the turbine. These are clas- is designed and is the sum of the ratings of the gen-
sified as low, medium and high head plants and erators in the plant. It should be greater than the
the common prime movers used based on range of maximum demand on the plant.
heads are Propeller (<30 m), Francis (30 to 200 m),   9.  Plant use or Utilisation factor: It is defined as
and Pelton (>200 m), respectively. Station output in kWh
  3.  Nuclear power plants: These are classified on Plant capacity × hours of use
the basis of nuclear fuel and reactor used
 10.  Plant capacity factor: It is defined as
A power system should essentially be stable, secure and
Average demand
reliable. and meet fundamental objectives of:
Plant capacity(maximum installed capacity)
  1.  Supplying power at minimum cost.
  2.  Meet the varying demand in active and reactive  11. Power types: Power guaranteed by the supplier
power. to be available at all times for the agreed period
  3.  Maintain the quality of power, in terms of volt- is called firm power. Power required when there is
age and meet the standards related to frequency, no other source of power available is prime power.
amplitude and wave shape. Power that is available as a backup in event of
a failure is called standby power. Electric power
These objectives are met with power system control generated by the system that is in excess of the
measures that are applied at power generation, trans- maximum need is called dump power.
mission and distribution stages.

5.1.1  Some Standard Terms used in Power 5.2  TRANSMISSION LINE


Systems PARAMETERS

Some standard terms related to power systems are listed


and defined as follows: The transmission lines can be represented by their
equivalent circuit model for analysing different transmis-
  1.  Load curve: It is the plot of load (power demand)
sion parameters. The equivalent circuit will consist of
vs. time. It can be plotted to show daily, weekly,
its resistance R, inductance L between the conductors
monthly or yearly variation in loads. It provides
and capacitance C between conductors and to ground.
inputs on variation in demand as well as the maxi-
These (R, L and C ) are generally configured as series, T
mum and minimum load requirement.
or Π models either as lumped or distributed parameters
  2.  Connected load: It is the total load connected
according to the length of the line. These parameters and
to the system.
their calculations are discussed in the following sections.
  3.  Maximum demand: It is the maximum load on
the system. It is generally less than connected load
and can have a maximum value equal to it. 5.2.1  Inductance
  4.  Demand factor: It is the ratio of maximum
demand to the connected load. The inductance L is the equivalent inductance between
  5.  Average load: It can be calculated on daily, the line conductors. The general expression for induc-
monthly or yearly basis, by dividing the total tance is given by
number of units supplied (in kWh) by the sta- y Nf
L= =
tion with time factor number of hours in the time I I
denomination.
  6.  Load factor: It is the ratio of average demand where y is the flux linkage, I is the current carried by
and maximum demand. It is always less than 1. A the conductor and f is the flux produced for current I.
value close to 1, shows good utilisation of the plant Thus, Nf is the flux linkage of the circuit, expressed in
capacity. weber turns.

Chapter 5.indd 372 3/23/2016 11:57:09 AM


5.2  TRANSMISSION LINE PARAMETERS     373

5.2.1.1 Inductance Calculations Inductance of Composite Conductors


The general expression for inductance of conductor A
Case 1: Two-wire transmission line
acting as a composite conductor having two groups, A
In a two-wire transmission line; there are two types of and B, with the numbers m and s in the two groups is
flux linkages occur; one is due to self-flux of one line given by,
m I D 
given by 0 and the other is due to flux produced by LA = 2 × 10−7 ln  m  H/m (5.3)
8p  Ds 

the other line given by where Dm is the geometric mean distance (GMD) given
m0 I  D  by
ln  
2p  r
¢ D12
Dm = mn (D11 ¢ ...D1¢n )(D21
¢ D22
¢ ...D2¢ n )...(Dm
¢ 1Dm
¢ 2 ...Dmn
¢ )
Here, D is the distance between the centres of the con-
ductors and r is the actual radius of the conductor. Thus, and Ds is the geometric mean radius (GMR) or self
total flux linkage of one conductor is GMD given by
m0 I m I  D Ds = m (r ¢D12D13 ...D1m )(r ¢D21D23 ...D2m )...(r ¢Dm1Dm2 ...Dmm )
2

y = + 0 ln  
8p 2p  r
Total inductance due to conductors A and B, L = LA+ LB.
The total inductance of the line due to both the conduc-
tors per unit length is given by
5.2.2  Capacitance
m   D 
L= 0
p 0.25 + ln  r   H (5.1) The capacitances along with conductance form the shunt
 
admittance of transmission line.
Case 2: Group of conductors in a transmission The electric field intensity E at a point A having dis-
line tance y metres from the conductor P with a charge of q
coulomb/metres can be given by
Although, there are a number of conductors, the self-
flux will be one and the mutual flux will be due to other q
conductors in the group. So, the total flux linkage of one E=
2pe y
conductor can be given by
where e is the permittivity of the medium, given by e0 er,
y 1 = 2 × 10−7 {I1 ln(1/r1 ) + I2 ln(1/D12 ) + I3 ln(1/D13 ) +
(here er is the relative permittivity of the medium and e0
... + I n ln(1/D1n )} (5.2) is the vacuum permittivity). Now the potential difference
 between any two points A and B as shown in Fig. 5.1, can
Case 3: Three asymmetrically conductors spaced be calculated as
in a transmission line
B
When the conductors of same resistance in a three-phase
line are asymmetrically spaced and they are carrying
currents as Ia, Ib and Ic; the individual inductance can q P A
be calculated as,
Conductor
−7  1 1 3 b y
La = 2 × 10 ln − ln −j ln 
 r1′ bc 2 c

 1 3 a Figure 5.1 |   Potential difference between points A and B.


Lc = 2 × 10−7 ln − ln
1
−j ln 
 r1′ ba 2 b
ò Edy = ò 2pe y dy V
q
VAB =
 1 3 c
Lb = 2 × 10−7 ln − ln
1
−j ln 
 r1′ ac 2 a
Now if the distances for A and B are D2 and D1 respec-
But, when this asymmetrically transmission line is trans- tively, then
posed, the average inductance can be calculated as
D2
 L + Lb + Lc  ò
q q D
VAB = dy = ln 2 V
L= a  2pe y 2pe D1
 (5.4)
 3 D1

Chapter 5.indd 373 3/23/2016 11:57:27 AM


374     Chapter 5:  POWER SYSTEMS 

5.2.2.1 Capacitance Calculations The same way we can write,


1  D r D
From the expression obtained in Eq. (5.4), the following VAB =  qA ln + qB ln + qC ln 
2pe r D D
cases of capacitor calculation can be considered.
1  D r D
 qA ln + qC ln + qB ln 
Case 1: Two wire line as shown in Fig. 5.2, in which the VAC =
lines are go and return path, so that qA = −qB 2pe r D D
Since for a balanced system qB + qC + qA = 0, therefore
A B
3qA D
rA rB VAB + VAC = ln  (5.7)
2pe r
Also,

D VAB + VAC = VAN − VBN + V AN − VCN


= 2VAN = (VBN + VCN ) = 3VAN

Figure 5.2 |   Two wire line. where N is the neutral point.


So, substituting in Eq. (5.7), we get
Thus the voltage between the two wires can be calcu-
lated as 3qA D
3VAN = ln
1  D rb  2pe r
VAB =  qA ln ra + qB ln D   (5.5)
2pe   Capacitance is given by

qA 2pe 0.0242
Since qA = −qB, Eq. (5.5) becomes, CAN = = = µF/km
VAN ln D/r log(D/r)
qA D2 Case 3: Three phase line with unsymmetrical spacing,
VAB = ln  (5.6)
2pe ra rb as shown in Fig. 5.4:

A
Now capacitance is given by
qA
q 2pe
CAB = =
VAB D2 DAB DCA
ln
ra rb

Substituting e = e0er where e0 = 8.854 × 10−12 F/m qB qC


DBC
0.0121
µF/km
( )
CAB = B C
Figure 5.4 |   Three phase line with unsymmetrical
log D/ ra rb
For ra = rb = r, spacing.
0.0121
CAB = µF/km The capacitance is given by
æDö
log ç ÷ qA 2pe 0.0242
èrø CAN = = = µF/km
VAN  D  Deq 
ln   log 
 r  r 
Case 2: Three phase line of equal spacing and of same
area, as shown in Fig. 5.3.
where Deq = DABDBCDCA
A
qA
5.2.3  Resistance

D D The transmission lines will also have resistance accord-


ing to their length, cross-sections and material used as
conductor, for example, copper, aluminium, etc. The DC
qB qC resistance of a conductor is given by
D
B C l
R=r
Figure 5.3 |   Three phase line with equal spacing. A

Chapter 5.indd 374 3/23/2016 11:57:47 AM


5.3  TRANSMISSION LINES     375

where r is the resistivity of the conductor material, The regulation of a transmission line is defined as the
l is the length of the conductor and A is the area of ratio of the change in receiving end voltage with respect
cross-section. The effective resistance of the conductor to its full load value expressed in per unit (pu) value.
is slightly greater than the DC resistance and given by The same also can be expressed in percentage when mul-
tiplied by 100.
Ploss
R= Thus, percentage voltage regulation
I2
 No load receiving − Full load receiving 
Transmission lines generally have low values of resis-  end voltage end voltage 
tance, so its effect is generally neglected for the calcula- = ×100
Full load receiving end voltage
tion of voltage regulation but is used while estimating
their efficiency. Resistance is affected by skin and prox-
5.3.2  Classification of Transmission Line
imity effect which are discussed as follows.
According to the length, the transmission lines are
5.2.3.1 Skin and Proximity Effect divided into the following three major categories.

Skin Effect 5.3.2.1 Short Transmission Lines


Unlike direct current (DC), where the current distribu-
tion is uniform throughout the entire cross-section of the The transmission lines below the length of 80 km are
conductor, the current density is non-uniform for AC classified as short transmission lines. For these lines,
supply. The degree of non-uniformity depends on the the effect of the shunt parameters such as the capaci-
frequency of supply. This effect is known as the skin tances is neglected, but the series parameters such as the
effect as the current density increases near the surface resistance and the inductances are considered for line
or the skin of the conductor due to magnetic flux having calculations.
set up by the alternating currents inside the conductor. From the equivalent circuit and the phasor diagram,
The skin depth is inversely proportional to the square shown in Fig. 5.5, the relation between the delivering
of the supply frequency. In that way, the effective AC (sending) end and the receiving end parameters can be
resistance of the conductor increases from its DC value. written as,
Vs cos f s = Vr cos f r + I r R
Proximity Effect
When two or more conductors carrying alternating cur- Vs sin fs = Vr sin f r + I r X
rent are placed side by side, the magnetic flux created by where Vr, Ir and fr are the receiving end voltage, current
one conductor sets up a circulating current and the cor- and phase angle and Vs and fs are the sending end volt-
responding flux in other conductor, affecting the current age and phase angle, respectively and X is reactance and
density or the uniformity of current distribution. Thus, R is resistance.
the effective resistance of the conductor changes due to
Thus, regulation (in pu) is expressed as
supply of the current in other conductor. This effect is
known as proximity effect. (V − Vr )
= s pu
Vr
 (5.8)
IR IX
5.3  TRANSMISSION LINES = r cos fr ± r sin f r
Vr Vr

The negative sign for Eq. (5.8) will occur for leading
5.3.1  Transmission Line Performance power factor load. From Eq. (5.8), the regulation can be
expressed as
The efficiency and regulations of transmission lines are = VR cos f r + V X sin f r
the two major performance parameters. The efficiency of
transmission line is defined as the ratio in percentage of where VR and VX are the per unit values of resistance and
the power received at the receiving end to the power sent reactance of the line. From the equivalent circuit, we have
from the delivering end. Thus, Vs = Vr + Ir (R + jX ) = Vr + Ir Z
%age efficiency of tramsmission lines Is = Ir
Power received at the receiving end
= ×100 Here, Z (= R + jX) is the series impedance per unit
Power delivered at sending end length (per kilometer or meter) with units ohms/m.

Chapter 5.indd 375 3/23/2016 11:57:56 AM


376     Chapter 5:  POWER SYSTEMS 
Vs
jIrX
Is R L Ir Is Ir

R + jX Z = R + jX
Load
Vs Vr e Vr
Vs Vr IrR ZL
fs Y = jwX
fr
Ir
Figure 5.6 |   Equivalent circuit of medium
(a)
transmission line.
Vs
jIrX The nominal T and Π models for medium transmission
L Ir lines are discussed as follows.
Nominal T Representation
+ jX
The equivalent circuit and vector diagram for nominal T
Vr representation are given in Figs. 5.7(a) and (b), respec-
Vr IrR tively. Here Z = R + jX.
fs Z/2
fr Is Ir
Ir
(b)
Z/2
Figure 5.5 |   Short transmission line. (a) Equivalent Vs Y = jwC Vr
circuit. (b) Phasor diagram.

In a four terminal passive network the voltage and cur-


rent on the receiving end and sending end are related by (a)
following pair of equations in terms of ABCD constants. E
V s = AV r + B I r
I s = C V r + DI r XL
Is
Thus the ABCD constants for the short transmission 2
lines can be calculated as, Is Z
A = 1, B = 2, C = 0 and D = 1 2 D
C
Also percentage efficiency is given by Is R
2
P
%h = × 100%
Vs
P + 3I r2R V1
XL
where P is the receiving end power and R is the equiva- Ir
2
lent resistance of the line.
Ir Z
2
5.3.2.2 Medium Transmission Lines A
B
Ir R
The transmission lines having the length between 80 km 2
to 160 km are classified as medium transmission lines.
Vr
For these lines, the effect of the shunt parameters such
as the capacitance or conductance is lumped at either fs Is
the sending or the receiving end as per the requirement IC
0
fr
of calculations. The series parameters such as the resis- Ir
tance and the inductances are also considered for the line
calculations. Thus, the possibility of either T or Π model (b)
is considered for these transmission lines. Figure 5.7 |   Nominal T-model for medium
The equivalent circuit representation of the medium transmission line. (a) Equivalent circuit.
transmission line is shown in Fig. 5.6. (b) Vector diagram.

Chapter 5.indd 376 3/23/2016 11:58:01 AM


5.3  TRANSMISSION LINES     377

The efficiency of the transmission line is given by Efficiency is given by

%h =
P
× 100% %h =
P
× 100%
P + 1.5(I s2 + I r2 )R P + 3I s2R
The ABCD constants for the medium transmission lines Percentage voltage regulation is given by
for nominal T representation can be calculated as,
Vr′ − Vr
A = D = 1+
YZ = ×100%
2 Vr

B = Z (1 + YZ /4 ) The ABCD constants for the medium transmission lines


with Π representation can be calculated as
C=Y
YZ
A = D = 1+
Here Z is the series impedance per unit length and Y 2
is the shunt admittance per unit length (expressed in B=Z
Siemens per metre).
C = Y (1 + YZ / 4)
Nominal Π Representation
The equivalent circuit and the phasor diagram for 5.3.2.3 Long Transmission Lines
­nominal Π representation are given in Figs 5.8(a) and (b),
respectively. The lines having a length of more than 160 km is known
as long transmission lines. The line parameters are com-
Is Ir puted using rigorous analysis with equivalent T or Π
models considering uniformly distributed parameters
Z throughout the entire length. The equivalent circuit for
Y = jwL Y = jwL long transmission lines is shown in Fig. 5.9.
Vs Vr
2 2 2 2
Z Z Z Z Z
+ +
Y Y Y Y Y Y
(a)
Vs Vr Zc
Vs C
− −
fs
Figure 5.9 |   Equivalent circuit for long transmission
0 A ILXL
fr Is Vr
ILR lines.
IC2
IL B
Here series impedance, Z = R + jwL; shunt admittance
IC1 Y = G + jwC and Zc is characteristic impedance. The
Ir
voltage and current at a point at distance x from the
receiving end is given by:
(b)
Figure 5.8 |   Nominal Π-model for medium V (x) = cosh(g x)Vr + Zc sinh(g x)I r
transmission line. (a) Equivalent circuit. 1
(b) Vector diagram. I (x) = sinh(g x)Vr + cosh(g x)I r
Zc
Taking the receiving end voltage (Vr) as the reference
of phasor, the current and no load receiving end voltage The current and voltage at the sending end can be
can be expressed as, obtained by replacing x with length of the line l, as
wC wC
I s = I r (cos fr − j sin fr ) + jVr +j
2 2 Vs = cosh(g l)Vr + Zc sinh(g l)I r

Vs (−2 j/wC ) 1
sinh(g l)Vr + cosh(g l)I r
Vr′ = Ir =
R + jX − 2 j/wC Zc

Chapter 5.indd 377 3/23/2016 11:58:18 AM


378     Chapter 5:  POWER SYSTEMS 

Equivalent T Representation The ABCD constants are given by


The equivalent T representation of long transmission A = D = cosh(g l)
line is shown in Fig. 5.10.
B = Zc sinh(g l)
Is Ir sinh(g l)
C=
Z¢/2 Z¢/2 Zc

Vs Y¢ Vr 5.3.3  Combination of Networks

The combination of networks can be cascaded or parallel


and their characteristics are listed as follows.
Figure 5.10 |   Equivalent circuit of nominal T-model   1.  Cascaded network: When two networks of con-
for long transmission line. stants A1, B1, C1 and D1 are connected in series
with another network with of constants A2, B2, C2
The Y and Z parameters are given by, and D2 as shown in Fig. 5.12, the resultant ABCD
constants are given by
Z = R + jX
sinhg l A B A1 B1 A2 B2
Y′=Y C D = C D  + C D 
gl    1 1  2 2
Z tanh(g l/2)
Z′ =
2 (g l/2) Is Ir
A1 B1 A2 B2
where g = a + jb Vs Vr
The ABCD constants are given by C1 D1 C2 D2
YZ
A = D = 1+
2 Figure 5.12 |   Cascaded network.
B=Z   2.  Parallel network: For two parallel networks as
C = Y (1 + YZ /4) shown in Fig. 5.13, the modified ABCD constants
are,
Equivalent Π Representation A B + A2 B1
A=D= 1 2
The equivalent Π representation of long transmission B1 + B2
lines is shown in Fig. 5.11.
B1B2
B=
Is Ir B1 + B2
Z¢ and AD − BC = 1

Y¢ Y¢
Vs Vr
2 2 A1 B1
Vs Vr

Figure 5.11 |   Equivalent circuit of nominal Π-model


C1 D1
of long transmission line.

The Y and Z parameters are given by


A2 B2
sinhg l
Z′ = Z
gl
C2 D2
Y ¢ Y tanh(g l/2)
=
2 2 (g l/2) Figure 5.13 |   Parallel network.

Chapter 5.indd 378 3/23/2016 11:58:37 AM


5.4 AC AND DC TRANSMISSION CONCEPTS     379

5.3.4  Ferranti Effect where Zoc and Zsc are the sending end impedances when
the receiving ends are open-circuited and short-circuited,
When a long transmission line is operated under no load respectively.
or light loaded condition, the charging current for the The characteristic impedance of a transmission
equivalent capacitance to ground can make the receiving line is expressed in terms of the  surge impedance
end voltage greater than the one in sending voltage. This ­loading (SIL), or natural loading of the line. The SIL
effect is known as Ferranti effect. The simplified equiva- is defined as the line loading at which reactive power is
lent circuit of the line considering lumped parameters neither produced nor absorbed and it is expressed as,
and the corresponding phasor diagram are as shown in
Figs. 5.14(a) and (b). VLL2
SIL =
Is L¢ Ir Zc

where, VLL is the line-to-line voltage expressed in Volts


and Zc is the characteristic impedance of the line.
Vs e Vr When the line is loaded below its SIL, it supplies reac-
tive power to the system, helping to raise the system
voltages. On the other hand, above the SIL loading, the
line absorbs reactive power and reduces the line voltage.
(a)
Is
5.4  AC AND DC TRANSMISSION
Vs CONCEPTS
q
IsjwL

Vr
The generation voltage level of the generating stations
(b) is in the lower range between 5−12 kV. Since the load
Figure 5.14 |   Ferranti effect. (a) Simplified centres are far away from the generation places, the gen-
equivalent circuit. (b) Phasor diagram erated power has to be carried to the load points. This
(q ≈ 90°, at no load). purpose is served by the transmission lines at a much
Now considering no load condition where Ir ≈ 0
higher voltage levels than the generated voltages, to
reduce the conductor size and the transmission losses.
Vr = Vs − Ι s ( jw L) As the generation is in AC, the conversion of gener-
Now, the sending end current Is can be given by ated voltage is to the transmission level voltage is easy
with the help of transformers. On the other hand, trans-
Vs
Is = mitting voltage through AC lines has several disadvan-
jwL +
1 tages too. In the AC system, the line will have both
jwC inductance and capacitance which can affect the trans-
Usually C is much smaller, given 1/w C  wL. mission capability of the line, more voltage fluctuation at
Thus, I s = Vs × jwC
the load terminals, skin effect of the line conductors and
proximity effect between the line conductors which can
So, Vr = Vs − Vs jw C × jw L = Vs (1 + w 2 LC ) affect different parameters of the line. Moreover, syn-
Therefore |Vr| > |Vs| which is termed as Ferranti effect. chronisation between the lines is also necessary during
connection of any line to the system. This has led to the
other option of transmitting power through a DC system
5.3.5  Surge Impedance which can eliminate most of the aforesaid disadvantages.
The characteristic impedance of a lossless line is known
as the surge impedance, given by 5.4.1  Comparison of AC and DC Transmission
Z The comparative characteristics of AC and DC trans-
Zc =
Y mission are listed as follows.
where, Z = R + jwL is the series impedance and   1.  Power transmitted: The power transmission
Y = G + jwC is the shunt admittance of the line. It is capabilities of an AC link and a DC link are dif-
also given by ferent. We know that, for the same insulation,
Zc = Zoc ⋅ Zsc the direct voltage Vd is equal to the peak value

Chapter 5.indd 379 3/23/2016 11:58:47 AM


380     Chapter 5:  POWER SYSTEMS 

( 2 × RMS value) of the alternating voltage Va,


for AC to obtain the same reliability. Thus, based
on these electrical and mechanical considerations;
that is Vd = 2Va .
smaller tower are suitable for transmitting DC.
For the same conductor size, the same current can   3.  Ground return: For the same length of trans-
be transmitted with both DC and AC (not consid- mission, the impedance of the ground path is lesser
ering skin effect), that is Id = Ia. for DC than for the corresponding AC transmis-
Thus, the power transmission per circuit for each sion, as the DC spreads over much larger width
conductor, using DC and AC links is given by: and depth. Hence use of ground return is possible.
DC power: Pd = Vd × Id
The ground path resistance in case of DC is almost
entirely dependent on the earth’s electrode resis-
AC power: Pa = Va × Iacosf tance at the two ends of the line and not on the
line length. However, on the flip side ground cur-
The ratio of the power transmitted with DC and
rents may cause electrolytic corrosion of the buried
AC links is given by
conductors and also interfere in the operation of
Pd 2 1.414 at pf = 1 signalling.
= =  (5.9)   4.  Skin effect: In AC systems, the current is not
Pa cos f 1.768 at pf = 0.8
uniformly distributed over the cross-section of the
In practice, AC transmission is carried out using conductor. The current density is higher in the
either single circuit or double circuit in three phase outer region (attributed to skin effect) and results
transmission using three or six conductors. In such in under-utilisation of the conductor’s ­cross-section.
a case, the ratio for power in Eq. (5.9) must be Skin effect is completely absent in DC and hence
multiplied by 2/3 or by 4/3. there is a uniform current in the conductor and the
Let Rd and Ra be the corresponding values of con- conductor metal is better utilised.
ductor resistance for DC and AC, respectively, not   5.  Stability and synchronous interconnec-
considering the skin resistance. Then tions: The DC link is an asynchronous link and
hence any AC supplied through converters or DC
P generation does not require synchronisation with
Id =
Vm the link. For AC links, interconnections between
power systems must be synchronised and thus, sys-
Power loss Ploss = (P/Vm)2 Rd tems of different frequencies cannot be intercon-
nected. Such systems with different frequencies
= (P/Vm)2 ⋅ (rI/Ad) (5.10) can be easily interconnected through high voltage
DC links.
Similarly,
  6.  Cable transmission: The working stress of the
P 2P DC cable insulation is three to four times higher
Ia = =
(Vm / 2 ) cos f Vm cos f than the working stress in AC cable. The absence
of continuous charging current in a DC cable per-
mits higher active power transfer, particularly
Power loss, Ploss =  2P /(V cos f ) 2 R
 m  a across long distances.
  7.  Short circuit fault: When an AC transmission
= 2(P/Vm )2 ⋅ ( rI/Aa cos2f )  (5.11) system is extended, the fault level of the whole
system goes up. This sometimes necessitates the
Equating power loss for DC and AC in Eqs. (5.10) replacement of circuit breakers with those of higher
and (5.11), we get fault levels which is costly. This problem can be
(P /Vm )2 ⋅ ( rI /Ad ) = 2(P /Vm )2 ⋅ ( rI /Aa cos2 f )
overcome with high voltage DC, as it does not con-
tribute current to the AC short circuit beyond its
This gives the result for the ratio of areas as: rated current.
  8.  Corona and radio interference: The corona
Ad cos f 2
0.5 at pf = 1 loss increases with frequency, for a given conduc-
= = tor diameter and applied voltage. There is much
Aa 2 0.2 at pf = 0.8 lower corona loss and hence less radio interference
  2.  Tower size: The DC insulation level required is with DC transmission in comparison to AC trans-
lower than that for AC, for the same power trans- mission. As a result, the use of bundle conductors
mission. Also the DC line will only need two conduc- become unnecessary and leads to substantial saving
tors instead of three conductors (or six) as required in line costs.

Chapter 5.indd 380 3/23/2016 11:58:57 AM


5.5  INSULATED CABLES     381

5.5  INSULATED CABLES atmosphere. The metallic sheath of lead or alumi-


num is generally provided over the insulation as
shown in Fig. 5.15.
Electric power can be transmitted or distributed either
by overhead conductors or by underground cables. The
underground cables have several advantages, such as Metallic
Conductor sheath Armouring
less liable to damage through storms or lightning, low
maintenance cost, smaller voltage drop, etc. However,
the major drawback for the cables is of having greater
installation cost and lower voltage withstanding capabil-
ity compared to the equivalent overhead system. For
this reason, underground cables are generally employed
where it is impracticable to use overhead lines, such as in Insulation Bedding Serving
thickly populated areas, around plants and substations
or where maintenance conditions do not permit the use (a)
of overhead construction.
An underground cable essentially consists of one
or more conductors covered with suitable insulation and Serving
surrounded by a protecting cover. In general, a cable
fulfills the following necessary requirements:
  1.  The conductors used in cables are generally
stranded copper or aluminum of high conductivity.
Stranding is done to increase flexibility and reduce
skin effect. Metallic
  2.  The conductor size should be such that the cable sheath
carries the desired load current with voltage drop
within permissible limits.
  3.  The cable must be provided with suitable mechani-
cal protection so that it may withstand stress while
laying.
  4.  The materials used to manufacture cables should
be such that it is capable of fighting corrosion after
it is laid underground. Conductor Insulation

5.5.1  Construction of Cables (b)


Figure 5.15 |   (a) General construction of cable.
Figure 5.15 shows the general construction of a three-
(b) Cross-section of cable.
conductor cable. The various parts of the cable are
described as follows.   4.  Bedding: It is provided over the metallic sheath
  1.  Cores or Conductors: A cable may have one or and consists of a fibrous material like jute or hes-
more than one core or conductor depending upon sian tape. The main purpose is to protect the metal-
the requirement. A three conductor cable is shown lic sheath against corrosion and from mechanical
in Fig. 5.15 which is used for three-phase system. injury due to armouring.
The conductors are made of stranded tinned copper   5.  Armouring: The armouring is provided over
or aluminum. the bedding, which consists of one or two layers
  2.  Insulation: Each core or conductor is provided of galvanised steel wire or steel tape. It protects
with a suitable insulation and the thickness of the cable from mechanical injury during laying or
which depends upon the voltage to be withstood during the course of handling. Armouring is done
by the cable. The common materials used for insu- in the case of some cables which are not subjected
lations are impregnated paper, varnished cambric to heavy mechanical stress or to reduce weight.
or rubber mineral compound.   6.  Serving: It protects the armouring from atmo-
  3.  Metallic sheath: This is provided to protect spheric conditions. Usually a layer of fibrous mate-
the cable from moisture, gases or other corrosive rial like jute similar to bedding is provided over the
liquids, such as acids or alkalies in the soil and armouring and is known as serving.

Chapter 5.indd 381 3/23/2016 11:58:59 AM


382     Chapter 5:  POWER SYSTEMS 

5.5.1.1 Insulating Materials for Cables


e
The general characteristics of the insulating materials
used in cables are listed as follows: D d
2 2
  1.  High insulation resistance.
  2.  High dielectric strength to avoid electrical break-
down of the cable.
  3.  High mechanical strength.
  4.  Non-hygroscopic, that is, it should not absorb
moisture from air or soil. gmax
  5.  Non-inflammable.
g gmin
  6.  Low cost.
  7.  Unaffected by corrosions from acids and alkalies.
The commonly used insulating materials for cables are
rubber, vulcanised rubber, impregnated paper and poly-
vinyl chloride. x
Figure 5.16 |   Stress distribution in a cable.
5.5.2  Classification of Cables
Hence,
Cables are classified in two ways: (i) according to the 2pe 0e r V
type of insulating material and (ii) according to the Q=  (5.14)
ln D/d
working voltage.
However, the latter method is generally preferred and Substituting for Q from Eq. (5.14) in Eq. (5.12) the
cables are divided into the following groups according to potential gradient is given by
the voltage rating:
V
  1.  Low-tension (LT) cables — up to 1000 V g= V/m  (5.15)
x ln D/d
  2.  High-tension (HT) cables — up to 11,000 V
  3.  Super-tension (ST) cables — from 22 kV to 33 kV It is clear from the above equation that potential gradi-
  4.  Extra high-tension (EHT) cables — from 33 kV to ent is inversely proportional to the distance x. Therefore,
66 kV potential gradient will be maximum where x is mini-
  5.  Extra super voltage cables — beyond 132 kV mum, that is, when x = d/2, which is at the surface of
the conductor. On the other hand, potential gradient
will be minimum at x = D/2, that is, at sheath surface.
5.5.3  Dielectric Stress in Cables Maximum potential gradient is given by

While in operation, the insulation of a cable is sub- 2V


gmax = V/m  (5.16)
jected to electrostatic forces known as dielectric stress. ln D/d

The dielectric stress at any point in a cable is generally
Whereas the minimum value is given by
the potential gradient or electric intensity at that point.
Considering a single core cable as shown in Fig. 5.16, 2V
with core diameter d and internal sheath diameter D, gmin = V/m  (5.17)
D ln D/d
the electric intensity at a point x metres away from the
centre of the cable is given by Hence from Eqs. (5.16) and (5.17), the ratio is given by,
Q gmax D
Ex = V/m
2pe 0e r x gmin
=
d
By definition the electric intensity is equal to potential
gradient. Therefore, potential gradient g is given by, 5.5.4  Most Economical Size of Conductor
Q
g=  (5.12)
2pe 0e r x We have seen that that the maximum stress in a cable
occurs at the surface of the conductor. For safe working
Potential difference V between conductor and sheath is
given by of the cable, dielectric strength of the insulation should
Q D be more than the maximums tress. Therefore, from the
V = ln V 
2pe 0e r
(5.13) expression in Eq. (5.16)
d

Chapter 5.indd 382 3/23/2016 11:59:09 AM


5.5  INSULATED CABLES     383

2V
gmax = V/m e3
ln D/d
e2
The gmax will be minimum when the denominator is
maximum, which can be obtained by, e1

d  d

dd 
d ln  = 0
D
or d
D 2
= e = 2.718
d
Therefore, the most economical conductor diameter is d1
D 2 d2
d= 2
2.718 D
The value of gmax under this condition is given by, gmax 2

2V gmin
gmax = V/m g
d

5.5.5  Grading of Cables

The stress in the dielectric of the cable is non-uniform. x


Figure 5.17 |   Capacitance grading.
The process of achieving uniform electrostatic stress in
the dielectric of cables is known as grading of cables. An
insulation of high dielectric strength is useful near the
conductor where stress is the maximum where as a low permittivity er1, er2 and er3 respectively, such that er1 >
dielectric strength is required as we move away from er2 > er3. If the three dielectrics are at the same maxi-
the conductor. The unequal stress distribution in a cable mum stress, then
e 1d = e 2d1 = e 3d2
results in insulation of greater thickness near the conduc-
tor which increases the cable size and cost. To overcome
the above disadvantages, a uniform stress distribution in gmax d
cables is necessary. This can be achieved by distribut- V1 = d ln 1
2 d
ing the stress in such a way that its value is increased
in the outer layers. This process is known as grading of gmax d
V2 = d1 ln 1
cables. The two main methods of grading of cables are: 2 d
(i) capacitance grading and (ii) inter sheath grading.
gmax D
V3 = d ln
2 2 d2
5.5.5.1 Capacitance Grading
The total potential difference between the core and
The process of achieving uniformity in the dielectric earthed outer sheath is given by
stress by using layers of different dielectrics is known as
capacitance grading. V = V1 + V2 + V3
In capacitance grading, the homogeneous dielectric is gmax  d1 d2 D
V = d ln + d1 ln + d2 ln 
replaced by a composite dielectric which consists of vari- 2  d d1 d2 
ous layers of different dielectrics. The relative permittiv-
ity er of any layer is inversely proportional to its distance 5.5.5.2 Inter Sheath Grading
from the centre. Thus the dielectric stress in the cable
is almost the same everywhere after the grading is done. In inter sheath grading, although a homogeneous dielec-
An ideal grading requires the use of an infinite number tric is used, it is divided into various layers by placing
of dielectrics which is impossible. Generally, two or three metallic inter sheaths between the core and lead sheath.
dielectrics are used in the decreasing order of permittiv- The inter sheaths are held at suitable potentials between
ity from the core as shown in Fig. 5.17. There are three the core potential and earth potential. This arrangement,
dielectrics of outer diameter d1, d2 and D and of relative as shown in Fig. 5.18, improves the voltage distribution

Chapter 5.indd 383 3/23/2016 11:59:21 AM


384     Chapter 5:  POWER SYSTEMS 

in the dielectric of the cable and more uniform potential Since the dielectric is homogeneous, the maximum stress
gradient is obtained as a result. in each layer is the same which can be written alter-
nately as,
g1 max = g2 max = g3 max = gmax
Lead
sheath V1 V2 V3
= =
d d1 d1 d2 d2 D
ln ln ln
2 d 2 d1 2 d2
Inter As the cable behaves like three capacitors in series, the
sheath 1 total voltage between conductor and earthed lead sheath
is given by, V = V1 + V2 + V3
Inter
The principal disadvantages associated with use of
sheath 2
inter sheath are:
  1.  The sheath potentials cannot fixed easily.
d   2.  There can be considerable losses in the inter sheaths
2 due to charging currents.
d1
2 d2
2
5.6  ELECTRIC LINE INSULATORS
D
gmax 2
Electric insulation is very important for design of any
gmin component of electrical power system. The performance
g
of electrical components of a power system is impacted
by the reliability of the insulation design used. Insulation
failure may either cause permanent damage to the elec-
trical equipment or it can temporarily break the load
x schedule. It can lead to considerable of financial losses.
The insulation of an electrical equipment is broadly
Figure 5.18 |   Inter sheath grading. divided into two general categories:
  1.  Internal insulation: It is the own insulation of
The cable shown in Fig. 5.18 has core diameter d and the apparatus enclosed in a grounded housing to
outer lead sheath of diameter D. The two inter sheaths protects it from other internal parts. For exam-
of diameters d1 and d2 are inserted into the homogeneous ple, consider the insulation of a large transformer
dielectric and are maintained at some fixed potentials. between turns and between primary and secondary
The voltages V1, V2 and V3 are the voltage between core coils. An over-voltage stress can produce internal
and inter sheath 1, between inter sheath 1 and 2 and insulation breakdown leading to a permanent fault.
between inter sheath 2 and outer lead sheath respec-   2.  External insulation: It is the insulation of the
tively. The maximum stress between core and inter equipment to protect it from the environment. For
sheath 1 is given by example, consider the ceramic (porcelain) insu-
lators used for supporting transmission line con-
V1 ductors. An over-voltage stress produces only a
g1 max =
d d temporary fault and the insulation restores itself.
log e 1
2 d The external insulators are subjected to a variety
of stresses, including mechanical, electrical and
Similarly, for the other two sheaths, these are, environmental. In addition to regular voltage, they
also need to withstand over voltages, corona losses,
V2
g2 max = switching events. Porcelain and glass insulators
d1 d
log e 2 have been generally used because of their proven
2 d1 resistance to environmental and electrical stresses.
In the recent decades, polymer and composite insu-
V3
g3 max = lators have been introduced and employed widely
d2 D for their better performance in different types of
log e
2 d2 stress.

Chapter 5.indd 384 3/23/2016 11:59:26 AM


5.6  ELECTRIC LINE INSULATORS     385

5.6.1  Electrical Stresses on External  Insulation as those over the width of the river, two or more
insulators can be used in parallel. For low volt-
The external insulations in transmission lines or in the age lines (< 11 kV), shackle insulators are used as
sub-stations are exposed to continuous electrical stress strain insulators.
arising from the applied voltage of an operating power   4.  Shackle insulator: This is typically employed in
system and sudden electrical stress caused by the surge low voltage distribution systems. It can be installed
voltages. Additionally, exposure to the weather and sur- both in horizontal and vertical positions. The
rounding conditions leads to environmental stress on the tapered hole of the shackle insulator distributes
insulators. The electrical stresses on insulation are cre- load more consistently and minimises the option of
ated by: breakage when it is heavily loaded. The conductor
  1.  Continuous power frequency voltages sits in the groove of this insulator and it is clamped
  2.  Temporary over-voltages with the assistance of a soft binding wire.
  3.  Switching over-voltages
  4.  Lightning over-voltages
5.6.3  Voltage Distribution in Suspension
The insulation of any electrical system is designed to Insulator
withstand normal operating voltages, which however,
fluctuate with changing load or external surges. The A string of suspension insulators consists of series of
normal permissible range of fluctuation is around ±10%. porcelain discs connected via metallic links as shown in
The line-to-ground voltage also causes voltage stress on Fig. 5.19. The overhead line design is considered for the
the insulators and hence taken into consideration for the design of suspension insulators and the voltage insulator
calculation of insulation requirement level. Thus, the division of the disk.
insulation requirement of a 220 kV line should be calcu-
lated as follows, considering 10% safety factor
220 kV
1.1 × = 140 kV
3 V1 C V1

5.6.2  Types of Insulators


C1
There common types of over-head line insulators used C V2
are described as follows: V2

  1.  Pin type: These insulators are used for transmis-


sion and distribution of electric power at voltages C1
C V3
up to 33 kV. Pin type insulator is fixed to the
cross-arm on the pole and the conductor is housed V3
in a groove at the upper end of the insulator. The
conductor sits in the groove of this insulator and it C1

Figure 5.19 |   Design and equivalent circuit of


is clamped with the assistance of an annealed wire
of the same material as the conductor.
  2.  Suspension type: These insulators are used for ­suspension insulator.
voltages higher than > 33 kV. A suspension type
insulator consists of a number of porcelain discs The porcelain portion of each disc is in between two
connected in series by metal links in the form of a metal links which cause it to behave as a capacitor with
string. The conductor is suspended at the bottom capacitance C which can be termed as self-capacitance.
end of this string and the other end of the string is There also exists capacitance between the metal link
fixed to the cross-arm of the tower. on the disc and the tower or earth represented by C1.
  3.  Strain type: These insulators are used when Due to these effects, voltage distribution across each disc
there is a dead end of the line or there is corner or will be different as the charging current will not be the
sharp curve, under the influence of which the line same.
is subjected to excessive tension. Strain insulators This unequal distribution of voltage is undesirable and
help relieve this tension in high voltage transmis- is expressed in terms of string efficiency. The string effi-
sion lines. It consists of an assembly of suspension ciency is defined as the voltage ratio across the whole string
insulators in which the discs are used in the verti- of the insulators to the product of number of discs and the
cal plane. In case of long transmission lines, such voltage across the disc nearest to the running conductor.

Chapter 5.indd 385 3/23/2016 11:59:28 AM


386     Chapter 5:  POWER SYSTEMS 

String efficiency Thus string efficiency % is given by


V V
Voltage across the string × 100%
(3 × V3 ) × 100% ( )
= =
n × voltage across the disc which is nearest to the string 3 × V1 1 + 3K + K 2

where n = number of discs in the string considered. The methods to improve the string efficiency are:
  1.  Using longer cross arms for the transmis-
sion line: This will ensure the ratio of shunt
capacitance to self capacitance will be smaller
C resulting in better voltage distribution.
V1
C1 I1   2.  Grading the insulators: In this method, the
i1
insulators are so chosen as to provide different
A
capacitances. The top insulator is kept at mini-
C mum capacitance and gradually increased capaci-
V2 V tance to the bottom. This method also equalises
C1 i I2 the voltage distribution.
2
B   3.  Using guard rings: A guard ring is a metallic
ring electrically connected to the line conductor
C which introduces capacitance across the conductor
V3
and metal links. This cancels out the stray capaci-
C1 I3 tances improving the voltage distribution.
i3
C
5.6.4  Electric Field Distribution

Ceramic or porcelain insulators are widely used in power


transmission lines to provide mechanical support for high

Figure 5.20 |   Voltage distribution in suspension


voltage conductors and to withstand electrical stresses. The
exact nature and magnitude of these stresses varies with
insulator. the design of the insulator, its application and location.
The performance of the insulator is affected by the defec-
In the equivalent circuit given in Fig. 5.20, C is the self
tive insulator discs and can result disruption of normal
capacitance and C1 is the shunt capacitance. Therefore,
operation of the power system. The insulating materials
C = KC1 where K is a fraction. Applying KCL at node
are highly sensitive to the electric field and potential dis-
A, we have
I2 = I1 + i1 tribution, so accurate determination of electric field distri-
bution is necessary in designing of high voltage apparatus.
This can be expressed as,
Various techniques are usually used for the computa-
V2w C = V1w C + V1w C1 = V1w C + V1w KC tion of the electric potential and field along the insulator
string. These can be broadly classified into:
V2 = V1(1 + K )  (5.18)   1.  Domain based methods: These include finite dif-

ference method (FDM) and finite element method.
Similarly, applying KCL at node B, we have
  2.  Boundary based methods: These include charge
V3 = V1(1 + 3K + K 2 )  (5.19) simulation method (CSM) and surface charge sim-
ulation method (SCSM).
Voltage across conductor and earth is given by
The equations for electric potential and electric field for
V = V1 + V2 + V3 charge distribution symmetric around the axis are given
by:
Substituting for V2 and V3 from Eqs. (5.18) and (5.19),
1 = 11
4pe 0 L∫44pe
we have V (r, z)V
V=((rr,, zz)) = rs r ′F∫ *r
rss(rr ′′F
z ′)**dl((rr ′′zz ′′))dl
F dl
pe 00 LL
V = V1(1 + K ) (3 + K )
′ 2 +drdz′′ 2′ 2+ dz ′′ 22
2
dl = dldl
dr= = dr + dz
of rrz′′ ′and zz ′′that
So, the voltage across topmost unit,
Here
* is F
Here FHere ** is
a functio
F n of r ′n
is aa functio
functio of
nand , such
and ,, such
such that
that
V 4FK (=m)44KK((m m))
V1 = F* = F ** =
(1 + K ) (3 + K ) a+b a a+ + bb

Chapter 5.indd 386 3/23/2016 11:59:40 AM


5.7  DISTRIBUTION SYSTEMS     387

Here L is the length of the segment in r-z plane, dl is to be distributed. The current remains constant in
the elemental segment length, e0 is the permittivity of feeders and the most important parameter is their
free space, rs is the linear charge density, (r, z) are the current carrying capacity.
coordinates of the measuring points and (r′, z′) are the   2.  Distributors: These are lines or conductors to
coordinates of the source points. which various consumers are connected. These dis-
The corresponding electric field is given by tribute power among these consumers.
  3.  Service mains: It is line or a cable that connects

ò rs r¢H * (r¢z ¢)dl


1 the distributor to the consumer’s terminal.
FE(r, z) =
4pe 0 L

dl = dr ¢ + dz ¢
2 2 5.7.1  Classification of Distribution Systems
Here H * is a functioon of r ¢ and z ¢, such that The distribution systems may be classified based on the
2 é r ¢ + r + (z − z ¢)
2 2 2 ù following criteria:
H* = êK(m) − E(m)ú a r
r a + b êë a-b úû   1.  Type of construction: Overhead and under-
ground distribution systems. Generally overhead
é 4(z − z ¢) ù
+ê E(m)ú a z systems are cheaper and employed in preference.
ë (a − b) a + b û However, underground cables are used where it is
impractical or prohibition to use overhead lines.
Generally the electric field distribution of a non-ceramic   2.  Nature of current: AC and DC distribution
long insulator is more non-linear than that of a porcelain system. The AC distribution system is universally
insulator because of the absence of intermediate metal used because it is simpler and more economical.
parts in the former. For three phase power line, the elec-   3.  Scheme of connection: Radial, ring main and
tric field strength in the vicinity of the non-ceramic insu- inter-connected systems.
lators is influenced by the conductors, the hardware, the
tower configuration and the presence of the other two
phases of the three phase system. The end fitting shape 5.7.2  AC and DC Distribution Systems
of non-ceramic insulators is carefully designed to control
The AC distribution system is for distribution for AC
the electric field strength in its vicinity and a grading
power, that is, AC voltage and current. It is further
ring is added if required.
divided into the following two systems.
The electric field strength on non-ceramic insulators
  1.  Primary distribution system: For this system
and associate transmission system needs to be controlled
the general voltage level is 11kV, 6.6 kV and
to prevent:
3.3 kV, that is, higher than the voltage level used
  1.  Discharge activity on the surface material under by the general consumers. For economic consider-
wet and dry conditions so that the pollution perfor- ations, this system is provided with three phase,
mance of the insulators is not impacted. three wire system.
  2.  Discharge activity inside the insulator (e.g., fibre-   2.  Secondary distribution system: In this case
glass rod) and the sheath rubber material to the distribution level is 230 V single phase or
circumvent mechanical or electrical failure of non- 415 V three phase four wire systems. This voltage
ceramic insulators. level is directly usable by the general consumers.
  3.  Corona activity from the non-ceramic insulator or
metal hardware, which can result in radio interfer- The DC distribution system is meant for distribution of
ence and acoustic emissions. DC voltages through the following two systems.
  1.  Two-wire DC distribution system: In this
case the power is transmitted by two wires one pos-
5.7  DISTRIBUTION SYSTEMS itive (+) and then other negative (−). The loads
are connected between the two wires as shown in
the Fig. 5.21(a).
The part of electrical system that distributes electrical   2.  Three-wire DC distribution system: This
power for local usage is known as distribution system. system consists of two outer wires and one middle
The distribution system is constituted by following three wire which is earthed in the substation. The volt-
components: age between the outer wires is twice the voltage
  1.  Feeders: These are lines or conductors that con- between one line and earth. The configuration is
nect major station to the area where the power is shown in Fig. 5.21(b).

Chapter 5.indd 387 3/23/2016 11:59:41 AM


388     Chapter 5:  POWER SYSTEMS 

+ The three connection schemes, for AC distribution


system are shown in Figs. 5.22(a), (b) and (c).
Distributor
V L L M

Feeder Load


(a)
+ Sub -
station Feeder
V L
Neutral
M 2V
wire
V Feeder
− (a)
(b) Distributor
Figure 5.21 |   DC distribution systems. (a) Two wire
system. (b) Three wire system. Loads

5.7.3  Connection Schemes

The different connection schemes in distribution system


are discussed as follows: Sub - Primary
Distributor
  1.  Radial system: In this system separate feeders station ring
radiate from a single substation and feed the dis- Loads
tributor at one end only. This system is used when
the power is generated at low voltage and the sub-
station is located at the centre of the load. The
drawbacks of the system are: Loads
(i) Consumers are dependent only on single feeder
and distributor, so in event of a fault, the Distributor
power supply will be cut-off completely. (b)
(ii) The distributor end nearest to the feeding
point is heavily loaded. Distributor
(iii) The consumers at the distant end of the
distributor (away from the feeder) will be Loads
impacted by voltage fluctuations as the load
on the distributor changes. Sub - Sub-
  2.  Ring system: In this system, two feeders are used station 2 station 1
to supply power to single substation at both ends.
Thus, a loop circuit starts from the substation bus Distributor Distributor
bars, covers the area of supply by forming a loop
and returns to the substation. The disadvantages Loads Loads
of the radial system, that is voltage fluctuation and
low reliability, are thus overcome. Since the dis-
tributor is fed by two feeders, in event of a fault at Loads
one end, the power continues to be supplied.
  3.  Interconnected system: In this system, the Distributor
feeder ring receives power from two or more gener-
ating substations. This increases the reliability of (c)
the supply and increases efficiency of the system. Figure 5.22 |   Connection schemes for AC
An area, during the peak load hours, can be fed distribution systems. (a) Radial,
with feeder from alternate generating substation. (b) Ring main and (c) Interconnected.

Chapter 5.indd 388 3/23/2016 11:59:47 AM


5.9  BUS ADMITTANCE MATRIX     389

5.8  PU (PER UNIT SYSTEM) in the system connecting the buses `i’ and `j’. Since in a
large power system, each bus is connected to less number
of buses through an incident element, the matrix, Ybus of
When a system of different interconnected generators, such systems would be highly sparse.
transformers with different KVA, voltage ratings and
impedances are considered , it is difficult to calculate
5.9.1  Gauss-Seidel Method of Load Flow Analysis
different electrical parameters like fault current, voltage,
etc. when individual ratings are taken during computa- For a power system with n number of buses, let np be the
tion. The calculation becomes easier when a single ref- number of P—Q (load) buses and the ng be the number
erence MVA, voltage or current are considered. These of P—V (generator) buses such that, n = np + ng + 1.
are denoted as base values of those quantities. Then the Both voltage and phase angles of the load buses are
system is known as per unit or pu system with a unknown and reactive power and load angles of the gen-
single base value defined for individual quantities like erator buses are unknown. The real and reactive powers
KVA, voltage, currents, etc. Thus, per unit value of any of load bus and real power and voltage for the generator
quantity is given by bus are known. Also, the voltage and phase angle of the
Quantity slack bus are known. Thus sufficient numbers of known
quantities are there to attain a solution of the load flow
Base value of that quantity
problem for the power system.
For a three- phase system base KVA or MVA or VA is The complex power at ith bus in a power system can
to be defined and similarly base voltage or KV is also to be given as,
be defined
n
Let the base KVA = KVAB and base KV = KVB and Pi − jQi = Vi * I i = Vi * å Yik Vk
base voltage = VB k =1
n
= Vi (cos d i − j sin d i ) å Y ik Vk (cos q ik + j sin q ik )
Therefore base current is given by
KVAB MVAB × 103 k =1
IB = = (cos d k + j sin d k )
3KVB 3KVB
n
Thus, base impedance is given by = å Y ik ViVk (cos d i − j sin d i )(cos q ik + j sin q ik )
k =1
(cos d k + j sin d k )
VB / 3 KVB / 3
ZB = = × 103
IB IB
Knowing the real and reactive power injected at any bus,
KVB × 103 / 3 (KVB )2
= = × 103 the above equation can be expanded as,
(KVAB / 3KVB ) KVAB n
Pi,inj − jQi,inj = Vi * ∑ Yik Vk
(KVB / 3 ) × 103 × 3KVB (KVB )2

k =1
= =
MVAB × 103 MVAB = Vi * [Y i1V1 + Y i2V2 +… + Y iiV i +… + Y inVn ]
Thus, Rewriting the above equation, we get
Z (Ω) MVAB
Zpu = = Z(Ω) × 1  Pi, inj − jQi, inj 
ZB (KVB )2 Vi =  − Y i1V1 − Y i2V2 … − Y in Vn 
Y ii  Vi * 
Z(Ω) × KVAB
= × 10−3 The voltages of all the buses can be updated using the
(KVB )2
above procedure.

5.9  BUS ADMITTANCE MATRIX For updating the generator bus voltage, the reactive
power has to be known. Thus for updating the voltage of
this bus, the reactive power is calculated as,
The bus admittance matrix (Ybus) plays a crucial role in
 n 
load flow analysis. This matrix is a complex, square and Qi,inj = − Im Vi * ∑ Y ik Vk 
symmetric matrix and therefore only n(n + 1)/2 elements  k =1 
of Ybus are required to be stored for an n-bus system. In = − Im[V i * (Y i1V1 + Y i2V2 + … Y iiVi + …Y in Vn )]
the Ybus matrix, Yij = 0, if an incident element is absent

Chapter 5.indd 389 3/23/2016 11:59:59 AM


390     Chapter 5:  POWER SYSTEMS 

For the kth iteration, Q(k) can be given as, where, the Jacobian matrix can be subdivided into sub-

{
matrices as,
Qi, inj(k) = − Im Vi *(k −1) [Y i1V1 + Y i2V2(k) + … Y iiV i(k −1) J J12 
J =  11 
+ … Y in V n(k −1) ]}  21 22 
J J

where,
The error between the actual and injected real and reac-
 ∂P2 ∂P2 
 ∂d 
∂d n 
tive powers is calculated as,
 2
DPi = Pi, inj − Pi, calc = PGi − PLi − Pi, calc and, J11 =    
 ∂Pn ∂Pn 
DQi = Qi, inj − Qi, calc = QGi − QLi − Qi, calc   
 ∂d 2 ∂d n 
When these values are below a small error limit the iter-
ation process is concluded.  ∂P2 ∂P2 
 V2  V1+ n 
Since the Gauss-Seidel method is sometimes very slow  ∂ V2 0
∂ V1+ n 
= 
0

  
to converge, a constant value is multiplied after itera-
 
J12
tions. The updated voltage of ith bus can then be given
 ∂Pn ∂Pn 
as,
 V2  V1+ n 
∂ V2 ∂ V1+ n
Vi,acc(k) = (i − l )Vi, acc(k −1) + lVi(k) = Vi, acc(k −1)
0

 0 

+ l V i(k) − Vi, acc(k−1) 


 ∂Q2 ∂Q2 
  
where, l is a constant and is called the acceleration  ∂d 2 ∂d n 
factor. J 21 =     and,
 ∂Q ∂Q1+ n 
Advantage of Gauss Seidel method is that the method  1+ n0  0 
 ∂d ∂d n 
 
is simple with small computer memory requirement.
2
Also it takes lesser computational time in calculation.
However, it is a slow method and the number of itera-
tions increases as the number of system buses increase.  ∂Q2 ∂Q2 
 V2  V1+ n 
The convergence speed is governed by the selection of
 ∂ V2 0
∂ V1+ n 
the slack bus.  0

J 22 =    
 ∂Q1+ n ∂Q1+ n 
5.9.2  Newton—Raphson Method for Load Flow  V2 0
 V1+ n 0

 ∂ V2 0
∂ V1+ n 
Analysis  0 
In an n-bus power system, let the number of P—Q (load)
buses be np and the number of P—V (generator) buses be The steps in Newton-Raphson algorithm are as follows:
ng. Thus, n = np + ng + 1. Bus 1 is assumed to be the Step 1: Choose initial values of the voltage magnitudes
slack bus. The approach to Newton—Raphson method |V |(0) of all the load buses considered and n − 1 angles d (0)
of load flow study is comparable to solving a system of
of voltages of all the buses except the slack bus.
non-linear equations using the Newton—Raphson itera-
tive method. After each iteration, a Jacobian matrix is Step 2: Employ the estimated |V |(0) and d (0) to cal-
formed and solved for the corrections. For the load flow culate n − 1 number of injected real power Pcalc(0) and
problem, this equation can be, DP (0) where,

 ∆d 2  ∆Pi = Pi, inj − Pi, calc = PGi − PLi − Pi, calc


  
   ∆P2  Step 3: Employ the estimated |V |(0) and d (0) to calcu-
 ∆d n    
    late np number of injected reactive power Qcalc(0) and
 =  ∆Pn 
V2
J ∆ V DQ(0) where,
 2   ∆Q2 
      ∆Qi = Qi, inj − Qi, calc = QGi − QLi − Qi, calc
   
 V1+ n0   ∆Q1+n0 
∆  Step 4: To formulate the Jacobian matrix J (0), use the
 V1+ n0  estimated values of |V |(0) and d (0).

Chapter 5.indd 390 3/23/2016 12:00:12 PM


5.10  VOLTAGE CONTROL     391

Step 5: Solve for d (0) and D |V |(0) ÷ |V |(0) using the the apparatus in the power system, the profile of the load
Jacobian matrix of the load flow problem. voltage can be improved by using shunt and series com-
pensators which inject reactive power into the system.
Step 6: Update,

d (1) = d (0) + ∆d (0) 5.10.1.1 Shunt Compensator


 ∆V
(0) 
1 + 
(1) (0)
V = V An ideal shunt compensator system can be used for a
 V
(0)
 line to keep load voltage supply constant in the presence
of fluctuating loads. Considering the condition of active
Step 7: Terminate the process if DP and DQ are less
power flow from sending to the receiving end, let the
than permissible error values. Otherwise go back to
sending and receiving voltages be given by Vs = V∠d
and Vr = V ∠ 0 respectively. The shunt compensator reg-
Step 1 for further iterations.
ulates the midpoint voltage (VM) of a transmission line
5.10  VOLTAGE CONTROL to VM = V∠(d /2) for any variation in the compensator
current. The schematic representation and voltage vs.
current characteristic of the ideal shunt compensator is
The basic requirement of a good quality power system shown in Fig. 5.23(a) and (b), respectively, where IQ is
is consistent supply voltage. Majority of the equipment the compensator current is at the midpoint of the line.
constituting the power system are designed to operate at
the rated voltage level or atmost the voltage variation Is V–d/2 Ir
within a limit say ±5% of rated voltage. The receiving
end voltage can vary due to the change in load or load jX/2 IQjX/2
power factor even though the sending voltage remains
constant. The voltage at any point in the transmission/ Vs = V– d + Vr = V–d
distribution network can be controlled locally by plac-
ing suitable voltage control devices. As the active and −
reactive power demand at the load-end of a transmission
line varies its voltage level is also bound to vary. This
(a)
voltage variation has to be controlled within the permis-
sible limits for safety of the loads connected to it. Thus VM
voltage control at the load terminals is extremely neces-
sary. Various methods are adopted for voltage control
ranging from basic tap changing transformer method to V
advanced reactive power injection techniques. The com-
monly used voltage control devices are:
  1.  Shunt capacitors which are line reactance
compensators.
  2.  Series capacitors and reactors which act as source IQ
Leading Lagging
and sink of reactive power, respectively.
  3.  Synchronous condensers which act as source and (b)
sink for reactive power. Figure 5.23 |   Ideal shunt compensator.
  4.  Regulating transformers like tap changing (a) Schematic diagram. (b) Voltage
transformer. vs. current characteristics.
  5.  Static VAR compensators.
In practical systems, the ideal behaviour of the compen-
  6.  Power electronics-based flexible AC transmis-
sator is not feasible and a slight droop is observed in
sion system controllers, which is a relatively new
the practical voltage characteristic for the compensator.
technology.
However, assuming that the shunt compensator strictly
regulates the midpoint voltage to VM = V∠(d /2), the
5.10.1  Voltage Control by Reactive Power expressions for the sending and receiving end currents
Injection for the transmission line from Fig 5.17(a) can be written
as,
We have seen that the receiving end voltage of a trans- VÐd − VÐ(d /2)
mission line does not remain constant because of reactive Is =  (5.14)
jX /2
power exchange. To maintain constant voltage supply to

Chapter 5.indd 391 3/23/2016 12:00:22 PM


392     Chapter 5:  POWER SYSTEMS 

and 5.10.1.2 Series Compensator


VÐ(d /2) − V
Ir =  (5.15) Consider that the series compensator is represented by
jX /2
an ideal voltage source as shown in Fig. 5.24 and sup-
In Fig 5.17 (a), applying KCL at the compensation mid- plies reactive power to the system. Unlike the shunt
point, we get compensator, the location of the series compensator is
I s + IQ = I r not crucial, and can be placed anywhere along the trans-
mission line.
Combining and simplifying the above three equations,
Is Ir
the compensator current is given by
VQ
jX
[1 − cos(d /2)] Ð(d /2) 
4V
IQ = j (5.16)
X
Vs = V– d

Vr = V–0°
From Eq. (5.16), it can be observed that the compen-
sator current is in quadrature with the midpoint volt-
age of VM = V∠(d /2), and has a magnitude of (4V/X )
[1 −cos(d/2)]. Thus the apparent power injected by the
shunt compensator to the AC bus at the midpoint of the Figure 5.24 |   Schematic representation of an ideal
line is given by series compensator.
4V 2
PQ + jQQ = VM IQ*
= −j [1 − cos(d /2)]  (5.17) In the series compensated system, the receiving end cur-
X
rent is equal to the sending end current, that is, Is = Ir.
Thus, an ideal shunt compensator injects only reactive The series voltage VQ is injected in such a way that the
power to the system. magnitude of the injected voltage is proportional and the
The apparent power supplied by the source is given by phase of the voltage is in quadrature to that of the line
current. Therefore, the injected voltage is given by,
 V∠ − d − V∠ − (d /2)  VQ = lI s ∓ j 90°
Ps + jQs = Vs I s* = V∠d  
 −jX /2 
The ratio l/X is called the compensation level and is
V 2 − V∠(d /2)
= often expressed in percentage. Thus, when the compen-
−jX /2 sation level is 50%, l = X/2. Applying (Kirchhoff’s volt-
2V 2 sin(d /2) 2V 2 [1 − cos(d /2)] age law (KVL) in the e series compensation circuit of
= +j Fig. 5.24, we get
X X
Vs − VQ − Vr = jXI s Þ Vs − Vr = ∓ jlI s + jXI s
Similarly the apparent power delivered at the receiving
Assuming  Vs = VÐd and Vr = VÐ0°, the line current
end is,
can be given by
Pr + jQr = Vr I r*
V∠d − V
 V∠ − (d /2) − V  Is =  (5.18)
=V   j(X ∓ l )
 −jX /2 

2V 2 sin(d /2) 2V 2 [cos(d /2) − 1] When VQ = lIse−j90°, Eq. (5.18) becomes


= +j
V∠d − V
X X
Is =
Hence the real power transmitted over the line is given by j(X − l )

2V 2 Thus, l  is subtracted from  X, which implies that the


PE = Ps + Pr = sin(d /2) voltage source is acting as a pure capacitor. Hence, this
X
mode is known as the capacitive mode of operation.
 and the reactive power consumed by the line is given by When VQ = lIse+j90°, Eq. (5.18) becomes
8V 2 V∠d − V
QE = Qs + QQ − Qr = [1 − cos(d /2)] Is =
X j(X + l )

Chapter 5.indd 392 3/23/2016 12:00:40 PM


5.10  VOLTAGE CONTROL     393

Here l  is added to  X, converting this to an  inductive Then


mode of operation. Since this voltage injection results in I
addition or subtraction of l from the line reactance; this Vs − Vr ≈ (RT cos f + XT sin f )
N r2
mode is known as the constant reactance mode of volt-
age induction operation. Vr (IRT cos f + XT sin f )
=
N r2 V r
5.10.2  Voltage Control using Tap Changing RT Pr + XT Qr
Transformers =
N r2 V r
Usually the tap changing transformers can be used at
both the sending and the receiving ends to regulate the Now equating this magnitude with
voltage at the load end. The basic configuration is shown Ns
in Fig. 5.25. Vs − Vr ≈ Vs1 ⋅ − Vr
Nr
Transmission line we get,
ZT
Vs1N s R P +X Q
Vs1 Vs Vr Vr1 − Vr = T r 2 T r ∵ N s N r = 1
Nr N r Vr
1 : Ns Nr : 1
or, Vs1Vr − Vr2 N r2 = RT PR + XT Qr 
Figure 5.25 |   Configuration of voltage control with
(5.19)
tap changing transformers.
Equation (5.19) is a quadratic equation of Vr and it can
Considering nominal taps, the sending end voltage is be obtained by solving the equation with known value of
Vs = N s ⋅ Vs1 other parameters.

whereas the receiving end voltage is


5.10.3 Voltage Control by Combination of
Vr
Vr1 = Tap Changing Transformer and Reactive Power
Nr Injection
Both the sending end and the receiving end transformers
have tap-changing facilities. At nominal taps Ns⋅Nr = 1. The configuration of combination of tap changing trans-
All the quantities can be referred to the load side with former and reactive power injection is shown in Fig. 5.27.
the equivalent circuit as given in Fig. 5.26. The compensator connected in shunt mode injects reac-
tive VAR to the system to maintain proper voltage pro-
Is = Ir file at the receiving end. The transformer is represented
by equivalent Y-connection and any impedance from Vs
Ns ZT/Nr2 or Vr to the transformer can be lumped together with
Vs = Vs1. Vr = Vr1
Nr the transformer branch impedances.

Figure 5.26 |   Equivalent circuit referred to load P S


end.
Vs Vr
Now, the voltage drop can be given by,
Ns
−Vr + Vs = −Vr + Vs1 . T
Nr
Vc
and ZT = RT + jXT (considering no shunt parameters)
If the current has a phase angle of −f and the line Compensator
impedance is denoted by ZT = RT + jXT
Figure 5.27 |   Combined tap-changing transformer
(RT + jXT )
Vs − Vr = I∠ − f and reactive power injection.
N r2
However, this configuration is not used frequently due
I
= 2
(cos f − j sin f )(RT + jXT ) to the advent of modern techniques of reactive power
Nr injection techniques.

Chapter 5.indd 393 3/23/2016 12:00:53 PM


394     Chapter 5:  POWER SYSTEMS 

5.11  FREQUENCY CONTROL Similarly, the receiving end apparent power is given by
 V∠ − d − V 
Pr + jQr = Vr I s* = V  
The load frequency control, as the name signifies, is the  −j(X ∓ l ) 
method to regulate the power flow between different V 2 sin d V 2 (cos d − 1)
= +j
areas through transmission network while keeping the X∓l X∓l
frequency constant. The system frequency rises when the
load decreases if governor opening, that is, the active  Hence the real power transmitted over the line is given by
V2
power input is unchanged. Similarly the frequency can PE = Ps + Pr = sin d
drop if the load increases. It is desirable to maintain X∓l
the frequency constant. Thus the load frequency control The power-angle characteristics of an uncompensated
(LFC) has the following objectives: and series compensated power system are shown in
•  The frequency should be held constant against any Fig. 5.28. The curve  P0  is the power-angle curve 
load variations. where the line is not compensated. The curve P1, with
the maximum value of power greater than the base
•  Each area must maintain the tie-line power flow to
power curve P0, corresponds to the capacitive mode of
operation, where l is negative. On the other hand, the
its pre-specified value.
The first step in the LFC is to develop the equation for curve P2 corresponds to the inductive mode of operation
the area control error (ACE) which is defined as where l is positive. It will have maximum value of power
ACE = (Ptie − Psch ) + Bf ∆f = ∆Ptie + Bf ∆f lower than that of curve P0

where Ptie and Psch are tie-line power and scheduled


power through tie-line respectively and the constant Bf P1
is called the frequency bias constant and Df is the fre-
quency deviation.
Any change in the reference of the power setting DPref,i, 1 P0
for any area-i can be obtained as Real power
∆Pref ,i = Ki ∫ ACEdt
(pu)

where Ki is the integral gain. The ACE is negative when P2


the net power flow out of an area is low or when the fre-
quency has dropped or both the situations has occurred.
In this case the generation must be increased by increas- 0
ing DPref,i. This negative sign actually accounts for this 0 d (rad) p
inverse relation between DPref,i and ACE. The tie-line
power flow and frequency of each area are monitored Figure 5.28 |   Power-angle characteristics in
in the individual control center for the particular area. capacitive and inductive mode.
When the ACE is computed and DPref,i is obtained from
the above equation, the commands are given to vari- Let us consider the capacitive mode of operation which
ous turbine-generator controls to adjust their reference represents the normal mode of operation in which the
power settings. power transfer over the line is enhanced. The reactive
power supplied by the compensator is given by
5.12  POWER SYSTEM STABILITY V∠d − V V∠ − d − V
QQ = VQ I s* = −jl ×  (5.20)
CONCEPTS j(X − l ) −j(X − l )

Simplifying Eq. (5.20), we get
5.12.1 Power-Angle Characteristics 2lV 2
QQ = −j (1 − cos d ) (5.21)
(X − l )2
The sending end apparent power is  given by, VsIs*,
therefore,
 V∠ − d − V  5.12.2 Swing Equation
Ps + jQs = Vs I s* = V∠d  
 −j(X ∓ l )  There are two torque acting on a three-phase alterna-
V 2 − V 2 ∠d V 2 sin d V 2 (1 − cos d ) tor driven by a prime mover (shaft), that is, mechanical
= = +j
−j(X ∓ l ) X∓l X∓l torque (Tm) and the electromagnetic torque (Te) of the

Chapter 5.indd 394 3/23/2016 12:01:02 PM


5.12  POWER SYSTEM STABILITY CONCEPTS     395

alternator. Neglecting losses, the accelerating torque (Ta) where


can be obtained as the difference these two torque as Tm wrI = M = angular momentum or inertia constant
tends to accelerate the generator in the positive direction
of rotation and Te in the negative direction. Thus wrTm = Pm = mechanical power input at the shaft
(neglecting rotational losses)
Ta = Tm − Te wrTe = Pe = electrical power output (neglecting losses)
The equation of motion for the machine rotor, in a Here Pm, Pe and Pa are all expressed in MW.
three-phase alternator driven by a prime mover, can be
expressed as, The normalised inertia constant can be defined as,

d 2q Stored kinetic energy at synchronous speed


I = Tm − Te = Ta  (5.21) H=
dt2 Generator MVA rating

where I is the total moment of inertia of the rotor mass Jw s2
= MJ/MVA (5.26)
in kg-m2, q is the rotor angle (angular position of the 2Srated

rotor) in radians and Tm, Te and Ta are all expressed
Substituting and re-arranging Eqs. (5.25) and (5.26), we get
in Nm.
Srated d 2d
In the steady state, the electrical torque is equal to 2H wr = Pm − Pe = Pa
the mechanical torque, and hence the accelerating torque w s2 dt2
d 2dd d
will be zero. Under the steady-state condition, the rotor 2
S Srated
moves at synchronous speed ws in rad/s. The rotor 2H2Hrated 2 = P= P−
mP −e P=e P=a Pa (5.27)
angle, q is measured with respect to a stationary refer- w s w sdt dt2 m
ence frame and converted into angle d (in rad) measured (as(wasr w=r w=swats at steady
steady state
state ) )
with respect to the synchronously rotating reference
frame. The relation is expressed as, Dividing Eq. (5.27) with Srated (generator MVA rating),
the per unit values of quantities Pm,  Pe and  Pa are
q = wst + d  (5.22) obtained as

Differentiating Eq. (5.22) with respect to time, we have
2H d 2d
= Pm − Pe = Pa per unit
dq dd
(5.28)
= ws + (5.23) w s dt2

dt dt 
Equation (5.28) describes the behaviour of the rotor
In terms of the angular speed of the rotor be,
dynamics and hence is known as the swing equation.
dq The angle d is the angle of the internal emf (electromo-
wr =
dt tive force) of the generator with terminal voltage and
it reflects the amount of power that can be transferred.
Eq. (5.23) can be modified as  This angle is therefore called the load angle.
dd
 wr − ws =
dt
5.12.3  Equal-Area Criterion
Thus, the rotor angular speed( is equal to the synchro-
nous speed only when dd/dt is equal to zero. Therefore, The real power transmitted over a lossless line is given by
the term  dd/dt can be defined as the error in speed.
Vs Vr
Combining Eqs. (5.21) and (5.23) in steady state with P = sind
synchronous speed constant, we get X

d 2d We know that in a synchronous machine operating in


I 2
= Tm − Te = Ta  (5.24) steady state, electric power output Pe = Pm. If a fault
dt
occurs in the system, the circuit breaker will open up in
Multiplying both side of Eq. (5.24) by  rotor speed wr, the faulted section to clear the fault and then restore
we have the system and the entire process will take a few cycles.
d 2d
w r I 2 = w r (Tm − Te )
However, in this time interval, the input power will con-
dt tinue to be supplied by a prime mover and can be con-
sidered to remain constant during the period electrical
d 2d
= Pm − Pe = Pa 
changes occur due to the fault. The transient stability
M (5.25)
dt2 of the power system is determined by the ability of the

Chapter 5.indd 395 3/23/2016 12:01:18 PM


396     Chapter 5:  POWER SYSTEMS 

system to recover from the fault and deliver power Pm Consider a generator is operating in steady state at load
with a new load angle d. angle  d0, dd/dt = 0. We have seen above that when a
Consider the power angle curve shown in Fig. 5.29, fault occurs, the machine will start accelerating and will
where the system is operating in the steady state and continue to do so after the fault is cleared, till it reaches
delivering a power Pm at an angle  d0  in steady state. its limiting value at dm, at which point again dd/dt = 0.
When the fault occurs in the system, the real power Thus, the area of acceleration (A1) is given by,
transferred is zero, so the output power line obtained dc
is a straight line coinciding with d-axis as shown in the A1 = ∫ (Pm − Pe )dd  (5.32)
P-d curve in Fig. 5.29. Since the steam input does not d0

change in this time interval, Pm remains constant and
Similarly, the area of deceleration (A2) can be defined as,
the accelerating power Pa is equal to Pm. The difference
between the two powers gives rise to the rate of change dm
of stored kinetic energy in the rotor masses in accor- A2 = ∫ (Pe − Pm )dd  (5.33)
dance with the Swing equation, as a result of which the dc
rotor accelerates and the load angle increases. If the fault For stable operation of the alternator, two areas given
is cleared and the circuit breaker re-closes at an angle dc, by Eqs. (5.32) and (5.33) have to be equal. Consider the
the power angle characteristics revert back to the normal following cases:
operating curve as shown in Fig. 5.29. At that d = dc,
Case (i): The area of acceleration A1 is larger than
the electrical power is more than the mechanical power
the area of deceleration A2, that is A1 > A2. The fault
and the machine begins to decelerate. However, the load
is cleared and the line reclosed at  any angle dc. The
angle continues to increase for some time due to the iner-
alternator load angle will cross the maximum point dm,
tia of the rotor masses and eventually stop at a point of
d = dm and the rotor starts to decelerate. beyond whichthe electrical power is again less than the
mechanical power which will result in further accelera-
Pe (per unit) tion. The generator will therefore start accelerating and
eventually will run out of step.
Case (ii): The area of acceleration A1 is smaller than
A2 the area of deceleration A2, that is A1 < A2. The fault
is cleared before dc, the available decelerating area will
always be larger than the accelerating area; the machine
Pm
would decelerate before becoming unstable.
A1 Case (iii): The areas area of acceleration and decelera-
tion are equal, that is, A1 = A2. This defines the bound-
d (rad)
ary of the stability limit. The clearing angle dc for
0 do dc dm p this mode is called the critical clearing angle and is
denoted by dcr. Thus, substituting dc = dcr in Eq. (5.34),
Figure 5.29 |   Power-angle curve demonstrating
equal area criterion. d cr dm

The relation for change in d can be written as, ∫ (Pm − Pe )dd = ∫ (Pe − Pm )dd  (5.34)
d0 d cr

d  dd   dd   d d 
2 2
   
dt   dt2 
= 2  (5.29) Thus, the critical clearing angle can be calculated from
dt  dt  the above equation. Since the critical clearing angle

depends on the equality of the two accelerating and
The Swing’s equation (5.28) can be modified after multi-
plying both sides by dd /dt and re-arranged as,
de-accelerating areas, so it is called the  equal area
criterion.
H d  dd  dd
2
The equal area criterion for two cases, that is, sudden
  = (Pm − Pe )  (5.30)
w s dt  dt  dt increase of load on synchronous motor and opening of
one parallel line, are discussed as follows.
Multiplying both sides of Eq. (5.30) by dt and integrat-
ing between angles of d0 to dc, the same can be expressed
Condition 1: Sudden increase of load on synchro-
nous motor.
as,
2 dc dc
H  dd  Consider the motor connected to infinite bus, as shown
  = ∫ (Pm − Pe )dd  (5.31)
w s  dt 
in Fig. 5.30(a) and the phasor diagram is shown in
d0 d0 Fig. 5.30(b)

Chapter 5.indd 396 3/23/2016 12:01:26 PM


5.12  POWER SYSTEM STABILITY CONCEPTS     397

E V (vi)  At point between c and b: The input is greater than


− + jx output and rotor under acceleration. w increases further
and d decreases.
I (vii)  At point b: The output is equal to input and w = wm
(a) and d = ds. d continues to decrease.
V (viii)  At any point between b and a: The output is
d greater than input and deceleration occurs resulting in
decrease in w and decrease in d.
E (ix)  At point a: Again, the output is greater than input
and the cycle repeats.
I
At final state, b will be the steady state operating point
(b)
with the condition of stability is given as,
Figure 5.30 |   Motor connected to infinite bus. d d d
(a) Circuit. (b) Phasor diagram.
∫ Ps − Pe dd = 0 ⇒ ∫ Pe dd = ∫ Ps dd
Here E = V − ( jX ) d0 d0 d0

From the power angle curve,


E V
Pe = sin d = Pmax sin d dm

∫ Pedd = area d 0 abcd m


X
The power angle curve for such a case is given as follows
d0
in Fig. 5.31. and
P
dm

∫ Ps dd = area d 0 abed m
Input power
c d0

d b A2 e Changed output From the curve we have that for stability, A1 = A2.
Ps
A1 Condition 2: Opening of one of the parallel lines.
Consider the parallel transmission lines shown in
a Initial output Fig. 5.32. When power is supplied through parallel
P0
transmission lines by the alternator, opening of the
lines will result in increase in equivalent line resis-
d tance. This will inevitably cause decrease in maximum
d0 ds dm power transferred. The generator may lose synchro-
Figure 5.31 |   Power angle curve for Condition 1.
nism with opening of one line when the other line can
supply power to the connected load.
When the load is increased suddenly the changes taking
place are listed as follows: G
(i)  At point a: Initially, input = output = P0
w = w s and d = d 0 Figure 5.32 |   Parallel transmission lines.
After sudden change in load, output = Ps, which is less Three phase fault occurring on one of the parallel lines is
than input. As a result w decreases and d increases from shown in Fig. 5.33(a) and the simplified circuit is shown
their initial values. in Fig. 5.33(b).
(ii)  At any point between a and b: The output is greater
than input and deceleration occurs resulting in decrease
in w and increase in d.
(iii)  At point b: The output is equal to input and w is
less than w s and also at its minimum value. d = d s. d +
continues to increase as w is less than w s. + Ei
(iv)  At point between b and c: The input is greater than Eg −
output, thus resulting in increase in energy and acceleration of −
the rotor. w increases but still is less than w s. d also increases.
(v)  At point c: The input is greater than output and (a)
rotor under acceleration. w = w s also, d = dm. w  increases Figure 5.33 |   Three phase fault on one parallel line.
and d decreases. (a) Fault circuit. (b) Simplified circuit.

Chapter 5.indd 397 3/23/2016 12:01:39 PM


398     Chapter 5:  POWER SYSTEMS 

From the power angle curves, we have:

+ + (i)  For stability dc < dcr and A1 = A2


 (ii) For critically stable system: dc = dcr and A1 = A2
(iii)  For unstable system: dc > dcr and A1 < A2
Eg Ei
− −
When this kind of fault occurs on any other point of the
Ig
line, there is finite impedance between the buses and fault.
The fault circuit as shown in Fig. 5.35(a) can be reduced to
(b)
Figure 5.33 |   (Continued).
a circuit of Fig. 5.35(b)

The current from the generator Ig flows through the fault


which lags the voltage Eg by 90° (electrical). Thus the real
power will be zero. Such a fault will cause load angle d to
increase indefinitely and whole input power will be used for +
purpose of acceleration which will result in instability. With
+ Ei
the opening of the fault circuit, with fault at one end of the
double circuit line, rotor angle reaches a particular value Eg −
then the system will be stable otherwise it will lose stability. −
The power angle curves for different characteristics
conditions are depicted in Fig. 5.34. (a)

Output Xb
power (pre-fault)
Output
power (post-fault) + +
A2 Eg Xa Xc Em
Ps
− −
Input power
A1

d0 dc dm
Output power (b)
(during fault)
(a) Figure 5.35 |   Three phase fault at any other point
in one parallel line. (a) Fault circuit.
(b) Reduced circuit.
Power can be transmitted during the fault and can be
given as,
A2
Eg Em
sind
A1 Xb

d0 dcr dm
The power angle curves for this condition are shown in
(b) Fig. 5.36.

Output power (pre-fault)

Output power (post-fault)


A2
A2
A1 Input power
A1
Output power
d0 dc (during-fault)
(c)
Figure 5.34 |   Power angle curves for Condition 2 d0 dc dm
(fault in one parallel line). (a) Stable.
(a)
(b) Critically stable. (c) Unstable.

Chapter 5.indd 398 3/23/2016 12:01:48 PM


5.13  POWER FACTOR CORRECTION     399

  1.  Capacitors: Since majority of loads in power


Output power (pre-fault) system are inductive, they demand reactive power
Output power (post-fault) from the system. The reactive power is compen-
sated by capacitor banks which can be installed
A2 parallel to the connected load. Figure 5.37(a)
Input power shows an arrangement for power factor improve-
A1
Output power ment using capacitor bank. Capacitor banks may
(during-fault) be connected in star or delta. Static power factor
correction can be achieved at each individual
d0 dcr dm motor load by connection of capacitors to the
motor starting arrangement. Figure 5.37(b) shows
(b) an arrangement for power factor correction for the
induction motor using capacitor bank.
Output power (pre-fault)

Output power (post-fault) Power


supply Load
A2
Input power
A1
Output power
(during-fault)

d0 dc C C
(c)
Figure 5.36 |   Power angle curves. (a) Stable.
(b) Critically stable. (c) Unstable.
For such a system, from the power angle curves, C
we have: Capacitor bank
(i)  For stability dc < dcr and A2 = A1 (a)
  (ii) For critically stable system: dc = dcr and A2 = A1
(iii)  For unstable system: dc > dcr and A2 < A1 Overload
Power
supply
5.13  POWER FACTOR CORRECTION
Induction
motor
Power factor correction is the term used to describe
different technologies to restore the system power Capacitor bank
factor close to unity. To achieve this, the compensa-
(b)
tion of reactive power demand of inductive load is to be
reduced. The benefits of power factor correction include Figure 5.37 |   Power factor correction (a) using
reduction: capacitor bank; (b) of induction motor
using star connected capacitor bank.
  1.  in power consumption due to increased energy
efficiency.
  2.  of current in distribution equipments and   2.  Synchronous condenser: It is three-phase syn-
transformers. chronous motor with no load attached to its shaft
  3.  of voltage drop. and operating in overexcited mode. The synchro-
  4.  of burden and hence, better equipment life. nous machine in this mode behaves as a capaci-
  5.  in overall electricity cost. tor thus drawing lagging current from the system
to which it is connected. In a power system, syn-
Usually three methods are adopted for power factor chronous condenser is connected towards load side.
correction in power system and these are discussed as Figure 5.38 shows schematic representation of syn-
follows. chronous condenser.

Chapter 5.indd 399 3/23/2016 12:01:51 PM


400     Chapter 5:  POWER SYSTEMS 

P bus Q bus P load Q load sequence on the other hand is a balanced three-phase
system with phase sequence set opposite to the original
Load phase sequence. Zero sequence is equal in magnitude and
phase components to that of the original system with no
rotational sequence.
Figure 5.40 shows a set of three unbalanced phasors,
P mot Q mot displaced in phase from each other by 120°, that are
resolved into the three sequence components. The origi-
nal phasors are denoted by Va, Vb and Vc. The positive,
SC
negative and zero sequence components are indicated by
the subscripts 1, 2 and 0, respectively. Similarly, unbal-
Synchronous anced currents can also be represented by symmetrical
condenser components.
Figure 5.38 |   Synchronous condenser. Vc Vc1 Vb2
  3.  Phase advancer: It is an AC exciter primarily
used to improve power factor of induction motor Va Va1 Va2
load. The induction motor draws exciting current Va0 Vb0 Vc0
which lags behind the supply voltage. The phase
advancer supplies this exciting current to produce Vb Vb1 Vc2
required flux at slip frequency which improves the (a) (b) (c) (d)
power factor of the system to which the motor is
connected. These can be mounted on shaft of the Figure 5.40 |   A three-phase system. (a) Unbalanced.
induction motor and connected to the rotor circuit. (b) Positive sequence. (c) Negative
sequence. (d) Zero sequence phasors.
5.13.1 Power Factor Correction Formula
5.14.1  Symmetrical Components Transformation
QL
cted The a-operator which is important in symmetrical com-
corre QC ponents transformation can be given as,
VA cted
f1 VA c
orre
a = e j120° = − + j
1 3
f2 2 2
Real power (P) The following relations can be deduced from the
Figure 5.39 |   Power triangle.
a-operator,

a2 = e j240° = − − j
From Fig. 5.39, the uncorrected and corrected values of 1 3
a*
reactive power are given as, 2 2
Quncorr = P ∗ tan f1 a3 = e j360° = 1
Qcorr = P ∗ tan f2 a4 = e j 480° = e j360°e j120° = a
where P is the real power. The required capacitance a5 = e j600° = e j360°e j240° = a2
value, Qc is,
Also,
Qc = Quncorr − Qcorr = P ∗ (tan f1 − tan f2 ) 1 3 1 3
1 + a + a2 = 1 − + j − −j =0
2 2 2 2
5.14  SYMMETRICAL COMPONENTS Using the a-operator from Fig. 5.40(b), we have
Vb1 = a2Va1 and Vc1 = aVa1
A system of three unbalanced phasor quantities can be
Similarly, from Fig. 5.40(c), we can write,
resolved into three symmetrical components viz. posi-
tive, negative and zero sequence components. Positive Vb2 = aVa 2 and Vc2 = a2 Va 2
sequence is a balanced three-phase system with the same
phase sequence as the original phase sequence. Negative Also from Fig. 5.40(d),

Chapter 5.indd 400 3/23/2016 12:02:02 PM


5.15  FAULTS     401

Va 0 = V b 0 = V c 0   1.  Before the fault occurs, the system is balanced in


a manner that among the three sequence networks
V a = V a 0 + V a1 + V a 2 only the positive sequence network is active and
Vb = Va 0 + a2Va1 + aVa 2 = Vb 0 + Vb1 + Vb2 when the fault occurs, these are connected only
through the fault location.
Vc = Va 0 + aVa1 + a2Va 2 = Vc 0 + Vc1 + Vc2   2.  The fault current is negligible, so the pre-fault pos-
itive sequence voltages are same at all nodes and at
Therefore, representing the above equations in matrix
the fault location.
form, we get
  3.  All the network resistances and line charging
é Va ù é1 1 1 ù éVa 0 ù
êV ú = 1 ê1 a a2 ú ê V ú
capacitances are negligible.
ê bú 3 ê ú ê a1 ú   4.  All loads are passive except the rotating loads
êë Vc úû êë a2 a úû êëVa 2 úû which are represented by synchronous machines.

The symmetrical component transformation matrix can The three sequence network in which the unsymmetrical
be derived as, faults may occur is shown in Fig. 5.41.
é1 1 1 ù éV ù Vf
éV a 0 ù
ê V ú = 1 ê1 a a2 úú êVb ú
a a
ê a1 ú 3 ê ê ú If(a)
êëVa 2 úû êë a2 a úû êë Vc úû
Thus, Vf
Va 012 = CVabc b

where, C is the symmetrical component transformation If(b)


matrix and Va012 is the symmetrical component matrix.
The unbalanced system matrix is represented by Vabc. Vf
c
Similarly, an unbalanced set of current phasors If(c)
Iabc and their symmetrical components as Ia012 can be
defined as,
Ia 012 = CIabc Figure 5.41 |   Three sequence network with
unsymmetrical faults.

5.15  FAULTS
5.15.1.1 Single-Line-to-Ground Fault

The faults can be broadly classified as short circuit and To understand single line-to-ground (LG) fault condi-
open circuit faults. Both the short and open circuit faults tion, consider three phase lines of a-b-c as shown in
can be classified as symmetrical and unsymmetrical Fig. 5.42, where a LG fault has occurred at node k of the
faults. In symmetrical faults, the three lines are affected network. It is assumed that the phase a has touched the
equally and in asymmetrical faults, the line currents ground through an impedance of Zf and the faulted seg-
will be unequal for the three phases. The short circuit ment shown in Fig. 5.42.
unsymmetrical (or asymmetrical) and symmetrical faults k
are discussed followed by the open-conductor faults. a
If(a) Zf

5.15.1  Unsymmetrical Fault Analysis


k
The unsymmetrical short circuit faults can be classified b
as shunt type and series type faults. The shunt type
faults include the following:
  1.  Single line-to-ground (LG) fault. k
c
  2.  Line-to-line (LL) fault.
  3.  Double-line-to-ground (LLG) fault. Figure 5.42 |   Representation of 1 LG fault.

We will consider these fault conditions and determine The system is unloaded before the occurrence of the
the voltage (Vf) at the faulted point and the fault cur- fault, therefore
rent (If) in the three phases, based on the following I f (b ) = I f (c ) = 0  (5.35)
assumptions:

Chapter 5.indd 401 3/23/2016 12:02:11 PM


402     Chapter 5:  POWER SYSTEMS 

The phase a voltage at the fault point k is given by 5.15.1.2 Line-to-Line Fault


Vk(a ) = Zf I f (a ) To understand line-to-line (LL) fault condition, consider
The sequence currents phase with the fault can be writ- three phase lines of a-b-c as shown in Fig. 5.44 where a
ten from Eq. (5.35) as, LL fault has occurred at node k between lines b and c of
the network. It is assumed that the system is unloaded
1 1 1   If (a ) 
1
a2   0 
before the occurrence of the fault, therefore
= 1 a I f (a ) = 0 
 
If (a 012) (5.39)
1 a2 a   0 
3
k
On simplification and solving, we get a
If (a )
If (a 0) = If (a1) = If (a 2) =
 (5.36)
3
k
From Eq. (5.36), we deduce that the three sequence cur- b
If(b) Zf
rents are in series for the LG fault. Let the positive and
negative sequence Thevenin impedances at the faulted
If(c)
point be  Zkk0,  Zkk1  and  Zkk2, respectively and the cor-
responding Thevenin voltage at the faulted phase is Vf. c
Thus the three sequence circuits can be drawn as shown k
in Fig. 5.43. Figure 5.44 |   Representation of LL fault.
+ Zkk1 The phases b and c are shorted through an impedance
Vf If(a1) Zf, therefore, the current for the shorted phases b and c
Vk(a1)
− can be written as,

If(a0) = If(a1) = If(a2)


I f (b) = −If (c)  (5.40)
Zkk2
If(a2) Therefore, from Eqs. (5.39) and (5.40), we have
Vk(a2)
 0   
 1 
0

If (a 012) = C  If (0)  = (a − a2 )If (0) 
 −I  3  2 
Zkk0  f (0)  (a − a)If (0) 
If(a0)
Vk(a0) The fault currents can be summarised as,
3Zf Ifa(0) = 0
 (5.41)
If (a1) = −If (a 2)
Figure 5.43 |   Thevenin equivalent of a single LG fault.
Therefore, no zero sequence current is injected into the
Thus from Fig. 5.43, we have. network for the LL fault at k. The positive and negative
Vk(a 0) = −Zkk 0 I f (a 0) sequence currents are equal having opposite sign.

Vk(a1) = V f − Zkk1I f (a1)


From Fig. 5.44, the voltage at the faulted point k is
given by,
Vk(a 2) = −Zkk2 I f (a 2)  (5.37)
Vk(b) − Vk(c) = Zf I f (0)  (5.42)
Then from the Eqs. (5.36) and (5.37), we can write
The above expression can be further simplified as,
Vk(a ) = Vk(a 0) + Vk(a1) + Vk(a 2)
Vk(b) − Vk(c) = Vk(b 0) + Vk(b1) + Vk(b2) − Vk(c 0) − Vk(c1) − Vk(c2)
= Vf − (Zkk 0 + Zkk1 + Zkk2 )I f (a 0)  (5.38)
= (Vk(b1) − Vk(c1) ) + (Vk(b2) − Vk(c2) )
Since
= (a2 − a)Vk(a1) + (a − a2 )Vk(a 2)
Vk(a ) = Zf I f (a ) = Zf (I f (a 0) + I f (a1) + I f (a 2) ) = 3Zf I f (a 0)

= (a2 − a)(Vk(a1) − Vk(a 2) ) (5.43)
From Eq. (5.38), we get

I f (a 0) =
Vf Since If(a0) = If(b0) = 0 and If(a1) = −If(b2), the expression
Zkk 0 + Zkk1 + Zkk2 + 3Zf for current can be written as,

Chapter 5.indd 402 3/23/2016 12:02:28 PM


5.15  FAULTS     403

Also voltages of phases b and c are given by


I f (0) = I f(01) + I f(02) = a2 I f(a1) + aI f(a 2)
Vk(b) = Vk(c) = Zf (I b − I c ) = 3Zf I f(a 0) 
= (a2 − a)If(a1)
(5.46)
 (5.44)
Therefore,
Therefore, combining the three voltage and current
expressions (Eqs. 5.42 to 5.43), we get Vk(a )   Vk(a ) + 2Vk(b) 
  1 
Vk(a1) − Vk(a 2) = Zf I f(a1) Vk(a 012) = C Vk(b)  = Vk(a ) + (a + a 2 )Vk(b)  (5.47)
V  3 
 k(b)  Vk(a ) + (a + a )Vk(b) 
2
Equations (5.42) and (5.44) indicate that the positive
and negative sequence networks are in parallel position
and the sequence network is as shown in Fig. 5.45. From We thus get the following two equations from Eq. (5.47),
this network we get,
Vf
I f(a1) = −I f(a 2) =
Vk(a1) = Vk(a 2)  (5.48)
Zkk1 + Zkk2 + Zf
3Vk(a 0) = Vk(a ) + 2Vk(b)
If(a1) If(a2)
+ Zkk1 Zf Zkk2 = Vk(a 0) + Vk(a1) + Vk(a 2) + 2Vk(b)  (5.49)

Vf Vk(a1) Vk(a2) Substituting Eqs. (5.46) and (5.48) in Eq. (5.49) and
− rearranging, we get
Figure 5.45 |   Thevenin equivalent of an LL fault.
Vk(a1) = Vk(a 2) = Vk(a 0) − 3Zf I f (a 0)  (5.50)

5.15.1.3 Double-Line to Ground Fault Also since If(a) = 0, we have,

To understand double-line-to-ground (LLG) fault con- I f (a 0) + I f (a1) + I f (a 2) = 0  (5.51)



dition, consider three phase lines of a-b-c as shown in
Fig. 5.46, where a LLG fault has occurred at node  k From the Thevenin equivalent circuit for LLG fault
between lines b and c of the network. It is assumed that shown in Fig. 5.47, we get
the phases b and c are shorted to ground through the
impedance  Zf  in Fig. 5.46 and the system is unloaded If(a1)
before the occurrence of the fault. Therefore, the phase
a current is given by
If(a2)
1 1
Zkk1 If(a0) Zkk0 Vk(a0)
I f (a 0) =(I + I f (0) + I f (c) ) = (I f (0) + I f (c) )
  3 f (a ) 3 Zkk2
 Þ 3I fa(0) = I f (0) + I f (c) (5.45) +
Vk(a1) Vk(a2)
k
a 3Zf
Vf
k −
a
Figure 5.47 |   Thevenin equivalent of a LLG fault.
k
b
If(b) k Vf
b I f ( a1) =
If(b) Zkk1 + Zkk2 (Zkk 0 + 3Zf )
k Vf
c =
Z (Z + 3Zf )
k Zkk1 + kk2 kk 0
c Zkk2 + Zkk 0 + 3Zf
If(c)

If(c) Zf  The zero and negative sequence currents can be obtained


Zf using the current divider principle as
 Zkk2 
I f (a 0) = −I f (a1)  
 Zkk2 + Zkk 0 + 3Zf 
Figure 5.46 |   Representation of LLG fault.

Chapter 5.indd 403 3/23/2016 12:02:45 PM


404     Chapter 5:  POWER SYSTEMS 

æ Zkk 0 + 3Zf ö Prior to the occurrence of the fault, the system is assumed
I f (a 2) = −I f (a 1) ç ÷ to be in balanced state and the bus voltage factor is
è kk2
Z + Z kk 0 + 3Z f ø
Vbus (0) = [V1(0)… Vk (0)… Vn (0)]T  (5.52)

5.15.2  Symmetrical Fault Analysis In the event of fault, the voltage of all the buses changes
due to flow of heavy current through the transmission
A three phase symmetrical fault results when three equal lines and is given by.
fault impedances Zf are applied to the three phases. Since DVbus = [ DV1 … DVk … DVn ]T  (5.53)

the applied impedance affects the three phases equally,
the system remains balanced and the fault is called sym- The change in voltage can be calculated by applying
metrical. The fault analysis is done for each phase. These voltage Vk (0) at the kth bus and short circuiting all
faults are of two types and are depicted in Fig. 5.48. other voltage sources. Since all three phases are short-
circuited simultaneously, the network remains balanced
  1.  Line-to-line-to-line (LLL) fault. and is analysed on per phase basis. The other two phases
  2.  Line-to-line-to-line-to-ground (LLLG) fault. carry the same current but with a phase shift of 120°.
a b c a b c Thevenin equivalent circuit is shown in Fig. 5.50, where
Zi and Zk are equivalent load impedances at buses’ i
Zf Zf Zf Zf Zf Zf and k, respectively, Zik is the impedance of line between
ith and kth bus, X d is the generator reactance and Ik(f ) is
the fault current.

(a) Bus i
Xd Zik
(b)
Figure 5.48 |   Representation of (a) Line-to-line-to-
Bus k
+
line (LLL) and (b) Line-to-line-to-line- Zi Vk(0)
to-ground (LLLG) faults. Zk −
S
The two types of faults are identical in their behaviour and
severe in their impact. If Zf = 0, the fault is called a solid
Zf Ik(f)
or a bolted fault and it is the most severe fault that can
occur in a system. However, the frequency of occurrence of
these faults is very low.
In the symmetrical fault analysis, the fault is simu- Figure 5.50 |   Thevenin equivalent of symmetrical
lated by connecting impedances in the network at the fault analysis.
fault location. The faulted network is analyzed by using
Thevenin’s equivalent network, as seen from the fault The bus voltage due to fault is given by
Vbus(f ) = Vbus (0) +  ∆Vbus  
point. Consider the n-bus power system network shown
(5.54)
in Fig. 5.49, in which the balanced three phase fault
occurs at the kth bus using fault impedance Zf . And the bus injected current is given by
Bus i I bus = éëVbus ùû éëY bus ùû  (5.55)

where Vbus is bus voltage vector and Y bus is the bus
admittance matrix.
Bus k
Sk The node equation for the network can be written in
Si terms of fault current Ik(f ), as all bus other currents are
zero.
Vk(f) é 0 ù é Y11  Y1k  Y1n ù é DV1 ù
ê  ú ê  úê  ú
Ik(f ) Zf
ê ú ê úê ú
−I
ê k(f ) ú = ê Yk1  Ykk  Ykn ú ê DVk ú
ê  ú ê  úê  ú
Figure 5.49 |   n-Bus power system network for ê 0 ú êY úê ú
ë û ë n1  Y nk  Y nn û ë DV n û
symmetrical fault analysis.

Chapter 5.indd 404 3/23/2016 12:03:09 PM


5.15  FAULTS     405

Also, from Eq. (5.5). 5.15.2.1 Short Circuit of Synchronous


Generator
éI bus(f ) ù = éY bus(f ) ù é DVbus(f ) ù
ë û ë ûë û
In the event of a symmetrical three-phase fault at the ter-
Hence Vbus can be calculated as minals of synchronous generator, a transient DC compo-
−1
éë DVbus ùû = éëY bus(f ) ùû éI bus(f ) ù = éZbus(f ) ù éI bus(f ) ù  (5.56)
ë û ë ûë û
nent gets added to the current flow through the phases,
The generator has AC current and voltages before the
fault but both AC and DC currents are present after the
where [Zbus(f ) ] = [Y bus(f ) ]−1 is the bus impedance matrix. fault as shown in Fig. 5.51.
Substituting this value of DVbus in Eq. 5.54, we get

Vbus(f )  = Vbus (0) + Zbus(f )  I bus(f ) 


      0 Time

which can be represented in expanded form as Phase a


Current
 V1(Vf V)1(1V f()f1 )(f )VV(V 10V ()f(V (10)()0Z
)0 )VZ 1Z 011
(11 
Z
)11 

ZZ1kZ 1k1Z
11Z 
k
1k
Z
Z1n1ZkZ 1n0 Z
1n1Zn 001n0  0 
     1  11   11                DC component
                    
 −I  −I−I−I 
  (5.57) 
)k(kV
 Vk(Vf V )fk)(f=
f(= )V= kV= )0(k+0)(=
(Vk0Vk()fk(V )0+Z 1+
)+kV Z 
(k01kZ
kZ +
)1k1
YZkkY
k1Y Y
kkkk 
kk
Z
Yknkk
ZZ  
knkn
Z  fZ
 k(
kn k(
)k( )f)−I
f )fk(
kn  k(f ) 
                      
                 Z   0 Z    
(a)
V V V V V VV( 0V ) (V 0
( 0
) () 0 Z) V Z Z( 0 
Z) 
Z
 n(f )nn(f()nf )(f ) n nn(fn) n  n1n nn1 1n1  nkn1nknknk nnnknnnn
Z Z Z Z  
Z
Z Z Z  nn  00nn0  0 
The bus voltage at the faulted bus can be given by
Vk(f ) = Vk (0) − Zkk Ik(f ) 
DC component
(5.58)

where Zkk is the Thevenin impedance of the network as Current
seen from the faulted bus k.
From Fig. 5.49, we have
0 Time
Vk(f ) = Zf Ik(f )
Phase b
For zero fault impedance, Zf = 0 , therefore, Vk (F) = 0 , (b)
so from Eq. (5.58), the fault current is
Vk (0)
Ik(f ) =  (5.59)
Zkk
0 Time
For non-zero fault impedance the fault current is Current
Vk (0)
Ik(f ) =
Zkk + Zf DC component Phase c

After the fault, the bus voltage for the unfaulted buses (c)
Figure 5.51 |   Current through the synchronous genera-
can be written from Eq. (5.57) as
Vi(f ) = Vi (0) − Zik Ik(f ) ∀i = 1, 2, … n, i ≠ k tor after three-phase symmetrical fault.
(a) Phase a, (b) Phase b and (c) Phase c.
Substituting for Ik(f ), from Eq. (5.59) in the above equa-
tion, we get voltage at the ith bus as The transient DC component of current is such that the
sum of the AC and DC components just after the fault
Zik is equal to the AC current just before the fault. The
Vi(f ) = Vi (0) − V (0)
Zkk + Zf k magnitude of DC components is different in different
phases because the instantaneous values of current at
The current flowing (post the fault) between the ith and the moment of the fault are different in each phase.
jth bus is given by
The AC component of the symmetrical fault current
Vi(f ) − V j(f ) can be divided into three periods as shown in Fig. 5.52:
I ij(f ) =   1.  Sub-transient period: This corresponds to first
Zij
cycle of the fault where the AC current is large
where zij is the impedance of line between i th and j  th bus. (about 10 times the steady state value) and also

Chapter 5.indd 405 3/23/2016 12:03:33 PM


406     Chapter 5:  POWER SYSTEMS 

falls rapidly. The sub-transient current I ′′ (rms conductors of a transmission line get disconnected due
value) is given by to storm. This can also be a result when fuses, isolators
or circuit breakers operate only on one or two phases
Eg
I ′′ = leaving others connected. These are known as open
Xd′′ conductor faults. These faults can be analysed with
the assistance of [Zbus] matrices of sequence networks.
where Eg is the internal generated voltage and Xd′′
Figure 5.53 shows a three-phase system with buses i
is the sub-transient reactance.
and j are shown, in which with Fig. 5.53(a) has one con-
  2.  Transient period: The rate of fall of the AC ductor open while Fig. 5.53(b) has two conductors open.
current is reduced in this phase, to about five These points are marked as k and k  ′. The fault analysis
times the steady state value. The transient current for such a system the Thevenin’s impedance between the
I′(rms) value is given by buses is to be found and the relationship between the
elements of ëéZbus ûù and Thevenin’s impedances at each
Eg
I′ = bus needs to be computed.
Xd ′ Ia k k¢
i Ia k k¢ j
  3.  Steady state period: Here, the current reaches i j
a steady state value, given by the ratio of induced Ib
voltage and synchronous reactance. i Ib j
i j
Eg
I ss = I
Xd i Icc j
i j
Sub-transient Transient Steady-state (a)
period period period (a)

Ia k k¢
i Ia k k¢ j
i j
Ib
i Ib j
Short circuit 0 Time i j
current

I
Extrapolation of Actual i Icc j
steady value envelope i j
Extrapolation
of transient (b)
Figure 5.53 |   Three-phase system open conductor
envelope (b)
Figure 5.52 |   Symmetrical fault current in faults. (a) Single conductor.
synchronous generator. (b) Two conductors.

For the synchronous generator, the variation of AC If [V (0) ] is the open-circuit bus voltage matrix corre-
fault current (rms value) with time (t) is given by sponding to the initial bus current vector before fault
′′ ′
[I (0) ] injected in a network. Then,
I (t) = (I ′′ − I ′ )e−t / T + (I ′ − I ss )e−t / T + I ss
[V (0) ] = [Zbus ][I (0) ]
where T  ′′ and T  ′ are the sub transient and transient
time periods, respectively. When the bus currents are altered to a new value,
[I (0) + DI ], the new bus voltage [V] [V] can be
given as:
5.15.3  Open Conductor Faults
[V ] = [Zbus ][I (0) + DI ]
Opening of one or two phases of a balanced three-phase
line due to faults creates an unbalance in the system. = [Zbus ][I (0) ] + [Zbus ][ DI ]

= [V (0) ] + [ DV ]
This results in the flow of unbalanced currents. Such
a fault may occur in power systems when one or two

Chapter 5.indd 406 3/23/2016 12:03:46 PM


5.15  FAULTS     407

where, [ DV ], is the change in the value of the original


(1) (2) (0)
Zkk ′ Zkk′ Zkk′
(1) (2)
V =V = V0′ = I ij
bus voltage [V (0) ]. kk ′ kk ′ kk (0) (1)
Zkk (1) (2) (2) (0)
′ Zkk′ + Zkk′ Zkk′ + Zkk′ Zkk′
Now, we consider the two cases for one and two- phase
I ij is the pre-fault current or load current in phase a.
open conductors.
The equivalent injected currents are,

5.15.3.1 One Phase Open Conductor (1) (2) ( 0)


Vkk¢ Vkk¢ Vkk ¢
(1)
, (2)
and ( 0)
Consider a circuit where only phase a conductor is open, zij zij zij
phase a current Ia = 0.
where zij is the line impedance between ith and jth bus.
Thus,
Ia(1) + Ia(2) + Ia0 = 0
The bus voltages after fault are calculated using
superposition as,

where, (1), (2) and (0) represents the symmetrical com- Vi(1)(f ) = Vi(1)(0) + DVi(1)
ponents of phase a current.
Vi(2)(f ) = DVi(1)
The phases b and c are closed, thus the voltage
drops are, Vi(0)(f ) = DVi(2)

kk′ (b) kk′ (c)


V = 0 and V =0
5.15.3.2 Two Phase Open Conductor
Across the fault, the symmetrical components of voltage
drops are, When the two phases b and c are open, the voltages and
currents are,
Va(0)  1 1 1  Vkk ′(a )  (1)
 (1)  1  Vkk′(a ) = Va(0) + Va(1) + Va(2) = 0
2  
 Va  = 1 a a   0 
V (2)  3
1 a 2 a   0 
 a  I b = 0 and I c = 0
Vkk ′(a )  Sequence components of line current are given as,
1 
= Vkk ′(a )  éIa(0) ù
3 é1 1 1 ù é Ia ù
 ê (1) ú 1 ê úê ú
Vkk ′(a )  ê Ia ú = ê1 a a2 ú ê0 ú
ê I (2) ú 3 ê1 a 2 a úú êê 0 úú
(1) êë a úû êë ûë û
The current Ia is,
(1) é (2) ( 0) ù After simplification,
¢ ëZkk¢ + Zkk¢ û
Ia( ) = I ij
Zkk
1
(0) (1) (1) (2) (2) (0) 1
Zkk ¢ Zkk¢ + Zkk¢Zkk¢ + Zkk¢ Zkk¢ Ia(0) = Ia(1) = Ia(2) =
I
3 a
The sequence network is shown in Fig. 5.54. The sequence Thus, the sequence circuits are in series and can be
voltage drops from the Fig. 5.54 are, drawn as shown in Fig. 5.55,

(1) (1) + (2) (2)


+ (0) (0)
+
Zkk¢ Ia k Zkk¢ Ia k Zkk¢ Ia k
(1) + (1) (2) (0)
Iij Zkk¢ − Vkk¢ Vkk¢ Vkk¢
k¢ k¢ k¢
− − −

Figure 5.54 |   Sequence networks to represent open phase a where, positive,


negative and zero sequence networks are connected in parallel.

Chapter 5.indd 407 3/23/2016 12:04:04 PM


408     Chapter 5:  POWER SYSTEMS 

The schematic of an electromechanical relay and its


(1) (1) +
Ia operation is shown in Fig 5.56. The power source is pro-
Zkk′ k
(1) + (1) vided to the electromagnet through a control switch and
Ii Zkk′ − Vkk′ through contacts to the load. When current starts flow-
k′ ing through the control coil, the electromagnet starts
− energizing and thus intensifies the magnetic field (see i).
Thus the upper contact arm starts getting attracted to
the lower fixed arm. This closes the contacts causing a
(2) (2)
+
Zkk′ Ia k short circuit for the power to the load (see ii). On the
(2) 1 other hand, if the relay was already de-energised when
I
Vkk′ 3 a the contacts were closed, these move in opposite direc-
k′ tion and make an open circuit (see iii). As soon as the
− coil current is off, the movable armature is returned back
to its initial position by a force provided by spring and
+ gravity.
(0) (0)
Zkk′ Ia k
(0)
Vkk′ From To load
power
k′
− source

Figure 5.55 |   Sequence networks to represent open Contacts


phase b and c, where positive, negative
Control open
and zero sequence networks are switch
connected in series. Control
(1) coil
Ia(1) = Ia(2) = Ia(0) =
Zkk ′
I ij (0) (1) (2)
Zkk ′ + Zkk ′ + Zkk ′
Iron core
The sequence voltages are, (a)
( )
( )
(1) ( 0) (2)
(1)
Zkk ′ Zkk ′ + Zkk ′
= Ia(1) Zkk ′ + Zkk ′ = I ij
( 0) (2)
Vkk ′ ( 0) (1) (2)
Zkk ′ + Zkk ′ + Zkk ′
(1) (2)
(2 )
Vkk ′ = −Ia(2)Zkk ′ = −I ij
(2) Zkk ′ Zkk ′
( 0) (1) (2)
Zkk ′ + Zkk ′ + Zkk ′
(1) (0)
(0 )
Vkk ′ = −Ia(0)Zkk ′ = −I ij
( 0) Zkk ′ Zkk ′
( 0) (1) (2)
Zkk ′ + Zkk ′ + Zkk ′
Similar to the single conductor fault, the other calcula-
tions can be done.
(i) (ii) (iii)
(b)
Figure 5.56 |   Electromechanical relay. (a) Schematic
5.16  RELAYS
representation. (b) Operation.
Protective relays are devices that detect abnormal con- Solid state relays (SSR) are similar in operating principle
ditions in the circuit, generally by measuring quantities to normal electromechanical relays. These not have any
(such as current, voltage) that change on occurrence of a movable contacts, instead they have semiconductor switch-
fault and comparing these against the threshold settings. ing elements. They also employ opto-couplers to isolate
These then detect the fault, issue trip signals to open the input and output. SSRs have a variety of advantages
the circuit breaker and isolate the faulted section from over the mechanical relays. They are ideal for a wide range
the rest of the power system, to prevent further damage. of applications as they provide high-speed and frequency
Protective relays respond to and operate in accordance switching, no mechanical failures and thus are more reli-
with the defined operating characteristics and applied able. These also generate almost no or sometimes low noise
settings. during operation. Figure 5.57 shows a typical SSR circuit.

Chapter 5.indd 408 3/23/2016 12:04:12 PM


5.17  PROTECTION     409

Input DC 5.17.1  Overcurrent Protection


Output
The occurrence of a fault in a line can cause the cur-
Ri device
rent to increase to a value several times greater than the
(triac)
Opto -coupler maximum load current. Overcurrent protection protects

Snubber
electrical power systems against continuous or transient
-
excessive (over) currents, caused by short circuits, ground
+ (earth) fault, phase faults or winding faults. It is the sim-
Zero-crossing plest and the most widely used ways to protect any power
detector and system elements, that is, transmission lines, transformers,
triggerring generators, or motors. This protection scheme generally
uses relays and fuses to indicate the presence of short cir-
Figure 5.57 |   Schematic of SSR relay. cuit or fault when the current exceeds a pre-determined
permissible value. This type of protection scheme is used
AC type SSR turns ON at the zero crossing point of in simple systems, such as in radial distribution system.
the AC waveform, avoids high in-rush currents when Consider a radial power distribution system with a single
switching inductive or capacitive loads. The turn OFF source, as shown in Fig. 5.58, with short-circuit faults F1
feature of thyristors and triacs provides performance and F2. The fault current is fed from only one end of the
enhancement over the arcing contacts of electromechani- feeder. For feeder protection, more than one overcurrent
cal relays. relays may be used to protect different sections of the
feeder. These overcurrent relays need to coordinate with
each other such that the relay nearest fault operates first.
Here, two relays R1 and R2 are used for the two sections.
5.17  PROTECTION For relay R1, both downstream faults F1 and F2 are vis-
ible, which means both IF1 and IF2 pass through the CT
of R1. For relay R2, the upstream fault F1 is not visible
A power system comprises a number of components, and only F2  is seen. This is because no component of
such as generator, transformer, transmission lines and IR1 will be pass through the CT of R2.
bus bars. In event of a fault in any of these, these com-
ponents need to be protected by means of protective
relays, circuit breakers and instrument transformers. F1 F2
The protection scheme may work at the system (unit)
level or the equipment level. The purpose of equip- R1 R2
ment protection is to rapidly detect the fault and dis-

Figure 5.58 |    Radial distribution system with


connect the faulted part from the rest of the system.
Dependability, speed of operation, security and selec-
tivity are some of the important performance param- short-circuit fault.
eters for protection. The clearing time for the fault is Generally the use of relays is based upon solely on the
generally defined for a system based on the backup magnitude of fault current. This type of protection
protection. scheme is said to be non-directional. Overcurrent
The protective elements can operate on the basis of: protection, thus helps detect abnormal conditions in
the power system or its components; isolate these faulty
  1.  Overcurrent protection function by operating when
parts and speed-up the operation to control damage due
the input current exceeds the permissible load cur-
to the fault.
rent value.
  2.  Directional protection function by selectively pick-
ing up the faults in one direction and remaining 5.17.1.1 Directional Overcurrent Protection
stable to faults in the other directions.
  3.  Distance protection function used in protection of When the power system is a radiant power system with
transmission lines, where the response is dependent source at both the ends, an overcurrent relay may not be
upon the electrical distance between the relay loca- able to provide adequate protection. This type of relay
tion and the fault. operates in on direction of current flow and blocks in the
  4.  Differential protection function by detecting the opposite direction. Consider a radial system with source
difference between incoming and outgoing current at both ends where the fault current is fed from both
through the equipment. the ends of the feeder, as shown in Fig. 5.59. Here, the

Chapter 5.indd 409 3/23/2016 12:04:14 PM


410     Chapter 5:  POWER SYSTEMS 

relays are required to be positioned at both the ends of some finite value, Zapp = ZL+ Zline, where ZL is the load
the feeder to control the fault current. It is not possible impedance and Zline is the total line impedance.
to distinguish whether the fault is in the section AB or The apparent impedance reduces considerably in the
BC from the magnitude of the current seen by the relay presence of a line fault. If the fault occurs at a pu dis-
R2, The faults in section AB are not under the control tance `x’, the impedance observed by the relay drops to
of relay R2, and some arrangements needs to be made to xZline. A distant relay compares this Zapp with the posi-
prevent tripping of the line due to the fault. For the pur- tive sequence impedance (Z1) of the transmission line. If
pose, a directional overcurrent relay is required, which the fraction of Zapp/Z1 is less than one (< 1), the pres-
use both the magnitude of current and the phase-angle ence of a fault is indicated. This ratio also indicates the
information to interrupt the fault distance of the fault from the relay.
Since impedance is a complex number and the dis-
A B If2 C tant protection is directional, that is, the impedance of
R1 R2 the transmission line is to be protected lying in the first
quadrant, which is in the forward direction. However,
If1 if we consider only the magnitude, the plot of imped-
ance relay is obtained as shown in Fig. 5.61(a) and
F2 F1 Fig. 5.61(b) shows the characteristics of mho relay. The
L1 L2 L3 L4
impedance relay trips if the magnitude of the impedance
VB is within the circular region. Since, the circle covers all in
the quadrants, the protection scheme is non-directional.
If2 If1 However, the mho relay covers mainly the first quad-
rant and therefore this protection scheme is directional
Figure 5.59 |   Directional overcurrent protection in nature.
scheme in a radial system with source
at both ends. X

Such directional relays are commonly used in sub-­ B


transmission networks where the ring mains are used.
AB = Zline

5.17.2  Distance Protection


R
When a fault occurs in the system, changes in the mea- A
sured impedance are observed due to changes in the
measured voltage and current. The distance protec-
tion element measures the ratio of voltage and current
(impedance, Z) considering the phase angle between V
and I and compares it against the set threshold value.
(a)
To understand this, consider a simple radial system
fed from a single  source, as shown in Fig. 5.60. The X
apparent impedance (V/I) is measured at the sending
B
end.

A B Zline
Z
Zline

xZline (I − x)Zline q
F Load l
R
(ZL) A
Figure 5.60 |   Distance protection scheme for fault in
transmission line.
(b)
For an unloaded system, I = 0, so the appar- Figure 5.61 |   R—X diagram of (a) impedance relay
ent impedance(Zapp) observed by the relay is infinite. In and (b) mho relay.
the loaded system,  the apparent impedance reduces to

Chapter 5.indd 410 3/23/2016 12:04:19 PM


5.18  CIRCUIT BREAKERS     411

Thus, the trip law for the impedance relay is given by: Thus it incorporates manual as well as automatic con-
trol with switching functions. The latter control employs
VR
if Zapp = < Zset ,   relays and operates only under fault conditions. The sche-
IR matic diagram of a circuit breaker is shown in Fig. 5.63.

then trip, else be restrained.


The trip law for mho relay is given by:
Fixed terminal
if  Zapp < Zk cos(q − l ),
then trip; else be restrained.

5.17.3  Differential Protection End


shield
Differential protection is based on the principle that in
event of a fault in the equipment, the current entering the
equipment differs from the current leaving it. Thus, by Electrodes
comparing the two currents in either magnitude or phase, Insulting
or both, the fault can be identified and tripping occurs - envelope
if the difference is more than the predetermined value. Shield
support
To understand this protection scheme, consider an
ideal transformer with the CT connections, shown in
Vapour
Fig. 5.62. Let the current rating of primary winding is
condensation
100 A and secondary winding is 1000 A. Then if CT
shield Bellows
ratios of 100:5 and 1000:5 are used for primary and sec-
ondary windings, respectively, under no-fault condition, shield
the scaled CT currents will match in their corresponding
magnitudes and there is no circulating current set up. Bellows
In case of is fault inside the transformer, the currents in
the CT secondaries will differ and a resulting circulating
current will be set up and this will be picked up by the Movable terminal
sensor coil and will activate the relay.
I1 I2 Figure 5.63 |   Schematic diagram of a circuit breaker.
N1 N2
A circuit breaker essentially consists of fixed and
moving contacts, called electrodes. Under normal
V1 V2 operating conditions, these contacts remain closed
and open automatically when the system becomes
faulty. The contacts also can be opened manually or
by remote control whenever desired even without fault
conditions. For automatic opening, the trip coils of the
Current sensor circuit breaker get energised and the moving contacts
are pulled apart by some mechanism, thereby opening
Boundary of differential the circuit.
protection When the contacts of a circuit breaker are separated
Figure 5.62 |   Differential protection for transformer.
under fault conditions or during running conditions, an
arc is formed between the fixed and the moving contacts.
The current thus continues until the discharge through
arc ceases. The production of arc delays the current inter-
5.18  CIRCUIT BREAKERS ruption process and also generates enormous heat which
may cause damage to the circuit breaker. Therefore, the
main problem in the use of circuit breaker lies in how to
A circuit breaker is a piece of equipment which can make extinguish the arc within the shortest possible time so
or break a circuit either manually or by remote control that heat generated by it cannot reach a value that can
and also break both under normal or fault conditions. damage the breaker.

Chapter 5.indd 411 3/23/2016 12:04:23 PM


412     Chapter 5:  POWER SYSTEMS 

5.18.1  Arc Phenomenon R1 Y3 B5 R1

In the event of a short circuit, a heavy current flows


through the contacts of the circuit breaker before they
are opened by the mechanism inside it. At the instant
when the contacts begin to separate, the contact area
between the two contacts decreases rapidly and the large Y6 B2 R4 Y6
current results in increased current density and hence (a)
rise in temperature. The heat produced in the medium
between contacts, usually oil or air, is sufficient to ionise
Y6,R1 R1,B2 B2,Y3 Y3,R4 R4,B5 B5,Y6 Y6,R1
the air or simultaneously vapourise and ionise the oil.
The ionised air or vapour acts as conductor and an arc
is struck between the contacts. The potential difference
between the contacts is still quite small and is still just
sufficient to maintain the arc which has low resistance. (b)
The greater the arc resistance, the smaller is the current Figure 5.65 |   (a) Thyristor voltage waveform (a = 0).
between the contacts. The arc resistance strictly depends (b) DC output waveform (a = 0).
upon the following two factors:
  1.  Degree of ionisation: The arc resistance It can be shown that for the six-valve bridge, the total
decreases with the increase in the number of ion- rms ripple is of the order of 4.2% of the DC value. The
ised particles between the contacts. ripple increases with the delay angle and has a value
  2.  Length of the arc: The arc resistance increases of 30% at a = p/2. With the twelve-pulse bridge, the
with the length of the arc, that is, separation rms ripple is of the order of 1.03% of the DC value and
between the two contacts. increases to 15% at a = p/2.
If E is the rms line-to-line voltage, the direct voltage
5.19  HIGH VOLTAGE DIRECT is given as,
CURRENT TRANSMISSION p 3


E 3
Vd = 2x x 2x cos q dq
3 2p −p 3
The use of high voltage direct current (HVDC) transmis-
3 2 1  p
sion gained significance with the advent of devices with =E⋅ ⋅ 2x sin 3 
ability for rectification and inversion. The three-phase p 3
six-pulse bridge rectifier, as shown in Fig. 5.64, is the 3 2
= ⋅ E = 1.350E
most extensively used in HVDC applications. The con- p
verter transformer is generally wound in delta-star but
can also be star-star wound as shown. The control angle for rectification is the angle by
which firing pulse is delayed beyond the natural takeover
+ for the corresponding thyristor. It is also called the delay
angle. The firing delay angle is usually indicated by a
T1 T3 T5
as shown in Fig. 5.65. Assuming no commutation delay
R for thyristors conducting simultaneously during transfer,
the voltage waveforms across the thyristors are shown
T4 T6 T2 in Fig. 5.66.
B Y −
Controlled rectifier R1 Y3 B5 R1
Transformer bridge
Figure 5.64 |   Configuration of AC to DC conversion a a
for HVDC transmission.

The conducting period for the thyristor T1 for phase


R is shown in the Fig. 5.57(a) in hatched lines. The Y6 B2 R4 Y6
corresponding DC output voltage waveform is shown in
Fig. 5.65(b). Figure 5.66 |  Thyristor voltage waveforms (with delay a).

Chapter 5.indd 412 3/23/2016 12:04:28 PM


5.19 HIGH VOLTAGE DIRECT CURRENT TRANSMISSION     413

The magnitude of the DC voltage is given by, the amount of harmonics transferred to the AC
p 3 +a system but DC system requires additional reactor

E 3
VDC = 2x x 2x cos q dq components,; thus adding to the cost.
3 2p −p 3 +a   4.  Circuit breaking is difficult with DC due to absence
of natural current zero. The lack of this protection
3 2 1  p  p 
=E⋅ ⋅ sin  3 + a  + sin  3 − a  
equipment imposes restrictions on increasing the
p 3   transmission voltage beyond pre-set values.
3 2   5.  Voltage transformation is not easy.
= E cos a = Vd cos a
p
  6.  Converters lack the overload capacity available in
transformers.
The commutation period between two thyristors on
the same side of the bridge is the angle by which one 5.19.2  Corona
thyristor commutates to the other as shown in Fig. 5.67.
During this period, the voltage follows mean voltage of When a power is transmitted though HV transmission
the two conducting thyristors at the same side. lines, usually ionisation around the conductor takes
places. If the ionisation becomes very intense due to
g R1 w Y3 B5 R1 several factor like humidity, pressure of changes in the
atmosphere, etc., a self-sustained discharge with sound
takes place. This phenomenon is known as corona.
a
The critical disruptive voltage is defined as the voltage
at which complete disruption of the dielectric occurs
Figure 5.67 |   Commutation between two thyristors.
between the two conductors.

With both the delay angle and commutation angle being


A B
present, the magnitude of the DC voltage may be deter-
mined from equation as follows: Q r −Q
P
p 3 +a x

E 3
VDC = 2 2 f (q ) dq
3 2p −p 3 +a
d

3 2E 
−p 3 +a +g
1  2p   Figure 5.68 |   Two transmission lines with charges Q
= ∫
3p  −p 3 +a 2 
cos q + 3  + cos q  dq
 and −Q per unit length.

p 3 +a  Let us consider two lines running such that their charges
are Q and −Q per unit length, as shown in Fig. 5.68. The
+ ∫ cos qdq 
−p 3 +a +g 
electric field intensity at any point P at a distance x from
the conductor with change Q is given by,
Vd
= [cos a + cos (a + g )]
2 Q Q Q 1 1 
Ex = − =  − 
2pe 0 x 2pe 0 (d − x) 2pe 0 x d − x 
5.19.1  Comparison of HVAC and HVDC
The potential difference between the two conductors can
There are some characteristic problems associated with be calculated as(assuming the conductors are at larger
HVDC in comparison to AC transmission and these distance).
include:
d −r d −r
q 1 1 
r
  1.  HVDC requires expensive converters at each end of VAB = − ∫ Ex dx = ∫ Ex dx = ∫ 2pe 0
 −  dx
x d − x
a DC transmission link; whereas, AC transmission d −r r r
requires only transformer stations
  2. Converters require high reactive power (up to 50% Q d −r Q  d−r r 
= (ln x − ln(d − x) r =  ln − ln 
of the active power rating) both for rectification and 2pe 0 2pe 0 r d − r
inversion. The reactive power requirement is met
with installation of synchronous or static capacitors. Q d−r
  3.  Convertors generate a lot of harmonics both on the = ln
pe 0 r
DC and the AC sides. Filters can be used to reduce

Chapter 5.indd 413 3/23/2016 12:04:37 PM


414     Chapter 5:  POWER SYSTEMS 

5.20  ECONOMICS OF POWER the power station are also semi fixed charges and
GENERATION charges. It also includes cost incurred on account of
transportation, labour, etc. Taxes, insurance pre-
mium, policies, etc. will also be regarded as semi
The process of determining the per unit cost of production fixed. The cost due to starting and shutting down
of electrical energy can be regarded as economics of power of plants are also included in this category.
generation. To determine the power generation econom-   3.  Running charges: This is the most important
ics effectively, the structure of annual expenditure of the parameter which is considered for power genera-
plant and the factors affecting them are to be known. The tion economics. It depends on how much electricity
generation plant economics can be sub-divided into dif- the plant can produce and the cost of fuel for the
ferent charges, viz. fixed, semi-fixed and running charges. production of electricity. This may also include the
These are all important parameters pertaining to the eco- fuel handling costs. Other than that, operational
nomics of power generation and are discussed as follows: and maintenance and variable labor costs come
under this category.
  1.  Fixed charges: These do not vary either with
the capacity of the plant or with plant operation. The total annual charges incurred in the power genera-
These charges include capital cost and land costs tion can be represented by the equation,
for the generation plant which largely remain fixed E = Rs. (a + b kW + c kWh),
under all circumstances. These also include the sal-
aries of the operating officials of the organisation where, a is the fixed charge for power generation. b is the
and the rent of the land if any. semi fixed charge and c is the running or variable charge
  2.  Semi fixed charges: These charges may include for power generation. Sometimes, the above expression
the interest and depreciation on the capital of the can be converted to a more convenient two part form, a
plant, its transmission and distribution network. fixed sum per kW of maximum demand represented by
The buildings and other civil engineering works d and a running charge per kWh of energy represented
and their depreciation can also be included in semi by e. Thus,
fixed charges. Development and construction of E = Rs. (d kW + e kWh).

IMPORTANT FORMULAS

1. Some terms related to power generation


y 1 = 2 × 10−7 {I1 ln(1/r1 ) + I2 ln(1/D12 )
Maximum demand
+ I3 ln(1/D13 ) + ... + I n ln(1/D1n )}
(a)  Demand factor =
Connected load
Average demand (c)  Three asymmetrically conductors spaced in a
(b)  Load factor =
Maximum demand transmission line
(c)  Diversity factor  1 3 b
La = 2 × 10−7 ln − ln
1
−j ln 
 r1′
Sum of individual maximum demands bc 2 c
=
Maximum demand of the system
 1 3 a
Lc = 2 × 10−7 ln − ln
1
Station output in kWh −j ln 
 r1′
(d) Utilisation factor = b
Plant capacity × hours of use ba 2

 1 3 c
Lb = 2 × 10−7 ln (1/) − ln
(e) Plant capacity factor 1
)− j ln 
=
Average demand  r1′ ac 2 a
Plant capacity(maximum installed capacity)
(d)  Composite conductors:
2. Inductance of:
D 
LA = 2 × 10−7 ln  m  H/m
 Ds 
(a)  Two-wire transmission line (per unit length)

m0   D 
0.25 + ln  r   H
L= where Dm is the geometric mean distance (GMD)
p  
′ D12
Dm = mn (D11 ′ ...D1′n )(D21
′ D22
′ ...D2′ n )...(Dm
′ 1Dm
′ 2 ...Dmn
′ )
(b)  Group of conductors in a transmission line:

Chapter 5.indd 414 3/23/2016 12:04:51 PM


IMPORTANT FORMULAS     415

and Ds is the geometric mean radius (GMR) (d)  Percentage efficiency


Ds = m (r ′D12D13...D1m )(r ′D21D23...D2m )...(r ′Dm1Dm2 ...Dmm ) %h =
P
2

× 100%
P + 3I r2R
3. Capacitance calculations of:
7. Medium transmission lines
(a)  Two wire line
(a) Nominal T representation
0.0121
µF/km
( )
CAB = (i)  Percentage efficiency
log D/ ra rb
%h =
P
× 100%
(b) Three phase line of equal spacing and of same P + 1.5(I s2 + I r2 )R
area
q 2pe 0.0242 (ii)  ABCD parameters:
CAN = A = = µF/km
VAN ln D/r log(D/r) A = D = 1 + YZ/2
(c)  Three phase line with unsymmetrical spacing B=Z
and C = Y (1 + YZ/4)
qA 2pe 0.0242
CAN = = = µF/km
 Deq   Deq 
where Z is the series impedance per unit
VAN
ln 
 r 
log 
 r 
length and Y is the shunt admittance per
unit length.
(b) Nominal Π representation
Deq = DABDBCDCA (i)  Current and no load receiving end voltage

wC wC
4. Effective resistance of a conductor I s = I r (cos fr − j sin fr) + jVr +j
P 2 2
R = loss Vs (−2 j/w C)
I2 Vr′ =
5. Performance of transmission lines R + jX − 2 j/w C
(a)  Percentage efficiency of transmission lines (ii)  Efficiency
P
=
Power received at the receiving end
×100 %h = ×100%
ding end
Power delivered at send P + 3I s2R
(b)  Percentage voltage regulation (iii)  Percentage voltage regulation
 No load receiving − Full load receiving Vr′ − Vr
 end voltage end voltage  ×100%
Vr
= ×100
Full load receiving end voltage (iv)  ABCD parameters
6. Short transmission lines YZ
A = D = 1+
(a) Relation between the delivering end and the 2
receiving end parameters B=Z

Vs cos fs = Vr cos fr + I r R C = Y (1 + YZ/4)

Vs sin fs = Vr sin fr + I r X 8. Long transmission lines


(a) Current and voltage at point x from the re-
where Vr, Ir and fr are the receiving end volt-
age, current and phase angle and Vs and fs
ceiving end:
are the sending end voltage and phase angle, V (x) = cosh(g x)Vr + Zc sinh(g x)I r
1
sinh(g x)Vr + cosh(g x)I r
respectively and X is reactance
I (x) =
IrR IX Zc
(b)  Regulation (in pu) = cos fr ± r sin fr
Vr Vr (b) Current and voltage at the sending end (l is
(c)  ABCD parameters of short transmission line the length of the line)
V s = AV r + B I r Vs = cosh(g l)Vr + Zc sinh(g l)I r
1
sinh(g l)Vr + cosh(g l)I r
I s = C V r + DI r
Ir =
A = 1, B = 2, C = 0 and D = 1 Zc

Chapter 5.indd 415 3/23/2016 12:05:20 PM


416     Chapter 5:  POWER SYSTEMS 

(c) Equivalent T representation 11. AC and DC transmission


(i)  Y and Z parameters (a)  Power transmitted with DC and AC links

sinhg l Z tanh(g l/2) 1.414 at pf = 1


Y′=Y and Z ′ = Pd 2
=
gl 2 (g l/2) =
cos f
Pa 1.768 at pf = 0.8
where g = a + jb
(b)  Power loss in DC transmission
(ii)  ABCD parameters:
YZ Ploss = (P/Vm)2 Rd = (P/Vm)2.(rI/Ad)
A = D = 1+
2 ••Power loss in AC transmission
B=Z
Ploss =  2P /(V cos f )  2 R
C = Y (1 + YZ/4)  m  a
(d) Equivalent Π representation = 2(P /Vm )2 ⋅ ( rI /Aa cos2 f )
••Ratio
(i)  Y and Z parameters
of areas:
sinhg l
Z′ = Z and Ad cos2 f 0.5 at pf = 1
gl = =
Aa 2 0.2 at pf = 0.8
Y tanh(g l/2)
Y ′/2 =
2 (g l/2) 12. Cables
(ii)  ABCD parameters (a)  Classification
A = D = cosh(g l) Low-tension (LT) cables up to 1000 V
High-tension (HT) cables up to 11,000 V
B = Zc sinh(g l) Super-tension (ST) cables from 22 kV to 33 kV
sinh(g l) Extra high-tension (EHT) from 33 kV to 66 kV
C= cables
Zc
Extra super voltage cables beyond 132 kV
where Zc is the characterstic impedance.
(b)  Dielectric stress in cables: Potential gradient
9. Combination of networks
V
(a)  Cascaded g= V/m
x ln D/d
éAB ù éA1B1 ù éA2 B2 ù
êCD ú = êC D ú + êC D ú
2V
gmax = V/m
ë û ë 1 1û ë 2 2û ln D/d
(b)  Parallel 2V
gmin = V/m
A B + A2 B1 D ln D/d
A=D= 1 2
B1 + B2 (c)  Most economical size of conductor
B1B2
B= D
B1 + B2 = e = 2.718
d
and AD − BC = 1 (d) Grading in cables: For capacitance grading,
10. Surge impedance total potential difference between the core
Z and earthed outer sheath
Zc =
Y gmax  d1 d2 D
V = d ln + d1 ln + d2 ln 
where, Z = R + jwL is the series impedance and 2  d d1 d2 
Y = G + jwC is the shunt admittance of the line.
Maximum stress between core and inner sheaths
(a)  Surge impedance loading
g1 max = g2 max = g3 max = gmax
V 2
SIL = LL V1 V2 V3
Zc = =
d d1 d1 d2 d2 D
ln ln ln
where VLL is the line-to-line voltage  2 d 2 d1 2 d2

Chapter 5.indd 416 3/23/2016 12:05:50 PM


IMPORTANT FORMULAS     417

13. Voltage distribution in suspension insulator


Pr + jQr = Vr I r*
2V 2 sin(d /2) 2V 2 [cos(d /2) − 1]
String efficiency
Voltage across the string = +j
= X X
n × voltage across the disc which is nearest to the string
(d)  Real power transmitted over the line
14. Per unit (pu) system
Per unit value of any quantity
2V 2
PE = Ps + Pr = sin(d /2)
Quantity X
Base value of that quantity (e)  Reactive power consumed by the line

15. Gauss—Seidel method of load flow analysis 8V 2


QE = Qs + QQ − Qr = [1 − cos(d /2)]
(a)  Complex power at i bus in a power system X
n 18. Series compensator
(b)  Pi,inj − jQi,inj = Vi * å Y ik Vk
(a)  Injected voltage
k =1

= Vi * [Y i1V1 + Y i2V2 + … VQ = lI s ∓ j 90°


+ Y iiVi + … + Y in Vn ] (b)  Line current
V∠d − V
(c)  Voltage of the ith bus using the procedure Is =
j(X ∓ l )
1 é Pi, inj − jQi, inj
Vi = ê − Y i1V1 (c)  Capacitive mode of operation
Y ii ë Vi *
ù V∠d − V
− Y i2V2 … − Y in Vn ú VQ = l I s e−j 90 ⇒ I s =
û j(X − l )
(d) Error between the actual and injected real
and reactive powers (d)  Inductive mode of operation

∆Pi = Pi, inj − Pi, calc = PGi − PLi − Pi, calc and V∠d − V
VQ = lI s e+ j 90 ⇒ I s =
j(X + l )
∆Qi = Qi, inj − Qi, calc = QGi − QLi − Qi, calc
19. Voltage control using tap changing transformers
(e)  Voltage of ith bus using acceleration constant l
Vs1Vr − Vr2 N r2 = RT PR + XT Qr
Vi,acc(k) = (i − l )Vi,acc(k −1) + lVi(k)
20. Frequency control
= Vi,acc(k −1) + l Vi(k) − Vi,acc(k−1) 
(a)  Equation for the area control error (ACE)
16. Newton—Raphson method for load flow analysis:
ACE = (Ptie − Psch ) + Bf ∆f = ∆Ptie + Bf ∆f
Steps described in Section 5.9.2
17. Shunt compensator where Ptie and Psch are tie-line and scheduled
(a)  Compensator current power through tie-line respectively; Bf is fre-
quency bias constant and Df is the frequency
[1 − cos(d /2)] ∠(d /2)
4V deviation.
IQ = j
X (b) Change in the reference of the power setting
(b)  Apparent power supplied by the source DPref,i, for any area i

Ps + jQs = Vs I s* ∆Pref , i = Ki ∫ ACE dt


2V sin(d /2)
2
2V [1 − cos(d /2)]
2
21. Power angle characteristics
= +j
X X
(a) Real power transmitted over the line is
(c)  Apparent power delivered at the receiving end given by

Chapter 5.indd 417 3/23/2016 12:06:12 PM


418     Chapter 5:  POWER SYSTEMS 

(iii) Positive, negative and zero sequence cur-


V2
PE = Ps + Pr = sin d rent for LLG fault:
X∓l
Vf
I f (a1) =
Zkk1 + Zkk2 (Zkk 0 + 3Zf )
(b)  Reactive power supplied by the compensator
Vf
2lV 2
=
QQ = −j (1 − cos d )
Zkk2 (Zkk 0 + 3Zf )
Zkk1 +
(X − l )2 Zkk2 + Zkk 0 + 3Zf
22. Normalised inertia constant (H  ) and swing equation
 Zkk2 
I f (a 0) = −I f (a1)  
 Zkk2 + Zkk 0 + 3Zf 
Stored kinetic energy at synchronous speed
H=
Generator MVA rating
Jw s2  Zkk 0 + 3Zf 
= MJ/MVA I f (a2) = −I f (a1)  
2Srated  Zkk2 + Zkk 0 + 3Zf 

2H d 2d 26. Symmetrical fault analysis


= Pm = Pe = Pa
w s dt2 (a)  Bus voltage at the faulted bus
23. Equal area criterion Vk(f ) = Vk (0) − Zkk Ik(f )
(a)  Area of acceleration
(b)  Fault current
dc
Vk (0)
A1 = ∫ (Pm − Pe )dd Ik(f ) =
Zkk
d0
(c)  Fault current for non-zero fault impedance
(b)  Area of deceleration
dm Vk (0)
∫ (Pe − Pm )dd
A2 = Ik(f ) =
Zkk + Zf
dc
(c)  From equal area criterion (d) Bus voltage for the unfaulted buses (after the
event of fault)
d cr dm

∫ (Pm − Pe )dd = ∫ (Pe − Pm )dd 


Vi(f ) = Vi (0) − Zik Ik(f ) ∀i = 1, 2, … n, i ≠ k
d0 d cr
(e)  Voltage at the ith bus
24. Symmetrical component transformation matrix
Zik
éV a 0 ù é1 1 1 ù éV ù Vi(f ) = Vi (0) − V (0)
ê V ú = 1 ê1 Zkk + Zf k
a 2úú ê Vb ú
a

ê a1 ú 3 ê ê ú
a
êëVa 2 úû êë a2 a úû ëê Vc ûú (f) Current flowing between the ith and jth bus
(post fault)
25. Unsymmetrical fault analysis Vi(f ) − V j(f )
I ij(f ) =
(a) Single-line-to-ground fault (LG) Zij
(i)  Current for single line to ground fault:
where Zij is the impedance of line between ith
Vf and jth bus.
I f (a 0) =
Zkk + Zkk + Zkk + 3Zf (g) For synchronous generator, the variation of
0 1 2

AC fault current (rms value) with time (t)


(ii)  Current for line to line fault:
I (t) = (I˝ − I´)e−t / T ˝ + (I´−I ss )e−t / T´ + I ss
Vf
I f (a1) = −I f (a2) =
Zkk1 + Zkk2 + Zf where I˝, I´ and Iss are sub-transient, transient
and steady state current, respectively.

Chapter 5.indd 418 3/23/2016 12:06:35 PM


SOLVED EXAMPLES     419

( )
27. Open conductor faults (ii)  Sequence voltages

( )
(1) ( 0) (2)
(1)
Zkk ′ Zkk ′ + Zkk ′
= Ia(1) Zkk ′ + Zkk ′ = I ij
(a) One phase conductor open ( 0) (2)
Vkk ′ ( 0) (1) (2)
(i)  Current Zkk ′ + Zkk ′ + Zkk ′
(1) (2)
Zkk ′ Zkk ′ + Zkk ′  (2 )
= −Ia(2)Zkk ′ = −I ij (0) kk ′(1)kk ′ (2)
(1) (2) ( 0) (2) Z Z
Ia(1) = I ij   Vkk ′
(0) (1) (1) (2) (2) (0) Zkk ′ + Zkk ′ + Zkk ′
Zkk ′ Zkk ′ + Zkk Zkk ′ + Zkk ′ Zkk ′
(1) (0)
(0 )
= −Ia(0)Zkk ′ = −I ij (0) kk ′(1)kk ′ (2)
( 0) Z Z
(ii)  Bus voltages after fault Vkk ′
Zkk ′ + Zkk ′ + Zkk ′
Vi(1)(f ) = Vi(1)(0) + DVi(1)
28. Economics of power generation
Vi(2)(f ) = DVi(1) (a)  E = Rs. (a + b kW + c kWh),
Vi(0)(f ) = DVi(2)  where, a is the fixed charge for power generation.
b is the semi fixed charge and c is the running or
variable charge for power generation.
(b) Two phase conductors open
(b)  E = Rs. (d kW + e kWh)
(i)  Current
where a fixed sum per kW of maximum
1 demand represented by d and a running charge
Ia(0) = Ia(1) = Ia(2) = I
3 a per kWh of energy represented by e.

SOLVED EXAMPLES

Transmission Lines
11
  1. The line impedance of a three-phase transmission (a) A (b) 11 A
3
line is given by Z = (10 + j5)Ω. For 100 MVA 200
power delivered at 132 kV, the transmission (c) 200 A (d) A
3
loss is
(a) 10 MW (b) 5.74 MW Solution:  For the line, the admittance
(c) 11.5 MW (d) 1
Y ≈ C = 0.05∠80° S/ph
Solution:  For the given transmission line 220
Thus per phase voltage = = 127 kV
3
Ir
Thus charging current is given by
Z = 10 + j5 Ω
Vs Vr = 132 kV V ⋅Y = 127 × 0.05 × 10−3
100 MVA
11
= 6.35 A = A
MVA × 103 3
Ir =
3 kV Ans. (a)

100 × 103 3. For 500 Hz frequency, a 30 km transmission line


= A = 437.4 A will be treated as a long transmission line. For
3 × 132 50 Hz, the same line will be treated as:
Transmission loss = I 2R = (437.4)2 × 10 = 5.74 MW (a) short line (b) medium line
Ans. (b) (c) long line (d) any of the above
2. A 220 kV, 50-km long, three-phase trans-
mission line has A = D = 0.96 × 10−3 ∠3° , Solution:  The series reactance of the transmis-
−3
B = 55 × 10 ∠ 65° Ω/ph, C = 0.05 × 10−3 ∠80° sion line will decrease by 10 times on reducing
S/ph. The charging current per phase will be the frequency by the same order. Also, the shunt

Chapter 5.indd 419 3/23/2016 12:06:51 PM


420     Chapter 5:  POWER SYSTEMS 

admittance also will decrease by 10 times. Hence R = 0.125 × 400 = 50.0 Ω


the line can be treated as a short line. X = 0.4 × 400 = 160.0 Ω

Y = 2.8 × 10−6 × 400∠90° = 1.12 × 10−3 ∠90°W


Ans. (a)
4. A three-phase load of 200 kVA is delivered at
11 kV over a short transmission line of R = 10 Ω, Z = R + jX = (50 + j160) = 168∠72.6°W
X = 0.5 Ω. Find the sending end power if the load
pf is 0.8 lag. YZ = 1.12 × 10−3 ∠90° × 168∠72.6° = 0.188∠162.6°

(a) 161.1 kW (b) 200 kW The source end voltage and current at no-load con-
(c) 100 kW (d) 165.3 kW dition are given by
Vs = AVr; (i)
Solution:  Given that source load = 200 kVA, Is = CVr  (ii)
VL = 11 kV, R = 10 Ω and X = 0.5 Ω The values for constant A can be obtained as:
10 + j 0.3 1 1
A  1 + YZ = 1 + × 0.188∠162.6° = 0.91 + j0.028
2 2
Vs ∠d
Is = Ir
Therefore, A = 0.91  (iii)
Voltage at the receiving end is Substituting value in from Eq. (iii) in Eq. (i), we
get the receiving end voltage as
11 kV
Vr = ∠0° = 6.35 kV 220 220
3 Vr = = = 242 kV
A 0.91
Load current
Similarly parameter C can be computed as
100 × 103
∠ cos 0.8°
 YZ 
IL =
3 × 11 × 103 C = Y 1 + 
 6 
= 10.48 A∠ − 36.87°
 
= 1.12 × 10−3 ∠90° 1 + ∠162.6°
0.188 (iv)
Voltage at the source end  6 
10∠ − 36.87 × (10 + j0.5) = 1.09 × 10−3 ∠90.55° 
Vs = 6.35 + kV
1000
Substituting from Eq. (iv) in Eq. (ii) and value of
= 6.36∠ − 0.05 Vr obtained, we get the sending end current as

I s = C Vr = 1.09 × 10−3 ×
242
Therefore, sending end power × 103 = 152 A
Ps = 3 × 6.46 × 10.48 cos (36.87 — 0.05) 3
= 161.1 kW Note: The receiving end voltage is more than the
Alternately: sending end voltage due to Ferranti effect.
Load end power = 200 × 0.8 = 160 kW and load Ans. (b)
current I = 10.48 A. Then
Ps = 160 kW + (10.48)2 × 10 = 161.1 kW 6. For the transmission line given in Question 5, deter-
Ans. (a) mine the maximum permissible line length if the receiv-
ing end no-load voltage should not exceed 235 kV.
5. A three-phase 50 Hz transmission line is 400  km
long. The voltage at the sending end is 220 kV.
Solution:  The required length l of the transmis-
The line parameters are r = 0.125 ohm/km,
x = 0.4 ohm/km and y = 2.8 × 10−6 ohm/km. When
sion line such that maximum permissible no-load
receiving end voltage is 235 kV, can be determined
there is no-load on the line, then the receiving end
as follows
voltage and sending end current are
Vs 220
(a) 220 V, 128 A (b) 242 V, 152 A A = = = 0.936
(c) 200 V, 116 A (d) None of the above Vr 235

Solution:  Given that the length of trans- Now


mission line is 400 m. Therefore the total line 1
A = 1 + YZ
parameters are: 2

Chapter 5.indd 420 3/23/2016 12:07:09 PM


SOLVED EXAMPLES     421

(a) 6600 V (b) 6396 V


= 1 + l2 × j2.8 × 10−6 × (0.125 + j0.4)
1
(c) 6400 V (d) 684 V
2
= (1 — 0.56 × 10−6l2) + j0.175 × 10−6l 2 Solution:  From the above solution, the sending
Neglecting the imaginary part, |A| can be approxi- end voltage is given by
mated as
|A| = 1 — 0.56 × 10−6l 2 = 0.936 Vs = 6600 + 37.9∠ 39.68(1 + j10)
= 6600 − 211.8 + j314.3
1 − 0.936
= 6395.9∠2.81° V = 6396 V
2
l =
0.56 × 10−6
Ans. (b)
l = 338 km
Ans. (388) 9. The corona loss of a three-phase, 160-km long line
7. A single-phase transmission line having a line at 50 Hz is 1.5 kW/km. The same at frequency of
impedance (1 + j10) is delivering power to a sub- 75 Hz is
station. The connected load to the substation is (a) 360 kW (b) 320 kW
250 kVA at 6.6 kV. If the sending end pf is 0.8 lead, (c) 540 kW (d) 427 kW
load pf is
(a) 0.77 lead (b) 0.77 lag Solution:  The ratio corona loss is given by
(c) 0.8 lead (d) 0.8 lag
PC1 (f + 25)
= 1
Solution:  The line current is given by PC2 (f2 + 25)

250 × 103 Therefore,


IL = = 37.9 A
6.6 × 10 3
160 × 1.5 × 100
PC2 = = 320 kW
From the phasor digram, the sending end voltage 75
is given by Ans. (b)
Is = Ir 10. A 75 Ω transmission line is short-terminated and
Vs
the minima locations are identified. After replacing
jIX the short by a resistive load RL, the minima locations
36.87° are not altered and the measured VSWR is 3. The
Ir
q value of RL is
d
Vr (a) 25 Ω (b) 50 Ω
(c) 225 Ω (d) 250 Ω
Vs = Vr + I∠q ⋅ (1 + j10)
= 6600 + 37.9∠q ⋅ 10∠84.3° Solution:  The reflection coefficient is given by
= 6600 + 379 cos(84.3° + q ) + j379 sin(84.3° + q ) ZL − ZS
G=
Then ZL + ZS
379 sin(84.3° + q )
d = tan−1
Given that ZS = 75, we have
6600 + 379(cos 84.3° + q )
ZL − 75
Now, G=
−d + q = 36.87°
ZL + 75

379 sin(84.3° + q ) 1+ Γ
tan−1 = q − 36.87° VSWR = =3
6600 + 379 cos(84.3° + q ) 1− Γ

or,  q = 39.68° Substituting for Γ, we have


Thus load point pf = 0.77 lead. ZL − 75
Ans. (a) 1+
ZL + 75
=3
8. Find the sending end voltage level for the single- Z − 75
1− L
phase transmission line in Question 7. ZL + 75

Chapter 5.indd 421 3/23/2016 12:07:25 PM


422     Chapter 5:  POWER SYSTEMS 

2ZL Solution:  Maximum stress in each layer


=3
75 × 2
g1 max = g2 max = gmax
ZL = 75 × 3 = 225 W
V1 V2
Ans. (c) =
d d1 d1 D
ln ln
Cables and Electric Line Insulators 2 d 2 d1

11. A single-phase line with two conductors having a In terms of radius


separation of 2 m operates at 50 Hz, the line-to-line V1 V2
capacitance for the system is 10 nF/km. The pos- gmax = =  (i)
r r
sible diameter for the conductor is r log 1 r1 ln 2
r r1

(a) 2 cm (b) 1.5 cm
(c) 2.5 cm (d) 5 cm where r is the radius of the core, r1 of the inter-
sheath and r2 is the overall radius.
Solution:  For the given single-phase line with two For a given cable voltage V and gmax, the minimum
conductors, the capacitance is given by overall diameter r2 is obtained when

2m = D V V V
V1 = , r= and r1 = (ii)
e egmax gmax

2r
Substituting from Eq. (ii) in Eq. (i)
2pe 0
C = ,
ln D/r r2 = r1eV2 /V
where D is the distance between conductors
(= 2 m) and r is the radius of each conductor. Then Give that gmax = 40 kV E = 66 2 kV

2p × 8.84 × 10−9
10 × 109 × 10−3 = Therefore, from Eq. (ii)
2
ln  
r 66 2
40 = r = 0.86 cm
r = 7.73 × 10−3 m ≈ 0.77 cm
2.718 r

Hence, diameter ≈ 1.5 cm r1 = 0.86 × 2.718 = 2.34 cm


Ans. (b)
12. Consider a 66 kV, lead-sheathed insulated cable with r2 = 2.34 ×(2.718)V2 /V
one inter-sheath as shown in the following figure. If
66
the maximum potential gradient of the insulating V2 = V − V1 = 66 − kV(rms) = 41.7 kV
material is 40 kV/cm, then calculate the voltage at 2.718
which the inter sheath must be maintained and the Therefore,
minimum overall diameter of the cable.

Lead sheath r2 = 2.34 ×(2.718)41.7 /66 = 4.39 cm

Inter sheath Thus, the minimum overall diameter of the inter-


Core sheathed cable 8.78 cm.
Ans. (41.7, 8.78)
13. In a three unit insulator string the voltage across
the lowest unit is 17.5 kV while string efficiency is
84.28%. The total voltage across the string will be
r
equal to
r1
(a) 882.5 kV (b) 44.25 kV
r2 (c) 88.25 kV (d) 442.5 kV

Chapter 5.indd 422 3/23/2016 12:07:46 PM


SOLVED EXAMPLES     423

Solution:  The string efficiency is given by 0.2/4 Ω = 0.0.05 Ω at the end. Hence, maximum
Total voltage voltage drop = 750 × 0.05 = 37.5 V.
Ans. (a)
n × voltage across the lowest insulator
17. A three-phase three-wire line 50 km long, supplies a
So, balanced load of 6 MW at pf 0.8 lagging at 33 kV.
Calculate the weight of copper required if the trans-
Total voltage mission efficiency is 80 %. Given, specific resistance
8428 =
3 × 17.5 kV of copper = 1.73 × 10−8 Ω and density = 8900 kg/m3.
Total voltage = 44.25 kV Solution:  Given a three—phase, 3-wire line distri-
Ans. (b) bution system, with P = 6 × 103 kW, V = 33 kV
(line to line) and cos f = 0.8 .
14. The insulation resistance of a cable of length 10 km
is 1 MΩ. For a length of 100 km of the same cable, Then the current
the insulation resistance will be 6000 × 1000
I= = 131.21 A
(a) 1 MΩ (b) 10 MΩ 3 × 33 × 1000 × 0.8
(c) 0.1 MΩ (d) 0.001 MΩ
Efficiency of transmission
Solution:  Insulation resistance of a cable is Power output
=
r R
R′ =
(Power output + line loss)
ln
2p l r
80 6000
Thus, the resistance is inversely proportional to the =
100 6000 + line losses
length of the cable
6000
R ′1 l 6000 + line losses =
= 2 0.80
R ′2 l1 Line losses = 7500 — 6000 = 1500 kW
l2
× 1 = 0.1 MW
10
⇒ R ′1 = × R ′1 = 1500
l1 100 Line losses per phase = = 500 kW/phase
3
Ans. (c)
I 2R = 500 × 1000

Distribution Systems 500 × 103


R= = 29.04 Ω
(131.21)2
15. A uniform DC radial distribution system of 500 m
length has a resistance of 0.2 Ω/km. If it is loaded Length of conductor L = 50 km
at 3 A/m, its maximum voltage drop will be Conductor cross-sectional area
rL 1.73510−8 × 50 × 103
(a) 100 V (b) 150 V
(c) Zero (d) 300 V A= = = 2.9786 × 10−5 m2
R 29.04
Solution:  The maximum current at the end = Volume of copper required
500 × 3 = 1500 A, resistance = 0.2/2 Ω = 0.1 Ω at
the end. Hence, maximum voltage drop = 1500 × 3 AL = 3 × 2.9786 × 10−5 × 50000 = 4.4679 m3
0.1 = 150 V. Weight of the copper required
Ans. (b) = volume × density
16. If the sytem given in Question 15 is fed at both = 8900 × 4.4679 = 3.976 × 104 kg m
ends with equal voltages, the maximum voltage Ans. (3.976 ×104)
drop will be
(a) 37.5 V (b) 150 V PU (per unit system)
(c) Zero (d) 75 V
18. The pu impedance of a transmission line is 0.1. If
Solution:  When fed from both ends, the maximum the base kV and base MVA are doubled, the new
current at the end = 250 × 3 = 750 A, resistance = pu value of the line is

Chapter 5.indd 423 3/23/2016 12:08:03 PM


424     Chapter 5:  POWER SYSTEMS 

(a) 0.1 (b) 0.05 size of Jacobian matrix in Newton—Rapson method


(c) 0.4 (d) 1.0 will be
Solution:  Given that Z = 0.1 pu (a) 376 × 376 (b) 378 × 378
Let the base MVA = MVA1. So, (c) 380 × 380 (d) 382 × 382

MVA1 × 103 Solution:  Total number of buses (n) = 200 =


I base1 =
3kV1 np + ng + 1. Given that ng = 20, ther number of
load buses np = 179.
Let base kV = kV1. So Size of Jacobian matrix is given by
kV1 × 3 kV1 (kV1 )2
Zbase1 = = V3 (n + np − 1) × (n + np − 1)
(MVA1 ) MVA1
= (200 + 179 − 1) × (200 + 179 − 1) = (378 × 378)
So Z = Zpu × Zbase1 Ans. (b)

The new base values on doubling are: base MVA = 21. An alternator with Xd = 1.2 pu is connected to
MVA2 = 2 MVA1 and base kV = kV2 = 2 kV1, infinite bus as shown the figure. If its no load emf
then new impedance is is 1.0 pu and it delivers 1.0 pu current at 0.8 pf lag,
its steady state power limit will be:
( 3 kVA1 ) = 2 Z
2
(2 kV1 )2
Zbase2 = V3 =2 base1
2 MVA1 MVA1
V = 1 ∠0°
Thus new pu value is

Z 0.1 × Zbase1 (a) 0.833 (b) 0.80


= = 0.05 pu
Zbase2 2Zbase1 (c) Zero (d) 1.25
Ans. (b)
19. The pu impedance of a generator is j0.2 on a base Solution:  The steady state power limit is
value of 11 kV, 50 MVA. The same with base EV 1
= = 0.833 pu
values of 22 kV and 100 MVA will be Xd 1.2
(a) j0.8 (b) j0.1 Ans. (a)
(c) j0.2 (d) j0.4
Voltage Control
Solution:  Given that Z = j0.2 pu, Vbase1 = 11 kV,
MVAbase1 = 50 and Vbase2 = 22 kV, MVAbase2 = 22. For a single-phase transformer, the equivalent imped-
ance referred to primary is 10 Ω and the same
referred to secondary is 2.5 Ω. If its primary voltage is
100. Then

Zbase2  KVA2 
increased by 10%, the % increase of secondary will be
MVA1  22 2 50
2
=   × =   ×
MVA2  11  100
=2
Zbase1  KVA1  (a) 10% (b) 2.5%
(c) Zero (d) 5%

Hence new Zbase2 = 2Zbase1 Solution:  For the given transformer

So, new impedance will be


N1 N2
=
j0.2
= j0.1 pu Zeq(p) = 10 Ω Zeq(s) = 2.5 Ω
2
Ans. (b)  N 2
Z1 + Z2  1  = 10
 N 
2
Bus Admittance Matrix
 N 2
20. A system has 200 buses of which 20 buses are gen- Z2 + Z1  2  = 2.5
erator bus and the rest of them are load bus. The  N 
1

Chapter 5.indd 424 3/23/2016 12:08:19 PM


SOLVED EXAMPLES     425

Assuming Z1 ≈ 0 i0 cos f
415√2
Q
 N 2
Z2  1  = 10, Z2 = 2.5
 N 
2 i0
i0 sin f
N1
or =2
N2
i0 cos f =
415 2
= 0.347 A
RC
N2
So, V2 = × V1
N1
i0 sin f =
415 2
= 3.93 A
1 149
= × 1.1 V = 0.55 V
2
Q = tan−1
4.19
= 85°
0.55 − 0.5 347
% increase = × 100% = 10%
0.5
Ans. (a) i0 = 3.95 sin(157 t − 85°)
23. A voltage of 415 2 sin 314 t is applied to a single-
Ans. (b)
phase transformer operating at no load. If the no
load current is 2.0 sin (314 t − 80°), the approxi- 25. A 240 V single-phase AC source is connected to a
mate magnetisation reactance can be load with an impedance of 10∠60° W. A capacitor
is connected in parallel with the load. If the capaci-
(a) j298 Ω (b) j250 Ω
tor supplies 1250 VAR, the reactive power supplied
(c) j300 Ω (d) j290 Ω
by the source is
Solution:  The components of no load current in I IC
single-phase transformer are
IL

415√2 V 240 V Z = 10∠60° 1250 VAR


80°

2A (a) 3600 W (b) 2880 W


(c) 2400 W (d) 1200 W
Here, i0cos f = 0.347A and i0sin f = 1.97 A
Therefore, Solution:  From the circuit, the current through
the load will be
= 298 W
45 2
Xm = I L = I + IC
1.97
Ans. (a) 240 1250
= +j
24. If the voltage for the transformer in Question 23 is 10∠60 240
changed to 415 2 sin 157 t, then the no load cur- = 24∠ − 60° + 5.20 j
rent assuming constant core line will be = 12 + 15.60 j
(a) 2.0 sin(157t — 80°) (b) 3.95 sin(157t — 85°)
(c) 5 sin(157t) (d) 4 sin(157t — 80°) The power supplied by the source
Ps + jQs = Vs I s*
Solution:  For the given transformer:
= 240 (12 + 15.60 j)
= 2880 + 3744 j
= 1630 Ω
400 2
RC =
0.347
Real power, Q = 2880 W
Assuming RC remains same, new Xm at w = 157 is Ans. (b)

= 140 W then
280 26. Two generators of 200 MW and 400 MW rating are
Xm =
2 operating in parallel. Both the governors have a

Chapter 5.indd 425 3/23/2016 12:08:42 PM


426     Chapter 5:  POWER SYSTEMS 

droop of 4%. When the total load is 300 MW, each (a) 48.50 Hz (b) 47.69 Hz
individual generator will be, (c) 47.5 Hz (d) 49.00 Hz
(a) P1 = 100 MW and P2 = 200 MW
Solution:
(b) P1 = 150 MW and P2 = 150 MW
(c) P1 = 200 MW and P2 = 100 MW 50 Hz
(d) P1 = 120 MW and P2 = 180 MW

Solution:  The output of the two generators can 48 Hz


47.5 Hz
be depicted as
50 Hz 600 MW
f1 200 MW 400 MW
48 Hz
For Generator 1:

300 MW f − 50 P −0
=
50 − 48 0 − 200
P
200 MW P1 P2 400 MW or P = (−100)(f − 50) = 5000 − 100f
Here, 4% drop of 50 Hz is 2 and hence the resultant For Generator 2:
f − 50 P −0
frequency is 48 Hz.
For a 300 MW load: = or P = 8000 − 160f
P1 + P2 = 300 50 − 47.5 0 − 400
Let the common frequency be f1. Let common frequency be f1
Then, P1 = 5000 — 100 f1 and P2 = 8000 — 160f1
For Generator 1:
Thus,
f − 50 P −0 5000 — 100 f1 + 8000 — 160f1 = 600
=
50 − 48 0 − 200 260 f1 = 12400
200 f1 = 47.69 Hz
P = (50 − f ) = 100(50 − f ) Ans. (b)
2
For Generator 2: Power System Stability
f − 50 P −0
= 28. If a generator of 250 MVA rating has an inertia
50 − 48 0 − 400 constant of 6 MJ/MVA, its inertia constant on
P = 200(50 − f ) 100 MVA base is

At frequency f1 (a) 15 MJ/MVA (b) 150 MJ/MVA


(c) 6 MJ/MVA (d) 41.4 MJ/MVA
P1 = 100(50 − f1 ) and P2 = 200(50 − f1 )
Solution:  Given that at 250 MVA, inertia con-
Thus stant H1 = 6 MJ/MVA. Then on 100 MVA base,
inertia constant is
100(50 − f1 ) + 200(50 − f1 ) = 300
300f1 = 5000 + 10000 − 300  250 
H2 = 
 100 
6 MJ/MVA = 15 MJ/MVA
15000 − 300
f1 = = 49 Hz Ans. (a)
300
So P1 = 100 MW and P2 = 200 MW 29. A power station consists of two synchronous
Ans. (a) ­generators A and B of ratings 250 MVA and
500 MVA with inertia 1.6 pu and 1 pu, respectively
27. Two generators rated 200 MW and 400 MW on their own base MVA ratings. The equivalent
having governor droop characteristics of 4% and pu. inertia constant for the system on 100 MVA
5%, respectively, are operating in parallel. If the common base is
generators operate on no load at 50 Hz, the fre-
quency at which they would operate with a total (a) 2.6 (b) 0.615
load of 600 MW is (c) 1.625 (d) 9.0

Chapter 5.indd 426 3/23/2016 12:08:55 PM


SOLVED EXAMPLES     427

Solution:  Given that inertia constants H1 = 1.6 pu Here d0 = ∠33.9° is the initial rotor angle.
and H2 = 1 pu. When there is no fault in the system, maximum
power is given by
Then
H1G1 H2G2 1.6 × 250 1 × 500 V E′ 1 × 1.075
Heq = + = + =9 Pmax = = = 1.79 pu
GB GB 100 100 X12 0.6
Ans. (d)
30. The generator shown in the following figure deliv- Therefore, in terms of power angle
ers 1.0 pu power to infinite bus and the generator Pe = 1.79 sind
terminal voltage is 1 pu. Determine the maximum
power that can be transferred when the system has When one line is open, from the reactance diagram,
(i) no fault and (ii) one line is open. we have

X12 = 0.25 + 0.1 + 0.5 = 0.85


j0.5
j0.1 1 ∠0° Therefore, maximum power is
Vt
∞ 1 × 1.075
Pmax = = 1.265
X′d = 0.25 0.85
j0.5
Pe = 1.265 sin d
Solution:  The simple reactance diagram for the
Ans. (1.265)
power system can be represented as
31. If the generator given in Question 30 has an integer
3 2 constant of 4 MJ/MVA, what is the initial angular
j0.5
1 acceleration? If this acceleration remains constant
j0.25 j0.1 for ∆t = 0.05 s, find the new acceleration at the
end this time interval.
+ Vt +
|E| ∠d j0.5 1 ∠0°
Solution:  When the fault occurs, the Swing equa-
tion can be written as,
Let the terminal voltage (Vt) be given by
H d 2d
= Pm − Pe
Vt = Vt ∠a 180f dt2

Then from power angle equation, we have d 2d


= 1 − 0.694 sin d
4
Vt V 180 × 50 dt2
sin a = Pe
X d 2d
 1 × 1  = 2250(1 − 0.694 sin d )
 sin a = 1 ⇒ a = 20.5° dt2
 0.25 + 0.1

The current in the infinite bus is given by Using initial rotor angle d 0 = 33.9° from the
above solution, we have
Vt ∠a − V ∠0° d 2d
I=
jX = 2250(1 − 0.694 sin 33.9°)
dt2
1∠20.5° − 1∠0°
t = 0+
=
j0.35 = 1379 elect deg/s2
= 1 + j0.18 = 1.016∠10.3° Ans. (1379)

The generator emf leading to transient reactance is Power Factor Correction


E ′ = 1∠0° + j0.6(1 + j0.18)
32. A three-phase, 11 kV, 50 Hz and 200 kW load is
= 0.892 + j0.6 = 1.075∠33.9° operating at 0.8 pf lag. If the pf is to be improved

Chapter 5.indd 427 3/23/2016 12:09:16 PM


428     Chapter 5:  POWER SYSTEMS 

to unity, using star connected capacitor, the per- Vs


phase value of the reactive VAR required is:
(a) 100 kVAR (b) 200 kVAR
(c) 50 kVAR (d) 10 kVAR d = 3.56°
Vr
Ir = Is
Solution:  The given three-phase load with star
The current at the sending and receiving ends is
connected capacitor.
100 × 103
Is = Ir = = 144 A
Three phase 3 × 400
Load The sending end voltage is
11 kV
400 144
Vs = + ∠0°×(1 + j100)
0.8 pf lag 3 1000
200 kW

C ≈ 231 + j144 = 231.4∠3.56° kV

Sending end pf = cos 3.56° = 0.998


Ans. (d)

250 kVA 34. A 415 V, three-phase, 50 Hz source is feeding a


Q = 150 kVAR load of 450 kVA at 0.6 pf lagging. Find the kVAR
cos−1 0.8 to of the capacitor bank required to improve the pf
to 0.95 lag.
P = 200 kW (a) 271 (b) 500
(c) Zero (d) 450
kW 200
Load kVA = = 250 kVA
cos f 0.8 Solution:  The power transmitted (kW of load) =
450 × 0.6 = 270 kW.
The reactive power without compensator consumed
Load kVAR = 2502 − 2002 = 150 kVAR by the line

The per phase capacitance VAR required for unity Q= (4502 − 2702 ) = 360 kVAR
power factor (upf)
For new pf of 0.95, the load kVA = 270/0.95
150 = 284.2 kVA
= kVAR = 50 kVAR
3 Thus, the reactive power with compensator is
Ans. (c) Qc = (284.2)2 − (270)2 = 88.7 kVAR
33. The sending end pf of a three-phase transmission The capacitor bank kVAr = 360 — 88.7 = 271.3 kVAR
line having impedance of (1 + j100) Ω/ph supply- Ans. (a)
ing a load of 100 MW at unity power factor (upf)
at 400 kV is: 35. In Question 34, if the capacitor bank has a loss of
50 kW, find the kVA supplied by the source.
(a) unity (b) 0.5
(c) 0 (d) 0.998 (a) Zero (b) 457
(c) 337 (d) 357
Solution:  For the given transmission line
Solution:  From the solution above, we have
PL = 450 × 0.6 = 270 kW and the reactive power
Is ZT = 1 + j100 without compensator Q = 360 kVAR.
Ir 400 kV, upf For new pf of 0.95, the total load kW = 270 + 50 =
Vs Vr
100 MW 320 kW, the load kVA = 320/0.95 = 336.8 kVA.
Ans. (c)

Chapter 5.indd 428 3/23/2016 12:09:30 PM


SOLVED EXAMPLES     429

36. A synchronous motor loaded at 200 kW is con- 10∠30°


nected to a load of 500 kW to improve pf to 0.95. a
The kVAr supplied by the motor is
R R
(a) 230
15∠−60° R
(b) 584 b
(c) Zero 15∠−220°
(d) 769 c

Solution:  Given that motor input = 200 kW; orig- (a) 13.95∠ − 42° (b) 12.69∠157°
inal pf(cos f1) = 0.65 and final pf(cos f2) = 0.95.
The input kVA (c) 38.07∠ −0° (d) 41.85∠42°

kW 700 Solution:  The sequence symmetrical component


= = ≈ 737 kVA
cos f1 0.95 current of line c is given by

Thus 1
ic1 = (i + aib + a2 ia )
KVAr = 7372 − 7002 = 230.27 3 c

Thus KVAr supplied by motor = 230.07 Substituting the values, we get


ic1 = 12.69 < 157°
Ans. (a)
37. An alternator is supplying a power of 0.8 pu to an Ans. (b)
infinite bus. The total reactance of the alternator,
transformer and the line is 0.8 pu. If Eg = 1.1 pu
and Eb = 1.0 pu, determine the operating angle of Faults
the alternator. What is the maximum power that
39. The maximum possible fault level on LV. (low
can be transferred? If, a series capacitor is added to
voltage) side of the 500 kVA transformer with 5%
bring down the total reactance to 50% of original
reactance with zero equivalent reactance of source
value, find the new operating angle.
will be:
Eg Eb sin d (a) 10 MVA (b) 13 MVA
Solution:  P =
X (c) 14 MVA (d) 15 MVA

1.1 × 1.0
0.8 = sin d Solution:  For 5% impedance, the current at short
0.8 circuit is

 0.8 

d = sin-1 1.375  = 35.58° 1
=
1
= 20 pu (∵ X = 0.05 pu)
X 0.05
1.1 × 1.0
Maximum power =
0.8
= 1.375 pu Hence, fault level = 500 × 20 = 10 MVA.
With compensation, Ans. (a)
40. Three  1 1000  V ,  1 00  M VA  t hree-phase  a lterna-
1.1 × 1.0
sin d ⇒ d = 16.91°
tors are operating in parallel. Each has positive
0.8 = sequence reactance of 15%, and the negative and
0.4
Ans. (16.91) zero sequence currents are 75% and 50% of posi-
tive sequence value. An earth fault occurs on
one bus bar. Find the magnitude of fault cur-
Symmetrical Components rent, if all the alternators neutrals are solidly
earthed.
38. A D-connected balanced resistive load is connected
across unbalanced three-phase supply. Find the (a) 40,600 A (b) 18,750 A
positive sequence symmetrical component current (c) 46,654 A (d) 18,551 A
of line c.

Chapter 5.indd 429 3/23/2016 12:09:46 PM


430     Chapter 5:  POWER SYSTEMS 

Solution:  The positive, negative and zero sequence Ppu = Vpu I pu cos q = 0.8
impedances are given as, X1 = .15 pu; X2 = .15 ×
0.75 = 0.1125 pu and X0 = 0.5 × 0.15 = 0.075 pu
0.8
I pu = = 0.8∠0° pu
The fault impedance is given by, 1×1

The induced emf behind subtransient reactance is


(0.15 + 0.1125 + 0.075) = 0.1125 pu
1
Zf =
3
E ″ = V (0) + I × jXd
Hence the fault current
= 1∠0 + 0.8∠0 × j0.2
MVA ´ 103
If =
= 1 + j0.16 = 1.013∠9.1°
3 KV
Zf
The circuit after a symmetrical or balanced fault
100 × 103/1.732 × 11
= = 46.66 kA can be represented as
0.1125
I ″sc
Ans. (c)
41. A 50 Hz alternator is rated 500 MVA, 20 kV, with + jX″d
Xd = 1.0 per unit and X˝d = 0.2 per unit. It sup- E″ Short circuit
plies a purely resistive load of 400 MW at 20 kV. −
The load is connected directly across the generator
terminals when a symmetrical fault occurs at the
load terminals. The initial rms current in the gen- Then the initial or subtransient fault current is
erator is per unit is
E″ 1.013∠9.1°
(a) 7.22 (b) 6.4 ″ =
I sc = = 6.065 − ∠80.9° pu
(c) 3.22 (d) 5.1 jX ″a j 0. 2

″ = 5.065 pu
Therefore, I 90
Solution:  Consider the following circuit for the
given generator. Ans. (d)

jX ″d I0 42. For the power system given in the following


figure, the pu reactance for various components
are marked. Assuming that the prefault voltage is
+
Load 1 pu, calculate the fault current If and the voltage
at the first and second bus V1(f) and V2(f) for a
− solid three-phase fault on bus 3.

Here Xd = 1 pu, and X d″ = 0.2 pu. 0.05 Bus 1


0.2 0.1 Bus 2 0.2
G1 G2
Let (MVA)B = 500 and (KV)B = 20. Then under
prefault condition, the generator is supplying resis- Y ∆Y Y∆ Y
tive load of 400 MW at 20 KV.
0.1 0.1
The prefault terminal voltage is
V0 = 20 KV = 1 pu
Load power
Bus 3
400
P = 400 MW = = 0.8 pu
(MVA)B
Solution:  The Thevenin equivalent network for
The load is resistive, so, pf = 1, q = 0°. Therefore, this system is as follows

Chapter 5.indd 430 3/23/2016 12:10:02 PM


SOLVED EXAMPLES     431

1 0.1 2 44. When a 50 MVA, 11 kV, three-phase generator is


subjected to a three-phase fault, the fault current
is —j5 pu (per unit).
0.05 0.1 3 0.1 0.05
When it is subjected to a line-to-line fault, the
positive sequence current is —j4 pu. The respec-
tive positive and negative sequence reactances
Consider the reference bus k, then the fault current can be,
at the kth bus is given by (a) j0.2 and j0.05 pu (b) j0.2 and j0.025 pu
Vk (0) (c) j0.025 and j0.2 pu (d) j0.05 and j0.2 pu
If =
Zkk + Zf
Solution:  Given a three-phase generator with
V (0) 1 50  MVA, 11 KV, subjected to three-phase fault,
For bus 3, I f = 3 = = −j5.71 then If = −j5 pu and for line to line (LL) fault,
Z33 j0.175
if = −j4 pu.

The voltage at the ith bus is given by For LLL fault

E
I i(f) = Vi (0) =
Zik
V (0) I a1 = = −j 5
Zkk + Zf k Z1

Then For LL fault


 Z 
V1(f ) = 1 − 13  = 1 −
0.125
E
= −j 4
= 0.286 V
 Z33   0.175 I a1 =
Z1 + Z2

and
 Z  1 
= 0.2 pu Z2 =  − 0.2 = 0.05 pu
1
V2(f ) = 1 − 23  = 0.286 or, Z1 =
−j 5  4 
 Z33 

Note: The two voltages are equal because of the Ans. (a)
symmetry of the given power network.
Protection
43. Consider a transmission system
45. The following data are given for a generator:
5 MVA, 6.6 kV, three-phase, star connected, 15%
X˝and X = 0.5 Ω per phase. The initial short cir-
cuit current will be
∆ Y
(a) 1575 A (b) 1500 A
(c) 2100 A (d) 1400 A
For the system with generator transformer used for
step-up, a LG fault on the secondary side of the
Solution:  The base impedance
transformer is equivalent to
(a) A LG fault on generator side of transformer V2 (6.6)2
(b) A LLG fault on generator side of transformer = = = 8. 7
MVA 5
(c) A LL fault on generator side of transformer
(d) A three-phase fault X = 0.5 Ω = 5.75% at given base.
Solution:  The current on one phase in the star Total impedance = 0.15 + 0.0575 = 0.2075 pu.
side will flow extra current between two lines in the The short circuit current
delta side. LG fault on the secondary side of the 5000
= = 2109 A
transformer is equivalent to LL fault on generator (1.732 × 6.6 × 0.2075)
side of transformer.
Ans. (c) Ans. (c)

Chapter 5.indd 431 3/23/2016 12:10:17 PM


432     Chapter 5:  POWER SYSTEMS 

46. The given figure shows the circuit for a percentage 5 1


differential relay to protect one phase of a genera- ×7 − × 0.1 × 17.5 = 0.3325 A
100 100
tor. A high resistance fault occurs near the neutral
end with the current distribution as shown in the Since this is more than the pickup value of 0.15 A,
figure. Given that the ratio of the number of turns the relay will operate.
in the restraining (Nr) and coils operating (No) is Ans. (b)
0.1 and a pickup of 0.15 A. The CT ratio is 500/5.
Identify the condition under which the relay will Circuit Breakers
operate.
47. Four identical 100 MVA, 33 kV generators are open-
CB I2 = 400+j 75 I1 = 435+j 75 ing in parallel, as shown in the following figure, in
two bus-bar sections, interconnected through a cur-
rent limiting reactor of X pu reactance on the gener-
ator-base. Each generator has a reactance of 0.2 pu.
100 MVA 33 kV
Xd′ = 20%
33 kV
33 kV

(a) Under fault condition shown CB


(b) No-load condition Xpu Load
(c) Both of the above
(d) None of the above F(a-b-c)
Load
Solution:  For the relay to operate,
The value of the reactor X to limit a symmetri-
cal short-circuit (a-b-c) current through the circuit
N r I1 + I2
I1 − I2 > breaker to 1500 MVA is
No 2 (a) 0.05 pu (b) 0.10 pu
(c) 0.15 pu (d) 0.20 pu
where I1 and I2 are currents at the two ends of the
protected element.
Solution:  For the given power system
Under the fault condition given in the figure
|I1 — I2| = |(435 + j 75) — (400 + j 75)| = 35 A

I1 + I2 1
= (435 + j75) − (400 + j75) = 424 A
2 2
j.2 j.2 j.2 j.2
Since 35 < 0.1 × 422 = 42.4, the relay will not operate.
If the power system operates under no-load condi- jX
tions I2 = 0 and I1 = 35 A
So Total reactance = (j1)  j(0.1 + X)
I1 — I2  = 35 A For 1500 MVA fault level, total reactance
1 j.1 + (0.1 + X )
I1 + I2 X= = = 0.05 pu
= 17.5 A 15 j0.2 + jX
2
Ans. (a)
The relay actually operates when 48. Consider the following figure depicting relay co-
ordination for the distance relays R1 and R on
N  I + I
k I1 − I2 − k  r  1 2
> Ip adjacent lines of a transmission system. The
 N o  2 zone  1 and zone 2 settings for both the relays
are also shown. Which of the following indicates
where Ip is the pick-up current and k is the current the correct time setting for the zone 2 of relays
transformation ratio of the two CTs. Therefore R1 and R2?

Chapter 5.indd 432 3/23/2016 12:10:27 PM


SOLVED EXAMPLES     433

Zone 2(R2) Zone 2(R2) 220 =


594.2
[cos 15° − cos d ]
2 × 0.286
⇒ cos d = 0.7541 ⇒ d = 41.05
R1 R2
Therefore angle of overlap is
Zone 1(R1) Zone 2(R1)
m = (d − a ) = 41.05° − 15° = 26.05°
(a) TZ2(R ) = 0.6s, TZ2(R ) = 0.3s The average direct voltage is
1 2
(b) TZ2(R ) = 0.3s, TZ2(R ) = 0.6s V
Vd = d 0 (cos a + cos d )
1 2
(c) TZ2(R ) = 0.3s, TZ2(R ) = 0.3s
1 2 2
(d) TZ2(R ) = 0.1s, TZ2(R ) = 0.3s
1 2
594.2
Solution:  Consider that the fault occurs at any VD = [cos 15° + cos 41.05°] = 511.02 V
2
point P. The relay R2 should be closed first then
Ans. (26.05, 511.02)
relay R1 because relay R1 provides back up protec-
tion for zone 2 while relay R2 provides primary
protection. So for zone 2, R2 time should be less Economic Power Generation
than R1 for proper relay co-ordination.
Ans. (a) 50. The power generated by two plants are P1 = 50 MW
and P2 = 40 MW. If the loss coefficients are
B11  = 0.001, B22 = 0.0025 and B12 = −0.0005,
High Voltage Direct Current Transmission
then the calculated power loss will be
49. Consider that for a three-phase bridge rectifier, the (a) 5.5 MW (b) 6.5 MW
transformer secondary leakage reactance is 0.3  Ω (c) 4.5 MW (d) 7.5 MW
and the line voltage is 440 V if the output current
Solution:  Given that P1 = 50 MW, P2 = 40 MW
is 220 A. Find the angle of overlap and DC output
and B11 = 0.001, B22 = 0.0025, B12 = −0.0005
voltage at a delay angle of 15°?
Power loss is given by
Solution:  Given that XC = 0.3 Ω, VLL = 440 V,
Id = 220 A, delay angle a = 15°. PL = B11P12 + B12P1P2 + B22P22
Also,
= 0.001 × 502 + 402 × 0.0025 — 0.0005 × 50 × 40
3 2 3 2
× 440 = 594.2 V
= 5.5 MW
Vd0 = VLL =
p p Ans. (a)

Equivalent commutation resistance per phase is 51. Fuel costs of the two plants of P1 and P2 MW are
given by,
3 × 0.3
= 0.286 Ω
3 C1 = 100 + 2.5P1 + 0.00 4P12
RC = X =
p C p
C2 = 250 + 1.8P1 + 0.008P22
The average direct current is If the plant dispatches a load of 450 MW, the eco-
nomic load scheduling will be
3Em
Id = (cos a − cos d ) (a) P1 = 271 MW, P2 = 179 MW
2wLC (b) P1 = 225 MW, P2 = 225 MW
(c) P1 = 179 MW, P2 = 271 MW
(d) P1 = 200 MW, P2 = 250 MW
3 3Em pRc
But Vd 0 = and wLC = Solution:  We know that
p 3
dC1 dC2
= 2.5 + 0.004 × 2P1 and = 1.8 + 2 × 0.08P2
where Em is the maximum phase to neutral voltage. dP1 dP2
Therefore ,
V For economic load scheduling,
I d = d 0 (cos a − cos d )
2RC dC1 dC2
=
Substituting value, we have dP1 dP2

Chapter 5.indd 433 3/23/2016 12:10:44 PM


434     Chapter 5:  POWER SYSTEMS 

or, 2.5 + 0.008P1 = 1.8 + 0.16 Thus, for P1 + P2 = 450,


0.016 P2 — 0.008 P1 = 0.7 P2 = 179 MW, P1 = 271 MW
2P2 — P1 = 87.5 Ans. (c)

PRACTICE EXERCISES

SET I (One Mark Questions) 7. A lossless transmission line of with characteristic


impedance of 300 Ω and length l/2 is shortened at
1. The load carrying capability of a long AC trans- one end and terminated in its characteristic imped-
mission line is ance at the other. The input impedance measured
at the mid-section of the line is
(a) always limited by the conductor size.
(b) limited by stability consideration. (a) 0 Ω (b) 200 Ω
(c) reduced at low ambient temperatures. (c) 300 Ω (d) 150 Ω
(d) decreased by the use of bundled conductors of
single conductors. 8. The reciprocity condition for a two-port transmis-
sion network can be expressed by
2. A single-phase transmission line of j0.8 Ω is sup-
plying a load of 40 A at 200 V and unity power A B A D
(a) =0 (b) =1
factor. The sending end power factor will be C D B C

(a) unity (b) 0.987
A D A B
(c) zero (d) 0.982 (c) = 1 (d) =1
C B C D
3. Two incoming lines with fault levels at their ter-
minals equal to 100 MVA and 150 MVA terminate 9. Consider the following statements regarding the
on a common bus in a substation. A 1 MVA step suitable choice of HVDC converter configuration:
down transformer having 10% reactance is con- 1. Pulse number should be high.
nected to this bus. The fault level on LV side of 2. Ratio of peak inverse voltage to no load DC
the transformer will be: output voltage should be as high as possible.
(a) 12.57 (b) 9.57 3. Transformer utilisation factor should be near
(c) 11.57 (d) 18.57 unity.

4. The use of high-speed circuit-breakers Which of the statements given above are correct?
(a) 1 and 2 (b) 1, 2 and 3
(a) reduces the short circuit current. (c) 2 and 3 (d) 1 and 3
(b) improves system stability.
(c) decreases system stability. 10. A transmission line with characteristic imped-
(d) increases the short circuit current. ance of 50 Ω is terminated by a load impedance of
(15 — j20) Ω. What is the normalised load impedance?
5. When a bundled conductor is used in place of a
single conductor, the changes in line parameters (a) 0.6 — j0.8 (b) 0.3 — j0.6
are: (c) 0.3 — j0.4 (d) 0.3 + j04

(a) L increases and C decreases 11. Which one of the following statements is correct?
(b) L decreases and C increases Corona loss increases with
(c) L decreases and C not affected (a) d ecrease in conductor size and increase in
(d) L and C both unaffected supply frequency.
6. Normally ZBus matrix is a (b) increase in both conductor size and supply
frequency.
(a) null matrix. (c) decrease in both conductor size and supply
(b) sparse matrix. frequency.
(c) full matrix. (d) increase in conductor size and decrease in
(d) unity matrix. supply frequency.

Chapter 5.indd 434 3/23/2016 12:10:49 PM


PRACTICE EXERCISES     435

12. A single-phase AC voltage source has 200 V rms (a) 16 (b) 12


and a system connected consumes an active power (c) 10 (d) 08
of 300 W. What is the reactive power consumed by 19. In a short transmission line the resistance and
the system, if 2.5 A rms current is drawn? inductance are found to be equal and regulation is
(a) 150 VAr (b) 250 VAr zero, then the possible load will
(c) 300 VAr (d) 400 VAr (a) have zero power factor.
13. Consider the following statements regarding the (b) be 0.707 leading.
fault analysis: (c) have unity power factor.
(d) be 0.707 lagging.
1. The neutral grounding impedance Z appears as
3Zn in zero sequence equivalent circuit. 20. One current transformer (CT) is mounted over a
2. For faults on transmission line, three-phase three-phase three-core cable with its after remov-
faults the least severe amongst other faults. ing sheath and armour from the portion covered by
3. The positive and negative sequence net- the CT. The CT secondary would measure
works are not affected by method of neutral (a) the positive sequence current.
grounding. (b) the zero sequence current.
Which of the statements given above are correct? (c) the negative sequence current.
(a) 2 and 3 (b) 1 and 2 (d) three times the zero sequence.
(c) 1 and 3 (d) 1, 2 and 3 21. A three 10 kW induction motor has a pf of 0.85
14. Two buses having fault level of 100 MVA and lag. Determine the size of the capacitor to raise the
50  MVA, respectively, are interconnected by a power factor to 0.97.
line of negligible impedance. The fault level at any 22. An overhead line with surge impedance of 400 Ω is
point of the line will be terminated through a resistance R. A surge travel-
(a) 80 MVA (b) 70 MVA ling over the line will not suffer any reflection at
(c) 150 MVA (d) 50 MVA the junction, if the value of R is

15. In load flow analysis, the load connected at a bus (a) 100 Ω (b) 200 Ω
is represented as (c) 400 Ω (d) 800 Ω
(a) constant current drawn from the bus. 23. At slack bus, which of the following combinations
(b) constant impedance connected at the bus. of variables is specified?
(c) voltage and frequency dependent source at
the bus. (a) V ,d (b) P,Q

(d) constant real and reactive drawn from the bus. (c) P,V (d) Q,V
16. In a DC transmission line 24. The propagation constant (b) of a transmission line is
(a) it is necessary for the sending end and receiving 0.15 × 10−3 + j1.5 × 10−3
end to be operated in synchronism.
(b) the effects of inductive and capactive reac- The wavelength of the travelling wave is
tances are greater than in an AC transmission
line of the same rating. 1.5 × 10−3 2p
(a) (b)
(c) there are no effects due to inductive and capac- 2p 1.5 × 10−3
tive reactances.
(d) power transfer capability is limited by stability 1.5 × 10−3 p
(c) (d)
considerations. p 1.5 × 10−3
17. A 200 kVA 6.6 kV/415 V; Y connected three-phase 25. The resistance of a 1 kW electric heater when ener-
transformer with 10% leakage impedance has a gised by a 230 V single-phase AC is
short circuit current of
(a) 52.9 Ω (b) 230 Ω
(a) 175 A (b) 100 A (c) 1000 Ω (d) 4.2 Ω
(c) 17.5 A (d) 200 A
26. A three-phase load of 0.8 pf lag is supplied from
18. The number of 2 mF, 300 V capacitors needed a balanced three-phase supply of phase sequence
to obtain a capacitance value of 2 mF rated for RYB. With VYB as reference, the current IR will be
1200 V is (Given cos−1(0.8) = 36.76°)

Chapter 5.indd 435 3/23/2016 12:10:55 PM


436     Chapter 5:  POWER SYSTEMS 

(a) in-phase with VYB (a) reactive power limit.


(b) lagVYB by 36.76° (b) short-circuit current limit.
(c) leadVYB by 53.14° (c) steady-state stability limit.
(d) transient stability limit.
(d) lagVYB by 53.14°
27. A signal of 10 V is supplied to a 100 Ω coaxial
35. The basic circuit of circulating current system of
transmission line which is terminated in 200 Ω.
protection is shown in the following figure. To
improve the through-fault stability, a stabilising
The magnitude of the reflected voltage will be,
resistor is connected between the points
(a) 6.66 V (b) 10 V
(c) 3.33 V (d) 1 V Zone
R S
28. A series RLC circuit with R = 1 Ω, L = 1 H and P
C = 1 F has a resonant frequency of w0. If the V1 Relay V2
values of R, L and C are tripled, the new resonant Q
frequency will be
(a) R and P in series
(a) 3w0 (b) 9w0 (b) P and S in series
w0 w0 (c) P and T in series
(c) (d) (d) P and Q in parallel
3 3
29. A transmission line is distortionless when 36. In a three-phase converter used in HVDC transmis-
sion, the three anodes conduct sequentially. Due
R C
(a) GR = LC (b) = to overlap caused by the circuit inductance, two
C L anodes conduct simultaneously during the overlap
(c) R = G (d) RC = LG period. The output voltage waveform during this
period is the
30. If the maximum and minimum voltages on a trans-
mission line are measured as 4 V and 2 V, respec- (a) v
 oltage of the first anode, because the second
tively, for a typical load, VSWR is anode has not completely taken over.
(b) mean of the two anode voltages, as they con-
(a) 0.5 (b) 1.0 duct together.
(c) 3.0 (d) 8.0 (c) v
 oltage of the second anode, because the volt-
31. For a transmission line with losses, the characteris- age of this anode is greater than that of the
tic impedance does not depend on first.
(d) sum of the first and the second anode voltages,
(a) the operating frequency of the line. because both the anodes are conducting.
(b) length of the line
(c) the conductivity of the conductors. 37. The values of A, B, C and D constants for a short
(d) conductivity of the dielectric separating the transmission line are, respectively
conductors.
(a) Z, 0, 1 and 1 (b) 0, 1, 1 and Z
32. When a transformer primary is connected to line (c) 1, Z, 0 and 1 (d) 1, 1, Z and 0
the zero sequence currents can flow from line into
a transformer bank if the windings are in 38. If a short transmission line is delivering to lagging
power factor load, the sending-end power factor
(a) delta/star. would be (notations have their usual meaning)
(b) star/grounded star.
(c) grounded star/delta. Vr cos f + I r sin f Vr cos f + I r
(a) (b)
(d) delta/delta Vs Vs

33. When the conductors are dead-ended or there is
a change in the direction of transmission line, the Vr cos f + I r Vr sin f + I r cos f
(c) (d)
insulators used as of the Vs Vs

(a) pin type. (b) suspension type.
(c) strain type. (d) shackle type. 39. If the positive, negative and zero sequence reac-
tances of an element of a power system are 0.3, 0.3
34. The critical clearing time of a fault in power system and 0.8, respectively, then the element would be a
is related to

Chapter 5.indd 436 3/23/2016 12:11:04 PM


PRACTICE EXERCISES     437

(a) synchronous generator. (c) line to line fault.


(b) synchronous motor. (d) double line to ground fault.
(c) static load.
48. In a 220 kV system, the inductance and capaci-
(d) transmission line.
tance up to the circuit breaker location are
40. A 12-bus power system has three voltage controlled 25  mH and 0.025 mF, respectively. The value
buses. The dimensions of the Jacobian matrix of ­resistor required to be connected across the
will be breaker ­contacts which will give no transient
(a) 21 × 21 (b) 21 × 19
oscillations, is
(c) 19 × 19 (d) 19 × 21 (a) 25 Ω (b) 250 Ω
(c) 500 Ω (d) 1000 Ω
41. A three-phase circuit breaker is rated at 2000 MVA;
33 kV. Its making current will be 49. The operating characteristic of a distance relay in
(a) 35 kA (b) 70 kA the R—X plane is shown in the following figure. It
(c) 89 kA (d) 161 kA represents operating characteristic of a

42. In Merz Price percentage differential protection


of a D−Y transformer, the CT secondaries connec-
X
Non-operating zone
tion in the primary and secondary windings of the
transformer would be in the form
Operating
(a) D − Y (b) Y − D zone
(c) D − D (d) Y − Y R
43. In the case of a HVDC system, there is
(a) charging current but no skin effect. (a) reactance relay.
(b) no charging current but skin effect. (b) directional impedance relay.
(c) neither charging current, nor skin effect. (c) impedance relay.
(d) both charging current and skin effect. (d) Mho relay.
44. Shunt compensation in an EHV line is used to 50. A surge of 100 kV travels along an overhead line
improve towards it junction with a cable. The surge imped-
(a) stability and fault level. ance for the overhead line and cable are 400 Ω
(b) fault level and voltage profile. and 50 Ω, respectively. The magnitude of the surge
(c) voltage profile and stability. transmitted through the cable is
(d) stability, fault level and voltage profile. (a) 11.11 kV (b) 22.22 kV
45. Bundled conductors are used for EHV transmission (c) 12.50 kV (d) 85.89 kV
lines primarily for reducing the
51. The YBus matrix of a 100-bus interconnected
(a) corona loss. system is 90% sparse. Hence, the number of trans-
(b) surge impedance of the line. mission lines in the system must be
(c) voltage drop across the line.
(a) 450 (b) 500
(d) I2R losses.
(c) 900 (d) 1000
46. The good effect of corona on overhead lines is to
52. A power system consists of two areas connected via
(a) increase the line carrying capacity due to a tie line. While entering the data for load flow, the
conducting ionised air envelop around the tie line parameters and its connectivity data were
conductor. left out by mistake. If the load flow programme is
(b) increase the power factor due to corona loss. run with this incomplete data, then the load flow
(c) reduce the radio interference from the conductor. calculations will coincide only if
(d) reduce the steepness of surge fronts.
(a) one slack bus is specified in the first area.
47. If all the sequence voltages at the fault point in a (b) one slack bus is specified in the second area.
power system are equal, then the fault is a (c) one slack bus is specified in either of the two
(a) three-phase fault. areas.
(b) line to ground fault. (d) two slack buses, one in each area, are specified.

Chapter 5.indd 437 3/23/2016 12:11:06 PM


438     Chapter 5:  POWER SYSTEMS 

53. The zero sequence current of a generator for line to 2. A 110 kV, 50-km long transmission line is having
ground fault is j2.4 pu Then the current through the following A, B, C, D parameters, A = D =
the neutral during the fault is 1.2∠5°, B = 40∠70°/ph C = 0.05∠80° S/ph. The
line has a per phase charging current of:
(a) j 2.4 pu (b) j 0.8 pu
(c) j 7.2 pu (d) j 0.24 pu (a) 3.175 kA (b) 5.5 kA
(c) 52.92 kA (d) 1.588 kA
54. In HVDC transmission, there are predominant
3. A single-phase two wire transmission line is con-
(a) voltage harmonics on DC side and current har-
nected to a load at 0.8 pf lag at the receiving end.
monics on AC side of converters.
Assuming the line to be a short transmission line
(b) current harmonics on DC side and voltage har-
and the line impedance to be a series combination
monics on AC side of converters.
of R and L, the sending end power factor can be:
(c) current harmonics only on the DC side of
converters. (a) 0.8 lag
(d) voltage harmonics only on the AC side of (b) less than 0.8 lag
converters. (c) 0.8 lead
(d) less than 0.8 lead
55. If A, B, C and D are generalised circuit constants,
then which one of the following equation is correct? 4. If the line impedance for the transmission line in
Question 3 is (1+j10) Ω and the load is 100 kVA
(a) −AB + CD = −1 (b) AD + CB = 1
(c) AB — CD = −1 (d) −AD + BC = −1
at 6.6 kV, the sending end pf can be
(a) 0.8 lag (b) 0.8 lead
56. D1 is the GMR of each subconductor of a four subcon-
(c) 0.79 lag (d) 0.79 lead
ductor bundle conductor and d is the bundle spacing.
What is the GMR of equivalent single conductor? 5. A line is operating at 50 Hz and the corona loss is
1.5 kW/ph/km. For a supply frequency of 75 Hz,
(a) 1.09 Ds × d 3 the per phase corona loss in the line per km will be
(a) 1.5 (b) 2.5
(b) 1.09 Ds3 × d 3 (c) 2.0 (d) No corona loss

(c) 1.09 4 Ds3 × d 3 6. The generated power by two plants are


P1 = 50 MW, P2 = 70 MW. The loss coefficients
(d) 1.09 4 Ds × d 3 are given by B11 = 0.001, B22 = 0.0025 and
B12 = −0.0004. The power loss in the system will be

57. For a fixed receiving end and sending end voltage (a) 10 MW (b) 12 MW
in a transmission system, what is the locus of the (c) 13.35 MW (d) 14.75 MW
constant power? 7. The following figure shows the single line diagram
(a) A straight line (b) An ellipse of a power system with all reactances’ marked in
(c) A parabola (d) A circle per unit (pu) on the same base.

58. The combined frequency regulation of 1500 MW F


G M
capacity generator operating at a nominal fre- j 0.2 pu
quency of 60 kHz is 0.1 pu on its own base capac- T1 T2
ZG1 = 0.1 pu ZG2 = 0.1 pu
ity. The relation in Hz/MW will be j0.08 pu j0.08 pu

(a) 0.1/1500 (b) 60/1500 Z G1 = Z G2 = j 0.1 pu, ZT1 = ZT2 = j 0.08 pu,
(c) 600/1500 (d) 1500/60 ZTL1 = j 0.2 pu
The system is on-load when a three-phase fault
Set 2 (Two Marks Questions) occurs at F on the high voltage side of the trans-
former T2. The fault current will be
1. A 200-km long three-phase transmission line is
transferring a power of 200 MVA having line losses (a) −j0.08187 pu (b) +j0.08187 pu
of 5 MW. If the receiving end voltage is 110 kV, (c) −j8.1871 pu (d) +j8.1817 pu
the line has a resistance of 8. For a circuit breaker with the arc extinction prin-
(a) 10 Ω/ph (b) 1.5 Ω/ph ciple, resistance switching is employed by placing a
(c) 1 Ω/ph (d) 2.5 Ω/ph resistance r in parallel with the poles of the circuit

Chapter 5.indd 438 3/23/2016 12:11:15 PM


PRACTICE EXERCISES     439

breaker as shown in the following figure. This pro- (a) 6 MW and 4 MW (b) 4 MW and 6 MW
cess introduces damping in the LC circuit. For (c) 5 MW and 5 MW (d) 10 MW and zero
critical damping, the value of resistance r across
14. The surge impedance of a three-phase, 400 kV
transmission line is 400 Ω. The surge impedance
CB should be equal to

r loading (SIL) is
I (a) 400 MW (b) 1000 MW
(c) 1600 MW (d) 800 MW
CB
C
C Fault 15. Two 50 Hz generating units operate in parallel
within the same power plant and have the follow-
ing ratings
C C Unit 1: 500 MVA, 0.85 power factor, 20 kV,
(a) (b) 0.5
L L 3000 rpm, H1 = 5 MJ/MVA
Unit 2: 200 MVA, 0.9 power factor, 20 kV,
L 1 L 1500 rpm, H2 = 2 MJ/MVA
(c) 0.5 (d)
C 2p C The equivalent inertia constant H in MJ/MVA on
9. An overhead line with a surge impedance of 400 Ω
100 MVA base is
is connected to a transformer by a short length of (a) 21.0 (b) 29.0
cable of surge impedance 100 Ω. If a rectangular (c) 25.0 (d) 35.0
surge wave of 40 kV travels along the line towards
16. Three generator of 100 MVA, 11 kV have an imped-
the cable, then the voltage of the wave travelling
ance of 0.15 pu each. If in the same plant, these
from the junction of the overhead, line through the
generators are being replaced by a single equivalent
cable towards the transformer would be
generator its effective impedance will be
(a) 40 kV (b) 24 kV
(c) 32 kV (d) 16 kV (a) 0.05 pu (b) 0.15 pu
(c) 0.35 pu (d) 0.45 pu
10. An alternator with transient reactance equal to 0.1 pu
on a 100 MVA base is connected to the high voltage 17. Two insulator discs of identical capacitance value
bus through a step up transformer of reactance 0.1 pu C make up a string for a 22 kV, 50 Hz, single-phase
on a 100 MVA base. The fault level at the bus is overhead line insulation system. If the pin to earth
capacitance is also C, then the string efficiency is
(a) 1000 MVA (b) 500 MVA
(c) 200 MVA (d) 10 MVA (a) 50% (b) 75%
(c) 90% (d) 86%
11. A single core cable, consisting of 1 cm diameter
cable inside a 2.8 cm diameter sheath, is 20 km 18. Match List I (Classification based on head) with
long and operates at 13 kV and 50 Hz. The relative List II (Type of turbine) and select the correct
permittivity of the dielectric is 5, and the open cir- answer using the codes given below:
cuit power factor of the cable is 0.08. Calculate the List I
capacitance of the cable and is charging current.
(a) Low head, 2−15 m
12. The cost function of a 50 MW generator is given by (b) Medium head, 16−70 m
(c) High head, 71−500 m
F(P1) = 225 + 53 P1 + 0.02 P12
(d) Very high head > 500 m
where P1 is the generator loading in MW.
List II
For 100% loading the cost is
1. Pelton
(a) Rs. 55 per MWh
2. Francis or Pelton
(b) Rs. 55 per MW
3. Kaplan or Francis
(c) Rs. 58.5 per MWh
4. Propeller or Kaplan
(d) Rs. 58.5 per MW
     A B  C D
13. Two alternators each having 4% speed regulation
(a)  4  3  2  1
are working in parallel. Alternator 1 is rated for
12 MW and alternator 2 is rated for 8 MW. When (b)  3  4  1  2
the total load is 10 MW, the loads shared by alter- (c)  4  3  1  2
nators 1 and 2 would be, respectively, (d)  3  4  2  1

Chapter 5.indd 439 3/23/2016 12:11:22 PM


440     Chapter 5:  POWER SYSTEMS 

2p 23. A 50 Hz, 10.00 kV generator is connected to a


j
19. If a = e = e 3 ,
and [I] = [A][IS ]where [I ] is equal power system. The system reactance and capaci-
to phase current vector and [ IS ] is equal to symmetri- tance per phase are 10 mH and 0.02 mF, respec-
cal current vector matrix, then the symmetrical com- tively. What is the maximum voltage across the
ponents transformation matrix [A] can be given as, contacts of the circuit breaker while the breaking
current passes through zero?
1 1 1  1 a a 2 
    (a) 28.28 kV (b) 29.28 kV
(a) 1 a a 2  (b) 1 1 1  (c) 30.28 kV (d) 31.28 kV
1 a 2 a  1 a 2 a 
    24. For a 100 MVA, 11 kV, 50 Hz, 4 pole turbo-­
generator, with 800 MJ stored energy in the rotor
1 1 1   
1 a a 
2
 
at synchronous speed, what is the inertia constant
(c) 1 a a 
2
(d) 1 a a 2  H?
1 a a 2  1 1 a 
   
(a) 2.0 MJ/MVA (b) 4.0 MJ/MVA
(c) 6.0 MJ/MVA (d) 8.0 MJ/MVA
20. A sample power system network is shown in the
following figure. The reactances marked are in pu. 25. The change in pu current and impedance if the
What is the pu value of Y22 of the bus admittance base VA and base voltages are halved will be
matrix (YBus)? (a) 1/2, 1/2 (b) 2, 1
(c) No change (d) 1, 1/2
j0.1 j0.1
j0.2 26. A load of three impedances (6+j9) Ω is supplied
Bus 1 Bus 2 from 400 V, 50 Hz three-phase source with line
impedance of (1+j2) Ω. If the load is delta con-
nected, the load power is
j0.2
(a) 5000 W (b) 9410 W
j0.2 (c) Zero (d) 1500 W

27. A two-port network is represented by the following


Bus 3 equations:

(a) j10.0 (b) j0.4 V1 = 60 I1 + 20 I2


(c) −j0.1 (d) −j20.0 V2 = 20 I1 + 40 I2

21. The direct axis reactance of an alternator Xd is The ABCD parameters of the above network
given as 0.4 pu based on the alternator’s name would be
plate rating of 10 kV, 75 MVA. The base for calcu-
 1  100 3 
lation is 11 kV, 100 MVA. What is the pu value of 2   
(a)  20  (b)  1
Xd on the new base? 3 100 2 
   20 
(a) 0.279 (b) 0.578
(c) 0.44 (d) 0.412 3 100
100 20  
22. A fault occurring at the terminals of an unloaded (c)   (d)  1 
6 3   2 
synchronous generator operating at its rated volt-  20 
age results in the following values of currents and
voltages: 28. A 440 V three phase distribution feeder has a load
of 75 kW drawing a current of 130 A. A capaci-
Ia(0) = j2.30 pu, tor rating 45 kVAr is connected across the load.
Ia(1) = j3.1 pu, Ia(2) = j0.72 pu Determine the (i) pf and reactive load before com-
Va(0) = Va(1) = Va(2) = 0.337 pu pensation and (ii) pf after compensation.
Indicate the correct type from the following faults. 29. For the network shown in figure the zero sequence
(a) LLfault (b) LG fault reactances in pu are indicated. The zero sequence
(c) LLG fault (d) LLL fault driving point reactance of the node 3 is

Chapter 5.indd 440 3/23/2016 12:11:34 PM


ANSWERS TO PRACTICE EXERCISES     441

(a) 1.152 pu (b) 11.52 pu


1 2 3 (c) 0.8 pu (d) 5.52 pu

XL0 = 0.05 31. A shunt reactor of 100 MVAR is operated at 98%


XG0 = 0.1 XG0 = 0.2 of its rated voltage and at 96% of its rated fre-
quency. The reactive power absorbed by the reac-
XL0 = 0.15 tor is

(a) 0.12 (b) 0.30 (a) 98 MVAR (b) 104.02 MVAR


(c) 0.10 (d) 0.20 (c) 96.04 MVAR (d) 100.04 MVAR

30. Consider a system consisting of a generator equiva- 32. The distribution system shown in figure is to be
lent reactance 0.50 pu connected to an infinite bus protected by over current system of protection.
through a series reactance of 1.0 pu. The terminal 1 2 3 4
33 kV 33 kV
voltage of the generator is held at 1.20 pu and the
voltage of the infinite bus is 1.0 pu. Then find the Supply Supply
steady state power limit of the system.
5 Load
Xd = 0.5 X = 1.0 I

+ + For proper fault discrimination directional over


current relays will be required at locations
E ′∠δ Vt = 1.2∠q V = 1.0∠0°
(a) 1 and 4 (b) 2 and 3
(c) 1, 4 and 5 (d) 2, 3 and 5

ANSWERS TO PRACTICE EXERCISES

Set 1 (One Mark Questions) Therefore,


Zin = Z0/2 = 150 ohm
1. (b)
8. (d) The reciprocity condition is AD - BC = 1
200
2. (b) Load impedance = = 5∠0 9. (b)
40
10. (c) Given that:
Total impedance = 5 + j0.8 = 5.06∠9.09° Ω
Zo = 50 ohm and ZL = (15 - j20).
Hence, pf = cos(9.09) = 0.987 Hence the normalised load impedance is
ZL 15 − j20
= 0.3 − j0.4 Ω
3. (b) Less than 10% as the lines will be in parallel.
=
Zo 50
4. (b) The faster operation of the circuit breaker,
removes the fault faster and restores the system to 11. (b) As corona loss is proportional to (f +25) and
normal. proportional to d (where d is the diameter of the
conductor).
5. (b)
12. (d) For the given system
6. (c)
7. (d) Given that Zc = 300 Ω and l = l/2 is shortened 2.5A
at one end.
V
200 V
Zo Zo
Zin VA supplied by the source = 200 × 2.5 = 500 VA

Chapter 5.indd 441 3/23/2016 12:11:45 PM


442     Chapter 5:  POWER SYSTEMS 

Then 22. (c) The reflected voltage is given by


VAR = 500 − 300 = 400 VAR
2 2
400 − R
13. (d) 400 + R

14. (c) With zero line impedance the fault level will be Thus for R = 400 Ω. There will be no reflected
the same at all points and given by sum of indi- voltage.
vidual fault levels. 23. (a) For the slack bus, reactive and constant power
15. (d) Power quantities are specified and so the load need not be considered and only V and d are
connected at bus in load flow analysis is repre- sufficient.
sented as constant real and reactive power drawn 2p 2p
24. (b) Wavelength l = then l =
b 1.5 × 10−3
from the bus.
16. (c)
25. (a) Given that single 1 kW electric heater ener-
17. (a) For the three-phase transformer, the full load gised at 230 V. Thus,
current is
200 V2 2302
If l = = 17.5 A = 52.9 Ω
3 × 6.6
R= =
W 1 × 103
Given that Z = 0.1 pu, so short circuit current is
17.5 26. (c) Given that load with 0.8 pf lag is supplied, there-
I sc = = 175 A fore, cos f = 0.8 or f = 36.76°(given). From the
phasor diagram, IR leads VYB by = 90° − 36.76° =
0.1
18. (a) 2 µF 300 V 53.14°

VR
IR
f
IB 90˚− f
VYB
Four capacitance in series will make voltage ratio f
f
of 1200 V as 1200/300 = 4 . IY VY

Now equivalent capacitance for one series pack


2 µF
VB
C
= =
4 4 27. (c) Reflected voltage is given by
Now the number of parallel packs to make 2  mF = 4.
Thus, number of capacitance needed = 4 × 4 = 16. (ZL − Z0 ) (200 − 100)
= × V = 10 × = 3.333 V
19. (b) For a short transmission line only with R-L (ZL + Z0 ) (200 + 100)
series, impedance is considered. Thus, for regula-
tion zero, the term I < q (R + jX) will have 90° 28. (d) From the RLC circuit, we have
angle when q = 45° as R = X.

wL
20. (b) In this case, the voltage induced in the CT is
R 1
wC
due to the zero sequence current flowing through
CT secondary. So the positive and negative
sequence currents will have zero resultant.
w0L = ⇒ w0 =
1 1
21. When pf is increased, the reactive power supplied w 0C LC
is given by
QC = P(tan f 1 - tan f 2) If the components are tripled, the new resonant
= 10 (0.6197 - 0.2506) = 3.691 kVAr frequency is
3.691
w 0′ =
Rating of capacitor/phase = = 1.23 kVAr
w
1 1 1
3 = =
Ans. (1.23) 3L ⋅ 3C 3 LC 3 0

Chapter 5.indd 442 3/23/2016 12:12:06 PM


ANSWERS TO PRACTICE EXERCISES     443

29. (d) 51. (c) Number of non-sparse elements


(100 − 90)
30. (c) VSWR =
(Vmax + Vmin ) 6
= =3 = 100 × 100 × = 1000
(V max − V min ) 2 100
Number of self-admittance elements = 100
31. (b) Thus number of transmission lines = 1000 − 100
32. (c) This can provide the path for the current in the = 900.
primary. 52. (d) Since the two areas are isolated with missed
33. (c) data, two slack buses, one for each area is required.

34. (c) 53. (a)

35. (c) P and T are in series as this can reduce any 54. (a)
oscillatory current through PQ. 55. (d) As AD − BC = 1 for the transmission lines
36. (b) 56. (d) When Ds is the GMR of conductor, the GMR
37. (c) for four-conductor bundle is given by,

Ds(4-bundle) = 16 (Ds × d × d × 2d )
38. (b) 2

39. (d) For transmission line, the positive and nega-


tive sequence impedances are equal but due to the = 1.09 4 Ds × d 3
grounding reactor the zero sequence reactance can
be higher. 57. (d) In this case the complex power locus is circle
when the sending and receiving voltages are same.
40. (a)
58. (c) The relation between Hz/MW will be
41. (c) Given that the circuit breaker is rated at 2000
MVA, 33 KV, therefore 0.1 × 60 × 103 600
2.55 × 2000 1500
=
1500
Making current = kA = 89 kA
3 × 33
42. (b) Set 2 (Two Marks Questions)
43. (c) 1. (b) The line current is given by
44. (c)
MVAbase 200 × 103
45. (a) IL = = 1049.73 A
3 kV 3 × 110
46. (a)
Thus if per phase resistance is RL, then
47. (b)
48. (c) For no oscillations, critical damping is required 3I L2 RL = Ploss
for which Substituting values, we have
1 L
r=
2 C 3 × (1049.73)2 × RL = 5 × 106
⇒ RL = 1.5 W/ph
−3
25 × 10
W = 500 W
1
r=
2 0.025 × 10−6
2. (a) Charging current can be approximately calcu-
lated as,
49. (d)
Ic = Vr × Y
50. (b) The transmitted surge is given by
Here Y = C.
2Zcable
VT = Vinput × Substituting from give values, we have
Zline + Zcable
2 × 50 110 × 103
= 100 × = 22.22 kV Ic = × 0.05 = 3.175 kA
400 + 50 3

Chapter 5.indd 443 3/23/2016 12:12:21 PM


444     Chapter 5:  POWER SYSTEMS 

The receiving end current 6. (c) The power loss is given by


I r = CVr + DI r B11P12 + B12P1P2 + B22P22
= 0.001 × (50) + 0.004 × 50 × 70 + 0.0025 × (70)
3. (b) Consider the following phasor diagram for the 2 2
given arrangement.
= 13.35 MW
Vs
7. (c) Equivalent fault impedance
ILZs
= (ZG1 + ZT1 + ZTL1 ) (ZG2 + ZT2 )
Vr
qr qs j0.38 × j0.18
= = j0.122
j(0.38 + 0.18)
IZ
It is observed from the figure that cosqr > cosqs. Therefore fault current is
Thus the sending end power factor can be less than
Voltage 1.0
0.8 lag. If = = = −j8.18
Fault impedance j0.122
4. (c) The line current is given by
100 × 103 8. (c) Consider the following equivalent circuit
IL = = 18.94 A
6.6 × 103 × 0.8
L r
So sending end voltage can be determined from the
phasor diagram as
C
Vs

Here the impedance is given by


1.15°
Vr r j wL
Z = jwL + ⇒ wL +
r
36.87°
r + 1 jw L 1 + jw rC
IZ = 18.94 A
Vs = Vr + I∠q(1 + j10)
Taking Laplace transform,

= 6600 + 18.94∠ − 36.87(1 + j10) Z(s) = Ls +


r
=
V (s)(1 + rCs)
1 + rCs r + Ls + rCLs2
= 6600 + 189.9∠47.4 V (s)(1 + rCs)
=
= 6728 + j139.8 = 6730∠ 1.19° V s2 +
Ls
+
r
Therefore, rCL rCL

pf = cos(36.87 + 1.19) = 0.79 lag Hence,

2z ⋅
5. (c) Corona loss is given by 1 1
=
LC rC
−5 (f + 25) r
P = 241 × 10 ⋅ (V − V0 )2
z =
1 L
d d p
2r C
where f is the supply frequency.
for z = 1, r =
1 L
If the line is initially operating at f1 = 50 Hz with
corona loss P1 = 1.5 kW/ph/km, then the corona 2 C
loss (P2) at f2 = 75 Hz is given by 9. (b) The given arrangement of transmission lines
P1 (f + 25) can be represented as
= 1
P2 (f2 + 25) Line Cable
75 + 25
or, P2 = × 15 kW/ph/km 100 Ω
50 + 25 Z0 = 400 Ω
= 2.0 kW/ph/km 40 kV

Chapter 5.indd 444 3/23/2016 12:12:44 PM


ANSWERS TO PRACTICE EXERCISES     445

Then the transmitted voltage is given by f − 50 P −0


For Generator 2: = or P = 200 — 4 f
50 − 48 0−8
400 − 100 3
× 40 kV = × 40 = 24 kV
400 + 100 5 Let common frequency = f1
Then, P1 = 200 — 4 f1 and P2 = 300 — 6 f1
Thus, 200 — 4 f1 + 300 — 6 f1 = 10
10. (b) The circuit can be represented as
or, 10 f1 = 500 — 10 = 490 or f = 49
TX P1 = 200 — 4 × 49 = 4 MW;
P2 = (300 — 6 × 49) = 6 MW
j 0.1 pu j 0.1 pu
Bus 14. (a) Given that

Rated MVA
Fault level = L
= 400 = Z0
Impedance(source or fault) C
100
= = 500 MVA Therefore surge impedance load is
0.2
2
VLL (400)2
2pk0kr −12
2 × p × 8.854 × 10 ×5
SIL = = = 400 MW
11. µ = = Z0 400
ln(R/r) ln(2.8/1)
= 0.270 µF/km  500 
15. (b) Unit 1 =  × 5 MJ/MVA on 100 MVA
Cable capacitance C = 0.27 × 20 = 5.4 mF  100 
Charging current base = 25 MJ/MVA
wCV = 2 × p × 50 × 5.4 × 10−6 × 13 × 103
 200 
Unit 2 =  × 2 MJ/MVA on 100 MVA base
 100 
= 22.05 A
Ans. (5.4, 22.05)
12. (b) Given that: = 4 MJ/MVA

F(P1) = 225 + 53 P1 + 0.02 P12 Thus equivalent inertia constant = 29 MJ/MVA


Therefore,
16. (a) The effective impedance is
dF
= 53 + 0.04 P1 0.15
dP1 = 0.05 pu
3
For P1 = 50 MW (i.e., 100 % performance)
Cost = 53 + 0.04 × 50 = Rs 55 per MW
17. (b) The voltage across capacitor nearest to con-
ductor is V2 = 2V1 (taking shunt capacitance into
13. (b) Given that P1 + P2 = 10 MW account).
Thus string efficiency is
50 Hz
f1 (V1 + 2 V1 ) 3
= = 75%
48 Hz (2 × 2 V1 ) 4
18. (a)
19. (c)
P1 P2
P = 12 MW P = 8 MW 20. (d) For the bus admittance matrix

f − 50 P −0 Y22 = Y22 + Y21 + Y23


For Generator 1: = or
50 − 48 6 − 12 Y22 =
1
+
1
+
1
= −j20.0
j ⋅1 j ⋅ 2 j ⋅ 2
 −12 
P =  (f − 50) = 300 − 6f
 2  21. (c) pu value of Xd on the new base is given by

Chapter 5.indd 445 3/23/2016 12:13:04 PM


446     Chapter 5:  POWER SYSTEMS 

 Base KV  Load power


Xd(new) = Xd(old) ×  old  Base MVAnew
 2×
 Base KVnew  Base MVAold PL = (39.6)2 × 2 × 3 = 9408.86 W
10 2 100
= 0.4 ×   27. (d) Given that:
 11  75
= 0.44 pu

V1 = 60 I1 + 20 I2 (i)


22. (c) For LLG fault, the sequence fault voltages are
equal but the currents are different. V2 = 20 I1 + 40 I2 (ii)

23. (a) The peak voltage is On comparing Eqs. (i) and (ii) with standard equa-
tions, (iii) and (iv)
Vm = 2 × 10 = 14.14 kV
Vs = AVr + BIr (iii)
Therefore the maximum voltage across the con-
tacts is Is = CVr + DIr (iv)
2Vm = 28.28 kV
we have, Vs = V1, Vr = V2 and Is = I1, Ir = I2
24. (d) Inertia constant is given by From Eq. (ii),

V2 − 40 I2
Stored energy 800 MJ I1 =  (v)
H= = = 8 MJ/MVA 20
Machine rating 100 MVA

From Eqs. (i) and (v), we have


I MVA2 kV1 1 MVA1 KVA1
25. (d) base2 = = . =1
2 MVA1 1 KVA
V2 − 40 I2
I base1 MVA1 kV2
2 1
V1 = 60 × + 20 I2 = 3V2 — 100 I2
20
Zbase2 (KVA2 )2 MVA1
= × and rearranging Eq. (ii)
Zbase1 MVA2 (KVA1 )2
1 2 1 40
V2 − I2 =
1
V − 2I2
 KVA1  × MVA1 I1 =
 2  1 20 20 20 2
= =
1 2
MVA1(KVA1 )2 So, option (d) is correct neglecting sign.
2
26. (b) For the given three-phase delta connected load 28. Given that VL = 440 and load current IL - 130 A.
Then
(1 + j2) Ω
(i) Reactive load before compensation

6 + j9 Ω 75 × 103
pf = cos f =
P
= = 0.757
400 V 3 VL IL 3 × 440 × 130
50 Hz
QL = P tan f = 64.73 kVAr

(ii) pf after compensation is

QC = 45 kVAr
(6 + j9)
Equivalent star load per phase =
3Ω Total Q = QL — QC = 19.73 kVAr

Total load per phase = (1 + j2) + 2 + j3 = (3 + j5) W  19.73 


pf = cos  tan−1  = 0.967 lag
Line current  75 

400 3 400 The pf has improved from 0.757 to 0.967.


I line = = × (32 + 52 ) = 39.6 A
3 + j5 3 Ans. (0.757, 64.73, 0.967)

Chapter 5.indd 446 3/23/2016 12:13:28 PM


ANSWERS TO PRACTICE EXERCISES     447

29. (c) The equivalent zero sequence network for the Using the value of q, we can determine terminal
given is voltage, current and generator emf as
Vt = 1.2∠73.87° = 0.332 + j1.152
3
0.332 + j1.152 − 1
XT0 XL0 I= = 1.152 + j0.668
j1
XG0 XG0
E = 0.332 + j1.152 + j0.5 (1.152 + j0.668)  1.728∠90°
Refrence bus
Steady state power limit is given by

XT0 and XL0 are in series. E V 1.728 × 1


Pmax = = = 1.152 pu
X = XT0 + XL0 = 0.15 + 0.05 = 0.20 Xd + X 1. 5

31. (d) Let rated voltage is V kV. Then reactance of


V2
reactor is X = .
X XG0 100

The reactive power absorbed by the reactor at


0.98 V and 0.96 X.
X and X will parallel, then net zero sequence driv-
ing point reactance of the node 3 is
(0.98)2 V 2 (0.98)2 V 2 × 100
XXG0 0.2 × 0.2 Since q = =
XL 0 = = = 0.10 0.96 X 0.96 × V 2
X + XG0 0.2 + 0.2 = 100.04 MVAR
30. (a) Taking the voltage of the infinite bus as refer-
ence, from the circuit, we have 32. (c) In the given distribution system, let the fault
occur in segments A and B.
V = 1.0∠0°, Vt = 1.2∠q
1 A 2 3 B 4
V −V 1.2∠q − 1.0 33 kV
33 kV
I= t =
jX f1

Therefore, generator emf is given by


C

E = Vt + jXd I Load
 1.2∠q − 1.0 
= 1.2∠q + j0.5   Then, for proper fault discrimination the direc-
 j1  tional over current relays should be placed such
= 1.8∠q − 0.5 = (1.8 cos q − 0.5) + j1.8 sin q that for a fault at segment B, the circuit breaker
2 should not trip and for a fault at segment A,
the circuit breaker 3 should not trip. This will
Steady state power limit is reached when E prevent any disruption of the supply to load since
has an angle of 90° or the real part is zero. current flow during fault is in one direction. So,
Therefore, directional over current relays are to be located
1.8cosq − 0.5 = 0 ⇒ q = 73.87° . at 2 and 3.

Chapter 5.indd 447 3/23/2016 4:16:42 PM


448        CHAPTER 5:  POWER SYSTEMS

SOLVED GATE PREVIOUS YEARS’ QUESTIONS

1. Bundled conductors are mainly used in high voltage Solution:  Auxiliary components in HVDC trans-
overhead transmission lines to mission are: DC line inductor and reactive power
sources.
(a) reduce transmission line losses.
(b) increase mechanical strength of the line. Ans. (b)
(c) reduce corona.
(d) reduce sag. 4. A round rotor generator with internal voltage
E1 = 2.0 pu and X = 1.1 pu is connected to a round
(GATE 2003: 1 Mark) rotor synchronous motor with internal voltage
E2 = 1.3 pu and X = 1.2 pu. The reactance of the
Solution:  Bundled conductors are used to reduce line connecting the generator to the motor is 0.5 pu.
corona loss. When geometric mean distance (GMD) When the generator supplies 0.5 pu power, the rotor
of the conductor is increased, critical disruptive volt- and the difference between the machines will be
age will be reduced, thus, reducing the corona loss.
Ans. (c) (a) 57.42° (b) 1°
(c) 32.58° (d) 122.58°
2. A power system consists of 300 buses out of which (GATE 2003: 1 Mark)
20 buses are generator buses, 25 buses are the ones
with reactive power support and 15 buses are the ones Solution:  Given that: P = 0.5 pu, E = 2 pu and
with fixed shunt capacitors. All the other buses are V = 1.3 pu.
load buses. It is proposed to perform a load flow anal-
ysis for the system using Newton-Raphson method. The reactance of motor + generator + connecting
The size of the Newton-Raphson Jacobian matrix is lines:
(a) 553 × 553 (b) 540 × 540 Xd = Xeq = 1.1 + 1.2 + 0.5 = 2.8
(c) 555 × 555 (d) 554 × 554
To determine the rotor angle difference, we know
(GATE 2003: 1 Mark) that
EV
Solution:  Given that: n = 300 P = [sin(d 1 − d 2 )]
Generator bus = 20 Xd
Reactive power support bus = 25 2 × 1. 3
Fixed bus with shunt capacitor = 15 0. 5 = sin(d 1 − d 2 )
2. 8
d 1 − d 2 = 32.58°
Therefore, number of slack buses,
ns = 20 + 25 − 15 = 30
Ans. (c)
Size of Jacobian matrix= 2(n − ns ) × 2(n − ns )
= 2(300 − 30) × 2(300 − 30) 5. The interrupting time of a circuit breaker is the
period between the instant of
= 2(270)× 2(270)
= 540 × 540 (a) initiation of short circuit and the arc extinction
Ans. (b) on an opening operation.
(b) energising of the trip circuit and the arc extinc-
3. Choose two appropriate auxiliary components of a tion on an opening operation.
HVDC transmission system from the following (c) initiation of short circuit and the parting of
P. DC line inductor primary arc contacts.
Q. AC line inductor (d) energising of the trip circuit and the parting of
R. Reactive power sources primary arc contacts.
S. Distance relays on DC line (GATE 2003: 1 Mark)
T. Series capacitance on AC line
(a) P and Q (b) P and R Solution:  The interrupting time of CB is the time
(c) Q and S (d) S and T period between energising of trip circuit and the
arc extinction on an opening operation.
(GATE 2003: 1 Mark) Ans. (b)

Chapter 5 solved question paper.indd 448 3/23/2016 12:48:32 PM


SOLVED GATE PREVIOUS YEARS’ QUESTIONS        449

6. A balanced delta connected load of (8 + j6) Ω per special conditions S1 is open and S2 is closed. In
phase is connected to a 400 V, 50 Hz, three-phase such a case, the magnitude of the voltage across
supply lines. If the input power factor is to be the LV terminals a and c is
improved to 0.9 by connecting a bank of star con-
nected capacitors, the required kVAR of the bank is HV ∆ Y LV
A a
(a) 42.7 (b) 10.2
(c) 28.8 (d) 38.4
(GATE 2003: 2 Marks)
B b
Solution:  A balanced delta connected load= 8 + 6j.
Given V2 = 400 V and cos f2 = 0.9. S2
 6
f1 = tan−1   = 36.85°
 8 C c
f 2 = cos−1 0.9 = 25.84°
S1

Therefore,
V 400 400 (a) 240 V (b) 480 V
I= = = = 40 ∠ − 36 − 86°
Z 8 + 6 j 10 ∠36.86 (c) 415 V (d) 0 V

I = 32 − 24 j (GATE 2003: 2 Marks)



The power factor is improved by connecting a delta Solution:  Under the given condition
connected capacitor bank. The phasor diagram is
given as follow: S1 ® ON and S2 ® OFF

b IC c
a Step down voltage output is obtained as
11000 V ® 415 V
f2
Under the special condition
f1 I1
S1 ® OFF and S2 ® ON
O
Therefore, Two-phase supply is connected to the transformer
f2 = I cos f1
oa = I ′cos
V1 N 11000
I ′cos 25.84 = 32 = 1 = = 26.5
V2 N2 415
32
I′ = = 35.55
0.9 As output terminals a and c are in opposite phase
ac = 24 A and cancel each other, terminal voltage = 0 V.
Ans. (d)
ab = I ′ sin f2 = 35.5 sin 25.84 = 15.5 A
IC = ac − ab = 24 − 15.5 = 8.5 A 8. The ABCD parameters of a three-phase overhead
3 × 400 × 8.5 transmission line are A = D = 0.9∠0°, B = 200∠90° Ω
and C = 0.95 × 10-3∠90°. At no-load condition, a
3VIC
= = = 10.2 kVAR
1000 1000
shunt inductive reactor is connected at the receiv-
Ans. (b) ing end of the line to limit the receiving-end volt-
age to be equal to the sending-end voltage. The
7. The following figure shows a ΔY connected three- ohmic value of the reactor is
phase distribution transformer used to step down
(a) ∞ Ω (b) 2000 Ω
the voltage from 11,000 V to 415 V line-to-line.
(c) 105.26 Ω (d) 1052.6 Ω
It has two switches S1 and S2. Under normal con-
ditions S1 is closed and S2 is open. Under certain (GATE 2003: 2 Marks)

Chapter 5 solved question paper.indd 449 3/23/2016 12:48:46 PM


450        CHAPTER 5:  POWER SYSTEMS

Solution:  Given that: 10. A DC distribution system is shown in the following


A = D = 0.9∠0° figure with load currents as marked. The two ends
of the feeder are fed by voltage sources such that
B = 200 ∠90°Ω VP − VQ = 3 V. The value of the voltage VP for a
C = 0.95 × 10−3∠90°S
minimum voltage of 220 V at any point along the
feeder is
Receiving end voltage = sending end voltage. We
know that VP VQ
Vs = AVr + BI r
0.1 Ω R 0.15 Ω S 0.2 Ω
Also,
P Q
Vr = AVr + BI r ∵ Vs = Vr 
Vr (1 − A) = BI r
Vr B 200 ∠90°
= 2000 ∠90°
10 A 20 A 30 A 15 A
= =
Ir 1 − A 1 − 0.9 ∠0°
(a) 225.89 V (b) 222.89 V
Ans. (b) (c) 220.0 V (d) 228.58V

9. A surge of 20 kV magnitude travels along a lossless (GATE 2003: 2 Marks)


cable towards its junction with two identical loss-
less overhead transmission lines. The inductance Solution:  If the total current = I A, then
and the capacitance of the cable are 0.4 mH and Current through section PR = (I − 10) A
0.5 mF per km. The inductance and capacitance Current through section RS  = I − 10 − 20 = (I − 30) A
of the overhead transmission lines are 1.5 mH and Current through section SQ = I − 30 − 30 = (I − 60) A
0.015 mF per km. The magnitude of the voltage at VP − VQ = 3 V
the junction due to surge is
By Kirchhoff’s voltage law (KVL),
(a) 36.72 kV (b) 18.36 kV
(c) 6.07 kV (d) 33.93 kV VP − VQ = (I − 10) 0.1 + (I − 30) 0.15 + (I − 60) × 0.2

(GATE 2003: 2 Marks) 3 = 0.45 I − 17.5


20.5
Solution:  For the cable: I= = 45.6 A
0.45
L1 = 0.4 mH/km C1 = 0.5 mF/km V = 20 kV Line drop, VL = 8.58 V [∵ I = 45.6 A]
For the overhead transmission line
Hence,
L2 = 1.5 mH/km C2 = 0.5 mF/km
VP = 220 + VL = 220 + 8.58 = 228.58 V
Then for the cable
Ans. (d)
−3
L1 0.4 × 10
Z1 = = = 28.284
0.5 × 10−6
11. A three-phase 11 kV generator feeds power to a
C1
constant power unity power factor load of 100 MW
For the overhead transmission line through a three‑phase transmission line. The line-
to-line voltage at the terminals of the machine is
L2 1.5 × 10−5 maintained constant at 11 kV. The per unit positive
Z2 = Z3 = = = 316.23
0.015 × 10−6
C2 sequence impedance of the line based on 100 MVA
and 11 kV is j0.2. The line‑to‑line voltage at the
Therefore, voltage at junction due to surge is given by load terminals is measured to be less than 11 kV.
The total reactive power to be injected at the termi-
2 × V × Z2 nals of the load to increase the line-to-line voltage at
V=
Z1 + Z2 the load terminals to 11 kV is
2 × 20 × 103 × 316.23 (a) 100 MVAR (b) 10.1 MVAR
= = 36.72 kV
316 + 28.284 (c) −100 MVAR (d) −10.1 MVAR
Ans. (a) (GATE 2003: 2 Marks)

Chapter 5 solved question paper.indd 450 3/23/2016 12:49:00 PM


SOLVED GATE PREVIOUS YEARS’ QUESTIONS        451

é Z12 ù
Solution:  Given that: PL = 100 MW, ZL(pu) =
0.2j, Vs = Vr = 11 kV êZ ú
Þ ´ ê 22 ú ´ éëZ21 Z22 Z23 Z24 ùû
1
Zij + Zb êZ23 ú
(kV)2 (11)2 êëZ24 úû
ZL = ZL(base) × = j0.2 × = j 0.242 W
MV 100
V s V r sin d Z = j0.2 W, reference bus j = 2, n = 4
PL =
ZL Substituting values, we have
11 × 103 × 11 × 103 é j0.2860 ù
100 × 106 = sin d ê j0.3408 ú
´ê ú ´
0.242 1
=
or, sin d = 0.2 Þ d = 11.54 j0.3408 + j0.2 ê j0.2586 ú
êë j0.2414 úû
Reactive power is given by [ j0.2860 j0.3408 j0.2586 j0.2414]
Vs V r cos d 2
Vr
Qr = − The required change is in Z22 and Z3
ZL ZL
11 × 103 × 11 × 103 (11 × 103 )2 j2 (0.3408)2
= cos(11.54) − ¢ =
Z22 = j0.2147
0.242 0.242 j(0.5408)
121 × 106 j2 (0.3408)(0.2586)
= [cos(11.54) − 1] = −10.1 MVAR ¢ =
Z23 = j0.16296
0.242 j(00.5408)
Ans. (d)
Therefore,

¢ = j0.3408 − j0.2417
Z22(new) = Z22(old) − Z22
12. The bus impedance matrix of a 4-bus power system
is given by
= j0.1260
é j0.3435 j0.2860 j0.2723 j0.2277 ù ¢ = j0.2586 − j0.16296
Z23(new) = Z23(old) − Z23
ê j0.2860 j0.3408 j0.2586 j0.2414 ú
Zbus =ê ú = j0.0956
ê j0.2723 j0.2586 j0.2791 j0.2209 ú
êë j0.2277 j0.2414 j0.2209 j0.2791 úû Ans. (b)
13. A 20 MVA, 6.6 kV, three-phase alternator is con-
A branch having an impedance of j 0.2Ω is con- nected to a three-phase transmission line. The per
nected between bus 2 and the reference. Then unit positive-sequence, negative-sequence and zero-
the  values of Z22,new and Z23,new of the bus sequence impedances of the alternator are j0.1, j0.l
impedance matrix of the modified network are, and j0.04, respectively. The neutral of the alternator
respectively, is connected to ground through an inductive reactor
(a) j 0.5408 Ω and j 0.4586 Ω of j0.05 pu. The per unit positive-, negative- and
(b) j 0.1260 Ω and j 0.0956 Ω zero-sequence impedances of the transmission line
are j0.1, j0.1 and j0.3, respectively. All per unit
(c) j 0.5408 Ω and j 0.0956 Ω values are based on the machine ratings. A solid
(d) j 0.1260 Ω and j 0.1630 Ω ground fault occurs at one phase of the far end of
the transmission line. The voltage of the alternator
(GATE 2003: 2 Marks) neutral with respect to ground during the fault is

Solution:  When a branch of 0.2j Ω is connected (a) 513.8 V (b) 889.9 V


between bus 2 and reference (c) 1112 V (d) 642.2 V
(GATE 2003: 2 Marks)
Zij 
 
×   × Z ji … Z jn 
1
= ZB(old) − Solution:  Zero sequence impedance for generator
Zij + Zb 
ZB(new)
 and transmission line,
Znj  ZL(0) + ZG (0) = Z0
Z0 = j0.04 + j0.3 = j0.34 pu

Chapter 5 solved question paper.indd 451 3/23/2016 12:49:11 PM


452        CHAPTER 5:  POWER SYSTEMS

Positive sequence impedance, (a) P1 = 242.86 MW; P2 = 157.14 MW; and
P3 = 300 MW
Z1 = ZL(1) + ZG(1) = j0.1 + j0.1 = j0.2 pu
(b) P1 = 157.14 MW; P2 = 242.86 MW; and
P3 = 300 MW
Negative sequence impedance, (c) P1 = 300.0 MW; P2 = 300.0 MW; and
Z2 = ZL(2) + ZG(2) = j0.1 + j0.1 = j0.2 pu P3 = 100 MW
(d) P1 = 233.3 MW; P2 = 233.3 MW; and
Also, given that ZN = j 0.05 pu P3 = 233.4 MW
For line to ground fault, the current is (GATE 2003: 2 Marks)
Ea Solution:  For optimal power generation,
Ia =
Z0 + Z1 + Z2 + 3ZN P3 = maximum load = 300 MW and IC1 = IC2
0.1 20 +
200+
.3 P =1 30
0.31 P =+300+
.4P
0.24P2
=
j0.2 + j0.2 + j0.34 + j0.15 Therefore,
Therefore,
Therefore,
= −j 1.12 p
01.3−
P10−
pu
0.3P .4P =2 10
0.24P = 10 (i)
The base current is given by Also,
P1 + P2 + P3 = 700
Generator MVA 20 × 106
IB = = = 1750 A
3 Generator KV 3 × 6.6 × 103 P1 + P2 + 300 = 700
P1 + P2 = 400  (ii)
Therefore, the fault current is
Solving Eqs. (i) and (ii), we get
I f = 3 Ia IB = 3(−j1.12) (1750) = − j 5897.6 A
P1 = 242.8 MW and P2 = 157.14 MW
Neutral voltage is given by
VN = I f ZN Ans. (a)

Where 15. A list of relays and the power system components


protected by the relays are given in Group I and
(6.6)2 Group II, respectively. Choose the correct match
ZN = ZB Zpu = × 0.05 = 0.1089 W from the codes given below.
20

Therefore,
Group I Group II
VN = 5897.6 × 0.1089 = 642.2 V
P.   Distance relay 1. Transformers
Ans. (d) Q. Under frequency relay 2. Turbines
14. Incremental fuel costs (in some appropriate unit) R. Differential relay 3. Busbars
for a power plant consisting of three generating S.    Buchholz relay 4. Shunt capacitors
units are
5. Alternators
IC1 = 20 + 0.3P1 6. Transmission
IC2 = 30 + 0.4P2 lines
IC3 = 30
where Pi is the power in MW generated by unit i, Code:
for i = 1, 2 and 3.
P Q R S
Assume that all the three units are operating all (a) 6 5 3 1
the time. Minimum and maximum loads on each (b) 4 3 2 1
unit are 50 MW and 300 MW, respectively. If the (c) 5 2 1 6
plant is operating on economic load dispatch to (d) 6 4 5 3
supply the total power demand of 700 MW, the
power generated by each unit is (GATE 2003: 2 Marks)

Chapter 5 solved question paper.indd 452 3/23/2016 12:49:23 PM


SOLVED GATE PREVIOUS YEARS’ QUESTIONS        453

Solution: p
2 sin 30°(110 − 30) = 2 (cos d c − cos 110°)
180
Group I Group II 80p
0.5 × = cos d c + 0.342
Distance relay Transmission lines 180
Under frequency relay Alternators cos d c = 0.698 − 0.342
Differential relay Busbars  d c = 69.138° Ans. (c)
Buchholz relay Transformers 17. A three-phase alternator generating unbalanced
voltages is connected to an unbalanced load through
Ans. (a)
a three-phase transmission line as shown in the fol-
16. A generator delivers power of 1.0 pu to an infi- lowing figure. The neutral of the alternator and the
nite bus through a purely reactive network. star point of the load are solidly grounded. The
The maximum power that could be delivered phase voltages of the alternator are Ea = 10 ∠ 0° V,
by the generator is 2.0 pu A three-phase fault E b = 10 ∠ −90° V, Ec = 10 ∠120° V. The positive-
occurs at the terminals of the generator which sequence component of the load current is
reduces the generator output to zero. The fault
Ea j1.0 Ω j1.0 Ω
is cleared after t c seconds. The original network
is then restored. The maximum swing of the
rotor angle is found to be dmax = 110 electri- Eb j1.0 Ω j2.0 Ω
cal degree. Then the rotor angle in electrical
degrees at t = t c is Ec j1.0 Ω j3.0 Ω
(a) 55 (b) 70
(c) 69.14 (d) 72.4
(a) 1.310 ∠−107°A (b) 0.332 ∠−120° A
(GATE 2003: 2 Marks) (c) 0.996 ∠−120°A (d) 3.510 ∠−81° A
(GATE 2003: 2 Marks)
Solution:  From the given data, we have
Solution:  The three phase currents are
Ea 10 ∠0°
= 5∠ −90°
P = 2sind
Pmax 2 Ia = =
Za 2j
A2 Eb 10 ∠ −90°
Ib = = = 3.33∠ −180°
P1 1 Zb 3j

A1 We know that
d E 10Ð120°
Ic = c = = 2.5Ð30°
30° dc d max Zc 4j

[I + aI b + a 2 I c ]
1
By equal area criterion, we get I a1 =
3 a

d max 1 é5Ð −90° + 3.33Ð(−180° + 120°)ù


Ps = (d max − d 0 ) = òd c
Pmax sin d × dd I a1 =
3 êë+2.5Ð(240° + 30°)
ú
û
Pmax sin éë(d 0 (d max - d 0 )ùû
[∵ a = 1 Ð120°a = 1 Ð240°]
2

= Pmax éëcos d 0 − cos d max ùû (i) 1


3 5Ð −90° + 3.33Ð −60° + 2.5Ð270°]
= [
Given that Pmax = 2 pu, therefore 1
= [ - 5 j + 1.665 − j2.883 − 2.5 j ]
3
P0 = Pmax sin d 0 = 1
= [1.665 − j10.383]
1
2 sin d 0 = 1 ⇒ d = 30° 3

= 3.5Ð −80. 89°
Substituting the value of d in Eq. (i) and using
Ans. (d)
given value d max = 110°, we get

Chapter 5 solved question paper.indd 453 3/23/2016 12:49:31 PM


454        CHAPTER 5:  POWER SYSTEMS

18. The rated voltage of a three-phase power system is 22. The transmission line distance protection relay
given as having the property of being inherently directional is
(a) rms phase voltage (a) impedance relay (b) mho relay
(b) peak phase voltage (c) ohm relay (d) reactance relay
(c) rms line to line voltage
(GATE 2004: 1 Mark)
(d) peak line to line voltage
(GATE 2004: 1 Mark) Solution:  One of the types of distance relay is mho.
This has the property of being inherently directional
Ans. (c)
which is used for transmission line protection.
19. The phase sequence of the three-phase system shown Ans. (b)
in the following figure is
23. A 800 kV transmission line is having per phase line
R
inductance of 1.1 mH/km and per phase line capaci-
tance of 11.68 nF/km. Ignoring the length of the
line, its ideal power transfer capability in MW is

Y (a) 1204 MW (b) 1504 MW


(c) 2085 MW (d) 2606 MW
B
(GATE 2004: 2 Marks)
(a) RYB (b) RBY
(c) BRY (d) YBR Solution:  Given that V = 800 kV, L = 1.1 MH/km
(2004: 1 Mark) and C = 11.68 nF/km. Then,

Surge impedance,
Solution:  The phase sequence is RBY
1.1 × 103
R Z = L/C = = 306.88 Ω
11.68 × 10−9
Y R

Y V2 (800)2
B Power P = = = 2085 MW
Z 306.88
B Ans. (c)
Ans. (b)
24. A 110 kV, single core coaxial, XLPE insulated
20. In thermal power plants, the pressure in the work- power cable delivering power at 50 Hz, has a capac-
ing fluid cycle is developed by itance of 125 nF/km. If the dielectric loss tangent
(a) condenser (b) super heater of XLPE is 2 × 10−4, then dielectric power loss in
(c) feed water pump (d) turbine this cable in W/km is
(a) 5.0 (b) 31.7
(GATE 2004: 1 Mark) (c) 37.8 (d) 189.0
Ans. (c)
(GATE 2004: 2 Marks)
21. For harnessing low variable water heads, the suit-
able check for coverage in text with high percent- Solution:  Given for the cross-linked polyethylene
age of reaction and runner adjustable vanes is power cable:
V = 110 kV; C = 125 nF/km tan d = 2 ´ 10 -4
(a) Kaplan (b) Francis
Dielectric power loss is given by
(c) Pelton (d) Impeller
(GATE 2004: 1 Mark) PL = 2V 2w C tan d
= 2 (110 × 103 )2 2p f × 125 × 10−9 × 2 × 10−4
Solution:  Kaplan turbines are used for harness-
ing low variable water heads. This is because of = 2 (110 × 103 )2 2p × 50 × 125 × 10−9 × 2 × 10−4
high percentage of reaction and runner adjustable = 189 W/ km
vanes.
Ans. (a) Ans. (d)

Chapter 5 solved question paper.indd 454 3/23/2016 12:49:40 PM


SOLVED GATE PREVIOUS YEARS’ QUESTIONS        455

25. A lightning stroke discharges impulse current of Also, for reactive power we have
10 kA (peak) on a 400 kV transmission line having
surge impedance of 250 Ω. The magnitude of tran- Qr = PR tan f = PR tan(cos−1 f )
sient over-voltage travelling waves in either direc-
tion assuming equal distribution from the point of = 50 tan(cos−1 0.9) = 24.2 MW

lightning strike will be
Vs sin(76.4 − d ) = 215 (ii)
(a) 1250 kV (b) 1650 kV
(c) 2500 kV (d) 2900 kV
From Eqs. (i) and (ii)
(GATE 2004: 2 Marks)
2
Vs = (215)2 + (83.46)2
Solution:  Given that: I = 10 kA V = 400 kV and
Z = 25 Ω.
IZ Vs = (215)2 + (83.46)2 = 230.63 kV
Magnitude of transient over voltage V =
2
Ans. (c)
This is because impulse current will be equally
divided in both directions. Therefore, substituting
values, we have 27. A new generator having Eg = 1.4∠30° pu
[equivalent to (1.212 + j 0.70) pu] and synchro-
10 × 103 × 250 nous reactance Xs of 1.0 pu on the system base,
V = = 1250 kV
2 is to be connected to a bus having voltage Vt in
Ans. (a) the existing power system. This existing power
system can be represented by Thevenin’s voltage
26. The generalised circuit constants of a three-phase, ETh = 0.9Ð0°pu in series with Thevenin’s imped-
220 kV rated voltage, medium length transmission ance ZTh = 0.25Ð90° pu. The magnitude of the
line are bus voltage Vt of the system in pu will be

A = D = 0.936 + j0.016 = 0.936Ð0.98° (a) 0.990 V (b) 0.973 V


(c) 0.963 V (d) 0.900 V
B = 33.5 + j138 = 142.0Ð76.4°W
C = (−5.18 + j914)× 10−6 W
(GATE 2004: 2 Marks)

Solution:  Given that:


If load at the receiving end is 50 MW at 220 kV
with a power factor of 0.9 lagging, then magnitude
of line to line sending end voltage should be Eg = 1.4Ð30 pu; ETh = 0.90Ð0 ;
(a) 133.23 kV (b) 220.00 kV Xs = 1.0 pu and ZTh = 0.25Ð90
(c) 230.78 kV (d) 246.30 kV
(GATE 2004: 2 Marks) Thevenin’s equivalent circuit is given by

ZTh Xs
Solution:  Given the circuit parameter constants
A, B, C and D and
I
PR = 50 MW a = 0.98o + + E
ETh Vt
o − − g
Vr = 220 kV b = 76.4
pf = 0.9 lag

We know that Therefore,

Eg − ETh
2
Vs Vr A Vr 1.4∠30° − 0.9∠0°
Pr = cos( b − d ) − cos( b − a ) I = =
B B ZTh + Xs j1.25
V s × 220 (0.936) (220)2 1.212 + j7 − 0.9
50 = cos(764° − d ) − cos(75.6°) =
j 1.25
142 142
Vs [cos(76.4 − d )] = 83.46 (i) = 0.56 − 0.2496 j

Chapter 5 solved question paper.indd 455 3/23/2016 12:49:57 PM


456        CHAPTER 5:  POWER SYSTEMS

Vt = Eg − IXs = 1.212 + j7 − (0.56 − 0.2496 j) (1j) (a) 5.78∠ −30° A (b) 5.78∠90°A
= 1.212 − 0.249 + j (0.7 − 0.56) (c) 6.33∠90°A (d) 10.00 ∠ −30° A

= 0.972Ð8.3° (GATE 2004: 2 Marks)


Ans. (b) Solution:  From the figure, we have Ia = 10Ð0°A, I b = 10Ð180°A
Ia = 10Ð0°A, I b = 10Ð180°A and I c = 0
28. A three-phase generator rated at 110 MVA,
11 kV is connected through circuit breakers to a Then positive sequence current component in line
transformer. The generator is having direct axis a will be
sub-transient reactance Xd² = 19%, transient 1
reactance Xd¢ = 26% and synchronous reactance I a 1 = [ I a + a I b + a 2 I c]
3
= 130%. The generator is operating at no load and
= [10 ∠0° + (1∠120°× 10 ∠180°) + 0]
1
rated voltage when a three phase short circuit fault
occurs between the breakers and the transformer. 3
= [10 ∠0° + 10 ∠300°]
The magnitude of initial symmetrical RMS current 1
in the breakers will be 3
15 −j 8.66
= [10 + 5 − j 8.66 ] =
1
(a) 4.44 kA (b) 22.20 kA
(c) 30.39 kA (d) 38.45 kA 3 2
17.32 ∠ −30°
(GATE 2004: 2 Marks) = = 5.78 ∠ − 30 °
3
Solution:  Given that, rating = 110 MVA, volt- Ans. (a)
age = 11 kV, sub-transient reactance Xd² = 19%,
transient reactance Xd¢ = 26% and synchronous 30. A 500 MVA, 50 Hz, three-phase turbo-generator
reactance = 130%. produces power at 22 kV. Generator is Y-connected
and its neutral is solidly grounded. Its sequence
Short circuit current I sc reactances are X1 = X2 = 0.15 and X0 = 0.05 pu. It
1 1 is operating at rated voltage and disconnected from
I sc = = = −j5.26 pu the rest of the system (no load). The magnitude
Xd ¢¢ j 0.19
of the sub-transient line current for single line to
Base current ground fault at the generator terminal in pu will be

Generator MVA 110 × 106 (a) 2.851 (b) 3.333


IB = = (c) 6.667 (d) 8.553
3 × generator KV 3 × 11 × 103
= 5773.67 A (GATE 2004: 2 Marks)

I rms = IB × I sc = 5773.67 × 5.26 = 30369.5 A Solution:  Given 500 MVA three-phase turbo gen-
= 30.37 kA erator, with

Ans. (c) f = 50 Hz, V = 22 kV, X1 = X2

29. A three-phase transmission line supplies Δ-connected = 0.15 and X0 = 0.05 pu


load Z. The conductor `c’ of the line develops an
open circuit fault as shown in the following figure. Then
The currents in the lines are as shown in the dia-
Ea
gram. The positive sequence current component in Ia 0 =
line `a’ will be X0 + X1 + X2
1Ð0°
Ia = 10–0°A =
a j 0.15 + j 0.15 + j 0.05
1
= = −2.857 j
Z Z 0.35 j
Ib = 10–180°A
b
Ic = 0 Sub-transient line current for a single line to ground
Z
c fault is given by

Chapter 5 solved question paper.indd 456 3/23/2016 12:50:12 PM


SOLVED GATE PREVIOUS YEARS’ QUESTIONS        457

I f = 3Ia(0) (∵ Ia(0) = Ia(1) = Ia(2)) 120 × 50


p= = 24
= 3(−2.857j) = 8.57 A 250
Ans. (d)
Ans. (d)

31. A50 Hz, 4-pole, 500 MVA, 22 kV turbo-generator 33. The pu parameters for a 500 MVA machine on its
is delivering rated megavolt-amperes at 0.8 power own base are inertia, M = 20 pu; reactance, X = 2 pu.
factor. Suddenly, a fault occurs reducing electric The pu values of inertia and reactance on 100 MVA
power output by 40%. Neglect losses and assume common base, respectively, are
constant power input to the shaft. The accelerating
(a) 4, 0.4 (b) 100, 10
torque in the generator in MNm at the time of the
(c) 4, 10 (d) 100, 0.4
fault will be
(GATE 2005: 1 Mark)
(a) 1.528 (b) 1.018
(c) 0.848 (d) 0.509
Solution:  Given that, at 500 MVA, M = 20 pu
(GATE 2004: 2 Marks) and X = 2 pu. We know that the relation between
old and new pu values is given by
Solution:  Given for the 500 MVA and 22 kV gen-
erator, that f = 50 Hz, number of poles, p = 4 and pu(new) = pu(old)
power factor lag = 0.8. Also, the output is reduced
by 40% when fault occurs. The pu values of inertia (M) can be determined as:
Power (without fault) = 500 × 0.6 = 400 MW
MVA(old)
Power (after fault) = 400 × 0.6 = 240 MW M(new)(pu) = M(old)(pu) ×
MVA(new)
Then accelerating torque at the time of fault can
be determined as 500
= 20 × = 100 pu
100
Pa = Ta ×w
Pa
Ta = (where w = 2p fmech) Similarly, reactance (X) can be determined as
w
2 2 50 MVA(new)
fmech = felec × = 50 × = = 25 X(new)(pu) = X(old)(pu) ×
p 4 2 MVA(old)
Pa = 400 − 240 = 160 MW
100
160 = 2× = 0.4 pu
Ta = = 1.019 MNm 500
2p × 25 Ans. (d)
Ans. (b)
34. A 800 kV transmission line has a maximum power
32. A hydraulic turbine having rated speed of 250 rpm transfer capacity of P. If it is operated at 400 kV
is connected to a synchronous generator. In order with the series reactance unchanged, then new
to produce power at 50 Hz, the number of poles maximum power transfer capacity is approximately
required in the generator are (a) P (b) 2P
(a) 6 (b) 12 (c) P/2 (d) P/4
(c) 16 (d) 24 (GATE 2005: 1 Mark)
(GATE 2004: 2 Marks)
Solution:  Given that P is the maximum power
Solution:  Given that for the turbine, rated speed
transfer capacity of 800 kV transmission line.
N = 250 rpm and f = 50 Hz. We know that,
Then, the maximum power transfer capacity of
400 kV can be determined from the following
Þp=
120f 120f
N =
p N relation

sin d Þ P µ V 2
EV
where p is the number of poles. Substituting values, P =
we get X

Chapter 5 solved question paper.indd 457 3/23/2016 12:50:23 PM


458        CHAPTER 5:  POWER SYSTEMS

Therefore, when V is halved; power transfer capacity (a) 0.85 lag (b) 1.00 lag
becomes 1/4 of the pervious value. (c) 0.80 lag (d) 0.90 lag
Ans. (d)
(GATE 2005: 2 Marks)
35. The insulation strength of an EHV transmission line
is mainly governed by Solution:  Given that: PL = 4 MW load,QC = 2
(a) load power factor MVAR for load and power factor pf = 0.97 lag. Then,
(b) switching over-voltages cos f = 0.97 Þ f = cos−1 0.97
(c) harmonics
(d) corona tan f = tan(cos−1 0.97) = 0.25

(GATE 2005: 1 Mark) Q −Q


QLL − QCC = 0.25
Ans. (b) P = 0.25
PLL
Substituting
Substituting values
values, we get
Substituting values
QL − 2
36. High voltage DC (HVDC) transmission is mainly
used for QL − 2 = 0.25 Þ QL = 3 MVAR
4 = 0.25 Þ QL = 3 MVAR
(a) bulk power transmission over very long distances. 4
(b) inter-connecting two systems with the same Q   3
f = tan−1  L  = tan−1   = 36°
 4
nominal frequency.
(c) eliminating reactive power requirement in the  PL 
operation. cos f = cos 36° = 0.8 lag
(d) minimizing harmonics at the converter stations.
Ans. (c)
(GATE 2005: 1 Mark)
39. The network shown in the given figure has imped-
Ans. (a)
ances in pu as indicated. The diagonal element Y22
37. The parameters of a transposed overhead transmis- of the bus admittance matrix Ybus of the network is
sion line are given as:
1 2 3
Self reactance, Xs = 0.4 Ω/km
+j0.1 +j0.1
Mutual reactance, Xm =0.1 Ω/km
The positive sequence reactance X1 and zero sequence
reactance X0, respectively, in Ω/km are −j20.0 −j20.0 −j10.0

(a) 0.3, 0.2 (b) 0.5, 0.2


(c) 0.5, 0.6 (d) 0.3, 0.6
(a) −j 19.8 (b) + j 20.0
(GATE 2005: 2 Marks) (b) + j 0.2 (d) −j 19.95
Solution:  Given that (GATE 2005: 2 Marks)

Xs = 0.4 Ω/km Xm = 0.1 Ω/km Solution:  For the given network

Then positive sequence reactance is I1 = V1Y11 + (V1 − V2 )Y12


= 0.05 V1 − j 10 (V1 − V2 )
X1 = Xs − Xm = 0.4 − 0.1 = 0.3 W/km
= −j9.95 V1 + j10V2

I2 = (V2 − V1 )Y21 + (V2 − V3 )Y23


Zero sequence reactance

X0 = Xs + 2Xm = 0.4 + 2(0.1) = 0.6 W/km = j10V1 − j 9.9 V2 − j0.1V3

Ans. (d) Then

38. At an industrial sub-station with a 4 MW load, a Y22 = Y11 + Y23 + Y2


capacitor of 2 MVAR is installed to maintain the = −j9.95 − j9.9 − j0.1 = −j19.95
load power factor at 0.97 lagging. If the capacitor
goes out of service, the load power factor becomes Ans. (d)

Chapter 5 solved question paper.indd 458 3/23/2016 12:50:35 PM


SOLVED GATE PREVIOUS YEARS’ QUESTIONS        459

40. A load centre is at an equidistant from the two (a) 10∠30°Ω (b) 40∠−45°Ω
thermal generating stations G1 and G2 as shown in
(c) 1 Ω (d) 0 Ω
the given figure. The fuel cost characteristics of the
generating stations are given by (GATE 2005: 2 Marks)
F1= a + b P1+ c P12 Rs/hour
Solution: We know that
F2 = a + b P2 + 2c P22 Rs/hour,
where P1and P2 are the generation in MW of G1 éV1 ù é A B ù éV2 ù
ê ú = êC D ú ê ú
ëI1 û ë û ëI1 û
and G2, respectively.

V1 I1
B= C =
I2 Y2
G1 G2 V2 = 0 I2 = 0
Load

For most economic generation to meet 300 MW of From the given figure,
load P1 and P2, respectively, are
æ V1 ) ö
ç ÷
(a) 150, 150 (b) 100, 200
(c) 200, 100 (d) 175, 125
C= è Z1 + Z2 ø
=
1
æ V1 ö Z2
ç ÷ × Z2
(GATE 2005: 2 Marks)
è 1
Z + Z2ø
Solution:  Given that

F1 = a + bP1 + CP12 Rs/h Therefore,

F2 = a + bP2 + 2CP22 Rs/h 1 1


Z2 = = = 40 ∠ −45°
P1 + P2 = 300 MW C 0.025∠45°
P1 = 300 − P2  (i)
Ans. (b)
For economical operation
42. A generator with constant 1.0 pu terminal voltage
∂F1 ∂F supplies power through a step-up transformer of
= 2
∂P1 ∂P2 0.12 pu reactance and a double-circuit line to an
0 + b + 2CP1 = 0 + b + 4CP2 infinite bus bar as shown in the following figure.
The ­infinite bus voltage is maintained at 1.0 pu.
P1 = 2P2  (ii) Neglecting the resistances and susceptances of the
system, the steady state stability power limit of
From Eqs. (i) and (ii),we get
the system is 6.25 pu. If one of the double-circuit is
(300 − P2 ) = 2P2 tripped, then resulting steady state stability power
300 = 3P2 limit in pu will be
Þ P2 = 100 MW; P1 = 200 MW
1 2 3
Ans. (c) x
41. Two networks are connected in cascade as shown in
x
the following figure. With the usual notations the
equivalent A, B, C and D constants are obtained.
(a) 12.5 pu (b) 3.125 pu
Given that, C = 0.025 ∠45°, the value of Z2 is
(c) 10.0 pu (d) 5.0 pu
Z1 = 10–30°Ω
(GATE 2005: 2 Marks)

Z2 Solution:  The given circuit can be represented


as

Chapter 5 solved question paper.indd 459 3/23/2016 12:50:45 PM


460        CHAPTER 5:  POWER SYSTEMS

44. The positive sequence driving point reactance at


1.0 pu 1 0.12 V 2 3
the bus is
x
(a) 2.5 Ω (b) 4.033 Ω
x (c) 5.5 Ω (d) 12.1 Ω
G T1 1.0 pu
(GATE 2005: 2 Marks)
Steady state stability power limit Pm is given by Solution:  Given that: substation level = 220 kV;
three-phase fault level = 4000 MVA and ground
EV
Pm = fault level = 5000 MVA. The fault current is
X
4, 000
I f =
Substituting values, we have 3 × 220
1×1 The reactance is given by:
6.25 =
X
0.12 + Vph 220/ 3
2 X= = = 12.1 W
If 4000/ 3 × 220
X = 0.08 pu
Ans. (d)
In one of the double circuit is tripped
45. The zero sequence driving point reactance at the
EV 1×1 1 bus is
Pm (new) = = = = 5 pu
X 0.12 + 0.08 0.2 (a) 2.2 Ω (b) 4.84 Ω
(c) 18.18 Ω (d) 22.72 Ω
Ans. (d)
(GATE 2005: 2 Marks)
43. An electric motor, developing a starting torque of
15 Nm, starts with a load torque of 7 Nm on its Solution:  The fault current is
shaft. If the acceleration at start is 2 rad/s2, then 5000
    I f =
moment of inertia of the systems must be (neglect- 3 × 220
ing viscous and Coulomb friction)
If 5000
(a) 0.25 kgm2 (b) 0.25 Nm2 Ia(1) = Ia(2) = Ia(0) = =
3 3 × 3 × 220
(c) 4 kgm2 (d) 4 Nm2
Vph
(GATE 2005: 2 Marks) X0 + X1 + X2 =
Ia(1)
Solution:  Given that Tst = 15 Nm, TL = 7 Nm 220/ 3 127
and a = 2 rad/s2.
= = = 29.036
5000/3 3 × 220 4.37
We know that
Since,
T = Tst − TL = 15 − 7 = 8 Nm X1 = X2 = −12.1 W
T = I a Therefore,
8
I = = 4 kgm 2 X0 = 29.036 − 12.1 − 12.1 = 4.836 W
2
Ans. (b)
Ans. (c)
46. The concept of an electrically short, medium, and
Common Data for Questions 44 and 45: At a long line is primarily based on the
220 kV substation of a power system, it is given (a) nominal voltage of the line
that the three-phase fault level is 4000 MVA and (b) physical length of the line
single-line to ground fault level is 5000 MVA. (c) wavelength of the line
Neglecting the resistance and the shunt suscep- (d) power transmitted over the line
tances of the system,
(GATE 2006: 1 Mark)
Ans. (b)

Chapter 5 solved question paper.indd 460 3/23/2016 12:50:58 PM


SOLVED GATE PREVIOUS YEARS’ QUESTIONS        461

47. Keeping in view the cost and overall effective- Real power, P = S cos f = 12 × 3 × 0.8 = 16.6 kW
sinff== 12 × 3 × 0.6==
ness, the following circuit breaker is best suited for
Reactive power, QQ==SSsin 12.5
12.kW
5 kW
capacitor bank switching
where S is the generator ratings.
(a) vacuum (b) air blast For unity power factor, we have to set capacitor
(c) SF6 (d) oil bank to Q = 12.5 kW.
Ans. (d)
(GATE 2006: 1 Mark)
Ans. (a) 51. The A, B, C and D constants of a 220 kV line are:
A = D = 0.94 ∠1°; B = 130 ∠73°; C = 0.001 ∠.90°
48. In a biased differential relay, the bias is defined as If sending end voltage of the line for a given load
a ratio of delivered at nominal voltage is 240 kV, then %
(a) number of turns of restraining and operating coil voltage regulation of the line is
(b) operating coil current and restraining coil (a) 5 (b) 9
current (c) 16 (d) 21
(c) fault current and operating coil current
(d) fault current and restraining coil current (GATE 2006: 2 Marks)
Solution:  Given that
(GATE 2006: 1 Mark)
A = D = 0.94 ∠1°; B = 130 ∠73° C = 0.001 ∠90°;
Ans. (b)
Vs = 240 kV
49. An HVDC link consists of rectifier, inverter trans-
mission line and other equipment. Which one of We know that % voltage regulation
the following is true for this link?
Vr(no load) − Vr(full load)
(a) The transmission line produces/supplies reac- = ×100
VR(full load)
tive power.
(b) The rectifier consumes reactive power and the Ir(no load) = 0. Therefore,
inverter supplies reactive power from/to the
respective connected AC systems. Vs 240
Vr(no load) = = = 255.32
(c) Rectifier supplies reactive power and the A 0.94
inverter consumes reactive power to/from the
Vr(full load) = 220 kV
respective connected AC systems.
(d) Both the converters (rectifier and inverter)
So, % Voltage regulation
consume reactive power from the respective
connected AC systems. 255.32 − 220
= × 100 = 16%
(GATE 2006: 1 Mark) 220
Ans. (c)
Ans. (b)
52. A single-phase transmission line and a telephone
50. A 400 V, 50 Hz, three-phase balanced source sup- line are both symmetrically strung one below the
plies power to a star-connected load whose rating other, in horizontal configurations, on a common
is 12 3 kVA , 0.8 pf (lag). The rating (in kVAR) tower. The shortest and longest distances between
of the delta-connected (capacitive) reactive power the phase and telephone conductors are 2.5 m and
bank necessary to bring the pf to unity is 3 m, respectively. The voltage (volt/km) induced
in the telephone circuit, due to 50 Hz current of
(a) 28.78 100 A in the power circuit is
(b) 21.60
(c) 16.60 (a) 4.81 (b) 3.56
(c) 2.29 (d) 1.27
(d) 12.47
(GATE 2006: 2 Marks)
(GATE 2006: 2 Marks)
Solution:  Let P be the power line and T be tele-
Solution: We know that phone line. Let points P1T1 and P2T2 denote the

Chapter 5 solved question paper.indd 461 3/23/2016 12:51:06 PM


462        CHAPTER 5:  POWER SYSTEMS

points of shortest (Ds) and maximum (Dm) distance Ib


between the two lines. Then inductance is given by
a
æD ö
b Vbn
LP = 2 × 10−7 ln ç m ÷ H/m
Ia
n Van Za Zb
è Ds ø k
æD ö Vcn Zc
LT = 2 × 10−7 ln ç m ÷ H/m Ic
è Ds ø c
Ans. (b)
Total inductance due to conductor P and T,
L = LP + LT, that is, 54. A generator is connected through a 20 MVA, 13.8/
138 kV step down transformer, to a transmission
æD ö
L = 4 × 10−7 ln ç P2T2 ÷ H/m
line. At the receiving end of the line a load is supplied
è DP1T1 ø
through a step down transformer of 10 MVA, 138/69 kV
rating. A 0.72 pu load, evaluated on load side trans-
æ 3 ö
= 4 × 10−7 ln ç −7
÷ = 0.73 × 10 H/m
former ratings as base values, is supplied from the
è 2. 5 ø above system. For system base values of 10 MVA
Hence, voltage induced by power line into tele- and 69 kV in load circuit, the value of the load (in
phone line is per unit) in generator circuit will be
(a) 36 (b) 1.44 (c) 0.72 (d) 0.18
VT = wLI = 2pfLI
(GATE 2006: 2 Marks)
= 2 × 3.14 × 50 × (0.73 × 10−7 ) × 100 Solution: We know that
= 2.29 V/km
æ MVA(old) ö æ KV(new) ö
2
Ans. (c) Znew = Zold × ç ÷×ç ÷
ç MVA(new) ÷ ç KV(old) ÷
è ø è ø
53. Three identical star connected resistors of 1.0 pu
20 æ 69 ö
2
are connected to an unbalanced three-phase supply. = 0.72 × ´ç ÷ = 36 pu
The load neutral is isolated. The symmetrical com- 10 è 13.8 ø
ponents of the line voltages in pu are: Znew = 36 pu
Ans. (a)
Vab1 = X∠q1, Vab2 = Y∠q2

If all the pu calculations are with the respective 55. The Gauss Seidel load flow method has the follow-
base values, the phase to neutral sequence voltages ing disadvantages. Tick the incorrect statement.
are: (a) Unreliable convergence
(a) Vanl = X∠(q1 + 30 ); Van2 =Y∠(q2 − 30 )
o o (b) Slow convergence
(c) Choice of slack bus affects convergence
(b) Vanl = X∠(q1 − 30o); Van2 =Y∠(q2 + 30o) (d) A good initial guess for voltages is essential for
1 1
(c) Van1 = X∠(q 2 − 30°); Van2 = Y∠(q 2 − 30°) convergence
3 3 (GATE 2006: 2 Marks)
Ans. (a)
(d) Van1 = 1 X∠(q1 − 60°); Van2 = 1 Y∠(q 2 − 60°)
3 3
Common Data for Questions 56 and 57:
(GATE 2006: 2 Marks) A generator feeds power to an infinite bus
through a double circuit transmission line. A
three phase fault occurs at the middle point of
Solution:  Given that Vab1 = X∠q1, Vab2 = Y∠q2. one of the lines. The infinite bus voltage is 1 pu,
The sequence for given star connected resistors can the transient internal voltage of the generator
be represented as shown in the following figure. We is 1.1 pu and the equivalent transfer admittance
know that in positive sequence, line voltage leads during fault is 0.8 pu. The 100 MVA generator
phase voltage by 30° and converse is true for nega- has an inertia constant of 5 MJ/MVA and it
tive sequence. Therefore was delivering 1.0 pu power prior of the fault
with rotor power angle of 30°. The system fre-
Van1 = X∠q1 − 30° and Van2 = Y∠q 2 + 30° quency is 50 Hz.

Chapter 5 solved question paper.indd 462 3/23/2016 12:51:15 PM


SOLVED GATE PREVIOUS YEARS’ QUESTIONS        463

56. The initial accelerating power (in pu) will be 58. The post fault voltages at buses 1 and 3 in per unit,
respectively, are
(a) 1.0 (b) 0.6 (c) 0.56 (d) 0.4
(GATE 2006: 2 Marks) (a) 0.24, 0.63 (b) 0.31, 0.76
(c) 0.33, 0.67 (d) 0.67, 0.33
Solution:  Given that V = 1 pu; E = 1.1 pu, PS = (GATE 2006: 2 Marks)
1.0 pu; 1/X = 0.8 pu; power angle d = 30° and f =
50 Hz. We know that accelerating power is given by Solution:  The pre‑fault voltage is

Pa = Ps − Pe é V1 ù é1Ð0°ù
1. 1 × 1 = êV2 ú = ê1Ð0°ú
ê ú ê
Vbus
= 1 −
EV
sin d = 1 − sin 30° ú
X 1/0.8 êëV3 úû ë1Ð0°û
= 0.56 pu
The fault occurs at bus 2, so r = 2. For solid fault
Ans. (c) Zf = 0. Then the fault current is

57. If the initial accelerating power is X pu, the initial V r(0) V2(0)
If = =
acceleration in elect deg/s2, and the inertia con- Zrr + Zf Z22 + Zf
stant in MJ-s/elect degree respectively, will be
1Ð0°
(a) 31.4 X, 18 (b) 1800 X, 0.056 = = −4 j
j0.24
(c) X/1800, 0.056 (d) X/31.4, 18
(GATE 2006: 2 Marks) Let the pre‑fault voltage be Vi (0), then post fault
voltage
Solution:  Let Pa be the initial acceleration power,
then acceleration is given by Vi( f ) = Vio (0) − Z ir × I f

Pa X(pu) × S Using values from the given impedance matrix


a= =
M SH /180f
V1(f ) = V1(0) − Z12 I f = 1Ð0° − j 0.08 (−j4)
where H is normalised inertia constant and S is gen-
erator MVA rating. Substituting values, we have = 0.68 pu
180 × 50 × X × S V3(f ) = V3 (0) − Z32 I f = 1Ð0° − j 0.16 (−j4)
a=
S ×5
= 1800 × deg/ s2 = 0.36 pu
1
Intertial constant = = 0.056 Ans. (d)
18
Ans. (b) 59. The per unit fault feeds from generators connected
to buses 1 and 2, respectively, are
Linked Answer Questions 58 and 59: For
a power system the admittance and impedance (a) 1.20, 2.51 (b) 1.55, 2.61
matrices for the fault studies are as follows. (c) 1.66, 2.50 (d) 5.00, 2.50
(GATE 2006: 2 Marks)
é−j8.75 j1.25 j2.50 ù
ê ú Solution:  The post fault voltage current is given by
Y bus = ê−j1.25 - 6.25 j2.50 ú
ê ú I g1(f ) = [Eg1 − V1(f ) ]Y10
ë - j2.50 - j2.50 - j5.00 û (i)
é j0.16 j0.08 j0.12 ù I g2(f ) = [Eg 2 − V2(f ) ]Y20 (ii)
ê ú
Zbus = ê j0.08 j0.24 j0.16 ú
ê ú From the given admittance matrix
ë j0.12 j0.16 j0.34 û
Y10 = Y11 + Y12 + Y13
The pre-fault voltages are 1.0 pu at all the buses.
The system was unloaded prior to the fault. A solid = −j8.75 + j1.25 + j2.5 = −j5
three phase fault takes place at bus 2.

Chapter 5 solved question paper.indd 463 3/23/2016 12:51:28 PM


464        CHAPTER 5:  POWER SYSTEMS

Y20 = Y21 + Y22 + Y23 61. Two regional systems, each having several syn-
chronous generators and loads are interconnected
= −j1.25 + j6.25 + j2.5 = −j2.5 by an AC line and a HVDC link as shown in the
following figure. Which of the following statements
Substituting values in Eqs. (i) and (ii), we get is true in the steady state?
I g1(f ) = [1∠0° − 0.68](−j5) = −j1.65
PDC
I g2(f ) = [1∠0° − 0)](−j2.5) = −j2.5

Ans. (c)

60. The incremental cost curves in Rs/, MW hour for HVDC line
Region 1 Region 2
two generators supplying a common load of 700 MW
are shown in the following figures. The maximum AC line
and minimum generation limits are also indicated.
The optimum generation schedule is: PAC
Generator A Generator B

800 (a) Both regions need not have the same frequency.
650 (b) The total power flow between the regions (PAC
600 + PDC) can be changed by controlling the
450 HVDC converters alone.
(c) The power sharing between the AC line and the
P HVDC link canP be changed by controlling the
200 MW 450 MW 150 MW HVDC converters alone.
400 MW
Incremental cost Rs/MWh (d) The direction of power flow in the HVDC link
(PDC) cannot be reversed.
Generator A Generator B
(GATE 2007: 1 Mark)
800
650  Ans. (c)
62. Consider a bundled conductor of an overhead line,
consisting of three identical sub-conductors placed
at the corners of an equilateral triangle as shown
P P in the figure. If we neglect the charges on the other
200 MW 450 MW 150 MW 400 MW phase conductors and ground, and assume that
cremental cost Rs/MWh spacing between sub-conductors is much larger
(a) Generator A: 400 MW, Generator B: 300 MW than their radius, the maximum electric field inten-
(b) Generator A: 350 MW, Generator B: 350 MW sity is experienced at
(c) Generator A: 450MW, Generator B: 250 MW
(d) Generator A: 425MW, Generator B: 275 MW Y

(GATE 2007: 1 Mark)


Solution:  Given that PA + PB = 700 MW X

Maximum incremental cost for, Z


Generator A = 600 at 450 MW. W
Generator B = 800 at 400 MW.
Minimum cost for
Generator B = 650 at 150 MW.
Generator A = 450 at 200 MW. (a) Point X (b) Point Y
(c) Point Z (d) Point W
The maximum cost of generator A is less than min-
imum cost of generator B, so (GATE 2007: 1 Mark)
PB = 700 − 450 = 250 MW. Solution:  The electric field intensity at the given
Ans. (d) points is

Chapter 5 solved question paper.indd 464 3/23/2016 12:51:34 PM


SOLVED GATE PREVIOUS YEARS’ QUESTIONS        465

E1 given by 0.045 pu and 1.2 pu, respectively. If the


velocity of wave propagation is 3 × 105km/s, then
E3 E2 E3 E2 approximate length of the line is
X (a) 122 km (b) 185 km
(c) 222 km (d) 272 km
E1
(GATE 2007: 2 Marks)

X Y Solution:  Given that X = 0.045 pu, then induc-


tance is

E1 E2 0.045 0.045
L= =
E2 E3 2pf 2p × 50
E3 Given that susceptance = 1.2 pu, then capaci-
E1
tance is
Z W
1 1
Thus, the maximum electric field intensity will be C= ×
observed at point Y as there will be minimum can- 2p × 50 1.2
cellation of the electric fields generated due to the
The velocity of wave propagation v = 3 × 105 km/s.
three points.
Ans. (b) We now that

Þ l = v LC
l
63. The following figure shows a three phase self-com- v=
mutated voltage source converter connected to a LC
power system. The converter’s DC bus capacitor is
marked as C in the figure. The circuit is initially Therefore, substituting values, length is
operating in steady state with d = 0 and the capac-
0.045
itor DC voltage is equal to VDC0. You may neg1ect l = 3 × 105 = 185 km
all losses and harmonics. What action should be 2p × 50 × 2p × 50 × 1.2
taken to increase the capacitor DC voltage slowly Ans. (b)
to a new steady state value?
65. Consider the protection system shown in the follow-
Three phase ing figure. The circuit breakers, numbered from 1 to
C voltage source 7 are of identical type. A single line to ground fault
converter with zero fault impedance occurs at the midpoint of
E–d E–0 the line (at point F), but circuit breaker 4 fails to
(a) Make d positive and maintain it at a positive
operate (“stuck breaker”). If the relays are coordi-
nated correctly, a valid sequence of circuit breaker
value
(b) Make d positive and return it to its original
operations is
value (a) 1, 2, 6, 7, 3, 5 (b) 1, 2, 5, 6, 7, 3
(c) Make d negative and maintain it at a negative (c) 5, 6, 7, 3, 1, 2 (d) 5, 1, 2, 3, 6, 7
value
(d) Make d negative and return it to its original 1 3
Transmission
value
line
Bus C
(GATE 2007: 2 Marks)
2 4 6
Stuck
Solution:  The value of the capacitor DC volt-
breaker
age can be increased to a new steady state value F
by making d negative and returning to its original Bus A 5 7
value.
Transmission
Ans. (d)
line
Bus B
64. The total reactance and total susceptance of a loss-
less overhead EHV line, operating at 50 Hz, are (GATE 2007: 2 Marks)

Chapter 5 solved question paper.indd 465 3/23/2016 12:51:40 PM


466        CHAPTER 5:  POWER SYSTEMS

Solution:  Given that the fault F occurs at the mid- 1.0–0


point and failure occurs at circuit breaker 4. So the cor-
x
rect sequence of operation is 5-6-7-3-2-1. This is based
on the back-up mechanism for protection, wherein x¢ = 0.1 pu
relays are so coordinated that relay in the section x
where fault occurs operate first and then the relays in
1.0–d
the adjoining sections, in case the first relay fails.
Ans. (c)
δ One line trips
66. A three-phase balanced star connected voltage
source with frequency w rad/s is connected to a star 130°
connected balanced load which is purely inductive.
The instantaneous line currents and phase to neu-
tral voltages are denoted by (ia, ib, ic) and (van, vbn,
vcn) respectively and their RMS values are denoted
by V and I. If t

é 1 1 ù
ê0 − ú
(a) 0.87 (b) 0.74
ê 3 3 ú éi ù (c) 0.67 (d) 0.54
R = éëvan vbn vcn ùû ê−
1 1 úê aú
ê 3 ú êi ú
0 ib (GATE 2007: 2 Marks)
ú êë c úû
3
ê 1
ê ú
1
− 0 Solution:  Let P1e be the electric power before trip-
ëê 3 3 úû ping of circuit and P2e be the electric power after
tripping. We know that.
then the magnitude of R is
(a) 3VI (b) VI EV EV EV
(c) 0.7 VI (d) 0 P = sin d ; Pmax = ; P2max =
X X X2
(GATE 2007: 2 Marks)
where X2 = (0.1 + X )pu
Solution:  We have from the given matrix
Since the system does not lose synchronism subse-
van vbn vcn quent to tripping,
R= (ib − ic ) + (ic − ia ) + (ia − ib )
3 3 3 P2e = Pm

sin d 2 = Pm
Setting EV
ib − ic X2
i −= iIc and van = V 1.0 × 1.0
3b = I and van = V sin 130° = 1
3
R = 3VI X2
We have, R = 3VI X2 = sin 130° = 0.766
Ans. (a)
We have
67. Consider a synchronous generator connected to an
X2 = 0.1 + X
infinite bus by two identical parallel transmission
lines. The transient reactance x of the generator is X = 0.766 − 0.1 = 0.666
0.1 pu and the mechanical power input to it is con-
stant at 1.0 pu. Due to some previous disturbance, Ans. (c)
the rotor angle (d) is undergoing an undamped 68. Suppose we define a sequence transformation
oscillation, with the maximum value of d(t) equal to between “a-b-c’’ and “p-n-o’’ variables as follows:
130°. One of the parallel lines trips due to relay mal-
operation at an instant when d(t) = 130° as shown éfa ù é1 1 1ù éfp ù
in the following figure. The maximum value of the êf ú = k êa 2 a 1ú êf ú
per unit line reactance, x, such that the system does ê bú ê úê nú
not lose synchronism subsequent to this tripping is êëfc úû êëa a 2 1úû êëfo úû

Chapter 5 solved question paper.indd 466 3/23/2016 12:51:50 PM


SOLVED GATE PREVIOUS YEARS’ QUESTIONS        467

2p Therefore,
j
where, a = e 3 and k = constant.
Now, if it is given that: VP = kA Z ′ IS 

éVp ù é0.5 0 0 ù éip ù  A−1 


êV ú = ê 0 = kA Z ′  ′ −1
 IP = AZ A IP
0 ú êin ú and  
ê nú ê úê ú
0 . 5 k
êëVo úû ë0 0 2.0 û êëio úû
é 1 0.5 0.5 ù
éV a ù éia ù V P = ê 0. 5 1 0. 5 ú I P
êV ú = Z ê i ú ê ú
ê bú ê bú ë 0. 5 0. 5 1 û
êëVc úû êëic úû
then So,
é1.0 0.5 0.75 ù é 1 0. 5 0. 5 ù
(a) Z = ê0.75 1.0 0.5 ú Z = ê 0. 5 1 0. 5 ú
ê ú ê ú
ë 0.5 0.75 1.0 û ë 0. 5 0. 5 1 û
é1.0 0.5 0.5 ù Ans. (b)
(b) Z = ê0.5 1.0 0.5 ú
ê ú
ë 0. 5 0. 5 1. 0 û 69. Consider the two power systems shown in
é1.0 0.75 0.5 ù
Figure (a) below, which are initially not intercon-
(c) Z = 3k2 ê0.5 1.0 0.75 ú
nected, and are operating in steady state at the
ê ú same frequency. Separate load flow solutions are
ë 0.75 0.5 1.0 û computed individually for the two systems, corre-
é 1. 0 − 0 . 5 − 0. 5 ù sponding to this scenario. The bus voltage phasors
k2 ê
(d) Z = − 0. 5 1. 0 − 0. 5 ú so obtained are indicated on Figure (a). These two
3 ê−0.5 −0.5 ú
ë 1.0 û isolated systems are now interconnected by a short
transmission line as shown in Figure (b), and it is
(GATE 2007: 2 Marks) found that P1 = P2 = Q1 = Q2 = 0
Solution:  We have fP = kAfS
Therefore from the given matrix equation

éfa ù éfp ù é1 1 1ù
fP = fb , fS = êfn ú and A =
ê ú êa 2 a 1ú 1.02–0° 1.02–-20° 1.0–-5° 1.02– 0°
ê ú ê ú ê ú
êë fc úû êë fo úû êë a a 2 1úû

Similarly,
X Y
VP = kAVS and IP = kAIS 1.02–10° 1.02–15°

Also, VS = Z ′ IS 
Then from the given matrix equation, we have (a)

0.5 0 0 
Z ′ =  0 0.5 0 
 
 0 0 2 P1, Q1 P2, Q2
 1 a a2
−1 1 
A =  1 a 2 a 
3 1 1 1 
  X Y
 1 a a 2  0.5 0 0  1 1 1
1   
VP =  1 a 2 a   0 0.5 0  a 2 a 2 1 IP
3 1 1  
1   0 0 2   a a 2 1
    (b)

Chapter 5 solved question paper.indd 467 3/23/2016 12:52:07 PM


468        CHAPTER 5:  POWER SYSTEMS

The bus voltage phase angular difference between Thus,


generator bus X and generator bus Y after the
XL (or 2pfL) = = = 23 W
V 230
interconnection is
IL 10
(a) 10° (b) 25°
(c) −30° (d) 30° L=
23
=
23
= 72.95 mH
2pf 2p × 50
(GATE 2007: 2 Marks)
Therefore, L = 72.95 mH in phase B and C =139 mF
Solution:  The two power systems are connected by
in phase C.
short transmission lines as P1 = P2 = Q1 = Q2 = 0
Ans. (b)
The phase angle difference for no energy transfer is
71. An isolated 50 Hz synchronous generator is rated
q = 30° − 20° = 10° at 15 MW which is also the maximum continu-
Ans. (a) ous power limit of its prime mover. It is equipped
with a speed governor with 5% droop. Initially, the
70. A 230 V (phase), 50 Hz, three-phase, 4-wire system generator is feeding three loads of 4 MW each at
has a phase sequence ABC. A unity power-factor 50 Hz. One of these loads is programmed to trip
load of 4 kW is connected between phase A and permanently if the frequency falls below 48 Hz. If
neutral N. It is desired to achieve zero neutral cur- an additional load of 3.5 MW is connected, then
rent through the use of a pure inductor and a pure frequency will settle down to
capacitor in the other two phases. The value of (a) 49.417 Hz (b) 49.917 Hz
inductor and capacitor is (c) 50.083 Hz (d) 50.583 Hz
(a) 72.95 mH in phase C and 139.02 mF in phase B (GATE 2007: 2 Marks)
(b) 72.95 mH in phase B and 139.02 mF in phase C
(c) 42.12 mH in phase C and 240.79 mF in phase B Solution:  Change in frequency w.r.t to power,
(d) 42.12 mH in phase B and 240.79 mF in phase C
Drop out frequency
(GATE 2007: 2 Marks) ∆f = × Change in power
Rated power
5
Solution:  Given that V = 230 V, P = 4 kW (load), = × 3.5 = 1.16
f = 50 Hz, IN = 0 15
Since, IN = 0, we have The change in terms of actual frequency

Df = 1.16 ×
I A + I B + IC = 0  (i) 50
= 0.58 Hz
100
We know that P = VI and therefore,
Therefore, the overall system frequency = 50 - 0.58
P 4 × 103 = 49.42 Hz.
IA = = = 17.39A
V 230 Ans. (a)
From Eq. (i), 72. A two machine power system is shown in the fol-
lowing figure. Transmission line XY has positive
I A = −IB − IC sequence impedance of Z1 Ω and zero sequence
impedance of Z0 Ω.
 3 3
I A = −  IB . + IC .  X Y
 2 2 
F
I A = 3IB = 3IC ∵ IB  IC 
An `a’ phase to ground fault with zero fault imped-
17.39 V 230 ance occurs at the centre of the transmission line.
IB = = 10 A ; XC = = = 23 Ω Bus voltage at X and line current from X to F
3 IC 10
for the phase `a’, are given by Va Volts and Ia
Amperes, respectively. Then, the impedance mea-
1 1 1
XC = ;C = = = 139 µF sured by the ground distance relay located at the
2pfC 2pfXc 2p × 50 × 23 terminal X of line XY will be given by

Chapter 5 solved question paper.indd 468 3/23/2016 12:52:20 PM


SOLVED GATE PREVIOUS YEARS’ QUESTIONS        469

Z Z Shunt capacitance of the line can be neglected. If


(a)  1 W (b)  0 W the line has positive sequence impedance of 15 Ω
and zero sequence impedance of 48 Ω, then the
2 2
(Z + Z1 ) V values of Zs and Zm will be
(c)  0 W (d)  a Ω
2 Ia (a) Zs = 31.5 Ω; Zm = 16.5 Ω
(b) Zs = 26 Ω; Zm = 11 Ω
(GATE 2008: 1 Mark)
(c) Zs = 16.5 Ω; Zm = 31.5 Ω
Solution:  Let phase voltage be Va and phase cur- (d) Zs = 11 Ω; Zm = 26 Ω
rent be Ia.
(GATE 2008: 1 Mark)
Impedance measured from ground distance
Bus voltage V Solution:  We know that,
= = aΩ
Current from phase a Ia
Ans. (d) é DVa ù é(Zs − Zm ) 0 0 ù é Ia ù
ê ú ê úê ú
ê DVb ú = ê 0 (Zs − Zm ) 0 ú êIb ú
73. An extra high voltage transmission line of length ê ú ê úê ú
300 km can be approximated by a lossless line êë DVc úû êë 0 0 (Zs + 2Zm )ûú êë I c úû
having propagation constant. Then percentage ratio
of line length to wavelength will be given by
Zero sequence impedance
(a) 24.24% (b) 12.12%
(c) 19.05% (d) 6.06% Zs + 2Zm = 48  (i)
(GATE 2008: 1 Mark)
Positive sequence impedance = Negative sequence
Solution:  Given that length l = 300 km,
b = 0.00127 rad/km. We know that I m = Zs − Zm = 15  (ii)

2p 2p Solving Eqs. (i) and (ii), we get


l= = = 4947.4 km
b 0.00217 Zs = 26 and Zm = 11
l 300
%= × 100 = 6.06% Ans. (b)
l 4947.4
75. Voltage phasors at the two terminals of a trans-
Ans. (d) mission line of length 70 km have a magni-
tude of 1.0 per unit but are 180° out of phase.
74. A 3-phase transmission line is shown in the follow-
Assuming that the maximum load current in
ing figure.
the line is 1/5 th of minimum three-phase fault
∆ Va
current, which one of the following transmission
line protection schemes will NOT pick up for
Ia this condition?
(a) Distance protection using mho relays with
∆ Vb Zone-1 set to 80% of the line impedance
Ib (b) Directional over current protection set to pick
up at 1.25 times the maximum load current
∆ Vc (c) Pilot relaying system with directional compari-
son scheme
Ic (d) Pilot relaying system with segregated phase
Voltage drop across the transmission line is given comparison scheme
by the following equation: (GATE 2008: 2 Marks)
é DVa ù éZs Zm Zm ù é I a ù
ê DV ú = êZ Zm ú ê I b ú
Solution:  Distance protection using mho relays
ê bú ê m úê ú
Zs with Zone‑1 set to 80% of the line impedance.
êë DVc úû êëZm Zm Zs úû êëI c úû
Ans. (a)

Chapter 5 solved question paper.indd 469 3/23/2016 12:52:34 PM


470        CHAPTER 5:  POWER SYSTEMS

76. A lossless transmission line having surge imped- Therefore, from Eq. (ii)
ance loading (SIL) of 2280 MW is provided with a
uniformly distributed series capacitive compensa- 1 + 0.11PG1 = 3 + 0.06PG2
tion of 30%. Then, SIL of the compensated trans-
0.11PG1 − 0.06PG2 = 2  (iii)
mission line will be
(a) 1835 MW (b) 2280 MW Solving Eqs. (i) and (iii), we get PG1 = 100 MW
(c) 2725 MW (d) 3257 MW and PG2 = 150 MW
Ans. (c)
(GATE 2008: 2 Marks)
78. A lossless single machine infinite bus power system
Solution:  SIL of the compensated transmission is shown in the following figure
line will be 2280 MW as there will be no effect
of capacitive compensation on the active power of 1.0–d pu 1.0–0 pu
transmission line.
Ans. (b) 1.0 pu

77. A lossless power system has to serve a load of The synchronous generator transfers 1.0 per unit of
250 MW. There are two generators (G1 and G2) in power to the infinite bus. The critical clearing time
the system with cost curves C1 and C2, respectively, of circuit breaker is 0.28 s. If another identical syn-
defined as follows: chronous generator is connected in parallel to the
existing generator and each generator is scheduled
C1(PG1 ) = PG1 + 0.055 × PG12
to supply 0.5 per unit of power. Then the critical
clearing time of the circuit breaker will
C2 (PG2 ) = 3PG2 + 0.03 × PG22
(a) reduce to 0.14 s
(b) reduce but will be more than 0.14 s
where PG1 and PG2 are the MW injections from (c) remain constant at 0.28 s
generators G1 and G2, respectively. Then the mini- (d) increase beyond 0.28 s
mum cost dispatch will be
(GATE 2008: 2 Marks)
(a) PG1 = 250 MW; PG2 = 0 MW
Solution:  When the generators are connected in
(b) PG1 = 150 MW; PG2 = 100 MW
parallel and supplied 0.5 pu of power, there will be
(c) PG1 = 100 MW; PG2 = 150 MW increase in the current. Hence, the clearance time
(d) PG1 = 0 MW; PG2 = 250 MW will be reduced, not less than 0.14 seconds, so as to
(GATE 2008: 2 Marks) maintain transient stability.
Ans. (b)

Solution:  Given that 79. Single line diagram of a 4-bus single source dis-
tribution system is shown in the following figure.
C1(PG1 ) = PG1 + 0.055 × PG12 Branches e1, e2, e3 and e4 have equal impedances.
The load current values indicated in the figure are
C2 (PG2 ) = 3PG2 + 0.03PG22 and in per unit.
PG1 + PG2 = 250 MW (i)

Then,

dC1 e1 e2
= 1 + 0.11PG1 and
dPG1

dC2
= 3 + 0.06PG2  (ii) e3 e4
dPG2
1+j0 5+j0
dC1 dC2
For lossless system, =
dPG1 dPG2 2+j0

Chapter 5 solved question paper.indd 470 3/23/2016 12:52:43 PM


SOLVED GATE PREVIOUS YEARS’ QUESTIONS        471

Distribution company’s policy requires radial reveals that decaying DC offset current is negative
system operation with minimum loss. This can be and in magnitude at its maximum initial value.
achieved by opening of the branch Assume that the negative sequence impedances
are equal to positive sequence impedances, and the
(a) e1 (b) e2
zero sequence impedances are three times positive
(c) e3 (d) e4
sequence impedances.
(GATE 2008: 2 Marks)
80. The instant (t0) of the fault will be
Solution:  We know that line loss is given by (a) 4.682 ms (b) 9.667 ms
(c) 14.667 ms (d) 19.667 ms
Line loss = S I 2R
where R is the impedance. (GATE 2008: 2 Marks)
Consider the case for removal for each of the branch:
(i) Without e1 ; losses Solution:  Given that

= (1) R + (1 + 2) R + (1 + 2 + 5) R = 74 R
2 2 2
v(t) = Vm cos w t
(ii) Without e2 ; loss
= 52 R + (5 + 2)2 R + (5 + 2 + 1)2 R = 138 R
For symmetrical three phase fault

i(t) = Ae−(R / L)t +


2Vm
(iii) Without e3 ; loss = (1)2 R + 22 R + (5 + 2)2 R cos(wt − a )
Z
= 54 R

(iv) Without e4 ; loss = 22 R + (2 + 1)2 R + 52 R = Let fault occur at t = t0


38 R
Therefore i (t) = 0 and so
So, minimal loss can be achieved by removing e4
Ae−(R / L)t0 +
2Vm
Ans. (d) cos(w t0 − a ) = 0
Z
Common Data for Questions 80 to 82: Consider
a power system shown in the following figure. Maximum value of DC offset current,

X Y
Ae−(R / L)t0 = −
2Vm
Vs1 Vs2 cos(w t0 − a )
Zs1 ZL Zs2 Z

For negative maximum, w t0 − a = 0 ⇒ t0 = a /w


IX F If

Given that: Vs1 = Vs2 = 1.0 + j0.0 pu It is given that, Z = 0.004 + j0.04 = 0.0402∠84.29°
The positive sequence impedance are: Therefore,
Zs1 = Za2 = 0.001 + j0.01 pu a = 84.29° or 1.471 rad
and ZL = 0.006 + j0.06 pu Thus
Three-phase base MVA = 100
Voltage base = 400 kV (Line to line)
1.471
t0 = = 0.00468 s = 4.68 ms
Nominal system frequency = 50 Hz 2p × f0
The reference voltage for phase a is defined as Ans. (a)
v(t) = Vmcos(wt).
81. The rms value of the AC component of fault cur-
A symmetrical three phase fault occurs at centre rent (IX) will be
of the line, that is, point F at time t0. The posi-
tive sequence impedance from source S1 to point (a) 3.59 kA (b) 5.07 kA
F  equals 0.004 + j0.04 pu. The waveform cor- (c) 7.18 kA (d) 10.15 kA
responding to phase a fault current from bus X (GATE 2008: 2 Marks)

Chapter 5 solved question paper.indd 471 3/23/2016 12:53:07 PM


472        CHAPTER 5:  POWER SYSTEMS

Solution:  Positive sequence impedance, the base load power plants are
(a) (i) and (ii) (b) (ii) and (iii)
= 0.0201Ð84.29°
Z
Z1 = (c) (i),(ii) and (iv) (d) (i), (ii) and (iv)
2
(GATE 2009: 1 Mark)
For three phase fault Ans. (a)
84. For a fixed value of complex power flow in a trans-
Z1 = Z2 = Z0
mission line having a sending end voltage V, the
real power loss will be proportional to
V 1Ð0°
I f (pu) = = = 49.8 (b) V2
Z1 0.0201Ð84.29° (a) V
1 1
Therefore, rms value of the AC component of fault (c)  2 (d) 
V V
current is
(GATE 2009: 1 Mark)
100 100
I X = If × = 49.8 × = 7.18 kA Solution:  We know that
3 × 400 3 × 400
Ans. (c) S = P + jQ = VI [cos f + j sin f ] = VIe jf
S
82. Instead of the three phase fault, if a single line to ⇒ I=
ground fault occurs on phase a at point F with zero Ie jf
fault impedance, then the rms value of the AC com-
ponent of fault current (IX ) for phase a will be Also, power loss

æ S ö
2
(a) 4.97 pu (b) 7.0 pu S 2R
Ploss = I 2R = ç jf ÷ R = 2 jf 2
(c) 14.93 pu (d) 29.85 pu è Ve ø V ×(e )
(GATE 2008: 2 Marks) Thus
1
Solution:  Line to ground fault is at phase a, PL ∝
therefore, V2

when S 2R e jf is constant
2

Z1 = Z2 = 0.0201∠84.29°
Z0 = 3Z1 = 0.0603∠84.29°
Ans. (c)
85. For the Y-bus matrix of a 4-bus system given in
I per unit, the buses having shunt elements are
Ia 0 = I a1 = I a 2 = a
3
é −5 2 2. 5 0ù
I a1 =
1. 0
= 9.95 pu ê 2 −10 2.5 4ú
Y bus = jê ú
0.0201 + 0.0201 + 0.0603
ê 2 . 5 2 . 5 − 9 4ú
Fault current, êë 0 4 4 −8 ûú

I f = Ia = 3Ia1 = 29.85 pu (a) 3 and 4 (b) 2 and 3


(c) 1 and 2 (d) 1, 2 and 4
Therefore, the rms value of the AC component of
(GATE 2009: 2 Marks)
fault current for phase `a’ is
Solution:  In general,
100
I X = 29.85 ×
 Y11 Y14 
= 4.9 kA
3 × 400 Y12 Y13
 
Ans. (a)  Y21 Y22 Y23 Y24 
Y bus = 
 Y31 Y32 Y33 Y34 
83. Out of the following plant categories Y Y 44 
 41 Y 42 Y 43
(i) Nuclear
(ii) Run-of-river Y11 = Y10 + Y12 + Y13 + Y14 = −5 j

(iii) Pump Storage
(iv) Diesel Y22 = Y20 + Y21 + Y23 + Y24 = −10 j

Chapter 5 solved question paper.indd 472 3/23/2016 12:53:27 PM


SOLVED GATE PREVIOUS YEARS’ QUESTIONS        473

Y33 = Y30 + Y31 + Y32 + Y34 = −9 j Codes:



A B C D
Y12 = Y21 = −Y12 = 2 j
(a) 2 3 4 1
Y13 = Y31 = −Y13 = 2.5 j (b) 2 4 3 1
(c) 4 3 1 2
Y14 = Y 41 = −Y 41 = 0 j (d) 4 1 3 2

Y24 = Y 42 = −Y24 = 4 j (GATE 2009: 2 Marks)

Therefore, Solution:
Y10 = Y11 − Y12 − Y13 − Y14 Elements Uses
= −5 j + 2 j + 2.5 j + 0 j = −0.5 j
Shunt capacitor Improving PF
Y20 = Y22 − Y12 − Y23 − Y24
Series reactors Reduce the current ripples
= −10 j + 2 j + 2.5 j + 4 j = −1.5 j
Series capacitor Increase power flow in line
Similarly Shunt reactors Reduce the Ferranti effect
Y30 = 0 Y 40 = 0
Ans. (b)
87. Match the items in List-I with the items in List-II
S1 S2 and select the correct answer using the codes given
Y10 Y20
below the lists.
1 2
Y12 List I List II
(Type of (Type of distance
Y24 transmission line) relay preferred)
Y23 A. Short line 1. Ohm relay
Y13 B. Medium line 2. Reactance relay
C. Long 3. Mho relay
4 3
Codes:
Y43
S4 S3 A B C
(a) 2 3 1
Therefore, branches (1) and (2) behave like shunt (b) 3 2 1
element. (c) 1 2 3
Ans. (c) (d) 1 3 2
86. Match the items in List I with the items in List II (GATE 2009: 2 Marks)
and select the correct answer using the codes given
below the lists. Solution:

List I List II Transmission line Distance relay


(To) (Use)
Short Ohm reactance
A. Improve power 1. Shunt reactor
factor Medium Reactance relay
B. Reduce the current 2. Shunt capacitor Long Mho relay
ripples
Ans. (c)
C. Increase the power 3. Series capacitor
flow in line
88. Three generators are feeding a load of 100 MW.
D. Reduce the Ferranti 4. Series reactor The details of the generators are given below.
effect

Chapter 5 solved question paper.indd 473 3/23/2016 12:53:38 PM


474        CHAPTER 5:  POWER SYSTEMS

Substituting values in Eq. (ii), we get


Rating Efficiency Regulation
(MW) (%) (pu) on 1  2p × 3000 
KE = × 27.5 × 103 ×  
100 MVA 2  60
base
= 1357.07 MJ
Generator-1 100 20 0.02
Generator-2 100 30 0.04 Therefore, from Eq. (i), inertia constant is

Generator-3 100 40 0.03 1357.07


H= = 2.44 s
555.56
In the event of increased load power demand, which Ans. (a)
of the following will happen?
90. Power is transferred from system A to system B by
(a) All the generators will share equal power an HVDC link as shown in the figure given below.
(b) Generator-3 will share more power compared to If the voltages VAB and VCD are as indicated in the
Generator-1 figure, and I > 0, then
(c) Generator-1 will share more power compared to
Generator-2 AC Power flow AC
(d) Generator-2 will share more power compared to system A C system
Generator-3
A VAB i VCD B
(GATE 2009: 2 Marks)
B D
Rectifier Inverter
Solution:  Generator having better regulation share
more power, for increased load power. (a) VAB < 0, VCD < 0, VAB > VCD
(b) VAB > 0, VCD > 0, VAB < VCD
Therefore, G-1 shares more power than G-2 (c) VAB > 0, VCD > 0, VAB > VCD
Ans. (c) (d) VAB > 0, VCD < 0
89. A 500 MW, 21 kV, 50 Hz, three-phase, 2-pole syn- (GATE 2010: 1 Mark)
chronous generator having a rated pf = 0.9 has a
moment of inertia of 27.5 × 103 kgm2. The inertia Solution:  From the given diagram, we understand
constant (H) will be that I > 0; rectifier output VAB > 0 and inverter
input VCD > 0. Therefore, I > 0 and VAB > VCD.
(a) 2.44 s (b) 2.71 s
(c) 4.88 s (d) 5.42 s Ans. (c)

(GATE 2009: 2 Marks) 91. A balanced three-phase voltage is applied to a


star-connected induction motor, the phase to neu-
Solution: Inertia constant tral voltage being V. The stator resistance, rotor
Kinetic energy storted resistance referred to the stator, stator leakage reac-
H= 
(i) (i) tance, rotor leakage reactance referred to the stator,
MVA rating of generator
and the magnetising reactance are denoted by rs, rr,
Rating xs, xr and Xm, respectively. The magnitude of the
P 500 MW starting current of the motor is given by
G= = = 555.56 MVA
cos f 0.9 V
(a) 
Therefore, kinetic energy stored (rs + rr ) + (xs + xr )2
2

V
1 æ 2pN ö (b) 
2
KE = mw 2 = M ç
1
÷ (ii) (rs + (xs + Xm )2
2
 (ii)
2 2 è 60 ø
V
(c) 
Here N is given in terms of frequency and number (rs + rr ) + (Xm + xr )2
2

of poles as
V
(d) 
120f 120 × 50 (rs + (Xm + xr )2
2
N = = = 3000 rpm
p 2 (GATE 2010: 1 Mark)
Ans. (a)

Chapter 5 solved question paper.indd 474 3/23/2016 12:53:50 PM


SOLVED GATE PREVIOUS YEARS’ QUESTIONS        475

92. Consider a step voltage wave of magnitude 1 pu Therefore, real power, P = 1035 cos 15° = 1000 W
travelling along a lossless transmission line that
terminates in a reactor. The voltage magnitude Reactive power, Q = 1035 sin 15° = 268 VAR
across the reactor at the instant the travelling Ans. (a)
wave reaches the reactor is
94. A three-phase, 33 kV oil circuit breaker is rated
A 1200 A, 2000 MVA, 3s. The symmetrical breaking
current is
Reactor (a) 1200 A (b) 3600 A
(c) 35 kA (d) 104.8 kA

(a) —l pu (b) 1 pu (GATE 2010: 1 Mark)


(c) 2 pu (d) 3 pu
Solution:  Symmetrical breaking current is given by,
(GATE 2010: 1 Mark)
MVA 2000
Ib = kA = = 34.9 kA
Solution:  From the circuit, we have 3 kV 3 × 33

V Ans. (c)
A
95. Consider a stator winding of an alternator with an
+ internal high-resistance ground fault. The cur-
VL rents under the fault condition are as shown in
− the following figure. The winding is protected
using a differential current scheme with current
transformers of ratio 400/5 A as shown in the fol-
lowing figure. The current through the operating
V + VL = 0 coil is
Therefore, Therefore,
VL = −V = −1 pu CT ratio 400/5 CT ratio 400/5
Ans. (a)
(200 + j0)A (250 + j0)A
93. Consider two buses connected by an impedance of
(0 +j5) Ω. The bus 1 voltage is 100 ∠30° V, and bus
2 voltage is 100 ∠0° V. The real and reactive power
supplied by bus 1, respectively, are
Operating coil
(a) 1000 W, 268 VAR
(b) −1000 W, −134 VAR
(c) 276.9 W, −56.7 VAR
(a) 0.1875 A (b) 0.2 A
(c) 0.375 A (d) 60 kA
(d) −276.9 W, 56.7 VAR
(GATE 2010: 1 Mark)
(GATE 2010: 1 Mark)
Solution:  Given that impedance = 5j Ω; Vbus1 = Solution:  Current through operating coil
100 ∠30° V and Vbus 2 =100 ∠0° V.
5
Therefore complex power is given by IOC1 = 220 × = 2.75
400
5
100Ð30° - 100Ð0° IOC2 = 250 × = 3.125
P + jQ = VI * = 100Ð30°
100Ð30°5-j 100Ð0° 400
P + jQ = VI * = 100Ð30°
= 2000Ð30° − 2000Ð60° 5 j Net operating coil current = 3.125 − 2.750 = 0.375 A
2000Ð
= 1035
= Ð15 − 2000Ð60°
30°°
= 1035Ð15° Ans. (c)

Chapter 5 solved question paper.indd 475 3/23/2016 12:54:02 PM


476        CHAPTER 5:  POWER SYSTEMS

96. The zero-sequence circuit of the three phase trans- Long transmission line Receiving
former shown in the given figure is end
R
r (a) The magnitude of terminal voltage decreases,
and the field current does not change.
b (b) The magnitude of terminal voltage increases,
and the field current does not change.
(c) The magnitude of terminal voltage increases,
Y and the field current increases.
B y (d) The magnitude of terminal voltage does not
change, and the field current decreases.
(a) R r (GATE 2010: 2 Marks)

Solution:  Due to Ferranti effect, the magnitude


G of terminal voltage does not change and the field
(b) R r current increases.
Ans. (d)

98. Consider a three-phase, 50 Hz, 11 kV distribution


G system. Each of the conductors is suspended by
(c) R r an insulator string having two identical porcelain
insulators. The self-capacitance of the insulator is
five times the shunt capacitance between the link
and the ground, as shown in the following figure.
G
The voltage across the two insulators are
(d) R r
5C e2

G 5C e1
C
(GATE 2010: 1 Mark)

Solution:  Three phase star connected transformer Conductor


with neutral grounded has infinite impedance to
zero sequence current. A delta connected circuit, (a) e1 = 3.74 kV, e2 = 2.61 kV
provided path for the zero sequence circuit to (b) e1 = 3.46 kV, e2 = 2.89 kV
flow through the line and it circulates within the (c) e1 = 6 0 kV, e2 = 4.23 kV
winding. (d) e1 = 5.5 kV, e2 = 5.5 kV
(GATE 2010: 2 Marks)
R r
Solution:  For the given distribution system

6C e2
G
Ans. (c) 5C
e1
97. A 50 Hz synchronous generator is initially con-
11 kV
nected to a long lossless transmission line which
is open circuited at the receiving end. With the 3
field voltage held constant, the generator is discon-
( 6C )
11
nected from the transmission line. Which of the fol-
11 6
lowing may be said about the steady state terminal e1 = 3 = × = 3.46 kV
voltage and field current of the generator? 6C + 5C 3 11
11 5
e2 = × = 2.89 kV
3 11

Chapter 5 solved question paper.indd 476 3/23/2016 12:54:14 PM


( 6C )
11
11 6
e1 = 3 = × = 3.46 kV SOLVED GATE PREVIOUS YEARS’ QUESTIONS        477
6C + 5C 3 11
11 5 Choose 25 kV as the base voltage at the generator
e2 = × = 2.89 kV G1, and 200 MVA as the MVA base. The imped-
3 11
ance diagram is
Ans. (b)
(a) 
99. Consider a three-core, three-phase, 50 Hz, 11 kV j0.27 j0.42 j0.27
cable whose conductors are denoted as R, Y and B
in the given figure. The inter-phase capacitance (C1)
between each pair of conductors is 0.2 μF and the j0.18 j0.18
capacitance (C2) between each line conductor and the
sheath is 0.4 μF. The per-phase charging current is G1 G2

C2
(b) 
j0.27 j0.62 j0.27
C1 R C1

j0.18 j0.18
B Y
C1
C2 C2 G1 G2

Outer sheath
(c) 
(a) 2.0 A (b) 2.4 A j0.27 j0.42 j0.27
(c) 2.7 A (d) 3.5 A
(GATE 2010: 2 Marks) j0.21 j0.21
Solution:  Given that C1 = 0.2 μF and C2 = 0.4 μF
Capacitance per phase is G1 G2
C = 3C1 + C2 = 3 × 0.2 + 0.4 = 1 mF
Charging current (d) 
j0.3 j0.42 j0.3
= VwC
V
IC =
XC
= V 2pf j0.21 j0.21

11 × 103
= × 2 × 3.14 × 50 × 10−6 G1 G2
3
= 2 A
Ans. (a) (GATE 2010: 2 Marks)

100. For the power system shown in the following figure, Solution:  For the given power system, using 25 kV
specifications of the components are following: as the base voltage at generator G1 and 200 MVA as
the MVA base, we have impedances for generators
G1: 25 kV, 100 MVA, X = 9% G1 and G2 as
G2: 25 kV, 100 MVA, X = 9%
MVA(new)  kV(old) 
2
T1: 25 kV/ 220 kV, 90 MVA X = 12%
XG1 = XG2 = Xold × ×
T2: 25 kV/25 kV, 90 MVA, X = 12% MVA(old)  kV(new)
Line 1:220 kV, X = 150 ohms
200  25 
2
= j0..9 × ×   = j0.18
T1 T2 100  25 
Line 1 Similarly impedances for the transformers T1 and
T2 are
Bus 1 Bus 2 200 æ 25 ö
2
G1 G2 XT1 = j0.12 × × ç ÷ = j0.27
90 è 25 ø

Chapter 5 solved question paper.indd 477 3/23/2016 12:54:26 PM


478        CHAPTER 5:  POWER SYSTEMS

(a) an alternator (b) a transformer


200 æ 25 ö
2
XT 2 = j0.12 × × ç ÷ = j0.27 (c) a transmission line (d) a bus bar
90 è 25 ø
(GATE 2011: 1 Mark)
The impedance for the transmission line is Ans. (a)
S
Xline(pu) = X(Ω) × base2 150 ×
220
= j0.62 103. For enhancing the power transmission in a long
(kV) (220)2 EHV transmission line, the most preferred method
So the impedance diagram is is to connect a
(a) series inductive compensator in the line.
j0.27 j0.62 j0.27
(b) shunt inductive compensator at the receiving end.
(c) series capacitive compensator in the line.
j0.18 XG1 XG2 j0.18 (d) shunt capacitive compensator at the sending end.

(GATE 2011: 1 Mark)


G1 G2
Solution:  For EHV transmission line,
Ans. (b)
P µ
1
101. A nuclear power station of 500 MW capacity is (XL − XC )
located at 300 km away from a load centre. Select
the most suitable power evacuation transmission Therefore, the most preferred method is to connect
configuration among the following options: a series capacitive compensator in the line.
(a) Ans. (c)
132 kV, 300 km double 104. A load centre of 120 MW derives power from two
circuit power stations connected by 220 kV transmission
Load lines of 25 km and 75 km as shown in the follow-
 centre ing figure. The three generators G1, G2 and G3
(b) are to 100 MW capacity each and have identi-
cal fuel cost characteristics. The minimum loss
Load generation schedule for supplying the 120 MW
centre load is
132 kV, 300 km single circuit with
40% series capacitor compensation
G2
(c) 25 km 75 km
G1
Load G3
centre 120 MW
 400 kV, 300 km single circuit
(d) (a) P1 = 80 MW + losses
400 kV, 300 km double P2 = 20 MW
circuit P3 = 20 MW
Load (b) P1 = 60 MW
 centre P2 = 30 MW + losses
(GATE 2011: 1 Marks) P3 = 30 MW
(c) P1 = 40 MW
Solution:  Double circuit transmission enables the
P2 = 40 MW
transfer of more power over a particular distance
and is hence more preferred at lower voltage. Hence P3 = 40 MW + losses
circuit shown in option (a) is most suitable. (d) P1 = 30 MW + losses
Ans. (a) P2 = 45 MW
P3 = 45 MW
102. A negative sequence relay is commonly used to
protect (GATE 2011: 2 Marks)

Chapter 5 solved question paper.indd 478 3/23/2016 12:54:36 PM


SOLVED GATE PREVIOUS YEARS’ QUESTIONS        479

Solution:  We know that:


é−15 5 0ù
Power loss µ length of transmission line (b) j ê 5 7.5 −12.5 ú
Power loss µ P2 ê ú
ë 0 −12.5 2.5 û
On comparing all the given options, based on this,
we have that minimum loss generation schedule is é 0.1 0.2 0ù
as given in option (a). (c) j ê0.2 0.12 −0.08 ú
ê ú
Alternately: ë 0 −0.08 0.10 û
All the three generators are identical (each 100 MW)
and have the same fuel characteristics. é−10 5 0ù
The total load is P = P1 + P2 + P3. (d) j ê 5 7.5 12.5 ú
ê ú
ë 0 12.5 −10 û
The generators G1 and G2 can be represented by
equivalent generator G23 and load by P23, so
P = P1 + P23. (GATE 2011: 2 Marks)
The length of the lines are L1 and 3L1, therefore if
resistance R12 = R, then R23 = 3R = 3R. Solution:  Number of bus = 3.
Power delivered by the generator is
 Y11 Y12 Y13 
P = 3VI cos f Þ I = Y = Y21 Y23 
P
3 cos fV  
Y22
Y31 Y32 Y33 
Therefore, 1 1
Y11 = + = −10 j − 5 j = −15 j
P ∝ I = kI (i) j 0.1 j 0.2
1 Ans. (b)
where is constant k.
3V cos f
Common Data for Questions 106 and 107:
Assuming that all generators are operating at Two generator units G1 and G2 are connected by
same power factor, total transmission loss can be 15 kV line with a bus at the mid-point as shown
expressed as below.

PL = I12R12 + I22R23 = R(I12 + 3I22 ) G2


25 km 75 km
= k2R(P12 + 3P22 ) [from Eq. (i)] G1
G3
Using this relation power losses can be worked out for 120 MW
each of the given options and minimum transmission
loss is with operating conditions given in option (a). G1 = 250 MVA, 15 kV, positive sequence reactance
X = 25% on its own base
Ans. (a) G2 = 100 MVA, 15 kV, positive sequence reactance
105. A three-bus network is shown in the following figure X = 10% on its own base
indicating the pu impedances of each element. L1 and L2 = 10 km, positive sequence reactance
X = 0.225 Ω/km
1 2 3 106. For the above system, the positive sequence diagram
with the pu values on the 100 MVA common base is
j0.2
j0.1 −j0.08 j0.1 (a)
1 3 2

j1.0 j1.0
The bus admittance matrix, Y-bus, of the network is j0.10 j0.10

é 0. 3 − 0. 2 0ù
(a) j ê−0.2 0.12 0.08 ú
ê ú
ë 0 0.08 0.02 û

Chapter 5 solved question paper.indd 479 3/23/2016 12:54:51 PM


480        CHAPTER 5:  POWER SYSTEMS

(b) 107. In the above system, the three-phase fault MVA at


3 the bus 3 is
1 2
(a) 82.55 MVA (b) 85.11 MVA
(c) 170.91 MVA (d) 181.82 MVA
j1.0 j1.0
j0.25 j0.25
 (GATE 2011: 2 Marks)
Solution:  For parallel connection, Thevenin’s fault
impedance is given by:
1.1 × 1.1
XTh = = 0.55
(c) 1.1 + 1.1
1 3 2
MVA(base) 100
Fault = = = 181.82 MVA
XTh 0.55
j2.25 j2.25
j0.10 j0.10
Ans. (d)
108. The bus admittance matrix of a three-bus three-
line system is
(d) é−13 10 5ù
Y = j ê 10 −18 10 ú
1 3 2 ê ú
ë 5 10 −13 û

j2.25 j2.25 If each transmission line between the two buses is


represented by an equivalent p-network, the mag-
j0.25 j0.10
nitude of the shunt susceptance of the line connect-
ing bus 1 and 2 is
(a) 4 (b) 2
(c) 1 (d) 0
(GATE 2011:2 Marks)
Solution:  From the bus admittance matrix,
Solution:  From the given values, we have
Y11 + (Y12 + Y line ) + Y13 = 0
XL1 = 0.225 × 10 = 2.25 W
−j13 + ( j10 + Y line ) + j5 = 0
XL2 = 0.225 × 10 = 2.25 W
Y line = −j2
Taking 100 MVA, 15 kV as base, we get the reac-
tance of generator G1 as The magnitude of Y line = +2
Ans. (b)
100 æ 15 ö
2
XG1 = 0.25 × × ç ÷ = 0.1 pu 109. A two-phase load draws the following phase currents:
250 è 15 ø
i1(t) = I m sin(wt − f1),
The reactance for the line is
i2 (t) = I m cos(wt − f2)
Sbase
XL1 = X ×
(KVbase )2 These currents are balanced if f1 is equal to
100 (a) −f2 (b) f2
= 2.25 × 2
= 1 pu = XL2
æp ö æp ö
(15)
(c) ç − f2 ÷ (d) ç + f2 ÷
Also è2 ø è2 ø
(GATE 2012: 1 Mark)
152
Zbase = = 2.25 W
100 Solution:  Given that
100
= 0.1 ×
i1(t) = I m sin(wt − f1)
XG2 = 0.1 pu
100
Ans. (a) i2 (t) = I m cos(wt − f2)

Chapter 5 solved question paper.indd 480 3/23/2016 12:55:09 PM


SOLVED GATE PREVIOUS YEARS’ QUESTIONS        481

Also cos(f − 90°) = sin f (a) PG1 = 20, PG2 = 22 (b) PG1 = 22, PG2 = 20
Therefore, (c) PG1 = 20, PG2 = 20 (d) PG1 = 0, PG2 = 40

i1(t) = I m cos(wt − f1 − 90°) (GATE 2012: 1 Mark)

i2 (t) = I m cos(wt − f2)  (i) Solution:  Penalty factor,


1
In the phasor form L1 = (Plant G1 )
∂PL
I1 = I m Ðf1 + 90°; I2 = I m Ðf 2 ; 1−
∂PG1

Under balanced condition, I1 + I2 = 0, so We have PL = 0.5PG21, so


I m Ðf1 + 90° + I m Ðf2 = 0
∂PL
I m éëcos(f1 + 90°) + cos f2)ùû ∂PG2
= 0.5(2PG1 ) = PG1

+ j I m éë(sin f2 + sin(f1 + 90°)ùû


Therefore,
Equating real part to zero, we get 1
L1 =
cos(f1 + 90°) + cos f2 = 0 1 − PG2

cos(f1 + 90°) = − cos f2 1


Penalty factor L2 = (Plant G2 )
cos(f1 + 90°) = cos(p + f2) ∂PL
1−
∂PG2
f1 + 90° = p + f2
p ∂PL
f1 = + f2 = 0 ⇒ L2 = 1
2 ∂PG2

Alternately: For power generation to be economical,


We know that for balanced current the angle differ‑­
ence between two currents should be 180° or −180°.
C1L1 = C2 L2
Therefore, from Eq. (i), we have æ 1 ö
(10000) ç ÷ = 12500 × 1
−f1 + f2 − 90° = −180° ⇒ f1 = 90° + f2 è 1 − PG2 ø
Ans. (d) 1
PG2 = pu
110. The figure shows a two-generator system supplying 5
a load of PD = 40 MW, connected at bus 2. 1
For 100 MVA, PG2 = × 100 = 20 MW
Bus 1 Bus 2 5

æ1ö
2
G2 1
PL = 0.5(PG2 )2 = 0.5 ç ÷ = pu
G1 è ø
5 50
PG2
1
PG1 PL = × 100 = 2 MW
50
Therefore, total power
PD = 40 MW
PL = PG1 + PG2 − PL
total
The fuel cost of generators G1 and G2 are
40 = 20 + PG2 − 2
C1(PG1) = 10000 Rs/MWh and C2(PG2) =
12500 Rs/MWh PG2 = 22 MW
Ans. (a)
and the loss in the line is Ploss(pu) = 0.5PG21(pu),
where the loss coefficient is specified in pu on a 111. The sequence components of the fault current are
100 MVA base. The most economic power genera- as follows: Ipositive = j1.5 pu, Inegative = −j 0.5 pu,
tion schedule in MW is Izero = −j1 pu. The type of fault in the system is

Chapter 5 solved question paper.indd 481 3/23/2016 12:55:36 PM


482        CHAPTER 5:  POWER SYSTEMS

(a) LG (b) LL Therefore,


(c) LLG (d) LLLG
V1 = 1∠0° V2 = 1∠ − 30°
(GATE 2012: 1 Mark)
V1 − V2 1∠0° − 1∠ − 30°
Solution: I12 = = = 1 − j 0.288 p
pu
Z j 0.5
For double line ground fault (LLG),

I positive = −(I negative + I 0 ) We know that complex power demand is given by

The given values satisfy this condition. SD2 = V2 I2∗

= 1∠ − 30°I ∗2 ⇒ I2 = 1∠ − 30° pu
Ans. (c)

112. For the system shown below, SDI and SD2 are IG = I2 − I12 = 1∠ − 30° − (1 − j 0.288)
complex power demands at bus 1 and bus 2,
respectively. If |V2| = 1 pu, the VAR rating of the = 0.268 ∠ − 120°
capacitor (QG2) connected at bus 2 is
Rating of the capacitor,
Bus 1 Bus 2
V1 = 1–0 pu V2 QG1 = V2 VG = 1 × 0.268 = 0.268 pu
Ans. (b)

SG1 Z = j 0.5 pu 113. A cylindrical rotor generator delivers 0.5 pu power


in the steady-state to an infinite bus through a
QG2 transmission line of reactance 0.5 pu. The genera-
tor no-load voltage is 1.5 pu and the infinite bus
voltage is 1 pu. The inertia constant of the genera-
SD2 = 1 pu SD2 = 1 pu tor is 5 MW-s/MVA and the generator reactance
is 1 pu. The critical clearing angle, in degrees, for a
(a) 0.2 pu (b) 0.268 pu three-phase dead short circuit fault at the genera-
(c) 0.312 pu (d) 0.4 pu tor terminal is
(GATE 2012: 2 Marks) (a) 53.5 (b) 60.2
(c) 70.8 (d) 79.6
Solution:  From given the figure, we have
(GATE 2012: 2 Marks)
V1 = 1∠0°pu V2 = 1 pu Z = j 0.5 pu
SD1 = 1 pu SD2 = 1 pu Solution:  Representing the generator on a single
line diagram, we have
Total complex power
E = 1.5 pu 3f fault V = 1 pu
SG1 = SD1 + SD2 = 1 + 1 = 2 pu ∞
j0.5 pu
Power from bus 1 to bus j1 pu

V1 V2 sin(f1 − f2) X = j1 + j0.5 = j1.5 pu


P =
X
Critical angle,
1×1
= sin(f1 − f2)
0.5 d cr = cos−1 éë(p − 2d 0 ) sin d 0 − cos d 0 ùû
We have
To find steady state torque angle d 0:
sin(f1 − f2) = 0.5
PM = Pmax sind 0
f1 − f2 = sin−1 0.5 = 30°
E V
Since,
f2 = f1 − 30o = 0 − 30° = −30°
Pmax =
X

Chapter 5 solved question paper.indd 482 3/23/2016 12:55:51 PM


SOLVED GATE PREVIOUS YEARS’ QUESTIONS        483

We have We know that complex power is given by

PM =
E V
sin d 0 S = VI *
X
(1.5)(1) = (100∠60°) × (10∠ − 150°)∗
0. 5 = sin d 0
1. 5 = 1000 ∠210° = −866.022 − j500
sin d 0 = 0.5
⇒ d 0 = 30° or d 0 = 0.523 in radian Active power = —866. 02 W
Reactive power = — j 500 VAR
Therefore, Since, both are negative, therefore load absorbs

d cr = cos−1 [p − (2 × 0.523)] sin 30° − cos 30°


real power and reactive power.
Ans. (b)
= cos−1 [(2.095)(0.5) − 0.866] 116. The angle d in the swing equation of a synchronous
= cos−1(0.1815) = 79.6°
generator is the

Ans. (d) (a) angle between stator voltage and current


(b) angular displacement of the rotor with respect
114. A source vs (t) = V cos 100pt has an internal imped- to the stator
(c) angular displacement of the stator mmf with
ance of (4 +j 3) Ω. If a purely resistive load con-
respect to a synchronously rotating axis
nected to this source has to extract the maximum
(d) angular displacement of an axis fixed to the rotor
power out of the source, its value in Ω should be
with respect to a synchronously rotating axis
(a) 3 (b) 4
(GATE 2013: 1 Mark)
(c) 5 (d) 7
Solution:  In the swing equation, the angle d is the
(GATE 2013: 1 Mark)
angular displacement of an axis fixed to the rotor
Solution:  Given that impedance = 4 + j 3 with respect to a synchronous rotation axis.
Ans. (d)
For maximum power transfer from load to source,
117. For a power system network with n nodes, Z33 of its
RL = R s2+ Xs2 = 42 + 32 = 16 + 9 bus impedance matrix is j 0.5 per unit. The voltage
at node 3 is 1.3∠—10° per unit. If a capacitor having
= 25 = 5 Ω reactance of −j 3.5 per unit is now added to the
Ans. (c) network between node 3 and the reference node, the
current drawn by the capacitor per unit is
(a) 0.325 ∠−100°
115. A single-phase load is supplied by a single-phase
(b) 0.325 ∠80°
voltage source. If the current flowing from the load
to the source is 10∠ − 150°A and if the voltage at
(c) 0.371∠−100°
the load terminals is 100 ∠ 60° V, then the
(d) 0.433 ∠80°
(a) load absorbs real power and delivers reactive
(GATE 2013: 2 Marks)
power.
(b) load absorbs real power and absorbs reactive Solution:  We know that
power.
V
(c) load delivers real power and delivers reactive I=
power. Z
(d) load delivers real power and absorbs reactive
1.3∠ − 10° 1.3∠ − 10° 1.3∠ − 10°
power. = = =
Z33 + Znew j 0.5 + ( −j3.5) −j 3
(GATE 2013: 1 Mark)
1.3∠ − 10°
Solution:  Given that = = 0.433∠80
3 − ∠ − 90°
I L = 10Ð − 150° and VL = 100Ð60°. Ans. (d)

Chapter 5 solved question paper.indd 483 3/23/2016 12:56:00 PM


484        CHAPTER 5:  POWER SYSTEMS

Statement for Linked Answer Questions 118  1 1 −1 


and 119: In the following network, the voltage X + X 
B =  12 
13 X23
magnitudes at all buses are equal to 1 pu, the volt-
 −1 1 
age phase angles are very small, and the line resis-
 X23 + X13 
tances are negligible. All the line reactances are  X23
equal to j1 Ω. 1 + 1 −1   2 −1
=
 −1 1 + 1 −1 2 
=
Bus 1 (slack) Bus 2
j1 Ω éQ2 ù é 2 −1ù é 0.1 ù é 0 ù
êQ ú = ê−1 2 ú ê−0.2 ú = ê−0.1ú
ë 3û ë ûë û ë û
P2 = 0.1 pu Ans. (b)
119. If the base impedance and the line-to-line base
j1 Ω j1 Ω voltage are 100 Ω and 100 kV, respectively, then
the real power in MW delivered by the generator
connected at the slack bus is
Bus 3 P3 = 0.2 pu
(a) —10 (b) 0
(c)     10 (d) 20
118. The voltage phase angles in rad at buses 2 and 3 are
(GATE 2013: 2 Marks)
(a) q2 = —0.1, q3 = —0.2
(b) q2 = 0, q3 = —0.1 Solution:  In the given network
(c) q2 = 0.1, q3 = 0.1 P1 + P2 + P3 = 0.
(d) q2 = 0.1, q3 = 0.2
P1 = −P2 − P3
(GATE 2013: 2 Marks)
P1 = −0.1 + 0.2 = 0.1 pu
Solution:  From the figure we have
(100 × 103 )2
P2 = 0.1 pu and P3 = 0.2 pu. VA(base) = = 100 × 106
100
For a load flow problem MVA(base) = 100
éQ2 ù −1 éP2 ù
êQ ú = [ B ] êP ú Real power P = 100 × P2 = 100 × 0.1= 10 MW
ë 3û ë 3û Ans. (c)

Chapter 5 solved question paper.indd 484 3/23/2016 12:56:09 PM


SECTION VI: CONTROL SYSTEMS

MARKS DISTRIBUTION FOR GATE QUESTIONS

9
8
Number of questions

7
6
5 Marks 1
4 Marks 2
3 Total number of questions
2
1
0
2009 2010 2011 2012 2013 2014 2015

TOPIC DISTRIBUTION FOR GATE QUESTIONS

Year Concepts
2015 Basic of control system, Time domain analysis, Root locus technique, Frequency
response analysis using Bode plot, State variable analysis
2014 Bode plots, Routh—Hurwitz criterion, Transfer function
2013 Transfer function, Frequency response, Signal flow graph, State space analysis
2012 Transfer function, State space analysis, Lead—lag compensator
2011 Frequency response, Steady state error, Transfer function, Root loci
2010 Transfer function, Time response, Frequency response, Steady state space analysis
2009 Transfer function, Frequency response, Time response, State space analysis

Chapter 6(487-554).indd 485 3/23/2016 2:18:23 PM


Chapter 6(487-554).indd 486 3/23/2016 2:18:23 PM
CHAPTER 6

CONTROL SYSTEMS

The discipline of control systems deals with analysis and tools for analysis and design: Bode plots, Nyquist
control law synthesis for engineering systems. The study plots, and Nichol charts.
of control system starts with the concept of an engineer-   5.  State space representation, state space realizations,
ing system, identifying its various constituents, devel- development of concepts such as stability, control-
opment of time-domain and frequency-domain models, lability and observability.
study of system behaviour using the mathematical prop-
erties of these models, assessment of system stability
and performance, etc. The study can be systematically
divided into following: 6.1  FUNDAMENTALS
  1.  Concept of transfer function, model simplifications
using block diagram and signal flow graphs. A system may be defined as an aggregation of com-
  2.  Time-domain analysis of system, concept of step ponents accomplishing a given task in a coordinated
response and time-domain specification. fashion. If a system’s behaviour can be manipulated in a
  3.  Assessment of system stability and performance, desired fashion then it is known as control system. In
prediction of closed-loop behaviour for a given such a system, input is the desired response and output
open-loop system using root locus approach, design is the actual response as shown in Fig. 6.1. Some exam-
of compensators using root locus approach. ples are, smart transportation system; steering control
  4.  The concept of frequency response and introduction of an automobile; health monitoring and diagnostic
to sinusoidal transfer function, frequency-domain systems.

Chapter 6(487-554).indd 487 3/23/2016 2:18:23 PM


488     Chapter 6:  CONTROL SYSTEMS 

Input Output transform of the output signal to the Laplace transform


Stimulator Control Response of the input signal, provided there is no initial stored
system energy, that is, under zero initial conditions.

Figure 6.1 |   Control system.


Y (s)
G(s) =
R(s) Initial conditions are zero
6.1.1 Reasons for Building Control Systems Thus, one of the limitations of using transfer function
is that system behaviour of a system with stored energy
We build control systems for four primary reasons: cannot be predicted.
  1.  Power amplification The transfer function of a system can be used to pre-
  2.  Remote control dict the behaviour of a system only when it is initially
  3.  Convenience of input form relaxed. A transfer function is a rational function in s,
  4.  Compensation for disturbance and thus can be represented as a ratio of two polynomi-
The study of control systems can be divided into two als in s, as
parts m −1
m
N (s) a0 s + a1s + … am
  1.  System analysis: Study of the behaviour of the G(s) = = −
D(s) n
b0 s + b1sn 1
+ … bn
system which involves assessment about its stabil-
ity and performance. This can be done by math- For physical realisable systems, the order of numerator
ematical modelling of a system. must be less than or equal to the order of the denominator
  2.  Control design: Design of control law or control polynomial. The salient points of the transfer function are
signal so that the overall system becomes stable   1.  The order of the numerator polynomial is always
and offers desired performance. less than or equal to the order of denominator poly-
For the purpose of analysis and control design, a system nomial m ≤ n.
may be represented equivalently in either time domain   2.  The roots of numerator polynomial N(s) = 0 are
or frequency domain. The discussion here is limited to known as zeros of the transfer function.

a0 sm + a1sm−1 + … am = 0
linear system with constant parameters. Such systems
can be conveniently represented using
a0 (s − z1 )(s − z2 )… (s − zm ) = 0
  1.  Differential equations for continuous time systems
and difference equations for discrete time systems. Here p1, p2 …, pn are called zeroes of transfer
  2.  Transfer functions function.
  3.  State space representation   3.  The roots of the denominator polynomials D(s) = 0
are known as poles of the transfer function.
6.1.1.1 Differential Equations
b0 sn + b1sn −1 + … bn = 0
Ordinary differential equations provide most generalised
b0 (s − p1 )(s − p2 )… (s − pn ) = 0
representation of systems. Every kind of system whether
it is time invariant or variant, linear or nonlinear, etc.
can be represented in terms of differential equations. For Here p1, p2 …, pn are the poles of the transfer func-
example, in a series R-L circuit, the input and output tion. Thus, poles are the complex frequencies over
relationship, in terms of voltage and current signals can which the transfer function becomes infinite.
be represented by the following differential equation   4.  Since physical systems have real parameters; there-
fore, the poles and zeros if complex always occur in
dt(t) conjugate pairs. This is mandatory as the root of an
Ri(t) + L = v(t) algebraic equation with real coefficients are either
dt
real or if complex they occur in conjugate pairs.
The stored energy in the circuit elements can be repre-
sented using initial conditions. The initial conditions can be 6.1.1.3 State Space Representation
incorporated into the solution of the differential equations.
The state space representation is derived from the dif-
6.1.1.2 Transfer Functions ferential equation representation. A higher-order differ-
ential equation can be transformed into a set of coupled
The transfer function (TF) of a system is the Laplace trans- first-order differential equations; a form which is particu-
form of its impulse response, that is, ratio of the Laplace larly suited for numerical computations.

Chapter 6(487-554).indd 488 3/23/2016 2:18:29 PM


6.1  FUNDAMENTALS     489

The typical state space representation of a system 6.1.2.1 Classification Based on Input/
is described by the following vector matrix first-order Output Relation
differential equation called state equation, and a vector
matrix algebraic equation called output equation. Linear Systems

x = Ax + Bu
Systems in which output varies linearly with the input
and can be represented using linear differential equa-
y = Cx + Du tions are called linear systems; otherwise they are called
non-linear systems. Linear systems follow the principle
where A is the plant matrix, B is the input coupling
of superposition and homogeneity as:
matrix, C is the output coupling matrix and D is the
direct transmission matrix. A typical control system   1.  Principle of superposition: The principle of
involves the following parts/signals as shown in Fig. 6.2 superposition states that if u1 signal produces an
and described as follows. output y1, and another signal u2 produces an output
y2, then the joint application of these signals, that
is, u1 + u2 must produce an output y1 + y2 in case
Load
Command disturbance if the system is linear. So
signal
if u1 → y1 and u2 → y2
Controller Plant
then u1 + u2 → y1 + y2
Feedback   2.  Principle of homogeneity: The principle of
conditioner Sensor homogeneity states that if u1 signal produces an
noise output y1, then the application of signal au1 must
Figure 6.2 |   Block diagram of general control system. produce ay1, for a system to be linear.

u1 → y1 then au1 → ay1


  1.  Plant: It is the given system whose behaviour is to
be analyzed and/or control law is to be designed so The equations for linear and non-linear systems are as
as to make it behave in a desired fashion. follows:
  2.  Input signal: These are the signals which can be Linear system:
manipulated and used to derive the given system.
Inputs which cannot be manipulated are usually d2x dx
2
+a + bx = f (t)
termed as disturbance inputs. dt dt
  3.  Output signals: These are the signals which can Non-linear system:
be measured and manifest the behaviour of the
system. In addition they are used to design control d2x dx
2
+a + b sin x = f (t)
law and also feed controllers. dt dt
  4.  Load disturbances: These are the signals, which
impinge upon the system output and try to devi- Time Invariant Systems
ate the behaviour of the system from the normal
Systems with constant parameters are called time invari-
one. One of the purposes of closed-loop control sys-
ant systems, for example, a mass-spring system in which
tems is to compensate or nullify the effect of load
mass M as well as stiffness k of the spring is constant and
disturbances.
do not change with time.
  5.  Sensor noise: These are inevitable signals which
particularly act in feedback control system. Sensor Mx + kx = F
outputs are used to drive the control blocks, and
one of the objectives of control blocks remains Such systems can be represented using differential equa-
rejecting noise signal. tions with constant coefficients.
If g (t) is the input: c(t) is the output
6.1.2 Classification of System If g (t + T) in the input: c(t + T) is the output.
Systems which are linear as well as time invariant can
System may be classified on the basis of
be represented using transfer function.
  1.  Input/Output relation On the other hand, systems with varying parameters
  2.  Type of control requirement are called time varying systems, for example, a missile
  3.  Type of control configuration during flight.

Chapter 6(487-554).indd 489 3/23/2016 2:18:36 PM


490     Chapter 6:  CONTROL SYSTEMS 

Time varying systems cannot be described in terms of   8.  Stable system: The system produces bounded
transfer function. output for bounded input. A signal is bounded if
its magnitude is finite.
Causal Systems   9.  Unstable system: The system output is
unbounded for bounded input.
Systems that follow cause-and-effect relationship are
called causal systems. Causality means output succeeds
input and output cannot appear before application of
6.1.2.2 Classification Based on Control
input. Casual systems have zero output in negative time. Requirement
Systems which do not follow cause and effect relation-
On the basis of command signals, control systems can be
ship are called non-causal systems or anticipatory sys-
classified into two types:
tems. Generally all physical systems are causal in nature.

Regulators
Lumped System
Control systems, which are required to maintain their
Systems in which there is no variation in parameters
output around a single constant operating point, are
with respect to the spatial coordinates are known as
known as regulators. For example: a temperature control
lumped parameter systems otherwise they are termed as
system maintaining room temperature at 21°C.
distributed parameter systems.
Lumped parameter dynamical systems are repre-
Tracking System
sented using differential equations whereas distributed
ones using partial differential equations. Control systems, which are required to follow continuously
changing input signals, are known as tracking systems. For
Some Other Types of Systems example: an intercepting missile seeking a moving target.
They are also known as servo-mechanism control systems.
Some other types of systems based on input/output rela-
tions are listed as follows:
6.1.2.3 Classification Based on Control
  1.  Deterministic systems: Systems that have Configuration
fixed parameters and input.
  2.  Stochastic systems: Systems with randomness On the basis of signal used to actuate the given system,
in parameters or input. the control systems can be classified as follows.
  3.  SISO system: Single input variable is applied to
the system and the output variable is also single. Open-Loop System (Non-Feedback System)
  4.  MIMO system: Input and output variables are
more than one. In open-loop systems, output of the system is not used
  5. Continuous system: If the system handles continu- to generate the control signal. An open-loop speed con-
ous time signals, that is, system variables are func- trol of a DC motor requires application of pre-calculated
tions of continuous time variable, t, then it is called voltage level to achieve the speed control. However, any
CT systems. For example: amplifiers and analog filters change in loading conditions will cause variation of speed
  6.  Discrete system: If the system handles discrete from the desired one. Such systems are also very sensi-
time signals, that is, system variables are known tive to changes in plant configuration, unable to com-
only at certain discrete intervals of time. pensate for unwanted disturbance signals etc.
  7.  Dynamic and static systems: A system is said An open-loop system can be represented as shown in
to be dynamic if its output depends on present and Fig. 6.3 and examples of these systems are mechanical sys-
past inputs. It has memory. A memory less system tems consisting of mass, spring and damper with a constant
is said to be static. force applied to the mass. There is no feedback or sensors.

Input Load
disturbance disturbance

Input Input + Process/ + Output or


Controller + + controlled
Reference transducer plant
  variable
Figure 6.3 |   Open-loop control systems.

Chapter 6(487-554).indd 490 3/23/2016 2:18:38 PM


6.1  FUNDAMENTALS     491

Input Output
disturbance disturbance
Error
Reference + + Output (or)
Input ++
− Controller Plant + controlled
Input transducer variable

Output transductor/
sensor

Figure 6.4 |   Closed-loop control systems.

Closed-loop system (feedback control system): di


  2.  Inductor: L = v(t)
In closed-loop systems, the output of the system is con- dt
tinuously measured and compared with the command
signal as shown in Fig. 6.4. The difference between the
C∫
1
desired and actual output signal is used to generate the   3.  Capacitor: idt
control signals. The prime objective of closed-loop sys-
tems is to make them less sensitive to the changes in
plant parameters. However, presence of feedback loop Series RLC Circuit
makes the overall system extremely sensitive to the vari- In a series of RLC circuit as shown in Fig. 6.5, the sum
ations in feedback path components. of the voltages across individual elements is equal to the
Generally, control systems are low pass filter and applied voltage, and the relation is
in majority of cases, the sensor noise is automatically
+ Ri + ∫ idt = v(t)
reduced. The basic purpose of using the closed-loop di 1
L
configuration is to make the overall system insensi- dt C
tive to variation in plant, which cannot otherwise be
achieved using open-loop configuration. Using the nega- i(t)
tive feedback, the gain of the overall system is reduced
and bandwidth is increased. vR = Ri
vL = L vc = ∫idt
di 1
v(t) dt C
6.1.3 Modeling of Physical Systems

Figure 6.5 |   Series RLC circuit.


Representation of a system by mathematical relations
or equations is known as mathematical modeling of sys-
tems. These equations may be in the form of
The transfer function of the series RLC circuit may be
  1.  Difference equations defined as the ratio of the Laplace transform of the current
  2.  Differential equations I(s) to the Laplace transform of the input voltage V(s) as
  3.  Transfer equations
  4.  Matrix 1
sLI (s) + RI (s) + I (s) = V (s)
Cs
Transfer function is defined as the ratio of Laplace trans-
form of output to Laplace transform of input with zero I (s) Cs
T (s) = =
initial condition. V (s) LCs2 + RCs + 1

LT of output
TF = Parallel RLC Circuit
LT of input Initial condition = 0
In a parallel RLC circuit as shown in Fig. 6.6, the sum of
6.1.3.1 Electric Systems the currents through individual elements is equal to the
total current, given a voltage across the parallel combi-
The dynamical input-output relations of the fundamen- nation, and the dynamical relation is
tal elements of electric circuits are given as follows
+ ∫ vdt + C
v(t) 1 dv
= i(t)
  1.  Resistor: i(t) = v(t)/R R L dt

Chapter 6(487-554).indd 491 3/23/2016 2:18:48 PM


492     Chapter 6:  CONTROL SYSTEMS 

i(t) Z2(s)
i1 i2

iL = ∫vdt
1 dv Z1(s)
v(t) iR = v(t) iC = C Ei(s)
R L dt + Eo(s)

Figure 6.6 |   Parallel RLC circuit.


Figure 6.7 |   General op-amp with negative feedback.
The transfer function of the parallel RLC circuit may be
computed as the ratio of the Laplace transform of the The transfer function of the arrangement is
current I(s) to the Laplace transform of the input voltage Eo (s) Z (s)
V(s) as =− 2
Ei (s) Z1(s)
I (s) 1 1 LCs2 + sL + R
= + + Cs = If the signal in Fig. 6.7 is applied on the positive termi-
V (s) R sL sRL
nal of op-amp, then the overall transfer function shall be

6.1.3.2 Active Circuits Eo (s) Z (s)


= 1+ 2
Ei (s) Z1(s)
Active electric circuit comprises of elements which can
impart energy into the circuit. For example: a transistor, Depending upon various types of Z1(s) and Z2(s), the cir-
operational amplifier (op-amp), a signal conditioner, etc. cuit can be used as an integrator, differentiator, lag and
Op-amps offer several advantages over other amplifiers lead or lag-lead compensator as shown in Figs. 6.8(a)
  1.  Very high input impedance ≈ 106 W which means
and (b).
that the op-amp does not load any preceding circuit.
  2.  Very low output impedance, which means that the
op-amp is capable of driving any load within per-
missible limits. − −
  3.  Very high open-loop gain, which means better gain + +
stabilization under negative feedback.
  4.  Very low bandwidth, which can be enhanced to a
high value using negative feedback. It is worth to (a)     (b)
remember that gain-bandwidth product of op-amp/
any device is constant. Bandwidth is recovered Figure 6.8 |   Op-amp as (a) integrator and
using negative feedback at the expense of gain. (b) differentiator.

General Op-Amp Circuit 6.1.3.3 Mechanical Systems


A general operational amplifier network with negative
The mechanical systems can be classified into transla-
feedback having two impedance blocks Z1(s) and Z2(s) is
tional and rotational mechanical systems and their char-
shown in Fig. 6.7.
acteristics features are described in Table 6.1.

Table 6.1 |   Elements of translational and rotational mechanical systems

Translational System Rotational System


Mass (M) Moment of inertia (J)
d 2 x(t) d 2q (t)
f(t) fm (t) ∝ T(t) θ(t) Tj(t) ∝
M dt2 J dt2
d 2 x(t) d 2q (t)
Tj(t) = J
fm (t) = M J is expressed in kg-m2/rad dt2
x(t) dt2
f (t) = fm (t)

(Continued)

Chapter 6(487-554).indd 492 3/23/2016 2:19:04 PM


6.1  FUNDAMENTALS     493

Table 6.1 |   (Continued)

Translational System Rotational System


Dash pot (B) Dash pot (B)
dx(t) dq (t)
f(t) fb (t) ∝ B Tb (t) ∝
dt dt
dx(t) dq (t)
x(t) fb (t) = B Tb (t) = B
dt dt

T(t) q(t)
B is the rotational friction coefficient
expressed in (N-m)/(rad/s)

fk (t) ∝ x(t) Tk (t) ∝ q (t)


Spring (k) Torsional spring (k)
x(t) fk (t) = kx(t) Tk (t) = kq (t)

k
T(t) q(t)
k is stiffness of the spring expressed in
N-m/rad

Translational Mechanical Systems f


k
Modeling of mechanical systems involves application J
of laws of mechanics, fluid dynamics, thermodynamics, q(t)
etc. A typical translational mechanical system, that is, t
mass-spring damper system excited by applying a force Figure 6.10 |   Rotational mechanical system.
is shown in Fig. 6.9.
The corresponding differential equation is

f x(t) d 2q dq
J 2
+f + kq = t (t)
dt dt
M F(t)
k The respective transfer function is given by
Figure 6.9 |   Mass spring damper translational system. q (s) 1
= 2
t (s) s J + sf + k
The system can be described in terms of the following
differential equation
6.1.3.4 Electrical Analogue of
Mechanical Systems
d2x dx
M 2
+f + kx = F (t)
dt dt The basic elements of electrical systems are listed in
Table 6.2.
The respective transfer function is given by It is clear that differential equation and transfer
function of RLC circuits, translational and rotational
X(s) 1 mechanical system are similar. Therefore, given a
= 2
F (s) s M + sf + k mechanical system, we can easily construct a RLC circuit
having same dynamics as that of a mechanical system.
Comparing the coefficient of dynamical relations of series
Rotational Mechanical Systems RLC circuit and that of mechanical systems as well as
Rotational mechanical systems involve inertia, frictional their variable, the observations in terms of electrical ana-
and torsional stiffness elements as shown in Fig. 6.10. logue of mechanical system is summarised in Table 6.3.

Chapter 6(487-554).indd 493 3/23/2016 2:19:20 PM


494     Chapter 6:  CONTROL SYSTEMS 

Table 6.2 |   Basic elements of electrical systems


Element Symbol Current, i(t) Voltage v(t)
Resistor (R) R R i(t)
v(t)
y(t) R

R v1(t) − v2 (t) R [(i1(t) − i2(t))]


R
v1(t) v2(t)
Inductor (Ls)

L∫
1 di(t)
v(t) v(t)dt L
dt

L∫ 1
d
i (t) − i2(t)
1
v (t) − v2 (t) dt
dt  1
v1(t) v2(t) L

C∫
1 dv(t) 1
Capacitor C i(t)dl
Cs v(t) dt

v1(t) v2(t)
C
d
(v (t) − v2(t))
dt 1
1
C ∫ (i1(t) − i2(t)) d

Table 6.3 |   Electrical analogous of mechanical systems


Series Translational Rotational Ea 1 ia Tm Td
1 wm
RLC Mechanical Mechanical Kt
+ sLa+ Ra sJ + f
System System −
Voltage v(t) Force F(t) Torque t(t)
Kb
Current i(t) Displacement x(t) Angle
displacement q(t) Figure 6.11 |   Armature controlled DC machine.
R f f
L M J where La is the armature winding inductance, Ra in the
C k k winding resistance, J is the moment of inertia, f in the
damping factor of mechanical system and Td represents
the load fluctuations. Kt and Kb are constants as
6.1.3.5 Electro-Mechanical Systems Tm = Kt ia
Electromechanical systems involve either a mechani- Eb = Kbw m
cal system driven by an electrical device or an electri- The transfer function of the armature-controlled DC motor,
cal device excited by a mechanical input. Either way it neglecting the armature winding time constant term is
amounts of coupling between mechanical variables and
electrical quantities. w m (s) Km
=
Ea (s) t ms + 1
Armature-Controlled DC Motor
q (s) Km
=
In this system, the voltage across the armature circuit Ea (s) s(t m s + 1)
of a DC motor is varied to change the motor speed. The
back emf of the motor acts as a negative feedback. The where Km is the equivalent motor gain constant and tm
block diagram of the system is shown in Fig. 6.11. is motor-equivalent time constant.

Chapter 6(487-554).indd 494 3/23/2016 2:19:42 PM


6.1  FUNDAMENTALS     495

Also,   4.  It reduces the effect of external disturbances as


q(s)
well as sensor noise if system is well designed.
Kt /RaB
=
Ea (s)  K K 
s (1 + sTa )(1 + sTm ) + b t  6.1.5 Sensitivity of Control Systems
 R aB 

where Ta is electrical time constant, given by La/Ra 6.1.5.1 Sensitivity to Variations in


System Parameters
Tm is mechanical time constant, given by J/B.
Given an open-loop plant G(s) with input R(s) and
Field-Controlled DC Motor output Y(s), then the sensitivity of the overall system is
defined as an incremental variation in the system output
In the case of field-controlled DC motor, the field wind- to the incremental variation in the plant G(s), as
ing time constant is comparable to that of the mechani-
cal time constant. Thus, the electrical part of the system ∂ Y /Y
SGY =
cannot be neglected. The transfer function of the field- ∂G/G
controlled DC motor is given by
For the open-loop systems, the sensitivity is always one.
w m (s) K′ For open-loop systems
=
Ea (s) (sL f + R f )(sJ + f )
∂Y /Y
q (s) K′ SGY =
∂G/G
=1
=
Ea (s) s(sL f + R f )(sJ + f )
which means that open-loop systems are extremely sensi-
where Lf  and Rf are the inductance and resistance of the tive to the changes in the plant.
field windings, and J and f are the inertia and damping On the other hand, the sensitivity of the closed-loop
of the mechanical system. Also, system with feedback elements H(s) is given by

q(s) K f / R fB ∂Y / Y 1
= SGY = =
E a(s) s(1 + sTf )(1 + sTm ) ∂G / G 1 + G(s)H(s)

DC Techno-Generator Sensitivity of a closed-loop system is therefore the func-


tion of the loop transfer function G(s) H(s). Since, the
The emf produced by the DC techno-generator is directly loop transfer function is a function of the frequency at
proportional to the shaft speed, and hence, the transfer which plant operates; therefore, sensitivity of a closed-
function given by loop system changes with the frequency.
E(s) It is clear from the above relation that sensitivity of
= Kt
w (s) the closed-loop system can be reduced by using high loop
E(s) gains.
= sKt
q (s)
6.1.5.2 Sensitivity with Respect to
where Kt is the sensitivity of the techno-generator in
volts per rad/s. Sensor Noise

In a closed-loop system the sensitivity of the output with


respect to the sensor noise is given by
6.1.4 Effects of Negative Feedback
Y ∂ Y /Y −G(s)H(s)
Some of the salient features of using negative feedback are: SN = =
∂ N /N 1 + G(s)H(s)
  1.  It increases the bandwidth of the system and
reduces the system gain. Thus, the system can be made less sensitive to the
  2.  The sensitivity of the output with respect to sensor noise, if the loop gain can be made almost neg-
plant gets reduced; however, the system becomes ligible over the frequency range in which the noise lies.
more sensitive to sensor noise, feedback ele- Since control systems are low pass filter, this objective
ments, etc. is easily achieved, if not then use of proper filter makes
  3.  In most cases, it enhances stability of the system. it possible.

Chapter 6(487-554).indd 495 3/23/2016 2:19:52 PM


496     Chapter 6:  CONTROL SYSTEMS 

6.1.5.3 Sensitivity with Respect to Feedback signal conditioners, models of external disturbances
Element Gain (includes both disturbances which influence the input
to plant as well as those which influence the output). It
The sensitivity of the closed-loop system output with respect is easy to calculate the transfer function of each com-
to the fluctuations in the feedback element gain H(s) is as ponent. Thereafter, assuming that no component loads
the preceding component and it is able to drive well
Y ∂ Y /Y −G(s)H(s) the succeeding component, the overall transfer func-
SH = =
∂H /H 1 + G(s)H(s) tion of the system, can be obtained using the block
diagram reduction rules listed below and summarized
Thus, in order to reduce the sensitivity of the output in Table 6.4.
with respect to feedback element gain, the loop gain G(s)
H(s) needs to be minimised.   1.  Summing points: These are the points where
two or more than two signals combine or enter into
Thus, feedback reduces the sensitivity to the for- the sub-systems. The value of the summing points
ward path variations, but makes the systems extremely is equal to the sum of all incoming signals.
sensitive to the feedback path elements. Therefore, the   2.  Take-off point: These are the points from where
feedback blocks should be of very high precision, other- one signal feeds two or more than two blocks.
wise even a small variation in feedback gain changes is   3.  Blocks in parallel: When two or more blocks
directly reflected into the output signal. are in parallel, then the overall equivalent transfer
function of the combination is equal to the sum of
6.1.5.4 Load Disturbance Rejection their individual transfer function.
  4.  Blocks in cascade: When two or more than two
Since the load disturbance directly acts as the output, blocks are placed in cascade configuration, then
the effective transfer function between the disturbance the overall transfer function of the combination is
and the output is equal to the product of their individual transfer
Y (s) 1 functions.
=   5.  Feedback configuration: When two blocks
N (s) 1 + G(s)H(s)
G(s) and H(s) are in negative feedback configura-
tion, then overall transfer function of the loop is
Thus, the effect of the load disturbance on the output
given by
can be minimised by using a very high loop gain, that
is, G(s)H(s) >> 1 over the frequency range in which the G(s)
output disturbance lies. Gc (s) =
1 + G(s)H(s)

6.1.5.5 Design Considerations


  6.  Shifting take-off point: When a take-off point
is shifted from right to left of a component, then
The control design problem poses conflicting requirements.
the take-off branch also adds one additional gain
  1.  Command following and sensitivity reduction for block equal to the gain of the block it jumped. On
G(s) variations, require G(s)H(s) >> 1 the other hand, when a take-off point is shifted
  2.  Disturbance rejection requires G(s)H(s) >> 1 from left to right of a component, then the take-off
  3.  Insensitivity to feedback element gain requires branch also adds one additional gain block equal to
G(s)H(s) << 1 the gain one over that of the block it jumped.
  4.  Insensitivity to sensor noise requires G(s)H(s) << 1   7.  Jump error detector: When the take of point
The design objectives can be achieved by needs to be shifted such that it jumps an error
detector, then one more error detector need to be
  1.  Using precise elements in the feedback path. inserted in the takeoff branch so as to compensate
  2.  Make high-frequency loop gain as small as possible. the missed signal.
  3. Making loop gain very high in the low-frequency range.
  4.  Using additional compensators for disturbance Common rules for block diagram algebra with illustrations
rejection. are given in Table 6.4.
Block diagram reduction rules are helpful in reducing
6.1.6 Model Simplifications using the complexity of overall interconnection. On the other
Block Diagram Algebra hand, another pictorial representation of the system
that helps in getting the overall transfer function of
A control system consists of various components, that the system with the help of Mason’s gain formula are
is, given plant with actuators and sensors, controllers, signal-flow graphs.

Chapter 6(487-554).indd 496 3/23/2016 2:19:55 PM


6.1  FUNDAMENTALS     497

Table 6.4 |   Common block diagram reduction rules for control systems
Block Diagram Overall Transfer Function Diagrammatic Representation

Blocks in parallel Overall transfer function is the


G2(s)
sum of the transfer functions of x(s) y(s) x(s) y(s)
the individual blocks in parallel ++
G1(s) G1(s)+G2(s)

Two blocks in parallel Equivalent representation


Blocks in cascade Overall transfer function is the x(s) y(s) x(s) y(s)
product of the transfer functions G1(s) G2(s) G2(s)G1(s)
of the individual blocks in cascade Two blocks in cascade Equivalent representation

Blocks in feedback Overall transfer function is x(s) y(s)


configuration via G(s)
G(s) x(s) G(s) y(s)
T (s) = +
feedback gain H(s) 1+ G(s)H (s) − I+G(s)H(s)
H(s)
Blocks in negative Equivalent
feedback representation
Shifting of take-off The outgoing branch must add y(s) y(s)
point before a block additional block with gain G1(s) G(s)
G1(s) x(s) y(s) x(s) y(s)
G(s) G(s)
+ +
− −
Shifting take-off point Equivalent
before a block representation
Shifting of take-off The outgoing branch must add e(s) 1 e(s)
point after a block additional block with gain 1 G(s)
G1 (s)
G1(s) x(s) y(s) x(s) y(s)
G(s) G(s)
+ +
− −
Shifting take-off point Equivalent
after a block representation
To jump a take-off Jumping an error detector will r(s)
point before the error add one more error detector in e(s) − e(s)
+
detector the take-off branch
G(s) G(s)
x(s) + y(s) x(s)+ y(s)
− −
r(s)
Take-off point jumping Equivalent
an error detector representation
Moving summing B B
point ahead of the G(s)
A
block ++ G(s) G(s) ++
A A+B
(A+B) G(s) A G(s) + B G(s)
=(A+B) G(s)
Moving summing B 1
point before the block B G(s)
A
G(s) ++ ++ G(s)
A
AG(s) + B A G(s) + B

Chapter 6(487-554).indd 497 3/23/2016 2:20:07 PM


498     Chapter 6:  CONTROL SYSTEMS 

In addition, when integrators are used in signal-flow   2.  Node: It represents a system variable, the value of
graphs to represent states of a system, the resulting which is equal to the sum of all incoming signals.
graph is known as state diagram. The state-space rep- The outgoing signals from a node do not affect
resentation of a system can be easily obtained from state the value of node. However, for representing input
diagrams. and output signals, two dummy nodes are created,
which are known as input and output nodes.
  3.  Input node: It is a node which has only outgoing
6.1.7  Signal-Flow Graphs
branches.
  4.  Output node: It is a node which has only incom-
A signal-flow graph (SFG) is a pictorial representation of
ing branches.
a system in which each component of the system is rep-
  5.  Loop: It is a closed path which starts and ends at
resented by means of a directed branch annotated with
the same node without re-visiting any node. The
the value of its gain as shown in Fig. 6.12. Generally
product of all the branches involved in a loop is
branches meet a point or emerge from a point, which is
called loop gain. The loop gain for the loop shown
known as a node.
in Fig. 6.14, is −g1g2h1.

Input Forward path g4 Output


g1 g1 g2
node g2 g3 node
h1 Loop −h1

Figure 6.14 |   Loop with gain −g1g2h1.


Branch
h2

Figure 6.12 |   Signal flow graph.   6. Forward path: A path from input node to the
output node without repeating any node is called for-
Each branches of a signal flow graph represents a sub- ward path as shown in Fig. 6.15. The product of the
system with gain g1 and signal flows in a pre-specified gain of all the branches taking part in the forward
direction as shown in Fig. 6.13. path is called forward path gain and is represented by
Pi, which is the forward path gain of ith forward path.
Gain or transmittance The figure shows a SFG which has two forward path;
of branch one P1 with gain g1g4 and second P2 with gain g1g2g3.

g P1 = g1g4
g4
i j

Figure 6.13 |   Sub-component of a system as a g1 g2 g3 1


directed branch.
P2 = g1g2g3

Figure 6.15 |   Forward paths and their gains.


Once the given system is represented in terms of directed
branches and nodes, the overall system gain is calculated
via analytical formula known as Mason’s gain for-
mula. Signal flow graphs are applicable to linear-time The loops present in the SFG may or may not
invariant (LTI) systems only. touch each other.
  7.  Non-touching loops: These loops do not have
In order to apply Mason’s gain formula, several
any node in common. Figure 6.16 shows two loops
notions about a node, branch, loops, forward paths, non-
L1 and L2 that do not touch each other, and have
touching loops, determinant of the graph and its cofac-
loop gains g2h1 and g4h2, respectively.
tor, etc. need to be a defined as follows:

  1.  Branch: A sub-component of a system is rep- g1 g2 g3 g4 g5


resented using a branch. A branch has a gain
h1 h2
or transmittance value equal to the gain of the
L1 = g2h1 L2 = g4h2
sub-component. The direction of arrow rep-
resents the direction of flow of signal in the Figure 6.16 |   Non-touching loops with no node in
sub-component. common.

Chapter 6(487-554).indd 498 3/23/2016 2:20:13 PM


6.2  TIME-RESPONSE ANALYSIS     499

6.1.7.1 Rules to Draw Signal Flow Graphs Δ = 1 − (sum of gains of all the loops) + (sum of
the products of all possible combination of non-touching
  1.  Rule 1 loops taken two at a time) − (sum of the products of all
possible combination of non-touching loops taken three
a
X1 a1 at a time) + …
X1 X2 X2 Δk is called the co-factor of the kth forward path and is
X2 = aX1 a2
obtained from Δ by considering gain of all such loops
X2 that touch kth forward paths zero as zero.
X3 = a1X1 + a2X2
The determination of system transfer function
using SFG requires correct identification of all the
  2.  Rule 2: Elimination of cascaded branch loops, forward paths and listing down of the loops
touching each individual forward path. Thereafter,
a b ab
= the solution is readily obtained using the Mason’s
X1 X2 X3 X1 X3 gain formula.

  3.  Rule 3: Elimination of parallel branch


6.2  TIME-RESPONSE ANALYSIS
a

a+b The behaviour of a system can be described in terms


X1 X2 = of its time response as well as equivalently in terms of
X1 X2
frequency response. The evolution of system’s output
b with time, in response to excitation, is termed as time
response. The time response of systems can be classified
  4.  Rule 4: Elimination of mixed node as follows:

X1 a X1 ac   1.  Transient response: It represents the behav-


c iour of the system output with respect to time just
after the application of input signal. The transient
X3 X4 X4 response characterises the nature and speed of the
X2 b X2 bc
response. For a stable-state, transient dies out as
  5.  Rule 5: Elimination of loop time approaches infinity.
  2.  Steady-state response: It represents the
behaviour of the system when transients have
a X2 b ab/1−bc
X3 died out or sufficient time has elapsed after appli-
X1 X1 X3 cation of input signal. The steady-state response
c characterises the steady-state accuracy of the
system.

6.2.1 Transient State Behaviour


6.1.7.2 Mason’s Gain Formula
Transient response of a system is studied using standard
Mason’s gain formula is used to find the input output test signal. The test signal should be sufficiently rich so
transfer of a given system using the signal-flow graph as to excite all the modes of the system. Such signals
representation. The transfer function G of the overall are called persistent excitation signals. The popular test
system is given by signals used to study the transient response are discussed
as follows.
1 m
G= åP D
D k =1 k k 6.2.1.1 Standard Signals

where m is the number of parallel paths from input to   1.  Unit step signal: It represents a signal of con-
output, Pk is the path gain of kth forward path and is stant magnitude and is used to represent sudden
equal to the product of transmittance of all branches in unit change in command signal. The unit step
the path. Δ is called the determinant of the graph and signal as shown in Fig. 6.17(a), denoted by u(t), is
is given by described by the following relations

Chapter 6(487-554).indd 499 3/23/2016 2:20:20 PM


500     Chapter 6:  CONTROL SYSTEMS 

1 t ≥ 0   4.  Unit impulse signal: Precisely speaking an


u(t) = 
0 t < 0
impulse signal is not a function but a distribu-
tion. Approximately, the unit impulse signal is
shown in Fig. 6.18 and is described by the follow-
The Laplace transform of the step signal is U(s) ing relation:
= 1/s.
  2.  Ramp signal: It represents a signal which varies 1
 0≤t≤e
directly with respect to the time and is used to d (t) =  e
represent continuously rising or falling signals. The  0 elsewhere
slope of the time evolution of a unit ramp signal is A.
The ramp signal as shown in Fig. 6.17(b), denoted
as r(t), is described by the following relations: δ(t)
At t ≥ 0 1/ε
r(t) = 
0 t<0

The Laplace transform of the ramp signal is


0 ε t
r(s) = A/s2
Figure 6.18 |   Unit impulse signal.
If A is 1, then the signal is called unit ramp signal.
  3.  Parabolic signal: It represents a signal which
varies as square of the time and is used to model The unit impulse function occurs for infinitesimally small
the acceleration commands. The parabolic signal, interval of time and has area under curve unity.
shown in Fig. 6.17(c), denoted as a(t), is described +∞
by the following relation ∫ d (t)dt = 1
−∞
 At2
 t≥0
a(t) =  2 Important concepts like convolution in time domain,
 0 t<0 sampling of signals, etc. involve the properties of impulse
signal. Above all, any linear system with constant
The Laplace transform of the parabolic signal is parameters (linear time invariant) can be completely
characterised in terms of its impulse response. It means
a(s) = A/s3 that impulse signal is a sufficiently rich signal capable of
exciting all the modes of a dynamical system.
When A = 1, then the signal is called unit para- The various test signals, their symbols and Laplace
bolic signal. transform are listed in Table 6.5

Table 6.5 |   List of test signals and their symbols


u(t) r(t) = tu(t)
1 Test-Signals Symbols Laplace Transform
Step u(t) A/s
0 t 0 t Unit step u(t) 1/s
Ramp r(t) A/s2
(a)       (b)
Unit ramp r(t) 1/s2
Parabolic p(t) A/s3
r(t) = t2/2 u(t)
Unit parabolic p(t) 1/s3
Unit impulse d (t) 1
0 t

(c) 6.2.1.2 Relation between Standard Signals

Figure 6.17 |   Standard signals: (a) Unit step, (b) unit The three standard signals described above are related
ramp and (c) unit parabolic signal. to each other mathematically using differential operator:

Chapter 6(487-554).indd 500 3/23/2016 2:20:33 PM


6.2  TIME-RESPONSE ANALYSIS     501

  1.  Ramp signals are obtained via differentiation of Applying the final value theorem of Laplace transforms,
parabolic signal with time. the steady-state error is

d 1
a(t) = r(t) lim e(t) = ess = lim s R(s)
dt t →∞ t →∞ 1 + G(s)
  2.  Step signals are obtained via differentiation of Thus, the steady-state error of unity feedback systems
ramp signal with time. shall differ for different types of inputs, for example,
step, ramp, parabolic, etc. The extension to the study
d of steady-state error for non-unity feedback systems is
r(t) = u(t)
dt not straightforward and one needs to compute the over-
all closed-loop transfer function before commenting on
  3.  Impulse signals are obtained via differentiation of
steady-state error.
step signals with time.

d 6.2.2.1 Steady-State Response of Unity


u(t) = d (t) Feedback Systems
dt
The steady-state behaviour of a system depends upon
6.2.1.3 Order of a System its type and the nature of command signal. The type of
a system can be defined as the multiplicity of the system
The order of the system is order of the differential equa- poles at the origin. If a system has no pole at origin, it is of
tion governing the system. Order of the differential equa- type 0. Type 1 system has one pole at origin, whereas type 2
tion is the maximum value of derivative in the equation. systems will have two poles at the origin. Type of a system
For example, for the following equations decides the steady-state behaviour of dynamical systems.
C(s) 3 Characteristics of the steady-state error shown by dif-
= (Order is 3) ferent system types to standard signals are discussed in
R(s) s3 + 5s2 + 2s + 1
the following subsection.
d 2 x(t) dx(t) Step Input
y(t) = 2
+3 + 4x(t) (Order is 2)
dt dt
For step inputs with R(s) = 1/s, the steady-state error
expression reduces to
6.2.2 Steady-State Behaviour 1 1 1
ess = lim s =
s → 0 1 + G(s) s 1 + lim G(s)
A negative feedback increases the complexity and may s→ 0
render a system unstable with an overall increase in cost. 1
Still it is an important tool to reduce the sensitivity of the =
1 + G(0)
system with respect to plant variations, and also it pro-
vides automatically the steady-state error information. where the term G(0) is known as DC loop gain and is
The mismatch between the system input and feed backed commonly known as position error constant Kp which
output signal, sufficient time after the application of can be defined as
input is called steady-state error. The concept of error is
Kp = lim G(s)
implicitly visible in unity feedback as shown in Fig. 6.19. s →0

1
Steady-state error for step input ess =
R(s) Y(s) 1 + Kp
G(s)
+
− It is clear that if the loop gain G(s) has type 0, then
Kp has some constant finite value, say k, and thus,
ess = 1/(1 + Kp ) = 1/(1 + k) shall also be finite. It means
Figure 6.19 |   A negative feedback system. that a type 0 system can follow step commands but
always admit finite steady-state error.
The tracking error between the input and output is
Ramp Input

E(s) =
1 For ramp inputs with R(s) = 1/s2, the steady-state error
1 + G(s) expression reduces to

Chapter 6(487-554).indd 501 3/23/2016 2:20:47 PM


502     Chapter 6:  CONTROL SYSTEMS 

1 1 1 1 Type 0 Systems
ess = lim s = lim =
s→ 0 1 + G(s) s2 s → 0 s + sG(s) lim sG(0) Systems without any pole at the origin are known as
s→ 0
type 0 systems, for example, G(s) = 1/(s + 1). The error
The term lim sG(s) is known as velocity error constant constant and respective steady-state errors for type 0
s →0 systems are as follows:
and is denoted by Kv.
Kp = k ess = 1/(1 + Kp ) = 1/(1 + k) (finite)
Velocity error constant Kv = lim sG(s)
s →0 Kv = 0 ess = 1/Kv = ∞
1 Ka = 0 ess = 1/Ka = ∞
Steady-state error for ramp input ess =
Kv
This implies that
It is clear that if loop gain G(s) is
  1.  Type 0 systems can follow step inputs but with
finite steady-state error [Fig. 6.20(a)].
  1.  Type 0, then Kv will be zero and, therefore,
  2.  Type 0 systems cannot follow ramp inputs
the steady-state error ess = 1/Kp = 1/0 = ∞ will
[Fig. 6.20(b)].
become infinite. Thus, type 0 systems cannot follow
  3.  Type 0 systems cannot follow parabolic inputs
ramp commands.
[Fig. 6.20(c)].
  2.  Type 1, then Kv will have finite value and thus
the steady-state error ess = 1/Kv = 1/k will also
be finite. It means that the type 1 system can follow
ramp commands but with some finite steady-state 1
error.

Parabolic Input 0 t 0 t
For parabolic inputs with R(s) = 1/s3, the steady-state
error expression reduces to (a) (b)
1 1 1 1
ess = lim s = lim 2 =
s→ 0 1 + G(s) s3 s → 0 s + s2 G(s) lim s2G(s)
s→ 0
2
The term lim s G(s) is known as acceleration error con-
s →0
stant and is denoted by Ka 0 t

Acceleration error constant Ka = lim s2G(s)


s →0 (c)

Figure 6.20 |   Command following for type 0 systems.


Steady-state error for parabolic input ess = 1/Ka
It is clear that if the loop gain G(s) is
  1. Type 0, then Ka will be zero and therefore the Type 1 Systems
steady-state error ess = 1/Ka = 1/0 = ∞ will become
infinite. Thus, type 0 systems cannot follow para- Systems having one pole at the origin are known as type
bolic commands. 1 systems, for example, G(s) = 1/s(s + 1). The error con-
  2.  Type 1, then Ka will be zero and therefore the stants and respective steady-state errors for type 1 sys-
steady-state error ess = 1/Ka = 1/0 = ∞ will become tems are as follows:

Kp = ¥ ess = 1/(1 + ¥) = 0
infinite. Thus, type 1 systems cannot follow parabolic
commands.
  3.  Type 2, then Ka will have finite value and thus Kv = k ess = 1/k = finite
the steady-state error ess = 1/Ka = 1/k will also Ka = 0 ess = 1/0 = ¥
be finite. It means that the type 2 system can
follow parabolic commands but with some finite These imply that
steady-state error.
  1.  Type 1 systems can exactly follow step inputs
The type of system and associated steady-state con- [Fig. 6.21(a)].
stants and steady-state errors are detailed type-wise in   2.  Type 1 systems can follow ramp inputs but with
the following subsections. finite steady-state error [Fig. 6.21(b)].

Chapter 6(487-554).indd 502 3/23/2016 2:21:07 PM


6.2  TIME-RESPONSE ANALYSIS     503

  3.  Type 1 systems cannot follow parabolic inputs as   3.  Type 2 systems can follow parabolic inputs but
shown in [Fig. 6.21(c)]. with finite steady-state error [Fig. 6.22(c)].

1
1

0 t 0 t
0 t 0 t
(a) (b)
(a) (b)

0 t

(c) 0 t
Figure 6.21 |   Command following for type 1 systems.
(c)

Type 2 Systems Figure 6.22 |   Command following for type 2 systems.


Systems having two poles at the origin are known as
The relation between type of the system and the
Type 2 systems, for example, G(s) = 1 / s2 (s + 1) The
steady-state errors/steady-state error constant is sum-
error constants and respective steady-state errors for
marised in Table 6.6.
Type 2 systems are as follows:

Kp = ¥ ess = 1/(1 + ¥) = 0 6.2.2.2 Steady-State Response of Non-


Kv = ¥ ess = 1/¥ = 0 Unity Feedback Systems
Ka = 0 ess = 1/k finite
The steady-state error is a mismatch between the system
This implies that output and the input signal. As far as the feedback type is
  1.  Type 2 systems can exactly follow step inputs unity, the feed backed signal is the output signal. The output
[Fig. 6.22(a)]. of the error detector gives the value of the error signal.
  2.  Type 2 systems can exactly follow ramp inputs However, under non-unity feedback, the feed backed
[Fig. 6.22(b)]. signal is different from the output, and hence steady-state

Table 6.6 |   Steady-state errors for systems of different types


Type 0 Type 1 Type 2 Error Constants
Unit step 0 0 Position error
1
signal constant:
1 + Kp
Kp = lim G(s)
s ®0

Unit ramp ∞ 1
0 Velocity error
signal constant:
Kv
Kv = lim sG(s)
s →0

Unit parabolic 0 ∞ 1
Acceleration error
input constant:
Ka
Ka = lim s2G(s)
s →0

Chapter 6(487-554).indd 503 3/23/2016 2:21:18 PM


504     Chapter 6:  CONTROL SYSTEMS 

error is to be calculated using the closed-loop transfer w n2


function. G(s) =
s2 + (2zw n s) + w n2
Let us take an example system with non-unity feedback
and try to calculate the steady state for unit step input. where wn is called the undamped natural frequency of
Given a system with plant transfer function the system and z is called the damping factor.
1
G(s) =
s(s + 1) ω n2
+ s(s + 2zwn)
The steady state for the unity feedback as well as feed- −
back with H(s) = 5 are as follows:
For unity feedback, steady-state error is zero

1 1 1 ω n2
Kp = lim =∞ ess (∞) = = =0
s →0 s(s + 1) Kp ∞ s2 + 2zwns + ω n2

For non-unity feedback systems, the output is given by Figure 6.23 |   General second-order system
configuration.
Y (s) G(s) 1
Gcl = = =
R(s) 1 + G(s)H(s) s(s + 1) + 5
The damped natural frequency of the system may be
1 1 1
Y (∞) = lim s = = 0.2 defined as w d = w n 1 + z 2 rad/s. The transformed unit
s → 0 s(s + 1) + 5 s 5
step response of the system may be calculated by multi-
plying the Laplace transform of the unit step signal with
Thus, the steady-state error is
G(s) as
ess(∞) = 1 − Y (∞) = 1 − 0.2 = 0.8
1 w n2 1
Y (s) = G(s) ⋅ = 2 ⋅
Note: For the calculation of steady-state error for non- s s + 2zw n s + w n2 + w n2 s
unity feedback systems, the steady-state output must be
subtracted from the steady-state input. Using the inverse Laplace transform, one can obtain the
solution for y(t) as

e−zw n t 
( )
6.2.3 Transient Response of Second-Order
Systems y(t) = 1 − sin w n 1 − z 2 t + f 
2 
 
1−z
It is well known that a second-order polynomial with real
 1−z2 
f = tan−1  
coefficients may either have real roots or a pair of com-
plex conjugate roots. Roots of the characteristic equa-  z 
tion are the poles of the closed-loop system and describe  
the nature of system response.
The transient response of a typical underdamped second-
The transient response is mainly contributed by the order system is shown in Fig. 6.24.
dominant pole(s). If roots are real, then the response of
system will be exponentially decaying or rising depend- 5% tolerance band
ing upon whether the system is stable or not. y(t)
A wide class of systems can be modeled in terms of Mp
1
the following open-loop transfer function.

w n2
T (s) =
s(s + 2zw n )

Under the unity feedback configuration, the overall t


0 tr tp ts
closed-loop system transfer function may be represented
as shown in Fig. 6.23, and described in terms of following Figure 6.24 |   Unit step response of a underdamped
transfer function: second-order system.

Chapter 6(487-554).indd 504 3/23/2016 2:21:31 PM


6.2  TIME-RESPONSE ANALYSIS     505

Dynamical systems can be characterised in terms of their Mp


transient response properties such as rise time, peak 1
time, peak overshoot settling time, etc. Such important
transient response measures are used for analysis and
drafting specifications for subsequent control design and
are detailed in the following list: z
0 1
  1.  Rise time (Tr): It is the time taken by the system
response to evolve from 0% of its final value to Figure 6.25 |   Variation of peak overshoot with
100% of its final value for an underdamped system. damping factor.
However, for the overdamped systems, the rise
time is the time taken by the system response to
  5.  Settling time (Ts): It is the time taken by the
rise from 10% of its final value to 90% of its final
system response to reach within 2% or 5% of its
value.
final value. The 2% range in the proximity of the
 1−z2 
p − tan−1 
final value (i.e., for a unit step response, it means

p −j  z  within value 0.98 to 1.02) is called the 2% toler-
Tr = = seconds ance band. Typically systems take time almost
wd wn 1 − z 2 four times their time constant to reach 2% toler-
ance band.
  2.  Delay time (T): It is the time taken by the Similarly, 5% range in the proximity of the final
system response to evolve from 0% to 50% of its value (i.e., for a unit step response, it means within
final value, and is given by value 0.95 to 1.05) is called the 5% tolerance band.
Typically systems take time almost three times
1 + 0.7z
T = seconds their time constant to reach 5% tolerance band.
wn
The settling time is given by:
  3.  Peak time (Tp): It is the time required by an (a) Settling time for 2% tolerance band Ts = 4t =
underdamped system response to reach its first 4
= (Four times system time constant)
maximum value. It can be obtained by making the zw n
derivative of time response with respect to time (b) Settling time for 5% tolerance band Ts = 3t
zero dy(t)/dt = 0. Using this the peak time is cal- 3
= (Three times system time constant)
culated as zw n
p p
Tp = = seconds
wd
6.2.4 Nature of System Response vs. Location of
wn 1 − z 2 Closed-Loop Poles
  4.  Peak overshoot (Mp): It is usually measured in From the above discussion, we have seen that the nature
terms of percentage of the error between the maxi- of step response of a system depends upon its damp-
mum value of system response and the final value. ing factor z. Depending upon the range for the damping
It may be defined as factor, the step response may be classified in terms of
y(Tp ) − y(∞) the following:
Mp =
y(∞)   1.  Overdamped response: It occurs when damping
Substituting the value of Tp = p /w d in the step factor z > 1 and the response is non-oscillatory. In
response expression and considering the final value this case, both the poles of the closed-loop system
as 1, we get are real and have negative real parts.
  2.  Critically damped response: It occurs when
Peak overshoot M p = e−pz / 1−z 2
damping factor z = 1. The response is still non-
oscillatory. In this case, both the poles of the
% Peak overshoot %M p = 100 × e−pz / 1−z 2 closed-loop system are real and equal.
  3.  Underdamped response: It occurs when the
The peak overshoot depends upon the value of damping factor lies between 0 < z < 1. The poles of
damping factor z. The graph between the peak the closed-loop underdamped system are complex
overshoot and the damping factor is shown in conjugate with negative real parts. The response
Fig. 6.25. now becomes oscillatory.

Chapter 6(487-554).indd 505 3/23/2016 2:21:43 PM


506     Chapter 6:  CONTROL SYSTEMS 

  4.  Undamped response: It occurs when the damp- y(t)


ing factor z = 0 and the system admits undamped 1
sustained oscillations.
The main points of transient response are summarised
as
0 t
  1.  Transient response dictates the nature and speed
Figure 6.26 |   Step response of non-minimum
of response of the system.
  2.  Generally, the systems are designed with damping phase systems.
factor 0.7, that is, z = 0.7.
  3.  Low value of damping factor z results in poorly
damped oscillatory response whereas large value z
reduce the risk time, peak overshoot increases and
vice versa.
  7. The time constant of a system t is the time taken
makes the system sluggish.

  4.  The system response is unstable, that is, the time
by the response to reach 63.7% of its final value
for the first time and is given by t = 1/zwn.
response diverges with time in case system has
right half-plane poles.
    8. Settling time of a system’s response may be
  5.  The non-minimum phase system, that is, systems
defined in terms of its time constant.
with right-half-plane zeros have undershoots in the
    9. Settling time for 5% tolerance band is three times
the system time constant t.
transient response, that is, the response first goes
in negative direction and the positive as shown in
  10. Settling time for 2% tolerance band is four times
the system time constant t.
Fig. 6.26.
  6.  Sometimes the requirements are mutually conflict-
ing, for example rise time and peak overshoot. Both The location of system poles and corresponding unit-step
cannot be made small simultaneously. If we try to transient response are summarised in Table 6.7.

Table 6.7 |   Dominant pole pair location and transient response

Nature of Response Location of Roots of Unit Step Response Remarks


Characteristic Equation
on the Complex Plane
Overdamped response jw y(t) System response is
z>1 exponentially rising
1
Roots are negative,
real and unequal s2 s1 0 s

0 t
Critically damped jw System response is
response z = 1
y(t)
exponentially rising
1
Roots are negative,
real and equal s1 = s2 0 s

0 t

Underdamped response jw y(t) System response is


0<z<1 oscillatory but oscillation
jw1 1 damp out with evolution
Roots are complex
conjugate with of time
negative real part s1 0 s
−jw1 0 t

(Continued)

Chapter 6(487-554).indd 506 3/23/2016 2:21:51 PM


6.3  STABILITY OF CONTROL SYSTEMS     507

Table 6.7 |   (Continued)

Nature of Response Location of Roots of Unit Step Response Remarks


Characteristic Equation
on the Complex Plane
Undamped response y(t) System response is
z=0
jw
purely oscillatory, as
jw1 2
Roots are purely there is no damping at
imaginary and are on 1 all.
jw axis 0 s
−jw1 0 t

6.3  STABILITY OF CONTROL   3.  The presence of repeated poles on the jw axis makes
SYSTEMS the system unstable.
  4.  The presence of non-repeated poles on the jw
axis makes the system marginally stable. It
6.3.1 Bounded Input Bounded Output Stability means that such system will admit sustained
oscillations.
A system is said to be stable in case a bounded excita- The task remains to find the location of closed-loop poles
tion produces a bounded response. in a unity feedback configuration, given the location of
open-loop poles. One way to solve this problem is to use
x(t) y(t) Routh—Hurwitz criterion.
g(t)
System

Figure 6.27 |   System with input and output. 6.3.2 Routh−Hurwitz (RH) Stability Criterion

Figure 6.27 shows a system with input x(t) and output, The overall closed-loop transfer function of an open-loop
y(t). It is bounded input bounded output stable, if a system with gain G(s) in negative feedback configuration
bounded input produces a bounded output, that is, with elements H(s) can be described as

∞ ∞ G(s)
if ∫ x(t) dt < M , ⇒ ∫ y(t) dt < N ; M , N < ∞ Gcl (s) =
1 + G(s)H(s)
0 0
Alternatively, in the absence of any input signal, the Then the roots of the characteristic equation
steady-state response must decay to zero. The stability 1 + G(s)H(s) = 0 are also the poles of the closed-loop
of a system is dictated by the location of poles in the system.
complex plane. Given a characteristic equation, Routh−Hurwitz
Some observations about the relation between the system (RH) stability criterion provides a systematic tabular
stability and the location of closed-loop poles are listed way to adjudge the stability of the overall system.
as follows: It states that: the necessary and sufficient condi-
tion for stability is that all the elements in the first
  1.  A system is stable if all the poles are located in
column of Routh array should be positive. If this con-
the open left half of s-plane. It means that all the
dition is not satisfied, then the system is unstable and
poles have negative real part. However, system
the number of sign changes in the elements of first
may have right half-plane zeros. The presence of
column of Routh array corresponds to the number of
right half-plane zero does not affect the system sta-
roots of characteristic equation in the right half of
bility, but makes the system non-minimum phase.
the s-plane.
Recapitulate that non-minimum systems experi-
ence an undershoot in their transient response. Let the characteristic equation is given as
  2.  The presence of right half-plane poles makes the a0 s4 + a1s3 + a2s2 + a3s1 + a4 = 0, then the RH array
system unstable. is formulated as in Fig. 6.28.

Chapter 6(487-554).indd 507 3/23/2016 2:21:58 PM


508     Chapter 6:  CONTROL SYSTEMS 

s4 a0 a2 a4 (this is because as e is very small number, any finite


number can be neglected in comparison with 1/e)
s3 a1 a3 0
Finally upon completion of RH array, substituting
a1a2 - a0a3 a4 0 e → 0+, we found that the first column entry of s2 row
s2 a1 is negative.
a1a2 - a0a3 There are two sign changes in first column, s3 row to
a1 a3 - a1a4 s row and s2 row to s1 row. Hence, there are two roots of
2
s1 0 0
a1a2 - a0a3 the characteristic equation lying in right half of s-plane,
a1 and the corresponding system is unstable.
Case 2: All the entries of a row are zero: This
s0 a4
indicates the presence of purely imaginary poles on the
Figure 6.28 |   Formulation of Routh—Hurwitz array
jw axis. Here also it is not possible to proceed for the
computations of further entries of RH array. In order
from the chateristic polynomial.
to proceed, we differentiate the polynomial represented
by the row just above the row containing all the zero
The RH array is formulated by separating the even and entries. The coefficients of the differentiated polynomial
odd polynomials of the characteristic polynomial. The are used to replace the all zero row, and further compu-
two polynomials form the first and second row of the tation of RH array proceeds.
table. The RH array is computed as per the standard Consider a system with characteristic equation
procedure. For a system to be stable, all the entries in
the first column must be of the same sign. s5 + 4s4 + 8s3 + 8s2 + 7s + 4 = 0
The RH array can be formulated as
6.3.2.1 Special Cases s5 1 8 7
4
Case 1: Row containing first column entry zero, but s 4 8 4
other entries in row non-zero. 3
s 6 6 0
In such a case, we cannot proceed for the computa- 2
s 4 4 0
tion of the RH table. To resolve this, the zero entry is s 1
0 0 0
replaced by a small positive number e and resume the
calculation of the RH array entries. Once the table is All the entries in s-row become zero, hence we formulate
completed, we check the sign of the first column entries an auxiliary equation from the s2 row as
by putting the limit e → 0+ and check their sign. The
number of right half-plane poles is equal to the number    A(s) = 4s2 + 4 = 0
of sign changes. A(s) = s2 + 1
which has the roots at s = ± j1, and it indicates that the
Consider a system with characteristic equation:
s + 2s4 + 2s3 + 4s2 + 11s + 10 = 0 system is marginally stable with sustained oscillations
The RH array can be formulated as of 1 rad/s.
Differentiating the auxiliary polynomial with respect
5 to s, we can formulate the s-row and proceed as
s 1  2 11
4 2  4 10 dA(s)
s
= 2s
s 3
0  e  6  0 ds
s 2
(4e − 12)/e = −12/e 10  0 s5 1 8 7
1 2 4 4
s 6 + 10e /12  0  0 s 4 8
0 3
s 10  0  0 s 6 6 0
2
s 4 4 0
The first column entry in s3 row becomes zero and is 1
replaced with very small positive number e, and RH s 2 0 0
array tabulation proceeds in same manner. The first s 4 0 0
entry in s2 row becomes
Since all the entries in the first column of RH array
(4e − 12)/e = 4 − 12/e = −12/e are non-negative, the system corresponding to the given

Chapter 6(487-554).indd 508 3/23/2016 2:22:01 PM


6.4  ROOT LOCUS TECHNIQUE     509

characteristic equation is stable. Some of the observa-   4.  Design of controller to obtain a desired behaviour.
tions for RH Stability formulation are as   5.  Effect of positive feedback.
  1.  The RH array is used to calculate whether the
system is stable or not depending upon the sign 6.4.1 Concepts of Root Locus − Intuitive
changes in its first column. Example
  2.  In root locus technique, RH array is generally used
to find the intercept of the root loci on the jw axis Let us consider a unit feedback closed-loop system as
and computation of the gain K that corresponds to shown in Fig. 6.29.
the purely imaginary poles.
  3.  Given any open-loop configuration, the RH array R(s) Y (s)
can be used to compute the gain of the propor- KG(s)
tional controller to achieve stability. +

  4.  To find the critical gain of the system and
undamped oscillation frequency.
Figure 6.29 |   Unity feedback system with variable
gain K.
6.4  ROOT LOCUS TECHNIQUE
where G(s) is the plant transfer function, and gain K is
Given an open-loop transfer function, the stability and allowed to vary from 0 to ∞. Let N(s) and D(s) be the
performance of a closed-loop system depends upon the numerator and denominator polynomials of the open-
poles and zeros of the closed-loop system. The stability loop transfer function. Hence
of the closed-loop system depends only upon the poles or
the roots of the characteristic equation N (s)
G(s) =
D(s)
1 + G(s)H(s) = 0
The roots of N(s) and D(s) are called the zeros and poles
The confinement of all the roots of the characteristic
of the open-loop transfer function, respectively.
equation to the left half of s-plane implies that the
system is stable. Numerical techniques can be applied Under unity feedback configuration, the closed-loop
to find the roots of the characteristic equation, and transfer function of the system is given by
subsequently draw inferences on system stability and
KG(s) KN (s)
performance. T (s) = =
1 + KG(s) D(s) + KN (s)
However, numerical results do not give any idea about
how the roots of the characteristic equation will vary in Thus, the term D(s) + KN(s) is the characteristic poly-
case if some of its parameters/coefficients change. nomial of the closed-loop system. The roots of the char-
The root locus technique developed by R.W. Evans, acteristic equation p(s) = D(s) + KN(s) = 0 are the
provides a systematic way to sketch the root locus plot poles for the closed-loop system.
for a given system when some of its parameter varies.
The technique provides a quick qualitative estimate   1.  For K = 0, the characteristic equation becomes
about the nature of closed-loop system. The performance D(s) = 0. It means that when gain K = 0, the
and nature of the closed-loop system response can be closed-loop poles are located at the same positions
adjudged by inspecting the root locus diagrams. The as those of the open-loop poles.
root locus technique can be used for sensitivity studies.   2.  For K = ∞, the characteristic equation becomes
Thus, root locus is a systematic graphical technique to N(s) = 0. It means that K → ∞, the closed-loop
have a qualitative estimate of the location of the roots poles approach the open-loop zeros.
of a polynomial. It provides a quick qualitative estimate The question arises as to how the roots of the character-
about the system stability and performance. Root locus istic equation change when the parameter K varies from
technique is useful for adjudging: 0 to ∞?
  1.  The movement of closed-loop system poles when The answer lies in root locus technique, which is
certain system parameter varies, given the location essentially a graphical technique to get fair idea about
of open-loop poles and zeros. variation of location of roots of the characteristic equa-
  2.  Effect of adding poles and/or zeros to the open- tion as K varies from 0 to ∞. The basic idea for the
loop transfer function of the system. root locus technique can be developed with the following
  3.  Sensitivity of the roots of the characteristic equation. example.

Chapter 6(487-554).indd 509 3/23/2016 2:22:06 PM


510     Chapter 6:  CONTROL SYSTEMS 

Consider a unity feedback type 1 second order system product of phasor length of the phasors drawn from
with open-loop transfer function as this point to zeros, divided by the phasor lengths
from the poles is unity.
K
G(s) =   2.  Phase angle condition: The root locus can exist
s (s + 1) at a point s if and only if ∠KG(s) = −180(2k + 1),
that is, the net angle casted (sum of angles casted
The location of closed-loop poles is given by the roots of at given point due to zeros minus the sum of the
the characteristic polynomial angles casted by all the poles) at the point s is odd
multiple of −180°[for k = 0, 1, …(n − m)].
p(s) = s2 + s + K
Few important points are summarised as follows:
The location of the roots of p(s) as the gain K varies is
  1.  The K times the product of distance of that point
given by
from the zeros divided by the product of distance
−1 1 − 4K from poles is equal to unity.
s1 , s2 = ±   2.  The net angle subtended from zeros minus the net
2 2
angle subtended from poles, is equal to 180° or
The following observations can be made (2n + 1)180°
  1.  When K = 0, the roots of p(s) are located at 0   3.  The root locus exists on the real axis if and only
and −1. if the total number of poles to the left of it is odd.
  2.  As K increases and K ≤ 0.25, the root of p(s) at 0   4.  Since a polynomial with real coefficient can either
starts shifting towards left, whereas the root of p(s) have real roots or real as well as complex roots,
at −1 starts shifting towards right. the complex roots always occur in conjugate pair.
  3.  So far as the value of K ≤ 0.25, the roots of the Therefore, the root locus is symmetrical about the
polynomial are real. However, as K > 0.25, the real axis.
roots become complex conjugate.
  4.  For complex conjugate roots of p(s), the real part 6.4.2 Rules of Sketching Root Locus
of roots is always constant and is −0.5.
Given an open-loop system, it is a tedious task to find
If we smoothly plot the variation in the roots of p(s) out all those points in the s-plane which satisfy the mag-
with respect of K, we obtain the locus of the closed-loop nitude and angle conditions. The root locus technique
system poles as shown in Fig. 6.30. The resulting locus provides a systematic graphical way to roughly sketch
is called root locus. the path traversed by the closed-loop poles. In general,
we are interested in finding the values of gain K keeping
K→∞

in view of stability and performance requirements:


  1.  The values of gain K, for which the closed-loop
poles will enter into, right half of s-plane.
K=0 K=0   2.  The values of gain K, for the points where closed-
loop poles need to be placed.
−1 0
The rules for sketching the rough root locus plot are
K→∞

discussed in the following subsections.

6.4.2.1 Root Locus is Symmetrical About


Figure 6.30 |   Root locus plot of G(s) = 1/s (s + 1). the Real Axis
The root locus technique is a set of rules to sketch the This rule is based upon the concept that the physi-
locus of roots of the characteristic polynomial, given the cal systems are modeled in terms of transfer functions.
location of open-loop poles and zeros. These transfer functions are the ratio of two polyno-
The root locus for the roots of 1 + KG(s) = 0 admits mials in complex frequency s with real coefficients. A
the following condition 1 + KG(s) = 0, which means polynomial with real coefficients can either have real or
that root locus will exist only on those values of s for complex roots. Thus, the roots of the polynomial with
which KG(s) = −1. Since s is a complex number, the real coefficient, if complex, shall always occur in complex
condition can be broken into the following two types: conjugate pair.
  1.  Magnitude condition: The root locus can exist It means that if −1 + j2 is a root of some polynomial,
at a point s if and only if |KG(s)| = 1 K times the then −1 − j2 shall also be a root of that polynomial.

Chapter 6(487-554).indd 510 3/23/2016 2:22:11 PM


6.4  ROOT LOCUS TECHNIQUE     511

Thus, a system with a root above the real axis shall have
another root located below the real axis, as if it is mirror
image of the previous one. 0° 0° −180° −180° 180°

Therefore, it is concluded that, since the roots of a


polynomial with real coefficients always occur in conju-
Point of
gate pair, the root locus is always symmetrical about the
consideration
real axis, as is clear from Figure 6.30 where in there is
branch moving towards 90° then there must be another Figure 6.31 |   Angle contribution at a point by poles
branch moving towards -90°. and zeroes.

6.4.2.2 Number of Poles are Always Equal 6.4.2.5 Break-away Points


to Number of Zeros
Consider n-branches of root locus moving towards each
Our current understanding about the transfer function other. They meet at a point and, thereafter, move along
says that zeros are the roots of the numerator poly- n different directions. The point on the s-plane where
nomial while the roots of denominator polynomial are different branches of root loci come, meet and depart, is
called poles. It means that for a given transfer function known as break-away points.
G(s) = 1/s + 1, there is one pole at s = −1 and no Let us again have a look at the earlier example described
zeros. Then, how can the number of poles be equal to by the open-loop transfer function G(s) = 1/s(s + 1). As
number of zeros? This is because we have not yet con- gain is increased from K = 0, two loci originate from s = 0
sidered zeros at s = ∞. It is clear that for s = ∞, we get and s = −1 and move towards each other on the real axis
G(s) = 0 and hence one zero of the considered transfer as shown in Fig. 6.32. They meet each other at s = −0.5,
function lies at s = ∞. when the gain K becomes equal to 0.25. Then the point
s = −0.5 on the s-plane is known as the breakaway point
for the given system.
6.4.2.3 Number of Root Locus Branches

K→∞
Recapitulate that
  1.  For K = 0, the closed-loop poles are located at the
position of open-loop poles. K=0 K=0
  2.  For K = ∞, the closed-loop poles are located at −1 0
K→∞

open-loop zeros.
Break-away
The root locus originates from the open-loop poles and point −0.5
terminates at the open-loop zeros as the parameter K is
varied from 0 to ∞. It means that the branches taking Figure 6.32 |   Break-away point.
part in root locus are nothing but a path traversed by
the location of closed-loop poles as K varies. Hence, the
Any further increase in gain shall result into two root
loci coming out of the point s = −0.5, one at +90° and
number of branches in the root locus is always equal to
the other at −90°.
the number of closed-loop poles.

The break-away points are the points where multi-


6.4.2.4 Condition for Existence of Root ple roots of the characteristic equation occur, and the
Locus on Real Axis Segments gain parameter K is maximum with respect to s at such
points. At break-away point, we have dK/ds = 0
This rule states that net angle contribution at a point on
root locus due to open-loop zeros and poles must always
be ±(2K + 1)180°.
6.4.2.6 Break-in Points
The angle contribution of a pole or zero, which lies to Figure 6.33 shows two branches of root locus moving
the left of a given point on real axis in s-plane is always towards the real axis. They meet each other at a point
zero, as shown in Fig. 6.31. and thereafter move along the real axis. The point, at
The angle contribution of a pole or zero, which lies to which two loci meet and then move along the real axis, is
the right of a given point on real axis in s-plane is −180° called the break-in point. Break-in points are the points
and +180° respectively, as shown in Fig. 6.31. which have minimum gain parameter K with respect

Chapter 6(487-554).indd 511 3/23/2016 2:22:19 PM


512     Chapter 6:  CONTROL SYSTEMS 

to  s. Thus, the break-in points can be evaluated using 6.4.2.8 Gain at a Point of Root Locus
the same condition dK/ds = 0.
The gain at any point on the root locus can be calculated
graphically as
Product of phasor lengths from poles
Gain K =
Product of phasor lengths from zeros
Break-in Break-away
point Let s be the point at which K is to be calculated, and s =
point
p1, p2 are the open-loop poles and s = z1 is the open-loop
Figure 6.33 |   Break-away and break-in points. zero. Then the value of gain K at s is
The only way to distinguish between a break-away and s − p1 s − p2
K=
break-in point is to evaluate the second order differential s − z1
of K with respect to s, and judge for minima or maxima.
If there are no open-loop system zeros, then the denomi-
nator of above relation may be taken as 1.
6.4.2.7 Root Locus Proceeds to Zeros at
Infinity Along Asymptotes with Angles fA 6.4.2.9 Marginal Stability − Intersection of
Centred at sA Root Locus with jw-Axis
The root locus starts from the open-loop poles and termi-
Recall that for a given system G(s) with n number of
open-loop poles and m number of open-loop zeros, n − m
nates at the open-loop finite as well infinite zeros. Let us
consider a system with n number of poles and m number
branches of root locus will approach towards infinity
along n − m number of asymptotes centered at sA with
of zeros. Since, only proper transfer functions are con-
sidered, therefore n ≥ m always. Suppose n > m, then
the system has n − m number of infinite zeros. It implies angles
180(2k + 1)
.
that n − m zeros of the open-loop transfer function lies n−m
at ∞. Since root locus terminates at open-loop zeros, it Table 6.8 lists the angle of asymptotes for different
means that it will move towards ∞. The root locus then values of n − m.
approaches ∞ along n − m number of directions along
the asymptotes. The point where the asymptotes meet Table 6.8 |   Pole-zero difference and angle of
on the real axis is called centroid. asymptotes

The centroid, that is, the starting point of the asymp- n−m Angle of Asymptotes
totes lies on the real axis at a point sA where sA is given by 1 180
∑ Real part of poles − ∑ Real part of zeros 2 +90, −90
sA =
n−m 3 +60, −60, −180
In addition, as K → ∞, the root locus branches become
straight lines originating from the centroid with angles qA’s.
4 +45, −45, +135, −135

Angle of asymptotes are given by Thus, in case if the given system is open-loop stable,
that is, all the open-loop poles are in left half of s-plane,
for k = 0, 1, …(n − m)
180(2k + 1)
qA = then the closed-loop system shall also be stable if
n−m n − m < 3. This is because, as K → ∞, the root locus
As K → ∞ say along angle qA, then the net angle casted moves along the asymptotes with angles given in table
by the poles and zeros shall be (n − m)qA. For a point above. Therefore, they can never centre the right half of
along that direction to exist on root locus, the net angle s-plane, if centroid is located in left half of s-plane.
contribution must be 180(2k + 1). In case if n − m ≥ 3, then the asymptotes enter into
It is possible only when contribution from each pole or the right half of s-plane and gain K increases. It becomes
180(2k + 1) desirable to find the value of the critical gain K for which
zero are . Therefore, the angle of asymptotes
n−m
the given system will intersect with jw-axis.
shall always be Remember, when the close loop poles occur on
jw-axis, the system is marginally stable, and the value of
180(2k + 1)
qA = gain K at which the condition occurs can be found using
n−m RH array formulation.

Chapter 6(487-554).indd 512 3/23/2016 2:22:35 PM


6.4  ROOT LOCUS TECHNIQUE     513

The steps to compute the gain K are as 1


G(s) = 2
  1.  Formulate the characteristic equation for the s(s + 2s + 2)
closed-loop system as 1 + KG(s) = 0.
  2.  Formulate the Routh−Hurwitz array. The open-loop poles of the system are located at s = 0,
  3.  Enforce entries in the row corresponding to s1 to go −1 ± j1.
zero for a suitable value of K.
Let us calculate the angle of departure at the com-
  4.  Formulate the auxiliary equation from the s2 row
plex pole at s = −1 + j1. The point p as shown in the
and find out the frequency of oscillation for gain K
figure is chosen in the vicinity of s = −1 + j1, and let
fdep be the angle which the point p makes with the
found in previous step.
pole at s = − + j1. The point p makes an angle of 135°
6.4.2.10 Angle of Departure and Angle of with the pole at s = 0 and an angle of 90° with the
Approach pole at s = −1 − j1, therefore as per the angle condi-
tion we have
We know that root locus originates from the open-loop
poles and terminates at the open-loop zeros. For the fdep + 90 + 135 = 180
fdep = −45
poles and zeros on the real axis, the angle of departure
and angle of approach are decided as per the existence
condition of the root locus on the real axis.
φdep = −45°
However, the situation becomes tricky in the case
of complex conjugate poles and zeros. Root locus may 135° + 90° + φdep = 180°
φdep = −45°
depart from a complex pole at any angle between 0° and
360°. The angle at which the root loci leave an open-loop 135°
complex pole is called angle of departure. Similarly, the
angle at which the root loci arrive at a complex zero is
called angle of approach. The problem remains to find
90°
the angle it leaves the open-loop pole, and to this the
angle condition for existence of root locus at a point in
Figure 6.35 |   Angle of departure for G(s) = 1/s(s2 +
s-plane comes to our rescue.
A point p is chosen near to the location of the pole 2s + 2).
at which angle of departure is to be calculated as shown
in Fig. 6.34, and let this point is at angle fdep from the
nearby pole. Since for the point p to exist on root locus, the net angle
casted by all the poles and zeros must be 180°, hence for
the given problem the angle of departure comes out as
45° as shown in Fig. 6.35.
φdep

6.4.3 Systems with Time Delay

Time delays are inherent in all the physical systems. The


inherent inertia present in the system causes delayed
system response. Further, systems with material transfer
Figure 6.34 |   Angle of departure from a complex pole. such as transportation, fluid flow, heat transfer, etc. are
inherently time delay systems.
For this point to exist on the root locus, the contribution A system is said to have a time delay of T seconds, if
of the angles from poles minus the contribution of angle the current output y(t) is produced by the input signal
from all the zeros must result in angle 180°. Therefore, T s earlier, that is, y(t) = f[u(t − T)]. For the sake of
we can write angle condition as simplicity, we assume the function f(.) to be a liner-time
invariant one.
fdep + ∑ Angle casted by other poles In terms of transfer function, the delay of T seconds
−∑ Angle casted by the zeros = 180° is modeled as e−Ts.
The exponential functions in s are irrational functions,
Figure 6.35 shows the location of the open-loop poles and and therefore, root locus technique cannot be directly
zeros for the open-loop system applied to such functions. However, the irrational

Chapter 6(487-554).indd 513 3/23/2016 2:22:41 PM


514     Chapter 6:  CONTROL SYSTEMS 

function e−Ts can be approximated in terms of ratio of Ke−2s


two polynomials in s using Pade’s approximation, as G(s) =
s
−Ts
e 2 1 − sT /2 s − 2/T then using the Pade’s approximation we have
e−Ts = = =−
+
Ts 1 − sT /2 s + 2/T −K(s − 1)
e 2 G(s) =
s(s + 1)
The systems with right half-plane zeros are known as Using the rules detailed above, the root locus plot of the
non-minimum phase systems. Such systems are known to system is as shown in Fig. 6.36.
experience undershoots in their step response. In addition,
the gain in the open-loop transfer function is negative.

6.4.4 Root Locus for System with Gain −K


−1 0 1
The following two situations demand for the develop-
ment of root locus for the negative gain K.

Figure 6.36 |   Root locus plot for G(s) = Ke-2s/s.


  1.  Root locus for the positive feedback systems.
  2.  Root locus for systems with dead time.
The closed-loop transfer function for system with nega-
tive gain parameter is given as 6.5  FREQUENCY RESPONSE
KG(s) ANALYSIS
Gcl (s) =
1 − KG(s)
The characteristic equation is given by 1 − KG(s) = 0 This section discusses the frequency response analysis
⇒ KG(s) = 1 = 1∠360°. using Nyquist plots and Bode plots. The major advan-
tage of using frequency response analysis is that it does
The root locus plot can be developed for such system
not require exact location of poles and zeros, that is,
with minor change in angle condition, that is, use of 360°
analytical model of the system. Most of the times, fre-
in place of (2k + 1)180°. The rules to sketch root locus
quency response of systems can be obtained using labora-
for negative gain parameter variations are as follows:
tory testing. The overall system behaviour can be easily
  1. Root locus exists on the real axis if and only of the interpreted from the frequency response of the open-loop
number of poles and zeros to the right of it are even. system. The stability can be directly interpreted from
  2.  Root locus still remains symmetrical to real axis. these plots, whereas relative stability can also be mea-
  3.  Root locus begins from the open-loop poles and sured using the stability margins: phase margin and gain
ends at open-loop finite as well as infinite zeros. margin. Important concepts such as resonant peak and
  4.  There will be (n − m) root locus branches approach- resonant frequency along with gain and phase margin are
ing the zeros at infinity. used to sketch the parallel between frequency domain
  5.  The angle of asymptotes is specification and time domain specifications. However,
2kp it is commented here that such an analogy between two
qA = k = 0, 1,…, n − m − 1
n−m
types of specifications is not direct, and sometimes tedious
to deal with. Effect of addition of poles and zeros to the
  6.  The centroid is given by open-loop transfer functions has also been discussed.

sA =
∑ Real part of poles − ∑ Real part of zeros Let recapitulate some important points
n−m
  1. Root locus plots are used for studying the transient
  7.  Angle of departure/approach response and stability of the closed-loop system from
fdep = net contribution due to other poles and zeros
the location of poles and zeros of open-loop system.
  2.  The Routh−Hurwitz criterion is used to adjudge
farr = (-1) × net contribution due to other poles the stability, as well as marginal stability of the
and zeros closed-loop system.
Let us consider an integrator system with gain K and a   3.  Both root locus as well as Routh−Hurwitz criterion
finite delay of 2 seconds, represented with the following requires the exact location of poles and zeros of
transfer function open-loop transfer function.

Chapter 6(487-554).indd 514 3/23/2016 2:22:51 PM


6.5  FREQUENCY RESPONSE ANALYSIS     515

  4.  On the other hand, frequency response analysis is


s-Plane Contour F(s) Plane Contour
based upon the frequency response of the open-loop
system and does not require the exact location of s −1
3. F (s) =
poles and zeros of the system. (s − 2)(s − 4)
  5.  The frequency response of an open-loop system can jv
be obtained via laboratory experiment and does jw
not explicitly require system model.
  6.  Using the frequency response of the open-loop s u
system function in the form of G(jw), the stabil-
ity can be determined from the graphical frequency
domain analysis procedures like Nyquist plots,
Bode plots and Nichol charts. s − 2s + 15
4. F (s) =
  7.  Frequency domain response provides various mea- s−2
sures of relative stability such as gain margin and
phase margin etc. The bandwidth of the system jw jv
also plays an important role in determining the
speed of response of the system.
s
u
6.5.1 Polar Plots

It is well known from the study of circuits that the fre-


quency response of a network excited using sinusoidal
inputs can be studied using the polar plots. The polar s2 − 2s + 5
5. F (s) =
plot is the plot of magnitude and phase of the system s2 − 4s + 8
function as frequency varies from 0 to ∞. The polar
jw jv
plots along with the principle of argument can be used
to sketch the Nyquist plot of an open-loop system
gain, from which stability of the overall system can be s
adjudged using Nyquist stability criterion. The s-plane
and F(s) place contours are depicted in Table 6.9.
u
Table 6.9 |   Polar plots
s-Plane Contour F(s) Plane Contour
1. F (s) = s − 2s + 5
2

6.5.1.1 Typical Sketches of Polar Plots


jw jv
Polar plot of different types and their order is shown in
Fig. 6.37. Polar plots start with type number and ends
s with order.
u

Type 3
1
2. F (s) =
s2 − 4s + 8
jw 3rd Order 4th Order
jv

Type 2 Type 0 2nd Order 1st Order


s u

Type 1

Figure 6.37 |   Polar plots.

Chapter 6(487-554).indd 515 3/23/2016 2:23:11 PM


516     Chapter 6:  CONTROL SYSTEMS 

The typical sketches of polar plots are depicted in the total gain is G(s)H(s). Then, the closed-loop transfer
Table 6.10. function of the system is given by
Table 6.10 |   Type and order of polar plots
Y (s) G(s)
=
Type Sketch R(s) 1 + G(s)H(s)
Type: 0, order: 1 −270° The denominator of the equation is known as character-
1
−180°
G(s) = istic equation. The roots of the characteristic equation
1 + st 0
are known as poles of the closed-loop system. Let the
ω increasing open-loop transfer function G(s)H(s) is given by

−90° K(s + z1 )(s + z2  (s + zm )


G(s)H(s) =
Type: 1 order: 2 −270° (s + p1 )(s + p2 ) (s + zn )
1
G(s) =
−180°
Substituting the relation in closed-loop transfer function
s(1 + st) 0
relation, we get
ω increasing
Y (s) K(s + z1 )(s + z2 ) (s + zm )
=
R(s) (s + p1 ) (s + zn ) + K(s + z1 ) (s + zm )
−90°
It means that the
Type: 0, order: 2 −270°
1   1.  Zeros of the closed-loop system are same as the
G(s) = −180° 0° open-loop transfer function zeros.
(1 + st1 )(1 + st2 )
  2.  The poles of the closed-loop system are the roots
ω increasing of the equation

−90° (s + p1) … (s + pn ) + K(s + z1) … (s + z m) = 0

Type: 0, order: 3 Nyquist stability criterion provides an estimate about


−270° the stability of the closed-loop system using a graphical
1 procedure based upon the principle of argument.
G(s) =
(1 + st1)(1 + st2 )(1 + st3 )
−180° 0
6.5.3 Principle of Argument
ω increasing
Let a closed clockwise contour in the s-plane enclosed n
number of poles and m number of zeros of an open-loop
−90° transfer function G(s). Then the mapping of the contour
via G(s) shall encircle the origin (n − m) number of times
Type: 1, order: 3 in the counter clockwise direction. If
−270°
1   1.  n < m the net encirclement will be m − n times in
G(s) =
s (1 + st1)(1 + st2 ) clockwise direction.
−180°
  2.  n > m then net encirclement will be n − m times in

the counter clockwise direction.
  3.  n = m then there will be no encirclement of the
origin at all.
−90°
Figure 6.38 illustrates the principle of argument for
various poles and zeros enclosed on the s-plane.
  1.  The contour is s-plane contains two poles and one
6.5.2 Closed-Loop Poles and Zeros of a zero.
Negative Feedback System   2.  Each pole results in one anticlockwise encirclement
of origin in the G(s)H(s) plane.
Consider a negative feedback system with forward path   3.  Each zero results in one clockwise encirclement of
gain G(s) and feedback element gain H(s), such that origin in the G(s)H(s) plane.

Chapter 6(487-554).indd 516 3/23/2016 2:23:26 PM


6.5  FREQUENCY RESPONSE ANALYSIS     517

  4.  Thus, two anticlockwise and one clockwise encir- 6.5.4.1 Procedure to Obtain the
clement, means that the G(s)H(s) mapping contour Nyquist Plot
will encircle the origin once in anticlockwise direc-
tion as shown in Fig. 6.38(a) and (b). Nyquist plot is the image of the Nyquist contour under
the map G(s)H(s). Given any open-loop transfer func-
tion G(s)H(s), the procedure to obtain the Nyquist plot
as shown in Fig. 6.39 is summarised in the following list:
  1.  Select a Nyquist contour on the s-plane, which
covers whole of the right half of s-plane and
bypasses all the singularities on the jw axis. For
s-plane G(s)H(s)-plane
G(s) = 1/s(s + 1).
(a) (b)   2.  Label the Nyquist contour in the clockwise direc-
tion starting with w = 0+. Every small indent is
Figure 6.38 |   Principle of argument G(s)H(s). considered to have three labeled points.
  3.  The movement of the contour from E to F to A
The encirclement of the origin can be checked by draw-
is the small semi-circular indent given by s = eejq
ing a ray going outwards from the origin and counting
with e → 0 where q varies from −90° to 0° to +90°.
the number of clockwise and counterclockwise intercepts
  4.  AB section is given by the equation s = jw where
made by the mapping G(s) onto the ray.
w varies from 0 to ∞.
  5.  BCD section is the infinite semi-circular loop with
6.5.4 Nyquist Contour radius R → ∞ and the relation is given by s = Rejq
with q varies from +90° to 0° to −90°.
The Nyquist stability criterion is used to
  6.  DE is the mirror image of the positive imaginary
  1.  Check the stability of the closed-loop system. axis and is given by the relation s = jw where w
  2.  If not found stable, then how many closed-loop varies from − ∞ to 0.
poles lie in right half of s-plane.   7.  The gain and phase of the open-loop transfer func-
In order to check the existence of closed-loop poles in tion G(s)H(s) is evaluated on the Nyquist contour
the right half of s-plane, a contour covering whole of the and is plotted on a complex plane called G(s)H(s)
right half of s-plane is chosen. All the singularities which plane. The resulting plot is known as Nyquist plot.
occur on jw-axis are bypassed using infinitesimally small
indents. Such a contour is called Nyquist contour. Mapping Small Semi-Circular Indent
Let us consider a system with open-loop transfer func- To map EFA, substitute s = eejq in G(s)H(s) as below
1 and neglect s = eejq in comparison with 1.
tion G(s) =
s(s + 1)
= ∞e−jq
1 1
G(s)H(s) s =eejq = jq jq
=
ee jq
The open-loop system has two poles, in left of the s-plane
and the origin. We must select a Nyquist contour such that ee (ee + 1)
it encloses whole of the right half s-plane and bypasses the As the Nyquist contour moves from E to F to A, the
pole at origin. Such a contour is shown in Fig. 6.39. respective Nyquist mapping follows as per Table 6.11.
Im B
Table 6.11 |   Mapping to EFA indent on G(s)H(s)
s = Rejq
plane


R Point G(s)H (s) ∠G(s)H (s)
A
F C
−1 0 s = eejq Re E s = ee−j90° ∞ 90°
E
F s = eej0° ∞ 0°

A s = ee+90° ∞ −90°
D

Figure 6.39 |   Nyquist contour selection for G(s)H(s).


The mapping of EFA for different type of systems is
shown in Fig. 6.40.

Chapter 6(487-554).indd 517 3/23/2016 2:23:38 PM


518     Chapter 6:  CONTROL SYSTEMS 

E can be selected in between 0 to ∞ and gain and


phase information can be tabulated as in Table 6.13,
∞ ∞
→ → and points can be joined using smooth curve as in
R E R Fig. 6.41.
F F
A The following guidelines are preferred for polar plots
  1.  Generally, phase is not calculated, but we are more
interested in how phase behaves as per frequency
A
approximately.
(a) (b)   2.  The phase angle of the point where magnitude is
Figure 6.40 |   Mapping of EFA indent. (a) Type 1
unity is most important and is calculated, rest of
plot is sketched in rough manner.
system. (b) Type 2 system.

Mapping Infinite Semi-Circular Indent Table 6.13 |   Gain and phase vs. frequency for
To map BCD, substitute s = Re jq in G(s)H(s) and polar plots
neglect 1 in comparison with Re jq in various factors as
∠G(s)H(s)
w G(s)H (s)
= −90° − tan−1(w )
= 0e−j2q
1 1
G(s)H(s) s = Rejq = jq jq
= 2 j 2q = 1/w 1 + w 2
Re (Re + 1) R e

As the Nyquist contour moves from B to C to D in


1. 1/ 2 −135°

−153.4°
Fig. 6.39, the respective Nyquist mapping for the system
2. 1/ 5
−166°
1
G(s) = 3. 1 / 17
s(s + 1)

follows as per Table 6.12.

Table 6.12 |   Mapping of BCD indent on G(s)H(s)


Im

plane

Point G(s)H (s) ∠G(s)H (s) Re

B s = Re j90° 0 −180°

C s = ee j0° 0 0
®
w

D s = Re−j90° 0 +180° G(s)H(s)−plane

Figure 6.41 |   Mapping of jw-axis (polar plot).


Thus, for the sample system taken, the infinite semi-
circular indent on the Nyquist contour gets mapped to a
point at origin and its phase changes from −180° to 0° For the example system taken, since the given system
to +180°. However, in general, is a type 0 system, the initial phase shall be −90° and
  1.  For the systems with number of zeros equal to due to one more pole final phase shall be −180°. Hence,
number of poles, the BCD will be mapped to non- the Nyquist plot will start from ∞, 90° and ends at 0,
zero point. −180°. The Nyquist plot starts from a point of infinite
  2.  For all systems with number of poles more than magnitude in −90° direction and ends at zero magnitude
zeros, the BCD will be mapped to zero point. point with direction −180°.
  3.  Depending upon the system type, the Nyquist plot
shall rotate accordingly at zero point. Mapping the Negative jw Axis DE
The Nyquist plot of the negative imaginary axis DE
Mapping jw Axis AB (Polar Plots) is the mirror image of the Nyquist plot of the posi-
We substitute s = jw in open-loop transfer function tive imaginary axis AB, as shown by dotted line in
G(s)H(s) with w varying from 0 to ∞. Few points Fig. 6.42.

Chapter 6(487-554).indd 518 3/23/2016 2:23:52 PM


6.5  FREQUENCY RESPONSE ANALYSIS     519

Im jω

s-plane
ω= ∞
Re
C1


C2

®
ω = 0−
σ

R

ω = 0+
®
w

G(s)H(s)−plane C3

Figure 6.42 |   Mapping of −jw axis as mirror image of


polar plot. ω = −∞

(a)
Once the Nyquist plot is ready following the procedure
discussed in the above section, the stability of the closed- jw
loop transfer function can be judged through Nyquist
stability criterion.
C1

0

R
6.5.5 Nyquist Stability Criterion C2
s
The Nyquist stability criterion is used to check whether R →∞
a given system is stable in closed-loop configuration, C3
given the location of poles and zeros of open-loop system,
and also if unstable, how many poles lie in right half of
s-plane?
(b)
Let P be the number of open-loop poles of G(s)
H(s) lying in right half of s-plane, Z be the number
of closed-loop poles (or the zeros of the characteris- C3
tic polynomial 1 + G(s)H(s)) lying in the right half
of s-plane and N be the number of counter clockwise C2 ∞
encirclement of the point (−1 + j0) on the Nyquist →
C1 R
plot. C4
C8
Then as per the principle of argument, the Nyquist
plot will encircle the point ( −1 + j0), N = (P − Z)

C7 R
number of times in counter clockwise direction. 0
For a system to be stable, the number of closed-loop C6
poles in right half of s-plane must be zero, that is, Z C5
must be zero for a stable system.
Hence, as per the relation, the given system is stable
if and only if N = (P − 0) = P. (c)

Thus Nyquist stability criterion states that: If the Figure 6.43 |   Nyquist contours: (a) no poles on
G(s)H(s) contour in the G(s)H(s) plane corresponding imaginary axis; (b) pole at origin and
to Nyquist contour in the s-plane, encircles the point (c) poles on imaginary axis.
(−1+j0) in the anticlockwise direction as many times
as the number of right half of s-plane poles of G(s)H(s),
then the closed-loop system is stable. 6.5.5.1 Stability Margins
The Nyquist contours for different cases: (i) no poles Nyquist stability criterion provides an estimate of sta-
on imaginary axis, (ii) pole at origin and (iii) poles on bility of the closed-loop system, as well as number of
imaginary axis are depicted in Fig. 6.43(a), (b) and (c), closed-loop poles in right half of s-plane. In addition, two
respectively. popular stability margins: gain margin and phase margin

Chapter 6(487-554).indd 519 3/23/2016 2:23:58 PM


520     Chapter 6:  CONTROL SYSTEMS 

provide an estimate of relative stability of the system. x(t) = sin(ωt) y(t) = Asin(ωt+φ)
Two important frequencies are required to describe sta- G(jω)
bility margins and these are listed as follows:
  1.  Phase crossover frequency: It is the frequency
at which the phase of the loop gain G(s)H(s)
becomes −180° and is denoted by wpc as shown in Figure 6.45 |   Linear system with sinusoidal input and
Fig. 6.44. output.
  2.  Gain crossover frequency: It is the frequency
at which the loop gain |G(s)H(s)| becomes 1 and is
Therefore, the gain and phase of the linear system,
denoted by wgc.
which is reflected in terms of amplitude and phase shift
of its output, is frequency dependent. The plot of the
gain and phase information with respect to frequency
Im
ain 1 is known as frequency response. System stability
= as well as marginal stability indices can be directly
marigin a
Unit circle derived from frequency response. Further, important
transient response specifications can also be indirectly
wpc Re calculated. Bode plots include the Bode gain plots and
−1 a Bode phase plots, sketched on a common scale on the
f same paper.
wgc
Phase
  1.  Bode magnitude (gain)plot: It represents

marigin
the variation in system gain in decibels (dB) with
ω→

(180−f)
respect to frequency represented over a logarithmic
scale. Bode magnitude plot − 20log10|G(jw)| versus
Figure 6.44 |   Nyquist plot. w over logarithmic frequency scale.
  2.  Bode Phase plot: It represents the variation in
system phase ∠G( jw ) versus w over logarithmic
The stability margins: gain margin and phase margin frequency scale.
can now be defined in terms of crossover frequencies as
follows: In order to sketch these plots, exact data is seldom
required, and one works only with asymptotic approxi-
  1.  Gain margin: It is the amount of additional loop mations for Bode gain plots. The Bode phase plot is gen-
gain required at phase cross over frequency so as to erally plotted by calculating the phase angle information
make the system unstable. Let a be the gain of the at few points and then joining the points with a smooth
system at phase crossover frequency wpc, then the curve.
gain margin is 1/a.
  2.  Phase margin: It is the amount of additional
6.6.1 Advantages of Bode Plots
phase lag required at gain crossover frequency so
as to make the closed-loop system unstable. Let f Some of the advantages of Bode plots are listed as
be the phase of the loop gain at gain crossover fre- follows.
quency, then phase margin will be 180° + f°.
  1.  Due to the logarithmic scale of frequency, low and
Important points to remember: high frequency behaviour of the system can be rep-
  1.  The gain margin for a stable system is always more resented on a single plot.
than 1.   2.  Simultaneous plot of gain as well as phase can
  2.  The phase margin for a stable system is always be used to infer the important properties of gain
positive. margin and phase margin directly from Bode plots.
  3.  Bode plots can be sketched with the help of experi-
mental data and no analytical model is required in
6.6  BODE PLOTS such cases. It means that Bode plots may be used
even if analytical representation of the system is
not available.
The response of linear system to sinusoidal inputs is also   4.  The use of logarithms allows study of effect of
a sinusoid, as shown in Fig. 6.45, whereas the system adding poles and zeros, just by adding the gain
produces a response y(t) = A sin(wt + f ) in response to and phase of the additional element to the existing
input x(t) = sin (wt). Bode plot.

Chapter 6(487-554).indd 520 3/23/2016 2:24:02 PM


6.6  BODE PLOTS     521

  5.  Since Bode plot easily provide estimate of system 6.6.1.2 Poles/Zeros Located at the
behaviour due to addition of poles and zeros, these Origin jw±1
plots can be used to design various compensator
such as lag, lead, lag-lead, proportional integral The effect of poles and zeros placed at the origin is felt
derivative (PID) controller, etc. at all the frequencies. The contribution of a pole at the
  6.  The concept of underdamped response and effect of origin to the log magnitude response of the system can
undamped imaginary poles can be directly observed be calculated as follows:
from Bode plots in the form of resonant frequencies Gain magnitude in dB: 20 log10 1/jw = −20 log10 w
and resonant peaks.
Thus, the log magnitude plot of the factor [1/jw ] versus
The advantages of using log magnitude and log scale log(w) is a line with a constant slop of −20 dB over one
enables us to combine the gain exerted by individual decade of frequency as
poles and zeros to obtain the Bode gain plot of the over-
all system. Let us study the gain and phase plots of Gain in dB
= −20
individual factors in transfer function, that is, constant log(w )
factor K, simple poles and zeros [1 + jw/wn]±1, complex
poles or zeros [jw/wn)2 + j2z(w/wn)+1]±1. The overall The phase angle of the factor [1/jw ] is given by
gain plot for the given system then can be obtained by
w 
= − tan−1   = − tan−1(∞) = −90°
adding the gain plots of each of the involved factors. 1

jw + 0  0
6.6.1.1 Constant Gain Term K
which implies that the phase contribution of the pole at
The constant factors admit constant gain at all the fre- origin is −90° at all the frequencies.
quencies, that is, the gain is constant and is independent The log magnitude and phase, versus logarithmic of fre-
of the frequency. The log magnitude of the constant gain quency plots are shown in Figs. 6.47 (a) and (b) respectively.
K is given by 20log10K.
In addition, the phase angle of the factor K is always
constant. So
20 dB
  1.  ∠K = 0 positive value of K
  2.  ∠K = −180° negative values of K
0
dB

The log magnitude plots with respect to the logarithm of


frequency are shown in Fig. 6.46. 0.01 0.1 1 10 100 1000
w
(a)
∠ in Deg

20 log10 K
0
dB

0
-45
0.01 0.1 1 10 100 1000
w -90
(a)
0.01 0.1 1 10 100 1000
w
∠ in Deg

(b)
0
Figure 6.47 |   (a) Bode gain plot and (b) Bode phase
−45
plot for G(s) = 1/s.
−90

0.01 0.1 1 10 100 1000 The log gain magnitude plot of a zero at the origin
w admits a slope of +20 dB per decade of frequencies.
(b) The contribution of a zero at the origin to the log
Figure 6.46 |   (a) Bode gain plot and (b) Bode phase
magnitude response of the system can be calculated as
plot for G(s) = K. Gain magnitude in dB: 20 log10 |jw| = 20 log10w

Chapter 6(487-554).indd 521 3/23/2016 2:24:11 PM


522     Chapter 6:  CONTROL SYSTEMS 

Thus, the log magnitude plot of the factor jw vs. log(w) order factor representing the pole on real axis, that is,
is a line with a constant slop of 20 dB over one decade wc = 1/t. The log magnitude of the pole on the real axis
of frequency as is given by

æw ö
2
Gain in dB 1
log(w )
= 20 20 log10 = −20 log10 1 + ç ÷
1 + w /w c è wc ø
The phase angle of the factor jw is given by Obviously, the log magnitude depends upon the value of
w  the frequency, that is, whether w is less than the corner
∠jw + 0 = tan−1   = tan−1(∞) = 90°
 0 frequency or greater than the corner frequency. We can
quickly sketch the log magnitude plot for the frequen-
which implies that the phase contribution of the zero at cies which are far below and far higher than the corner
origin is 90° at all the frequencies. The log magnitude frequency.
and phase versus logarithmic of frequency plots are as Such an approximate plot which describes the asymp-
shown in the figure. totic behaviour of the log magnitude in low and high
frequency region is known as asymptotic Bode plots.
6.6.1.3 Multiple Poles or Zeros at The asymptotic approximation may be made as
Origin [±(jw)±n] follows:
  1.  For w << wc, we carry approximation as 1 >> (w/wc )2,
The overall effect of the two poles or zeros at the origin and hence (w/wc )2 can be neglected. Thus, the log
is equal to twice of the effect of single pole or zero. For magnitude is given as
example, if there are two poles at the origin, the log
magnitude plot shall have a slope of −40 dB per decade
æw ö
2
of frequency with a constant phase angle of −180°. For −20 log10 1+ç ÷ = −20 log10 (1) = 0 dB
a factor 1/(jw)2 we can calculate the log magnitude as: è wc ø

2 which is represented by a line of slope 0 dB per


1
Gain magnitude in dB:  20 log10 = −40 log10 w decade of frequencies for the frequencies lower than
w the corner frequency, that is, a line with constant
gain 0 dB from w = 0 to w = wc.
and the phase angle is calculated as
  2.  For w >>  wc, we carry output approximation as
 1  1  1 << (w/wc)2, and hence 1 can be neglected. Thus,
∠ = −90° − 90° = 180°
 jw   jw  the log magnitude is given as

æw ö æw ö
The concept can be generalised to the existence of n 2 2
poles at the origin, where the log magnitude shall be −20 log10 1 + ç ÷ = −20 log10 ç ÷
given by −20nlog10w with slope of −20n per decade of è wc ø è wc ø
frequencies, and the phase angle is given by −90n° con- = −20 log10 w + 20 log10 w c dB
stant at all the frequencies. Similarly, the existence of
two zeros at the origin shall lead to a slope of 40 dB which is represented by a line of slope −20 dB per
per decade of frequencies with a constant phase angle of decade of frequencies for the frequencies higher
180°. For n number of zeros at the origin, the net slope than the corner frequency, that is, a line starting
of the log magnitude curve is given by 20n with respect from w = wc with constant slope of −20 dB per
to log of frequency with constant phase angle of 90n°. decade of frequencies.
  3.  The two asymptotic approximation shall meet
6.6.1.4 Poles or Zeros on the Real Axis a w  = wc with a gain of 0 dB, the gain which
[ ± (1 + jw/wc)+n ] is satisfied by both of the above asymptotic
approximations.
A pole on the real axis is represented by a factor
The phase angle for the pole on the real axis with corner
1 frequency w = wc is given by
1 + j(w )/(w c )
æw ö
Ð = − tan−1 ç
1
÷
where wc is known as the corner frequency. The corner w è wc ø
1+ j
frequency is one over the time constant of the first wc

Chapter 6(487-554).indd 522 3/23/2016 2:24:21 PM


6.6  BODE PLOTS     523

This may be approximated for the frequencies which 6.6.1.5 Second Order Factors: Complex
are far lower and far higher than the corner frequency, Poles or Zeros
shown as follows:
The complex poles or zeros constitute second-order
  1.  For w << wc, the ratio w /w c = 0, and therefore, factors.
the phase angle contribution shall be 0°.
  2.  For w >> wc, the ratio w /w c ® ¥, and therefore é æw ö ù
2 +n

ê1 + j zw
the phase angle contribution shall be −90°. ÷ ú
2
−ç
ê wn è wn ø ú
ë û
To have an asymptotic approximation, we observe that
due to the properties of the tangent of an angle, the The complex poles or zeros occur in complex conjugate
phase angle varies significantly within the range of 0.1 pairs, so the general form of a second order factor is
and 10 radians of the phase angle. Further, the phase given by
angle at the corner frequency w = wc is 45°. Therefore,
±1
the phase plots can be approximated as per the following é zw æw ö ù
2
procedure: ê1 + j 2
−ç ÷ ú
ê wn è wn ø ú
  1.  The effect on the phase angle by a first-order pole ë û
factor starts one decade before the corner fre-
quency (0.1wc) and lasts up to one decade after The log magnitude of a complex conjugate pole pair is
(10wc). given by
  2.  The net change in angle over two decades of fre-
quencies is +90°. Therefore, the phase plots can be
approximated by a line with a constant slope of
−45° per decade of frequency starting one decade 20 log10
1
2zw æ w ö
before the corner frequency and up to one decade 2
1+ j −ç ÷
wn è wn ø
after the corner frequency.
It is observed that the error between asymptotic approx-
imation of the phase and its actual value is always less
é ö ù
than 6 as shown in Fig. 6.48. Although, it is easy to 2
æ æ ö
2 2
have asymptotic plots for the phase angle of the first = 20 log10 ê1 − ç w ÷ ú + ç 2zw ÷
order factors, yet the phase plots are plotted by joining ê
ë è w n ø úû è wn ø
smoothly the actual values of phase angle of chosen five
éé 2ù
æw ö ù
2 2
æ 2zw ö ú
to six points on the log frequency range.
êê ú
= −10 log10 ê ê1 − ç w ÷ ú + ç w ÷ ú
êë ë è nø û è n ø ú
û

This can be approximated via two extreme asymptotic


0 −20 dB decrease behaviours; one for the frequencies far lower than the
dB

corner frequency and the other for the frequencies far


higher than the corner frequency.
0.01 0.1 1 10 100 1000
w
(a)
6.6.2 Measures of System Performance in
∠ in Deg

Frequency Domain
0
Recapitulate that the response of a system to sinusoidal
−45 inputs in steady state is known as frequency response.
−90 For such inputs, at any frequency, the system behaves
like a complex gain having a magnitude and phase angle,
0.01 0.1 1 10 100 1000
w and therefore the magnitude and phase of the output
(b) signal varies with frequency. The variation in gain and
phase of the system may be plotted with respect to fre-
Figure 6.48 |   (a) Bode magnitude and (b) Bode quency to obtain the frequency response plots as shown
phase plots of first-order pole. in Fig. 6.49.

Chapter 6(487-554).indd 523 3/23/2016 2:24:29 PM


524     Chapter 6:  CONTROL SYSTEMS 

Mr 6.6.2.2 Resonant Peak Mr and Resonant


Frequency wr

The notion of a resonant peak has its origin in the concept


Gain in dB

Resonant peak of resonance. In a pure LC circuit, the impedance of the


undamped system circuit becomes zero at resonant frequency w r = LC
rad/s, and thus the circuit offers an infinite gain at the
0 w
wr
frequency. This frequency is also known as the undamped
natural frequency wn of the circuit. The frequency
G(s)H(s) response of a system is studied using a plot of system gain
Mr and phase f versus frequency. Therefore, at resonant
Figure 6.49 |   Infinite resonant peak of undamped frequency, a peak of infinite value shall appear on the
system. frequency response plot as shown in Fig. 6.49.
Inclusion of resistance in this circuit shall limit the
Several measures like bandwidth (w b), resonant peak gain at all the frequencies including resonant frequency.
(Mr), resonant frequency (wr), etc. are used to character- Presence of resistance in a circuit amounts to dissipation
ise the system behaviour under frequency response. Using of energy in the form of heat, and therefore adds damping
these performance measures, it is possible to predict the to the circuit, which can be measured in terms of damp-
transient behaviour of the system. These measures are ing factor z. Depending upon the damping factor of the
defined as follows. system, that is, in case of lightly damped systems, a peak
will be observed in the frequency response at the resonant
6.6.2.1 Bandwidth w b frequency, and is known as resonant peak denoted by Mr.
The frequency at which the resonant peak is observed is
The bandwidth of a system is defined in terms of the known as the resonant frequency denoted by wr.
frequency range over which the system has appreciable
gain. Classically, the bandwidth is defined as the fre-
6.6.2.3 Correlation between Time Response
quency range over which system gain is more than 1 2,
and Frequency Response
that is, 0.707 times its maximum gain. Since the power
of a signal is proportional to the square of its amplitude,
Though, it is very difficult to sketch a parallel between
therefore, the bandwidth may also be defined as the
the location of poles and zeros of a system, and its time
range of frequencies over which the output signal power
response, yet the concept of resonant peak and resonant
 1 
2
1 frequency is clearer in frequency domain. This is due to
remains more than   = of its maximum value.
 2 2 the fact that these concepts are related to the existence
of complex poles. Still for the sake of clarity and with
Equivalently, the bandwidth may also be defined in
the purpose to draw a parallel between the time response
terms of gain in decibels. Reduction of gain by a factor
and frequency response, we shall limit our discussion to
of 0.707 amounts to loss of 3 dB gain. Therefore, for a
the study of a system represented in terms of its domi-
system with 0 dB maximum gain, the bandwidth may
nant pole pair. Let us consider a second order system as
be defined as range of frequencies where system gain
remains above −3 dB value. The bandwidth of a system K
dictates its speed of response. More is the bandwidth; G(s) =
the faster will be its transient response. For a sample s + 2zw n s + w n2
2

system, the bandwidth is marked in Fig. 6.50. In order to have a zero DC gain, we can assume that
K = w n2 , such that
Bandwidth
wb w n2
Gain in dB

G(s) =
G(s)H(s) s2 + 2zw n s + w n2
w
0
−3 dB Under sinusoidal excitation, we can replace s by jw to
obtain the sinusoidal transfer function G(jw) as

w n2
G( jw ) =
1
=
−w + j2zw nw
2
+ w n2 æw ö
2
æw ö
Figure 6.50 |   Bandwidth for control systems −3 dB 1−ç ÷ + j2z ç ÷
point. è wn ø è wn ø

Chapter 6(487-554).indd 524 3/23/2016 2:24:40 PM


6.7  COMPENSATORS     525

Let us define the notion of normalised frequency as 6.7  COMPENSATORS


p = w w n which results in
1
G( jp) =
(1 − p ) + j2 z p 2 The desired performance is seldom achieved using
a unity feedback configuration. Static gain blocks
At any particular frequency, the system can be charac- Gc(s) = K may be inserted in the forward path in order to
terised in terms of a complex gain with magnitude M
and phase angle f given by   1.  Improve the steady-state errors.
1   2.  Reduce the sensitivity of the output to forward
M = path variations.
(1 − p ) + (2z p)2
2
However, when desired control objective cannot be
é 2z p ù
−1 ê ú
achieved using simple gain adjustments, for example,
f = − tan
ê 1 − p2 ú elimination of steady-state errors or to fine tune the
ë û transient response of the system, dynamic blocks with
The maxima of M occurs at the resonant frequency. poles and zeros may be required as
Therefore, we can calculate the resonant frequency by N c (s)
imposing dM/dp = 0. Using simple calculations, one can Gc (s) =
show that Dc (s)

Normalised resonant frequency pr = 1 − 2z 2 These additional blocks required to obtain the desired con-
trol system behaviour are called compensators. The ter-
1 minology stems from the facts that insertion of these blocks
Resonant peak M r =
2z 1 − z 2 may result into adjustment of system gain and/or phase at
particular frequencies. The compensators are classified as
Since, pr is the normalised resonant frequency given by
pr = wr/wn, we can get the resonant frequency and peak as   1.  Lead compensator
  2.  Lag compensator
w r = w n 1 − 2z 2   3.  Lag-lead compensator
1 In addition to compensator, popular and simple control
Mr =
2z 1 − z 2 system designs can be achieved using
  1.  Proportional (P) control
Further, it can be verified from the phase plots that the   2.  Integral (I) control
slope of the phase curves varies inversely with damp-   3.  Derivative (D) control
ing factor and for systems without damping the slope   4.  Combination − PI, PD, PID control
of phase curves approaches infinity. The effect of the
damping factor on the resonant peak and nature/slope The concepts involved in the P, I, D, PID type of control
of the phase curves is as shown in Fig. 6.51. help us understand the theory behind other compensa-
tors and thus are discussed first.
z = 0.3 z = 0.5
z = 0.7
6.7.1 Proportional (P) Control
0 ζ= 1
The proportional control may be achieved by inserting
−20 an adjustable static gain block in cascade with the open-
dB

−40 loop system as shown in Fig. 6.52.

0.01 0.1 1 10 100 1000


R(s) Y (s)
z= 0 z = 0.5 K G(s)
0° +
−45° −
∠ in Deg

z = 0.7
−90°
−135° z= 1
−180° Figure 6.52 |   Proportional control.

0.01 0.1 1 10 100 1000 The purpose of using proportional control is to


w
  1.  Improve the steady-state errors.
Figure 6.51 |   Effect of damping factor on resonant   2.  Reduce the sensitivity of the output to forward
peak and phase plots. path variations.

Chapter 6(487-554).indd 525 3/23/2016 2:24:50 PM


526     Chapter 6:  CONTROL SYSTEMS 

  3.  Improve the speed of response to some extent, but 1 1


may also destabilise the system if pole zero differ- ess = =
1 + Kp 2
ence is more than two.
Let us consider a type 0 system with G(s) = 1/(s + 1), By adding a proportional controller with gain say
then the position error constant is given by K = 50, the steady-state error becomes
Kp = lim G(s) = 1 2 1
s ®0 Kp = =
1 × 2 × 50 50
1 1 1 1
ess =
1 + Kp
=
2 ess = = ≈ .0196
1 + Kp 1 + 50
In order to reduce the steady-state error, proportional
The steady-state error becomes almost 1/50 as compared
control may be employed. Suppose we insert a gain
to that of uncompensated system. The closed-loop trans-
K = 50 in the forward path, then the proportional error
fer function of the system is given by
constant and steady-state error for unit step input are
Kp = lim KG(s) = 50 100/(s + 1)(s + 2) 100
s ®0 Gcl (s) = = 2
1 + 100/(s + 1)(s + 2) s + 2s + 101
1 1
ess = =
1 + Kp 51 The poles of the system are located at −0.5 ± j100, thus
it is clear that the undamped natural frequency of the
system is w n = 101 = 10.05 rad/s. The damping factor
As we increase the gain in the forward path, the steady-
is z = 0.1. The transient response of the system is highly
state error reduces.
The sensitivity of the system to variation in forward under-damped and not acceptable.
path gains is also reduced as
1 1
SGY = ⇒ 6.7.2 Integral (I) Control
1 + G(s) 1 + KG(s)
Integral control is employed to eliminate the steady-
More is the controller gain K, less sensitive the system
state error, and is achieved by placing an integrator in
will be to the fluctuation of forward path gains.
cascade with the given system as shown in Fig. 6.53.
Although the proportional gain reduces the steady-
state error and improves sensitivity, yet it suffers from R(s) Y (s)
two major drawbacks: K G(s)
+ s
  1.  If uncompensated system shows the steady-state −
error, then by using the proportional control the
steady-state error may be reduced, but cannot
completely eliminate it. Figure 6.53 |   Integral control.
  2.  The closed-loop system with proportional control is
50 The transfer function of the integral controller is
G(s) = which means that the dynamics of
s + 51
the closed-loop system becomes very fast, that is, K
Gc (s) =
impulse response decays at a rate of e−51tu(t) and s
hence may not acceptable in majority of cases.
  3.  From the root locus technique, we know that pole The insertion of integral block into the system loop
zero difference is three of more than three, then increases the type of the system by 1. For example,
asymptotes are bound to enter the right half plane G(s) = 1/(s + 1) becomes G(s) = 1/s(s + 1) and steady-
as gain K increases and, hence, will make the state error for step commands become zero as:
system unstable for high values of gain K. For uncompensated system
Let us consider a system with open-loop transfer func- 1 1
2 G(s) = ; Kp = 1; ess =
tion G(s) = s+1 2
(s + 1)(s + 2)
For compensated system
The steady-state error to the unit step input is
1
Kp =
2
=1 G(s) = ; Kp = ¥; ess = 0
1× 2 s(s + 1)

Chapter 6(487-554).indd 526 3/23/2016 2:25:09 PM


6.7  COMPENSATORS     527

Further, the gain K can be adjusted to achieve desired Thus,


sensitivity reduction and transient response shaping. The   1. PI control also adds one zero to the system at
major reason for employing integral control is to elimi- s = −3. Due to the added zero, the centroid in the
nate the steady-state error, but stability of the overall root locus moves to origin and asymptotes will move
system may decrease. along imaginary axis. It means that at high gain, the
response of the system will be completely undamped.
6.7.3 Proportional plus Integral (PI) Control   2.  If we place the zero near the jw axis, then it will
affect the transient response of the system.
The PI control is a parallel combination of a proportional The location of zeros and hence the ratio KI/K poses a
gain block and an integral block as shown in Fig. 6.54. design constraint in design of PI controller.

K + 6.7.4 Proportional plus Derivative (PD) Control


R(s) Y(s)
G(s)
+ The PD control is a parallel combination of a proportional
− KI + gain block and a differentiator block as shown in Fig. 6.55.
s

Figure 6.54 |   Proportional plus integral control.


K
R(s) + Y(s)
G(s)
+
The transfer function of the PI control block is − sKD +

KI sK + KI K(s + KI /K )
Gc (s) = K + = =
s s s Figure 6.55 |   Proportional plus derivative control.
Essentially, PI control:
The transfer function of the PD control block is
  1. Places a pole at the origin, as a result, type of the Gc(s) = K + sKD
system is increased and steady-state error is eliminated.
  2.  Places a zero in the left half of s-plane. The zero The PD compensator results in improper transfer
tries to shift the centroid of the root locus towards function. Using PD control, a zero is added on the open-
the right, thus making system comparatively less loop transfer function, which has the following effects on
stable at high gains. Hence the zero must not be the overall system behaviour
located far away from jw axis   1.  The transient response of the system improves as
  3. Reduces the sensitivity of the overall system with system dynamics become fast. System becomes
the help of proportional gain. Also it shapes the more stable, as pole zero difference is reduced.
transient response of the system, after the steady-   2.  Due to the derivative of the sKD, an undesired
state error requirement is met using integral control. high frequency noise is present in the input to the
system.
In general, suitable choice of KI and K allows one to
  3.  Intentionally, a pole is added along with PD con-
have desired transient response with improved steady-
state accuracy. trol, too far away from jw axis so that its effect on
dominant poles is the minimum.
For the considered system with   4.  The gains K and KD can be suitably adjusted to
2 obtain desired closed-loop behaviour.
G(s) =
(s + 1)(s + 2)
The steady-state error to the unit step input is 0.5. Let 6.7.5 Proportional Integral Derivative (PID)
us incorporate a PI controller with K = 1 and KI = 3 such Control
that Gc(s) = (s + 3)/s is in cascade with the system.
PID control adds the advantages of all the three control-
The open-loop transfer function now becomes lers and generates the control output as a sum of propor-
2(s + 3)
with Kp = = ¥
2 tional, integral and derivative controllers. The transfer
G(s) Gc(s) =
s(s + 1)(s + 2) 0 function of the PID controller is
1 æ 1 ö
ess =
1 + Kp
=0 Gc (s) = Kp ç 1 + + Td s ÷
è TI s ø

Chapter 6(487-554).indd 527 3/23/2016 2:25:18 PM


528     Chapter 6:  CONTROL SYSTEMS 

where Kc is called the controller gain, TI is called inte-   4.  Use the f1 information to find the location of the
gral time and Td is called derivative time constant. compensator pole.
Various tuning methods are used to find the value of   5.  Repeat the procedure if the steady-state conditions
these terms for a given system, such as Ziegler Nichols are not met.
method, Ultimate cycle method, etc.

6.7.6.2 Lead Compensator Design using


6.7.6 Lead Compensator
Bode Plots
Lead compensator is used to
The design specification may be mapped as frequency
  1.  Improve the transient response. response measures as follows:
  2.  Increase the system bandwidth.
  3.  Faster speed of response with reduced overshoot.   1.  Gain margin and phase margins
  2.  Bandwidth, resonant frequency and resonance
The transfer function of the lead compensator is given by peak
s + zc   3.  Steady-state errors and error constants
Gc (s) =
s + pc Given the design specification in terms of steady-state
error and phase margin, the steps to design lead compen-
Assuming a = zc/pc < 1, and zc = 1/t, where t is the sator using Bode plots are as follows:
time constant related to compensator zero, we get
  1.  Design a proportional gain controller to achieve the
s + 1/t desired steady-state error requirement.
Gc (s) =
s + 1/at   2.  Sketch the Bode gain and phase plots of the
system
  3.  Mark the gain crossover frequency wgc.
The pole-zero diagram of the lead compensator is shown
  4.  Check the phase margin say fpm (available). Calculate
in Fig. 6.56.
additional phase lead required
flead = fpm (desired) − fpm (available) +E

  5.  Calculate the value of a as


−1/at −1/t
1 + sin f lead
a=
1 + sin f lead
Figure 6.56 |   Pole zero locations for lead
æ 1−a ö
compensator. f lead = sin−1 ç ÷
è1+a ø
6.7.6.1 Lead Compensator Design using   6.  Calculate the gain provided by the lead compensa-
Root Locus tor at frequency corresponding to maximum phase
lead
The time domain specifications are generally given in
 1 
Gain = −20 log 
 a 
terms of damping ratio and undamped natural fre-
quency. The knowledge of z and wn can be used to
find the location of the dominant pole pair. Thereafter,
the following procedure may be adopted for design of   7.  Refer Bode gain plot and mark the frequency at
compensator. which the gain is −20 log(1/ a ), let that frequency
be wm.
  1.  Select location of the dominant pole pair-based upon
the desired time response specifications. Generally   8.  Calculate the value of the time constant t as
system is kept well damped with z = 0.707. wm =
1
  2.  Calculate the deficiency in phase angle at t a
  9.  Once a and t are known, the compensator transfer
­dominant pole with positive imaginary part, let it
be f1.
ts + 1
  3.  Place the zero of the compensator on the negative function is given by Gc (s) =
real axis just below the dominant pole with posi- ats + 1
tive imaginary part. The procedure is illustrated with the help of Fig. 6.57.

Chapter 6(487-554).indd 528 3/23/2016 2:25:27 PM


6.7  COMPENSATORS     529

6.7.7.1 Lag Compensator Design using


Root Locus
w gc = 10.4 rad/s
dB

The following procedure may be adopted for the design


w g = 14 rad/s of compensator:
−20log(1/√a)
  1.  Assuming the desired transient response shap-
ing has been carried out using proportional gain
adjustments; find out the gain at the position of
dominant pole pair using root locus plots.
f

20°   2.  Calculate the steady-state errors with the value of


gain obtained at the position of dominant poles.
  3.  With the desired steady-state error value given,
0.01 0.1 1 10 100 w determine the factor by which the steady-state
error is to be reduced. Chose the value of b slightly
Figure 6.57 |   Phase lead compensator design using more than this value.
Bode plots.   4.  Select the location of zero near the origin such that
the line joining the dominant pole with the origin
and with that zero should not make angle more
6.7.7 Phase Lag Compensator than 8° to 10°. This position is marked as −1/t.
  5.  With the value of b calculated as per point 3 above,
The phase lag compensator improves the steady-state place the pole at a position −1/bt.
behaviour of the system without affecting the transient
response of the system. The bandwidth of the system Thus, the overall compensator transfer function is
gets reduced with response slower. s + 1/t
obtained as Gc (s) =
s + 1/bt
The transfer function of the phase lag compensator is
s + zc
given by Gc (s) =
s + pc 6.7.7.2 Lag Compensator Design using
Assuming b = zc/pc > 1, and zc = 1/t, where t is Bode Plots
the time constant related to compensator zero, we get
Given the design specification, the steps to design lead
s + 1/t compensator using Bode plots are as follows:
Gc (s) =
s + 1/bt   1.  Sketch the Bode gain and phase plots of the uncom-
pensated system and adjust the system gain K as
The pole-zero diagram of the phase lag compensator per the steady-state error requirements.
is as shown in Fig. 6.58.   2.  Mark the frequency wg at which the phase of the
system matches the given phase margin conditions
f = −180° + PM + margin
A margin of about 5°−10° is sufficient to obtain
tolerable design.
−1/t −1/bt   3.  Note down the gain K at frequency wg, and then
calculate b as

Figure 6.58 |   Pole zero location of phase lag 20log b = K


compensator.
  4.  The gain at this frequency shall have to be reduced
to zero to complete the design, that is, we will have
We observe that the poles and zeros are located very to bring the gain curve down without affecting the
near to the origin, so that they have no effect on the phase plots.
dominant pole pair.   5.  Select the upper corner frequency, that is, frequency
Lag compensators are only used in a situation where related to compensator zero, about a decade below
the desired transient response can be obtained via simple wg, it means
gain adjustments and lag compensator is designed only wg 1
to reduce the steady-state errors. Transient response =
shaping cannot be achieved using lag compensation. 10 t

Chapter 6(487-554).indd 529 3/23/2016 2:25:34 PM


530     Chapter 6:  CONTROL SYSTEMS 

  6.  With these calculated value of t and b, the designed 6.7.8.3 PI Controller Realisation
lag compensator is
PI controller can be realised using a series combination
ts + 1 of resistor and capacitor in the feedback as shown in
Gc (s) =
bts + 1 Fig. 6.61.
In case the overall design specifications are not met, the
design procedure may be repeated with a larger value of
allowable margin. R R2 C
R

6.7.8 Realisation using Op-amps −
+ R1
vi(t) +
Operation amplifier circuits can be used to realise the vo(t)
compensators as discussed in the following subsections.

6.7.8.1 Proportional Controller Realisation Figure 6.61 |   PI controller using op-amps.

Op-amp gain circuits in negative feedback configuration


are placed in tandem as shown in Fig. 6.59. The gain of the circuit is

R R2 vi (t) R Cs + 1
= 2
R vo (t) R1Cs


+ R1
vi(t) + 6.7.8.4 PD Controller Realisation
vo(t)
The PD controller can be realised by shunting the input
resistor of the second stage with a capacitor as shown in
Figure 6.59 |   Op-amp schematics for proportional Fig. 6.62.
gain controller.
The gain of the circuit is
R C R2
vi (t) R
= 2 R
vo (t) R1 −

+ R1
vi(t) +
6.7.8.2 Integral Control Realisation vo(t)

Integral control can be realized using an integrator cir-


cuit as shown in Fig. 6.60.
Figure 6.62 |   Op-amps schematics of PD control.
R C
R
− The gain of the circuit is

+ R1
vi(t) + vi (t) R
vo(t) = 2 (R1Cs + 1)
vo (t) R1

Figure 6.60 |   Integrator using op-amps.


6.7.8.5 Lag/Lead Compensator Realisation
The gain of the circuit is Lag or Lead compensator can be realised by placing a
vi (t) 1 capacitor in parallel with feedback resistor as well as
=
vo (t) R1Cs input resistor as shown in Fig. 6.63.

Chapter 6(487-554).indd 530 3/23/2016 2:25:45 PM


6.8  STATE SPACE REPRESENTATION     531

C2   5.  Many significant properties of the systems can be


R C1 defined in state space representation like observ-
R ability, controllability, stability, etc.
−   6.  The state space representation provides easy design

R2
+ R1 procedures using state feedback which at least the-
vi(t) + oretically guarantees that poles of the closed-loop
vo(t)
system can be placed arbitrarily.
  7.  The state space representation is also applicable
to general nonlinear dynamical systems as well as
Figure 6.63 |   Op-amp schematics for lag/lead time varying systems.
compensator.

6.8.1 Concept of State


The gain of the circuit is
Most important part of state space representation is the
vi (t) R (R C s + 1) concept of a state.
= 2 1 1
vo (t) R1 (R2C2s + 1)   1. State: A state of a dynamical system is defined as
the minimal set of variables that can be indepen-
The value of the resistors and capacitors can suitably be dently specified at t = t0, such that the knowledge of
chosen to make the schematics behave like either a lag which along with the future inputs completely defines
or a lead compensator. the behaviour of the system for t > t0. These minimal
  1.  If R1C1 >> R2C2, then the circuit behaves like a set of variables are called state variables. States rep-
lead compensator. resent the degree of freedom of a dynamical system.
  2.  If R1C1 << R2C2, then the circuit behaves like a The number of states in a system is equal to the
lag compensator. number the initial conditions that can be specified
in a system. For a n-th order system, we can specify
a set of `n’ state variables. General state space rep-
resentation of a system incorporates two equations
6.8  STATE SPACE REPRESENTATION
namely; state equation and output equation.
  2.  State equation: This equation of a dynamical
The transfer function approach is best suited for single- system relates the differential of the state vari-
input single-output systems. Most of the systems occur- able to the state variable and the input signals, as
ring in nature are multi-input and multi-output (MIMO) x(t) = Ax(t) + Bu(t), where A is called the system
in nature. To deal with such systems, an alternative matrix and B is known as the input coupling
time domain approach known as state space representa- matrix. It is a first-order vector matrix linear dif-
tion was developed, which has the following significant ferential equation.
features:   3.  Output equation: It formulates the system
output using a linear combination of states and
  1.  The state space approach is a unified approach for the input signals, as y(t) = Cx(t) + Du(t); where
analysis and design technique to deal with MIMO C is called output coupling matrix and D is known
systems. as direct transmission matrix. It is a vector matrix
  2.  The state space approach represents a system in linear algebraic equation.
terms of a set of first order differential equations
and well established results from linear algebra can The general block diagram for state space representation
be used. of a system is as shown in Fig. 6.64.
  3.  Most of important transfer function representation
properties are lost when systems are represented in
state space form, and mapping of such properties in D
state space representation is not direct. Therefore, +
∫v
today design is thought in the frequency domain
u B . C
and computations are done in time domain using + x x + y
state space representation. −
  4.  The concept of zero input response, that is, response A
due to initial conditions can be studied using state
space representation which is not possible using Figure 6.64 |   State space representation of a general
transfer functions. linear time invariant (LTI) system.

Chapter 6(487-554).indd 531 3/23/2016 2:25:50 PM


532     Chapter 6:  CONTROL SYSTEMS 

The state space representation of a general linear time variables is to define a state variable which is derivative
invariant (LTI) system is given in terms of a pair of of its previous state variable. States variables chosen in
equations as follows: such a way are called phase variables. Let x1, x2, … xn
be the phase variables. The state variables in the form of
x(t) = Ax(t) + Bu(t) phase variables are defined as

y(t) = Cx(t) + Du(t) x1 = y


dy
where x(t) is state vector, u(t) is the input vector and x2 =
dt
y(t) is the output vector. For a nth order LTI system,
with m number of inputs and p number of outputs, A is d2y
x3 = ,
n × n matrix, B is n × m matrix, C is p × n matrix and dt2
D is p × m matrix. The state variable representation in d n −1y
xn =
dtn −1
an expanded form is given as follows:

é x1(t) ù é a11 a12 .… a1n ù é x1(t) ù The clear observation from the above equations is that
ê x (t) ú ê a .… a2n ú ê x2 (t) ú
ê 2 ú = ê 21
a22
úê ú x1 = x2 , x2 = x3 ,… x n−1 = xn . Finally, the derivative of
ê  ú ê   …  úê  ú the nth state xn is obtained by substituting the rela-
êë n úû êë n1 n2
x (t) a a .… ann úû êë xn (t)úû tions for the derivative of other states into the given nth
é b11 .… b1m ù é u1(t) ù order differential equation as
êb .… b2m ú ê u2 (t) ú
+ ê 21 úê ú xn + an −1xn −1 + an −1xn −2 +  + a1x1 + a0 x1 = b0 u
ê  …  úê  ú
êë n1 …
b . bnm ûú ëêum (t)ûú xn = −a0 x1 − a1x2 − a2 x3 −  − an −1xn + b0 u

é y1(t) ù é c11 c12 … a1n ù é x1(t) ù The above equation can be represented in vector matrix
ê y (t) ú ê c c … c2n ú ê x2 (t) ú form as follows:
ê 2 ú = ê 21 22 úê ú
ê  ú ê   …  úê  ú  x1   0 1 0  0   x1  0 
êëy p (t)úû êëc p1 c p2 … c pn úû êëxn (t)úû  x   0  0   x2  0 
 2 
0 1
 
é d11 … d1m ù é u1(t) ù  x3  =  0 0 0  0   x3  + 0  u
êd … d2m ú ê u2 (t) ú     0 0 0  1      
+ ê 21 úê ú   −a −a1 −a2  −an −1  xn  1 
ê  …  úê  ú xn   0
ëêdn1 … dnm úû êëum (t)úû
since y is the output of the system which is represented
The state space representation of a linear time invari- by x1, therefore the output equation is given by
ant system can be converted into an equivalent transfer
 x1 
x 
function representation or nth order differential equation
 2
y = [1 0 0  0 ]  x3 
representation, and vice versa.

  
6.8.2 Transforming Differential Equation x 
 n
Representation into State Space
Representation The given differential equation had no differential terms
for the input signal, therefore, there are no terms con-
Let us convert a given nth order differential equation taining derivative of input variable in phase variable
into state space representation. state space representation. The phase variable represen-
Consider a nth order differential equation given by tation of a general nth order may involve terms con-
taining derivative of the input signal. In order to avoid
dny d n −1y dy such input signal derivative terms, we may have to judi-
n
+ an −1 n −1
+  + a1 + a0 y = b0 u
dt dt dt ciously select the state variable. Apart from this phase
variable canonical form of state representation, various
In order to obtain the state space representation, we other useful state space representations exists, such as
need to define a set of n number of state variables. parallel form, cascade form, observable canonical form,
One mathematically convenient way of choosing state controllable canonical form, etc.

Chapter 6(487-554).indd 532 3/23/2016 2:26:01 PM


6.8  STATE SPACE REPRESENTATION     533

6.8.3 Time Response from State Representation Some of the properties of state matrix are as follows:
  1.  f(t1) transform a given system state from t = 0 to
Given state space representation of a system as t = t1. Also, f(t) is a non-singular matrix for any
x(t) = Ax(t) + Bu(t) time t.
    2.  f(t2 − t1) transforms a given system state at time
y(t) = Cx(t) + Du(t) t1 to system state at time t2.
  3.  Substituting time t = 0, we get f(0) = I and dif-
Taking Laplace transform of both equations, we have
ferentiating we have df/dt = Af.
sX(s) − X(0) = AX(s) + BU(s)
(sI − A)X(s) = BU(s) 6.8.6 Similarity Transformations
−1 −1
X(s) = (sI − A) BU(s) + (sI − A) X(0)
It is well known that state space representation of a
Substituting the value of X(s) in the output equation system is not unique. There can be infinite number of
and rearranging, we get different state space representations of a system corre-
sponding to unique transfer function. Let x be one state
Y(s) = [C(sI − A)−1B + D] U(s) + (sI − A)−1X(0) space, then we may obtain another state space x such
that the new state space is related to the existing state
Using inverse Laplace transform, we obtain the time space via similarity transformations as x = Px, where P
response of the given system as is an n × n non-singular matrix.
t It can be proved that the state space representa-
∫ Ce
A(t −t )  both lead to the
Bu(t )dt + CeAt x(0) tions in state space x as well as x;
 
y(t) =

0  Free response
same transfer function. The use of similarity transfor-
Forced response of system of the system due mation makes it possible to explore different state space
due to external input u(t) to initial conditions
representation of a system from point of view of their
applicability.
6.8.4 Obtaining Transfer Function
Representation from State Space
6.8.7 Canonical Forms
Representation
Depending upon the choice of variables, a given transfer
Here we take the Laplace transform of the state as well
function may be realized in terms of many realisations
as output equation, considering zero initial conditions
such as
  X(s) = (sI − A)−1 BU(s)   1.  Jordan form
Y(s) = CX(s) + DU(s)   2.  Observable canonical form
  3.  Controllable canonical form
Substituting the value of X(s) in Y(s), we get
We shall limit out the discussion to single-input single-
Y(s) = [C(sI − A)−1 B + D] U(s), output (SISO) systems only, the extension to the multi-
input multi-output (MIMO) is fairly natural.
thus, the transfer function representation of the given
state space representation is given by   1.  Jordan form: Let an LTI SISO system is repre-
sented in terms of general transfer function
Y (s) 
= C(sI − A)−1 B + D 
U (s) 
G(s) =
b0 sn + b1sn −1 +  + bn
G(s) =
sn + a1sn −1 +  + an
6.8.5 State Transition Matrix
The partial fraction expansion of the above equation
The Laplace inverse transform of matrix (sI − A)−1 is leads to:
known as state matrix, represented by f(t). The state
Case 1: When the poles of the given transfer function
transition matrix f(t) = eAt transforms system state at
are real and distinct.
time t = 0 to time t, that is,

x (t ) = f (t ) x (0 ) G(s) = b0 +
r1 r
+ 2 + +
rn
x (t ) = eAt x(0) s −l 1 s − l 2 s − ln

Chapter 6(487-554).indd 533 3/23/2016 2:26:12 PM


534     Chapter 6:  CONTROL SYSTEMS 

then the state space representation can be x = Px


obtained as
0.5 −j0.5 … 0
 l1 0 … 0 1 0.5 j0.5 … 0
0 l … 0 1 P = 
x =  2
x+  u    … 
  …      0 0 …. 1 
 0 0 … ln  1
and the resulting state variable representation is
y = r1 r2 … rn  x + b0 u
given by
Case 2: When the poles of the given transfer  s 1 w1 … 0 2
function are real and repeated, for example, l1 is −w s … 0 0
x =  1 1
 x +   u
repeated two times, then the partial fraction expan-    …    
sion are as follows:  0 0 … ln  1 
r11 r rn y =  g h … ru  x + b0 u
G(s) = b0 + + 12 +  +
s − l1 s − l1 s − ln   2.  Observable canonical form: Given a system with
State space representation is given as b0 sn + b1sn −1 +  + bn
transfer function G(s) =
sn + a1sn −1 +  + an
 l1 1 … 0 0
0 l … 0 1 The observable canonical form is given by
x =  1
x+  u
  …    0 … 0 −an  bn − an b0 
 0 0 … ln   1  1 … 0 −an −1    
x =  x+  u
y = r11 r12 … rn  x + b0 u  …     b2 − a2b0 
0 … 1 −a1   b1 − a1b0 
y [0 … 0 1] x + b0 u
The main consideration here is to note that when
l1 gets repeated, then the system matrix has l1
as first two diagonal entries, and term just above   3.  Controllable canonical form: Given a system
the second l1 is 1 (see underlined 1 in A matrix).
n−1
Further, the row in B matrix corresponding to the n
with transfer function G(s) = b0s + b1s +  + bn
first l1 gets 0 entry (see underlined 0 entry in B n n−1
s + a1s +  + an
matrix).
The controllable canonical form is given by
Case 3: When the given system has a complex con-
jugate pole pair, then the residues corresponding to  0 1 … 0  0 
 0 0 … 0  0 
x = 
…  
x+  u
the complex poles in the partial fraction expansion
of the transfer function shall also occur in conjugate     
pairs as follows: −an −an −1 … −a1  1 
g + jh g − jh y = bn − an b0 bn −1 − an −1b0 … b1 − a1b0  x + b0 u
G(s) = b0 + +
s − (s 1 + jw1 ) s − (s 1 − jw1 )
rn 6.8.7.1 Controllability
+ +
s − ln Given an LTI system with the state space description as
then the Jordan form is given as x = Ax + Bu
 y = Cx + Du
s 1 + jw1 0 … 0 1
 0 s 1 − jw1 … 0  1
x =  x+  u
State space representation of a system is said to be con-
        trollable if there exists a control input u(t) [ 0, r ] which
 … ln  1
1
0 0
transfers the system from any given arbitrary state x(0)
y =  g + jh g − jh … rn  x + b0 u to any arbitrary state at time t1; x(t1), else it is uncon-
trollable. In order to check we formulate a controllabil-
The complex entries in the system matrix A and ity matrix Uc = [B AB A2B … An−1 B], and the system
output matrix C can be converted into real one is controllable if rank of the controllability matrix is n.
using the following similarity transformations: Few important points are listed as follows:

Chapter 6(487-554).indd 534 3/23/2016 2:26:27 PM


IMPORTANT FORMULAS     535

  1.  System is controllable if the input to the system x = Ax + Bu


affects each and every state of the system.
  2.  Controllability is the property of the system and it  y = Cx + Du
cannot be changed via feedback.
then, the system is said to be observable if and only
  3.  For a system to become stable, the uncontrollable
if the knowledge of the output y(t) at time t = 0, and
the input u(t) over a finite horizon t ∈[0, t1 ] is suffi-
state has to be asymptotically stable.
  4.  Loss of controllability may infer cancellation of
cient to determine the system states x(0), else system is
poles and zeros in corresponding transfer function.
unobservable.
  5.  Stability, controllability and observability are
property of the system and in general have In order to check we formulate a observability matrix
2 n −1
nothing to do with each other. Loss of control- U o = [C T A T C T A T C T  A T C T ]T , and the system
lability does not immediately infer instability or is observable if rank of the observability matrix is n. Few
unobservability. important points are listed as follows:
  6.  Uncontrollable system can be made controllable by
reconfiguring the system configuration only, say by   1.  System is observable if the output of the system is
means of putting more sensors. constructed using each and every state.
  2.  Observability is the property of the system and it
cannot be changed via feedback.
6.8.7.2 Observability
Loss of observability may infer cancellation of poles and
Given an LTI system the state space description as zeros in corresponding transfer function.

IMPORTANT FORMULAS

I Fundamentals (d) Translational mechanical system,


X(s) 1
1. The Laplace transform of the system’s output to the = 2
Laplace transform of system’s input provided initial F (s) s M + sf + k
conditions are zero, is called transfer function. (e) Rotational mechanical system
G(s) =
Y (s) q (s) 1
=
R(s) All initial conditions are zero t (s) s2J + sf + k
2. The input-output relations of fundamental elements (f) Armature-controlled DC motor,
for electric circuit:
•  Resistor v(t) = Ri(t) and V(s) = RI(s) w m (s) Km
=
•  Inductor v(t) = Ldi/dt and V(s) = sLI(s) Ea (s) t m s + 1

Capacitor v(t) = 1/C òidt and V(s) = 1/[C(s)I(s)]


•  q (s) Km
=
3. Transfer functions of: Ea (s) s(t m s + 1)

(a) Series RLC circuit where Km is the equivalent motor gain constant
and tm is motor-equivalent time constant.
I (s) Cs
T (s) = = 2
(g) Field-controlled DC motor
V (s) LCs + RCs + 1
(b) Parallel RLC circuit w m (s) K¢
=
Ea (s) (sLf + R f )(sJ + f )
I (s) 1 1 LCs2 + sL + R
= + + Cs = q (s) K¢
V (s) R sL sRL =
Ea (s) s(sL f + R f )(sJ + f )
(c) General op-amp with negative feedback
where Lf and Rf are the inductance and resistance
Eo (s) Z (s)
=− 2 of the field windings, and J and f are the inertia
Ei (s) Z1(s) and damping of the mechanical system.

Chapter 6(487-554).indd 535 3/23/2016 2:26:39 PM


536     Chapter 6:  CONTROL SYSTEMS 

(h) DC techno-generator 1
(b) Steady-state error for step input ess =
E(s) 1 + Kp
= Kt
w (s)
(c) Velocity error constant Kv = lim sG(s)
E(s) s ®0
= sKt
q (s) 1
(d) Steady-state error for ramp input ess =
where Kt is the sensitivity of the techno-generator Kv
in volts per rad/s.
(e) Acceleration error constant Ka = lim s2G(s)
4. Sensitivity of the overall system is defined as an s ®0
incremental variation in the system output to the
incremental variation in the plant G(s), as (f)   Steady-state error constant for parabolic input
1
∂ Y /Y
ess =
SGY = Ka
∂G/G 13. Steady state errors for different types of systems
(a) For open-loop systems given in Table 6.6.

∂Y /Y 14. Steady state response of unity feedback systems,


SGY = =1
∂G/G
output is given by
Y (s) G(s)
(b) For closed-loop system Gcl = =
R(s) 1 + G(s)H(s)
∂Y /Y 1
SGY = = Y(∞) = lim s.Gcl
s ®0
∂G/G 1 + G(s)H(s)

15. Highest power of `s’ in the denominator is the order
5. Sensitivity of the output of closed-loop system with of the system.
respect to the sensor noise
16. Transfer function of first-order (RC) system:
Y ∂ Y /Y −G(s)H(s) vo (s)
t = RC
SN = = 1
∂ N /N 1 + G(s)H(s) =
vi (s) 1 + sRC
6. Sensitivity of the closed-loop system output with
respect to the fluctuations in the feedback element 17. Response of first-order system to unit step input
gain H(s) 1/T
C(s) =
Y ∂ Y /Y −G(s)H(s) s[s + (1/T )]
SH = =
∂H /H 1 + G(s)H(s) C(t) = 1 − e−t/T
7. Load disturbance rejection 18. Response of second-order system to unit ramp input
Y (s) 1
; C(t) = t + T (e−t / T − 1)
= (1/t)
C(s) = 2
N (s) 1 + G(s)H(s) s [s + (1/T )]
8. Reduction rules for block diagram in Table 6.4 19. Transfer function of second order system;
9. Mason’s gain formula C(s) w n2
G(s) = = 2
1 m R(s) s + 2zw n s + w n2
G = å Pk Dk
D k =1 20. Response of second-order system to unit step input:
II Time Response Analysis (a) Undamped, z = 0; C(t) = 1 − cos w n t

10. Total time response = Transient response + (b) Overdamped, z > 1


Steady-state response = Ct (R) + Css
wwnn ééee−−s1st1t ee−−s2st2 tùù
11. Test signals and their Laplace transform given in CC((t)t)==11−− êê −− úú
Table 6.5 22 zz22−−11êëêë s1s1 ss22 úûúû

where s1 = zw n − w n z − 1
12. Steady-state error

lim e(t) = ess lim S


1
R(s) s2 = zw n + w n z − 1
t ®¥ t ®¥ 1 + G(s)
(c) Critically damped, z = 1 ; C(t) = 1 − e−w n t éë1 + w nt ùû
  (a) Position error constant Kp = lim G(s)
s ®0 (d) Under damped, 0 < z < 1

Chapter 6(487-554).indd 536 3/23/2016 2:27:13 PM


IMPORTANT FORMULAS     537

e−zw n t 30. Irrational function e−Ts can be approximated in


y (t) = 1 − [sin(w d t + f )] terms of ratio of two polynomials in s using Pade’s
1 −z 2 approximation
é 1 −z 2 ù −
Ts
−1 ê ú;
f = tan wd = wn 1 −z 2 2 1 − sT /2 s − 2/T
e−Ts =
e
ê z ú = =−
ë û +
Ts 1 + sT /2 s + 2/T
e 2
æ 1 −z 2 ö
−1 ç ÷
p − tan 31. Angle of departure/approach
ç z ÷ fdep = net contribution due to other poles and zeros
21. Rise time, Tr = è ø
wn 1 −z 2 farr = (-1) × net contribution due to other poles
and zeros
p
22. Peak time, Tp = V Frequency Response Analysis
wn 1 −z 2
23. Peak overshoot 32. Polar plots and type and order of polar plots given
in Tables 6.9 and 6.10.
%M p = e−zp / 1−z 2
× 100 33. Nyquist stability criterion states that if the G(s)
24. Settling time Ts = 4t (2% error), ts = 3t (5% error) H(s) contour in the G(s)H(s) plane corresponding
to Nyquist contour in the s-plane encircles the point
25. Type of a system is the number of poles at the (−1+j0) in the anticlockwise direction as many times
origin. as the number of the number of right half of s-plane
poles of G(s)H(s), then the closed-loop system is stable.
III Stability of Control Systems
VI Bode Plots
26. Routh—Hurwitz (RH) stability criterion states that 34. Basic factors of G(jw) are
the necessary and sufficient condition for stability is
that all the elements in the first column of Routh (a) Constant gain, K
array should be positive. If this condition is not sat- K K
(b) Integral factor, or
isfied, then the system is unstable and the number of jw ( jw )n
sign changes is the elements of first column of Routh
(c) Derivative factor, K(jw) or K(jw)n
array corresponds to the number of roots of charac-
(d) First-order factor in the denominator,
teristic equation in the right half of the s-plane. 1
1
jw m
or
jw
IV Root Locus Technique 1+
w
11 + wc 2
c
27. Under unity feedback configuration, the closed- (e) First-order factor in the numerator
loop transfer function of the system is (1 + jwT) or (1 + jwT)m.
(f) Quadratic factor in the denominator
KG(s) KN (s) 1
T (s) = =
1 + KG(s) D(s) + KN (s)
æ jw ö æ jw ö
2
1 + 2z ç ÷+ç ÷
è wn ø è wn ø
(a) The roots of the characteristic equation
p(s) = D(s) + KN(s) = 0 are the poles for the
closed-loop system (g) Quadratic factor in the numerator:
(b) The location of the roots as gain K varies is  1 + 2z (jw/wn) + (jw/wn)2
-1 1 - 4K
±
s1, s2 = 35. Frequency domain specifications
2 2 1
(a) Resonant peak, M r =
28. The centroid, that is, the starting point of the asymp- 2z 1 − z 2
totes sA is given by
(b) Resonant frequency, w r = w n 1 − 2z 2
∑ Real part of poles − ∑ Real part of zeros
sA = (c) Bandwidth w b = w n (1 − 2z 2 + 2 − 4z 2 + 4 z 4 )1/2
n−m
(d) Phase margin
æ ö
29. Angle of asymptotes is given by
2z
180(2k + 1) g = 180° + tan−1 ç ÷
qA = for k = 0, 1, …(n − m) ç (−2z 2 + 4z 4 + 1)1/2 ÷
n−m è ø

Chapter 6(487-554).indd 537 3/23/2016 2:27:34 PM


538     Chapter 6:  CONTROL SYSTEMS 

VII Compensators 39. Obtaining transfer function from state mode:


36. Types of compensators Given: x = Ax + Bu and y = Cx + Du

= [C(sI − A)−1 B + D ]
(a) Proportional (P) controller, G(s) = Kp Y (s)
G(s) =
U (s)
KI
(b) Integral (I) controller, G(s) = 40. Solutions of homogeneous state equation:
s
(a) x(s) = [sI − A]−1x(0)
(c) Derivative (D) controller, G(s) = K + sKD x(s) = f(s)x(0)
f(s) → Resolvant matrix
  (b) x(t) = f(t) x(0)
(d) PID controller, G(s) = Kp 1 + + Td s
1
 TI s  f(t) → state transition matrix.
41. Solution for non-homogeneous equation
VIII State Space Representation
t
37. State model for armature controlled DC motor x(t) = eAt x(0) + ò eA(t−x)Bu(x).dx
0
é x1(t)ù é -Ra /La −Kb/La 0 ù é x1(t)ù é1/La ù
êx (t)ú = ê K /s −B/J 0 ú êx2 (t)ú + ê 0 ú u 42. Condition for controllability
ê 2 ú ê úê ú ê ú
êëx3 (t)úû ë 0 1 0 û êëx3 (t)ûú ë 0 û
[Uc ] = [B AB A2 B … An −1
38. Standard form: x = ax + bu
B]

 x1(t) must have rank n.


y = [ 0 0 1] x2 (t)
 
43. Condition for observability
x3 (t) 2 n −1
Standard form: y = Cx + Du [U o ] = [C T A T C T A T C T ... A T C T ]T

SOLVED EXAMPLES

Fundamentals

1. Given a translational mechanical system as shown G1G2G3


in the figure, perturbed by a unit impulse, the (a)
1 + G1G2 + G2G3 + G3G1 + G3
overall system oscillates with frequency

G1G2G3
F = d(t)
M=2 (b)
k=2 1 + G1G2 + G2G3 + G1G2G3 + G3
(a) 2 rad/s (b) 4 rad/s
(c) 1 rad/s (d) Does not oscillate at all G1G2G3
(c)
1 − G1G2 − G2G3 − G3G1 + G3
Solution:  The natural frequency of the system is
k/M = 2/2 = 1 rad/ s G1G2G3
(d)
Ans. (c) 1 − G1G2 − G2G3 + G1G2G3 + G3
2. Consider the following block diagram. The overall
transfer function Y(s)/R(s) is Solution:  Negative feedback around G3 gives
R(s) Y(s) G3
G1 G2 G3 . Then we can take the takeoff point after
+ + + 1 + G3
− − − G2 to the output and manipulate the feedback
block accordingly as shown in the figure.

Chapter 6(487-554).indd 538 3/23/2016 2:27:57 PM


SOLVED EXAMPLES     539

R(s) Y(s) s2 2s
G1 G2 G3/(1 + G3) (a) (b)
+ + 2
s + 2s + 3 s2 + 2s + 3
− −

(1 + G3)/G3 2s2 2s2


(c) (d)
s2 − 3s + 3 s2 + 2s + 3
Now the inner loop gain is
Solution:  There is only one forward path from
G1G2 the input node to the output node with gain
1 + G3 + G2G3 P1 = 2. There are three loops with gain −3s2, −s−1
Therefore and −2s−1, hence value of Δ = 1 + 3s−2 + 3s−1.
Also, since all the loops touch the forward path,
Y (s) G1G2G3 /1 + G3 + G2G3
R(s) 1 + (1 + G3 ) G1G2G3  / G3 (1 + G3 + G2G3 )
= thus Δ1 = 1. Thus, the transfer using Mason’s
gain formula is
G1G2G3
= C 2 2s2
1 + G1G2 + G2G3 + G1G2G3 + G3 = =
R 1 + 3s2 + 3s−1 s2 + 3s + 3
Ans. (b)
3. The feedback control system with negative feed- Ans. (d)
back is 5. The impulse response of an initially relaxed LTI
system is e−2tu(t). To produce a response of
t.e−2tu(t), the input must be
(a) equally sensitive to both forward and feedback
path parameter changes.
(b) insensitive to both forward and feedback path (a) u(t) (b) e−2tu(t)
parameter changes. (c) tu(t) (d) e−1u(t)
(c) less sensitive to the feedback path parameter
changes and more sensitive to forward path Solution:  Since transfer function of a system is
parameter changes. the Laplace transform of its impulse response,
(d) less sensitive to the forward path parameter therefore,
changes and more sensitive to feedback path
parameter changes. 1
G(s) =
s+2
Solution:  The sensitivity to the forward path and
The Laplace transform of
feedback path parameter changes is given as
1
y(t) = t ⋅ e−2t u(t)
M
SG = 1
Y (s) =
1 + G(s)H(s) (s + 2)2
M G(s)
SH =
1 + G(s)H (s) Since Y(s)/R(s) = G(s), therefore, R(s) = Y(s)
G(s). Substituting the values of Y(s) and G(s), we
Since the gain of the forward path element is always get Y(s) = 1/s + 2. Taking inverse Laplace trans-
more than that of the feedback path gain (con- form y(t) = e−2tu(t).
trol systems are stabilised by reducing gain and Ans. (b)
increasing bandwidth using negative feedback),
SGM
< SHM
. Hence, (d) is the correct option. 6. To reject the load disturbance of 1 rad/s frequency,
Ans. (d) a feed forward controller Gff (s) is employed. Then
gain of Gff  (s) at 1 rad/s frequency is
4. The transfer function C/R of the system repre-
sented in terms of the signal flow graph shown in d(s)
the figure, is Gf f ( s )

1 s−1 1 s−1 + +
R(s) Y(s)
−1 1
R −2 + s(s+ 1) +
2 −
C −3

Chapter 6(487-554).indd 539 3/23/2016 2:28:13 PM


540     Chapter 6:  CONTROL SYSTEMS 

p p the sensitivity of the output gain due to variation in


(a) 1/ 2 , ∠ − (b) 1/ 2 , ∠ + forward path and feedback element gain variations is
4 4
p p M 1 1
(c) 2, ∠ + (d) 2, ∠ − SG = = = 1/2
4 4 1 + G(s)H(s) 1 + 10/10
M G(s) 10
Solution:  Gain from load disturbance d(s) to y(s) is SH = = =5
1 + G(s)H(s) 1 + 10/10
1 s(s + 1) Percentage variation in overall gain due to varia-
=
1+
1 2
s + s+1 tion in G is ∆G ⋅ SG
M
= 10 × 1/2 = 5%
s(s + 1) Percentage variation in overall gain due to varia-
Gff (s) tion in H is ∆H ⋅ SH
M
= 1 × 5 = 5%
Gain via feed forward controller is Ans. (c)
2
s + s+1
9. The transfer function of a zero-order hold with
For complete rejection of d(s),
sample period T is
Gff (s) = −s(s + 1) w =1 rad/ s
(a) s(1 + e−sT ) (b) (1/s)(1 + e−sT )
Gff ( jw ) w =1 rad /s = 1 1 + 1 = 2 (c) s(1 − e−sT ) (d) (1/s)(1 − e−sT )

ÐGff ( jw ) =1 rad /s = −180 − 90 − tan−1 1 Solution:  The time evolution of zero-order hold
is given by g(t) = u(t) − u(t − T ). Taking Laplace
p
= 45° = − transform, we get G(s) = 1/s − 1/s (e−sT )
  4
Ans. (d)
Ans. (d)
7. The transfer function of a system is given by 10. The unit impulse response of a unity feedback
G(s) = (1 − s) / (1 + s), then which of the following closed-loop system is y(t) = e−2t u(t), then the
about it is true? open-loop transfer function G(s) can be

(a) Non-minimum phase system 1 1


(a) (b)
(b) Stable system with undershoot s + 1 s+2
(c) All pass system s 1
(d) All of the above (c) (d)
s + 2 s+3
Solution:  A zero in right half of s-plane makes it Solution:  The closed-loop transfer function is the
non-minimum phase system. Since pole location is Laplace transform of its given impulse response, thus
in left half of s-plane, the system is stable. Non- 1
minimum phase system always experience under- Gcl (s) =
s+2
shoots in their time response. Further, there is a
zero in right half of s-plane symmetrical to the left Also in terms of open-loop transfer function G(s),
half-plane pole (there is zero at s = 1 and there is we have
a pole at s = −1), thus, it is all pass network also. G(s)
Gcl (s) =
Hence all the option are true, thus option (d) is the 1 + G(s)
right answer.
G(s)
Þ 1 + G(s) = sG(s) + 2G(s)
1
Ans. (d) =
s + 2 1 + G(s)
8. A negative feedback system with G(s) = 10 and
H(s) = 1/10 have at most 10% and 1% variation Thus,
1
approximately. Then, the percentage variation in 1 = (s + 1)G(s) ⇒ G(s) =
the output due to each factor will be s+1
(a) 1%, 10% (b) 2%, 5% Ans. (a)
(c) 5%, 5% (d) 10%, 1%
Time Response Analysis
Solution:  The overall gain of the system is given as
11. The unit step response of a unity feedback system
10
M = =5 with G(s) = K/(s + a) is as shown in the following
1 + 10 × 1 / 10 figure.

Chapter 6(487-554).indd 540 3/23/2016 2:28:44 PM


SOLVED EXAMPLES     541

Y(t) 13. The settling time of an underdamped second order


system for 5% tolerance band is 10 s and the peak
0.75 time is 1 s. Then number of cycles of damped response,
before the system settles are
(a) 5    (b) 10    (c) 20    (d) 40
t
Solution:  Peak time is 1 second, therefore
The values of K and a can be
(a) (3, 1) (b) (6, 2) p
Tp = =1
(c) (9, 3) (d) All of the above wn 1 −z 2

Solution:  The value of Kp = K/a and the steady- p = wn 1 −z 2


state error is
Squaring both sides,
1 1 a
p 2 = w n2 − w n2z 2 
ess = = = (i)
1 + Kp 1 + K /a a+K
Also, the settling time for 5% tolerance band is
Here the time response settles to 0.75, therefore the
10 s, therefore
steady-state error is (1 − 0.75) = 0.25.
Since, there is one value to be achieved, that is, 3
ts = = 10
0.25 and there are two parameters K and a, various zw n
combinations are possible. zw n = 0.3
Option (a) gives ess = 1/(1 + 3) = 0.25 and
hence is a correct option. Similarly other options
z 2w n2 = 0.09  (ii)
(b) and (c) are also correct. Hence (d) is the answer.
Ans. (d) Substituting the value of z 2w n2 from Eq. (ii) in
12. The second-order response of a second-order system Eq. (i), we have
is as shown in the following figure.
p = w n2 − 0.09
Y(t) w n = 3.15 rad/s
1.25
z =
0. 3
1 = 0.095
3.15

wd = wn 1 − z 2
0 t
= 3.15 1 − (0.95)2
The value of damping ratio is = 3.13 rad/s
(a) 0.35    (b) 0.48    (c) 0.29    (d) 0.707
wd in hertz is given as 0.5 Hz. Therefore, the
Solution:  The peak overshoot is 0.25, and this is number of damped frequency cycles in 10 seconds
given by will be 10/0.5 = 20.
−pz Ans. (c)
Mp = e 1−z 2
= 0.25 14. The step response G(s) = 10e−s/(s + 1) is

Taking log of both sides, (a) Y(t) (b) Y(t)


−pz 1 10
= ln(0.25) = −1.386
1 −z 2
0 t 0 1 t
Squaring both sides and calculating, we get

9.87z2 = 1 − 1.92z2
(c) Y(t) (d) Y(t)

    z2 = 1/3.43
10 9.1
    z = 0.29
Ans. (c) 0 t 0 1 t

Chapter 6(487-554).indd 541 3/23/2016 2:29:05 PM


542     Chapter 6:  CONTROL SYSTEMS 

Solution:  For the given system Solution:  The characteristic equation of given
system is
Kp = lim G(s) = 10,
s ®0 s2 + 6s + 5 = (s + 5)(s + 1) = 0,
1
ess = = 0.9 The poles are located at s = −5, −1, hence the
(1 + Kp ) system is stable. So response due to point −12e−t
will itself decay to zero as t → ∞.
Therefore, step response reaches up to (10 − 0.9)
The Laplace transformed output of the overall
= 9.1
system due to step of 12 units is
Further, there is a delay of 1 second due to delay
factor, hence the step response shall start after 12
1 second of application of input. Y (s) =
s(s2 + 6s + 5)
Ans. (d)
15. The step response of a system is y(t) = 1 -1.15e-t
y(t) t →∞ = lim sY (s)
s→ 0
sin(1.73t + f), then the impulse response is 12s 12
given by = lim =
s→ 0 s(s2 + 6s + 5) 5
(a)  1.15e−t sin(1.73t + f ) − e−t cos(1.73t + f )
(b) 1.15e−t sin(1.73t + f ) + e−t cos(1.73t + f )
Thus, the final value of output will be 12/5 = 2.4
Ans. (c)
(c) 1.15e−t sin(1.73t + f ) + 1.99e−t cos(1.73t + f ) 18. For the system shown in the figure, the steady
(d) 1.15e−t sin(1.73t + f ) − 1.99e−t cos(1.73t + f ) state error to the unit step input is

Solution:  The impulse response is obtained by dif- R(s) 1 Y(s)


ferentiating the step response as + s(s+10)

dy 10
g(t) =
dt
= 1.15e−t sin(1.73t + f )
(a) 0     (b) 10     (c) 0.1     (d) 0.9

−1.15 × 1.73e−t cos(1.73t + f ) Solution:  The closed loop transfer function is


−t −t
= 1.15e sin(1.73t + f ) − 1.99e cos(1.73t + f ) 1
G(s) = 2
Ans. (d) s + 10s + 10
16. A second-order system has 5% peak overshoot for The error transfer function can be calculated from
a unit step input, if the amplitude of the input is the overall transfer function as
increased five times, then the peak overshoot and E(s)
percent peak overshoot will be = 1 − G(s)
R(s)
(a) 0.5 and 10% (b) 0.25 and 5%
E(s) 1
(c) 0.05 and 10% (d) 0.25 and 25% = 1− 2
R(s) s + 10s + 10
Solution:  The percent peak overshoot is always s2 + 10s + 9
the same, irrespective of the amplitude of the =
s2 + 10s + 10
step input. However, since the amplitude input
is increased five folds, then output shall also be ess (¥) = lim sE(s)R(s)
increased five times (linearity). Thus, s® 0
For u(t): Peak overshoot is 5% of 1, that is, 0.05 2
s + 10s + 9 1
For 5u(t): Peak overshoot is 5% of 5, that is, 0.25 = s. .
s2 + 10s + 10 s
Ans. (b)
ess (¥) = 0.9
17. A control system is defined by the following relation
Ans. (d)
−t
 + 6x + 5x = 12(1 − e )
x 19. For a second-order feedback system as shown in
The response of the system x(t) as t → ∞ is
the following figure, the damping ratio is 0.707 and
undamped natural frequency is 2 rad/s, then the
(a) 6     (b) 2     (c) 2.4     (d) −2 value for K, a, b is

Chapter 6(487-554).indd 542 3/23/2016 2:29:24 PM


SOLVED EXAMPLES     543

R(s) K Y(s) Stability of Control Systems


+ s2+as+1
− 21. For what range of K is the system shown in the
−β following figure, asymptotically stable

s−5
(a) 1, 0.707, 3 (b) 3, 0.707, 1 s+4
+
(c) 3, 2.818, 1 (d) 1, 2.818, 3 −
K
Solution:  The closed-loop transfer function is
given as (a) K < 4 (b) K > 5/4
K (c) K > 0 (d) None of these
Gcl (s) =
s + as + (1 + Kb )
2
Solution:  The closed-loop system characteristic
Comparing characteristic equation equation is
s2 + as + (1 + Kb) s(1 + K) + 4K − 5 = 0
with s 2
+ 2zw n s + w n2 2
= s + 2.818s + 4, we have Then it is clear that the system will be stable for
a = 2.818 and 1 + Kb = 4 K > 5/4.
Ans. (b)
⇒ Kb = 3, if we take K = 1, then b = 3.
22. The condition for stability of s4 + 3s3 + 5s2 + 6s
Ans. (d) + K + 10 = 0 is
20. The open-loop transfer function of a unity feedback (a) K > 5 (b) −10 < K
system is given as (c) K > −4 (d) −10 < K < −4
K
G(s) = Solution:  Formulating the RH array
s(s + 1)
s4 1 5 K+10
the factor by which gain K should be reduced, so
3
that the peak overshoot is reduced from 40% to s 3 6 0
30% is s 2
3 K+10 0
(a) 32% (b) 63% s1 0 0
(c) 142% (d) None of these (−12−3K)/3
s0 K + 10 0 0
Solution:  The closed-loop transfer function is
In order to keep all the entries of first column
K positive,
Gcl (s) =
s2 + s + K
−12 − 3K > 0 or K > −4.
The undamped natural frequency is K rad/s Further to make last entry positive, K > −10.
and damping factor is z = 0.5/ K . Ans. (b)
Thus, 1 / z 2 = 4K 23. A unity feedback system is as shown in the follow-
For 40% peak overshoot, we have ing figure

−pz 1 / 1−z 12 R(s) K(1 + Ts) Y(s)


e = 0. 4
+ s2(1+s)
−pz 1 / 1 − z 12 = −0.916 −

−p −p
= = −0.916 The system is stable for all positive values of K, if
1 − z 12 −1 4K1 − 1
(a) T = 0 (b) −1 < T < 0
(c) T > 1 (d) 0 < T < 1
Thus, K1 = 3.19. Similarly for 30% peak overshoot,
we get K2 = 1.96, and K1/K2 = 1.63. Thus, K must Solution:  The characteristic equation of the
be reduced by 63%. closed-loop system is s3 + s2 + KTs + K = 0.
Ans. (b) Formulating the RH array

Chapter 6(487-554).indd 543 3/23/2016 2:29:40 PM


544     Chapter 6:  CONTROL SYSTEMS 

s3 1 KT which means that damping factor is 0.5. On the


other hand, for K = 0, the characteristic equation
s2 1 K is s2 + 1 = 0 which means that damping factor is
1 0. Hence, the ratio of 0.5 to 0 is ∞.
s KT − K 0
Ans. (d)
s0 K 0
26. Find the range of K for which the system shown in
the figure remains stable.
For stability, all entries in first column are to be
positive, so R 1 K s−1 1 C
(KT − K) > 0, it means that T > 1, since
already K > 0. −1 −1
Ans. (c)
−2
24. For the system shown in the following figure, the
limiting value of K for stability of inner loop is (a) K > −1/3 (b) K < 0
X < K < Y. (c) K > 0 (d) K > −1
Solution:  The overall transfer function can be
R(s) K Y(s) determined either directly or using signal flow
+ + (s−a)(s+2a)(s+3a) graph (SFG) as
− − K
Gc (s) =
s(1 + K ) + (1 + 3K )
Stability requires that simultaneously
Then the overall system will be stable if and only if 1 + K > 0, that is, K > −1, and
(a) 2X < K < 2Y (b) 4X < K < 4Y 1 + 3K > 0, that is, K > −1/3
(c) X < K < Y (d) X/2 < K < Y/2 Hence, the correct answer is option (a).
Ans. (a)
Solution:  It is given that the inner loop is stable, 27. Given an all-pass filter with the open loop gain
that is, no root of the characteristic equation
formed with inner loop lies in right half of s-plane. K(s2 − s + 1)
G(s) =
(s2 + s + 1)
(s − a)(s + 2a)(s + 3a) + K = 0
the value of K for which the system shall observe
The characteristic equation of the overall system is
undamped oscillations under unity feedback is
(s − a)(s + 2a)(s + 3a) + 2K = 0 (a) 1 (b) −1
(c) 2 (d) None of these
Since K is now multiplied with 2, it means that
Solution:  Under unity feedback the closed-loop
acceptable range of K shall also be reduced by 2.
characteristic equation is given by
Hence the range is X/2 < K < Y/2
Ans. (d) (1 + K)s2 + (1 − K)s + (1 + K) = 0
25. A system with G(s) = 1/s2 is stabilised as shown Formulating the RH array
in the following figure.
s2 1+K 1+K
R(s) 1
Y(s) s1 1−K 0
+ + s2 0
− −
s 1+K 0
Ks For sustained oscillations 1 − K = 0, it means that
K=1
Ans. (a)
The ratio of damping ratio with K = 1 to that with
K = 0 is
Root Locus Technique
(a) 2         (b) 1        (c) 0        (d) ∞
28. The open-loop transfer function of a unity feedback
Solution:  For K = 1, the characteristic equation system has four poles located at 0, − 1, 0.5 + j 0.5,
of the overall system is given as s2 + s + 1 = 0 0.5 − j 0.5.

Chapter 6(487-554).indd 544 3/23/2016 2:29:50 PM


SOLVED EXAMPLES     545

The angle of departure at the complex pole with Solution:  The root locus will exist on the real axis
negative imaginary part is if and only if the total number of poles and zeros to
(a) 45     (b) 90     (c) 0     (d) 180 the right of the point are odd. So in this case, the
root locus must exist between s = 0 and s = −2.
Solution:  The root locus diagram of the given The angle of asymptotes are
open-loop transfer function is as shown below.
180(2k + 1)
Angle of asymptotes = = ±90°
2
Thus, the root locus moves along the direction
−1 0 +90° and −90°.
−0.5 Ans. (a)
K
The poles are symmetrically located horizontally 31. Given that G(s) = , then the value of
s(s + 2)
as well as vertically. It is clear from the root locus
that the angle of departure from the complex pole K for which one of the closed-loop pole lies at
with negative imaginary part is 90°. Hence, −1 + j 10 is
fdep − 90 − 45 − 135 = −180
1
fdep = 90 (a) 11    (b) 101     (c) 101    (d)
101
Ans. (b)
29. The number of breakaway points in root locus plot Solution:  The pole zero diagram with the point
1 −1 + j 10 is as shown in the following figure.
of G(s) = are
(s + 20)(s2 + s + 4) −1+j10
(a) 1      (b) 2      (c) 0      (d) 3 (10)2 + 1

Solution:  Since the pole at s = −20 is very far


away from the complex conjugate pole pair, there
will not be any breakaway point. The same deci- −2 0
sion can also be arrived by using the dK/ds = 0
procedure and finding out that there are no break- The gain at the given point is equal to product of
away points. The characteristic equation is lengths from poles divided by product of lengths
s3 + 21s2 + 24s + 80 + K = 0 from zeros.
    K = −(s3 + 21s2 + 24s + 80)
K = 101 × 101 = 101
dK/ds = −3s2 − 42s − 24 = 0
s = −13.4, −0.59
Ans. (c)

None of these points lie on the segments where root locus 32. For a system with unity positive feedback, which
exists. Hence, none of them can be breakaway point. one of the following cannot be the root locus.
Ans. (c)
30. The root locus plot of the system with open-loop (a) (b)
poles at s = 0 and s = −2, is given by
(a)

−2 0
(b)

−2 0 (c) (d) None of these


(c)

−2 0

(d) None of these

Chapter 6(487-554).indd 545 3/23/2016 2:30:01 PM


546     Chapter 6:  CONTROL SYSTEMS 

Solution:  For systems with positive feedback, the Solution:  Given that peak overshoot value is 30%,
root locus exists at a point, the total number of therefore
poles and zeros to the right of it are even. This is
the case in option (c). Further, the angle of asymp- M p = e−pz / 1−z 2
= 0.3
totes should be 0, 120 and 240 degrees for positive
feedback. Taking log of both sides
Ans. (c) pz
−1.2 = −
33. Given a system with open-loop poles and zeros as 1−z2
shown in the following figure.
11.31z 2 = 1.45
z = 0.357
The undamped natural frequency can be calculated
−20 −2 −1 0
using RH array as
s3 1 60
Then which of the following is false? 2
s 19 K
(a) There exists a gain K for which system is mar- 1
ginally stable
s (1140 − K)/19 0
0
(b) System is stable always for all gains s
(c) The system has one asymptote with angle 180°.
(d) None of above By making the s1 row as zero with K = 1140, we
can formulate the auxiliary equation for s2 row as
Solution:  The centroid of the root locus shall be 19s + 1140 = 0
        s = ± j 7.74
−1 − 2 − (−20) 17
sA = = = +8.5 With z = 0.357 and wn = 7.74, we have the dominant
3 −1 2
pole pair at −zw n ± jw n 1 − z 2 = −2.75 ± j7.24
The angle of asymptotes are ±90°. Thus, the root Ans. (b)
locus will approach towards infinity along the
asymptotes at angles ±90° originating from +8.5. 35. For the unity feedback system given in Question
It means that root locus will enter into right half of 34, the value of gain K for which the closed-loop
s-plane, and hence it will cross the imaginary axis system will admit 30% peak overshoot is
as shown in the following figure. (a) 900   (b) 812.3   (c) 813.5   (d) 808.4

Solution:  The required gain can be calculated


using the product of phasor lengths from open-loop
poles and zeros as
−20 −2 −1 0
s − p1 s − p2
Gain K =
s − z1
Hence, there exists some gain K for which the The phasor lengths can been calculated with the
system response is purely oscillatory and system help of following pole zero diagram
marginally stable.
Ans. (a) −2.75 ± j7.24
34. Given a unity feedback system with 14.23 7.74
7.34

G(s) =
K −15 −4 0
s(s + 4)(s + 15)

The dominant poles of the system for 30% over- Therefore,


shoot are located at 7.74 × 7.34 × 14.23
K= = 808.4
(a) −2.5 ± j4.3 (b) −2.75 ± j 7.24 1
(c) −1.25 ± j4.3 (d) −1.25 ± j 7.24 Ans. (d)

Chapter 6(487-554).indd 546 3/23/2016 2:30:14 PM


SOLVED EXAMPLES     547

Frequency Response Analysis Solution:  The loop gain of the system is


10(s + 1)
36. The Nyquist plot can be used to adjudge G(s)H(s) =
s(s + 10)
(a) stability and transient response
(b) steady-state errors The crossover frequency wgc can be calculated as

G( jw gc )H( jw gc ) = 1
(c) absolute as well as relative stability
(d) absolute as well as relative stability and number
of closed-loop poles in right half of s-plane. Substituting, we get

Solution:  The Nyquist stability criterion tells 10 w gc


2
+1
about the stability of the closed-loop system in =1
terms of encirclements of the −1 + j 0 point. Also w gc w gc
2
+ 100
it is used to calculate the stability margins; phase
margin and gain margin, which represent the rela- Squaring, we get
tive stability of the system. Also the number of 10(w gc
2
+ 1)
right half-plane closed-loop poles can be adjudged =1
from the net encirclement of −1 + j 0 point. w gc (w gc
2
+ 100)
Ans. (d)
Let w gc
2
= p, then we get a second-order equation as
37. The Nyquist plot of a system is as shown in the
following figure. Which one of the following can be 10( p + 1)
open-loop transfer function associated with it? =1
p( p + 100)
p2 + 90p − 10 = 0
which gives p = 90 and hence wgc = 9.48, at which
−1/3 1 the phase of the loop is given as
−90 + tan−1(9.48) − tan−1(9.48/10) = −85°
s −1 s+1 Thus, the phase margin is −180 + 85 = 95°.
(a) (b)
s + 3 3(s + 2) Ans. (d)
s+1 39. The phase crossover frequency of the system
(c) (d) None of these 10
s + 3 G(s) = is
s(s + 20)(s + 10)
Solution:  The static gain at w = 0 is 1/3, hence
(a) 8.3    (b) 28.2    (c) 14.14    (d) 20
(a) and (b) cannot be the correct options. The
Nyquist plot starts with a negative gain −1/3, it
Solution:  At the phase crossover frequency, the
means that there is either a pole or a zero of the
phase of the loop transfer function is −180°.
open-loop transfer function lying in the right of
s-plane. The final gain of the system at w = ∞.
Thus,

s −1 s −1 = −90 − tan−1(w /10) − tan−1(w /20) = −180


lim = −1/3 and lim
s→ 0 s + 1 s→ ∞ s + 1
which upon solving gives w = 14.14 rad/s.
1 − 1/s
= lim =1 Ans. (c)
s→∞ 1 + 1/s
40. Which of the following is the transfer function cor-
Ans. (c) responding to the Nyquist plot shown below?
38. In the Nyquist plot for the following system,

R(s) 10 Y(s)
+ s+10

1+ 1s

the phase margin of the system is

(a) 45°     (b) 60°    (c) 107.5°    (d) 95°

Chapter 6(487-554).indd 547 3/23/2016 2:30:33 PM


548     Chapter 6:  CONTROL SYSTEMS 

K(s + 1) K Solution:  The gain of the system at w = 0 is


(a) (b) finite, that is, −2 and angle −180°. It means
s2 (s + 1)(s + 5) s2 (s + 2)(s + 5) that
K K(s + 3)(s + 6) (i) There cannot be poles at the origin.
(c) (d)
s(s + 2)(s + 5) s(s + 2)(s + 5) (ii) There cannot be zeros at the origin.
So the options (a) and (d) are ruled out.
Solution:  The polar plot starts from −180° which
Since the initial starting angle is −180, it can
means that it is type 2 system. So, options (c) and
happen only when either pole in right half of
(d) cannot be the correct choices. The Nyquist plot
s-plane or zero in right half of s-plane. Thus, here
of option (a) will first enter in the third quadrant
option (b) is the answer.
and then may go to second. But the given plot first
Ans. (b)
goes into second quadrant, that is, initial −180°
angle due to double pole at origin and still another
pole in continuity, means first corner frequency Bode Plots
should be a pole. This is the case in option (b).
Ans. (b) 43. Asymptotic Bode gain plots for a non-minimum
phase system are as follows:
41. A unity feedback system has open-loop transfer
function as

G(jω)
1 60
G(s) =
(s − 1)(s + 2)(s + 3) 40
The Nyquist plot of the system encircles the origin

20log10
20
(a) once (b) twice ω
0.1 1 10
(c) thrice (d) never

Solution:  The question can be solved using


Routh—Hurwitz array. The closed-loop system’s The corresponding transfer function is
characteristic equation is
10(s + 1) 100(s + 1)
(s − 1) (s + 2) (s + 3) + 1 = 0 (a) 2
(b)
s3 + 4s2 + s − 5 = 0 s(s + 10) s(s + 10)2

Using RH criterion, one root (zero) of the characteris- 10(s + 1) 100(s + 1)


tic equation lies in right half of s-plane. Z = 1. There (c) 2
(d)
s(0.1s + 1) s(s + 10)2
is one unstable open-loop of G(s), that is, P = 1. As
per the Nyquist criterion, the total encirclements are
Solution:  The initial slope of the Bode plot is
N = P − Z = 1 − 1 = 0. −20 dB per decade, which implies that it is type 1
Ans. (d) system.
42. The polar plot of a system is as shown in the fol- The corner frequencies corresponding to a zero and
lowing figure. double pole are wc1 = 1 and wc2 = 10, respectively.
Thus, the transfer function is
K(s + 1)
G(s) =
s(1 + s/10)2
−2 Gain at w = 0.1 is 60 dB, thus

20log10 (K/0.1) = 60
  log10 (K/0.1) = 3
Then the corresponding transfer function is
   K/0.1 = 103 = 100
1 (s − 4)
(a) 2 (b) 2         K = 100
s (s + 4)(s + 5) s +s+2
Hence, the transfer function in option (d) is
(s + 4) −s2 correct.
(c) 2 (d)
s +s+2 (s + 4)(s + 5)(s + 6) Ans. (d)

Chapter 6(487-554).indd 548 3/23/2016 2:30:52 PM


SOLVED EXAMPLES     549

44. The Bode gain plot of a minimum-phase system is 46. For a system with open-loop transfer function
10
G(s) = 2
20 (s + 2s + 10)
−20 dB/decade the resonant frequency and resonant peak are given
dB

0 2 by
−1 1 w (a) 2, 32% (b) 6, 60%
(c) 2.707, 23% (d) 2.828, 66%
−40 dB/decade
Solution:  Comparing the denominator polynomial
with the standard equation s2 + 2zw n s + w n2 = 0
The steady-state error to a unit ramp input is we get
(a) 2     (b) 10     (c) 1     (d) ∞ w n2 = 10 Þ w n = 10
Solution:  The given system is type as the initial 2zw n = 2 Þ z = 0.3166
slope is −20 dB/decade. There occurs a pole at With this, we have resonant frequency
wc = 2. Thus, the system transfer function is

K w r = w n 1 − 2z 2 = 2.828
G(s) =
s(s / 2 + 1) Resonant peak is given by
In order to evaluate K, we find the system gain at 1
Mr =
w = 0.1 2z 1 − z 2
20log10(K/0.1) = 20 = 1.665 = 66%
     K/0.1 = 10 Ans. (d)
Thus, K = 1. 47. The open-loop transfer function of a system is
Hence the velocity error constant is s+2
G(s) = 2 , then the phase margin is
s
1
Kv = lim s . =1
s ®0 s(s/2 + 1) (a) 36.8°    (b) 45.1°    (c) 33.1°    (d) 30.2°

Steady state error is Solution:  At gain cross over frequency

G( jw ) = 1
1
e ss = =1
Kv
jw + 2 4 + w2
Ans. (c) Þ = =1
−w 2 w2
45. For the Bode gain plot of a minimum-phase system
given in Question 44, the damping ratio of the Squaring both sides, we get
system under unity feedback is
w4 = 4 + w2
(a) 1     (b) 0.5     (c) 2     (d) 0.707
Let w2 = p, then we have quadratic equation
Solution:  The characteristic equation of the p2 − p − 4 = 0
system under unity feedback is given by
The roots of equation are given by 1.5 and −2.5.
s2 + 2s + 2 = 0 The negative value cannot be value of w2 and hence
Comparing it with standard equation, we get w2 = 1.5. The phase of the transfer function at this
frequency is
w n = 2 and 2zw n = 2
−180° + tan(1.5/2) = −143.13°
Therefore, hence the phase margin is
1
z = = 0.707 −143.13° + 180° = 36.86°
2
Ans. (d) Ans. (a)

Chapter 6(487-554).indd 549 3/23/2016 2:31:08 PM


550     Chapter 6:  CONTROL SYSTEMS 

Compensators (a) 52° at 4 rad/s


(b) 52° at 6 rad/s
48. The phase lead compensator (c) 55° at 12.3 rad/s
10(1 + 0.3s) (d) None of above
Gc (s) =
Solution:  Here a can be calculated as ratio of the
1 + 0.1s
would provide a maximum phase shift of time constant of pole to the time constant of zero
as a = 0.05/0.5 = 0.1. Thus,
(a) 45°      (b) 30°      (c) 60°      (d) 20°
æ 1 − a ö æ 1 − 0. 1 ö
fmax = sin−1 ç ÷=ç ÷
è 1 + a ø è 1 + 0.1 ø
Solution:  From the compensator transfer func-
tion, t = 0.3 and at = 0.1; thus, a = 0.1/0.3 = 1/3
The maximum phase lead provided is = sin−1(.9 / 1.1) = 55°

æ 1 − a ö æ 1 − 1/3 ö wm =
1 1
fmax = sin−1 ç
= = 6.32 rad/s
÷=ç ÷ t a
è 1 + a ø è 1 + 1/3 ø
0. 5 0 . 1

= sin−1(2/4) = 30° Ans. (d)


Ans. (b) 51. A control system with a PD controller is shown in
the following figure. If the velocity error constant
49. The transfer function of a phase lead network as
Kv = 1000 and the damping ratio z = 0.5, then the
shown in the figure is
value of K and KD are
K(1 + 0.3s)
Gc (s) =
(1 + 0.17s) R(s) 100 Y (s)
K + KDs
+ s(s + 10)
C −

R1 (a) K = 100 and KD = 0.09


Input R2 Output (b) K = 100 and KD = 0.9
(c) K = 10 and KD = 0.09
(d) K = 10 and KD = 0.9
Then the value of R1 and R2 are, respectively
(a) 300 kΩ and 300 kΩ Solution:  The total loop gain is
(b) 300 kΩ and 400 kΩ
(c) 400 kΩ and 300 kΩ 100(K + KDs)
G(s) =
(d) 400 kΩ and 400 kΩ s(s + 10)

Solution:  The transfer function of the circuit Thus, Kv = 10K, in order to obtain the desired
shown is given as Kv = 1000, K = 100.
R2 (1 + R1Cs) The closed-loop characteristic equation now
(R1 + R2 )[(1 + R1R2Cs)/(R1 + R2 )] becomes
s2 + s(10 + 100KD) + 100K = s2 + s(10 + 100KD)
Comparing the transfer function with the given + 10000
one R1C = 0.3 and R1R2C/(R1 + R2) = 0.17. The undamped natural frequency is
Substituting the value of R1C in the second rela-
tion, we get R2/(R1 + R2) = 0.56, which means w n = 10000 = 100 rad/s,
that the resistances are in ratio of 3:4, and hence
option (b) is the answer. and the damping factor becomes
Ans. (b)
2z × 100 = 10 + 100KD
50. A phase lead compensator has the transfer func-
0.5s + 1 (2 × 0.5 × 100 − 10 )
tion Gc (s) = . The maximum phase lead KD = = 0. 9
0.05s + 1 100
provided is Ans. (b)

Chapter 6(487-554).indd 550 3/23/2016 2:31:22 PM


SOLVED EXAMPLES     551

State Space Representation Solution:  The state transition matrix is inverse


Laplace of (sI − A)−1. Given A, we have
52. State space representation of a system is
[sI − A] = éê3
s 0 ù
é0 0 ù é0 ù s + 4 úû
ë
1
x = ê 0 0 1 ú x + ê0 ú
ê ú ê ú
ë−1 −2 −3 û ë1 û és + 4 0 ù
adj [sI − A] = ê
y [1 2 1] x ë −3 s úû

The transfer function Y(s)/R(s) is det [sI − A] = s2 + 4s = s(s + 4)


é s+4 0 ù
s2 + s + 1 s2 + 3s + 1 ê s(s + 4) ú
[sI − A ]−1
(a) (b)
s3 + 4s2 + 2s + 1 s2 + 2s2 + 2s + 1 =ê ú
ê −3 s ú
s2 + 3s + 1 s2 + 2s + 1 êë s(s + 4) s(s + 4) úû
(c) (d)
s3 + 3s2 + 2s + 1 s3 + 3s2 + 2s + 1 é 1 0 ù
ê s ú

1 úú
Solution:
−1 ê −3
 s −1 0  êë s(s + 4) (s + 4) úû
adj(sI − A)
[sI − A ] −1
= 0 s −1  =
  sI − A
 1 2 s + 3 Taking inverse Laplace transforms, we get

|sI − A| = s3 + 3s2 + 2s + 1  u(t) 0 


f(t) = 
s2 + 3s + 2 s + 3 1 (3 / 4)(e
−4 t
− 1)u(t) e u(t)
−4 t

adj(sI − A) =  −1 2 
 s + 3s s  Ans. (b)
 −s −2s − 1 s2 
54. A dynamical system is modeed as
C . adj(sI − A)B
G(s) =
sI − A  0 1 1 
x =   x+  u
− 3 − 6  0 
és2 + 3s + 2 s + 3 1 ù é0 ù y = [1 0] x
[1 2 1] êê −1 ú
s + 3s s ú êê 0 úú
2

êë −s −2s − 1 s2 úû ë 1 û then which of the following is true?


G(s) =
D (a) System is stable, controllable and observable.
(b) System is stable, controllable but not observable.
(s2 + 2s + 1) (c) System is unstable, controllable but not observable.
=
(s3 + 3s2 + 2s + 1) (d) System is unstable, uncontrollable and unob-
servable.
Ans. (d)
53. The state transition matrix corresponding to the Solution:  It can be shown that the system is
system matrix stable, as

0 1  s −1 
sI − A =  = s2 + 6s + 3
3 s + 6 
A=
4
is
− 3 −

 0 
2u(t) has negative roots. Further, the controllability
(a)  −4 t −4 t
u(t)
matrix
(3 / 4)(e − 1)u(t) e
é1 0 ù
U c = [B AB ] = ê

(b) 
u(t) 0  ë0 −3 úû
(3 / 4)(e
−4 t
− 1)u(t) e u(t)
−4 t
has rank 2, thus system is controllable also. In
 u(t) 0  addition, the observability matrix
(c) 
(3 / 4)(e
−4 t
− 1)u(t) e u(t)
−3 t
é1 0 ù
U o = éC T AT C T ù = ê
ë û ë0 1 úû
(d) None of these

Chapter 6(487-554).indd 551 3/23/2016 2:31:46 PM


552     Chapter 6:  CONTROL SYSTEMS 

also has rank 2, thus, system is observable. Both of the states are affected by the input,
Therefore the given system is stable, control- hence system is controllable. However, the second
lable and observable. state has no connection with the output. Thus,
Ans. (a) output contains no information about the state x2,
55. An LTI system is represented using the state dia- which makes the system unobservable.
gram as follows. Ans. (b)
56. For the system given as
1/s

y + 3y + 2y + y = 3u(t)
2 4
u 1 1 y
−1
the system matrix A related to observable canoni-
cal form is
3 1/s
0 0 −1 −3 0 0
(a) 1 0 −2 (b)  0 −2 0 
   
−2 0 1 −3  0 0 −1

The given system is 0 1 0 0 1 0


(c)  0 0 1 (d)  0 0 1
   
(a) stable, not controllable and not observable.
(b) stable, controllable and not observable. −1 −2 −3 −3 −2 −1
(c) stable, not controllable but observable.
(d) stable, controllable as well as observable. Solution:  In observable canonical form, the last
column of the system matrix is constructed using
Solution:  The state differential relations are given as the negative of the coefficients of the characteris-
tic polynomial in reverse order. The characteristic
x1 = x1 + 2u polynomial for the given differential equation is
x2 = −2x2 + 3u s3 + 3s2 + 2s + 1, therefore the coefficients of the
last column of A matrix must be −1 −2 −3, and
Then it is clear that the poles of the system are −1 hence option (d) is the correct answer.
and −2. Hence, the given system is stable. Ans. (d)

PRACTICE EXERCISES

Set 1 (One Mark Questions)   (a) Tracking a varying control command


(b) Rejecting some known disturbances
1. In a signal flow graph, the value of a node is equal to   (c) Sensitivity to feedback path parameter variations
(d) Sensitivity to forward path parameter variations
(a) number of outgoing branches
(b) sum of the gains of the output branches 4. The transfer function of two cascaded RC circuits is
(c) sum of all incoming signals not equal to the product of their individual transfer
(d) sum of transmittance of incoming branches functions because of
2. If we make the system insensitive to the variation (a) different concepts of energy storage and release
in forward path gain using negative feedback, it (b) reflection of the waves at their connecting
becomes points
(c) loading effects of the second section on the first
(a) slow and stable
section
(b) more sensitive to feedback element variations
(d) loading effects of the first section on the second
(c) stable as well as insensitive to variation in feed-
section
back path gains
(d) transient response improves at the cost of 5. Unit step response of a system can be obtained
stability from the unit impulse response by
3. Which one of the following cannot be achieved (a) differentiating the impulse response
using open-loop control systems? (b) integrating the impulse response

Chapter 6(487-554).indd 552 3/23/2016 2:31:54 PM


PRACTICE EXERCISES     553

(c) multiplying the impulse response by 1 − e−t.


(d) None of above. h2 h3
R C
6. Signal flow graphs are g1 g2 g3
(a) the technique to simplify the graphical repre- h1

g1g2 g3 (1 − h2h3 )
sentation and obtain a single block representing
overall transfer function
(a)
(b) graphical technique to obtain transfer function 1 − h2 − g2h1 − h3 − h2 g3
using analytical results
(c) arrangements of subcomponents in a feed for- g1g2 g3
(b)
ward manner only 1 − h2 − g2h1 − h3 − h2 g3
(d) None of the above
g1g2 g3
7. For a tachometer, if q(t) and w(t) are rotor angular (c)
displacement and angular speed, e(t) in the output 1 + h2h3 − h2 − g2h1 − h3
voltage and Kt is the tachometer constant, then (d) None of these
the transfer function E(s)/q(s) and E(s)/w(s) is
given by 12. The open-loop DC gain of a unity negative feed-
back system with overall DC gain 2/3, is
K
(a) Kt and Kts (b) t and Kts
s (a) 1/2    (b) 2/5   (c) 3/2   (d) 2
K
(c) Kts and Kt (d) Kts and t 13. A system follows a triangular wave as shown in the
s following figure
8. The transfer function of a second-order all pass
network is
r(t) y(t)
s2 s2 + 1
(a) (b)
s2 + as + b s2 + as + b 0 t
s s − as + b
2
(c) (d)
s + as + b
2
s2 + as + b Then type of the system is

9. In a force-voltage analogy, the mass M, stiffness (a) 0    (b) 2    (c) 1    (d) None of these
k and viscous friction f are equivalent to which of 14. A system with G(s) = 1/(s + 2) is controlled using
the following corresponding electrical quantities (in an integral controller by inserting integrator in the
order) as forward path. The steady-state error for the overall
(a) R, L, C (b) L, C, R system to step commands is
(c) C, L, R (d) R, C, L (a) ∞     (b) −∞   (c) 2/3   (d) 0
10. The input output relations of a system are as 15. An integral controller K(s) = 1/s is required to be
shown in figure, the system is placed in tandem with the plant G(s) so as to make
the steady-state error for ramp input as zero. Then
y(t) the type of G(s) is
(a) 2      (b) 1    (c) 0    (d) ∞
16. The step response of a dynamical system repre-
0 r(t) sented using y + y − 2y = u is

(a) y(t)
1
(a) linear (b) non-causal
(c) time varying (d) None of above
11. The overall transfer function C/R from the signal
flow graph shown in the following figure is 0 t

Chapter 6 (555-577).indd 553 3/23/2016 3:24:53 PM


554     CHAPTER 6:  CONTROL SYSTEM

(b) y(t) (c) calculate the number and angle of asymptotes


(d) All of above
1
23. The root locus for a unity feedback with open-loop
transfer function having gain parameter K exist on
real axis if and only if the angle subtended by poles
0 t and zeros is
(c) y(t) (a) kp (b) (2k + 1)p
(c) 2kp (d) None of these
1
24. The asymptotes in the root locus plot of unit feed-
1
back configuration of G(s) = inter-
0 t s(s + 1)(s + 2)
sect with the real axis at
(d) y(t)
(a) −1.25   (b) −1.5     (c) −1    (d) −1.15
25. An open-loop transfer function has three zeros at
infinity, then the relationship between the number
of poles (P) and the number of zeros (Z) is given by
0 t
(a) P − Z = 3/2 (b) 3P = Z
17. None of the poles of a linear control system lie in (c) P − Z = 3 (d) Z − P = 3
the right half of s-plane. For a bounded input, the
26. The Nyquist plot of a system passes through
−1 + j0 point, then the phase margin of the system
output of this system
(a) is always bounded is given by
(b) always tends to zero
(a) 45°    (b) 90°     (c) 180°    (d) 0°
(c) could be unbounded
(d) none of above 27. The polar plot of the system is shown in the follow-
ing figure. The type of the system is
18. The number of positive real roots of the character-
istic equation s3 − 2s + 2 = 0 are Im
(a) 1 (b) 2 Re
(c) 0 (d) cannot be obtained
0
19. The unit step response of a system is 1 − et (1 + t).
The system is
(a) unstable (a) 1 (b) 0
(b) stable (c) 2 (d) none of these
(c) critically stable
28. A pure transportation lag has transfer function
G(s) = e−sT. The polar plot of the system is given
(d) stability depends upon the input
20. Which of the following is a bounded signal? by
(a) e2t (b) t
(c) e−3t (d) et sint (a) (b)

21. A system with G(s) = −1/(s(s + 1)) is stabilised


with proportional controller G(s) = K. The value
of K for which the system will be marginally
stable is (c) (d)
(a) 1 (b) 2
(c) 2 2 (d) Does not exist
22. The root locus technique can be used for which of
the following 29. A unity feedback system has loop transfer function
(a) transient response study K
as G(s) =
(b) variation of the poles of the closed-loop system s−3

Chapter 6 (555-577).indd 554 3/23/2016 3:25:06 PM


PRACTICE EXERCISES     555

The range of gain, for which the closed-loop system


is stable, is (c) w b/wn

(a) K > 0 (b) K < 0


(c) K < 3 (d) K > 3
30. The polar plot of an open-loop transfer function is z
as follows. (d) w b/wn

z
−1
34. A pole/pair of complex conjugate poles are called
dominant poles, when the distance of other poles
from jw axis is ———— times the distance of dominant
poles from jw axis.

Which of the following statement is true? (a) 5 times (b) 2 times


(c) 10 times (d) 3 times
(a) System is type 0 and has one open-loop unsta-
ble pole. 35. The proportional controller is able to
(b) System is type 1 and is closed-loop stable. (a) reduce the steady-state error
(c) System is type 1 and closed-loop unstable with (b) eliminate the steady-state error
one closed-loop pole in right half of s-plane. (c) increases the gain margin
(d) System is type 1 and closed-loop unstable with (d) none of above
two closed-loop pole in right half of s-plane.
36. The integral control is used for
31. Bode gain plots alone can be used to find the trans-
fer function of a system when (a) reducing the steady-state error
(b) eliminate the steady-state error
(a) system is stable (c) stabilising an unstable system
(b) system is stable and observable (d) improve the transient response of the system
(c) system is non-minimum phase
(d) none of above 37. The response of a system is very fast and also it
shows significant steady-state error. Which com-
32. A compensator is used to increase the bandwidth pensator is it likely to be?
of a system. Which one of the following is true?
(a) Proportional controller
(a) Delay time increases (b) Lead compensator
(b) Rise time increases (c) Lag compensator
(c) System becomes more stable (d) PD controller
(d) System starts observing undershoots in step
response 38. The transfer function of a phase lead compensator
is (s + a)/(s + b) and that of a phase lag com-
33. Which one of the followings represents correct plot
pensator is (s + p)/(s + q), then which one of the
of z for a second-order system?
following sets of conditions must be met?
(a) w b/w n (a) a > b and p > q
(b) a > b and p < q
(c) a < b and p < q
(d) a < b and p > q
z
39. With regards to the filtering property, the lead com-
(b) w b/wn pensator and the lag compensator are respectively
(a) low pass and high pass filters
(b) high pass and low pass filters
(c) both are high pass filters
z (d) both are low pass filters

Chapter 6 (555-577).indd 555 3/23/2016 3:25:12 PM


556     CHAPTER 6:  CONTROL SYSTEM

40. A compensator transfer function is (a) jω (b) jω


(2s + 1)
( 0.2s + 1) σ σ
Gc (s) =

What is the nature of compensator and its


parameter?
(a) Lag controller, b = 10
(c) jω (d) jω
(b) Lag controller, b = 2
(c) Lead controller, a = 0.1
(d) Lead controller, a = 0.9 σ σ
41. A composite RC network has the following transfer
function
1 + 21s + 20s2 47. A unity feedback system with G(s) = 1/s2 in cas-
G(s) = 2
1 + 11s + 10s cade with a PD controller
The network can be used as which one of the Gc(s) = Tp(1 + Tds),
following?
where Tp and Td are adjustable parameters, then
(a) Phase lead compensator which one of the followings is not true?
(b) Phase lag compensator
  (c) Lag Lead compensator (a) The system is always stable.
(d) None of above (b) The system may have damped oscillations for
a step input.
42. A temperature control system is very sluggish. To (c) The system amplifies the noise.
improve its dynamics (d) The system has zero steady-state error for par-
 (a) a PI controller can be used abolic inputs.
(b) an integral control can be used 48. A compensator with the transfer function
  (c) a PID controller with large integral action and
very low derivative action can be used (1 + 0.1s)
(d) a PD controller can be used G(s) =
(1 + s)
43. The phase lead compensation is used to
can give a maximum gain and minimum gain of
 (a) increase rise time and decrease overshoot
(a) −20 dB and −40 dB
(b) decrease both rise time and overshoot
(b) −10 dB and −40 dB
  (c) increase both rise time and overshoot
(c) 0 dB and −20 dB
(d) decrease rise time and increase overshoot
(d) 0 dB and −40 dB
44. Introduction of an integral action in the forward
49. PD controllers are used to improve
path of a type 0 unity feedback system result in a
(a) transient response
  (a) marginally stable system
(b) steady-state accuracy
(b) system with increased margin of stability
(c) increase stability
 (c) system with better speed of response
(d) both (a) and (c)
(d) system with no steady-state error to unit steps
50. What is the effect of the phase lead compensator on
45. A second-order unity feedback system is found
the gain cross-over frequency wgc and bandwidth w b?
to be oscillating with high frequency. The
oscillations (a) Both are increased.
(b) wgc is increased but w b decreased.
 (a) can be eliminated using proportional action
(c) wgc is decreased but w b increased.
(b) can be eliminated using integral action
(d) Both are decreased.
 (c) can be eliminated using derivative action
(d) cannot be reduced 51. Consider the following statements:
46. The pole zero configuration of a phase lead com- I. Phase lead compensation shifts the gain cross-over
pensator is given by frequency to the right.

Chapter 6 (555-577).indd 556 3/23/2016 3:25:17 PM


PRACTICE EXERCISES     557

II. The maximum phase lead angle occurs at the arith- (a) 2 (b) 1/2
metic mean of the corner frequencies of the phase (c) 1 (d) K can have any value
lead network.
3. The signal flow graph of system is as shown in the
III. Phase lead compensation is effective when the
figure. The value of C/R is
slope of the uncompensated system near the gain
cross-over is low.
Which one of the statements given above is true? R a b c d C
(a) I, II, and III (b) I and II
(c) II and III (d) I and III −f e

52. The maximum value of a controller output is 100 V abcd abcd


(a) (b)
and is obtained when the input error is 1 V. If the 1 + bf − ce − g 1 − bf + ce + g
controller is working at 20% proportional band, the
error and output will be, respectively, abcd
(c) (d) None of these
1 − gf + ce + g
(a) 0.2 V and 100 V (b) 1 V and 20 V
(c) 1 V and 100 V (d) 0.2 V and 80 V 4. The overall transfer function C/R of the system
represented using signal flow graph as in the fol-
53. Given an autonomous system x = Ax non-singu-
lowing figure, is
lar A matrix and non-zero initial state x(0), the
state transition matrix is a b c
R C
−1 −1
(a) [sI − A] (b) [sI −A] x(0) g
(c) eAtx(0) (d) Laplace inverse of [sI − A]−1 d
f
e
Set 2 (Two Marks Questions) h

1. For a closed-loop system shown in figure, the over-


all gain Y(s) /R(s) is (1 + h)abc (1 − h)(abc + adfc)
(a) (b)
(1 − h)adfc

(1 + h)
R(s) Y(s)
G1 G2 abc + adfc
+ + (c) abc + agfc (d)
− + 1+h
H1 sH2 5. The dynamics of a system are governed by
y(t) − x
(t) − x(t − 1) = 0 . Then the transfer func-
sG1G2 tion of the system is
(a)
1 − s G1G2 H1H2 (a) s2 + se−s (b) 1 + s2e−s
2

G1G2 (c) 1 + s−2e−s (d) 1 + s−2es
(b)
1 + sG1G2 H1H2 − sG2 H2
1
6. A system with G(s) = is controlled using
G1G2 s(s + 1)
(c)
1 + sG1G2 H1H2 + sG2 H2 a first-order compensator and unity negative feed-
back. The closed-loop system has poles at −1±1
sG1G2 and −4. Then the transfer function of the compen-
(d)
1 − s G1G2 H1H2
2 sator block is

2. The system shown in the following figure has gain s+3 2s + 3


(a) (b)

2, then the value of K is 2(s + 5) (s + 5)

R(s) Y(s) 5(s + 1.6) 3(2s + 3)


(c) (d)
G 2/G s+5 s+4
+ +
− +
7. A RLC circuit with ideal diodes is as shown in the
l/G KG following figure. The order of the system is

Chapter 6 (555-577).indd 557 3/23/2016 3:25:45 PM


558     CHAPTER 6:  CONTROL SYSTEM

G1(1 − H1G2 )
R L (c)
1 + G1H2G2 H1G1
D1 C D2 G1(1 + H1G2 )
(d)
1 + G1H2G2 H1G1
(a) 1    (b) 2    (c) 4    (d) 3
11. The order of the system shown in figure is ––.
8. A series RLC circuit has R = 10  Ω, L = 2 mH and
C = 20 mF, then the decay rate of the circuit is
C1
given as C2
(a) 10000 (b) 5000 L1 L2
(c) 4330 (d) 2500
v1 L3 R v2
9. The transfer function X(s)/F(s) of the system
shown in figure is
(a) 5    (b) 2    (c) 4    (d) 3
x(t) 12. The transfer function from N(s) to Y(s) for system
shown in the following figure is
k1 k2 M k3
F(t) G2
f +
R(s) + Y(s)
G1
+
(a)
1 − +
+ N(s)
æ kk ö H1
s2M + sf + ç k3 + 1 2 ÷
è k1 + k2 ø
G1 + G2 −G1H1
1 (a) (b)
s2M + sf + (k1 + k2 + k3 )
(b) 1 + G1H1 1 + G1H1

−G1H1 + G2 −(G1 + G2 )H1


1 (c) (d)
(c)

k1k2 1 + G1H1 1 + G1H1
s2 (M + k3 ) + sf +
k1 + k2
13. A system with step response y(t)= (1 − e− t )u(t) is
1 placed in negative feedback with a feedback ele-
ment having impulse response (1 − e− t )u(t), then
(d) 2
s M + s(f + k3 ) + (k1 + k2 )
the overall transfer function of the system is
10. For the block diagram shown in the following
s(s + 1) s(s + 1)
figure, the ratio Y(s)/U(s) is (a) (b)
1 − s(s + 1) 2
1 + s(s + 1)2
U(s) Y(s)
G1(s) 1 (s + 1)
+ + (c) (d)
− 2
s (s + 1) 2
s(s + 1)2 − 1
H1(s)
+
14. The system with G(s) = 1/s(ts + 1), when con-
H2(s)
− nected in a unity feedback produces error in step
response as e(t) = Ke−4t sin(10t + f ). The system
V(s) +
G2(s)
time constant t is
G1(1 + H1G2 ) (a) 0.01    (b) 0.03    (c) 10    (d) 4
(a)
1 - G1H2G2 H1G1 15. In a series RLC circuit with R = 100  Ω, L = 1 mH
and C = 0.1 µF. Then the peak overshoot will be
G1(1 − H1G2 )
(b)
1 - G1H2G2 H1G1 (a) 10% (b) 12.5%
(c) 24% (d) No overshoot

Chapter 6 (555-577).indd 558 3/23/2016 3:26:13 PM


PRACTICE EXERCISES     559

16. A forced dynamical system is represented by 23. For what value of gain K, the closed-loop system given
in Question 21, shall have sustained oscillations?
y + y + y = (1 − e−t )
1 1
2 18 (a) −5    (b) 4    (c) 1.414    (d) 10
The natural time constants are 24. A second-order system has closed-loop transfer
(a) 1 s, 18 s (b) 2 s, 5 s function as
(c) 1 s, 9 s (d) 3 s, 6 s
(8/a)(s + a)
Gcl (s) =
17. The step response of a first-order system is as (s + 1)(s + 16)
shown in the following figure.
The value of a such that position error constant is
y(t)
100, is
1 (a) 2   (b) 0.5   (c) 4   (d) None of above
0.6
25. The values of K and a for which system shown in
the figure will oscillate with frequency 3 rad/s are,
0 0.48 t respectively
The time constant of the system is
R(s) K(s + 2) Y(s)
(a) 0.612 (b) 0.524
+ s3+ s2+ as+ 1
(c) 0.48 (d) None of these −
18. The time taken by the output of system G(s)
= 1/(s + 1) to settle at 95% of its final value is
(a) 2, 2    (b) 4, 5    (c) 3, 4    (d) 2, 3
(a) 1 s    (b) 2 s     (c) 3 s       (d) 4 s
26. The system with characteristic equation
19. The time response of a system is given as
y(t) = 2 + e−2t sin(wt + f ) + cos(wt + f ) + 1 − e−t s3 + Ks2 + 9s + 18 = 0
then the steady-state value of y(t) is
has capability to have sustained oscillations, then
(a) e−2t sin(wt + f ) − e−t the value of K and frequency of sustained oscilla-
(b) 2 tions wn are, respectively,
(c) 3 + sin(wt + f ) + cos(wt + f ) (a) 4, 5    (b) 4, 6    (c) 2, 3    (d) 2, 5
(d) 3 + cos(wt + f )
27. A derivative feedback minor loop control system is
20. A second-order delay system with dominant second- shown in the following figure.
80e−s
order dynamics is as G(s) =
s(s + 1)(s2 + 16s + 80) R(s) 1 Y(s)
The steady-state error of the system under unity
+ + s2(s + 4)
feedback when excited by tu(t) is − −
(a) 0      (b) ∞     (c) 80      (d) 1 s
21. For the closed-loop system given as follows, the
value of K for which steady-state error to unit step
inputs becomes zero, if t = 1, is
The value of K for which the system has undamped
oscillations is
R(s) Y(s)
1 (a) 1     (b) 2     (c) 3     (d) 4
+ s + 0.2K
− 28. A unity feedback system with the closed-loop
0.5 K
s+t transfer function Gc (s) = 2
is marginally stable. (s + 2s + 2)(s + 2)
(a) 1     (b) −2    (c) 2.5     (d) 3.4
What is frequency of undamped oscillations?
22. With the value of K obtained in Question 21, the
peak overshoot will be (a) 2 rad/s (b) 2/ 2 rad/s

(a) 0.1    (b) 0.3    (c) 0.707    (d) 0.2 (c) 6 rad/s (d) 3 rad/s

Chapter 6 (555-577).indd 559 3/23/2016 3:26:31 PM


560     CHAPTER 6:  CONTROL SYSTEM

29. For the unity feedback system, given in Question 36. Given that the system G(s) has poles and zeros
28, the range of K for which the system is stable is with negative real parts only, and has three zeros
which lie at infinity. How many zeros need to be
(a) K > 0 (b) K > −4
added to G(s) so as to make it always stable under
(c) −4 < K < 20 (d) −1 < K < 10
the unity feedback configuration?
30. Given a unity feedback system with
(a) 1
K
Gc (s) = 2 (b) 3
(s + 6s + 25)(s + 4)(s + 2) (c) 2
The value K for which the system will have sus- (d) Impossible to make it closed-loop stable by
tained oscillations is adding zeros.
(a) 666    (b) 798    (c) 848    (d) 1200 37. A system with open-loop transfer function
1
31. For the unity feedback system given in Question G(s) = is placed in tandem with
30, the range of K for which the system is stable is s(s + 1)(s + 2)
proportional controller. The value of controller
(a) −100 < K < 666 gain K for which the unity feedback configuration
(b) −200 < K < 1332 is always stable is
(c) 0 < K < 666
(d) None of these (a) 10     (b) 12.5     (c) 13     (d) 5
32. The open-loop transfer function of a unity feedback 38. The maximum value of K below which the unity
system is K
feedback configuration of G(s) = exhib-
1 s(s + 10)
G(s) = its non-oscillatory response is
s(s + 2)(s + 3)
the break-away point lies in the interval (a) 25     (b) 10     (c) 2     (d) 10

(a) 0 and −2 (b) −2 and −3 39. A unity feedback configuration has open-loop poles
(c) −3 and −∞ (d) None of these at s = 0, −2, −1 ± j 1, then the angle of departure
of the root locus originating from −1 + j1 is
33. By suitable choice of the parameter K, the unity
feedback configuration of system with open-loop (a) 0°     (b) −90°     (c) 90°    (d) 45°
1
transfer function G(s) = can be 40. The range of K for which the root locus of the
s(s + 2)(s + 4)
made to oscillate at frequency K(s + 8)
open-loop transfer function G(s) = lies
1 on the real axis is s
(a) rad/s (b) 2 2 rad/s
2 (a) K ∈[0, 4] (b) K ∈[0, 8]
(c) 2 rad/s (d) 4 rad/s (c) K ∈ [0, ∞ ] (d) None of these
34. Which of the following system has break-in point
at s = −3.14 41. A unity feedback configuration of an open-loop
K
transfer function G(s) = , the value of K
K(s + 3) K s(s + 2)
(a) (b)
for which the closed-loop system will admit under-

s(s + 6) s(s + 6)
damped response with z = 0.707 is
K(s + 4) K(s + 2)
(c) (d) 1
(s + 3) s(s + 1) (a)      (b) 2      (c) 2     (d) 0.5
2
35. The unity feedback configuration of a system with
open-loop transfer function G(s) is always stable, then K
42. A unity feedback system has G(s) = ,
which one of the following can never represent G(s) s(s + 4)(s + 8)
K K(s + 4) the root locus plot will intersect with the jw axis at
(a) (b) points

s(s + 2) (s + 2)
(a) ±j2 2 (b) ±j3 2
K
(c) (d) None of these
s2 (s + 1) (c) ±j4 2 (d) Does not intersect

Chapter 6 (555-577).indd 560 3/23/2016 3:27:02 PM


PRACTICE EXERCISES     561

43. The characteristic equation of a unity feedback The polar plot of the system is
configuration is s2 + (2 + k)s + 3 = 0. The break-
away point in the root locus plot is (a) (b)
(a) −2    (b) −1    (c) −3    (d) −1.732
44. Given a system with open-loop transfer function 0 1/2 -1/2 1/2
−K(s + 1)
G(s) = 2 then the break-in point is
s + 2s + 1
located at (d)
(c)
(a) Never as P − Z = 1 (b) −0.5
(c) 1 (d) 0
-1 -1/2 -1/2 0
45. A unity feedback system has the open-loop transfer
function

K(s + 1)(s + 1) 49. For the unity feedback system given in Question 48,
G(s) =
(s − 2)(s − 1) the closed-loop system is

the range of K for which the system is stable is (a) marginally stable
(b) critically stable
(a) K ∈[0, 1] (b) K ∈ (1, ∞)
(c) asymptotically stable
(c) K < 1 (d) K ≤ 1 (d) unstable
46. Given a unity feedback system with 50. A unity feedback system has loop gain
K(s − 1)(s − 2) Ke−0.1s
G(s) = G(s) =
s(s + 1) s + 10
Which of the following is true? The value of gain K corresponding to phase margin
of 78° is about
(a) The system will always be stable for all K.
(b) The system will be stable or low gain K. (a) 29.2 (b) 38.5
(c) The system is stable for high gain K. (c) 42.1 (d) None of these
(d) The root locus never crosses jw − axis.
51. The open-loop transfer function of a system is
47. A system has zero steady-state error for step and s+4
G(s) =
ramp inputs and is closed-loop stable also. Which s(s − 1)
one of the followings can be its Nyquist plot?
The polar plot of the system is
(a) (b) (a) (b)

–1

(c) (d) None of the above

(c) (d)

–1
52. For the system given in Question 51, the closed-
loop system is
(a) stable
48. A unity feedback system has loop transfer function
(b) unstable with two poles in right half of s-plane
0.5 (c) unstable with one poles in right half of s-plane
as G(s)H(s) =
−1 + s (d) none of these

Chapter 6 (555-577).indd 561 3/23/2016 3:27:20 PM


562     CHAPTER 6:  CONTROL SYSTEM

53. The asymptotic Bode plot of a first-order system is 126(10s + 1)(14s + 1)


as shown in the following figure (a)
s3
20 12500(s / 10 + 1)(s / 14 + 1)
dB (b)
s2
0.1 1 3 10 w
0 12500(10s + 1)(14s + 1)
(c)
∠G −45°/decade s3
−90°
126(s / 10 + 1)(s / 14 + 1)
(d)
The maximum error in asymptotic approximation s2
of phase and gain at w = 3 rad/s is 58. For the system given in Question 57, the closed-loop
system is excited with an input r(t) = t2/2, then the
(a) 4°, 1 dB (b) 0°, −3 dB
steady-state error is
(c) 6°, 3 dB (d) 4°, dB
(a) 1/126   (b) 126    (c) 0     (d) ∞
54. In the asymptotic Bode plot given in Question 53,
the maximum error in gain and phase approximation 59. For the system given in Question 57, the Bode gain
at frequency w = 1 rad/s is plot for the closed-loop system is
(a) 1.5, 6° (b) 0.46, −5.1° (a) 42
(c) 2.5, 10° (d) 4.5, 10°
55. A system has open-loop transfer function

dB
G(s) =
s2 + 1 ω
s(s + 1)
The output of the system is zero at
(b)
(a) 2 rad/s (b) 0.1 rad/s
dB

(c) 1 rad/s (d) 4 rad/s


10 14
56. A unity feedback configuration has the open-loop ω
transfer function represented by

s−7
G(s) =
(s + 2)(s + 3) + (s − 7)
(c)
Then the closed-loop system is
 (a) stable and minimum phase
dB

(b) unstable and non-minimum phase


   (c) stable and non-minimum phase
10 14 ω
(d) unstable and minimum phase
57. The Bode gain plot of a system is as shown in the
following figure. (d) None of the above

42 60. The system shown in the figure is compensated


using a PD controller. The phase diagram margin
is 45° at 1 rad/s, then the value of K is
dB

10 14
ω R(s) 1 Y (s)
s/2 + K
+ s(s + 2)
−38 −20 dB/decade −

The open-loop transfer function G(s) is given by (a) 1.2    (b) 3.1    (c) 1.45    (d) 1.82

Chapter 6 (555-577).indd 562 3/23/2016 3:27:36 PM


PRACTICE EXERCISES     563

61. A double integrator plant with G(s) = 1/s2 under 66. The state space representation of a system is
unity feedback is to be compensated to achieve the
damping ratio z = 0.5 and undamped natural fre- é0 1 ù é0 ù
x = ê ú x+ ê úu
quency wn = 5 rad/s. Which one of the following ë 0 − 2 û ë2 û
compensator will be suitable? y = [1 0] x
s+3 s + 9. 9 then the damping ratio and decay factor are
(a) (b)
s + 9.9 s+3
(a) 0.707, 2 (b) 1.414, 1
s−6 s+6 (c) 0.5, 1 (d) 0.5, 0.5
(c) (d)
s + 8.33 s
67. Given three forms, that is, controllable canonical
62. The state representation of a system with transfer form, observable canonical form and Jordan form
function G(s) = (s + 4)/(s2 + 5s + 6) is of a dynamical system. Which one of the following
é−5 −a ù é1 ù statements is true?
x = ê ú x+ ê úu
ë 1 0 û ë0 û (a) If controllable canonical form is observable,
y = [1 4] then there is no pole zero cancellation in sys-
tem’s transfer function.
then the value of a is
(b) Observability cannot be judged from control-
(a) 6      (b) 5       (c) 3       (d) 4 lable canonical form.
(c) Stability of the system can only be judged from
63. The transfer function of a system is
Jordan form and no other form.
s+2 (d) Controllability of the given system can be
G(s) = 2
s + 5s + 4 improved using negative feedback.
with state space representation given by 68. A dynamical system is represented in terms of
b 
state space representation as
0 1
x =   x +  1u
− 4 − 5  b2  é0 1ù é1 ù é -1ù
x = ê ú x + ê ú u x(0) = ê ú
ë 0 -3 û ë û
0 ë 3û
then b1 and b2 are
(a) 2, 4    (b) 0, 4    (c) 0, 2    (d) 0, 1 where x(0) is the initial state of the system. Also
system is given a unit step input.
64. A mass-spring system has transfer function as The state transition matrix is
X(s) 1
(a) é1 ù
(1 − e−3t )ú
= 1
F (s) s2M + K ê
ê0 ú
3
and the state space representation is ë e−3t û
 0 1  0 é1 ù
x =  1 −t
(e − e−3t )ú
−b 
x+  u
−a g  (b) ê
ê0 ú
3
then the parameters a, b and g are ë e−t û
(a) −M, −K, 1/M (b) −M, −K, −1/M é1 1 −t ù
(c) −M/K, 0, 1/M (d) −K/M, 0, 1/M (e − e−3t )ú
(c) ê
ê0 ú
3
65. The state space representation of a system is ë e−3 t û
é0 1 ù é0 ù é1 (1 − e−3t )ù
x = ê ú x+ ê úu
ë 0 − 2 û ë2 û (d) ê ú
ë0 e−3t û
y = [1 0] x
69. The state transition equations are
then the position error constant Kp and velocity
é −t ù é e−t ù
(a) x(t) = êt −−et ú (b) x(t) = êt − −
error constant Kv are
(a) Kp = 2 and Kv = 0 ë e û ë 3e t úû
(b) Kp = ∞ and Kv = 0
é −3 t ù é e−3t ù
(c) Kp = ∞ and Kv = 1 (c) x(t) = êt − e− t ú (d) x(t) = êt − −
(d) None of these ë 3e û ë 3e 3t úû

Chapter 6 (555-577).indd 563 3/23/2016 3:28:00 PM


564     CHAPTER 6:  CONTROL SYSTEM

−1 0 0 
70. The free response of a system with state space
0 1 0
 
(b)  0 −2 1 
representation
(a)  0 0 1 
  0 0 0
1 0  0 1 −1 −2 0 
x =   x+  u x(0) =   is
1 1  1 0  0.5 7 /2 0   0.5 7 /2 0 
   −5 0 
(c) − 7 /2 0.5 0  (d) − 7 /2 
é tù é1 ù
(a) êtet ú (b) ê t ú  0 0 0  0 0 0
ëe û ëte û    
74. A unity negative feedback is applied to a system
é tù é t ù with state space representation
(c) ê e t ú (d) ê te t ú
ëte û ë1 − e û é0 1 ù é1 ù
x = ê
ë−2 −3 úû
x+ ê úu
ë0 û
71. A second-order system with system matrix y = [1 0] x

−3 0  3  then the poles of the overall closed-loop system are


A=  , and x(0) =  
 0 − 1 4
located at
(a) −1 ± 0. 707 (b) −1 ± 0.866
then the states of the system after 2 seconds (c) −1.5 ± 0.707 (d) −1.5 ± 0.866
will be
75. The dynamics of a system are described as follows.
 −6   −6  é 3 0ù é1 ù
(a) x(2) = 4e−8  (b) x(2) = 4e−8  x = ê ú x+ ê úu
3e  e  ë− 2 2 û ë0 û
y = [1 1] x
 −6   −6 
(c) x(2) = 3e−2  (d) x(2) = 7e−2  Then the system is
4e  e 
(a) controllable, observable and unstable.
72. The state diagram of an LTI system is as shown (b) not controllable, observable and stable.
(c) not controllable, not observable and unstable.
1/sx2 (d) not controllable, observable and unstable.
2 k 76. The dynamic relations of an LTI system are described
by 
y + 3y + 2y + y = 3u(t), then the system matrix
u 1 i 1 y
j −1 A related to controllable canonical form is
0 0 0 0
3 1/sx1 l 1 1
−2 (a)  0 0 1  (b)  0 0 1
   
−3 −2 −1 −1 −2 −3
m −3 0 0 0 0 −1
(c)  0 −2 0  (d) 1 0 −2
   
If the overall system is unstable as well as uncon- 0 0 −1  0 1 −3 
trollable, then the values of i, j, k, l, m are
77. The state space representation of the system shown
(a) 2, 1, 3, 2, 4 (b) 1, 1, 3, 2, 3 in the following figure, is
(c) −1, 1, 2, 2, 3 (d) −2, 1, 2, 2, 4
C1
73. The system matrix A in the Jordan form of state
space representation of the system shown in the
following state diagram is 2R C2 R
+
v(t) −
u 1 1/s x3 1/s x2 1/s x1 1 y R R/2

−1 (a) neither controllable nor observable.


−2 (b) stable but not controllable.

Chapter 6 (555-577).indd 564 3/23/2016 3:28:28 PM


ANSWERS TO PRACTICE EXERCISES     565

(c) controllable but not observable. Then, which one of the followings is true about the
(d) observable but not controllable. system?
(a) It is controllable but not observable.
78. The state diagram of a system is as follows.
(b) It is observable but not controllable.
(c) It is unstable, but controllable as well as
u 1 1/s 1/s 1 y
observable.
−2 (d) It is stable, controllable as well as observable.
2

ANSWERS TO PRACTICE EXERCISES

Set 1 (One Mark Questions)

1. (c) 6. (b) 15. (b) It is clear that only type 2 systems can exactly
follow the ramp commands, which implies two
2. (b) 7. (c)
poles at the origin. The integral controller K(s) =
3. (d) 8. (d) 1/s provides one pole at the origin. The other pole
at origin is provided by the G(s), that is, G(s) is of
4. (c) 9. (b)
type 1.
5. (b) 10. (b)
16. (d) It is clear that the system is unstable and has
poles at s = −2, 1. Hence, the step response will
11. (c) There is one forward path with gain P1 = g2g2g3
rise unboundedly.
and three loops with gains L1 = h2 L2 = g2h1 and
L3 = h3, further 17. (c) Although, the poles of the system do not
belong to right half of s-plane, but presence of
D = 1 − ( h2 + g2h1 + h3 ) + h2h3 simple poles on jw axis will make the system mar-
D1 = 1
ginally stable and repeated poles on jw axis make
the system unstable. Hence, (c) is the appropriate
C PD g1g2 g3 answer.
Hence = 1 1 =
R D 1 + h2h3 − h2 − g2h1 − h3 18. (c) We can see that the roots of the equation are
complex conjugates, hence, there are no positive
12. (d) The DC gain of the overall system is real roots.

G(0) 2 19. (b) The Laplace transform of the unit step response
Gcl (0) = =
1 1 1 s+2
is C(s) = − −
1 + G(0) 3
=
s s + 1 (s + 1)2 (s + 1)2
Simple calculation of the above relation yields
G(0) = 2.
The final value of the output is C(∞) = 2. Also the
13. (c) It is clear that the input signal is of ramp poles are located in left half of s -plane, the system
type and the system output is able to follow is stable.
it but with finite error. It means that error for
20. (c) All the given signals approach infinity (∞)
except e−3t.
ramp signal ess = 1/Kv is finite. Recapitulate
that only type 1 systems can follow ramp signals
with steady-state errors. Thus, the system is of 21. (d) The closed-loop characteristic polynomial is
type 1. given by s2 + s − K = 0. It is clear that for no
14. (d) By adding the integrator in the forward path value of K, the system can have sustained oscilla-
means that G(s) is multiplied with 1/s, so that tions. Therefore, no value of K exists.
loop gain is G(s)/s. So the type of the system 22. (d)
becomes 1. Type 1 systems can exactly follow step
commands and steady-state error is zero. 23. (b)

Chapter 6 (555-577).indd 565 3/23/2016 3:28:35 PM


566     CHAPTER 6:  CONTROL SYSTEM

24. (c) The point where the asymptotes intersect each Tp (1 + Td s)


other is called centroid, and is given as 47. (d) The closed loop system Gc (s) = is
s2
always stable, amplifier noise is zero, and it may
−1 − 2
sA = = −1 have damped oscillation to a step input. But it
2 cannot have steady state error to parabolic inputs
25. (c) We know that number of zeros at infinity is as it is of Type 2.
Number of poles − Number of finite zeros 48. (c) 50. (a)
P−Z=3 49. (d) 51. (d)
26. (d) Since the Nyquist plot passes through −1 + j 0 52. (b)
point, the system is critically stable. Thus, there
is no further phase margin available. Hence phase 53. (d) The transformed form of the state transition
margin is 0°. matrix is [sI − A]−1; therefore, the state transition
matrix is given by the Laplace inverse of [sI − A]−1.
27. (b) For w = 0, the system has finite gain, that is,
4 and angle 0°, hence it cannot have any pole at
origin, therefore option (b) is the correct choice. Set 2 (Two Marks Questions)

28. (c) The magnitude of a delay system e−sT


1. (b) The simplified block diagram is shown in the
is
following figure.
always 1, however its phase ∠G( jw ) = −Tw Y(s) R(s)
always becomes more and more negative as w G1 G2
+ +
increases. − +
sH2
29. (d) The closed-loop system is given as
sH1H2
K /(s − 3) K
G(s) = = .
1 + K /(s − 3) s − 3 + K The inner loop can be solved equivalent to transfer
So the system will become stable after K > 3. function
G2
30. (d) 36. (b)
1 − sG2 H2
31. (c) 37. (c) Therefore,

32. (b) 38. (d) Y (s) G1G2 / (1 − sG2H2 )


=
R(s) sG G H H
1= 1 2 1 2
1 − sG2H2
33. (a) 39. (b)

34. (c) 40. (c) G1G2


=
35. (a) 1 + sG1G2H1H2 − sG2H2

41. (a) The transfer function of a lag—lead compensa- 2. (d) The overall transfer function can be calculated
tor is as follows: via block diagram reduction as

(1 + t 1s) (1 + t 2s) Y (s) 2


= −2
G1(s) = K × × =
R(s) 1 − 2K + 2K
(1 + bt 1s) (1 + at 2s)
which is independent of K and hence can have any
where a < 1 and b > 1.
value.
Thus, the given second order RC component net-
work can serve as a lag−lead compensator. The 3. (a) Here there is only one forward path with gain
values of K, t1, t2, a and b can be computed by P1 = abcd, and three loops with loop gain L1 = −bf,
comparing the coefficients. L2 = g. Further, there are no two non-touching
42. (d) 45. (c) loops. Also all the loops touch the forward path.
Hence Δ = 1+bf − ce − g, and Δ1 = 1, therefore
43. (b) 46. (a)
abcd
C /R =
44. (d) 1 + bf − ce − g

Chapter 6 (555-577).indd 566 3/23/2016 3:28:48 PM


ANSWERS TO PRACTICE EXERCISES     567

4. (c) There are two forward paths with gains The natural frequency of the circuit is
P1 = abc and P2 = agfc. There is only one loop w n = 1/ LC . Substituting the given values, we
with gain L1 = h. Thus, Δ1 = 1−h, and since this get wn = 5000 rad/s. The decay rate of the circuit
loop does not touch the forward paths, we have is given as s = zwn. Comparing the coefficients of
Δ1 = 1−h, and Δ2 = 1−h. first-order term
C abc(1 − h) + agfc(1 − h)
2zw n = 2 × 5000z =
= = abc + agfc R 10 10000
R 1−h L
=
002
=
2
z =
5. (c) Taking the Laplace transform of the given dif- 1
ferential equation 2

Then the decay rate is given by s = zwn = 0.5 ×


s2 Y (s) − s2 X(s) − e−s X(s) = 0
5000 = 2500 per second.
Therefore, 9. (a) For a mass-spring-damper system, the general
2 −s second-order relations are given as
= 1 + s−2e−s
Y (s) s + e
= 1
X(s) s2 G(s) = 2
s M + sf + ks
6. (c) Let the transfer function of the compensator be
Here k1 and k2 are in series, their equivalent stiff-
k(s + z) ness is given by
Gc (s) =
s+ p k1k2
The transfer function of the overall system under k1 + k2
unity negative feedback is
(1/s(s + 1)) Gc (s)
And the combination of k1 and k2 is in parallel with
k3. Therefore the total stiffness is given as
1 + (1/s(s + 1)) Gc (s)
Gcl (s) =

(1/s(s + 1)) k(s + z)/(s + p) k3 +


k1k2

1 + (1/s(s + 1)) k(s + z)/(s + p)


= k1 + k2

k(s + z) Substituting the value of equivalent stiffness in the


=
s(s + 1)(s + p) + k(s + z) second-order relation, we get
k(s + z) 1
= G(s) =
s3 + ( p + 1)s2 + ( p + k)s + z  kk 
s2M + sf +  k3 + 1 2 
 k1 + k2 
Since the system has poles at −1 ± 1 and −4,
therefore the characteristic equation of the system 10. (b)   The given block diagram can be simplified as
is given by s3 + 6s2 + 10s + 8. Comparing the
coefficients with the above equation, we get k = 5, U(s) Y (s)
G1(s)
p = 5 and z = 1.6. + +
+ −
7. (a)  Since the diodes are ideal, anti-parallel arrange-
ment of diodes across the capacitor will create H2G2H1G1 H1G2

short circuit across C, hence the capacitor volt-
age becomes zero for all times. Thus, only induc- which can be solved as
tor contributes to the mechanism of energy storage Y (s) G1
= (1 − H1G2 )
U (s) 1 − G1H2G2 H1G1
and release. Since, the order of the system is equal
to the number of energy storage elements, the
order of the system is 1. 11. (d) The order of a network = Number of energy
storage elements − Number of CV loops − Number
8. (d) The characteristic equation of RLC series cir- of IL cut sets.
cuit is given as Here there is one IL cut set, that is, a node where three
inductors are joining with no other element. Also there
s2 + 2zw n s + w n2 = s2 +
sR 1
+ is one CV loop, that is, loop formed by C1, C2, v2 and
L LC v1. Thus, the order of the network is 5 − 1 − 1 = 3.

Chapter 6 (555-577).indd 567 3/23/2016 3:29:04 PM


568     CHAPTER 6:  CONTROL SYSTEM

12. (b) Presence of the feed forward from R(s) to Y(s) The peak overshoot is
e−zw n e−p ×0.5
does not affect the transfer function from N(s) to
Y(s). For rest of the system the loop gain is −G1H1. Mp = = = 0.24
Hence, the transfer function from N(s) to Y(s) is 1 −z 2 1 − 0.52
−G1H1
given by 16. (d) Using the Laplace transforms, the characteris-
1 + G1H1
tic equation is
13. (b) The Laplace transformed step response of the
1 1
forward path element is s2 + s + =0
2 18
1 1 1 1
G(s) = − = which has the poles at s = −1/6, −1/3 and hence
s s s + 1 s(s + 1)
characteristic equation can be written as
Therefore,
1 (s + 1/6) (s + 1/3) = 0
G(s) =   (6s + 1) (3s + 1) = 0
s+1
The last relation is in time constant form, and
Also the transfer function of the feedback ele- hence the system has 3 s and 6 s time constant.
ment is the Laplace transform of its given impulse
response 17. (b) For a first-order system, the step response
is y(t) = k(1 − e−t/t). Here k = 1 since the step
1 1 1

response settles to 1. Given that at t = 0.48, the
H(s) = =
s s + 1 s(s + 1) output is y = 0.6.

Then, the transfer function of the overall feedback 0.6 = 1 − e−.48/t


system is given as
0.48
− = ln(0.4) = −0.916
Y (s)
=
G(s)
=
s(s + 1) t
R(s) 1 + G(s)H(s) s(s + 1)2 + 1 t = 0.524

14. (b) By inspection of the error equation, we get 18. (c) The output reaches 95% means it enters the 5%
zw n = 4 and w n 1 − z 2 = 10 z = 0.37, w n = 5.714 tolerance band, for which the settling time is equal
to three times the system time constant. By inspec-
The closed-loop system is given as tion, the given system has t = 1 (time constant
t /(ts + 1) t is equal to the coefficient of s when denominator
Gcl (s) = = 2
1 + t /(ts + 1) ts + s + 1 factors are written in the form (as + 1) Therefore,
settling time is 3 times 1, that is, 3 s.
1
=
s2 + s/t + 1/t 19. (d) As t → ∞, the time response equation becomes

Comparing this with standard equation y(∞) = 2 + 0 + cos(wt + f ) + 1


s2 + 2zwns + w n2 = 3 + cos(wt + f )
we have 1/t = w n2 , which gives t = 0.03 20. (d) The input given is a ramp signal, thus

15. (c) The characteristic equation of a RLC series Kv = lim sG(s) = 1


s ®0
circuit is 1
R 1 ess = =1
s2 + s + =0 Kv
L LC
21. (c) The error transfer function for the given
Substituting given values, we get system is
100 1
s2 + s −3
+ −3
=0 æ ö
10 10 × 0.1 × 10−6 E(s) = ç 1 −
1
÷ R(s)
è (s + 0.2K )(s + t ) + 0.5 ø
Comparing it with standard equation
s2 + 2zwns + w n2 = 0 æ ö1
ess (¥) = lim s . ç 1 −
1
÷ =0
s® 0 è (s + 0.2K )(s + t ) + 0.5 ø s
we get z = 0.5 and wn = 105 rad/s.

Chapter 6 (555-577).indd 568 3/23/2016 3:29:29 PM


ANSWERS TO PRACTICE EXERCISES     569

1 26. (c) Formulating the RH array we have


0 = 1−
0.2Kt + 0.5
s3 1  9
0.2Kt = 0.5
K = 0.5/0.2 = 2.5 s2 K 18
s1 (9K − 18)/K  0
22. (d) With K = 2.5 and t = 1, the closed-loop transfer 0
s 18  0
function is
1
Gcl (s) = 2 For sustained oscillations 9K − 18 = 0, it means
s + 0.5s + 1.5 that K = 2, and then the frequency of sustained
oscillations is given by
Comparing the characteristic equation with stan-
dard dominant pole characteristic equation, we get Ks2 + 18 = 0
w n2 = 1.5, that is, wn = 1.22. Also 2zwn = 0.5, 2s2 + 18 = 0
which gives z = 0.2.   s2 + 9 = 0
      s = ± j3
23. (a) The transfer function of the closed-loop system
is given as For K = 2, the system has oscillations of 3 rad/s
1
Gcl (s) = 27. (d) The characteristic polynomial of the overall
(s + 0.2K )(s + 1) + 0.5
system is s3 + 4s2 + s + K. Formulating the RH
1 array, we have
= 2
s + (1 + 0.2K )s + 0.5 + 0.2K
The system shall have sustained oscillations if the s3 1 1
coefficient of the s term, that is, 2
s 4 K
(1 + 0.2K ) = 0 ⇒ K = − = −5
1
0.2 s1 (4 − K)/4 0
0
24. (d) The steady-state error in terms of Kp is s K 0
1
ess = For sustained oscillation, the s1 row must be all
1 + Kp
zeros, that is, K = 4.
For Kp = 100, we get ess(∞) = 1/101. The steady- 28. (c) The characteristic equation of the system is
state error can be compared as s3 + 4s2 + 6s + 4 + K = 0
æ (8 / a)(s + a) ö 1 Formulating the RH array, we have
ess (¥) = lim s. ç 1 − ÷ . = −0.5
s® 0 è (s + 1)(s + 16) ø s
s3 1 6
Irrespective of the value of a, we have constant steady- 2
s 4 4+K
state error, thus option (d) is the correct answer.
s1 (20 − K)/4 0
25. (b) The characteristic equation of the system is 0
s3 + s2 + (a + K)s + (1 + 2K) = 0 s 4+K 0
Formulating the RH array as
For sustained oscillations, 20 − K = 0, it means
s3 1 a+K that K = 20. Then forming the auxiliary equation
s2 1 1 + 2K with s2 row with the calculated value of K, we get
s1 a − K− 1 0 4s2 + 24 = 0
0
s 1 + 2K 0 s = ±j 6
1
For sustained oscillation the s row must be all
zeros, that is, a − K − 1 = 0. That is a = 1 + K. 29. (c) For the above RH array 4   +   K   >   0 and
The auxiliary equation will be s2 + (1 + 2K) = 0. 20 − K   >   0, it means that −4 < K < 20
For oscillations of 3 rad/s, the auxiliary equation 30. (a) The characteristic of the closed-loop system is
should be equal to s2 + 9 = 0, thus, 1 + 2K = 9
and hence K = 4. For this value of K, a becomes 5. s4 + 12s3 + 96s2 + 198s + 200 + K = 0

Chapter 6 (555-577).indd 569 3/23/2016 3:29:38 PM


570     CHAPTER 6:  CONTROL SYSTEM

Formulating the RH array (a) (b)

s4 1 69 200 + K
s3 12 198 0
2 (c) (d)
s 52.5 200 + K 0
s1 (7995 − 12K)/52.5 0 0
1
s 200 + K 0 0
It is clear that break-in point can only exist in case
For sustained oscillations: 7995−12K = 0, that is, of (d), and conceptually there is no need to calcu-
K = 666 late the break-in points.
However, to verify, we can calculate the break-in
31. (c) All the entries in the RH array must be posi- points as
tive for system to be stable, that is, K < 666 and
K > −200. It means that the range of K is −s(s + 1)
=0ÞK =
K(s + 2)
1+
−200 < K < 666. s(s + 1) s+2
dK (2s + 1)(s + 2) − s(s + 1)
32. (a) Root locus exists on the real axis if and only if =− =0
the total number of poles and zeros to the right of ds (s + 2)2
it are odd. Clearly, such an interval is 0 and −2. s2 + 4s + 2 = 0
Thus, option (a) is the answer. s = −3.14, 0.59
The breakaway point can be calculated from
dK/ds = 0. 35. (c) All the options have poles and zeros in the left
half of s-plane. The unity feedback configuration of
1 + KG(s) = 0 ⇒ 1 +
K option (a) will always be stable, as the pole zero
=0
s(s + 2)(s + 3) difference is 2 and asymptotes will not enter right
half of s-plane.
K = −s(s + 2)(s + 3) = −s3 − 5s2 − 6s = 0
In option (b), the pole zero difference is zero, hence
dK
= −3s2 − 10s − 6 = 0 root locus exists in between pole and zero and the
ds closed-loop system will be stable.
s = 2.66, −0.66 In option (c), the pole zero difference is 3, the angle
of asymptotes will be ±60°, −180°.
Since −2.66 point is not on the root locus; there- Thus, two branches of root locus will move towards
fore, the breakaway point is s = −0.66 which lies right half of s-plane and hence the closed-loop
between 0 and −2. system will be unstable. Thus, option (c) is the
33. (b) The characteristic equation of the overall system answer.
is s3 + 6s2 + 8s + K = 0 36. (a) All the poles and zeros have negative real parts,
Formulating the RH array we have it implies that there will be three asymptotes with
s3 1 8 angles −60°, 60°, −180°.
Adding one zero to the open-loop transfer func-
s2 6 K tion shall make pole zero difference as 2, and hence
angle of asymptotes becomes 90° and −90°. It
s1 (48 − K)/6 0 means that the root locus will not enter into the
0
s K 0 right half of s-plane and closed-loop system will be
stable. Hence we need to add one zero only.
For sustained oscillations, the s1 row must be all
zero, which can happen when K = 48. Substituting 37. (d) The characteristic equation of the system is
the value of K in the auxiliary equation formed s3 + 3s2 + 2s + K = 0
using s2 row, we get Formulating the RH array
s3 1 2
6s2 + 48 = 0
Þ s = ± j2 2 rad/s s2 3 K
s1 (6 − K)/3 0
34. (d) The sketches of the root locus of all the four 0
s K 0
transfer functions are as follows:

Chapter 6 (555-577).indd 570 3/23/2016 3:29:43 PM


ANSWERS TO PRACTICE EXERCISES     571

For the system to be stable, all the entries in the 43. (d) The given characteristic equation can be writ-
first column of the RH array must be of same sign. ten as
Thus, we need to have 6 − K > 0, that is, K < 6.
0 = s2 + (2 + K )s + 3
38. (a) The system will certainly exhibit non-oscil-
latory response if it has no complex poles. The −s2 − 2s − 3
K=
open-loop poles are real ones, that is, before the s
breakaway point the system poles will be only real.
So we need to find the breakaway point first, and dK (−2s − 2)s + s2 + 2s + 3
=
then calculate the system gain at that point. ds s2
The breakaway points can be calculated using
−s2 + 3
dK/ds = 0, and there is one breakaway point at = =0
s = −5. s2
5×5
The value of gain K at s = −5 is K = = 25. which gives s = ± 3 = ±1.732
1
39. (b) In order to calculate the angle of departure at 44. (d) The characteristic equation of the closed-loop
the pole at −1 + j10, let us sketch the pole zero system is
map, take a point in the vicinity of −1 + j 10 and
angle casted on it by all the poles and zeros is −K(s + 1)
1+ =0
−180(2k +1). s2 + 2s + 1
φdep s2 + 2s + 2
K=
s+1
+45
−135 The condition for existence of break-in point is
−2 0 dK/ds = 0, thus

+90 dK s2 + (2s + 2)(s + 1) − (s2 + 2s + 2)


=
ds (s + 1)2

45 + 135 + 90 + fdep = 180(2k + 1) =


s2 + 2s
=0
fdep = −90° (s + 1)2

40. (c) As we know that the root locus starts from the s = 0, −2
open-loop poles and ends at open-loop zeros, the The point s = −2 is not a root locus, therefore,
root locus in this case will lie between s = 0 and s = 0 is the break-in point. Hence (d) is the answer.
s = −8 as K increases from 0 to ∞. Also, the given system has negative gain param-
Hence K ∈ [0, ∞ ] eter, and hence root locus shall exist on sections of
41. (a) The characteristic equation of the system is real axis, the number of poles and zeros to the right
s2 + 2s + K = 0, thus w n = K rad/s.
of which is even. There shall be one asymptote at
angle 0 degree. The root locus plot is as shown in
Also, 2zwn = 2 implies z = 1/ K . the following figure.
To get z = 0.707 = 1/ 2 we have gain K = 2.

42. (c) The point can be calculated using RH array


s3 1 32
2
s 12 K
s1 (384 − K)/12 0 45. (b) The characteristic equation of the closed-loop
system is
s0 K 0
K(s + 1)(s + 1)
1+ =0
for K = 384, the s1 row becomes zero. Then formu- (s − 2)(s − 1)
lating the auxiliary equation we get   (s − 2)(s − 1) K(s + 1)(s + 1) = 0
12s2 + 384 = 0 which gives s = ± j 32 = ± j4 2. (1 + k)s2 + s(3K − 3) + (2 + k) = 0

Chapter 6 (555-577).indd 571 3/23/2016 3:30:00 PM


572     CHAPTER 6:  CONTROL SYSTEM

The RH array can be formulated as 50. (b) The phase margin of 78° requires us to find the
frequency corresponding to −102° that is
s2 1+K 2+K
− tan−1(w /10) = −102
180
s1 3K − 3 −0.1w ×
0 p
s0 2+K 0 The equation is approximately satisfied with
w = 10, and hence the gain at this frequency must
The system will be stable if all entries in the first be 1.
column of RH array are of same sign, that is, K ≥ 1, Ke−1 0.367
hence K ∈ (1, ∞) is the answer. =K =1
100 + 100 14.14
46. (b) The root locus starts at open-loop poles and K = 38.5
ends at open-loop zeros. In this case, root locus
starts from s = 0 and s = −2 and will end at the 51. (a) It is clear that the initial angle of the polar plot
zeros in the right half of s-plane. Thus, system will for w = 0 is −90° to pole at origin −180° due to
be stable for low gain and will become unstable (s + 4)/(s − 1) term
at high values of gain K. Thus, option (b) is the So polar plot must start from angle −270°, and
answer. The root locus of the given system is given hence option (a) is the answer.
as follows.
52. (a) One can see that the polar plot passes through
−1 + j 0 point and hence the complete Nyquist
plot shall encircle the −1 + j 0 point once in the
anticlockwise direction, that is, N = 1.
−1 0 1 2 Also there is one pole of open-loop transfer function
in right half of s-plane, that is, P = 1.
Substituting in Z = N − P, that is, Z = 1 − 1 = 0,
that is, no pole of the closed-loop transfer function
47. (d) The system is stable and does not have any lies in right half of s-plane and hence the closed-
right half-plane poles. Therefore, the net encircle- loop system is stable.
ment must be zero. So options (b) and (c) cannot The poles of closed-loop system can also be verified
be the answers. Further, the system has zero- using G(s)/(1 + G(s)).
steady-state errors for step and ramp inputs, which
53. (b) In case of asymptotic phase plots, there is no
means that system is of type 2. Type 2 system
error in phase approximation at corner frequency.
results in double rotation of Nyquist plot of the
However, asymptotic Bode gain plots have an error
small semicircular indent. Hence option (d) is the
of −3 dB at corner frequency.
correct answer.

48. (d) The system gain is 0.5/ 1 + w 2 54. (b) The DC gain in dB is 20 dB, which means that
DC gain is 10. The system function is thus given as
For s = 0, |G(s)H(s)| = 0.5 ∠G(s)H(s) = −180° G(s) = 10/(1 + s/3).
The gain at w = 1 is
0.5
For s = 1, G(s)H(s) = ∠G(s)H(s) = −135° æ ö
10 æ 3 × 10 ö
10 log10 ç
ç 1 + 1 / 9 ÷÷
2 = 20 log10 ç ÷
è ø è 10 ø
For s = ∞, G(s)H(s) = 0 ∠G(s)H(s) = −90°
= 19.54 dB
Hence, option (d) represents the correct answer.
Error in magnitude approximation is (20−19.54) =
49. (d) The given system has one unstable pole, thus 0.46 dB and phase at w = 1 is
P = 1. The net encirclement of −1 + j 0 point are
− tan−1(1 / 3) = −18.43°
N = 0. Since N = P − Z, it implies that Z = 1.
There is one closed-loop pole of the system which Since error changes at a rate of −45° per decade, it
lies in right half of s-plane. It is clear that the given means that asymptotic phase change at 1 rad/s is
system is unstable in the closed-loop.
−45 × (log− log 0.3)
The answer can be verified as follows: = 23.52°
log 3 − log 0.3
0.5 / (−1 + s) 0. 5
Gcl (s) = = Error in phase approximation is
1 + 0.5 / (−1 + s) −0.5 + s (−23.52 + 18.43) = −5.1°

Chapter 6 (555-577).indd 572 3/23/2016 3:30:14 PM


ANSWERS TO PRACTICE EXERCISES     573

55. (c) The output of a system can either be zero at: It means that where G(s) is less than 1 or 0 dB,
the Bode gain plot of the closed-loop system shall
(i) Infinite frequency if number of poles are more
be same as that of open-loop system. Hence option
than finite zeros.
(b) is the answer.
(ii) The value of frequency at which pair of zeros
occur on imaginary axis. 60. (c) Since required phase margin is 45°, means that
at 1 rad/s, system must have a phase of −145°,
Looking at the options, there occurs a pair of zeros
at ±j1, hence the gain of the system shall be zero
at 1 rad/s. tan−1(1/2K ) − 90 − tan−1(1/2) = −135
56. (b) The closed-loop transfer function is given as tan−1(1/2K ) = −90 − 26.56 + 135

s−7 tan−1(1/2K ) = 19 Þ K = 1.45


Gcl (s) =
(s + 2)(s + 3) + (s − 7)
61. (a) Options (c) and (d) can be altogether rejected.
s−7 In order to stabilise a double integrator plant, the
= 2
s + 6s − 1 corner frequency of the zero of compensator should
occur first and then that of pole. Hence option (a)
The closed-loop system has right half-plane zero is a suitable candidate for answer. Forming the
means that it is non-minimum phase system, also closed-loop with option (a), we get
it has a pole in right half of s-plane implying that
is it unstable also. s+3 s+3
Gd (s) = 2
=
57. (a) The initial slope of the Bode gain plots is s (s + 9.9) + s + 3 s + 9.9s2 + s + 3
3

−60  dB/decade of frequency. It is clear that


options (b) and (d) cannot be answers as the Bode 62. (a)
plot refers to type 3 system. Neglecting all factors
other than 1/s3 and considering gain as K, the
[sI − A] = 
s + 5 a
 −1 s 
gain at 1  rad/s should be 42 dB as given in the
figure.    
|sI − A| = s(s + 5) + a = s2 + 5s + 6
20log(K/1) = 42
      K = 126 Comparing the coefficients of the characteristic
polynomial |sI − A| with the denominator of
Thus, option (a) is the correct answer. the given transfer function s2 + 5s + 6, we get
58. (c) The given input signal is a unit acceleration a = 6.
signal. Since the given system is type 3 system, it 63. (d)
can easily follow acceleration input without error.
The acceleration error constant is  s −1 
[sI − A ] = 
126(10s + 1)(14s + 1)   4 s + 5
Ka = lim s2 =¥
s® 0 s2
és + 5 1ù
adj[sI − A ] = ê
ë −4 súû
ess = 1/Ka = 0 ,

59. (b) Since the closed-loop system transfer function sI − A = s2 + 5s + 4


is given as
s + 5 1  b1 
C adj[sI − A ]B = [2 1] 
G(s)
 −4 s b2 
Gcl (s) =
1 + G(s)
  
This implies that when G(s) is high, then the over-
all closed-loop gain is 1 or 20log(1) = 0 dB. = s(2b1 + b2) + 6b1 + 2b2e
When the system gain falls below 1 that is G(s) has
small value, then Comparing the coefficients of the above polynomial
with the numerator of the transfer function, we get
G(s) 2b1 + b2 = 1, 6b1 + 2b2 = 2 which gives b1 = 0,
Gcl (s) = = G(s)
1 + G(s) and b2 = 1

Chapter 6 (555-577).indd 573 3/23/2016 3:30:25 PM


574     CHAPTER 6:  CONTROL SYSTEM

64. (d) Converting the given transfer function into phase 68. (a)
variable canonical form, we have
é s −1 ù
[sI − A ] = ê sI − A = s(s + 3)
x1 = x2 ë0 s + 3 úû
,
−K 1 és + 3 1ù
x2 = x + f (t) adj[sI − A ] = ê
M 1 M ë 0 súû
é 0 1ù é 0 ù
x = ê
ë−K /M 0 úû
x+ê
ë1/M úû adj[sI − A ] 1 / s 1 / s(s + 3)
[sI − A ]−1 = =
sI − A  0 1 / (s + 3) 

Comparing the terms with the given state space
representation, we get Using inverse Laplace transform, we get f(t)
a = −K/M, b = 0 and g = 1/M
1 
(1 − e−3t )
1
f(t) = 
0 
65. (c) 3
 e−3t 
−1 ù
[sI − A] = éê0
s
, sI − A = s(s + 2)
ë s + 2 úû 69. (d) The response due to initial conditions as well as
és + 2 1ù
input signal is
adj[sI − A ] = ê
ë 0 súû 1
C .adj[sI − A ]B x(t) = f (t)x(0) + ∫ f (t − t )Bu(t )dt
G(s) =
sI − A 0

és + 2 1 ù é 0 ù Substituting the value of f(t), B and u(t) = 1, we


[1 0] ê
súû êë2 úû
= ë 0 get

é1 1 − e−3t ù
s(s + 2)
2 x(t) = ê ú é−1ù
G(s) = ê 3 ú êë 3 úû
êë0 e−3t úû
s(s + 2)
t é1 1 ù
2
=¥ (1 − e−3(t−t ) )ú é1 ù
Kp = lim G(s) = lim +òê dt
s® 0 s® 0 ê ú êë0 úû
s(s + 2) 3
0 ë0 e−3(t−t ) û
2
é −3 t ù é t ù é e−3t ù
= ê−e−3t ú + ê ú = êt − −
Kv = lim sG(s) = lim =1
s® 0 s ® 0 (s + 2)
ë 3e û ë û ë 3e 3t úû
0
66. (d) The characteristic equation of the system is 70. (c)
s −1 és − 1 0 ù
sI − A = = s2 + s + 1 [sI − A ] = ê sI − A = (s − 1)2
1úû
,
1 s+1
ë − 1 s −
és − 1 0 ù
adj[sI − A ] = ê
The standard characteristic equation for a second-
order system is s2 + 2zw n s + w n2 . ë 1 s − 1úû
adj[sI − A ]
We get z = 0.5 and s = zw n = 0.5 × 1 = 0.5 [sI - A ]−1 =
sI − A
é 1 / (s − 1) ù
67. (a) Stability, observability and controllability of 0
the system can be judged by any state space repre- =ê 2 1 / (s − 1)ú
sentation, and is not dependent upon any particular ë1 / (s − 1) û
form of state space representation. Controllability The inverse Laplace transform results
or observability can never be improved using nega-
é t 0ù
tive feedback.
f(t) = ê e t
et úû
Further, if a controllable canonical form is observ-
able, then the state space representation is minimal ëte
é t 0 ù é 1 ù é et ù
and there are no cancellation of poles and zeros in x(t) = f(t)x(0) = ê e t
et úû êë0 úû êëtet úû
=
respective transfer function. ëte

Chapter 6 (555-577).indd 574 3/23/2016 3:30:40 PM


ANSWERS TO PRACTICE EXERCISES     575

71. (c) The state transition matrix f(t) can be calcu- Applying unity negative feedback around G(s), we
lated as get the closed-loop system Gcl(s) as

 −3 t 0  1 / (s2 + 3s + 2)
f(t) = e
G(s)
−t  Gcl (s) = =
 0 e  1 + G(s) 1 + 1 / (s2 + 3s + 2)
1
Then the state after two seconds is =
s2 + 3s + 3
é −6 0 ù é3 ù é3e −6 ù
x(2) = f(2)x(0) = êe
e−2 úû êë 4 úû êë 4e−2 úû
=
ë 0 The poles of the closed-loop system are the roots of
s2 + 3s + 3, that is, −1.5 ± 0.866.
72. (d) The given form is a parallel form. x2 is a stable 75. (a) The controllability matrix Uc = [B AB] as well
state due to negative feedback. However, in case as the observability matrix
if the value of m becomes more than 2, then the
feedback around state x1 shall become positive and Uo = [CT ATCT], both have rank 2, and therefore
system will become unstable. Thus, m > 2. the system is controllable as well as observable.
Since observability is not in question, that means However, the |sI − A| = s2 − 6s + 8 has roots in
that any value of k, l will serve the purpose and are the right half of s-plane and therefore the given
not the deciding factors. system is unstable.
Carefully inspecting the input connection of 76. (b) In controllable canonical form, the last row
states, it is clear that x1 will become uncontrol- of the system matrix is constructed using the
lable if j = −3, and x2 will become uncontrollable if negative of the coefficients of the characteristic
i = −2. Thus, for the system to be uncontrollable polynomial in reverse order. The characteristic
either j = −3 or i = −2 or both. polynomial for the given differential equation is
Thus, option (d) is the answer, as it makes x2 s3 + 3s2 + 2s + 1; therefore, the coefficients of the
uncontrollable and system unstable with m = 4. last row of A matrix must be −1 −2 −3, and hence
73. (c) The dynamic relations between the states option (b) is the correct answer.
are  x1 = x2 , x2 = x3 and x3 = −2x2 − x3 + u
and hence state space representation in phase vari- 77. (a) As the bridge is balanced, there appears no volt-
able canonical form is age across C2, hence vC2 state is not controllable.
Further, even if C2 has any initial voltage across
0 1 0 0  it, it can never appear/affect the output, as bridge
x = 0 0 1  x + 0  u is balanced.
   
0 −2 −1
Thus, the given network although stable, is neither
1 
controllable nor observable.
|sI − A| = s(s2 + s + 2), and the roots of the equa- 78. (c) The state space representation of the system can
tion are at s = 0, −0.5 ± 7 /2. Since, one of the be obtained via inspection of the state diagram as
root is real and zero, and other two roots are com-
plex conjugates, the normalised form of the Jordan x1 = x2 x2 = 2x1 − x2 + u
system matrix is é0 1 ù é0 ù
x = ê ú x+ ê úu
ë − û ë1 û
0 ù é 0. 5 7 /2 0 ù
2 1
és w
ê−w s ê 0. 5 0 ú
0 ú = ê− 7 /2 ú The eigenvalues of the system matrix are −1, +2,
ê ú
ë 0 0 l2û ê 0 0 0ú
ë û and hence the system is unstable. The controllabil-
ity matrix is

é0 1 ù
74. (d) The transfer function corresponding to the
U c = [B AB ] = ê
ë1 −1úû
given state space representation is

és + 3 1ù é 0 ù
[1 0] ê
G(s) =
C .adj(sI − A)B
= ë −2 súû êë1 úû which has rank 2. So, the given system is con-
sI − A s(s + 3) + 2 trollable. Also, the rank of the observability
matrix is 2, and hence system is observable. So
1
G(s) = the system is unstable but controllable as well as
s2 + 3s + 2 observable.

Chapter 6 (555-577).indd 575 3/23/2016 3:30:54 PM


576        CHAPTER 6:  CONTROL SYSTEM

SOLVED GATE PREVIOUS YEARS’ QUESTIONS

1. A control system is defined by the following math- For such a lead compensator
ematical relationship (a) a < b (b) b < a
(c) a > Kb (d) a < Kb
d2x
+ 5x = 12(1 − e−2t)
dx
2
+6 (GATE 2003: 1 Mark)
dt dt
The response of the system as t → ∞ is
Solution:  Given that lead compensator transfer
(a) x = 6 (b) x = 2
function is
(c) x = 2.4 (d) x = -2
K(1 + s/a)
(GATE 2003: 1 Mark) (1 + s/b)
Main important criterion for a lead compensation
is that phase qn should be positive.
Solution:  Given
d2x
+ 5x = 12(1 − e−2t ) 
dx (i) Substitute s = jw.
2
+6 (i)
dt
K(1 + jw /a)
dt
H( jw ) =
By final value theorem, (1 + jw /b)
lim f (t) = lim sF (s) (ii) Find the phase angle for the given transfer
t →∞ s→ 0
function
RHS:
tan−1(w /a)
(i) Convert Eq. (i) into frequency domain by q n = ∠H( jw ) =
applying Laplace transform. tan−1(w /b)
(ii) Then apply the limit s → 0 to find f(t)|t → ∞
(LHS) Note that

Taking LT on both sides a + jb


1 1  b
s2 X(s) + 6sX(s) + 5X(s) = 12  −  f = tan−1  
 s s + 2   a 
 s + 2 − s  a−b 
X(s)[s2 + 6s + 5] = 12   tan−1 a − tan−1 b = tan−1  
 s(s + 2)   1 + ab 
 
24 Therefore,
X(s) =
s(s + 2)(s2 + 6s + 5)
q n = tan−1(w /a) − tan−1(w /b)
As LT(e−at ) = w w 
1
 − 
s+a  b 
On applying the limit s → 0 = tan−1  a
 2
1 + w 
 
lim sX(s) = s 
24  = 24  ab 
s→ 0   wb − wa 
 s(s + 2)(s + 6s + 5)  (2)(5)
2
 
−1  
24 = tan  ab 
= = 2.4  ab + w 
2
10  
Ans. (c)  ab 
2. A lead compensator used for a closed-loop control-
 w (b − a) 
ler has the following transfer function q n = tan−1   > 0 [should be greater than
 ab + w 2 
 s  
K 1 + 
 a zero for lead compensation]
 
1 + 
s Therefore, b > a ⇒ a < b
 b  Ans. (a)

Chapter 6 (578-614).indd 576 3/23/2016 4:01:17 PM


SOLVED GATE PREVIOUS YEARS’ QUESTIONS        577

3. A second-order system starts with an initial To find the natural time constants, we need to
 2 (i) Find the response of the system.
condition of   without any external input. The
 3 (ii) Determine the pole nearest to the imaginary
  axis.
state transition matrix for the system is given by
e−2t 0
Taking Laplace transform of Eq. (i) on both the
 sides, we get
 0 e−t  . The state of the system at the end of
  1 1 10 5 2
1 s is given by s2 X(s) + sX(s) + X(s) = + +
2 18 s s+4 s+5
 0.271  0.135  1  10(s + 5)(s + 4) + 5s(s + 5) + 2s(s + 4)
(a)   (b)  
s
X(s) s2 + +  =
1.100  0.368

     2 18  s(s + 4)(s + 5)

 0.271  0.135
10(s + 5)(s + 4) + 5s(s + 5) + 2s(s + 4)
(c)   (d)  
X(s) =
 1  1
 0.736 1.100  s(s + 4)(s + 5) s +  s + 

     3  6
(GATE 2003: 1 Mark)
Pole plot jw
Solution:  Given
e−2t 0 
State transition matrix, f(t) = 
 0 e−t  s
5 4 3 2 1
 2
Initial condition X(0) =  
3
We have to find X(t) at t = 1, without any external
input
So, Therefore, poles nearest to the imaginary axis are

X(t) = f (t)X(0) −1 1
s1 = s2 = −
e−2t 0  2 2e−2t 
3 6
=   = 
e−t  3 3e−t 
Therefore, natural time constants are
 0
T1 = 3 s and T2 = 6 s
At t = 1 Ans. (b)
2e−2   0.271
=  −1  = 
Common Data for Questions 5 and 6: The
X(t) 
 3e  1.100
t=1 block diagram shown in the following figure gives a
unity feedback closed-loop control system.
Ans. (a)
u(t) + 3 15 y(t)
4. A control system with certain excitation is gov- s+15 s+1
erned by the following mathematical equation

d2x
+ x = 10 + 5e−4t + 2e−5t
1 dx 1 5. The steady-state error in the response of the above
2
+
dt 2 dt 18 system to unit step input is
The natural time constants of the response of the (a) 25%   (b) 0.75%   (c) 6%   (d) 33%
system are (GATE 2003: 2 Marks)
(a) 2 s and 5 s (b) 3 s and 6 s
(c) 4 s and 5 s (d) l/3 s and l/6 s
Solution:  Given that:
(GATE 2003: 2 Marks) Unity feedback system H(s) = 1
3
Solution:  Given: Open-loop transfer function G1(s) =
s + 15
d2x
+ x = 10 + 5e−4t + 2e−5 t
1 dx 1 15
2
+ ⋅ (i) G2 (s) =
dt 2 dt 18 s+1

Chapter 6 (578-614).indd 577 3/23/2016 4:01:41 PM


578        CHAPTER 6:  CONTROL SYSTEM

Steady-state error, 7. The following equation defines a separately excited


DC motor in the form of a differential equation
sR(s)
ess = lim
s→0 1 + G(s)H (s)
d 2w B dw K2
w =
K
+ + V
1 dt2 J dt LJ LJ a
R(s) = (unit step input)
s
G(s) = G1(s)G2 (s) [series combination]] The above equation may be organised in the state-
space form as follows
 3   15 
=   45
 s + 15   s + 1 (s + 1)(s + 15)
=
 d 2w   
   
s ⋅ 1/s  2  dw 
 dt =P  
ess = lim
s→ 0  dw   dt  + QVa
45
   
1+
 dt  w 
(s + 1)(s + 15)  
1 1 1
= = = = 0.25
45 1+3 4 where P matrix is given by
1+
15
 B 2  K2 B 
%ess = 0.25 × 100 = 25% − − K  − −
(a)  J LJ  (b)  LJ J 
Ans. (a)  
 1 0  0 1
6. The roots of the closed-loop characteristic equation
of the system are  0 1  1 0
   
(a) -l and −15 (c)  K 2
B  (d)  B 2 
(b) 6 and 10 − − − − K 
(c) −4 and −15 (d) −6 and −10  LJ J   J LJ 
(GATE 2003: 2 Marks)  (GATE 2003: 2 Marks)

Solution:  To find the roots of the closed-loop Solution:  Given that the differential equation of a
transfer function, we need to separately excited DC motor is
Y (s)
(i) Find the closed-loop transfer function = . d 2w B dw K2
w =
K
U (s) + + V
dt2 J dt LJ LJ a
(ii) Equate the denominator (characteristic equa-
tion) to find the roots.
State space matrix is
Y (s) G(s)
=  d 2w 
U (s) 1 + G(s)H(s)    dw 
 2  
45  dt =P  dt  + QVa
 dw  w 
(s + 1)(s + 15)    
=
45  dt 
1+
(s + 1)(s + 15)
We first assign the state variables,
45 Let
=
(s + 1)(s + 15) + 45
dw d 2w
Therefore, the characteristic equation is = x1 , w = x2 , = x1
dt dt2
(s + 1)(s + 15) + 45 = 0
2 Therefore, state equation becomes,
s + 15s + s + 15 + 45 = 0
s2 + 16s + 60 = 0 B K2 K
x1 + x1 + x2 = V
(s + 6)(s + 10) = 0 J LJ LJ a
s = −6, −10
K B K2
x1 = Va − x1 − x
Ans. (d) LJ J LJ 2 (i)

Chapter 6 (578-614).indd 578 3/23/2016 4:02:01 PM


SOLVED GATE PREVIOUS YEARS’ QUESTIONS        579

From the state variable assigned, For the system to be unstable, element in the first
column should be negative
x2 = w
dw  48 − K 
 ≤0
x2 =
dt
= x1  6 
x2 = x1  (ii) 48 − K = 0

−K = −48 ⇒ K = 48
Substituting coefficients of x1 and x2 and coeffi-
cient of Va from Eqs. (i) and (ii) in the given state  Ans. (c)
matrix, we get 9. The asymptotic Bode plot of the transfer function
 2 K
 x1   −B −K   x1 
K
  is given in the following figure.
 = J   +  LJ  Va s
 x2   LJ   x2    1+
 1 0   0  a
The error in phase angle and dB gain at a frequency
 −B −K 2  of w = 0.5 a are respectively

P matrix =  J LJ 

 1 0  |G(jw)| dB
Ans. (a) 20 log K 20 dB/decade
8. The loop gain GH of a closed-loop system is given
K
w
by the expression
s(s + 2)(s + 4) a
The value of K for which the system just becomes
unstable is
(a) K = 6 (b) K = 8
(c) K = 48 (d) K = 96 0.1a 10a w

(GATE 2003: 2 Marks) 45°/decade


Ph°
Solution:  Given that the loop gain transfer
function
K (a) 4.9°, 0.97 dB (b) 5.7°, 3 dB
G(s) H(s) =
s(s + 2)(s + 4)
(c) 4.9°, 3 dB (d) 5.7°, 0.97 dB
To find K when system becomes unstable, we need to
(i) Find the characteristic equation (GATE 2003: 2 Marks)
1 + G(s) H(s) = 0 Solution:  Given that transfer function
K K
1+ =0
s(s + 2)(s + 4) s
1+
s(s + 2)(s + 4) + K = 0 a

s(s2 + 6s + 8) + K = 0 The maximum error occurs at corner frequency


between exact and asymptotic plot.
s3 + 6s2 + 8s + K = 0 For exact plot:
(ii) From the RH array (i) Gain (dB) w =0.5a

s3 1 8 w2
= 20 log K − 20 log 1 +
s2 6 K a2
s1 (0.5a)2
48 − K = 20 log K − 20 log 1 +
a2
6
= 20 log K − 20 log 1 + 0.52
s0 K
= 20 log K − 0.96

Chapter 6 (578-614).indd 579 3/23/2016 4:02:15 PM


580        CHAPTER 6:  CONTROL SYSTEM

(ii) Phase angle at w = 0.5a Eliminating the feedback path 1


w 
f = − tan−1   1
 a 
+
− s
 0.5a 
= − tan−1  = −26.56°
 a  3
1 1
For asymptotic plot: s 1
= = s =
1 s+3
1+ ⋅3 s + 3
Gain (dB) w = 0.5a = 20 log K s s

Phase angle at w = 0.5a Eliminating the feedback path 2.


f = −22.5°
+ 1
Error in gain, 20 log k - (20 log k - 0.96) = +0.96 dB. − s
Error in phase angle = -225 - (-26.56) = 4.9°
Ans. (a) 12
10. The block diagram of a control system is shown in 1
the following figure. The transfer function G(s) = 1
= s =
Y(s)/U(s) of the system is 1
1 + ⋅ 12 s + 12
s
9
Therefore the block diagram is reduced as,
U(s) − Y(s)
Integrator 2 Integrator 9
+ − +

3 12 U(s) − Y(s)
+ 1 2 1
s+3 s + 12
1
(a) 
 s  s
18 1 +  1 +  Eliminating the series block
 12   3
1 9
(b) 
  s
27 1 +  1 + 
s
U(s) − Y(s)
 6  9 +
2
(s+3)(s+12)
1
(c) 
 s  s
27 1 +  1 +  Eliminating feedback path, we get
 12   9

1 2
(d) 
 s  s
27 1 +  1 + 
Y (s) (s + 3)(s + 12) 2
=
 9  3
=
U (s) 1 + 2 (s + 3 )(s + 12) + 18
9
(s + 3)(s + 12)
(GATE 2003: 2 Marks)
2 2
= 2 = 2
Solution:  The given block diagram can be redrawn s + 15s + 36 + 18 s + 15s + 54
as 2 2
= =
(s + 9)(s + 6) 9 × 6  s  s
9 1 +  1 + 
9 6
U(s) Y(s) 1
1 1 =
+
− 2 +
−   s
27 1 +  1 + 
s s s
3 12  9  6
1 2
Ans. (b)

Chapter 6 (578-614).indd 580 3/23/2016 4:02:34 PM


SOLVED GATE PREVIOUS YEARS’ QUESTIONS        581

11. The Nyquist plot of loop transfer function G(s) 13. For a tachometer, if q(t) is the rotor displacement
H(s) of a closed-loop control system passes through in radians, e(t) is the output voltage and Kt is the
the point (−1, j 0) in the G(s) H(s) plane. The tachometer constant in V/rad/s, then the transfer
phase margin of the system is E(s)
function, will be
(a) 0°      (b) 45°      (c) 90°      (d) 180° Θ(s)
(GATE 2004: 1 Mark) K
(a) Kts2    (b)  t     (c) Kts    (d) Kt
s
Solution:  Given that G(s) H(s) passes through (GATE 2004: 1 Mark)
(-1 + j 0) point.
We know that the phase margin is the amount of
Solution:  Given:
q(t) is the rotor displacement in radians.
additional phase log required to bring the system
to verge of instability.
e(t) is the output voltage.
Therefore, phase margin of the system is 0°
Kt is the tachometer constant in V rad/s.
Ans. (a)
In AC tachometer,
5
12. Consider the function, F (s) = 2
where e(t) ∝ w (t)
s(s + 3s + 2)
dq (t)
F(s) is Laplace transform of the function f(t). The e(t) ∝ (as w = dq /dt)
dt
initial value of f(t) is equal to
dq (t)
5 5 e(t) = Kt
(a) 5   (b)  (c)  (d) 0 dt
   2       3     
Taking Laplace transform on both sides,
(GATE 2004: 1 Mark)
E(s) = Kt ⋅ sQ(s)
Solution:  Given: E(s)
= sKt
Q(s)
5
F (s) = 2
s(s + 3s + 2) Ans. (c)

f (t) ←
→ F (s) 14. For the equation, s3 − 4s2 + s + 6 = 0 the number
LT
Inverse LT
of roots in the left half of s-plane will be
To find initial value of f(t) at t → 0, we convert (a) zero (b) one
the given function F(s) to f(t) by inverse Laplace (c) two (d) three
transform.
(GATE 2004: 2 Marks)
By partial fraction method,
Solution:  Given:
5 A B C
2
= + + s3 - 4s2 + s + 6 = 0
s(s + 3s + 2) s s+1 s+ 2
5 = A(s + 1)(s + 2) + Bs(s + 2) + Cs(s + 1) By Routh-Hurwitz criterion,
s3 1 1
s = 0; 5 = A(2) A = 2.5
s = 1; 5 = B(−1)(−1 + 2) B = −5 s2 -4 6
1
s = −2; 5 = C(−2)(−2 + 1) C = 2.5 s
−4 + −6
Therefore, −4
s1 2.5
2.5 5 2.5
F (s) = − + s0 6
s s+1 s+ 2
Taking inverse LT There are two changes of sign from s3 to s2 and s2
to s1. Therefore, number of poles lying on right half
f(t) = 2.5u(t) - 5e-t + 2.5e-2t of s-plane = 2
Number of poles lying on left half of s-plane is
lim f (t) = 2.5 − 5 + 2.5 = 0 Total number of poles - Right half poles = 3 - 2 = 1
t→ 0
Ans. (d) Ans. (b)

Chapter 6 (578-614).indd 581 3/23/2016 4:02:49 PM


582        CHAPTER 6:  CONTROL SYSTEM

15. For the block diagram shown in the following 16. The state variable description of a linear auton-
C(s) omous system is X = AX where X is a two-­
figure, the transfer function is equal to dimensional state vector and A is the system
R(s)
matrix given by
 0 2
R(s)
1 + 1 + C(s) A= 
s s  2 0
.
+ +
The roots of the characteristic equation are
(a) −2 and +2 (b) −j 2 and +j 2
s2 + 1 s2 + s + 1 (c) -2 and −2 (d) +2 and +2
(a)  2 (b) 
s s2 (GATE 2004: 2 Marks)
1 2
s + s+1 Solution:  Given that:
(c)  2 (d) 
s + s+1 s X is a two-dimensional state vector
 0 2
(GATE 2004: 2 Marks) A is the system matrix =  
 2 0
Solution:  From the given block diagram State variable, X = AX
The characteristic equation is given by,
1 2
R(s) C(s) sI − A = 0
1 1
+ +  1 0  0 2
sI − A = s  − 
s + s +
 0 1  2 0
 
=  s −2 
−2 S 
For 1: The output is
sI − A = s2 − 4 = 0
1.
s R(s) s2 = 4 ⇒ s = ±2
R(s)
1 + Ans. (a)
s +
17. The block diagram of a closed-loop control system
R(s)
is given in the following figure. The values of K and
1 P such that the system has a damping ratio of 0.7
s
R(s) + R(s) and an undamped natural frequency wn of 5 rad/s,
 1
are respectively equal to
= R(s) 1 + 
 s  R(s) + K C(s)
s(s+2)
For 2: The output is −
1+sP
1 R(s) (1+ 1 )[
s [ s (a) 20 and 0.3 (b) 20 and 0.2
R(s)(1+ 1s )
1 + (c) 25 and 0.3 (d) 25 and 0.2
s +
(GATE 2004: 2 Marks)
R(s)
Solution:  Given that:
Damping ratio z = 0.7
1  1 Undamped natural frequency wn = 5 rad/s.
C(s) = R(s) ⋅ ⋅ 1 +  + R(s)
s 
We need to obtain the closed-loop transfer function
s
 1 1
K
C(s) = R(s) 1 + + 2 
 s s 
C(s) s(s + 2)
=
R(s) 1 + K
C(s) s2 + s + 1 (1 + sP )
= s(s + 2)
R(s) s2 K
=
Ans. (b) s(s + 2) + K(1 + sP )

Chapter 6 (578-614).indd 582 3/23/2016 4:03:07 PM


SOLVED GATE PREVIOUS YEARS’ QUESTIONS        583

Characteristic equation is
t→∞
s(s + 2) + K(1 + sP ) = 0
s2 + 2s + K + KsP = 0 
s2 + s(2 + KP ) + K = 0 (i)

By final value theorem,
In general characteristic equation of a second-order
system is given by, lim y(t) = lim s ⋅ Y (s)
t →∞ s→ 0
s2
+ 2zw n + w n2 = 0 (ii) = lim s ⋅ H(s)R(s)
s→ 0
100 1
Comparing Eqs. (i) and (ii), we get = lim s ⋅ ⋅
s→ 0 2
s + 125 + 100 s
w n2 = K ⇒ w n = K (∵ R(s) = 1/s; unit step input)
5 = K ⇒ K = 25 =1
Similarly Ans. (d)
2zw n = 2 + KP 19. In the system shown in the following figure, the
2 × 0.7 × 5 = 2 + 25P input is x(t) = sin wt, In the steady-state, the
7 − 2 = 25P ⇒ P = 0.2 response y(t) will be
Ans. (d)
x(t) s y(t)
18. The unit impulse response of a second-order under- s +1
damped system starting from the rest is given by
c(t) = 12.5e− et sin 8t, t≥0 1 1
(a)  sin(t − 45°) (b)  sin(t + 45°)
The steady-state value of the unit step response of 2 2

(c) sin(t - 45°)


the system is equal to
(d) sin (t + 45°)
(a) 0    (b) 0.25    (c) 0.5    (d) 1.0
(GATE 2004: 2 Marks)
(GATE 2004: 2 Marks)
Solution:  For the given system
Solution:  Given that:
x(t) → sinwt
c(t) = 12.5e−6 t sin 8t, t ≥ 0
Y (s) s
Transfer function = H(s) =
where c(t) is the unit impulse response of the sec- X(s) s+1
ond-order system.
Transfer function Substituting s = jw, we get

12.5 × 8 jw
H(s) = LT[c(t)] = H( jw ) =
2
(s + 6) + 8 2 jw + 1
Magnitude of H (jw),
LT (e−at sin bt) =
b
as
(s + a)2 + b2 w
H( jw ) =
Therefore, w2 +1

H(s) =
100 (∵ a + jb = R ⇒ R = a2 + b2 )
2
s + 36 + 12s + 64
100 From the data given w = 1 rad/s.
=
s2 + 12s + 100 H( jw ) w =1 =
1
=
1
1+1 2
Steady-state value is reached as time t tends to
infinity. Phase of H (jw)

Chapter 6 (578-614).indd 583 3/23/2016 4:03:28 PM


584        CHAPTER 6:  CONTROL SYSTEM

∠H( jw ) = 90° − tan−1(w /1)


Squaring both sides, we get

∠H( jw ) 90° − tan−1(1) w gc 4 = (w gc a) + 1


2
w =1=
= 90° − 45°
w gc 4 − w gc
2 2
a −1 = 0  (ii)
= 45°
As, phase margin = 180° + ∠G( jw gc )
Therefore, output of the system = H(jw ) x[t − H(jw )]

=
1
sin(t − 45°) tan−1(w gc a) − 180° + 180° = 45°
2
tan−1(w gc a) = 45° ⇒ w gc a = 1

Ans. (a)
20. The open-loop transfer function of a unity feedback Substituting in Eq. (ii), we get
as + 1
control system is given as G(s) = .
s2 (1/a)4 − 1 − 1 = 0 ⇒ (1/a)4 = 2
The value of `a’ to give a phase margin of 45° is
equal to a 4 = 0. 5
(a) 0.141   (b) 0.441   (c) 0.841   (d) 1.141 a = 0.841
(GATE 2004: 2 Marks) Ans. (c)
21. A system with zero initial conditions has the
Solution:  Given that: closed-loop transfer function
as + 1
G(s) = s2 + 4
s2 T (s) =
(s + 1)(s + 4)
Phase margin = 45°
Phase margin = 180° + ∠G( jw gc ) The system output is zero at the frequency
(a) 0.5 rad/s (b) 1 rad/s
45° = 180° + ∠G( jw gc ) (c) 2 rad/s (d) 4 rad/s
where wgc is the gain crossover frequency. It is the
(GATE 2005: 1 Mark)
frequency at which the magnitude of open-loop
transfer function is unity or dB magnitude is zero.
Solution:  Given
Therefore,
G( jw gc ) = 1 s2 + 4
T (s) =
(s + 1)(s + 4)
To find G( jw gc ) , substitute s = jw in the given
We have to find the frequency at which the system
transfer function. We get
output is zero.
jw a + 1
G( jw ) =
Substituting s = jw in the transfer function, we get
jw 2
( jw )2 + 4
Using w = wgc T ( jw ) =
( jw + 1)( jw + 4)
jw gc a + 1
G( jw gc ) =  (i) The system output is zero, so
jw gc
2

Since −w 2 + 4
T ( jw ) = =0
jw + 1 jw + 4
G( jw gc ) = 1 and ∠G( jw gc ) = 180° − 45°
this implies
Equation (i) becomes
4 − w2 = 0 w2 = 4 w = 2 rad/ s
(w gca) + 1
2

1= Ans. (c)
−w gc2
22. The following figure shows the root locus plot
(w gca)
2
−w gc =
2 (location of poles not given) of a third order system
+1
whose open-loop transfer function is

Chapter 6 (578-614).indd 584 3/23/2016 4:03:54 PM


SOLVED GATE PREVIOUS YEARS’ QUESTIONS        585

Im We have:
2√3 1
Gain margin (GM) =
√3 G( jw pc )

−3 −2 −1 1 2 3 Re 1
−√3 or, GM in dB = 20 log
G( jw pc )
−2√3
where wpc is the phase cross over frequency. It
K K is the frequency at which the phase of open-loop
(a)  (b)  2
s3 s (s + 1) transfer function is 180°, that is,

K K ∠G( jw pc ) = −180°
(c)  2 (d)  2
s(s + 1) s(s − 1)
Substituting s = jw in the transfer function, we get
(GATE 2005: 1 Mark)
s +1
G(s) =
Solution:  For a third-order system, highest power s2
of s = 3 jw + 1 jw + 1
G( jw ) = =
Number of poles = 3 ( jw ) 2
−w 2
±180°(29 + 1)
Asymptote:
n−m w2 +1
G( jw ) = (i)
w2
where n is the number of poles and m is the number
of zeros. Also,
Centroid is the point of intersection of asymptote
on real axis. ∠G( jw ) = tan−1 w
Sum of poles − Sum of zeros
Centroid = For w = wpc, we have
No. of poles − No. of zeros
From the given options: ∠G( jw pc ) = −180°

K 0−0 tan−1 w pc = −180°


(a) ⇒ =0
s3 3−0 w pc = 0
K −1 − 0
(b) ⇒ = −1/3 1
2
s (s + 1) 3−0 Gain margin =
G(Jw pc )
K 0+ j−0
(c) ⇒ = j/3
s(s2 + 1) 3−0 From Eq. (i), we get

K 0−j−0 w pc
2
⇒ = −j/3
1 1
(d) = = =0
s(s2 + 1) 3−0 G(Jw pc ) w pc
2
+1 1 + w pc
2

Therefore, solution option (a) is correct. w pc


2
Ans. (a)
23. The gain margin of a unity feedback control system Ans. (a)
(s + 1) 24. A unity feedback system, having an open-loop gain
with the open-loop transfer function G(s) =
is s2 K(1 − s)
G(s)H(s) = , becomes stable when
1 (1 + s)
(a) 0     (b)       (c)  2     (d) ∞
2 (a) |K| > 1
(GATE 2005: 2 Marks) (b) K > 1
(c) |K| < 1
Solution:  Given the transfer function:
(d) K < − 1
s+1
G(s) =
s2 (GATE 2005: 2 Marks)

Chapter 6 (578-614).indd 585 3/23/2016 4:04:23 PM


586        CHAPTER 6:  CONTROL SYSTEM

Solution:  Given ess = lim s . E(s)


s→0
K(1 − s)
G(s)H(s) = From the block diagram given,
(1 + s)
E(s) = R(s) -Y(s) (i)
Stability the given system can be analysed using
Routh-Hurwitz criterion. Y(s) can be written as
(i) The characteristic equation for the system is   2
Y (s) = [R(s) − Y (s)] − [R(s)]
3
given by  s  s + 2
1 + G(s)H(s) = 0 6 6 2
Y (s) = ⋅ R(s) − Y (s) − R(s)
K(1 − s) s(s + 2) s(s + 2) (s + 2)
1+ =0
1+s  6   6 2 
(1 + s) + K(1 − s) = 0 Y (s) 1 + = R(s)  −
 s(s + 2)   s(s + 2) (s + 2) 
  
1 + s + K − Ks = 0
 s(s + 2) + 6   6 − 2s 
s(1 − K ) + 1 + K = 0 Y (s)   = R(s)  
 s(s + 2)   s(s + 2) 
   
 6 − 2s 
s1 1-K Y (s) = R(s)  2 
 s + 2s + 6 
(ii)
 
s0 1+K
From Eqs. (i) and (ii), we have
For a system to be stable, s1 and s0 should be positive.  6 − 2s 
E(s) = R(s) − R(s)  2 
Therefore,  s + 2s + 6 
 
1− K > 0  s2 + 4s 
1+ K > 0 = R(s)  2 

 s + 2s + 6 
⇒ −1 < K < 1
or, K < 1 For unit step, R(s) = 1s

Ans. (c) Therefore, steady state error is


ess = lim s ⋅ E(s)
25. When subjected to a unit step input, the closed- s→ 0
loop control system shown in the following figure
1 (s2 + 4s)
will have a steady-state error of = lim s ⋅ ⋅ 2
s→ 0 s (s + 2s + 6)
=0

+
R(s) 2 Y(s)
3/s Ans. (c)
s+2
− +
26. In the GH(s) plane, the Nyquist plot of the loop
transfer function
(a) −1.0    (b) −0.5    (c) 0    (d) 0.5 pe−0.25s
G(s)H(s) =
(GATE 2005: 2 Marks) s
passes through the negative real axis at the point
(a) (−0.25, j 0) (b) (−0.5, j 0)
Solution:  The given control system can be repre-

(c) (−1, j 0) (d) (−2, j 0)
sented as

(GATE 2005: 2 Marks)
E(s) R(s)
− 2 Y(s)
+ 3/s + Solution:  Given

pe−0.25
R(s) s+2
G(s)H(s) =
s
To find the point where the Nyquist plot of
Steady-state error is given by,
G(s) H(s) passes through negative real axis:

Chapter 6 (578-614).indd 586 3/23/2016 4:04:37 PM


SOLVED GATE PREVIOUS YEARS’ QUESTIONS        587

−270°  1 
G(s)H(s) = (K + 0.3665)  
 s(s + 1)
−ve real axis

−180°
Substituting s = jw in the transfer function, we get
 
G( jw )H( jw ) = (K + 0.366 jw )    (ii)
1
−90°
 jw ( jw + 1)

When the Nyquist plot passes through negative Putting w = wgc, we get
real axis, G( jw gc )H( jw gc )
∠G( jw )H( jw ) = −180° K + 0.366 jw gc
=
jw gc ( jw gc + 1)
Substituting s = jw in G(s)H(s), we get
 0.366w gc 
= tan−1   − 90° − tan−1(w gc )
−0.25s
pe  K 
G( jw )H( jw ) =
jw
Substituting for G(jwgc)H(jwgc) from Eq. (ii) in
∠G( jw )H( jw ) = −180° Eq. (i) and wgc = 1, we get
−0.25 jw − 90° = −180°  0.366 w gc 
−0.25 jw = −90° Phase margin = 180° + tan−1   − 90° − tan−1 w gc
 K 
−90°
jw = = 360°  0.366 w gc 
−0.25 60° = 180° + tan−1   − 90° − tan−1 w gc
 K 
Therefore, s = jw = 360° = 2p
 0.366 
= 90° + tan−1  − tan−1(1)
 K 
Therefore,

pe−0.25×2p  0.366 
= −0.5
= 45° + tan−1  [∵ tan−1(1) = 45]
G(s)H(s) =
2p  K 
Therefore, the plot passes through the point
 0.366 
tan−1  = 15°
 K 
(-0.5, j 0)
Ans. (b)
0.366
27. If the compensated system shown in the following = tan 15°
figure has a phase margin of 60° at the crossover K
frequency of 1 rad/s, then value of the gain K is 0.366
K = = 1.366
0.267
R(s) + 1 Y(s)
K+0.366s Ans. (c)
s(s+1)
− Statement for Linked Answer Questions 28
and 29: A state variable system
0 1 1 
X (t) =   X(t) +   u(t),
(a) 0.366   (b) 0.732   (c) 1.366   (d) 2.738
 0 −3  0
 
(GATE 2005: 2 Marks)
with initial condition X(0) = [-1 3]T and the unit
step input u(t) has
Solution:  Given that phase margin = 60°, so
28. The state transition matrix
Phase margin = 180° + ∠G( jw gc )
 
Therefore gain crossover frequency wgc = 1rad/s. 1
(a) 
1
3
(
1 − e−3 t 

)
 
For loop transfer function,  0 e−3 t 
Phase margin = 180° + ∠G( jw gc )H( jw gc )  (i)  

We can determine ∠G( jw gc )H( jw gc ) after elimi-
1
(b)  3
(
1 −t
e − e−3 t 

)
 
nating the series combination blocks, as follows  0 e− t 

Chapter 6 (578-614).indd 587 3/23/2016 4:04:55 PM


588        CHAPTER 6:  CONTROL SYSTEM

    1 1 −3 t 
(c) 
1
3
(
1 −t
e − e−3 t 

) −1
Therefore, L (sI − A) −1
1
=
 − e 
 3 3 
   
 0 e−3 t  0 e−3 t 
  

(d) 
(
 1 1 − e− t 


) 
1
1
(
1 − e−3 t

)
0 −t = 3
 e   
 0 e−3 t 
(GATE 2005: 2 Marks)
Ans. (a)
Solution:  Given that:
29. The state transition equation
0 1   1
X (t) =   X(t) +   u(t) (i)
 0 −3  0  t − e− t   t − e− t 

(a) X(t) =    
−1 e− t   3e−3 t 
X
(b)  ( t) =
X(0) =   (ii)  
 
 3     

 t − e−3t   t − e−3 t 
(c) X(t) =    
For unit step input u(t),
−3 t  (d) X (t) =  e− t 
1  3e  
U (s) =
s (GATE 2005: 2 Marks)
State transition matrix,
−1
f(t) = L−1 [sI − A ]
Solution:  State transition equation,
 (iii)

State equation is X(s) = f (s)X(0) + f (s)BU (s)
X (t) = Ax(t) + Bu(t) (iv)

1 1  1 1 
From Eqs. (i) and (iv)    
s  
s(s + 3) −1   s s(s + 3)   1 1
0 1   1 X(s) =   3 +   0  
A=  B=   1     1     s 
 0 −3  0 0


s + 3 
0
 s + 3 

To find f(t)
 −1 3  1
 s 0   0 1   s −1     
s(s + 3)   s 
+
(i) sI − A =  − =   s

 0 s  0 −3  0 s + 3 =  +  1/s
   
 0
3

s + 3 1    
(ii) (sI − A)−1 =
1 s+3
 
s(s + 3)  0 s  −(s + 3) + 3   1   −1   1 
     
 s(s + 3)   s2   s + 3   s2 
1 1  =    + 
      3   
+ =

  0
3
s s(s + 3)    
f(s) =      s + 3   
0
  s+3
 1 
0  1 − 1 
 s + 3   2 
= 
s s + 3
 1  
(iii) To determine L−1   we use partial  3 

 s(s + 3)   
   s+3 
fraction method.
1 A B
= +  t − e−3 t 
s(s + 3) s s+3 Then, X(t) = L−1 X(s) =  −3 t 
1 = A(s + 3) + Bs  3e 
s = 0; A = 1/3 Ans. (c)
s = −3 : B = −1/3
30. For asystem with the transfer function
−1  1/3 1/3 
L  −  3(s − 2)
 s s + 3  H(s) = 2
4s − 2s + 1
, the matrix A in the state
1
1 − e−3 t 
.
space form x = Ax + Bu is equal to
3 
=

Chapter 6 (578-614).indd 588 3/23/2016 4:05:22 PM


SOLVED GATE PREVIOUS YEARS’ QUESTIONS        589

 1 0 0  0 1 0 32. The Bode magnitude plot of


   
(a)  0 1 0 (b)  0 0 1 104 (1 + jw )
−1 2 −4 −1 2 −4 H(jw ) = is
    (10 + jw )(100 + jw )2

0 1 0  1 0 0 (a) |H (jw)|dB
   

(c)  3 −2 1 (d)  0 0 1
 1 −2 4  −1 2 −4 40
   
20
−1
(GATE 2006: 1 Mark) log (w)
0 +1 +2 +3
Solution:  Given that: −20
3(s − 2) −40
.
H(s) = 2 ; x = Ax + Bu
4s − 2s + 1 (b) |H (jw)|dB
The characteristic equation for the system is 40

4s2 − 2s + 1  (i) 20
−1
log (w)
In the standard form for a characteristic equation 0 +1 +2 +3
−20
an sn + an −1sn −1 +  + a1s + a0 = 0  (ii) −40

From Eqs. (i) and (ii), n = 2 (c) |H (jw)|dB

a2 = 4 a1 = −2 a0 = 1 40
Therefore, 20
−1
log (w)
 0 0  0 1 0 0 +1 +2 +3
  
1

A= 0 0 1 =0 0 1 −20
  
−a0 −a1 −a2  −1 2 −4 −40

Ans. (b) (d) |H (jw)|dB


31. A discrete real all pass system has a pole at z = 2 40
∠30°, it therefore
20
(a) also has a pole at ∠30°
1
−1
2 log (w)
0 +1 +2 +3
(b) has a constant phase response over the z-plane:
−20
arg |H(z)| = constant.
(c) is stable only, if it is anticausal. −40
(d) It has a constant phase response over the unit
circle: arg |H(ejW)| = constant.
(GATE 2006: 2 Marks)

(GATE 2006: 1 Mark) Solution:  Given that

Solution:  Given that z = 2∠30° 104 (1 + jw )


H( jw ) =
Z-transform of a discrete all-pass system (10 + jw )(100 + jw )2
1 − z0*
H(z) = The standard form or Bode form is 1+ jw. Therefore,
z − z0
Therefore, pole
104 (1 + jw )
z − z0 = 0 H( jw ) =
 jw  2  jw 
2
10 1 +   
10  100 
z = z0 10 1 +

Given system has pole at z = 2∠30° = 3 + j 0.1(1 + jw )
Therefore, the system is stable if |z| < 1 and for this
=
  
1 + jw  1 + jw 
condition it should be anti-causal.  10 
 
 100 
Ans. (c)

Chapter 6 (578-614).indd 589 3/23/2016 4:05:47 PM


590        CHAPTER 6:  CONTROL SYSTEM

Corner frequency K(s − 1)


1+ =0
1 + jw ⇒ w c = 1 rad/s (s + 2)(s − 2)(s + 1)
1 + 0.1jw ⇒ w c = 10 rad/s [∵ s2 − 4 = (s + 2)(s − 2)]
1 + 0.01jw ⇒ w c = 100 rad/s
Therefore,
The observations can be explained by Bode magni-
tude plot shown in option (a): K(s − 1)
(i) Type zero system starts at 0 dB/decade. G(s) =
(ii) Between wc1 (1) to wc2 (10) → 20 dB/decade. (s + 2)(s − 2)(s + 1)
(ii) Between wc2 (10) to wc3 (100) → 0 dB/decade.
(iii) After wc3 (100) → 40 dB/decade No. of poles = 3(n) No. of zeros = 1 (m)
s= -2 s=1
Ans. (a) s= 2
33. A closed-loop system has the characteristic func- s= -1
tion (s2 − 4) (s + 1) + K (s − 1) = 0. Its root locus
plot against K is
(a) jw

s −3 −2 −1 1 2 3
−2 −1 +1 +2 Poles

(b) jw

s
(i) Number of root locus branches = 3 = number
−2 −1 +1 +2 of poles.
(ii) Number of root locus branches ending at zero
at infinity = n - 3 = 3 - 1 = 2.
(c) jw (iii) Number of root locus branches ending at finite
zero = 1.
Angle of asymptote, qA =
2K + 1
s × 180°
−2 −1 +1 +2 For K = 0, n−m

qA1 =
1
× 180° = 90°
(d) jw 2
For K = 1,

q A2 =
3
s × 90° = 270°
−2 −1 +1 +2 2
Centroid,

sA =
∑ Poles − ∑ Zeros
(GATE 2006: 2 Marks) n−m
(−1 − 2 + 2) − (−1)
= = −1
Solution:  Given that characteristic equation is 3 −1
Between two poles, s = -1 and s = -2, there exists
(s2 − 4)(s + 1) + K(s − 1) = 0  (i)
a break away point
To sketch the root locus it is necessary to obtain (s2 − 4)(s + 1)
the open-loop transfer function, G(s). K =−
(s − 1)
Therefore, Eq. (1) can be written as
dK
K(s − 1) = 0 ⇒ s = − 1. 5
1+ =0 ds
(s2 − 4)(s + 1) Ans. (b)

Chapter 6 (578-614).indd 590 3/23/2016 4:06:05 PM


SOLVED GATE PREVIOUS YEARS’ QUESTIONS        591

34. The quadratic equation (3) Im


F(s) = s5 − 3s4 + 5s3 − 7s2 + 4s + 20
is given. F(s) = 0 has
(a) a single complex root with the remaining roots ω =∞
−1
Re
being real ω
(b) one positive real root and four complex roots,
all with positive real parts
(c) one negative real root, two imaginary roots, and
two roots with positive real parts (4) Im
(d) one positive real root, two imaginary roots, and
two roots with negative real parts
ω =∞
−1
(GATE 2006: 2 Marks) Re
ω
Solution:  Given that

F (s) = s5 − 3s4 + 5s3 − 7s2 − 4s + 20


(a) (1) only (b) all, except (1)
(c) all, except (3) (d) (1) and (2) only
By Routh-Hurwitz criterion,
s5 1 5 -4 (GATE 2006: 2 Marks)
s 4
-3 -7 20
3 Solution:  Main condition for stability according
s 1 1
to the Nyquist stability criterion is that, if there is
s2 -4 20 no encirclement of -1 +j 0 point, then the system
is stable. Also there should not be any right half
s1 6 poles.
s 0
20 Therefore, option (a) is correct as it does not have
any encirclement.
If we observe the first column, sign is changing Ans. (a)
two times, so we have two poles on right half side
of imaginary axis and 5s2 + 20 = 0. So, s = ±2j 36. The system shown in the following figure is
and 1 pole on left side of imaginary axis.
Ans. (c) u1 + s−1
S
s+2
35. Consider tile following Nyquist plots of loop −
transfer functions over w = 0 to w = ∞. Which −
u2
S
of these plots represents a stable closed-loop 1
system? s−1 +

(1) Im (a) stable


(b) unstable
R (c) conditionally stable
ω=∞ (d) stable for input u1, but unstable for input u2
Re
−1 (GATE 2007: 1 Mark)
ω
Solution:  Case 1: Consider the input u1 and assume
u2 = 0.
(2) Im Therefore,

ω  s − 1   s − 1 
 
ω =∞  s + 2   s + 2  s −1
−1 Re H(s) = = =
 s − 1   1 
1 +   
s+ 2 +1 s +3

 s + 2   s − 1 s+2

Chapter 6 (578-614).indd 591 3/23/2016 4:06:14 PM


592        CHAPTER 6:  CONTROL SYSTEM

Poles are at s = -3 Separating real and imaginary parts,


−3w 2 j(w 3 − 2w )
Case 2: Consider the input u2 and assume u1 = 0
 1   1  G( jw ) = +
  w 2 (1 + w 2 )(4 + w 2 ) w 2 (1 + w 2 )(4 + w 2 )
 s − 1 
 s − 1
H(s) = =
 1   s − 1  −3w 2
1 +   
s+ 2 +1
 x = Re[G( jw )] =
 s − 1  s + 2  s+2 w 2 (1 + w 2 )(4 + w 2 )
(s + 2) −3
=
(s − 1)(s + 3) As w → 0 x = ,y=0
4
Poles are at s = 1 and s = -3. From Case 1, there Ans. (b)
are no right half poles. Therefore, system is stable.
38. The system 900 is be compensated such
From Case 2, one pole lies on right half of s-plane.
Therefore, system is unstable. s(s + 1)(s + 9)
Ans. (d) that its gain-crossover frequency becomes same as
its uncompensated phase-crossover frequency and
37. If x = Re G(jw), and y = Im G(jw) then for w → 0+, provides a 45° phase margin, to achieve this, one
may use
1
the Nyquist plot for G(s) = becomes
s(s + 1)(s + 2) (a) a lag compensator that provides an attenuation
asymptotic to the line of 20 dB and a phase lag of 45° at the frequency
3 of 3 3 rad/s.
(a) x = 0 (b) x = − (b) a lead compensator that provides an amplifica-
4
y tion of 20 dB and a phase lead of 45° at the
(c) x = y - 1/6 (d) x = frequency of 3 rad/s
 3
(c) a lag-lead compensator that provides an ampli-
fication of 20 dB and a phase lag of 45° at the
(GATE 2007: 2 Marks)
frequency of 3 rad/s.
(d) a lag-lead compensator that provides an atten-
Solution:  Given that:
uation of 20 dB and phase lead of 45° at the
1 frequency of 3 rad/s
G(s) =
s(s + 2)(s + 1)
(GATE 2007: 2 Marks)
Substituting s = jw in the transfer function, we get
Solution: Given that:

G( jw ) =
1 900
 (i)
( jw )( jw + 1)( jw + 2)
H(s) =  (i)
s(s + 1)(s + 9)

To separate real and imaginary parts, H(s) is the uncompensated transfer function and
HC(s) is the compensated transfer function.
G( jw ) = x + jw
R(s) C(s)
+ GC(s)

G(s)
Multiplying Eq. (i) with respective complex conju-
gates, we get
HC (s) = G(s) ⋅ GC (s)
(−jw )(1 − jw )(2 − jw )
G( jw ) =
900  (ii)
= ⋅ GC (s)
(−jw )(1 − jw )(2 − jw ) s(s + 1)(s + 9)

1
×
( jw )(1 + jw )(2 + jw ) Substituting s = jw in Eqs. (i) and (ii) for the
transfer functions, we have
(−jw )(2 − jw − 2 jw − w 2 )
=
H( jw ) =
900
w 2 (1 + w 2 )(4 + w 2 )
jw ( jw + 1)( jw + 9)
−2 jw − 3w 2 + jw 3
= HC ( jw ) =
900
G ( jw )
w 2 (1 + w 2 )(4 + w 2 ) jw ( jw + 1)( jw + 9) C

Chapter 6 (578-614).indd 592 3/23/2016 4:06:33 PM


SOLVED GATE PREVIOUS YEARS’ QUESTIONS        593

It is given that, The gain crossover frequency of compensated


wgc of compensated system = wpc of uncompen­­ system is lower than uncompensated system, it is
sated ­system necessary to use lag-lead compensator.
−180° = ∠H( jw pc )
Ans. (d)
−180° = ∠H( jw pc )
w pc 
−180° = − 90° − tan−1 w pc − tan−−1 1   −1 w pc 
39. Consider the discrete-time system shown in
−180° = − 90° − tan wpc9−tan  
 9 
the figure where the impulse response of G(z) is
g(0) = 0, g(1) = g(2) = 1, g(3) = g(4) = … = 0.
  w 
−90° = −  tan−1 w pc +  tan−1  pc 
 −1 w pc 
−90° = −  tan−1 wpc9+tan   

+
  
 
 S
 9  G(z)
  +
 w + w pc  
 pc  w + w pc 
tan−1  9  =pc90° 
2 −1  9  = 90° K
 w pc
tan
 
 1−  w pc 
2

 9  1 −  This system is stable for range of values of K


 9 
w pc  1
w pc + w pc  , 
(a) −1 (b) [−1, 1]
9 w + 
 2 

pc
=∞
= 9
w pc 2
1− w pc2
 1   1 
1− 
(c) − , 1 
(d) − , 2
9
9 
 2    2 
w pc
2
= 0 w pc
2
⇒ 1−
9 ⇒ 1− =0 (GATE 2007: 2 Marks)
9
w pc = 3 rad/s
w pc = 3 rad/s Solution:  Given that, impulse response of G(z) is
g(0) = 0
g(1) = 1 = g(2)
The phase margin is given by, g(3), g(4) … = 0.
PM = 180°∠HC ( jw gc ) G(z)
The transfer function is H(z) =  (i)
−1 1 − G(z)K
45° = 180° − 90° − tan w gc
Also it is given that, g(n) = d (n - 1) + d (n - 2).
 w gc 
−1 
− tan   + ∠GC ( jw gc ) Therefore,
 9 
G(z) = z-1 + z-2 (ii)
 3 + (1/3) 
−45° = − tan−1   + ∠GC ( jw gc )
 1 − (3)(1/3)  Substituting Eq. (ii) in Eq. (i), we get
 
−45° = −90° + ∠GC ( jw gc ) z −1 + z −2
H(z) =
45° = ∠GC ( jw gc ) 1 − K(z−1 + z−2 )

Bring the above relation into positive powers of z


From the definition of gain crossover frequency,
the magnitude should be unity (or) dB magnitude z−2 (z + 1)
H(z) =
z−2 (z 2 − Kz − K )
should be zero.

HC ( jw gc ) = 1 For a stable system, the poles should lie on or


inside the unit circle, which implies that
900 GC ( jw gc )
=1 |z| ≤ 1
w gc 1 + w gc
2
81 + w gc
2

Characteristic equation, z2 - Kz - K = 0. Therefore,


G( jw gc ) =
3 9 + 1 9 + 81 1
=
900 10 K ± K 2 + 4K
z=
1
G( jw gc ) in dB = 20 log   = −20 dB
2
10  z ≤1

Chapter 6 (578-614).indd 593 3/23/2016 4:06:47 PM


594        CHAPTER 6:  CONTROL SYSTEM

Therefore,
16 + K = 0 ⇒ K = −16
K ± K + 4K
2
≤2
From auxiliary equation
K 2 + 4K ≤ 2−K
Squaring on both sides, s2 + (64 + 3K ) = 0
s2 + 64 + 3 (−16) = 0
K 2 + 4K ≤ 4 + K 2 − 4K s2 + 64 − 48 = 0
8K ≤ 4
s2 + 16 = 0 ⇒ s2 = −16 ⇒ s = 4 j
K ≤ 1/2
Substituting s = jw, we get
So, (-1, 1/2)
Ans. (a) jw = 4 j ⇒ w = 4 rad/s
40. If the loop gain K of a negative feedback system
 Ans. (b)
K(s + 3)
having a loop transfer function is to be 41. The system shown in the following figure
(s + 8)2
adjusted to induce a sustained oscillation then
(a) the frequency of this oscillation must be b0 c0 b1 c1
4
3
rad/s. S

S S
(b) the frequency of this oscillation must be must
1/s 1/s P
be 4 rad/s.
(c) the frequency of this oscillation must be must
a0 a1
4
be 4 or rad/s.
3
(d) such a K does not exist. can be reduced to the form
(GATE 2007: 2 Marks) +
X S Y P
Solution:  Given that +

K(s + 3) Z
G(s) =
(s + 8)2
with
To find K for sustained oscillation, Routh-Hurwitz 1
(a) X = c0 s + c1 , Y = 2
, Z = b0 s + b1
criterion is used. Sustained oscillation can be shown (s + a0 s + a1 )
in Routh array with a row of zero.
(c0 s + c1 )
K(s + 3) (b) X = 1, Y = 2
, Z = b0 s + b1
1 + G(s)H(s) = 1 + (s + a0 s + a1 )
(s + 8)2
(b1s + b0 )
= (s + 8)2 + K(s + 3) (c) X = c1s + c0 , Y = 2
,Z = 1
(s + a1s + a0 )
2
= s + 64 + 16s + Ks + 3K
1
= s2 + s(K + 16) + (64 + 3K ) (d) X = c1s + c0 , Y = 2
,
(s + a1s + a0 )
s2 : 1 64 + 3K (Auxillary equation) Z = b1s + b0 z

s1: K + 16 (To make a row of zero (GATE 2007: 2 Marks)


for sustained oscillation)
Solution:  From the given block diagram, the
s0 : 64 + 3K overall transfer function is given as,

Chapter 6 (578-614).indd 594 3/23/2016 4:07:06 PM


SOLVED GATE PREVIOUS YEARS’ QUESTIONS        595

(c0 + c1s)P Solution:  Impedance of the given network is


s2 + a1s + a0  1 
Z = R + j wL −
C(s)

 wC 
=
P (b + sb1 )
1− 2 0
R(s)
(s + a1s + a0 ) Admittance is,
 (i)
From the reduced transfer function, eliminating 1 1
Y = =
 1 
R + j wL −
feedback and series block, we get Z

 wC 
R(s) C(s)  1 
R − j wL −
+

X Y P
wC 
+
1 
= ×
 1   1 
R + j wL −  R − j wL −
Z

 wC   wC 
R(s) C(s)
+  1 
j  wL −
X YP
+ 
R  wC 
= −
Z  1 
2
 1 
2
R 2 + wL −  R 2 + wL − 
 wC   wC 
C(s) YP
=X⋅  (ii) = Re(Y ) + Im(Y )
R(s) 1 − YPZ

From Eqs. (i) and (ii) By varying frequency for Re(Y) and Im(Y), we can
obtain the admittance locus as follows
1
X = c0 + c1s; Y = 2
; Im
s + a1s + a0
Z = b0 + sb1
Ans. (d) Re

42. The RLC series circuit shown is supplied from a


variable frequency voltage source. The admittance Ans. (a)
locus of the RLC network at terminals AB for
43. Consider the feedback control system shown in
increasing frequency w is
the following figure which is subjected to a unit
step input. The system is stable and has the fol-
A R lowing parameters kp = 4, ki = 10, w = 500 and
z = 0.7.
w L
C ki
s Z
B 1
+
Σ w2
Σ
0 +
(a) Im (b) Im kp
− + s2 + 2zws +w 2
  

Re Re The steady-state value of Z is


w w (a) 1    (b) 0.25    (c) 0.1    (d) 0
(GATE 2007: 2 Marks)
(c) Im     (d) Im Solution:  Given unit step input u(t)

u(t)
w w
Re Re

t
(GATE 2007: 2 Marks)

Chapter 6 (578-614).indd 595 3/23/2016 4:07:23 PM


596        CHAPTER 6:  CONTROL SYSTEM

Therefore, U(s) = 1/s In general, transfer function of the system is


kp = 4
ki = 10 Y (w ) = G(w )H(w )
w = 500
z = 0.7 Therefore, H(w ) = Ke−jw td
ki Magnitude is K
s Phase is -wtd
f = −w td ⇒ f ∝ w
Z
+
+ (∵ td is constant)
+
E(s)  Ans. (c)
45. G(z) = az−1 + bz−3 is a low-pass digital filter with
k
Z = E(s) ⋅ i a phase characteristics same as that of the above
s question if
(a) a = b (b) a = −b
sR(s)
E(s) = lim
s→ 0 1 + G(s)H (s) (c) a = b(1/3)
(d) a = b−(1/3)
s ⋅ 1/s
= lim (GATE 2007: 2 Marks)
s→ 0 k   w n2 
1 +  i + kp   2 
 s   s + 2zw n s + w n2  Solution:  Given low pass filter
G(z) = az−1 + bz−3
Therefore,
G(z) = z = e jw = ae−jw + be−j3w
 
 
 
 s ⋅ 1/s   ki 
Therefore, from the linear phase characteristic we have
Z = lim    a =b
s→ 0     w    s 

2
 1 +  + k  
k i n
 
 p  2
 s   s + 2zw n s + w n2  
 Ans. (a)
  Linked Answer Questions 46 and 47: Consider
ki k the RLC circuit shown in the figure.
Z = lim = i =1
s→ 0  wn 
 ki

2
s + (ki + kp s)  2  R = 10 Ω L = 1mH
 s + 2zw n s + w n 
2

Ans. (a)
ei C = 10 µF eo
Statement for Linked Answer Questions
44 and 45:
44. A signal is processed by a causal filter with transfer 46. For a step-input e, the overshoot in the output e0
function G(s). For a distortion free output signal will be
waveform, G(s) must (a)  0, since the system is not under-damped
(a) provide zero phase shift for all frequency (b)  5%
(c)  16%
(b) provide constant phase shift for all frequency
(d)  48%
(c) provide linear phase shift that is proportional
to frequency. (GATE 2007: 2 Marks)
(d) provide a phase shift that is inversely propor-
tional to frequency. Solution:  Applying Kirchhoff’s voltage law,
1
(GATE 2007: 2 Marks) e i(s) = RI (s) + Ls I (s) +I (s)  (i)
Cs
Solution:  For a distortion free output, = I (s)[R + L + 1/Cs]
y(t) = Kg(t - td)
I (s) Þ I (s) = Cs e0 (s) 
1
eo (s) = (ii)
Taking Fourier transform on both sides, Cs
Y (w ) = KG(w )e−jw td Substituting Eq. (ii) in Eq. (i)

Chapter 6 (578-614).indd 596 3/23/2016 4:07:36 PM


SOLVED GATE PREVIOUS YEARS’ QUESTIONS        597

é RCs + S 2 LC + 1 ù Steady-state value of output


ei (s) = Cs eo (s) = ê ú e−s
êë Cs úû lim y(t) = lim sY (s) = lim s ⋅ =0
t →∞ s→ 0 s→ 0 s2 + 3s + 2
eo (s) 1/LC
Þ
1
= 2 Ans. (a)
ei (s) R 1
s LC + RCs + 1 s2 + s +
L LC 49. The transfer functions of two compensators are

The standard second order characteristic equation is, 10(s + 1) s + 10


C1 = , C2 =
s2 + 2 t w n s + w n2 (as w n2 = 1/LC and w n = 1/ LC )
(s + 10) 10(s + 1)

Which one of the following statements is correct?


R 1 R R
2 xw n = ⇒ 2x ⋅ = ⇒x = ⋅ C (a) C1 is a lead compensator and C2 is a lag
L LC L 2p compensator
10 1 × 10−3 (b) C1 is a lag compensator and C2 is a lead
z = × = 0.5 compensator
2 10 × 10−6 (c) Both C1 and C2 are lead compensators
So, x < 1 (under-damped) (d) Both C1 and C2 are lag compensators
So, %peak overshoot = e−tp / 1−t 2
× 100 = e−0.5p / 1−0.52
× 100 = 16% (GATE 2008: 2 Marks)
%peak overshoot = e−tp / 1−t
× 100 = e−0.5p / 1−0.5
× 100 = 16%
2 2

Solution:  Given that:


Ans. (c)
10(s + 1) s + 10
47. If the above step response is to be observed on a C1 = and C2 =
non-storage CRO, then it would be best to have s + 10 10(s + 1)
the ei as a
Substituting s = jw in the transfer functions, we get
(a)  step function
(b)  square wave of frequency 50 Hz 10( jw + 1)
C1 =
(c)  square wave of frequency 300 Hz jw + 10
(d)  square wave of frequency 2.0 kHz
w
(GATE 2007: 2 Marks) Phase lead f C1 = tan−1 w − tan−1
10
 
w − w 
Solution:  On a non-storage CRO, it is best to
 
= tan−1  10 
w 2 
have the ei as a square wave of frequency 300 Hz.

Ans. (c)
1 + 
48. The transfer function of a linear time invariant
 10 
system is given as jw + 10
C2 =
G(s) = 2
1 ( jw + 1)
s + 3s + 2
w
The steady-state value of the output of this system Phase lag fC 2 = tan−1 − tan−1 w
for a unit impulse input applied at time instant 10
 w 
 − w 
t = 1 will be
 
= tan−1  10 2
(a) 0     (b) 0.5     (c) 1     (d) 2
1 + w /10 
(GATE 2008: 2 Marks)
 
 −9w 
Solution:  Given that:
= tan−1  <0
10 + w 2 
1
 9w 
G(s) =
= tan−1 
2
s + 3s + 2 > 0
r(t) = d (t − 1) 10 + w 2 

R(s) = e−s Ans. (a)


−s
e 50. The asymptotic Bode magnitude plot of a minimum
Y (s) = R(s)G(s) =
s2 + 3s + 2 phase transfer function is shown in the following figure.

Chapter 6 (578-614).indd 597 3/23/2016 4:07:52 PM


598        CHAPTER 6:  CONTROL SYSTEM

G(jω) The RH array is


(dB) -40 dB/decade
s3 1 30
−20 dB/decade ω (rad/s)
20
s2 13 K
0
0.1 (log scale)
−20 (30 × 13) − K
s1
0 dB/decade 13

This transfer function has s0 K


(a) three poles and one zero
(b) two poles and one zero For a stable system, first column of elements should
(c) two poles and two zeros be positive, that is

⇒− K<> 00 ⇒ K<
(d) one pole and two zeros 1
s1 : 390 − Kss1>:: 0390
390 −KK >390 ⇒K < 390
390
0 :K >0
0
s :K>0 s :K>0
(GATE 2008: 2 Marks) 0 s
Therefore, Therefore
Therefore,,
00 < K<
Solution:  The slope at 40 dB/decade shows pres-
ence of 2 poles. 0 < K < 390 <K < 390
390
The slope at 20 dB/decade shows presence of one Ans. (c)
zero. 52. The transfer function of a system is given as
The slope at 0 dB/decade shows presence of one
100
zero. 2
.
Therefore, the transfer function has two poles and s + 20s + 100
two zeros.
Ans. (c) This system is
51. The following figure shows a feedback system (a) an overdamped system
where K > 0. (b) an underdamped system
+ (c) a critically damped system
S K
(d) an unstable system
s(s+3)(s+10)
(GATE 2008: 2 Marks)

The range of K for which the system is stable will


be given by Solution:  Given that transfer function is
(a) 0 < K < 30 (b) 0 < K < 39 100
(c) 0 < K < 390 (d) K > 390 2
s + 20s + 100
(GATE 2008: 2 Marks)
Type depends on the value of damping ratio (z).
Standard transfer function of second-order system
Solution:  Applying Routh-Hurwitz criterion to
obtain the range of K for a stable system, we get w n2
= 2
s + 2zw n s + w n2
C(s) K K Therefore,
= =
R(s) s(s + 3)(s + 10) s(s + 3)(s + 10) + K
K w n2 = 100 ⇒ w n = 10
1+
s(s + 3)(s + 10) 2zw n = 20

z =
20
The characteristic equation is given by =1
20
K + s[s2 + 13s + 30 ] = 0
z = 1 shows that the system is critically damped.
s3 + 13s2 + 30s + K = 0 Ans. (c)

Chapter 6 (578-614).indd 598 3/23/2016 4:08:04 PM


−1
s − 1   0
0]    
0 s + 2 1 
= [1

−1
1 1 
 
s s(s + 2)   0
= [1 0 ]    
SOLVED GATE PREVIOUS1YEARS’ 1 
 QUESTIONS 
0   
      599
 s + 2 
 1 
Statement for Linked Answer Questions 53
and 54: The state space equation of a system is  
 s(s + 2) 
de­scribed by = [1 0]  
 1 
x = Ax + Bu  
 s + 2 
y = Cx
1
where x is state vector, u is input, y is output and =
s(s + 2)
0 1  0
A=  , B =   , C = [1 0 ]
 0 −2   Ans. (d)
1 
54. A unity feedback is provided to the above system
53. The transfer function G(s) of this system will be G(s) to make it a closed-loop system as shown in
the figure below.
s s+1
(a)  (b) 
(s + 2) s(s − 2) r(t) + Y(t)
Σ G(s)
s 1
(c)  (d) 
(s − 2) s(s + 2)

(GATE 2008: 2 Marks)
For a unit step input r(t), the steady-state error in
the output will be
Solution:  Given that:
(a) 0     (b) 1     (c) 2     (d) ∞
x = Ax + Bu (i) (GATE 2008: 2 Marks)
y = Cx (ii)

0 1  0 Solution:  We have
A=  B= 
0 − 2  1 
   H(s) = 1
1 0 R(s) = 1/s
I= 
0 1
C = [1 0 ]
 Steady-state error is given by
Taking Laplace transform of Eq. (i), we get ess = lim s ⋅ E(s)
s→ 0
sX(s) = AX(s) + BU (s)  
= lim s ⋅  
R(s)
sX(s) − AX(s) = BU (s) s→ 0  1 + G(s)H(s) 
 
X(s) [sI − A ] = BU (s)
 
−1  1 
X(s) = [sI − A ] BU (s)  
= lim s  s 
s→ 0  1 
1 + ⋅ 1
Taking Laplace transform on Eq. (ii), we get  s(s + 2) 
 s(s + 2) 
= lim  =0
Y(s) = C X(s)
s→ 0  s(s + 2) + 1 
Y(s) = C [sI − A]-1 BU(s)
Ans. (a)
Transform function is given by
55. The measurement system shown in the following
= C(sI − A)−1 B
Y (s) figure uses three sub-systems in cascade whose
U (s) 1
gains are specified as G1, G2 and . The relative
−1
s − 1   0
G3
0]     small errors associated with each respective sub-
0 s + 2 1 
= [1
 system G1, G2 and G3 are e1, e2 and e3. The error
−1
1 1  associated with the output is
 
s s(s + 2)   0
0]    
1 
= [1
   
1
0 1  Input G1 G2 Output
 s + 2  G3
 1 
 
 s(s + 2) 
= [1 0]  
 1 
 
 s + 2 
1
Chapter 6 (578-614).indd 599 = 3/23/2016 4:08:21 PM
600        CHAPTER 6:  CONTROL SYSTEM

e1 ⋅ e 2
(a) e 1 + e 2 +
1
(b)
e3 e
  3

(c) e 1 + e 2 − e 3 (d) e 1 + e 2 + e 3 -1.42



(GATE 2009: 1 Mark)

Solution:  Given that:


e1 is the error associated with G1 Number of closed-loop poles in right half of s place,
e2 is the error associated with G2
Z = P −N
e3 is the error associated with 1/G3.
1 where P is the number of open loop poles and N is
Overall gain in series combination= G1G2
G3 the number of encirclements (-1, j 0)
From above sketch N = −2 and P = 0, therefore
Therefore, error as applicable for product of more
Z = 0 - (−2) ⇒Z = 2,
than two quantities is given by e 1 + e 2 + e 3
So the system is unstable with 2 poles on right half
Ans. (d) of the s-plane.
 Ans. (d)
56. The polar plot of an open-loop stable system is
shown in the following figure. The closed-loop 57. The first two rows of Routh’s tabulation of a third-
system is order equation are as follows:

Im s3 2 2
s2 4 4

w=∞
This means there are
(a) two roots at s = ± j and one root in right half
−1.42 Re s-plane
(b) two roots at s = ± j 2 and one root in left half
s-plane
(c) two roots at s = ± j 2 and one root in right half
w=0 s-plane
(d) two roots at s = ± j and one root in left half
s-plane
(a) always stable
(b) marginally stable (GATE 2009: 1 Mark)
(c) unstable with one pole on the RH s-plane
(d) unstable with two poles on the RH s-plane Solution:  From the given Routh’s array, the aux-
(GATE 2009: 1 Mark) iliary equation is,

2s2 + 2 = 0
Solution:  Mapping the mirror image of the given
plot, we have 2s2 = −2
s2 = −1 s = ±j

The characteristic equation is

2s3 + 4s2 + 2s + 4 = 0
-1.42
Dividing both sides by 2, we get

s3 + 2s2 + s + 2 = 0
Mapping the circle of R → 0 to an infinite radius
R → ∞ gives To find the roots

Chapter 6 (578-614).indd 600 3/23/2016 4:08:37 PM


SOLVED GATE PREVIOUS YEARS’ QUESTIONS        601

Therefore, transfer function is


s2 (s + 2) + 1(s + 2) = 0
1000(s + 5)
(s + 2) (s2 + 1) = 0
2
s = −2 s = ± j s (s + 2)(s + 25)
Ans. (b)
Ans. (d)
59. The unit-step response of a unity feedback system
58. The asymptotic approximation of the log-­magnitude with open-loop transfer function
vs. frequency plot of a system containing only real
poles and zeros is shown in the following figure. Its K
G(s) =
transfer function is (s + 1)(s + 2)
80dB −40dB/decade is shown in the following figure. The value of K is
−60dB/decade
1

Response
0.75
0.5
0.25
0
w rad/s 0 1 2 3 4
0.1 2 5 25 Time(s)
10(s + 5) 1000(s + 5)
(a)  (b)  2 (a) 0.5     (b) 2     (c) 4     (d) 6
s(s + 2)(s + 25) s (s + 2)(s + 25)

(GATE 2009: 2 Marks)
100(s + 5) 80(s + 5)
(c)  (d)  2 Solution:  Given that
s(s + 2)(s + 25) s (s + 2)(s + 25)
K
(GATE 2009: 1 Mark) G(s) = .
(s + 1)(s + 2)
Solution:  From the given data: H(s) = 1
(i) Bode plot starts with slope −40 dB/dec which From the figure,
implies the presence of two poles at origin. Steady-state error (ess) = 1 − 0.75 = 0.25
(ii) Slope change from −40 dB/dec to −60 dB/ We know that
dec (−20 dB/dec added) implies that there is ess = lim s ⋅ E(s)
a pole in the transfer function. wc1 = 2 rad/s. s→ 0
(iii) Slope change from −60 dB/dec to −40 dB/dec  
0.25 = lim s ⋅  
R(s)
(+20 dB/dec added) implies that there exists s→ 0  
a zero in the transfer function wc2 = 5 rad/s.  1 + G (s)H (s) 
(iv) Slope change from −40 dB/dec to −60 dB/ 0.25 = lim s ⋅
1/s
[∵ R(s) = 1/s]
dec. (−20 dB/dec added implies that there is s→ 0 K
1+
a pole in the transfer function. wc3 = 25 rad/s. (s + 1)(s + 2)
(s + 1)(s + 2)
0.25 = lim
Therefore, s→ 0 K + (s + 1)(s + 2)
K(s + 5) 2
T (s) = 0.25 =
2
s (s + 2)(s + 25) K +2
2 = 0.25K + 0.5
To find K, substitute s = jw. We get 1. 5
K= =6
K(s + 5) 6.25
T ( jw ) =
( jw ) ( jw + 2)( jw + 25)
2 Ans. (d)
60. The open-loop transfer function of a unity feedback
T (jw)|at w =0 = 80. Therefore,
system is given by
(e−0.1s )
K(5)
80 = 20 log ⇒ K = 1000
0.1 × 2 × 25
2 G(s) =
s

Chapter 6 (578-614).indd 601 3/23/2016 4:08:56 PM


602        CHAPTER 6:  CONTROL SYSTEM

The gain margin of this system is 2s + 5 2s − 5


(a) 11.95 dB (b) 17.67 dB (c) (d)
s + 5s − 6
2 2
(c) 21.33 dB (d) 23.9 dB s + 5s + 6

(GATE 2009: 2 Marks) (GATE 2009: 2 Marks)

Solution:  Given that:


Solution:  Given that:
(e−0.1s )
dx1(t)
= −3x1(t) + x2 (t) + 2u(t)  (i)
G(s) =  (i) dt
s
dx2 (t)
= −2x2 (t) + u(t) 
1
Gain margin = 20 log  (ii) (ii)
G( jw pc ) dt

where wpc is the phase crossover frequency.


y(t) = x1(t) (iii)

∠G( jw pc ) = −180°  (iii) Taking Laplace transform on Eqs. (i), (ii) and (iii),

we get
Substituting j = jw in the transfer function given
in Eq. (i), we get sX1(s) = −3X1(s) + X2 (s) + 2 U(s)

e−0.1jw
G( jw ) = X1(s) (s + 3) = X2 (s) + 2 U(s)  (iv)
jw
 180°  sX2 (s) = −2 × 2 (s) + U (s)
∠G( jw ) = −0.1w ×  − 90°
X2 (s) (s + 2) = U (s)
 p 
Substituting w = wpc, we have U (s)
 180° 
X2 (s) =  (v)
∠G( jw pc ) = −0.1w pc ×  − 90° (iv) s+2
 p  Substituting Eq. (v) in Eq. (iv), we get
Substituting value from Eq. (iv) in Eq. (iii), we get
U (s)
 180°  X1(s)(s + 3) = + 2U (s)
−0.1w pc ×  − 90° = −180° s+2
 p  [1 + 2(s + 2)]
⇒ w pc = 15.7 rad/s
X1(s)(s + 3) = U (s)
s+2
Substituting value in Eq. (ii), we get gain margin as U (s)(2s + 5)
X1(s) =
(s + 2)(s + 3)
1
GM = 20 log
15.7 From Eq. (iii),
= 20 log 15.7 = 23.9 dB Y (s) = X1(s)
Ans. (d) U (s)(2s + 5)
Common Data for Questions 61 and 62: Y (s) =
(s + 2)(s + 3)
A system is described by the following state and
output equations Transfer function is
Y (s) (2s + 5)
dx1(t) =
= −3x1(t) + x2 (t) + 2u(t) U (s) (s + 2)(s + 3)
dt
Ans. (c)
dx2 (t)
= −2x2 (t) + u(t) 62. The state-transition matrix of the above system is
dt
y(t) = x1(t)  e−3t 0 e−3t e−2t − e−3t 
(a)  −2t −2 t 
(b)  
e2t 
−3 t

where u(t) is input and y(t) is the output. e +e e   0
e−3t e−2t + e3t  e3t e−2t − e−3t 
61. The system transfer function is (c)  
−2 t  (d)  
−2 t 
s+2 s+3  0 e   0 e 
(a)  2 (b)  2
s + 5s − 6 s + 5s + 6 (GATE 2009: 2 Marks)

Chapter 6 (578-614).indd 602 3/23/2016 4:09:28 PM


SOLVED GATE PREVIOUS YEARS’ QUESTIONS        603

Solution:  From Eqs. (i) and (ii) given in Question 61, Gain without error = 10%
we have Gain with 10% error = 10 + 0.1= 10.1
x1(t) = −3x1(t) + x2 (t) + 2u(t) Gain with −10% error = 10 − 0.1 = 9.9.
Therefore, 10 ± 1%
x2 (t) = −2x2 (t) + u(t)
Ans. (a)
Therefore, state matrix is 2
64. For the system , the approximate time taken
 x1  −3 1  x1  2 (s + 1)
 =    +   u(t)
 x   0 − 2   x  1  for a step response to reach 98% of its final value is
  
2   2  
⇒ x(t) = Ax + Bu (a) 1 s     (b) 2 s     (c) 4 s     (d) 8 s

State transition matrix f(t) = L−1[f (s)]


(GATE 2010: 1 Mark)

f(s) = (sI − A)−1 Solution:  Given that


s 0 −3 1 s + 3 − 1 
sI − A =  − = 
2
Y (s) =
0 s  0 − 2  0 s + 2
 s+1
s + 2 1 
[sI − A ]−1  
1 Convert in s domain to time domain, we have, step
 s + 3
=
(s + 3)(s + 2)  0 response,

y(t) = 2(1 − et )u(t) = 0.98 × 2


Therefore,
⇒ t = 3. 9 s ≈ 4 s
 s+2 1 
  Ans. (c)
 (s + 3)(s + 2) (s + 3)(s + 2) 
f(s) =  
  65. If the electrical circuit of Fig. (b) is an equivalent
0 s + 3 
 (s + 3)(s + 2) 
of the coupled tank system of Fig. (a), then

 1 1 
 
 s + 3 (s + 2)(s + 3) 
=  
 
0 1 
 s + 2  h1 h2
e−3t −2 t
−e −3 t 
f = L−1 f (s) =  
e
−2 t 
 0 e 
(a) Coupled tank
Ans. (b)

63. As shown in the following figure, a negative feed- B D


back system has an amplifier of gain 100 with
± 10% tolerance in the forward path, and an atten- A C
uator of value 9/100 in the feedback path. The
overall system gain is approximately
+ (b) Electrical equivalent
100 ± 10%
− (a) A, B are resistances and C, D capacitances
(b) A, C are resistances and B, D capacitances
9/100 t
(c) A, B are capacitances and C, D resistances
(a) 10 ± 1% (b) 10 ± 2%
(c) 10 ± 5% (d) 10 ± 10% (d) A, C are capacitances and B, D resistances

(GATE 2010: 1 Mark) (GATE 2010: 1 Mark)


Solution:  Given that
Gain = 100 with ± 10% tolerance and Solution:  From the given figure, h1 and h2 are
9/100 attenuator. compared as voltage across the capacitors.

Chapter 6 (578-614).indd 603 3/23/2016 4:09:39 PM


604        CHAPTER 6:  CONTROL SYSTEM

B and D are compared as the resistance across the −270°


two points.
Therefore, A and C are capacitances and B and D
are resistances. w=∞
Ans. (d) −180° 0°

1
66. The frequency response of G(s) =
w=0
[s(s + 1)(s + 2)]
plotted in the complex G(jw) plane (for 0 < w 0 −90°
< ∞ is
To find the point of intersection at the real axis,
we divide and multiply Eq. (1) with complex con-
(a)  Im (b)  Im jugate of the expression.
−3/4 ω=0
Re
G( jw ) =
1
( jw )(1 + jw )(2 + jw )
(−jw )(1 − jw )(2 − jw )
−3/4
Re ×
ω= 0 (−jw )(1 − jw )(2 − jw )
−jw (2 − jw − 2 jw − w 2 )
=
(c) (d) Im w 2 (1 + w 2 )(4 + w 2 )
Im
ω =0 −2 jw − 3w 2 + jw 3
=
Re w 2 (1 + w 2 )(4 + w 2 )
−3w 2 jw (2 − w 2 )
G( j w) = −
Re
ω= 0 w 2 (1 + w 2 )(4 + w 2 ) w 2 (1 + w 2 )(4 + w 2 )

−1/6 −1/6
At real axis, imaginary part |G (jw)| = 0. Therefore,
(GATE 2010: 2 Marks) w (2 − w2) = 0

w2 = 2; w = 2 rad/s
Solution:  Given that
At w = 2 rad/s
1
G(s) =
G( jw ) =
1 1
s(s + 1)(s + 2) < 3/4
2 ( 2 + 1) 2 + 4
=
6
Substituting s = jw in the transfer function, we get

−3/4 Im
G( jw ) =
1
 (1)
jw ( jw + 1)( jw + 2)

Re
G( jw ) =
1
and
w 1 + w2 4 + w2
w
∠G( jw ) = −90° − tan−1 w − tan−1
Ans. (a)
2
−1 2  0
x = Ax + Bu A=  , B =  
1 
67. The system with
(i) At w = 0, G( jw ) = ∞ and ∠G( jw ) = −90°  0 2  
−1 2  0
A=  ,B =   is
(ii) At w − ∞, G( jw ) = 0 and ∠G( jw ) = −270°  0 2 1 
 

Chapter 6 (578-614).indd 604 3/23/2016 4:10:01 PM


SOLVED GATE PREVIOUS YEARS’ QUESTIONS        605

(a) stable and controllable By Routh-Hurwitz criterion


(b) stable but uncontrollable
(c) unstable but controllable s(s2 + 4s + 3) + Ks + 2K = 0
(d) unstable and uncontrollable
s3 + 4s2 + 3s + Ks + 2K = 0
(GATE 2010: 2 Marks)
s3 + 4s2 + s(3 + K ) + 2K = 0

Solution:  Given: To form Routh array,

x = Ax + Bu s3 1 3+K
−1 2  0 2
A=  B =  . s 4 2K
0   
 2 1  s1 12 + 2K
For 2 × 2 matrix, first check for [B : AB]. 4

   0   2 s0 2K
AB = −1 2   =  
 0 2 1  2 It is given that K > 0, therefore, from s1
 
[B : AB ] =  0 2 ≠ 0 12 + 2k
 1 2 > 0.
4
Therefore, system is controllable. Thus, there is no sign change in the first column of
Next check for stability Routh array which indicates the absence of right-
half poles.
A − lI = 0
To sketch the root locus:
   
A − lI = −1 2 −  l 0 Equation (i) can be written as,
 0 2  0 l 
K(s + 2)
 2
= −1 − l
1+ =0
s(s + 1)(s + 3)
 0 2 − l  Standard form
= (−1 − l )(2 − l ) − 0
1 + G(s)H(s) = 0
= −2 + l − 2 l + l 2 = l 2 − l − 2
For unity feedback system H(s) = 1. Therefore,
A − lI = 0;
K(s + 2)
l2 − l − 2 = 0 ⇒ l = 2, −1 G(s) =
s(s + 1)(s + 3)
Eigen values are of opposite sign, so the system is Poles: Zeros
unstable. N = 3 m=1
 Ans. (c) s = 0  s = −2
s = −1
68. The characteristic equation of a closed-loop system s = −3
is s(s + l)(s + 3) + k(s + 2) = 0, k > 0. Which of jw
the following statements is true?
(a) Its roots are always real
(b) It cannot have a breakaway point in the range
-l < Re[s] < 0 s
(c) Two of its roots tend to infinity along the −3 −2 −1 0
asymptotes Re[s] = −1
(d) It may have complex roots in the right half
plane
Number of root locus branches (n) = 3.
(GATE 2010: 2 Marks) Number of root locus branches ending at zero at
infinity = n − m = 2.
Solution:  Given that Number of root locus branches ending at first zero = 1.
Breakaway point occurs between two open-loops
s(s + 1)(s + 3) + K(s + 2) = 0; K > 0.  (i) poles say between s = 0 and s = −1. Therefore,

Chapter 6 (578-614).indd 605 3/23/2016 4:10:14 PM


606        CHAPTER 6:  CONTROL SYSTEM

SReal part of poles − SReal part of zeros Solution:  For step input ess = 0.1
Centroid = We know that
n−m
−2)
0 − 1 − 3 − (− 1
= = −1 ess =
2 1+ K
As N = 3, the root loci branches terminate at infin- 1
0. 1 =
ity along asymptotes, so +1 < Re(s) < 0. 1+ K
Ans. (c) ⇒K =9
69. The frequency response of a linear system G(jw) is Also,
provided in the tabular form below K 9
G(s) = =
s+1 s+1
|G(jw)| 1.3 1.2 1.0 0.8 0.5 0.3
From the figure, input
ÐG(jw) −130° −140° −150° −160° −180° −200°
r (t) = 10 [ u(t) − u(t − 1)]
The gain margin and phase margin of the system
are Taking Laplace transform
(a) 6 dB and 30° (b) 6 dB and −30°
(c) −6 dB and 30° (d) −6 dB and −30°  1 e−s   1 − e−s 
R(s) = 10  −  = 10  
(GATE 2011: 1 Mark)  s s   s 
 
Therefore, steady-state error for pulse input is,
Solution:  Gain margin = −20 log G( jw )
w =w pc

1 ess (p) = lim s ⋅ E(s)


= 20 log s→ 0
G( jw pc ) R(s)
= lim s ⋅
s→ 0 1 + G(s)H(s)
Phase margin = 180° + fgc
where fgc is the gain cross-over frequency or angle = lim s ⋅
R(s)
at wgc s→ 0 s + 100
From the given data, the frequency at which the s+1
response crosses −180° is 0.5. 10(1 − e−s )/s
= lim s ⋅
G( jw ) = 0.5, at ∠G( jw ) = −180° s→ 0 s + 10
1 s+1
Therefore GM = 20 log
0.5
= 6 dB ( ∵ R(s) = 10(1 − e−s ) and H(s) = 1)
Phase margin = 180 + fgc 10 (1 − e0 )
= =0
= 180° + (−150°) = 30° 10/1

Ans. (a) Ans. (a)

70. The steady state error of a unity feedback linear 71. A point z has been plotted in the complex plane, as
system for a unit step input is 0.1. The steady- shown in the following figure.
state error of the same system, for a pulse input
r(t) having a magnitude of 10 and a duration of Im
one second, as shown in the following figure is
Unit
r(t)
z · circle

Re
10
t
1s

(a) 0     (b) 0.1     (c) 1     (d) 10


1
(GATE 2011: 1 Mark) The plot of the complex number y = is
z

Chapter 6 (578-614).indd 606 3/23/2016 4:10:28 PM


SOLVED GATE PREVIOUS YEARS’ QUESTIONS        607

(a) Im y = 1/z
y > [∵ z < 1]
Unit
circle z − is positive real and imaginary part. Therefore,
`y’ should have positive real and negative imagi-
Re nary part
Ans. (d)

72. An open-loop system represented by the transfer
(s − 1)
function G(s) = is
(b) Im (s + 2)(s + 3)
(a) stable and of the minimum phase type.
Unit
(b) stable and of the non-minimum phase type.
circle
(c) unstable and of the minimum phase type.
Re
(d) unstable and of the non-minimum phase type.

y· (GATE 2011: 2 Marks)

Solution:  Given that


(c) Im
(s − 1)
G(s) =
Unit (s + 2)(s + 3)
· y circle
Zeros (m) =1; s = 1
Re Poles (n) = 2; s = −2; s = −3.
(i) All the poles lie on the left half of the s-plane,
therefore open-loop system is stable.
(ii) One zero lies on the right half of the s-plane,
therefore non-minimum phase type.
(d) Im Ans. (b)
73. The open-loop transfer function G(s) of a unity
Unit
feedback control system is given as
circle
 2
Re K s + 
 3
G(s) =
s2 (s + 2)
·y
From the root locus, it can be inferred that when K
tends to positive infinity,
(GATE 2011: 1 Mark) (a) three roots with nearly equal real parts exist on
the left half of the s-plane.
Solution:  Given (b) one real root is found on the right half of
s-plane.
(c) the root loci cross the jw axis for a finite value
Unit of K; K ≠ 0.

z circle (d) three real roots are found on the right half of
the s-plane.
y = 1/z
(GATE 2011: 2 Marks)

Solution:
K(s + 2/3)
Point z lies inside the unit circle hence the value G(s) =
of z < 1 s2 (s + 2)

Chapter 6 (578-614).indd 607 3/23/2016 4:10:35 PM


608        CHAPTER 6:  CONTROL SYSTEM

Sum of poles − Sum of zeros Overall transfer function


Centroid(s A ) =
No. of poles − No. of zeros 1
−2 − (−2/3)
= −2/3
Y (s) s(s + 1) + Ks 1
= = =
3 −1 R(s) 1 + 1 s(s + 1) + Ks + 1
(2k ± 1) × 180° s(s + 1) + Ks
Angle of asymptote q A = ;
n−m Y (s)
= 2
1
=
1
k = 0, 1 n − m R(s) s + s + Ks + 1 2
s + s(K + 1) + 1

180°
q1 =
Standard second-order characteristic equation is
For k = 0; = 90°
2 given by
3 × 180° s2 + 2zw n s + w n2 = 0
k = 1; q 2 = = 270°
2
Therefore,

w n2 = 1 2zw n = 1 + K

wn = 1 z =
1+ K
2

Therefore, peak overshoot = e−z p/1 −z


2

−2 −2/3
Ans. (a)
75. A system with transfer function

So, three roots with nearly equal real parts exist on (s2 + 9)(s + 2)
the left half of the s-plane. G(s) =
(s + 1)(s + 3)(s + 4)
Ans. (a)

74. A two-loop position control system is shown in the is excited by sin (wt). The steady-state output of
following figure. the system is zero at
(a) w = 1 rad/s (b) w = 2 rad/s
Motor (c) w = 3 rad/s (d) w = 4 rad/s
1
R(s) + + Y (s) (GATE 2012: 1 Mark)
− − s(s + 1)

Ks Solution:  The steady state error shall be zero


when the overall function is stable and the factor
w
Tacho-generator
due to sinusoidal input, that is, gets can-
s + w2
2
The gain K of the Tacho-generator influences mainly celled by zero at (s2 + 9). It implies that
(a) peak overshoot.
s2 + w 2 = s2 + 9 ⇒ w 2 = 9 ⇒ w = 3 rad/s
(b) natural frequency of oscillation.
(c) phase shift of the closed-loop transfer function
at very low frequencies (w → 0). Ans. (c)
(d) phase shift of the closed-loop transfer function
at very high frequencies (w → ∞). 76. The state variable description of an LTI system is
given by
(GATE 2011: 2 Marks)
 x1   0 a1 0  x1  0
      
Solution:  Solving for inner loop, transfer function is x2  =  0 0 a1  x2  + 0 u

       
1 x3  a3 0 0 x3   1
 x1 
s(s + 1) 1  
y = (1 0 0)x2 
=
1
1+ ⋅ K(s) s(s + 1) + K(s)
 
s(s + 1) x3 

Chapter 6 (578-614).indd 608 3/23/2016 4:10:52 PM


SOLVED GATE PREVIOUS YEARS’ QUESTIONS        609

where y is the output and u is the input, The


system is controllable for R(s) + K(s + 1) Y (s)
(a) a1 ≠ 0, a2 = 0, a3 ≠ 0 s3 + as2 + 2s + 1

(b) a1 = 0, a2 ≠ 0, a3 ≠ 0
(c) a1 = 0, a2 ≠ 0, a3 = 0
(a) K = 2 and a = 0.75
(d) a1 ≠ 0, a2 ≠ 0, a3 = 0
(b) K = 3 and a = 0.75
(GATE 2012: 2 Marks) (c) K = 4 and a = 0.5
(d) K = 2 and a = 0.5
Solution:  Given that
(GATE 2012: 2 Marks)
x1   0 a1 0 x1  0
      
x  =  0 0 a2  x  + 0 u − (1);
     2   
 Solution:  Overall transfer function of the system is,
x  a3 0 0 x  1 
2

3 3 Y (s) G(s)
=
 x1  R(s) 1 + G(s)H(s)
 
y = (1 0 0)  x2  =
K(s + 1)
(∵ H(s) = 1)
x  s + as2 + 2s + 1
3
 3 
K(s + 1)
1+ 3
For a 3 × 3 matrix to check the countability, the s + as2 + 2s + 1
condition is K(s + 1)
= 3 2
s + as + 2s + 1 + K(s + 1)
U = [B : AB : A2 B ]  (i)
K(s + 1)
= 3 2
We know that s + as + s(2 + K ) + (K + 1)
x = Ax + Bu  (ii)
By Routh array, the characteristic equation is
On comparing Eqs. (i) and (ii),
s3 + as2 + s (2 + k) + (k + 1) = 0
 0 a1 0 0
    s3 : 1 2+K
A =  0 0 2  B = 0
 a
a3 0 0   2
s : a K +1
  1 
a(2 + K ) − K + 1
0  s1 :
   0 0 a1a2 
  a

AB = a2  A = a2a3 
0
2
   0 s0 : K +1
0   0 a3a1 0
For an oscillatory system, we have s1: 0 which
0 0 a1a2  0 a1a2 
implies
      a(2 + K ) − K + 1
A2B = a2a3 0 0  0 =  0  =0  (i)
   
     a
0 a3a1 0  1   0  (K + 1)
a=
0 a1a2 
 0

(K + 2)
0 

U = 0

a2
 Auxiliary equation is,
1 0 0  (K + 1)
as2 + K + 1 = 0 ⇒ s2 = −
a
This implies that a1 ≠ 0, a2 ≠ 0 and a3 may or may
not be zero. Substituting value of a from Eq. (i), we have
Ans. (d)
(K + 2)
s2 = −(K + 1) ⋅ = −(K + 2)
77. The feedback system shown in the following figure (K + 1)
oscillates at 2 rad/s when s= j K +2  (ii)

Chapter 6 (578-614).indd 609 3/23/2016 4:11:05 PM


610        CHAPTER 6:  CONTROL SYSTEM

df 1/a 1/b
Substituting s = jw in Eq. (ii), we get = − =0
dw w  2
1 + (w /b)2
jw = j K + 2 1  
a
w = K +2 1/a 1/b
=
2 = K +2 ⇒ K = 2 1 + (w /a)2 1 + (w /b)2

K +1 2 +1 3 1   w 2  1 
  = 1 +  w  
 2 
a   b   b   a  
Therefore, a = = = = 0.75 1 +
K +2 2+2 4    
Ans. (a) 1 1  w 2  1 1  w 2 
+  = +  
Statement for Linked Answer Questions a a  b2  b b  a2 
78 and 79: The transfer function of a compensa-
 1 1  w 2  1 1 
 −  =  −
 a b  ab  a b 
tor is given as
s+a
Gc (s) =
s+b
w 2 = ab ⇒ w = ab = 1 × 2 = 2 rad/s
Ans. (a)
78. Gc(s) is a lead compensator if
(a) a = 1, b = 2 (b) a = 3, b = 2 80. The transfer function V2 (s) of the circuit shown in
(c) a = −3, b = −1 (d) a = 3, 6 = 1 V1(s)
the following figure is
(GATE 2012: 2 Marks)
100 µF
Solution:  Substituting s = jw in the given trans-
+ +
fer function, we get
10 kΩ
jw + a
Gc ( jw ) = V1(s) V2(s)
jw + b 100 µF
For a lead compensator f > 0 − −

w w
f = tan−1 − tan−1  (i)
a b 0.5s + 1 3s + 6
(a)  (b) 
 w w 
s+1 s+2
 − 

−1 
= tan  a b  = tan−1 w (b − a) s+2 s+1
 2
w   w + ab 
(c)  (d) 
 2 s+1 s+2
1 + 
 ab  (GATE 2013: 1 Mark)
f > 0 implies that
Solution:  The given circuit can be represented as
w(b − a) > 0
b−a > 0 ⇒ b > a C1 100 µF
+ +
This condition is satisfied by options (a) and (c). R
However, in case of lead compensator, zero is nearer 10 kΩ
to the origin as compared to pole, therefore option V1(s)
C2 V2(s)
(c) is not valid and the correct option is (a). I(s)
Ans. (a) 100 µF
79. The phase of the above lead compensator is maxi- − −
mum at
(a)  2 rad/s (b)  3 rad/s For loop 1: Let C1 = C2 =100 mF = C

1
(c)  6 rad/s (d)  rad/s 1 1
3 V1(s) = I (s) + R.I (s) + I (s)
C1s C2s
(GATE 2012: 2 Marks)
 1 1 
V1(s) = I (s)  +R+   (i)
Solution:  From Eq. (i) of Question 78, we have  Cs Cs 

Chapter 6 (578-614).indd 610 3/23/2016 4:11:29 PM


SOLVED GATE PREVIOUS YEARS’ QUESTIONS        611

w = K 1/N
For loop 2:

1 6.309 = K 1/2 (∵ N = 2)
V2 (s) = R I (s) + I (s)
Cs K = 39.8
 1  V2 (s)
V2 (s) = I (s) R +  ⇒ I (s) =  (ii) 39.8
 Cs  (R + 1/CCs)
G(s) = 2
s

Ans. (b)
Substituting Eq. (ii) in Eq. (i), we get
82. The signal flow graph for a system is given below.
V (s) ⋅ Cs  RCs + 2 
V1(s) = 2   1
Rcs + 1  Cs 
U(s) 1 s−1 s−1 1 Y(s)
V2 (s) 10 × 103 × 100 × 10−6 s + 1 s+1
= −
= −4
V1(s) 10 × 10 × 100 × 10 s + 2 s + 2
3 6
−2
Ans. (d)
81. The Bode plot of a transfer function G(s) is shown Y (s)
The transfer function for this system is
in the following figure. U (s)
s+1 s+1
(a)  2 (b)  2
5s + 6s + 2 s + 6s + 2
40 s+1 1
(c)  2 (d)  2
32 s + 4s + 2 5s + 6s + 2
Gain (dB)

20 (GATE 2013: 2 Marks)

Solution: Using Mason's gain formula,


0
−8
1 10
100 Y (s) s−2 + s−1
=
ω (rad/s) U (s) 1 − [−2s−2 − 4s−1 − 2s−1 − 4] + 0
s−2 (1 + s)
The gain (20 log|G(s)|) is 32 dB and -8 dB at
=
1 + 2s−2 + 6s−1 + 4
s−2 (s + 1)
1 rad/s and 10 rad/s, respectively. The phase is
negative for all w, then G(s) is =
s−2 (5s2 + 6s + 2)
39.8 39.8 32 32
(a)      (b)  2     (c)      (d)  2 s+1
s s s s =
5s2 + 6s + 2
(GATE 2013: 1 Mark)
Ans. (a)
Solution:  Any two points on same line segment of Common Data for Questions 83 and 84: The
Bode plot satisfies the equation of a straight line. state variable formulation of a system is given as
 x1  −2 0  x1  1
The slope of straight line is
 = 
 x   0 −1  x  1
+ u,
G2 − G1  2  1  
= −40 dB/decade
log w 2 − log w1 x1(0) = 0 x 
and y = [1 0 ]  1 
0 − 32 x 
= −40 x2 (0) = 0  2
 w 

log  2
 w1  83. The system is

0 − 32 (a) controllable but not observable.


= −40 (Set w 2 = w )
w  (b) not controllable but observable.
log  
1 (c) both controllable and observable.
−32 = −40 log(w ) ⇒ w = 6.309 (d) both not controllable and not observable.

We know that (GATE 2013: 2 Marks)

Chapter 6 (578-614).indd 611 3/23/2016 4:11:51 PM


612        CHAPTER 6:  CONTROL SYSTEM

Solution: On inverse Laplace transform, we get


−2 0 1 e−2t
A=  B=  0 
 0 − 1 1
C = [1 0] L−1[f(s)] =  
     0 e−t 
To check for controllability: e2x 0 
e−Ax =  
 x = −t ]
[using x

AB = −2 0 1 = −2  0 ex 
 0 −1 1 −1  e2 x 0  1 e2x 
e−Ax ⋅ B =    =  
  x 1  x 
[BAB ] = 1 −2 ≠ 0  0 e    e 
1 −1
t t e2x 
 
Therefore, system is controllable. ∫ e−Ax ⋅ B(x) = ∫  x  ⋅ dx.
To check for observability: 0 0
e 
 0 = [−2  e2 t − 1 
CA = [1 0 ] −2  
 
0]
 0 −1 = 2 
C   1 0   et − 1 
 = =0  
CA −2 0  
   1 
Therefore, the system is not observable. Therefore,
Ans. (a)
 e2 t − 1 
84. The response y(t) to a unit step input is  
 2 
x(t) = eAt  
(a)  − e−2t
1 1
(b) 1 − e−2t − e−t
1 1  et − 1 
 
2 2 2 2  
 1 

(c) e−2t − e−t (d) 1 − e−t  e2 t − 1 

e−2t  
0   2 
=    
(GATE 2013: 2 Marks)
 0 e−t   et − 1 
 
 
Solution:  Solution for the given state equation in  1 
time domain is,  1 − e−2t 
 
 
 
t 2
x(t) = eAt x(0) + eAt ∫ e−Ax BU (x)dx =
 1 − e−t 
 
0  
 1 
Given x(0) = 0. We know that
Therefore,
eAt = L−1 [sI − A ]−1
eAt = L−1 [sI − A ]−1 y(t) = [1 0]x(t)
s 0 −2 0 s + 2 0   1 − e−2t 
sI − A = s 0 − −2 0  = s + 2 0   
sI − A =  0 s −  0 − 1 = 0 s + 1
 0 s  0 − 1 0 s + 1 = [1 0]  

     2
−1 s + 1 0 1 − e−t 
[sI − A ] =
1 
 0   
[sI − A ]−1 = (s + 1)(s + 2)  0
s + 1
s + 2
1
(s + 1)(s + 2)  0 s + 2 = − e−2t
1 1
 1 
 1 0  2 2
= s + 2 0 
= s + 2  Ans. (a)
 0 1/(s + 2)
 0 1/(s + 2)

Chapter 6 (578-614).indd 612 3/23/2016 4:12:06 PM


SECTION VII: ELECTRICAL AND ELECTRONIC MEASUREMENTS

MARKS DISTRIBUTION FOR GATE QUESTIONS

4
Number of questions

Marks 1
2 Marks 2
Total number of questions
1

0
2009 2010 2011 2012 2013 2014 2015

TOPIC DISTRIBUTION FOR GATE QUESTIONS

Year Concepts
2015 Measurement of basic electrical quantities, Electronic measuring instrument
2014 Measuring instruments
2013 PMMC, Moving iron, AC bridges
2012 PMMC, Bridges, Multiplier
2011 Wattmeter, AC bridge, Error analysis
2010 Wattmeter, Ammeter, AC bridge
2009 Dynamometer, Oscilloscope, Wattmeter

Chapter 7 Theory.indd 613 3/23/2016 1:21:19 PM


Chapter 7 Theory.indd 614 3/23/2016 1:21:19 PM
CHAPTER 7

ELECTRICAL AND ELECTRONIC MEASUREMENTS

Measurement techniques have played a significant role   1.  Primary (or absolute) and secondary type:
from the starting from fair exchange of goods in early Instruments that measure the absolute physical
civilizations to regulation of trade in industrialised societ- quantity directly in terms of the constants of the
ies. Better measurement and instrumentation techniques instrument and the deflection are called primary or
evolved as production of goods became industrialised absolute instruments (e.g., tangent galvanometer).
and advent of computers saw their enormous applica- If the actual value of the quantity being measured
tion to measurement, process control and monitoring. In is proportional to some other absolute value of the
this chapter, we will discuss the instruments used com- quantity, the instrument is called secondary. The
monly for electrical and electronic measurements and instrument is pre-calibrated using the absolute
about their error analysis. These include instruments for instrument (e.g., voltmeter, ECG recorder, etc.)
measurement of voltage, current and power; instrument   2.  Active and passive type: Instruments that can
transformers; oscilloscopes and transducers. be directly used for the quantity being measured
are known as the active-type. If the quantity being
measured simply modulates the magnitude of some
7.1  CLASSIFICATION OF MEASURING
external power source the instrument is known as
INSTRUMENTS passive type.
  3.  Deflection and null type: In a deflection-type
instrument, the physical effect generated by the
Instruments can be classified based on their mode of
quantity being measured, produces an equivalent
operation, manner of energy conversion, measuring tech-
opposing effect in some other part of the instru-
niques and kind of output signal. The main instrument
ment, which in turn causes deflection (or mechanical
types are discussed as follows.

Chapter 7 Theory.indd 615 3/23/2016 1:21:19 PM


616     Chapter 7:  ELECTRICAL AND ELECTRONIC MEASUREMENTS 

displacement) which is a measure of the quantity. where W is the control weight put at a distance l
In the null-type instrument the physical effect gen- from the axis of rotation (spindle) of the moving
erated by the physical quantity under measure- system and kgr is the gravity constant.
ment is nullified by either a manual or automatic Note: In instruments with spring control gradu-
balancing device. The equivalent null causing effect ated scales are used and cramped scales are used
is the measure of the quantity. for gravity control instruments.
  4.  Analog and digital type: In analog instrument,
the physical quantities under measurement show   3.  Damping torque: In an instrument, the combined
continuous (step-less) variation with time. In digi- effect of deflecting and controlling torques on the
tal instruments, the physical quantities are discrete movement of the pointer on the scale, causes it to
and vary in steps with time. oscillate when indicating the final reading. These
oscillations are prevented by using a damping mech-
Based on the mode of operation, the secondary instru- anism, either by generating air or fluid friction or by
ments are further classified into three types: action of eddy currents. This torque is proportional
  1.  Indicating type: In this category the measur- to the angular velocity of the moving system and
ing instrument indicates the quantity being mea- hence operated only when the system is in motion.
sured through a pointer or some type of indicator. dq
Majority of the measuring instruments fall under Tdamp = kdamp
dt
this category, for example, voltmeter, ammeter, etc.
  2.  Integrating type: In this category of measuring The effect of damping on deflection (q ) is depicted
instruments, the measurement is done with the in Fig. (7.1), where graphs I, II and III represent
help of integrating device or arrangement over a under-damped, critically damped and over-damped
period of time. For example, in the case of energy instruments, respectively.
meter the rotation of disc over a period of time
I
gives the reading of the energy consumed.
  3.  Recording type: In this category, the measur-
ing instrument is used to record certain quantities q
to be used for analysis. For example, plot chart
recorder, ECG, EEG, etc. II
Deflection
III
7.1.1  Indicating Type Instruments

The different types of torques that function in any indi-


cating measuring instruments are: Time
  1.  Deflecting torque: This torque, also called oper- Figure 7.1 |   Effect of damping on deflection.
ating torque, is developed by the magnetic, electro-
static, chemical or thermal effects produced by the
quantity to be measured. 7.2  TYPES OF INDICATING
Td ∝ Operating quanity
INSTRUMENTS
  2.  Controlling torque: The instruments are so
designed that the controlling torque acts on its The basic components of all indicating instruments
moving part. It can (i) control/stop the move- include:
ment of the pointer beyond the desired reading   1.  Support for moving system: This can be achieved
and (ii) bring the pointer back to its zero position, either by pivoting or suspension.
when the operating quantity is removed. The con-   2.  Permanent magnets: These should have constant
trolling torque (Tc) is usually obtained either by a strength over a period of time.
spring control.   3.  Pointers and scales: The pointers should be light
Tc ∝ q (deflection) ⇒ Tc = kcq in weight with low constant of inertia. A strip of
mirror is mounted on the scale beneath the pointer
where kc is the spring constant or by gravity ­control and reading is taken after removing the parallax
mechanism error between the pointer and its mirror image.
  4.  Cases: These are the outer covering of the instrument
Tc ∝ Wl sin q ⇒ Tc = kgr Wl sin q
and should be made-up of non-magnetic material.

Chapter 7 Theory.indd 616 3/23/2016 1:21:25 PM


7.2  TYPES OF INDICATING INSTRUMENTS     617

Depending on the mode of operation of the permanent proportional to the measured quantity, that is, volt-
magnet, the indicating type instruments can be classified age or current. This electromagnetic torque is counter
into the following types: balanced by the mechanical torque of control springs
  1.  Moving coil-type instruments: This is further (bronze hair springs) attached to the movable coil. The
categorised into: coil is wound on an aluminium former which moves in
(i) Permanent magnet moving coil: This can the magnetic field of the permanent magnet to provide
be used for direct current and voltage eddy current damping. When the torques are balanced,
measurements. the pointer attached to the moving coil will stop and its
(ii) Dynamometer type: This can be used either angular deflection will represent the amount of electri-
directly or through alternating current and cal current to be measured against a fixed reference or
voltage measurements. scale. The light weight pointer is carried by the spin-
  2.  Moving iron-type instruments: This can be dle and it moves over this graduated scale. The scales
used for AC/DC current and voltage measurement. of the PMMC instruments are usually linearly spaced
as the deflecting torque and hence the pointer deflections
These instruments are discussed in detail in the follow- are directly proportional to the current passing through
ing sections. the coil. The scale and pointer on the pivot are depicted
in the top view of PMMC in Fig. 7.3.
7.2.1  Permanent Magnet Moving Coil Instruments

Figure 7.2 shows the construction of a permanent Scale


magnet moving coil instrument. The instrument has a Pivot
moving coil of fine wire (circular or rectangular), with
N-turns suspended in the uniform, horizontal and radial
magnetic field of a permanent magnet in the shape of a Pointer
horse-shoe. It is free to turn about its vertical axis. The
coil with is placed around an iron core, which is spheri- Balance
Control weight
cal if the coil is circular, and is cylindrical if the coil is
spring
rectangular. Since the coil is moving and the magnet is
permanent, the instrument is called permanent magnet
moving coil or a PMMC instrument.

Figure 7.3 |   Top view of PMMC instrument.


Scale
The electromagnetic torque is equal to the multiplication
Control
spring of force with distance to the point of suspension. The
total deflection torque is given by
Pointer Permanent
magnet Td = NBIlb = NBIA
S
This torque will cause the coil to rotate until an equilib-
rium position is reached at an angle q with its original
N
orientation. At this position,
Rotating coil of N turns Electromagnetic torque = Control spring torque
Stationary iron core
Td = Tc
Figure 7.2 |   Permanent magnet moving coil
instrument. NBIA = kcq
When the current is passed through the coil it pro-  
q=
NBIA
⇒ q = KI where K = NBA 
duces another magnetic field and the interaction of this
 
field with the magnetic field of the permanent magnet kc kc 
produces an electromagnetic torque. The amount of
force experienced by the coil is proportional to the The angular deflection is thus linearly proportional to
current passing through the coil which again becomes the current I.

Chapter 7 Theory.indd 617 3/23/2016 1:21:32 PM


618     Chapter 7:  ELECTRICAL AND ELECTRONIC MEASUREMENTS 

The advantages of PMMC instruments are listed as follows: and move towards the coil. The spindle is rigidly con-
  1.  Uniform scale. nected to the pointer, controlling weight, moving iron
  2.  Accurate and reliable. and to the piston. The repulsion-type MI instrument
  3.  High sensitivity. consists of two cylindrical soft iron vanes mounted
  4.  Free from hysteresis error and not affected by within a fixed current carrying coil. One iron vane is
external (stray) magnetic fields. kept fixed to the coil frame and other, attached to the
  5.  Simple and effective damping mechanism. pointer shaft, is free to rotate. Two irons lie in the mag-
  6.  Low power consumption. netic field produced by the coil and current in the coil
  7.  Extension into multirange instruments possible. makes both vanes to become magnetised with the same
polarity. The repulsion between the similarly magne-
7.2.2  Moving Iron Type Instruments tised vanes produces a proportional deflection of the
pointer. In MI type instruments, rotation is opposed
The moving iron (MI) type instruments can measure by a hairspring that produces the restoring torque. The
AC signals at frequencies up to 125 Hz, in addition to damping is achieved by air or fluid friction damping.
DC signals. In these instruments, the signal (current) For an excitation current I carried by the stationary
to be measured is allowed to flow through a station- coil, the torque produced that causes the iron disc to
ary coil, which produces a magnetic field proportional move inside the coil is given by
to the quantity to be measured. The moving iron piece
(made of soft iron) is fixed with the moving system I 2dM
Td =
(a spindle and pointer), gets attracted/repelled pro- 2dq
portionately and gives reading on the calibrated scale. where M is the mutual inductance between the coil and
These instruments are accordingly classified as attrac- the iron disk and q is the angular deflection of the vane.
tion or repulsion type. Figure 7.4(a) shows the schematic Rotation is opposed by a spring connected with the vane
of attraction type moving iron type instrument and that and the pointer which produces a backwards torque
for the repulsion type is shown in Fig. 7.4(b). given by,
Cramped scale
Ts = kq
At equilibrium, the deflecting and the controlling torques
are equal, that is, Td = Ts and therefore the deflecting
Fixed angle q is given by
coil Pointer
I 2 dM
q=
2k dq
Spring
Thus the instrument has a square-law response where
Moving iron
the deflection is proportional to the square of the
disc signal being measured. Thus, the output reading is a
root-mean-squared (rms) quantity. As the deflecting
(a) torque is proportional to the square of the coil cur-
rent, these type of instruments possess scales that
are non-linear and cramped in the lower range of
calibration.
Coil Scale The advantages of MI-type of instruments are as follows:
  1.  Suitable for AC and DC circuits.
  2.  Simple construction and low cost.
  3.  Measures voltage in the range of 0−30 V but a
series resistance can be inserted in the circuit to
Pointer measure higher voltages.
  4.  Accuracy is high.
Spring
Moving iron piece (vane)   5.  Frictional error is less as torque/weight ratio is high.

(b)
Figure 7.4 |   Moving-iron meter: (a) Attraction type
7.2.3  Electrodynamic Type Meters
and (b) repulsion type.
Electrodynamic type meters, also known as dynamom-
The attraction-type moving-iron meter, the moving iron eter type meters, can measure both DC as well as AC sig-
disc (vane) placed near the coil is free to get attracted nals up to a frequency of 2 kHz. The schematic diagram

Chapter 7 Theory.indd 618 3/23/2016 1:21:38 PM


7.3  BRIDGES AND POTENTIOMETERS     619

for dynamometer type instrument is shown in Fig. 7.5. the moving-iron type instrument. The limitation of low
The instrument has a moving circular coil which is placed permissible frequency can be overcome, to an extent, by
in the magnetic field produced by two circular stationary rectifying the voltage signal and then applying it to a
coils which are wound separately and connected in series. moving-coil meter, as shown in Fig. 7.6.

Bridge rectifier
Scale
Moving-coil
Pointer meter

Moving coil
Fixed coils Figure 7.6 |   Measurement of high-frequency
voltage signals.

Figure 7.5 |   Schematic representation of electrodynamic


The circuit with bridged rectifier extends the upper limit
of measurable frequency to 20 kHz but makes the mea-
meter.
surement more sensitive to change in temperature of the
The deflection torque in this type of wattmeter is pro- environment and resulting non-linear behavioursignifi-
duced by the interaction of two magnetic fluxes. One of cantly impacts measurement accuracy for voltages. An
the fluxes is produced by a fixed coil, called current coil, alternative method to overcome the low frequency limit
which carries a current proportional to the load current. is provided by the thermocouple meter.
The other flux is created by the moving coil, called the
voltage or potential coil, which carries a current pro- 7.3  BRIDGES AND POTENTIOMETERS
portional to the load voltage. The deflecting torque is
dependent upon the mutual inductance between the two
coils and can be given by The AC bridge networks are used for the measurement
dM of inductance and capacitance in the circuits. These are
Td = I1I2
dq modified form of Wheatstone bridge; consisting similarly
of four arms, an excitation source and balance detector.
where I1 and I2 are the currents flowing in the fixed and
moving coils, M is the mutual inductance and q represents
DC bridges along with potentiometers are used for the
measurement of resistance. This is depicted in the flow
the angular displacement between the coils. The torque
chart shown in Fig. 7.7.
is thus proportional to square of the current. If the mea-
sured current is alternating, the meter is unable to follow
the alternating torque values and instead displays the
mean value of square of the current. The squared or rms Bridges
value of the measured current (or any other quantity)
can be obtained by suitable modification of the scale.
The advantages of electrodynamic type meters are as follows:
  1.  More accurate than moving-coil and moving-iron
instruments but expensive. AC Bridges DC Bridges
  2.  Voltage, current and power can all be measured by
suitable connections of fixed and moving coils.
  3.  Used to measure voltages in the range of 0−30 V
but can be modified by placing a series resistance Inductance (Resistance
Capacitance measurement)
to measure higher voltages. measurement
measurement Wheatstone
Maxwell bridge
Hay’s bridge De Sauty’s bridge bridge
7.2.4  Measurement of High-Frequency Signals Owen’s bridge Schering bridge Kelvin double
Anderson’s bridge Wein’s bridge bridge
In the instruments discussed in the sections above, the
maximum frequency limit is of the order of 2 kHz for the Figure 7.7 |   Bridges for measurement of inductance,
dynamometer type meters and only 100 Hz in the case of capacitance and resistance.

Chapter 7 Theory.indd 619 3/23/2016 1:21:41 PM


620     Chapter 7:  ELECTRICAL AND ELECTRONIC MEASUREMENTS 

The general form of an AC bridge under balance condi- IC I1


tion is shown in Fig. 7.8, where all four arms are consid-
ered as impedance (frequency dependent components). If
Z1, Z2, Z3 and Z4 are the impedance of AC bridge arms, e
I1R1
then at balance point:
I4wL1
EBA = EBC
I1Z1 = I2Z2
V V
I1 = and I2 =
Z1 + Z3 Z2 + Z4
I4R4 e3 = I3R3 I3
The complex and polar form of equations are:
(b)
Figure 7.9 |   Maxwell’s bridge (a) Circuit diagram.
Z1Z4 = Z2Z3  (7.1)

Z1Z4 (∠q1 + ∠q 4 ) = Z2Z3 (∠q2 + ∠q3 )  (7.2) (b) Phasor diagram.

The construction of Maxwell’s bridge shows:
B
  1.  One arm consisting of a capacitor C1 in paral-
Z1 Z2 lel with a resistor R2. Both these variables have
adjustable values.
  2.  The opposite arm consisting of an inductor L1 in
V A D C series with a resistor R4. Both these variables are
unknown values and need to be measured.
Z3 Z4   3.  The other two arms consist of resistors R1 and R3.,
for which the values are known.
D
The Maxwell bridge measures inductance after adjust-
Figure 7.8 |   AC bridge under balance condition. ment of C1 and R2 such that the current through the
bridge between points A and B becomes zero, which
7.3.1  Measurement of Inductance occurs when the voltages at points A and B are equal.
This is known as balancing of circuit.
Commonly used bridges for measurement of inductance When the Maxwell bridge is balanced, the impedances
are Maxwell’s bridge, Maxwell—Wien bridge and Hay’s can be written as
bridge; other’s include Owen’s bridge and Anderson bridge.
Z1 R
= 3 (7.3)
7.3.1.1 Maxwell’s Inductance-Capacitance R1 Z2

Bridge
where Z1 is the impedance of resistor R2 in parallel with
The Maxwell bridge is used to measure unknown induc- capacitor C2, and Z2 is the impedance of inductor L1 in
tance in terms of calibrated resistance and capacitance. series with resistor R4. Thus, from Eq. (7.3), the relation
The circuit arrangement for Maxwell’s bridge is shown can be mathematically represented as:
in Fig. 7.9(a) and the corresponding phasor diagram is
and Z2 = R4 + jwL1 
1
Z1 = R2 + (7.4)
shown in Fig. 7.9(b). jwC1

Here w = 2pf
C1 R3 Thus, when the bridge is balanced,

R2 + 1 / ( jwC1 ) R3
=
A B R1 R4 + jwL1
L1 or
R1 R1R3 = R2 + 1 / ( jwC1 ) R4 + jwL1 
R4
When the bridge is balanced, the negative and positive
(a) reactive components cancel out, so

Chapter 7 Theory.indd 620 3/23/2016 1:21:53 PM


7.3  BRIDGES AND POTENTIOMETERS     621

R1R3 When the Hay’s bridge is balanced, the impedances can


R1R3 = R2R4 ⇒ R4 = (7.5) be written as
R2
Z1 R
From the value of R4 determined by Eq. (7.5), L1 can be = 3 (7.6)
R2 Z4
determined using Eq. (7.4).
where, Z4 is the impedance of the arm containing C4 and
7.3.1.2 Hay’s Bridge R4 while Z1 is the impedance of the arm containing L1
and R1.
A Hay’s bridge is another AC bridge circuit, which is a
modification of Maxwell’s bridge. Figure 7.10 shows the Thus, from Eq. (7.6),
circuit diagram of the Hay’s Bridge. It can be used for
and Z1 = R1 + jwL1
1
measuring an unknown inductance by balancing its four Z4 = R4 +
arms, one of which contains the unknown inductance. jwC4
One of the arms of a Hay’s Bridge has a capacitor of
known value, which is the principal component that is Mathematically, when the bridge is balanced,

R1 + jwL1
used to determine the unknown inductance value.
R3
=
R2 R4 +1 / ( jwC4 )
e1 B
I1 e3
R1 + jwL1  =
L1 R3R2
R3 or,   R + 1 / ( jwC )
R1 4 4
I1
A C R1 L
D or, R2R3 + = R1R4 + jwL1R4 + 1  (7.7)
jwC4 C4
R2 C4
R4 Equating the real and imaginary terms for the Eq. (7.7),
I2 I2 we get
D
L1
e2 e4 R2R3 = R1R4 +  (7.8)
C4

e R1
wL1R4 = or  R1 = (w)2 L1R4C4 . (7.9)
(a) wC 4

I2 Substituting for R1 from Eq. (7.9) in Eq. (7.8), we get

e1 = e2 e R2R3C4
L1 =
(wR4C4 )
2
I1wL1 1 +
and
I2R4
90° (wC4 ) 2R2R3R4
I1R1 e3 = e4 I1 R1 =
1 + (wR4 C4 )
2

I2/wC4 These expressions can be used to find R1 and L1.


(b)
Figure 7.10 |   Hay’s bridge. (a) Circuit diagram. 7.3.1.3 Owen’s Bridge
(b) Phasor diagram.
The circuit for Owen’s bridge is shown in Fig. 7.11. The
The construction of Hay’s bridge is as follows: construction of Owen’s bridge is as follows:
  1.  One arm of the bridge consists of a capacitor C4 in   1.  One arm consists of a capacitor C2 in series with a
series with a resistor R4. The resistor R4 and C4 are resistor R2.
both adjustable.   2.  Second arm consists of an inductor L1 in series with
  2.  The second arm consists of an inductor L1 in series a resistor R1. These are the values to be determined.
with a resistor R1. These are the unknown values.   3.  Third arm contains a known capacitor C4.
  3. The other two arms contain known resistors R2 and R3.   4.  The fourth arm just contains known resistor R3. 

Chapter 7 Theory.indd 621 3/23/2016 1:22:07 PM


622     Chapter 7:  ELECTRICAL AND ELECTRONIC MEASUREMENTS 

B Equating real and imaginary parts, we have


e1 e3
I1 L1 = R2R3C4
L1 R3
and
R1 I1 C4R3
R1 =
A C C2
D

R2 7.3.1.4 Anderson Bridge


C4
C2 Anderson bridge is a modified version of Maxwell bridge,
I2 I2 used for measurement of unknown value of inductance
D by comparison with known values of electrical resis-
tance and capacitance. The circuit for Andersen bridge
e2 e4 is shown in Fig. 7.12. The construction of the bridge is
as follows:
e   1.  One arm contains the unknown inductor L1 con-
nected with resistance R1 (purely resistive).
(a)   2.  The other three arms consist of resistances R2, R3
and R4 (purely resistive).
I2   3.  A standard capacitor C is connected in series with
variable resistance r and this combination is con-
nected in parallel with arm CD.
I2/wC2
e
B
I1wL1 I1
R1
R3
I1 L1 D
A IC I3 C
I1R1 e3 = I1R3 = I2/wC4
r C
(b) I4
R2
Figure 7.11 |   Owen’s bridge. (a) Circuit diagram.
R4
I2
(b) Phasor diagram. D

When the bridge is balanced,

Z1 R (a)
= 3
Z2 Z4
e1
where Z2 is the impedance of C2 and R2 and Z1 is the e
impedance of the arm containing L1 and R1, and Z4 is I1wL1
the impedance of the arm containing C4.  IC I2 e2 = I2 R2
Then e4 = I4 R4
1 ICr I4
Z4 = IC r
jwC4
I1
1 I1(R1 + r) e3 = I1R3 = IC/w C
Z2 = R2 +
jwC2
= R1 + jwL1
(b)
Z1
Figure 7.12 |   Andersen bridge. (a) Circuit diagram.
When the bridge is balanced, (b) Phasor diagram.

1 / j (wC4 ) R3 When the bridge is balanced


=
R2 + 1 / j (wC2 ) R1 + jwL1 I1 = I3 and I2 = IC + I 4  (7.10)

Chapter 7 Theory.indd 622 3/23/2016 1:22:20 PM


7.3  BRIDGES AND POTENTIOMETERS     623

IC e3, e4
I1R3 =
jwC e1 = e2
I1(R1 + R + jwL1 ) = I2R2 + ICR e

 1 
(b)
IC R +  = (I2 − IC )R4
Figure 7.13 |   De Sauty’s bridge. (a) Circuit diagram.
 jwC 
(b) Phasor diagram.
Substituting for IC from Eq. (7.10), we have It measures an unknown capacitance by comparing it
I1(R1 + jwL1 − jwCR3R) = I2R2 
with a known standard capacitance. Two ratio arms of
(7.11)
this bridge consist of non-inductive resistors (R1 and R2)
And and two consist of capacitors (C1 and C2) where one is of
unknown value and another is standard capacitor. If C1
I1( jwCR3R + R3 + jwCR3R4 ) = I2R4  (7.12) is the capacitor whose capacitance is to be measured and

C2 is the standard (known) capacitance, then the circuit
From Eqs. (7.11) and (7.12), we have is balanced is by varying either R1 or R2. In the balanced
circuit, B and D are at the same potential. Then
I1(R1 + jwL1 − jwCR3R)
I1R1 = I2R2  (7.13)
R R jwCR2R3R 
= I1  2 3 + + jwCR2R3  −j −j
 R R  and ⋅ I1 = ⋅I  (7.14)
4 4
wC1 wC2 2

Equating real and imaginary parts we have, Dividing Eqs. (7.13) and (7.14) we get
R1 C R
R2R3 R = 1 ⇒ C1 = C2 1
R1 = and L1 = C 3 [R(R2 + R4 ) + R2R4 ] R2 C2 R2
R4 R4
The bridge has maximum sensitivity when C1 = C2. The
method is simple in construction and use but perfect balance
7.3.2  Measurement of Capacitance is difficult to achieve if the capacitors show dielectric loss.

The AC bridges commonly used for measurement of 7.3.2.2 Schering Bridge


capacitance are De Sauty’s bridge and Schering bridge.
These are also based on the principle of Wheatstone The Schering bridge is used to measure unknown electri-
bridge and have two arms; one of which has the unknown cal capacitance and its dissipation factor. The dissipa-
capacitance to be determined. tion factor of a capacitor is the ratio of its resistance to
its capacitive reactance. The circuit for a Schering bridge
is shown in Fig. 7.14.
7.3.2.1 De Sauty’s Bridge
e1 B e3
The De Sauty’s bridge is a modified form of the I1
Wheatstone bridge with the DC source replaced by an R1 R3
AC source. Figure 7.13 shows the circuit for De Sauty’s
bridge. C1 I1
A D C
B
e1 C4
e3
R1 C1 C2 IC
R4
D I2
A C I4
D
R2 C2 e2 e4
e4 e2
D
e e
(a)

(a) Figure 7.14 |   Schering bridge. (a) Circuit diagram.


(b) Phasor diagram.

Chapter 7 Theory.indd 623 3/23/2016 1:22:34 PM


624     Chapter 7:  ELECTRICAL AND ELECTRONIC MEASUREMENTS 

IC I2
7.3.2.3 Wien’s Bridge
I1R1 I4 Wien’s bridge is used for measurement of unknown
I1
e3 = I1R3 e4 = IC/wC4 = I4R4 capacitance and also frequency. The circuit for the bridge
d is shown in Fig. 7.15. The construction is as follows:
  1.  One arm consists of a capacitor C2 in series
e with a resistor R2 These are the quantities to be
I1/wC1
e1 = e2 = I2/wC2 determined.
  2.  The second arm consists of a capacitor C1 in par-
(b) allel to a resistor R1.  Both these quantities are
Figure 7.14 |   (Continued). adjustable.
  3.  The other two arms consist of one known resistors
  1.  One arm consists of a known capacitor (C1) con-
each R3 and R4 respectively. 
nected in parallel to a known resistance (R1). This
is used as one of the variable arms of the bridge,
where R1 and C1 are adjusted until the current e1 e3
through the ammeter between points a and b C1
becomes zero. I1
  2.  Capacitor with known capacitance C2 and resistor R3
IC R1
with known resistance R3 are present on the other
two arms. I1
IR
  3.  The fourth arm has the unknown capacitor C4 with Coupling D
a resistor (R4) connected in series (or parallel).
I2 R4
Under the balanced condition, we have
R2 C2 I4
Z1 R
= 3
C2 Z4 I2
e2 e4
where, Z4 is the impedance of R4 in parallel with C4
and Z1 is the impedance of R1 in series with C1.
The impedances Z4 and Z1 can be written as,
e
R4 R4
Z4 = =
jwC4 (1 / jwC4 ) + R4  (1 + jwC4R4 ) (a)

1 IC
and Z1 = + R1
jwC1
Thus, when the bridge is balanced: e3 = I1R3
 1  90° = e4 = I2R4
 
R4 R3
 jwC + R1  (1 + jwC R ) = jwC I2R2 I1 I2
1 4 4 2

jR4 jR RRC
R1R4 − =− 3 + 3 4 4  (7.15) IR
wC1 wC2 C2

Equating the real and imaginary parts of Eq. (7.15), we get
e1 = e2 = I1R1 e
I2/wC2
C4R3
R1 = (b)
C2
Figure 7.15 |   Wien’s bridge. (a) Circuit diagram.
RC (b) Phasor diagram.
C1 = 4 2
R3 The bridge can be balanced by adjusting the values of R1
and C1 such that voltages at the terminals of the detec-
Note: The balancing of a Schering bridge is independent tor D are equal and the current through them becomes
of frequency. zero (balance condition). Thus for balanced circuit,

Chapter 7 Theory.indd 624 3/23/2016 1:22:45 PM


7.3  BRIDGES AND POTENTIOMETERS     625

Z1 R3 B
=
Z2 R4 M L1 I1
C1
where Z1 is the impedance of the arm containing C1 and I1
R3
R1. Z2 is the impedance of the arm containing C2 and
R1
R2. Then, C
A D
 
 R1

1 + jwC R 
I2 R4
R3 R2 I2
1 1
=
 
R4 R − j  I = I1 + I2
  L2
2
wC  2
D
w (C2R1R4 ) = w (C2R2R3 + C1R1R3 )

(
−j R3 − w 2C2C1R1R2R3 ) E
Equating real and imaginary parts we have (a)

R4 R C e
= 2 + 1
R3 R1 C2
(I1 + I2)wM
And,

w=
1
R1R2C1C2 I2wL2

1 1 I1wL1
⇒f = ×
2p R1R2C1C2

Thus, the balanced equation involves a factor of fre- I1R1 = I2R2 I1R3 = I2R4 I1
quency, even though individual bridge elements may be (b)
Figure 7.16 |   Heaviside bridge. (a) Circuit diagram.
independent of frequency.

(b) Phasor diagram.


7.3.3  Measurement of Mutual Inductance
Under balanced condition,
Mutual inductance can exist only in the presence of self-
inductance. An important measure of mutual inductance I1R3 = I2R4  (7.16)

(M) is its ratio to the geometric mean of the two self- And
inductances (L1 and L2) it connects, called the coefficient
of coupling,
I1[R1 + jw (L1 − M )] = I2 [R2 + jw (L2 + M )]  (7.17)
M
k=
L1L2 Dividing Eqs. (7.16) and (7.17), we get

Different AC bridge circuits are used for measurement of R1 + jw (L1 − M ) R2 + jw (L2 − M )


=
mutual inductance. These include: R3 R4

Equating real and imaginary parts, we get


7.3.3.1 Heaviside Bridge
RR
In Heaviside bridge, the mutual inductance is measured R3R⇒RR1⇒=R 2= 3R2R3
R1R4R=RR2=
1 4 2 3 1R4
in terms of self-inductance. The circuit (Fig. 7.16) con- R 4
sists of four non-inductive resistors R1, R2, R3 and R4 and
connected on the four arms of the bridge. The mutual L1 − L
M −M L2 + L
M+M
inductor with unknown inductance is connected in series 1 = 2
R3 R =
R4 R
of this bridge circuit. 3 4

Chapter 7 Theory.indd 625 3/23/2016 1:22:59 PM


626     Chapter 7:  ELECTRICAL AND ELECTRONIC MEASUREMENTS 

Thus mutual inductance is given by 7.3.4.1 Kelvin Double Bridge


L1R4 − L2R3 Kelvin bridge, also known as Kelvin double bridge or
M =
R3 + R4 Thomson Bridge is widely used to measure an unknown
electrical resistance below 1 Ω. Its circuit configuration
The modified Heaviside-Campbell bridge is used to mea- and principle of operation is similar to the Wheatstone
sure the unknown value of self inductor in terms of mutual bridge except for the presence of additional resistors.
inductance. A balancing coil L and resistance r is added to The bridge uses a second set of ratio arms and hence the
the arm to which mutual inductor is connected in series name double bridge. The schematic circuit configuration
(Fig. 7.17). A short circuit switch is connected across R3 for Kelvin double bridge is shown in Fig. 7.18. The first
and L2 to obtain two sets of mutual inductance readings, ratio arms is R1 and R2. The second set of ratio arms R1¢
M1 and M2, when R3 and L2 are short-circuited and open- and R2¢ is used to connect the galvanometer to a point
circuited, respectively. Then self-inductance is given by D at a suitable potential between points M and N such
that the effect of connecting lead resistance r between
M1 − M 2 the unknown resistance R3 and the standard resistance
L2 =
1 + R3 / R4 R4 is eliminated. The ratio of resistances in the first arm
(R1/R2) and second arm (R1¢/R2¢) are made equal.

(I1 + I2) B
S1 A
I2 M G
I1 R1 R2
L2 L1
R2 R1 D
r R′1 R′2
L
D D B
I2 I1 A R3 r R4 C
M N
R4 R3 I I

Rb
C

Figure 7.17 |   Heaviside Campbell bridge.


E
Figure 7.18 |   Kelvin double bridge circuit.

7.3.4  Measurement of Resistance Under balance conditions there is no current through the
galvanometer which means that the voltage drop between
Resistance measurement techniques include: A and B, EAB is equal to voltage drop EAMD between A
and C. Then
  1.  Bridge circuits
  2.  Voltmeter-ammeter method and ohmmeter R1
  3.  Substitution method EAB = E
R1 + R2 AC
Different methods may be used for measurement of resis-  (R ′ + R ′ )r 
tance, depending on resistance value: EAC = I R3 + R4 + ′ 1 ′ 2 
  1.  Low resistance: If the resistance is low (of the  R1 + R2 + r 
order of 1 Ω or low); ammeter-voltmeter method,  R1′ r 
EAMD = I R3 + ′ 

R1 + R2′ + r 
Kelvin’s double bridge method and potentiometers
are used. 
  2.  Medium resistance: If the resistance is medium
For zero galvanometer deflection, EAB = EAMD.
(1 to 10,000); Wheatstone bridge, Carey-Foster
bridge or substitution method are used. Therefore from the above equations,
  3. High resistance: If the resistance is high (>10,000 Ω);  ′ ′   
Megohm bridge or direct deflection, loss of change R1I R + R + (R1 + R2 )r  = I R + R1′ R2′ 
 3 4 ′ ′   3 ′ ′ 
and Megger methods are used. R1 + R 2  R1 + R2 + r   R1 + R 2 + r 

Chapter 7 Theory.indd 626 3/23/2016 1:23:06 PM


7.3  BRIDGES AND POTENTIOMETERS     627

R2′ r R ′ is relatively large. The voltage drop in the ammeter may


R
R3 = 1 R4 + ′  1 − R1  
  lead to error in the reading.
R1 + R2′ + r  R2 R2′ 
(7.18)
R2
In the circuit shown in Fig. 7.19(b), the measured
Since resistance is given by
R1 R′ V V Rx
= 1′ R= = =
R2 R2 I Ix + I V IV
1+
Ix
The Eq. (7.18) reduces to
If Ix >> IV, the unknown resistance is equal to the mea-
R sured value of the resistance. So this connection of volt-
R3 = 1 R4
R2 meter-ammeter is used when resistance to be measured
is relatively small. The current through voltmeter may
There are some commercial devices available with accu- lead to error in the reading.
racies of 2% for resistance ranges between 0.000001 Ω
to 25 Ω.
The method may be used for low and medium level
resistances.

7.3.4.2 Ammeter-Voltmeter Method An ohmmeter, using only one meter, that is voltmeter
or ammeter, is also used for measurement of resistance.
This method is mainly used in the laboratories but not in Here, one of the parameters (current or voltage) is kept
practical applications. It involves use of two meters and constant. A basic series ohmmeter consists of a perma-
the accuracy is determined by the accuracy of both the nent magnet moving coil instrument connected in series
voltmeter and ammeter. Their possible arrangements are with standard resistance.
as shown in Figs. 7.19 (a) and (b).
7.3.4.3 Wheatstone Bridge
VA
+ − Wheatstone bridge is most commonly used for measur-
A
+ ing medium level resistances. Fig. 7.20 shows the circuit
+ for Wheatstone bridge. It consists of three known resis-
I
Vs V V Rx Vx tances (R1, R2, R3) and unknown resistance R4 arranged
− to form two parallel circuits. A galvanometer is bridged
− − between B and D and the values of known resistances
are adjusted to balance the bridge.
(a)
A
I

A R2
+ + R1
IV Ix
I1 I2
Vs V V Rx
V D G B
− − I3 I4
R3 R4
(b)
Figure 7.19 |   Ammeter-voltmeter method circuits.

In the circuit shown in Fig. 7.19(a), the measured resis- C


Figure 7.20 |   Circuit diagram for Wheatstone bridge.
tance is given by

V V + VA V Under balanced condition


R= = x = Rx + A
I I I VAD = VAB and I1R1 = I2R2
If Vx >> VA, the unknown resistance is equal to the mea- Also,
sured value of the resistance. So this connection of volt-
meter-ammeter is used when resistance to be measured VDC = VBC and I3R3 = I 4R4

Chapter 7 Theory.indd 627 3/23/2016 1:23:20 PM


628     Chapter 7:  ELECTRICAL AND ELECTRONIC MEASUREMENTS 

Since I1 = I3 and I2 = I4, we have Comparing the two balanced conditions, simplifying and
solving, we get
R1 R RR
= 2 ⇒ R4 = 2 3
R3 R4 R1 R4 + (l − l1 )r = R3 + (l − l2 )r

R4 − R3 = (l1 − l2 )r
7.3.4.4 Carey-Foster Bridge
Here l1 and l2 are the length of slide rule when slide wire
is calibrated using known resistance R4. Let l1′ and l2′ be
The Carey-Foster bridge is a more elaborate modifica-
tion of Wheatstone bridge, particularly useful for mea-
the length of slide wires at balance points, when known
resistance R4′ is used. Then
suring or comparing two nearly equal resistances. The
circuit for Carey-Foster bridge is shown in Fig. 7.21.
Here R1 and R2 are known non-inductive resistances,
R4 is the standard resistance and, R3 is the unknown R4 (l1′ − l2′ ) − R4′ (l2 − l1 )
R3 =
resistance to be measured. A slide wire having length l l1′ − l2′ − l1 + l2
is added between resistances R3 and R4. Resistances R1
and R2 are adjusted so that the ratio R1/R2 is approxi-
mately equal to R3/R4 by sliding contact on slide wire. 7.3.4.5 Substitution Method
Let r be the resistance per unit length of the slide wire.
The circuit diagram for measurement of medium resis-
tance by substitution method is shown in Fig. 7.22. It
B consists of a known (standard) resistance R, a regulating
resistance Rr and the unknown resistance Rx. To mea-
R1 R2 sure Rx, first switch S1 is put on point 1, switch S2 is
closed and reading of ammeter is noted. Then switch S1
G is moved to point 2 and the known variable resistance R
is adjusted until the ammeter gives the same deflection
A C as in the first case. The value of the unknown resistance
Slide wire
is obtained directly from the known variable resistance,
R3 R4 producing the same deflection.

l1 R
l2 S1
L Rx

A
Rr

S2
Figure 7.22 |   Circuit for substitution method.
Figure 7.21 |   Carey-Foster bridge.

When the bridge is balanced, let l1 be the distance of the 7.3.4.6 Measurement of High Resistances
sliding contact from the left hand end of the slide-wire.
Next the resistances R3 and R4 are interchanged and the The methods that can be adopted for measuring resis-
bridge is balanced by moving the slide rule to distance l2. tances of the order of 0.1 MΩ and higher are:
Then:   1.  Direct deflection method
For first balance condition   2.  Loss of charge method
  3.  Megohm bridge
R1 R3 + l1r   4.  Megger method
=
R2 R4 + (l − l1 )r
These are discussed as follows.
And for second balance condition
Direct Deflection Method
R1 R3 + l1r
= Figure 7.23 (a) shows an arrangement for measurement
R2 R4 + (l − l1 )r
of high resistance in cables having metallic sheath by

Chapter 7 Theory.indd 628 3/23/2016 1:23:31 PM


7.3  BRIDGES AND POTENTIOMETERS     629

direct deflection method. The leakage current IL is car-


ried by guard wire wound on the insulation and therefore
does not flow through the galvanometer as shown.
V V R C
Metallic
IR Guard sheath
IL wire
G Figure 7.24 |   Loss of charge method of measuring high
resistances.
V
The terminal voltage during discharge is given by,

V = ve(−t /CR) ⇒ = e(−t /CR) ,


V
Conductor Insulating v
material
The insulation resistance is thus,
(a)
t
R=
IL C lnV / v

IR Megohm Bridge
+ G Guard
wire Figure 7.25 shows the circuit for the Megohm bridge.
The circuit is completely self-contained and includes
V inbuilt power supplies, amplifiers, bridge members, and
Cable
− an indicating instrument. It has range from 0.1 MΩ to
106 MΩ.

P = 100 kΩ
Q

Guard +
RAG G terminal − V

A
(b) RBG
Figure 7.23 |   Direct deflection method for measurement
R
B S 10 kΩ to
of high resistance.
100 MΩ
Figure 7.23 (b) shows the arrangement for measurement
of high resistance in cables without metal sheaths. The E
ends of the cable are immersed in water in a tank. The
water and the tank then form the return path of the cur- Figure 7.25 |   Megohm bridge.
rent. The insulation resistance of the cable is,
The accuracy is usually within 3% to possible 10% above
V 10000 MΩ. Sensitivity of balancing at a high resistance
R = is obtained by usage of adjustable high voltage supplies
IR
of 500 V to 1000 V. The use of a sensitive null indicating
arrangement such as a high gain amplifier with an elec-
Loss of Charge Method tronic voltmeter or a cathode ray oscilloscope can also be
Figure 7.24 shows the circuit for loss of charge method of used for the purpose.
measuring high resistances. In this method, the unknown
resistance is connected in parallel with capacitor and Megger Method
electrostatic voltmeter. The capacitor is charged initially
to a voltage V and then allowed to discharge through the Figure 7.26 shows Megger arrangement for measuring
resistance R. The voltmeter reading is v. high resistances. The current coil is the same as that in

Chapter 7 Theory.indd 629 3/23/2016 1:23:39 PM


630     Chapter 7:  ELECTRICAL AND ELECTRONIC MEASUREMENTS 

PMMC instrument. V1 and V2 are the two voltage coils the use of these meters should not change the quantity to
and V1 encloses the annular magnetic core. be measured. For this, ideally, the voltmeter should have
an infinite resistance, so that current component is not
Annular magnetic altered by the inclusion of the voltmeter and an ammeter
core should have zero resistance so that the load voltage is
not altered by the inclusion of the ammeter in the circuit.
Current coil However, for practical purposes, voltmeters have very
Magnet high resistance and ammeters have very low resistance.
Rx These meters can eb used in both AC and DC circuits.
H

− + 7.4.1  Shunts and Multipliers
V2 V1 G
Generally moving iron instruments are used as ammeter
R 0 and voltmeters. The use of these meters for operating a
S Generator
moving coil instrument would be impractical due to bulk
R′ Magnet and weight of the coil required. So, to enhance use of these
R″ meters and extend their range, shunts (in case of amme-
ters) and multipliers (in case of voltmeters) are used.
Figure 7.26 |   Megger method for measuring high
resistance. 7.4.1.1 Extension of Range of Ammeters

The voltage coil V1 is in weak magnetic field when the The circuit for extension of an ammeter is shown in
pointer is at infinity and so this coil exerts lesser torque. Fig. 7.27.
The torque exerted by V1 increases as it moves into a
stronger field. This torque becomes maximum when it is Rm
under the pole face of the magnet and under this condi-
tion the pointer will be at zero of the resistance scale. Vm
In order to modify the torque in the voltage circuit, Im
another coil V2 is placed in the circuit. This coil is
located in such a way that it can move from infinity to I = Ish + Im Rsh I
zero. Coils V1 and V2 combined function like a spring
of variable stiffness, such that. It is very stiff near zero Figure 7.27 |   Ammeter with a shunt resistance.
when the current in the current coil is very small due
to the presence of unknown resistance Rx which is very Here Rm is the internal resistance of movement (coil) in Ω;
large. As a consequence, the low resistance portion of the Rsh is resistance of shunt in Ω;
scale is compressed and high resistance part of the scale
Im = Ifs is full-scale deflection current ( in amperes);
opens up.
Ish is the shunt current in amperes and I is the current
The voltage range for measurement using a Megger to be measured in amperes.
circuit can be controlled by varying the series resistance
Then
R (to R′or R′′) connected with the current coil. The test
voltages can be varied as 500, 1000 or 2500 V and can be I sh R sh = I m R m
supplied using generator G.
Im R m
R sh =
I sh
7.4  MEASUREMENT OF CURRENT
AND VOLTAGE I sh = I − I m

Thus
An ammeter is used to measure the current in a circuit I m Rm
which is connected in series with the components carry- R sh =
ing the current. A voltmeter is required to measure the (I − I m )
voltage across a particular element in the circuit and is I Rm
connected in parallel with the component across which −1 =
the voltage is to be measured. For accurate measurement, Im R sh

Chapter 7 Theory.indd 630 3/23/2016 1:23:46 PM


7.5  MEASUREMENT OF POWER AND ENERGY     631

I R The sensitivity of a voltmeter is determined in ohms


= 1+ m per volt. It is found by the division of the sum of the
Im Rsh
resistance of the meter (Rm), plus the series resistance
I (Rs), by the full-scale reading in volts. Mathematically
=m sensitivity is expressed as:
Im
R s + Rm
The ratio of current to be measured and the full scale Sensitivity =
deflection current is known as the instrument constant or Full scale deflection (FSD)
the multiplying factor. For the same instrument with dif-
ferent shunts, the instrument constant will be different.
7.5  MEASUREMENT OF POWER AND
resistance of the shunt is ENERGY
Rm Rm
R sh = =
(m − 1)  I 
 − 1

Power consumed in DC circuits is measured as a product
 I m of reading of ammeter and voltmeter, that is,
Multiple shunts can be used for multiple range ammeters.
P = VI

7.4.1.2 Extension of Range Voltmeters However, in order to obtain correct power consumed by
a load, corrections must be applied for the power loss in
For extension of range of voltmeters, a series resistor or a the instrumentthat is it should include the power con-
multiplier is required as shown in the circuit in Fig. 7.28. sumed by the instrument closer to the load terminal.
Therefore, considering power loss in ammeter, power
Rs Rm consumed by a load is
Im VL = VI − I 2R a
And considering power loss in voltmeter, power con-
sumed is
V V2
VL = VI −
Figure 7.28 |   Voltmeter with series resistor RV
(multiplier).
For measurement of power in AC circuits, the expres-
Here Im is the deflection current of movement, Rm is the sion is
internal resistance of movement, Rs is multiplier resis- P = VI cos f
tance and V is full range voltage of instrument.
where cosf is the power factor of the load.
Then

V = I m (Rs + Rm ) 7.5.1  Measurement of Power in AC Circuits


V − I m Rm V
Rs = = − Rm For measurement of AC power wattmeter is used instead
Im Im of voltmeter and ammeter. The most commonly used
is electrodynamic- or dynamomter-type wattmeter.
V R
= s +1 Induction-type wattmeter gives the integrated measure
I m Rm Rm of power with respect with time, which is the measure of
The circuit for multiple range of extension of voltmeter energy. It is hence called induction type energy meter.
can be achieved using more number of multiplier resis-
tances in series. 7.5.1.1 Dynamometer-Type Wattmeter

7.4.2 Sensitivity of Ammeter and Voltmeter Figure 7.29 shows the circuit for electrodynamometer-
type wattmeter. In this instrument, there are two low-
The amount of current required by the meter coil to resistance current coils (CC), which are fixed at their
produce full-scale deflection of the pointer is known as positions. A high-resistance moving coil called the poten-
ammeter sensitivity. If the amount of current required tial or pressure coil (PC) is placed between the two fixed
to produce the full-scale deflection is low, the sensitivity coils, such that it may cut the magnetic field created by
of the meter is high. the two coils. The spindle, carrying spring S and pointer

Chapter 7 Theory.indd 631 3/23/2016 1:23:59 PM


632     Chapter 7:  ELECTRICAL AND ELECTRONIC MEASUREMENTS 

P on it, is rigidly connected with the fixed coil. The two Since the deflection of such instruments is proportional
current coils are connected in series with each other. to the average power they almost have uniform scales.
The sense of winding in them is such that they produce So, this type of instruments can be used in both ac and
magnetic field in the same direction (Fleming’s right- dc power measurement.
hand rule).
Errors in Measurement using Electrodynamic
Scale Wattmeter
Pointer The errors associated with use of electrodynamic watt-
meter for measuring of power are listed as follows.
I1
  1.  Due to inductance of potential coil: The
Fixed coil potential coil used in the wattmeter, though
(CC) assumed to be purely resistive, will have some
inductance also. As a result, the current through
Spring the coil lags the voltage and is not in phase with
Moving coil it. Hence the angle between the current in the cur-
(MC) rent coil and current in the potential coil is less
than f. If the angle is f ′ = f − b , then the watt-
I2 meter reading is proportional to VIcos(f - b)′cos f
whereas the true power should be VIcosf. So the
Supply High correction factor is
Load
voltage resistance (R)
cos f
cos(f − b ) cos b
Figure 7.29 |   Dynamometer type wattmeter.
Similarly if the current through the coil leads by
One of the fluxes is produced by a fixed coil which carries angle b, the correction factor is
a current proportional to the load current and, therefore,
is called the current coil. The other flux is created by cos f
a coil which carries a current proportional to the load cos(f + b ) cos b
voltage and thus called the voltage or potential coil. A
high non-inductive resistance is connected to the poten- This error can be prevented by connecting a capac-
tial coil so that its current is almost in phase with the itor across part of external resistance R.
load voltage. The deflecting torque is produced by the   2.  Due to capacitance of potential coil: This
magnetic effect of electric current. The control torque error arises because of the presence of capacitance
is provided by control springs. The damping torque is in the potential coil in addition to inductance. This
provided by air friction damping. error results in effects that are opposite to those
induced by presence of inductance.
Instantaneous power, P = vi.   3.  Due to mutual inductance of coils: These
Assuming that voltage and current waves are sinusoidal, errors resulting from mutual inductance between
v = Vm sin wt
current and potential coils are more operative at
higher frequencies. As a consequence of this error,
the phase angle for connection of voltage coil on
i = I m sin (w ± f ) the load side is increased and phase angle when
where f is the phase by which the current lags or leads current coil is connected to load is decreased.
the voltage.   4.  Due to Eddy current: Alternating magnetic
field of the current coil, leads to the generation of
So, instantaneous power, Eddy currents. The magnetic field generated by
P = vi = Vm I m sin wt sin (wt ± f ) the Eddy currents modifies the phase and magni-
tude of current in the current coil, thus introduc-
Average power over a cycle ing errors.
T   5.  Due to connections: The diagram for connection
∫ V m I m sin wt sin(wt ± f )  (T = 2p )
1 of a wattmeter in a circuit with small load current
P =
T is shown in Fig. 7.30. Error is introduced in power
0
measurement due to loss of power in current or
P = Vrms I rms cos f Watt voltage coils.

Chapter 7 Theory.indd 632 3/23/2016 1:24:12 PM


7.5  MEASUREMENT OF POWER AND ENERGY     633

W There are two electromagnets in this instrument which


comprise the driving system. The series magnet is con-
C C IL nected in series with the load and is energised by the
CC. The shunt magnet takes the pressure coil which
P C
Source VL Load carries the current proportional to the supply voltage.
The magnetic fields produced by the two magnets act
upon an aluminium disc, which is free to rotate around a
Figure 7.30 |   Connections of wattmeter in a circuit.
spindle. It cuts the fluxes of both the magnets. A deflect-
ing torque is produced by the flux of each magnet, which
The voltage across the potential coil is equal to the sum of tries to rotate the disc. This is known as the moving
voltages across the current coil and load. The wattmeter system of the energy meter. The magnetic effects of cur-
measures the power consumed by the load and power loss rents through CC and PC will keep acting on a rotating
in the current coil. This error can be removed by use of a disc to give a cumulative value of power consumed with
compensating coil connected in series with the potential respect to time (i.e. energy).
coil and identical and coincident with the current coil. A permanent magnet known as the brake (or drag)
magnet is used to control the movement of the disc.
7.5.1.2 Induction Type Energy Meter It forms the braking system of the energy system. It
placed near the edge of aluminium disc. Eddy currents
Measurement of energy means integrated measurement are induced due to the rotation of the disc in the field
of power with respect to time. of braking magnet and the flux produced by the eddy
E= ∫ P dt current opposes the main flux. This produces a torque
proportional to the speed of disc.
Hence, an energy meter would work to continuously The spindle of the disc is connected to a counting
measure the cumulative effects of currents and volt- mechanism, which records the number of revolutions of
ages in the circuit. The energy meter is an instrument disc and indicates the energy consumed directly in kWh.
that measures the electrical energy consumed. It is also The number of revolutions made by the disc for con-
known as watt-hour or kilowatt-hour meter. sumption of one kilowatt hour of energy is known as
In an energy meter, a single pointer is not sufficient to meter constant (K). Therefore,
Number of revolutions = K ×Energy in kWh,
measure the total energy consumed. The time dependent
measurement can be achieved by having a continuous
rotation of a disc rather than a deflection. If the number of Number of revolutions
K=
revolution at a constant speed are proportional to the time kWh
and the speed is proportional to the power; the energy
7.5.2  Measurement of Power in a Three-Phase
consumed can be obtained easily. Figure 7.31 shows the
schematic diagram for induction type energy meter.
Circuit

Consider the star connected three-phase circuit shown


Shunt in Fig. 7.32.
Voltage magnet Iph = IL
coil R
(PC)
Z∠f
VL(rms)
Moving
Break aluminim Z∠f Z∠f
(drag) disc
magnet Y
Current CC
coil B
Figure 7.32 |   Three-phase star circuit.
(CC)

Load Then if Iph is the current through each phase, then


Series expression of power in each phase is
magnet V
Apparent power = L × I ph VA
Figure 7.31 |   Schematic of induction type energy meter. 3

Chapter 7 Theory.indd 633 3/23/2016 1:24:19 PM


634     Chapter 7:  ELECTRICAL AND ELECTRONIC MEASUREMENTS 

VL Various methods of measurement of three phase power


Real power = × I ph cosf Watts use different number of wattmeters:
3
  1.  One-wattmeter method: In this method, the
VL voltage coil of the wattmeter is connected across
Reactive power = × I ph sinf VAR
3 the phase and the current coil is connected in series
with the phase. Thus, at a time, the wattmeter
Thus the real power in three-phases gives power in one phase only. So after the measure-
VL ment in one phase, the wattmeter is disconnected
= 3× × I ph cosf Watts and then reconnected in each of the two remaining
3
phases. The three reading are finally added to get
= 3V L I ph cosf = 3VL I L cosf total power. The method is thus time consuming.
  2.  Two-wattmeter method: In this method
as Iph = IL in a start connection. two wattmeters are connected in an appropriate
Similarly, in a delta connected three-phase circuit manner, such that the sum of the two wattmeter
shown in Fig. 7.33. readings gives the total three-phase real power.
This is discussed in detail in the next section.
IL   3.  Three-wattmeter method: In this method, three
R wattmeters are used; one connected across each
phase. This gives the total three-phase power by
summing up the readings of the three wattmeters.
Vph = VL(rms) Z∠f Z∠f The method is faster but expensive as it requires
use of three wattmeters simultaneously.

Z∠f 7.5.2.1 Two Wattmeter Method of Power


Y Measurement

B Figure 7.34 shows the schematic circuit diagram for two-


Figure 7.33 |   Three-phase delta circuit.
wattmeter method of power measurement in a three-
phase star-connected system.
If the per phase voltage is VL and magnitude of per W1
phase current is I L / 3 then per phase IRN
R
IL
Apparent power = VL VA
3 VRY Z∠f

IL
cos f Watts
Y
Real power = VL
3 Z∠f Z∠f
IYN
IL
Reactive power = VL sin f VAR
3 B
IBN
The real power in an three-phase electrical circuit is
measured using the wattmeter. It consists of two coils, W2
Figure 7.34 |   Schematic circuit diagram for two-wattmeter
namely the voltage coil (also called the potential coil or
the pressure coil) carrying large number of thin turns method of power measurement in a three-
and the current coil carrying less number of thick turns. phase star-connected system.
The voltage coil is connected in parallel and the current
coil is connected in series to the load in which the power The current coils of the wattmeters 1 and 2, are in series
is to be measured. The reading given by the wattmeter with the two phases, R and B. The pressure or volt-
is the product of the rms values of the voltage (across age coils are connected between RY and BY phases for
the voltage coil), rms value of the current (in the current the two wattmeters. The total instantaneous power con-
coil) and the cosine of the angle between them. sumed in the load circuit is given by,

P = VI cos f W = iRN ⋅ vRN + iYN ⋅ vYN + iBN ⋅ vBN

Chapter 7 Theory.indd 634 3/23/2016 1:24:32 PM


7.5  MEASUREMENT OF POWER AND ENERGY     635

From Fig. 7.34, the voltage across the pressure coil in The reading of the first wattmeter W1 is given by,
the wattmeterW1 is
W1 = VRY ⋅ IRN ⋅ cos(VRY + IRN )
vRY = vRN − vYN
= VRY ⋅ IRN cos(30° + f )
and the current through the current coil is iRN. Then the
instantaneous power measured by this wattmeter W1 is = 3 V ph ⋅ I ph ⋅ cos(30° + f)  (7.21)

given by,
W1 = iRN ⋅ vRY + iRN (vRN − vYN ) Similarly the reading of the second wattmeter W2 is
expressed as,
Similarly, the instantaneous power measured by the W2
wattmeter, W2 = VBY ⋅ IBN ⋅ cos(VBY − IBN )
W2 = iBN ⋅ vBY + iBN (vBN − vYN ) = VBY ⋅ IBN ⋅ cos(30° − f )
= 3 Vph I ph cos(30° − f)
The sum of the two wattmeter readings is
(7.22)
W1 + W2 = iRN (vRN − vYN )

+iBN (vBN − vYN ) The line voltage, VRY leads the respective phase voltage,
= iRN ⋅ vRN + iBN ⋅ vBN VRN by 30o, and the phase voltage, leads the phase cur-
rent, IRN by f. Therefore, the phase difference between
−vYN (iRN + vBN )  (7.19) VRY and IRN is 30o+ f which can also be seen from the
Also, phasor diagram. From Eqs. (7.21 and 7.22), the sum of
iRN + iYN + iBN = 0 ⇒ iYN = −(iRN + iBN )  (7.20) the two wattmeter readings is given by,

Substituting the value of iYN from Eq. (7.20) in Eq. (7.19), W1 + W2 = 3Vph I ph [cos(30° + f ) + cos(30° − f )]
we get
= 3 Vph I ph ⋅ 2 cos 30° cos f
W1 + W2 = iRN ⋅ vRN + iBN ⋅ vBN + iYN ⋅ vYN
= 3 Vph I ph cos f = 3VL I L cos f
So, it can be concluded that the sum of the two watt-
meter readings is the total power consumed in the three-
So, the total power consumed by the balanced load is
phase circuit.
equal to (W1+W2). This method is also applicable to
The phasor diagram for a three-phase balanced star- balanced delta-connected load.
connected circuit is shown in Fig. 7.35. Here VBY and
VRY are line voltages and VRN is the phase voltage.
Determination of Power Factor for the Bal-
Angle between VBY and IB is (30o − f) and that between
anced Load
VRY and IR is (30o +f).
VBY From Eqs. (7.21 and 7.22), the difference of the two
wattmeter readings is

W1 − W2 = 3Vph I ph [cos(30° − f ) − cos(30° + f )]

= 3 V ph I ph 2 sin 30° sin f = 3 Vph I ph sin f

VBN −VYN VRY If the two sides of the above expression are multiplied
30°
by 3 then

IB 3 (W2 − W1 ) = 3Vph I ph sin f = 3VL I L sin f


f 60° Thus,
30° W2W−2 W
−1W1 1 1
f = = ⋅ tan f f
⋅ tan
IY
f
VRN W2W+2 W
+1W1 3 3
IR
or
   W W− W  
 2 −1W
= tan− 13 ⋅ 3 ⋅  2 1 
−1
f=
f tan  
  
 W 2 2 1  1 
 W+ W
+ W
VYN
The changes in the reading of the two wattmeteres based
Figure 7.35 |   Phasor diagram for two-wattmeter on the power factor of the load is given in Table 7.1.
method of power measurement.

Chapter 7 Theory.indd 635 3/23/2016 1:24:48 PM


636     Chapter 7:  ELECTRICAL AND ELECTRONIC MEASUREMENTS 

Table 7.1 |   Variation of two-wattmeter readings with change in power factor of the load current

Wattmeter Readings (W)


Power Factor of Load Inference
W1 W2

pf = 1.0 (f = 0°) Positive Positive W1 = W2

0.5 < pf < 1.0 (60° > f > 0° ) Positive Positive W1 > W2

pf = 0.5 ( f = 60°) Positive Zero Total power = W1

0 < pf < 0.5 (90° > f > 60°) Positive Negative | W1 | > | W2 |, so total power > 0

pf = 0 (f = 90°) Positive Negative | W1 | = | W2 |, so total power = 0

7.6  INSTRUMENT TRANSFORMERS

Instrument transformers are used for metering and pro- IP


tection in a power system. Electrical measurements and Laminated
relaying decisions in a power system are made based iron core
on the current and voltage obtained from the system. Conductor
Relays work with smaller magnitudes of these signals. carrying
current IS
Real life currents and voltages thus have to be scaled
to lower levels. This job is done by current and volt- A
age transformers also known as instrument transformers.
They also electrically isolate the relaying system from
the power equipment and working personnel.

7.6.1 Current Transformer Secondary


winding
Current transformers (CTs) are extensively used in around core
power system for power measuring circuits. CTs are gen-
erally used in panel boards in substations or grid station
to measure the high valued bus bar currents. These are

Figure 7.36 |   Current transformer.


also used in combination with the relays for protection
purposes. Current transformer can measure high cur-
rents in a conductor. The conductor carrying high cur-
rent passes through circular and laminated iron core of There may be different kinds of current transformers
the transformer. The conductor is the single turn pri- based on their use in metering and protection circuits.
mary winding. The secondary winding will constitute of When a current transformer is used for both metering
large number of turns of wire wound around this circular and protection purposes, it has to be of required accu-
core. The secondary current (IS) in turn is reduced to a racy class to suit both accuracy of measurement and
lower value than the higher valued primary current (IP) protection. It has to be precise and sensitive for both
as the secondary voltage is stepped up. The secondary small and large values of current.
is connected to an ammeter for current measurement. A Circuit for measurement of current using a CT is given
typical current transformer is shown in Fig. 7.36. in Fig. 7.37.

Chapter 7 Theory.indd 636 3/23/2016 1:25:06 PM


7.6  INSTRUMENT TRANSFORMERS     637

IP 90°
[90° − (g + f0)] B (g + f0)
AC C
Load A I1
Supply

I2
K
q

=
I0
Primary E1
winding g I0
IS f0 Iw
Secondary 0
A winding
Im fm
a
Figure 7.37 |   Circuit for measurement of current using
g
a CT.
I2
d
Three types of core construction can be employed for
V2
CTs namely, core type, shell type and ring type. The I2R2
core type construction has an advantage that sufficient
space is available for insulation purposes which makes
I2X2
it more suitable for high voltage work. Shell type gives
E2
better protection for the windings. Ring type is the most
common of the core constructions for CT. It has very Figure 7.38 |   Phasor diagram for CT.
small leakage reactance as it has no joints in the core.
7.6.2  Voltage Transformer
7.6.1.1 Transformer Burden
Voltage or Potential transformer (PT) is a type of trans-
In CTs, the secondary has very small impedance referred former used to measure high voltages; they basically
to as burden, so the CT practically operates on short function as step down transformers. They have smaller
circuit conditions. The burden for CT is the volt-ampere number of secondary turns than turns in the primary.
(VA) loading which is imposed on the secondary at rated Figure 7.39 shows a voltage transformer circuit.
current. The burden can also be expressed as the ratio
between secondary voltage and secondary current.
Voltage to be
A metering CT has lower VA capacity than a pro- V Voltmeter
measured
tection CT. A metering CT has to be accurate over its
complete measuring range. Such a CT’s magnetising
impedance at low current and hence low flux should be Figure 7.39 |   Voltage transformer.
very high. The magnetising impedance is not constant
for a CT’s operating range due to the non-linear charac- The voltage to be measured is connected across the pri-
teristics of the B-H curve. It cannot give linear response mary circuit. The low voltage secondary circuit is con-
during large fault currents. For protection CT, linear nected to a voltmeter. The power rating of this type of
response is expected for up to 20 times the rated current. transformer is usually lower. The circuit for measure-
It is also expected to give precise performance in the ment of voltage using a PT is given in Fig. 7.40.
normal operating currents up to high fault level currents. Primary
winding
7.6.1.2 CT Phasor Diagram

Figure 7.38 shows the phasor diagram for the current


transformer. Here, flux fm is taken as the reference. The AC
V Load
no-load current I0 has two components, that is, the mag- Supply
netising Im and the core loss component Iw. E1 and E2
are respectively the induced emf for the primary and Secondary
secondary windings lagging behind the flux by 90°. The winding
magnitudes of the emf are proportional to their number Figure 7.40 |   Circuit for measurement of voltage
of turns in windings. using a PT.

Chapter 7 Theory.indd 637 3/23/2016 1:25:11 PM


638     Chapter 7:  ELECTRICAL AND ELECTRONIC MEASUREMENTS 

7.6.2.1 PT Phasor Diagram 7.6.3  Linear Variable Differential Transformer


(LVDT)
Figure 7.41 shows the phasor diagram for a voltage
transformer. Starting with the flux reference, E2 is the The linear variable differential transformer (LVDT) is a
induced emf in the secondary winding and V2 is the ter- passive inductive transformer which requires an external
minal voltage across the secondary. Then source of power. It is used to measure linear displace-
ment. It consists of a primary winding and two second-
V2 = E2 − I2R2 cosf2 − I2 X2 sinf2 ary windings. These windings are wound over a hollow
The primary induced emf E1 is in phase opposite to E2. tube and the primary winding is kept between the two
secondaries. Figure 7.42 shows the schematic and work-
ing of LVDT.
90°
V1 q
Iron core
I2X2
1
I 1X q f
E1 f
KTI2X2 KTI2X2 Primary Secondary Primary Secondary
coil coil coil coil
C K T V2
b
KT I1
I 1=I 2/ Vin Vin
f Vout Vout
q I0
f0 Iw
0
Im fin Figure 7.42 |   Linear variable differential transformer
f2                  (LVDT).

I2
I2R2 V2 A movable iron core slides within the hollow tube and
affects the magnetic coupling between the coils. When
I2X2 E2 the iron core is centrally placed, the voltage induced is
Figure 7.41 |   Phasor diagram for PT. equal in the two secondaries. When the core is moved in
direction of the centre, voltage is increased in one of the
7.6.2.2 Differences between a PT and a CT windings while voltage in the other gets decreased.
Advantages of LVDT: It can produce high output
The main points of difference are listed as follows: voltage with relatively low change in core position. It is
  1.  The secondary of the CT is under short circuit as the also less costly, solid and robust in construction.
primary circuit is energised, a PT can operate with
its secondary under open circuit conditions without 7.6.4  Errors in Instrument Transformers
any damage to the transformer or to the operator.
  2.  Under normal operating conditions, the line volt- There can be two types of errors in instrument trans-
age of the PT is nearly steady. The flux density formers. These are listed as follows:
and the exciting current of a PT vary between
  1.  Ratio error: For CTs, the current transformation
small ranges. On the other hand, the primary cur-
ratio should be constant. It should also be within
rent of a CT varies over wide ranges under normal
the given limits. Practically, it can be seen some-
operating conditions.
times this ratio may vary with power factor. This
  3.  The primary current of CT is independent of sec-
gives an error known as the ratio error.
ondary winding conditions, while primary current
The ratio between actual current transformation
in a PT depends on the secondary burden.
and the normal ratio is known as ratio correction
  4.  For a PT, the primary is connected across full volt-
factor (RCF). Mathematically,
age. In case of CT, the primary is in series with a
line and therefore a very small voltage exists across
Actual transformation ratio k
the terminals. The CT primary on the other hand RCF = =
carries full line current. Normal transformation ratio kn

Chapter 7 Theory.indd 638 3/23/2016 1:25:14 PM


7.8  CATHODE RAY OSCILLOSCOPE     639

  2.  Phase angle error: This is the angle by which   5.  Potentiometric type: It is based on used of a
the secondary current differs in phase from the pri- calibrated potentiometer. The unknown voltage
mary current when reversed. This error is due to is measured by comparison with reference voltage
no-load or exciting current in the transformer. whose value is fixed using the potentiometer.
The advantages of DVM over conventional voltmeters are:
7.6.4.1 Error Minimisation Methods
  1.  Higher accuracy (+0.5% or better in some cases).
The magnetising and core loss component of currents   2.  Less human error as reading is displayed not read.
have to be kept at low values. The core material should   3.  Voltage input range from +1.000 V to +1000 V
have high value of permeability, large cross-section and with the automatic range selection and indication
shorter magnetic path in order to minimise these cur- for overload condition.
rents. The materials from which the core can be con-   4.  Higher resolution as 1 µV reading can be measured
structed for this purpose are hot rolled silicon, cold on 1 V range.
rolled grain oriented silicon steel and nickel iron alloys.   5.  Input impedance is as high as 10 MΩ
Suitable turns ratio can be provided and number of   6.  Small in size and hence portable.
secondary turns can be minimised by one or two turns. A digital multimeter on the other hand is an electronic
Large current on the secondary should be reduced by put- volt ohm meter with a digital display. It is capable of
ting a suitable valued shunt on either side. This process also measuring AC and DC voltages and circuits and resis-
reduces phase angle error in the instrument transformers. tances over several ranges. The basic circuit of a digital
multimeter is generally a DC voltmeter, so any param-
eter that is to be measured is first converted into voltage
7.7  DIGITAL VOLTMETERS AND form. The analog voltage form is then converted into
MULTIMETERS digital form using analog digital converter and the digi-
tal data displayed in decimal or BCD form. It has supe-
rior accuracy than analog instruments.
Digital voltmeters or DVM are analog to digital convert-
ers with a digital display unit. These measure voltage 7.8  CATHODE RAY OSCILLOSCOPE
across two points in a circuit and display the voltage in
the form of discrete numerical instead of pointer deflec-
tion. They can be used to measure both AC and DC The cathode-ray oscilloscope (CRO) is a common labo-
voltages. The most important component of DVM is ratory instrument which can provide accurate time and
analog to digital converter (ADC), which converts any amplitude measurements of voltage signals over a wide
analog signal into digital. For any input analog voltage, range of frequencies. Its reliability, stability and ease of
the output is in the form of binary digital values. operation make it suitable for usage as a general purpose
Digital meters achieve the required measurements laboratory instrument for various signal measurements.
by converting the analog input into digital signal by a The block diagram for a CRO is shown in Fig. 7.43.
sequence of digital samples spaced uniformly in time. The
input signals are processed in discrete time domain with Vetrical Vetrical
the measured signal displayed in digital structure. Thus input amplifier CRT
unlike analog instruments whose signals are processed in
continuous time domain, digital instruments have the sig-
nals processed in discrete time domain hence the name. External Sweep
trigger generator Horizontal
Different types of digital voltmeters are: amplifier
  1.  Ramp type: It uses ramp signals as reference to
convert analog input into digital form. AC line Sweep
  2.  Continuous balance type: It uses a number of signal trigger
test voltages in succession to calibrate the voltmeter. Horizontal
  3. Successive approximation type: It uses a sequence input
of test voltages to calibrate the voltmeter. It is more
Figure 7.43 |   Block diagram for cathode ray
rapid in operation that continuous balance type but
more complex in construction and expensive.
oscilloscope.
  4.  Integrating type: It is based on voltage to fre-
quency conversion and measures the actual average The most important component of CRO is the cathode
of the input voltage over a fixed measuring time. ray tube (CRT) (Fig. 7.44).

Chapter 7 Theory.indd 639 3/23/2016 1:25:16 PM


640     Chapter 7:  ELECTRICAL AND ELECTRONIC MEASUREMENTS 

Electron gun Deflecting


system
− High +
voltage Green spot
supply
Vacuum

Electron beam
6V

Filament Y-plates Fluorescent


Grid Focusing X-plates screen
Cathode anode Graphite
Accelerating coating
anode
Figure 7.44 |   Schematic of cathode-ray tube.

The important components of cathode ray tube their same time a voltage that increases linearly with time is
and functions are listed as follows: applied to the horizontal deflection plates. This causes
  1.  Electron gun: It is the total assembly of the fila- the beam to be deflected horizontally at a uniform or
ment, cathode, control (intensity) grid, focus grid constant rate. The signal applied to the vertical plates
and accelerating anode. It generates the electron can be thus displayed on the screen as a function of time.
beam and control its intensity and focus. The horizontal axis thus serves as a uniform time scale.
(i) Filament gets heated up when current is The linear deflection or sweep of the beam horizontally
passed through it and then heats up the is actually accomplished by the use of a sweep generator
cathode. which is part of oscilloscope control circuitry. The volt-
(ii) Cathode (a negative electrode)emits electrons age output of such a generator is a saw tooth wave as
when heated. shown in Fig. 7.45.
(iii) Control (intensity) grid controls the number
of electrons reaching the fluorescent screen. V
(iv) Accelerating anode accelerates the electrons
towards the fluorescent screen.
(v) Focusing gird (anode focuses the beam of elec-
t
trons on the screen. 0 a b c d
Figure 7.45 |   Voltage difference V between horizontal
  2.  Deflecting plates: These are pair of metal plates
that are oriented in a manner that one set provided
horizontal deflection (Y-plates) and the other set plates as a function of time t.
vertical deflection (X-plates). The combined effect
Application of one cycle of this voltage to the horizon-
of these plates help control the deflection of elec-
tal plates causes the beam to deflect across the tube
tron beam to reach any desired point on the fluo-
face also linearly with time. When the voltage suddenly
rescent screen.
falls to zero at the end of each sweep (points a, b, c, d
  3.  Fluorescent screen: It is a glass screen coated
in Fig. 7.45), each of the beam flies back to its initial
with fluorescent material, which converts the kinetic
position. The horizontal deflection of the beam is thus
energy of the electrons colliding with the screen into
repeated periodically and the frequency of this periodic-
heat and light. So wherever the electron beam hits
ity is adjustable by external controls.
the screen, the fluorescent material (phosphor) is
excited and light is emitted from that point. The working of CRO involves the following steps:
  1.  The signal to be displayed is first amplified by the
7.8.1  Working of CRO vertical amplifier and then applied to the vertical
deflection plates of the CRT.
To study a signal in oscilloscope, it is first amplified and   2.  A portion of the signal in the vertical amplifier is
then applied to the vertical deflection plates and at the also applied to the sweep trigger as a triggering signal.

Chapter 7 Theory.indd 640 3/23/2016 1:25:18 PM


7.8  CATHODE RAY OSCILLOSCOPE     641

  3.  The sweep trigger then generates a pulse which is (ii) Sweep time/cm variable: Provides continu-
coincident with a selected point in the cycle of the ously variable sweep rates. The calibrated
triggering signal. This pulse turns on the sweep position is fully clockwise.
generator and thereby initiating the saw-tooth (iii) Position: Controls horizontal position of trace
wave form. on screen.
  4.  The saw-tooth wave is then amplified by the hori- (iv) Horizontal variable: Controls the attenuation
zontal amplifier and applied to the horizontal or reduction of the signal applied to horizontal
deflection plates. amplifier through external horizontal. connector.
  5.  Some additional provisions for external signals are   4.  Trigger: This section selects the timing of the
usually made for applying an external triggering beginning of the horizontal sweep and has the fol-
signal or utilising the 60 Hz line for triggering. lowing related controls.
Also, in some cases, the sweep generator may not
(i) Slope: Selects whether the triggering occurs
be used and an external signal applied directly to
on an increasing (+) or decreasing (−) por-
the horizontal amplifier.
tion of trigger signal.
(ii) Coupling: Selects whether triggering occurs at
a specific DC or AC level.
7.8.2  CRO Controls (iii) Source: Selects the source of the triggering signal.
(iv) Level: Selects the voltage point on the trigger-
The controls present in most oscilloscopes provide a wide
ing signal at which sweep is triggered. It also
range of operating conditions and thus make the instru-
allows automatic (auto) triggering.
ment suitable for measuring a wide variety of signals. A
brief description of controls that are common to most
oscilloscopes (as depicted in Fig. 7.44) are as follows: 7.8.3  Measurements of Voltage
  1.  Cathode-ray tube (CRT): This section per- Consider the circuit shown in Fig. 7.46(a) that can be
forms the following control functions: used for the measurement of voltage. The signal genera-
(i) Power and scale illumination: Turns the tor is used to produce a 1000 Hz sine wave. The AC
instrument on and controls illumination of voltmeter and the leads to the vertical input of the oscil-
the screen. loscope are connected across the generator’s output. By
(ii) Focus: Adjusts the focus the spot or trace on adjusting the horizontal sweep time/cm and trigger, a
the screen. steady trace of the sine wave may be displayed on the
(iii) Intensity: Regulates the brightness of the spot screen as shown in Fig. 7.46(b). The trace represents a
or trace. plot of voltage vs. time. The vertical deflection of the
  2.  Vertical amplifier section: This section com- trace about the line of symmetry (DC) is proportional
prises of the following control functions: to the magnitude of the voltage at any instant of time.
(i) Position: Controls vertical positioning of oscil-
loscope display.
Voltmeter
(ii) Sensitivity: Selects the sensitivity of the verti-
cal amplifier in calibrated steps. AC
(iii) Variable sensitivity: Provides a continuous Signal
range of sensitivities between the calibrated generator Vertical
steps. input
(iv) AC-DC-GND: Selects the desired coupling
( AC or DC) for incoming signal applied to (a)
the vertical amplifier or grounds (GND)the
amplifier input. If DC coupling is selected, the
input is directly connected to the amplifier.
When AC coupling is selected, the signal is
first passed through a capacitor to block out
any constant or DC component, before enter- C Vm D
ing the amplifier. Vp-p
  3.  Horizontal sweep section: This section com-
prises of the following control functions:
(i) Sweep time/cm: Selects the desired sweep
(b)
rate from calibrated steps or admits external
signal to horizontal amplifier. Figure 7.46 |   Measurement of voltage.

Chapter 7 Theory.indd 641 3/23/2016 1:25:21 PM


642     Chapter 7:  ELECTRICAL AND ELECTRONIC MEASUREMENTS 

To determine the size of the voltage signal appearing at 7.8.4.1 Lissajous Figures
the output of terminals of the signal generator, an AC
voltmeter is connected in parallel across these terminals. When the inputs to the horizontal and vertical amplifi-
The AC voltmeter is designed to read the effective DC ers are sine-wave signals of different frequencies, a sta-
value of the voltage. This effective value is also known as tionary pattern is formed on the CRT. When the ratio
the rms value of the voltage. The peak or maximum volt- of the two input frequencies is an integral fraction such
age is Vm volts and is represented by the distance from 1 2 4 1
as , , , , etc., these stationary patterns are known
the symmetry line CD to the maximum deflection. The 2 3 3 5
magnitude of the peak voltage displayed on the oscil- as Lissajous figures and can be used for comparison and
loscope is related to the effective or rms voltage (Vrms) measurement of frequencies (Fig. 7.47). Using two oscil-
displayed on AC voltmeter as lators of different frequencies, some simple Lissajous
figures can be generated, like those shown in Fig. 7.48.
Vrms = 0.707 Vm (for a sine or cosine wave).

Vrms
Vm =
0.707
For a symmetric sinusoidal wave, the value of peak volt-
age Vm can be taken as 1/2 the peak to peak of the volt-
age signal (Vp-p). (a) (b)
Note: The mathematical realtion for rms signals is valid (a) (b)
only for sinusoidal signals.

7.8.4  Measurement of Frequency


(c) (d)
With the application of horizontal sweep voltage, the (c) (d)
voltage measurements can be obtained from the ver-
tical deflection and the signal can be displayed as a Figure 7.47 |   Lissajous figures for horizontal-to-vertical
function of time. If the time base (or the, sweep) is frequency ratios of (a) 1:1, (b) 2:1, (c) 1:2
calibrated, such that measurements of pulse duration and (d) 3:1.
or signal period can be made, then frequency of the
signals can then be determined as reciprocal of the Frequency ratio y/x
time period.
To measure the frequency of a signal, the following
steps are required: 1
  1.  Set the oscillator to 1000 Hz and display the signal
on the CRO.
  2.  Then set the horizontal gain so that only one com- 2
plete wave form is displayed.
  3.  Measure the period of the oscillations using the
horizontal distance between two points such as C
to D as shown in Fig. 7.46(b). 3
  4.  Then reset the horizontal gain until five waves are
seen on the screen. Keep the time base control in a
calibrated position.
  5.  Measure the distance (and hence time scale) for 1.5
five complete cycles and calculate the frequency
from this measurement.

1 1.33
f (Hz) = seconds
T
0° 45° 90° 135° 180°
  6.  Repeat the measurements for other frequencies
Phase difference x-y
(e.g. 150 Hz, 5 kHz and 50 kHz) as set on the
signal generator. Figure 7.48 |   Lisajous patterns.

Chapter 7 Theory.indd 642 3/23/2016 1:25:27 PM


7.9  Q-METER     643

Frequency Measurement 1
A signal generator can be used to measure the frequency Y2 (t) V2 sin(wt+f) Y1 (t) Vy
of an unknown sinusoidal signal. It is connected to the 0 0
Vx
vertical amplifier (or horizontal) and the calibrated
signal source of frequency is fed to the horizontal ampli- −1
0 0 Y2 (t)
fier (or vertical). The frequency of the signal generator is
adjusted so that a steady Lissajous pattern is obtained. T − 2p
w
If fv and fh are the frequencies of the signals applied
to vertical and horizontal amplifiers, respectively, then Y1 (t)
these are related to the number of tangencies (points
at the edge of arcs) along the vertical and horizontal
lines as
V1 sinwt
fh Number of vertical tangencies(crossings)
=
fv Number of horizontaltangencies(crossings)

If fh is known, the unknown frequency fv can be calcu-


lated using the above relation.
t=0
Note: It is difficult to maintain the Lissajous figures in a −1 0 1

Figure 7.49 |   Measurement of phase difference.


fixed configuration because the two oscillators are not in
phase and frequency locked. Their frequencies and phase
drift slowly causes the two different signals to change
slightly with respect to each other.
7.9  Q-METER
7.8.5  Measurement of Phase Difference
The quality factor (Q) of a coil is the ratio of reactance
When both applied waveforms are sinusoidal, the result- to resistance in a frequency dependent circuit configura-
ing Lissajous pattern may take many forms depending tion. The Q factor or quality factor of an inductance is
upon the frequency ratio and phase difference between commonly expressed as the ratio of its series reactance to
the waveforms. If straight line is obtained, the phase its series resistance. For inductor in series or parallel, the
angle difference will either be zero or 180°. However if an ratio of reactance to resistance in a frequency dependent
ellipse is obtained, the phase difference between the two circuit configuration is given by
signals can be determined from the Lissajous figure. The
formation of ellipse is illustrated in Fig. 7.49. Xs wLs
Q= =
Let the two sinusoidal voltage signals be given by Rs Rs

Vx = V1 sin wt Q=
Rp
=
Rp
wLp
Vy = V2 sin (wt + f )
Xp
The value of Q varies from 5 to 1000.
where f is the phase difference. The sinusoids are written The Q factor of a capacitance is the ratio of its series
as Vx and Vy as these voltages are put into horizontal reactance to its series resistance, although for capaci-
and vertical axes of the oscilloscope respectively. tors, generally dissipation factor (D) is used which is the
Since deflection is directly proportional to the ampli- reciprocal of Q. For capacitor in parallel or series, the
tude of voltage, we have from the figure ratio of reactance to resistance in a frequency dependent
circuit configuration is given by
Y1 X
sin f = = 1 Xp 1
Y2 X2 D= =
Rp w Cp Rp
Note: The ellipse forms can be used to determine only
Rs
the phase angle between two sinusoidal voltages. It D= = w Cs Rs
does not indicate which one is leading and which one is Xs
lagging. The value of D varies from 10-4 to 0.1

Chapter 7 Theory.indd 643 3/23/2016 1:25:35 PM


644     Chapter 7:  ELECTRICAL AND ELECTRONIC MEASUREMENTS 

Unknown coil

Tunable Lx
High
signal Low
V1 resistance Cx impedance V2
source C
(R) interface
(Oscillator)

Figure 7.50 |   Schematic diagram for Q-meter.

A Q-meter is used to measure the Q-factor of a coil and An active transducer is one which does not require
related electrical properties. It is based on the principle that any power source for operation. The input of physical
Q-factor of a resonant circuit is equal to its voltage magni- quantity generates a proportional electric signal. A pas-
fication factor and can be expressed as the ratio of voltage sive transducer requires external power source for opera-
developed across its reactive elements to the voltage injected tion. The output signal represents variation of electrical
in series with the circuit to produce the developed voltage. parameters (R, C, etc.) and needs to be converted into
The basic circuit for a Q-meter is shown in Fig. 7.50. equivalent current or voltage signal.
The circuit contains terminals for connecting the induc-
tance (Lx) to be measured and this is brought in resonance 7.10.1  Strain Gauge
by a variable tuning capacitor (C). There are terminals
available for adding capacitance (Cx), if required. A strain gauge is an instrument used to measure strain
When an unknown coil is connected to the test termi- produced on a wire by a force generated by varying the
nals, the circuit is excited by a tunable signal source. electrical resistance of the wire. It is an example of a passive
This is achieved by setting the oscillator to a given fre- transducer. It can effectively measure strain, displacement,
quency and varying the internal capacitor (C) or adding weight, pressure or mechanical force. A bonded strain
a capacitor Cx of the desired value and adjusting the gauge is made of fine wire looped from side to side on a
frequency of the oscillator. This leads to development of mounting plate which is attached to the element which is
voltage across a resistor in series with the tuned circuit. experiencing the stress. Consider the equation of resistance,
The resistance of the resistor should be very small (frac- rL
R=
tional in ohms) so that it can be neglected in comparison A
to the loss resistance of the components to be measured. where r is the specific resistance of conductor wire. L is
The AC injection voltage (Vin) across the series resistor the length of the conductor (in m) while A is the cross
and the AC output voltage (Vout) across the terminals of sectional area (in m2). When under strain, the length will
the tuning capacitor are measured using voltmeters. The increase and the area will decrease. As a result, the value
circuit for output measurement must be a high input of resistance will increase. The gauge factor is given as
impedance circuit so that loading of the tuned circuit by ∆R
the metering circuit is prevented.
k= R
The Q-factor is measured by adjusting the source fre- ∆L
quency and/or the tuning capacitor for a peak output L
voltage corresponding to resonance. Q-factor is deter- where, Δ terms the change in resistance and length when
mined as the ratio of output voltage measured across the pressure is applied.
tuned circuit to voltage injected into it. When Lx and Cx
are at resonance:
V 7.10.2  Velocity Transducers
Q = out
Vin
Velocity transducers can be of two types, translational
The voltage across the variable capacitor is measured and angular transducers. Velocity is often measured by
with electronic voltmeter V2, in which the scale is cali- converting it into frequency. The conversion is usually
brated to read Q directly. done by a strip or disc on which a large number of mark-
ings have been placed at equal distances x. The velocity
can be calculated as,
7.10  TRANSDUCERS
∆xn
V = = ∆xf
t
An electrical transducer is a device which can convert a where, n is the number of detection elements which
non-electrical physical quantity into an electrical quantity, passes the detector in t seconds. f is the frequency of
like current or voltage and generates an electrical signal. output signal.

Chapter 7 Theory.indd 644 3/23/2016 1:25:41 PM


7.10  TRANSDUCERS     645

This is usually performed optically, mechanically, Figure 7.51 shows a typical configuration of thermocouple.
inductively or capacitively.
Wire A
7.10.2.1 Tachometer
Hot Cold
Tachometers are used to measure speeds or rotational
movements, revolutions per minute, or sometimes they T1 T2
can be used to measure rate of flow. A scale factor can be Wire B
Figure 7.51 |   A typical thermocouple.
applied to produce readings of the desired type. They may
be the analog or digital types with contact or non-contact
types. Tachometers can be energised by either AC or DC.
The temperature ranges of thermocouple vary with the
metals used for making them. Temperature ranges of
7.10.3  Temperature Sensing Devices some common thermocouples are listed below:

7.10.3.1 Resistance Temperature Detector


Chromel-Alumel −270 to 1370oC
(RTD)
Chromel-Constantan −270 to 790oC
RTDs are temperature detectors that are based on the
change in electrical resistance of some materials with Iron-Constantan −210 to 1050oC
−270 to 400oC
change in temperature. The resistance changes linearly
Copper-Constantan
with temperature, within a limited temperature range.
The relation is given as, Nicros-Nisil −260 to 1300oC

R T = R0 + 1 + a (T − T0 )
The temperature measured can be expressed as a poly-
where, RT is the resistance at temperature T in ohms. R0 nomial of the measured voltage. Sometimes it is possible
is the resistance at standard temperature T0 and a is the to get a linear approximation over a limited temperature
temperature coefficient of resistance in °C−1.
range which can be given by,

The RTDs are encapsulated in probes and are usually E = c(T1 − T2 ) + k(T12 − T22 )
made of platinum as it has high chemical stability and
highly reproducible electrical properties. Although they where, c and k are thermocouple material constants,
are accurate and stable, they have higher initial cost. T1 and T2 are temperatures of the hot and cold or refer-
They are known to be less rugged in vibration locations. ence junctions respectively.

7.10.3.2 Thermistors 7.10.4  Transducers for Fluid Flow Measurement


Thermistors are also temperature sensing devices. They 7.10.4.1 Pitot Tube
can be negative temperature coefficient type used for tem-
perature sensing or positive temperature coefficient type Pitot tube is extensively used for velocity measurement
used for temperature control. They have a typical tem- in aircraft. It works on the principle that if a blunt object
perature range of −70 to 300°C (~ −100°F to 600°F). is placed in the flow channel, the velocity of fluid before
Thermistors are widely accepted as the most advanta- it will be zero; considering the fluid to be incompressible.
geous and less costly sensors for a number of applications Thus from Bernoulli’s equation,
for temperature measurement and control.
p1 v12 p v2
+ = 2 + 2
7.10.3.3 Thermocouples g 2g g 2g

Thermocouple is a temperature sensing device which where, p1 and p2 are the pressure of the two regions in
works on the principle of Seebeck effect. This effect the tube. Also, as v2 = 0,
causes a voltage generation in a circuit containing two
2g
different metals with their junctions kept at different v1 = ( p2 − p1 )
temperatures. These devices are rugged but sensitive g
and widely as they are inexpensive and can be used over
a wide temperature range. A typical Pitot tube is shown in Fig. 7.52.

Chapter 7 Theory.indd 645 3/23/2016 1:25:47 PM


646     Chapter 7:  ELECTRICAL AND ELECTRONIC MEASUREMENTS 

7.10.4.3 Rotameter

Rotameter works as a constant pressure drop and vari-


able area meter for measuring fluid flow. It is easy to
install and simple in construction but has lower accuracy
and can only be installed in vertical pipelines.

7.10.4.4 Venturimeter
p1 p2
Venturimeter can measure horizontal flow rate and has
Figure 7.52 |   Pitot tube. two pressure taps. It has high mechanical strength but
has higher cost. Figure 7.54 shows a venturimeter.
7.10.4.2 Orifice Meter

Orifice meter is the most common type of instruments


used to measure the fluid flow. Here an orifice plate is
used in a pipeline. Figures 7.53(a) and (b) show an ori-
fice meter and the structure of orifice plate respectively.
The volumetric flow rate Q in an orifice meter can be
p2
obtained at different diameters d1 and d2 of the orifice
plate as, p1
A2 2g Figure 7.54 |   Venturimeter.
Q = v2 A2 = ( p − p2 )
1− b 2 g 1
7.10.5  Capacitive Transducer
where, b = d2/d1, A is cross-sectional area. g and g are
Capacitive transducer can be used to measure force. It
acceleration due to gravity and specific weight of fluid
has a static plate and a flexible diaphragm with a dielec-
respectively.
tric placed between them (Fig. 7.55). A force exerted
on the diaphragm will cause a change in distance between
Flow profile the diaphragm and the static plate causing a change in
Orifice plate
Vena contacta capacitance. This change in capacitance is measured
using a bridge circuit or tank circuit.

Diaphragm Static plate


Flow
d1 d2
Insulating
material

Pressure

p1 p2
(a)
Dielectric
Figure 7.55 |   Capacitive transducer.

7.10.6  Piezo-Electric Transducer

Piezo-electric transducer is used to measure pressure as


a function of voltage generated. Force and acceleration
(b) can also be measured using this transducer. As the name
Figure 7.53 |   (a) Orifice meter. (b) Orifice plate. suggests, it contains a piezo-electric crystal in which an

Chapter 7 Theory.indd 646 3/23/2016 1:25:54 PM


7.11  ERROR ANALYSIS     647

electric charge is produced on the surface on application 7.10.8  Bellows


of force. Voltage produced will be proportional to the
applied force. Usually a quartz crystal is a piezoelectric Bellows are made of soft material and are pressed to form
crystal. An instrument amplifier is used on its output for convolutions. One end is fixed where air can go inside
measurement purpose. Figure 7.56 shows a piezoelectric using a port while the other end is free to move (Fig. 7.58).
transducer. A spring is used often to counter the bellow movement.
The pressure to be measured can be obtained as,
Pressure port kx
p =
A
Force
summing where, A is bellow area, k is spring constant and x is
member displacement of the bellow.

Quartz Output k
crystal p
voltage
Base x
Figure 7.56 |   Piezo-electric transducer. Figure 7.58 |   Bellows.

7.10.7  Bourdon Tube


7.11  ERROR ANALYSIS
Bourdon tube is used for pressure measurement. It con-
sists of a C-shaped hollow tube whose one end is fixed
and connected to a pressure tapping while the other end Error analysis is an essential part of measurement. Error
is kept free (Fig. 7.57). When a pressure is applied, the in measurement can be defined in as,
circular tube tries to straighten, which makes the free
end to move. A deflecting mechanism attached to an Error = Instrument reading — true reading.
indicating instrument is attached to the free end that
Percentage error
moves a pointer. The materials commonly used for this
Instrument reading − true reading
tube are brass, phosphor bronze, and beryllium-copper. = ×100
They can measure very high differential pressures of True reading
700 MPa.
7.11.1  Types of Errors

There can be three types of errors, namely gross error,


systematic error and random error and these are
explained as follows.
er
int   1.  Gross error: It arises due to human mistakes lead-
Po
ing to wrong reading of values, mistake in record-
ing measured data, parallax error, etc.
  2.  Systematic error: Zero error and instrument bias
Free end of are source of systematic error. There may be con-
the tube structional error in instruments. They may be due
to error in construction of the instrument scale,
Deflecting pointer, etc. This error can lead to error in every
mechanism reading taken while measuring.
  3.  Random error: It occurs when the exact cause
of error cannot be determined. These can never be
corrected but can be reduced to a certain extent by
p averaging or finding the error limits. The random
Figure 7.57 |   Bourdon tube. errors have a deviation of the readings and it

Chapter 7 Theory.indd 647 3/23/2016 1:26:00 PM


648     Chapter 7:  ELECTRICAL AND ELECTRONIC MEASUREMENTS 

follows a particular distribution which is generally deviation or variance can now be used to measure the
normal distribution. deviation from a set of readings as,

1 n
7.11.2  Mean Value and Deviation V = ∑ (x − x)2 = s 2
n − 1 i=1 i
Mean value is the most probable value for a set of read-
ings, which can be given as, where, s is the standard deviation.
Calibration is the process of comparing the
1 n
x = ∑ xi ­ easured value obtained from a particular instru-
m
n i=1 ment to that of a standard instrument. Comparison of
actual input values with the output indication of the
where, n is the total number of readings and xi is the system will result determination of systematic errors.
individual readings value. The deviation d of readings Errors at these calibrating points are then reduced
from this value can be derived as, by adjusting the components or by using calibration
charts.
di = xi − x
The true values can also be obtained from standard
look up tables prepared for this purpose. Calibration can
7.11.3  Precision and Calibration be re-performed from time to time to reduce the effects
of instrument wear over time.
The precision of measurement is sometimes defined as
this deviation from the mean value. The mean square

IMPORTANT FORMULAS

1. Types of torque in indicating instruments 4. Electrodynamic instrument:


(a)  Deflecting torque
dM
Deflecting torque, T = I1I2
dq
Td ∝ Operating quanity
5. AC bridges: If Z1, Z2, Z3 and Z4 are the impedance
(b)  Controlling torque
of AC bridge arms, then at balance point:

Tc = kcq (spring control) EBA = EBC


I1Z1 = I2Z2
Tc = kgr Wl sinq (gravity control) V V
I1 = and I2 =
Z1 + Z3 Z2 + Z4
(c)  Damping torque
6. Measurement of inductance
dq
Tdamp = kdamp (a)  Maxwell bridge
dt

and Z2 = R4 + jwL1
1
2. Permanent magnet moving coil (PMMC) instrument: Z1 = R2 +
jwC1
Deflecting torque T = NbAI
R1R3
3. Moving iron type instrument: R1R3 = R2R4 ⇒ R4 =
R2
I 2dM
Deflecting torque, T =
2dq (b)  Hay’s bridge

and Z1 = R1 + jwL1
1
I 2 dM
Deflecting angle, q =
Z4 = R4 +
jwC4
2k dq

Chapter 7 Theory.indd 648 3/23/2016 1:26:16 PM


IMPORTANT FORMULAS     649

R2R3C4 (b)  Heaviside-Campbell bridge


L1 =
(wR4C4 )
2
M1 − M 2
1 +
L2 =
(wC4 ) 2R2R3R4 1 + R3 / R4
and R1 =
1 + (wR4C4 )
2
    9. Measurement of resistance
(c)  Owen’s bridge (a)  Kelvin double bridge

1 R1
Z4 =
jwC4 R3 = R
R2 4
1
Z2 = R2 + (b)  Ammeter-voltmeter method
jwC2
V V + VA V
Z1 = R1 + jwL1 R= = x = Rx + A
I I I
C4R3 V V Rx
L1 = R2R3C4 and R1 = R= = =
C2 I Ix + IV I
1+ V
Ix
(d)  Anderson bridge
(c)  Wheatstone bridge
R2R3 R
R1 = and L1 = C 3 [R(R2 + R4 ) + R2R4 ]
R4 R4 R2R3
R4 =
R1
7. Measurement of capacitance
(a)  The De Sauty’s bridge (d)  Carey—Foster bridge

C1 = C2
R1 R4 (l1′ − l2′ ) − R4′ (l2 − l1 )
R3 =
R2 l1′ − l2′ − l1 + l2
(b)  Schering bridge
(e)  Direct deflection method
R4 1
Z4 = and Z1 = + R1
(1 + jwC4R4 ) jwC1
V
R =
IR
R4C2
C1 =
R3 (f)  Loss of charge method

(c)  Wein’s bridge


V = ve(−t /CR) ⇒ = e(−t /CR)
V
R4 R C v
= 2 + 1
R3 R1 C2
t
R=
C lnV
p
1 1
f=
2p
v
R1R2C1C2
10. Measurement of current and voltage
8. Measurement of mutual inductance
(a)  Extension of ammeter range
(a)  Heaviside bridge
I R
L R − L2R3 = 1+ m = m
M = 1 4 Im Rsh
R3 + R4

Chapter 7 Theory.indd 649 3/23/2016 1:26:41 PM


650     Chapter 7:  ELECTRICAL AND ELECTRONIC MEASUREMENTS 

Rm Rm 14. Q-Meter
Rsh = =
(m − 1)  I 
 − 1 Vout
 I m  Q=
Vin
where m is multiplying factor or instrument
constant 15. Transducers
(b)  Extension of voltmeter range (a)  Strain gauge

V R DR / R
= s +1 k=
I m Rm Rm DL / L

(c)  Sensitivity of ammeter and voltmeter (b)  Velocity transducer

∆xn
Rs + Rm V = = ∆xf
Sensitivity = t
Full scale deflection (FSD)
(c)  Resistance temperature detector
11. Measurement of power and energy in AC circuits
(a)  Dynamometer type wattmeter RT = R0 + 1 + a (T − T0 )

P = Vrms I rms cos f (d)  Thermocouple

(b)  Induction type energy meter E = c(T1 − T2 ) + k(T12 − T22 )

(e)  Pitot tube


Number of revolutions = K×Energy in kWh
2g
where K is meter constant. v1 = ( p2 − p1 )
g
(c) In three-phase (star) circuits total power
consumed by the balanced load is equal to
(f)  Orifice meter
(W1+W2).
A2 2g
  W − W  Q = v2 A2 = ( p − p2 )
12. Power factor angle f = tan−1  3  2 1  1− b 2 g 1

  W2 + W1 
16. Error analysis
13. Cathode ray oscilloscope (CRO)
Percentage error
(a)  Voltage measurement Instrument reading − true reading
= ×100
True reading
Vrms
Vm =
0.707
1 n
(b)  Frequency measurement
(a)  Mean x = ∑x
n i=1 i

fh Number of vertical tangencies (crossings) (b)  Deviation di = xi − x


=
fv Number of horizontaltangencies (crossings)
(c)  Variance (mean square deviation)
(c)  Measurement of phase difference
1 n
V = ∑ (x − x)2 = s 2
n − 1 i=1 i
Y1 X
sin f = = 1
Y2 X2

Chapter 7 Theory.indd 650 3/23/2016 1:27:03 PM


SOLVED EXAMPLES     651

SOLVED EXAMPLES

Types of Indicating Instruments a current of 10 A. Given that the mutual induc-


tance at 0° deflection is 2 µH and the change in the
1. The coil of a PMMC instrument has dimensions mutual inductance is linear with deflection.
15 mm × 12 mm. The flux density is given as
(a) 4.42 µH (b) 7.2 µH
1.8 × 10−3 Wb/m2, with the number of turns as 136.
(c) 2.21 mH (d) 2.21 µH
The current through the coil is 5 mA. Determine
the value of deflecting torque produced in Nm.
Solution:  We have
(a) 1.62 × 10−9 (b) 1.74 × 10−9
(c) 2.20 × 10−7 (d) 2.52 × 10−7 dL k × full scale reading
=
dq I2
Solution:  Deflecting torque 110 × 0.1 × 10−6
BNAI = 1.8 × 10−3 × 136 × 15 × 12 × 10−6 × = = 0.21 m H
5 × 10−3 = 2.20 × 10−7
100
Ans. (c) Thus total inductance = 2 + 0.21 µH = 2.21 µH
Ans. (d)
2. For the PMMC instrument in Question 1, calcu-
late the spring constant in Nm/rad if the deflection
is 90°. Bridges and Potentiometers

(a) 0.14 × 10−6 (b) 0.198 × 10−6 5. A Maxwell’s capacitance bridge is used to mea-
(c) 0.24 × 10−6 (d) 0.18 × 10−6 sure an unknown inductance in comparison with a
known capacitance as shown in the figure below.
R2 = 400 Ω, R3 = 600 Ω, R4 = 1000 Ω, C4 = 0.5 µF.
Solution:  Let the spring constant be k. Then
2 2 2
I1 = (I m ) + (I1 ) = (80) + (1000) = 1003.2 A2 Calculate the inductance L1.

k ×q = 2.20 × 10−7× ⇒ k = 0.14 × 10−6 Nm/rad


180 L1
p R1
Ans. (a) R3
3. An electrostatic voltmeter reading up to 2000 V is
controlled by a spring with a torsion constant of
5 × 10−6 Nm/rad has a full-scale deflection of 90°.
E D
The capacitance at zero voltage is 15 pF. What is C4
the change in capacitance when the pointer indi-
R2
cates 2000 V?
R4
(a) 21.17 pF (b) 6.17 pF
(c) 8.83 pF (d) 3.93 pF
(a) 0.24 H (b) 0.46 mH
Solution:  We have
(c) 0.12 H (d) 0.12 mH
V 2 dC
q=
2k dq Solution:  For a Maxwell bridge

90 × 2 × 10−6 p L1 = R2 × R3 × C4 = 400 × 600 × 0.5 × 10−6


= 3.93 × 10−12 F
dC
=
(2000)
dq 2 180 = 0.12 H
Ans. (c)
Ans. (d)
6. Calculate the value of resistance R1 from the
4. A 10 A electrodynamic ammeter is controlled by
a spring having a constant of 0.1 × 10−6 Nm/
Maxwell bridge given in Question 5.
degree. The full scale deflection is 110°. Determine (a) 240 Ω (b) 24 Ω
the inductance of the instrument when measuring (c) 2.4 kΩ (d) 240 kΩ

Chapter 7_Solved Examples and Practice exercise.indd 651 3/23/2016 1:29:22 PM


652     CHAPTER 7:  ELECTRICAL AND ELECTRONIC MEASUREMENTS

Solution:  The resistance for the Maxwell bridge 100


can be determined as V1 = 100 V, 5 mA ⇒ R V 1 = × 103 = 20 kW
  5
RR 600 × 400
R1 = 2 3 = = 240 W V2 = 100 V, 250 Ω/V⇒ R V2 = 250 × 100 = 25 kV
R4 1000
Ans. (a) Thus, R1 = 10 kΩ  20 kΩ = 6.67 kΩ
7. An Owen’s bridge is used to measure the impedance
of a steel specimen at 2 kHz. Given that R2 = 10 kΩ  20 kΩ = 7.14 kΩ

Thus,
R2 = 834 Ω, C2 = 0.124 µF,
R3 = 100 Ω, C4 = 0.1 µF. 6.67
Find the effective impedance. V1 = 100 × = 48.3 V
7.14 + 6.67

C4 V2 = 100 − 48.3 = 51.7 V


R3
7.14
V2 = 100 × = 51.7 V
6.67 + 7.14
D
Ans. (b)
C2 Lx
R2 Rx 9. For the given circuit, if the voltmeter reads 60 V,
the value of the unknown resistance R will be
(a) 132 Ω (b) 264 Ω +
(c) 164 Ω (d) 72 Ω 100 kΩ V
100 V
Solution:  The effective impedance is DC R
Z1 × Z4 = Z2 × Z3 ⇒ Z1 = 132.24 W

Ans. (a) (a) 2.2 kΩ (b) 50 kΩ
(c) 32.2 Ω (d) 100 Ω
Measurement of Current and Voltage
Solution:  Voltmeter reading = 60 V
8. The specifications of the two voltmeters are
Thus,
V1 = 100 V, 5 mA
100
V2 = 100 V, 250 Ω/V × R = 40
50 × 100
× 10 + R
3
150
10 kΩ 10 kΩ
Solving we get R = 2.2 kΩ
V1 V2 Ans. (a)
100 V DC voltmeter 1 DC voltmeter 2 10. A moving coil instrument of resistance 5 Ω requires
a potential difference of 75 mV to give a full scale
deflection. The value of shunt resistance needed to
The readings of the two voltmeters, respectively give a full scale deflection at 30 A is
will be
(a) 2.5 mΩ (b) 9.99 mΩ
(a) 50 V, 50 V (b) 48.3 V, 51.7 V
(c) 5 Ω (d) 9.95 Ω
(c) 51.7 V, 48.3 V (d) 55.3 V, 44.7 V
Solution:  Consider the following circuit Solution:  Consider the following circuit
10 kΩ 10 kΩ 75 mV

MC
I = 30 A 5Ω I1
100 V 20 kΩ 25 kΩ
R1
Rsh Ish

Chapter 7_Solved Examples and Practice exercise.indd 652 3/23/2016 1:29:42 PM


SOLVED EXAMPLES     653

Full scale meter current Thus, equivalent resistance


= RV || 100×103= 61.54 kΩ
V 7.5 × 10−3 61.54 × 200
I1 = = A = 15 mA Reading of the voltmeter = = 39.5 V
R 5 250 + 61.54
Thus, for the circuit Ans. (b)

15 × 10−3 = 30 ×
Rsh 13. For the circuit given in Question 12, find the error in
5 + Rsh the reading expressed as a percentage of true value.

Rsh = 2.5 mW (a) 5.76% (b) −30.74%


(c) −41.82% (d) 2.24%
Ans. (a)

11. A 0.5 Ω resistance is required to be connected in Solution:  For ideal voltmeter, the reading
parallel to a moving coil instrument whose full 200 × 100
= = 57.15 V
scale deflection is 1 mA; so that this instrument 350
100
can measure 10 mA current. Internal resistance of Error = (−57.15 + 39.5) × = −30.7%
this instrument is 57.5 Ans. (b)
(a) 5.0 Ω (b) 4.5 Ω 14. The galvanometer in the following circuit has resis-
(c) 2.25 Ω (d) 0.45 Ω tance of 10 mΩ and can measure a minimum cur-
rent of 5 µA. The minimum value of Rx that can
Solution:  Consider the circuit for the moving coil be measured is
instrument
1 mA I 100 Ω 100 Ω
MC
Rm 10 mA
G
Rsh = 0.5
Rx
Then 100 Ω 100 Ω
× 10 × 10−3 = 1 × 10−3
0.5
R m + 0.5
⇒ R m = 4.5 Ω
10 V
Ans. (b) (a) 200 µΩ (b) 2 µΩ
(c) 10 µΩ (d) 1 µΩ
12. The voltage across a 100 kΩ resistor is to be mea-
sured as shown in the following circuit. The sensi- Solution:  Consider the following equivalent circuit
tivity of the voltmeter is 2000 Ω/V. The range of V1
the voltmeter is 0−80 V.
250 kΩ 100 Ω 5 µA 100 Ω
+
10 mΩ
+
200 V 100 kΩ V G

100 Ω Rx
− 100 Ω

Calculate the reading of the voltmeter. V2


(a) 95.24 V (b) 39.5 V
(c) 158.18 V (d) 102.24 V 10 V
V1 − V2
Then
−3
= 5 × 10−6
Solution:  For the given circuit: 10 × 10
RV = Full scale deflection × Sensitivity  
= 80 × 2000 = 160 kΩ, V1 − V2 = 50 × 10−3

Chapter 7_Solved Examples and Practice exercise.indd 653 3/23/2016 1:29:55 PM


654     CHAPTER 7:  ELECTRICAL AND ELECTRONIC MEASUREMENTS

Applying node voltage equation at V1 and V2, we get (a) 10 W (b) 11.11 W
(c) 0 W (d) 110 W
V1 − 10 V V − V2
+ 1 + 1 (i)
100 100 10 × 10−3 Solution:  Consider the following equivalent circuit

V2 − V1 V2 V − 10 100 Ω
−3
+ − 2 =0 (ii)
10 × 10 100 + Rx 100

Solving, we get Rx = 2 Ω
10 Ω
R
Ans. (b) V

15. A moving coil instrument having internal resis-


tance of 25 Ω indicates full-scale deflection with a
current of 10 mA. How would the instrument func- The equivalent source resistance
tion as a voltmeter to read 100 V on full scale? 100 R
Rs = W
Solution:  Given that: 100 + R
Resistance of the instrument coil, R m = 25 Ω
Current flowing through the instrument for full-scale PL = I L2 × 10
deflection, I m = 10 mA = 0.01 A.
Then series resistance required to measure 100 V 100
IL =
Rs + RL
− 50 = 9975 Ω
V 100
Rse = − Rm =
Since RL is fixed at 10 Ω, Rs has to be minimum
Im 0.01
Ans. (9975) to have maximum IL and maximum power at load
Thus
16. How can the moving coil instrument given in
Question 15 work as an ammeter of 1  A on full R s (min) = 0 for R = 0
scale?
Ans. (c)
Solution:  To function as an ammeter a shunt
required to measure 1 A current would need to be 18. For the circuit given in Question 17, the maximum
added to the circuit. It can be calculated as: power will be
Multiplying power of shunt (a) 10 W (b) 1000 W
I 1 (c) 0 W (d) 100 W
m= = = 100
Im 10 × 10−3
Solution:  Maximum power
Shunt resistance required for full-scale deflection
100 2
P =  × 10 = 1000 W
 10 
at 100 A
Ans. (b)
Rm
= 0.2525 Ω
25 25
R sh = = =
m − 1 100 − 1 99 19. Two wattmeters are to be connected to measure
Ans. 0.2525 three-phase power in a three-phase system. The
connections are given in the figure below. When
Measurement of Power and Energy the system is balanced, the wattmeters will read as

17. For the following circuit, the value of R so that the W1


wattmeter reads the maximum value is i
A
100 Ω
W
i Load
B

10 Ω
R
100 V i
C
W2

Chapter 7_Solved Examples and Practice exercise.indd 654 3/23/2016 1:30:18 PM


SOLVED EXAMPLES     655

(a) Both will read the same value always phase sequence is ABC. If the wattmeter with its
(b) W1 = 0 always current coil in A-phase line reads zero, then the
(c) W2 = 0 always power factor of the three-phase load will be
(d) W1 + W = 0 always with resistance of 1 W (a) zero lagging (b) zero leading
(c) 0.5 lagging (d) 0.5 leading
Solution:  From the phasor diagram
Solution:  The two wattmeter circuit and the
VCA VC corresponding phasor diagram is
i
30° C VAB
f VCB C
30° iC
f
VA
W1 VC VAB
i i 30°
A 30° VA
VB
60°
W1 = VAB i cos(30 + f )
B
iA

W2 = VCA i cos(30 + f ) C
VB
For balanced system, VAB = VCA and line cur- Given that W1 = 0, then
rents are the same. So, both will read the same
value always. W1 = VAB iA ⋅ cos 90 = 0
Ans. (a)
pf = cos 60 = 0.5 lag
20. In a balanced three-phase 200 V circuit, the line
current is 115.5 A. When the power is measured Ans. (c)
by two wattmeter method, one of the wattmeters
reads 20 kW and the other one reads zero. What is 22. Two wattmeters are used to measure the power in
the power factor of the load? a three-phase balanced system. What is the power
factor of the load when one wattmeter reads twice
(a) 0.5 (b) 0.6 the other?
(c) 0.7 (d) 0.8
(a) 0 (b) 0.5
Solution:  Consider the balanced three-phase (c) 0.866 (d) 1
­circuit shown below.
W1 115.5 A Solution:  We know that
R W1 − W2
tan f = 3
W1 + W2
Y
When, W1 = 2W2 ,
200 V B
2W2 − W2
tan f = 3 ⇒ f = 30
1
W2 =
3W2 3
Given that: W1 = 20 kW and W2 = 0
Then, pf = cos 30 = 0.866
W1 − W2
tan f = 3 = 3 ⇒ f = 60 Ans. (c)
W1 + W2
23. The meter constant of a 230 V single-phase induc-
Then pf = cos f = cos 60° = 0.5 tion type watt hour meter is 400 revolutions per
Ans. (a) kWh. For a current 10 A of 0.9 pf lagging, speed of
the meter disc will be
21. Two-wattmeter method is employed to measure
power in a three-phase balanced system with the (a) 13.80 rpm (b) 26.16 rpm
current coil connected in the A and C lines. The (c) 18.20 rpm (d) 24.30 rpm

Chapter 7_Solved Examples and Practice exercise.indd 655 3/23/2016 1:30:43 PM


656     CHAPTER 7:  ELECTRICAL AND ELECTRONIC MEASUREMENTS

Solution:  Power is given by Secondary winding current, I2 = 5 A


W = VI cos f = 230 × 16 × 0.9 = 2070 W The secondary reflected current is

2070 I1 ′ = K ⋅ I2 = 200 × 5 = 1000 A


The meter makes × 400 revolutions per hour.
1000
Thus, Now primary current,
2070 400
Speed = × = 13.8 revolutions per min I1 = (I m )2 + (I1 )2 = (80)2 + (1000)2 = 1003.2 A
1000 60
Ans. (a)
1003.2
24. A single-phase energy meter operating on 230 V and Actual transformation ratio, Kc = = 200.64
5
5 A for 5 h makes 1940 revolutions. Meter constant
is 400 rev k Wh. The power factor of the load is  80 
Phase angle, f = tan−1 = tan−1 
Im
= 4°34′
(a) 1.0 (b) 0.8 I1 1000 
(c) 0.7 (d) 0.6
Ans. (200.64, 4.34°)
Solution:  For single phase energy meter
Cathode Ray Oscilloscope
Number of revolutions
Energy(kWh) =
K 26. Calculate the ratio of vertical to horizontal fre-
quencies for an oscilloscope which displays the
For 1940 revolutions, the energy is following Lissajous figures.
1940
kWh = 4.85 VA
400
So,
(i) (ii) (iii) (iv)
4.85 × 103
230 × 5 × cos f = ⇒ cos f = 0.8
5 (a) i-1:2, ii-2:1, iii-2:5, iv-3:1
Ans. (b) (b) i-2:1, ii-1:2, iii-5:2, iv-3:1
(c) i-2:1, ii-1:2, iii-5:2, iv-1:3
Instrument Transformers (d) i-1:2, ii-2:1, iii-5:2, iv-3:1
Solution: 
25. A CT has one turn primary and 200 turns second-
ary winding. The secondary carries a current of fh Number of vertical tangencies (crossings)
5 A passing through a secondary burden of 1 Ω. fv
=
Number of horiizontaltangencies (crossings)
The flux is set up in the core by current of 80 A.
The frequency is 50 Hz and cross section area of
Thus the correct option is (d)
core is 1000 sq-mm. Calculate the actual transfor-
Ans. (d)
mation ratio and phase angle for the transformer.
Solution:  For the given CT, 27. An oscilloscope has 25 mV/divisions and a square
Number of primary turns, N1 = 1. pulse is applied to its vertical plate. In the output,
the leading edge has three divisions and trailing
Number of secondary turns, N2 = 200.
edge two divisions. Find the pulse amplitude.
Impedance on secondary, Z2 = 1 Ω
(a) 125 mV (b) 62.5 mV
200 (c) 25 mV (d) 50 mV
Therefore, turn ratio, K = = 200
1
Voltage induced in secondary circuit, Solution:  The phase amplitude is given by
E2 = I2 . Z2 = 5 V.
Sum of rising and trailing edges
Also, I1 = K . I2. × calibration
Neglecting core loss component of no load current, 2
(3 + 2)
the magnetising component of no load current = = × 25 mV = 62.5 mV
80 2
mmf/primary turns. So, Im = = 80 A
1 Ans. (b)

Chapter 7_Solved Examples and Practice exercise.indd 656 3/23/2016 1:31:03 PM


SOLVED EXAMPLES     657

Transducers Solution:  We have

28. A resistance strain gauge with a gauge factor of Load = 20 × 0.9 × 240 = 4320 W = 4.32 kW
2.0 is fixed to a steel part, subjected to a stress
Actual speed = 45 rpm
of 100 N/mm2 . If modulus of elasticity of steel
is 2 × 105 N/mm2 , then the percentage change in
45 × 60
resistance is Expected load = = 4.5 kW
600
(a) 1.50 (b) 0.001
4.5 − 4.32
(c) 0.15 (d) 0.10
Percentage error = × 100 = 4%
Solution:  We know that 4.5
Ans. (b)
Stress
Strain =
Young’s modulus 31. The meter constant of a single-phase energy meter
∆L
= 50 × 10−5
100 is 500 rev/kWh. With a load of 5 kW, it makes 40
⇒ =
L 2 × 10 5 revolutions in 50 s. The percentage error is
(a) 5.25% (b) 10.5%
∆R
(c) 15.25% (d) 20%
R
Gauge factor =
∆L Solution:  Given that with load = 5 kW and revo-
L lution per hour
∆R
= 50 × 10−5 × 2 = 0.001
40
= × 3600 = 2880
R 50
Ans. (b) Expected load by meter is
29. The resistance of 125 Ω strain gauge changes by Number of revolutions = K× Energy in kWh
1 Ω when the strain is 4 × 10−3. The gauge factor
2880 = 500 × Energy in kWh
for the strain gauge is
Energy = 5.76 kW
(a) 2.0 (b) 1.5
(c) 2.5 (d) 3.0 5.76
Error = × 100% = 15.2%
∆L
= 4 × 10−3 and
5
Solution:  Given that
L Ans. (c)
∆R 124
= . Then gauge factor is 32. A voltmeter of scale 0−300 V has an error at zero
R 125
of 1% of its full scale reading. The voltage measured
∆R 124 through this instrument is 200 V. The maximum
limiting error is
= 125−3 ≈ 2.5
R
Gauge factor =
∆L 4 × 10 (a) 1% (b) 1.5%
L (c) 0% (d) 0.67%
Ans. (c)
Solution:  The assuming is 1% of full scale, that
Error Analysis 1
is, × 300 V = 3 V.
100
30. The meter constant of a single phase 240 V energy
meter is 600 rev/kWh. The percentage error of
Thus, for an error of 3 V at a reading of 200 V the
reading when running at a speed of 45 rmp at a
3
load of 20 A, 0.9 pf lag will be percentage error is × 100% = 1.5%
200
(a) +1.16% (b) 4%
(c) −4% (d) −1.16% Ans. (b)

Chapter 7_Solved Examples and Practice exercise.indd 657 3/23/2016 1:31:22 PM


658     CHAPTER 7:  ELECTRICAL AND ELECTRONIC MEASUREMENTS

PRACTICE EXERCISES

Set 1 (One Mark Questions) 6. If one of the control springs of a permanent magnet
coil ammeter is broken then it will read
1. In an induction type energy meter, the steady
speed attained by the revolving disc is (a) zero
(b) half the correct value
i. proportional to the deflecting torque. (c) twice the the correct value
ii. proportional to the resistance of the path of eddy (d) an infinite value
currents. 7. To minimise voltmeter loading
iii. inversely proportional to the square of brake (a) voltmeter operating current has to be very
magnet flux. small.
iv. inversely proportional to the effective readings of (b) voltmeter operating current has to be very high.
disc from its axis. (c) resistance connected in parallel with the coil
should be low.
Which of the above statements are correct? (d) resistance connected in parallel with the coil
(a) i, ii and iii only (b) i, ii and iv only should be high.
(c) ii, iii and iv only (b) i, ii, iii and iv
8. During making connections, the current and
2. In moving iron instruments, eddy current damping potential coils of a wattmeter are accidentally
cannot be used as interchanged. When the circuit is energised, the
(a) they have a strong operating magnetic. wattmeter does not show the reading. This is due to
(b) they are not normally used in vertical position. (a) damage done to the potential coil.
(c) they need a large damping force, which can only (b) damage done to the current coil.
be proved by air friction. (c) damage done to both potential and current coil.
(d) the introduction of a permanent magnet required (d) loose connection.
9. A current i = 5 + 14.14 sin(314t + 45 ) is passed
for eddy current damping would distort the
existing weak operating magnetic field.
through a centre-zero PMMC and a moving-iron
3. In Kelvin’s double bridge, two sets of readings are
instrument respectively, the respective readings are:
taken by reversing the battery terminals. This is
done to (a) −5 and 15 (b) 5 and 125
(c) −5 and 19.14 (d) 5 and 10
(a) eiminate the effect of contact resistance.
(b) eliminate the effect of thermo-electric emfs. 10. In induction type energy meter, creep error may
(c) correct for changes in battery voltages. occur due to:
(d) eliminate the effect of resistance of leads.
(a) incorrect position of brake magnet.
4. The current in primary winding of a current trans- (b) incorrect adjustment of position of shading of
former depends on shading band.
(c) overvoltage across voltage coil.
(a) voltage and power factor of secondary winding.
(d) increase in temperature.
(b) secondary winding current.
(c) load connected to the system in which CT is 11. An LVDT is used for measuring1 mm displacement.
installed. For this purpose, a voltmeter of range 0 to 2 V
(d) anyone of these. is connected at the output of the LVDT through
an amplifier having a gain of 500. Determine the
5. When the secondary winding of a voltage transformer
sensitivity of LVDT if the output is 2 mV.
(PT) is suddenly open circuited with primary
winding excited (a) 0.1 V/mm (b) 0.05 V/mm
(c) 1 V/mm (d) 0.5 V/mm
(a) the primary winding draws only the no-load
current. 12. In a low power factor wattmeter, sometimes com-
(b) the large voltages so produced may rupture the pensating coil is connected in order to
insulation.
(a) neutralise the capacitive effect of pressure coil.
(c) may be hazardous for operating personnel.
(b) compensate for power loss in pressure coil.
(d) None of these.

Chapter 7_Solved Examples and Practice exercise.indd 658 3/23/2016 1:31:24 PM


PRACTICE EXERCISES     659

(c) reduce the error caused by eddy current. 22. A signal of 20 mV at 100 kHz is to be measured
(d) compensate for inductance of pressure coil. with a meter. The type of meter most suitable for
measurement is
13. Determine the value of resistance to be added in
series with an ammeter whose full scale deflection (a) induction type (b) dynamometer type
is 0.1 mA and internal resistance is of 500 Ω, to (c) electrostatic voltmeter (d) CRO
make it suitable to measure (0−10) V is
23. A signal of 6 kV, 50 Hz is to be measured with a
(a) 0.02 kΩ (b) 99.5 kΩ voltmeter, the type of voltmeter most suitable is
(c) 500.02 Ω (d) 499.98 Ω
(a) electrostatic type (b) induction type voltmeter
14. A 0 to 300 V voltmeter has an error of ±2% for full (c) PMMC voltmeter (d) miron type voltmeter
scale reading. What is the range of readings if true
24. For the Lissajous pattern shown in the following
voltage is 30 V?
figure, if the frequency of x-axis signal is 100 Hz,
(a) 24−36 V (b) 20−40 V the frequency of y-axis signal is
(c) 29.4−30.6 V (d) None of the above
(a) 200 Hz  (b) 100 Hz  (c) 0 Hz  (d) 500 Hz
15. The deflecting torque developed by a PMMC meter
movement when N = 5000 turns, B = 3000, A = 1 cm2
and I = 70 A is
(a) 1.05 × 105 Nm (b) 105 Nm
(c) 10.5 Nm (d) 1050 Nm
16. The phase difference between the signals 25. A PMMC ammeter of 0−10 A range is used to
v1(t) = 20 sin wt and v2(t) = 20 sin(wt +f), the measure a current of 4 A. While measuring, it is
Lissajous pattern observed on the CRO will be a observed that the pointer is fixed at the maximum
circle, if the value of f is scale when the meter is connected. The possible
p p reason could be that the
(a) p   (b)    (c)    (d) 0
2 4 (a) instrument has very high deflecting torque.
17. The most stable type of ammeter to measure cur- (b) measured current is AC in nature.
rents at radio frequency is (c) control spring of the instrument is faulty.
(d) moving coil of the instrument is shorted.
(a) moving-iron type (b) moving-coil type
(c) induction type (d) thermocouple type 26. The typical power consumption of a PMMC instru-
ment is about
18. Hay’s bridge used for measurement of inductance
is suited for (a) 1−2 W (b) 10−20 mW
(c) 0−1 mW (d) 1−2 mW
(a) low Q coils (b) high Q coils
(c) medium Q coils (d) low and medium Q coils 27. The capacitive potential transformers are used
because
19. A 2000 Ω/V meter is used to measure a resistance
on 120 V scale. The meter resistance is (a) these can be used at very high voltage above
100 kV.
(a) 16.67 kΩ (b) 0.0005 Ω (b) they are cheaper.
(c) 240 kΩ (d) 240 Ω (c) of their ability to keep ratio error to a very low
20. The term phantom loading is used in connection value after making certain adjustments.
with a (an) (d) of their small.

(a) ammeter (b) wattmeter 28. The ratio and phase angle errors of a current
(c) energy meter (d) frequency meter transformer can be increased if

21. The Lissajous pattern of an oscilloscope has five hor- (a) the conductor of both primary and secondary
izontal and three vertical tangencies. If the horizon- winding are taken with large cross sections.
tal input has 120 Hz frequency then the frequency (b) the leakage reactance of both primary and
of the vertical input is secondary are increased.
(c) materials with low permeably and higher core
(a) 120 Hz (b) 60 Hz loss are used as core matter.
(c) 200 Hz (d) 100 Hz (d) the core is made of non-magnetic material.

Chapter 7_Solved Examples and Practice exercise.indd 659 3/23/2016 1:31:33 PM


660     CHAPTER 7:  ELECTRICAL AND ELECTRONIC MEASUREMENTS

29. The phenomenon of creep in a single phase induc- The meter constant is 600 rev/kWh. The number
tion type energy meter can arise due to of revolution the meter would make in 3 h is
(a) low control torque. (a) 4000 rev (b) 4200 rev
(b) over compensation due to friction. (c) 4210 rev (d) 4140 rev
(c) higher deflecting torque.
37. A 0−15 mA PMMC meter reads 6 mA in a circuit.
(d) improper design of disc.
Its only bottom control spring snaps suddenly. The
30. Two thermocouples are used in a true rms reading meter will now read nearly
voltmeter to
(a) 15 mA (b) 0
(a) avoid the failure of one of the thermocouples. (c) 3 mA (d) 6 mA
(b) reduce zero error.
38. A PMMC voltmeter has a voltage range of 0−300 V
(c) remove the non-linearity introduced by the first
with an internal resistance 2 kΩ. There is a mul-
thermocouple.
tiplier resistance of 298 kΩ attached with the
(d) have more deflecting torque.
meter. Find the sensitivity of the moving coil
31. A dynamometer type wattmeter can read high or instrument.
lagging pf and low or leading pf due to
(a) 2 kΩ/V (b) 3.5 kΩ/V
(a) high resistance of its pressure coil. (c) 1 kΩ/V (d) 1.5 kΩ/V
(b) high resistance of its current coil.
39. A thermo-electric ammeter gives a full scale deflec-
(c) high inductance of its pressure coil.
tion for a current of 50 A. Calculate the current
(d) high capacitance of its pressure coil.
which causes half scale deflection.
32. A meter with spring control follows square law.
(a) 5 A (b) 25 A
For a full scale deflection of 90°, it takes a cur-
(c) 38.47 A (d) 35.36 A
rent of 1 A. For a current of 0.707 A, its deflec-
tion will be 40. An energy meter is intended to make 100 revo-
lutions of disc for one unit of energy measured.
(a) 30° (b) 60°
Calculate the number of revolutions made by the
(c) 45° (d) 50°
disc when connected to load carrying current of
33. Time response of an indicating instrument is given by 40 A at 230 V and 0.4 power factor for an hour.
(a) controlling mechanism. (a) 428 (b) 368
(b) bearing mechanism. (c) 272 (d) 300
(c) deflecting mechanim.
41. Two milli-ammeters with a full scale current of
(d) damping mechanism.
2 mA and 20 mA are connected in parallel. They
34. For the given circuit, the reading of the moving- read 1 mA and 5 mA, respectively. Their internal
iron type instrument will be resistances are in the ratio

10 A (a) 5:1   (b) 10:1   (c) 1:10   (d) 1:5


42. A moving coil instrument gives a full scale deflec-
V V
tion of 10 mA when voltage across its terminal is
PMMC Moving
AC source iron 100 mV. For a 100 A current, calculate the shunt
resistance required for a full scale deflection.
(a) 0.01 Ω (b) 0.001 Ω
(a) 10 A (b) 0 A (c) 0.1 Ω (d) 1 Ω
(c) 15.7 A (d) 20 A 43. Calculate the power dissipation in the above instru-
35. The zero of a voltmeter is having an error of 5% of ment in Question 42.
its full scale. If the voltmeter reads 30 V, the pos-
(a) 100 W (b) 1000 W
sible range of actual voltage is
(c) 10 kW (d) 10 W
(a) 28 − 32 V (b) 28.5 − 31.5 V
(c) 20 − 40 V (d) 29 − 31 V
44. The inductance of a 25 A electrodynamic ammeter
changes uniformly at the rate of 0.0035 µH/degree.
36. A single phase energy meter is working at 230 V, The spring constant is 10−6 N-m/degree. Determine
50 Hz and a 10 A load current at unity power factor. the angular deflection at full scale.

Chapter 7_Solved Examples and Practice exercise.indd 660 3/23/2016 1:31:35 PM


PRACTICE EXERCISES     661

(a) 115° (b) 118.7° (a) 0.5 H (b) 0.6 H


(c) 125° (d) 120° (c) 1.0 H (d) 1.5 H

45. Which of the following formula is used to express 3. A shunt resistance of 25 Ω is necessary to extend the
active power in a balanced three-phase circuit? range of an ammeter from 100 mA to 500 mA. The
value of internal resistance of this ammeter is
(a) VL I L cos f (b) 3VL I L cos f
(a) 25 Ω (b) 50 Ω
(c) Vph I ph cos f (d) 3Vph I ph cos f (c) 100 Ω (d) 1000 Ω
46. Which one of the following is not a torque pro- 4. A single-phase watt hour meter is connected with
duced in an integrating type of instrument? load at 240 V taking a current of 1 A at 0.6 pf lag.
(a) Operating torque (b) Braking torque If the meter runs at a speed of 2.16 rpm, the meter
(c) Damping torque (d) None of the above constant will be
(a) 400 rev/kWh (b) 600 rev/kWh
47. The resistances of the current coil and the poten- (c) 900 rev/kWh (d) 1000 rev/kWh
tial coil in an electrodynamometer type wattmeter
are respectively, 5. The input power of a three-phase induction meter
is to be measured by two wattmeter method. If the
(a) high and low (b) low and high machine absorbs 300 W at a line voltage of 415 V
(c) High and high (d) low and low and meter line current of 1.4 A, the possible reading
48. In a PMMC-type instrument, the magnetic field of the two wattmeter are
produced by the eddy currents (a) 1104 W, 804 W (b) 627 W, −327 W
(a) does not affect the motion of the coil. (c) 425 W, 125 W (d) 425 W, −125 W
(b) acts in the same direction of the motion of the 6. For the circuit given below, the reading of the
coil. voltmeter will be
(c) acts in the direction opposite to the motion of
the coil. V
(d) none of the above.
400 V RV = 50 kΩ
100 kΩ
Set 2 (Two Marks Questions) AC supply
50 Hz
1. The voltage and current waveforms for an element (a) 133.3 V (b) 0 V
are show in the figures. (c) 400 V (d) 200 V
i(t) v(t) 7. If the voltmeter shown in the following circuit reads
20 V, the value of the unknown resistance Rx will be

V
2A 2V
50 kΩ
100 V Rx

2s Time 2s Time
(a) 0 V (b) 200 kΩ
The circuit element and its value are (c) 50 kΩ (d) 100 kΩ
(a) capacitor and 2 F (b) inductor and 1 H
(d) resister and 1 Ω
8. A strain gauge with gauge factor 2 has a nominal
resistance of 100 Ω. For a strain of 10−5 the change
(c) inductor and 2 H
2. For the AC current given below, if the rms voltages of its resistance is
(a) 1 Ω
across the resistor is 120 V. What is the value of
(b) 1 mΩ
the inductor?
(c) 2 mΩ (d) 5 mΩ
9. The resistance of a thermostat is 5 Ω at 30° and
+ 1 kΩ 100 Ω at 60°, using linear approximation, its resis-
V = 150√2 sin 500tV tance temperature coefficient is
− L (a) 3.17 Ω/°C (b) 3.17 kΩ/°C
(c) 10 Ω/°C (d) 10 mΩ/°C

Chapter 7_Solved Examples and Practice exercise.indd 661 3/23/2016 1:31:51 PM


662     CHAPTER 7:  ELECTRICAL AND ELECTRONIC MEASUREMENTS

10. For the given connection, the phase sequence is


ABC and the system is balanced power factor of 200 Ω 100 Ω
this load can be
+
i W1 = 100 W V
A

B
i To load 100 Ω 200 Ω

i W2 = 0
C
12 V
(a) unity (b) zero
(c) 0.5 lead (d) 0.5 lag (a) zero (b) −4 V
(c) 12 V (d) 6 V
11. A moving-coil ammeter is used to measure maxi-
mum value of current of 100 mA at its full scale. 15. If for the voltmeter shown in Question 14, the
If a resistance of 0.5 Ω is connected in parallel resistance is 100 Ω, the reading of the voltmeter is
with it, the new value of current at full scale (a) −1.71 V (b) 0 V
will be (c) 12 V (d) 6.8 V
(a) 100 mA (b) 300 mA 16. A three-phase, 500 V load has a power factor
(c) 500 mA (d) 1 A of 0.4. Two wattmeters are connected to mea-
12. A moving-coil ammeter with resistance of 0.5 W sure the input power. The input power is mea-
gives full scale deflection at 200 mA. To use this sured to be 30 kW. Find the reading in each
meter to measure a voltage of 200 V at full scale instrument.
(a) a resistance of 999.5 W is to connected in series (a) 26 kW, 4 kW (b) 32 kW, −2 kW
with the meter. (c) 31.62 kW, −1.62 kW (d) 34.85 kW, −4.85 kW
(b) a resistance of 0.5 W is to connected in series
17. In a two wattmeter method of measuring three-phase
with the meter.
power, power factor is 0.5, one wattmeter reads W
(c) a resistance of 999.5 W is to connected in paral-
then the other meter will read
lel with the meter.
(d) a resistance of 0.5 W is to connected in parallel W
with the meter. (a) 3 W    (b)    (c) Zero   (d) 2 W
2

18. An ammeter having a resistance of 1 W and a full


13. The expressions v(t) and i(t) of the given circuit
scale reading of 5 A is to be used for measuring a
v(t) = 100 sin 314t + 50 sin 3 × 314t + 10 sin 5 × 31 V current a current up to 50 A. The value of shunt
resistance to be connected will be
i(t) = 5 sin(314t − 300 ) + 1 sin(3 × 314t)
(a) 20 mΩ (b) 1 Ω
The reading of the wattmeter will be (c) 50 mΩ (d) 100 Ω

i(t) 19. A 50 mA meter with an internal resistance of 1 kΩ


is to be used as a DC voltmeter of range 50 V.
Then the voltage multiplying factor m is
v(t) Load (a) 100 (b) 10
(c) 1000 (d) 10,000
20. A 1 mA, 50 Ω galvanometer is required to measure
(a) 229 W (b) 550 W 5 A full scale reading. Determine the value of resis-
(c) 350 W (d) 0 W tance to be added across (shunt) the galvanometer
to accomplish this measurement.
14. Assuming that the voltmeter given in the circuit
below has very large resistance, the reading of the (a) 10 Ω (b) 0.01 Ω
voltmeter will be (c) 1.0 Ω (d) 0.001 Ω

Chapter 7_Solved Examples and Practice exercise.indd 662 3/23/2016 1:32:08 PM


ANSWERS TO PRACTICE EXERCISES     663

21. For the three-phase balanced circuit with voltage 23. Determine the ratios of potential transformer and
supplied 415 V, current 10 A and power factor lag current transformer respectively, required to mea-
0.8, as show in the circuit, the reading of the watt- sure power in a circuit rated at 5500 kW, 11 kV
meter will be with a wattmeter rated at 5 A and 110 V.
± CC I = 10 A
24. Two voltmeters with same range 0−500 V having
R
internal resistances 10,000 Ω and 20,000 Ω are
±
415 V Y IM connected in series. A voltage of 600 V is applied
PC across them. The readings in the respective meters
B will be
(a) 2490 (b) 0
(c) 1000 W (d) Wattmeter will burn (a) 240 V and 360 V (b) 300 V each
(c) 200 V and 400 V (d) one will be out of range
22. A moving coil instrument has a resistance of 1 Ω and
it reads up to 250 V when a resistance of 10,000 Ω
25. A moving coil instrument has coil of 200 turns. The
is connected in series with it. Find the current
width and depth of coil are 20 mm and 30  mm,
range of the instrument when it is used as ammeter
respectively. Determine the deflecting torque in
with the coil connected across a shunt resistance of
1 × 10−3 Ω.
the instrument when the air gap flux density is
0.12 Wb/m2 and the current measured is 1 A. What
(a) 24.99 A (b) 24.965 A is the angle of deflection, if the spring constant is
(c) 25.02 A (d) None of the above 4.8 × 10−4 nm/ degree?

ANSWERS TO PRACTICE EXERCISES

Set 1 (One Mark Questions)


0.1 × 10−3 =
10
1. (b) R s + 500
⇒ R s = 99.5 kΩ
2. (d)
3. (b) 14. (c) Error is 30 ×(±2%) = ± 0.6 V
4. (c) So, voltage range is 29.4 to 30.6 V
5. (a) 15. (a) Td = NBAI = 5000 × 3000 × 70 × 1
6. (a) = 1.05 × 105 Nm
7. (c) 16. (b) Lissajous pattern is a straight line when the
8. (b) phase difference is 0 or 180° and an ellipse for 45°.

9. (b) Bcecause, centre zero PMMC shows DC value 17. (d) Because current at radio frequency causes
whereas moving iron instrument shows rms values. heating which causes thermo-electric emf propor-
tional to heating. This drives the micro-amme-
10. (c) ter connected to it to record the proportional
11. (c) Given that LVDT output = 2 mV current.
Amplifier output 18. (b) If Q is sufficiently large, the term 1/Q2 which
= 2 × 10−3 × 500 = 100 0 mV = 1 V appears in the expression of unknown inductance
Sensitivity = 1 V/mm L1 can be neglected.
12. (b) 19. (c) Meter resistance = 2000 × 120 = 240 kW
13. (b) Consider the circuit for the ammeter
20. (c) It is a common term used for energy meter test-
0.5 mA ing. When the current rating of the energy meter
A used is high, the current coil is supplied from a
R Rm
500 Ω low voltage supply. Circulation of rated current
through the current coil is thus possible as the
10 V impedance of this circuit is low.

Chapter 7_Solved Examples and Practice exercise.indd 663 3/23/2016 1:32:17 PM


664     CHAPTER 7:  ELECTRICAL AND ELECTRONIC MEASUREMENTS

21. (c) Thus, number of revolutions


2300
fh
=
Number of vertical tangencies (crossings)   = × 600 × 3 = 4140 rev
fv Number of horiizontaltangencies (crossings) 1000

120 3 120 × 5 37. (b) If bottom spring snaps, meter reads 0 as no


= ⇒ fv = = 200 Hz deflection will occur.
fv 5 3
38. (c)
22. (d) It can measure signals with wide range of
frequencies. (Meter resistance + series resistance)
Sensitivity =
Full scale deflection
23. (a) Large voltage values typically kilo-volts range
(2 + 298)
are measured by electrostatic voltmeters. = = 1 kW/V
300
24. (a)
39. (d) For the moving coil instrument
fx Number of vertical tangencies (crossings)
=
fy Number of horiizontal tangencies (crossings) Half scale I2
=
Full scale (50)2
100 1
= ⇒ fy = 200 Hz 0.5 I2
fy 2 = ⇒ I = 35.36 A
1 2500
25. (c)
40. (b) Load = 230 × 40 × 0.4 = 3680 W = 3.68 kW
26. (d)
Thus, the number of revolutions = 3.68 kWh ×
27. (c) 100 = 368
28. (c) 41. (b) Voltage in two branches will be same, so
29. (b) R1 10
I1XR1 = I2 XR2 ⇒ 1XR1 = 5XR2 ⇒ =
30. (c) R2 1

=10 Ω.
31. (c) 100
42. (b) Rm =
10
q1 i2 Thus, for shunt resistance R,
32. (c) = 12
q2 i2 100R/(10 + R) = 100 × 10−3 ⇒ R = 0.001 Ω
q 2 = q1 × (i2 ) = 90° × (0.707 ) = 45° 43. (d) Power dissipation = 99.92 × 0.001 + 0.12 × 10
2 2

= 10 W
33. (d)
44. (c) Deflection = (i) ×
dL 180
×
2
34. (c) For the circuit dq p /k
Im
= 10 ⇒ I m = 10p = (25 × 25) × 0.0035 × 10−6 ×
180
= 125°
p p
45. (b)
Reading of MI instrument will be
46. (c)
Im
= 15.7 A 47. (b)
2
35. (d) For full scale reading, error is 5%, so change in 48. (c)
reading will be
30 × 0.05 = 1.5 V Set 2 (Two Marks Questions)
So range of actual values is 1. (c) From the given graphs, we have
30 ± 1.5 ⇒ 28.5 − 31.5
di
36. (d) Load = 230 × 10 = 2300 W
v=L
dt

Chapter 7_Solved Examples and Practice exercise.indd 664 3/23/2016 1:32:39 PM


ANSWERS TO PRACTICE EXERCISES     665

8. (c) In a strain gauge



1
i= vdt
∆R
2t = Strain × gauge factor = 2 × 10−5
= R
L
∆R = 2 mΩ
Then, at t = 2 s
9. (a) From the graph below, we have
i=2A
2 R
Then 2 = 2.
L 100 Ω
or, L=2H
5Ω
2. (d) From the given circuit
°C
120
i= = 120 mA 30° 60°
1 × 103
R −5 q − 30
=
−3 150 5 − 100 30 − 60
120 × 10 =
(1 × 103 )2 + wL2
(q − 30°) + 5
95
R=
30
= 3.17q − 90°
L = 1.5 H
3. (c) Consider the circuit for the ammeter
Therefore, temperature coefficient = 3.17 Ω/°C
500 µA Rm 100 µA
A 10. (d) The phasor diagram for the given circuit is

VCB

25 Ω
VC i VAB
100 × 10 −6
=
25
× 500 × 10−6 f 30°
25 + R m
30°
R m = 100 Ω
f
VA

4. (c) Load = 240 × 1 × 0.6 = 144 W = 0.144 kW i


i
Meter speed = 2.16 rpm, therefore
VB
2.16 × 60
Meter constant = = 900 rev/kWh
0.144 W1 = VAB i cos(30° + f )
5. (d) Power factor is given by W2 = VAB i cos(30° − f )

pf = cos f =
300
= 0.298 Given that W2 = 0. Therefore,
3 × 415 × 1.4
⇒ f = 72.65° p
cos(30° − f ) = 0 = cos
2
Then readings of the two wattmters are
W1 = 415 × 1.4 × cos(30° + 72.65°) = −128 W f = −60 ⇒ pf = cos f = 0.5 lag
W2 = 415 × 1.4 × cos(30° − 72.65°) = 427 W
11. (b) Consider the following circuit
6. (a) Voltmeter reading =
50
× 400 = 133.32 V 0.5 Ω
150
7. (b)
50 × 103 I
.100 = 20 A
50 × 103 + R x 1 Ω 100 mA
⇒ R x = 200 kW PMMC

Chapter 7_Solved Examples and Practice exercise.indd 665 3/23/2016 1:33:44 PM


666     CHAPTER 7:  ELECTRICAL AND ELECTRONIC MEASUREMENTS

Now, the current through the ammeter is given by 16. (d) Power factor angle f = cos−1(0.4) = 66.42°
0.5 Thus,
100 mA = I × (W1 − W2 )
1 + 0. 5 tan (66.42°) = 3 and W1 + W2 = 30 kW
(W1 + W2 )
1.5
Thus, I = × 100 mA = 300 mA Hence,
0.5
W1 = 34.85 kW and W2 = −4.85 kW
12. (a) Consider the following circuit
17. (c) Given that W1 = W
R
A (W − W2 )
and tan (f) = 3 1
0.5 Ω W1 + W2
PMMC
pf = cos(f) = 0.5 or f = 60°
200 V Thus, W2 = 0
The maximum current is given by 18. (a) Consider the following circuit
Rsh
= 200 × 10−3
200 49 A
I=
R + 0. 5
R + 0.5 = 1000 or R = 999.5 Ω
13. (a) The reading of the wattmeter will be A
50 A 1Ω 1A

cos 30 + cos 60 = 229 W


100 5 50 10
W = × × Current through ammeter I = 50 ×
1
2 2 2 2 1 + Rsh
So,
14. (b) The reading of the voltmeter will be Rsh
I = 50 ×
12 12 1 + Rsh
× 100 − × 200 = −4 V
300 300
or 49Rsh = 1 ⇒ Rsh = W ≈ 20 mW
1
15. (a) Consider the circuit given below. 49
A 19. (a) Consider the circuit for the meter
V1 1 kΩ 50 µA
A
200 Ω 100 Ω
100 Ω
R
50 V
V
I = 50 µA

100 Ω 200 Ω Vm = 50 × 10−6 A × 1 × 103 W = 50 mV


50 V
Voltage multiplying factor = = 100
B V 50 mV
2
20. (b) Consider the circuit for the galvanometer
12 V
50 Ω 1 mA
Then at node A
5A
V1 − 12 V V − V2
+ 1 + 1 =0 (i)
200 100 100
node B
At Rsh
V2 − V1 V2 − 12 V Then
+ + 2 =0 (ii)
1 × 10−3 =
Rsh
×5
100 100 200
Solving (i) and (ii), we get Rsh + 50
⇒ Rsh = 0.01 Ω
V1 − V2 = −1.72 V

Chapter 7_Solved Examples and Practice exercise.indd 666 3/23/2016 1:34:40 PM


ANSWERS TO PRACTICE EXERCISES     667

21. (a) Given that power factor Primary current,


5500 × 103
cosf = 0.8 ⇒ f = cos−1 0.8 = 36.87°
P
I1 = = = 500 A
V1 11000
Then wattmeter reading is
Secondary voltage, V2 = 110 V
W = 415 × 10 × sin 36.87 = 2490 W
So, PT ratio =
22. (c) Given that resistance of the instrument coil, V1 11000 100
Rm = 1 Ω V2
=
110
=
1

   
Current flowing through the instrument for full-scale I1 500 100
deflection and, CT ratio    = = =
I2 5 1
Full scale reading Ans. (100, 100)
Im =
R m + Series resistance 24. (c) Total current in circuit =
250 600
= = 0.02499775 A = 24.99 mA = 0.02 A
1 + 10, 000 (10000 + 20000)
Therefore, for voltmeter 1,
Shunt resistance, Rsh = 1 × 10−3 W voltage reading = 0.02 × 10000 = 200 V
and for voltmeter 2,
Current through shunt resistance, voltage reading = 0.02 × 20000 = 400 V

I m Rm 24.99 × 10−3 × 1 25. Deflecting torque Tc = NBAI nm


I sh = = = 24.99 A = (20 × 10−3) (0.12 × 1 × 30 × 10−3 × 200) =
R sh 1 × 10−3 0.01440 nm
Current range of instrument = Full-scale deflection At the point of operation, Td = Tc. Therefore,
current Tc
Angle of deflection =
= I m + I = 0.0249975 + 24.99 = 25.02 A Spring constant
0.01440
23. Given that: = = 30°
Power rating, P = 5500 kW 0.00048
Voltage across primary, V1 = 11 kV
Ans. (30)

Chapter 7_Solved Examples and Practice exercise.indd 667 3/23/2016 1:35:07 PM


668        CHAPTER 7:  ELECTRICAL AND ELECTRONIC MEASUREMENTS

SOLVED GATE PREVIOUS YEARS’ QUESTIONS

1. A Manganin swamp resistance is connected in For C ′= 200 pF, the Q-meter reading will be given
series with a moving coil ammeter consisting of a by
milli-ammeter and a suitable shunt in order to
1
Q=
(a) minimise the effect of temperature variation. wC ′R
(b) obtain large deflecting torque.
When a lossless capacitor Cx is connected in paral-
lel with C ′ = 200 pF,
(c) reduce the size of the meter.
(d) minimise the effect of stray magnetic fields.
1
Q=
w (C¢ + Cx )R
(GATE 2003: 1 Mark)

Solution:  In a permanent magnet moving coil C = C ′ + Cx


(PMMC) instrument as temperature increases the
coil resistance increases. Compensation is done by 300 = 200 + Cx
adding a swamp resistor in series with the moving Cx = 100 pF
Ans. (a)
coil. Swamp resistor is made of Manganin with
­zero-temperature coefficient. Thus the effect is to 4. The items in Group I represent the various types of
minimise effect of temperature variation. measurements to be made with a reasonable accu-
Ans. (a) racy using a suitable bridge. The items in Group II
represent the various bridges available for this pur-
2. The effect of stray magnetic fields on the actuating
pose. Select the correct choice of the item in Group
torque of a portable instrument is the maximum
II for the corresponding item in Group I from the
when the operating field of the instrument and the
following.
stray fields are
(a) perpendicular (b) parallel Group I Group II
(c) inclined at 60° (d) inclined at 30° P. Resistance in the 1. Wheatstone bridge
(GATE 2003: 1 Mark) milli-ohm range 2. Kelvin double bridge
Q. Low values of 3. Schering bridge
Solution:  The effect of stray magnetic field is the capacitance
4. Wien’s bridge
maximum when the operating field and stray fields R. Comparison of
are parallel. resistances which 5. Hay’s bridge
Ans. (b) are nearly equal 6. Carey-Foster bridge
3. A reading of 120 is obtained when a standard S. Inductance of a
inductor was connected in the circuit of a Q-meter coil with a large
and the variable capacitor is adjusted to a value time constant
of 300 pF. A lossless capacitor of unknown value
Cx is then connected in parallel with the variable (a) P — 2 Q — 3 R — 6 S — 5
capacitor and the same reading was obtained when (b) P — 2 Q — 6 R — 4 S — 5
the variable capacitor is readjusted to a value of (c) P — 2 Q — 3 R — 5 S — 4
200 pF. The value of Cx in pF is (d) P — 1 Q — 3 R — 2 S — 6
(a) 100 (b) 200
(GATE 2003: 2 Marks)
(c) 300 (d) 500

(GATE 2003: 1 Mark) Solution:  The bridges matched with the purpose
are as follows:
Solution:  Given that when C = 300 pF, Q-meter Kelvin bridge for low value of resistance (P → 2)
reading = 120. We know that Schering bridge for low values of capacitance
(Q → 3)
1 Carey-Foster bridge for comparison of resistances
Q=
wCR which are nearly equal (R → 6)

Chapter 7 Solved Question Paper_1 .indd 668 3/23/2016 1:32:57 PM


SOLVED GATE PREVIOUS YEARS’ QUESTIONS        669

Hay’s bridge for inductance of coil with large Solution:  Case (i): When the current coil is
time constant (S → 5) ­connected in the R phase and pressure coil is con-
Ans. (a) nected between R phase and neutral of three-phase
5. A rectifier type AC voltmeter consists of a series system supplying 0.8 pF inductive load.
resistance Rs, an ideal full-wave rectifier bridge and R W
a PMMC instrument as shown in the given figure.
The internal resistance of the instrument is 100 Ω
and a full scale deflection is produced by a DC cur-
rent of 1 mA. The value of Rs required to obtain
full scale deflection with an AC voltage of 100 V
(rms) applied to the input terminals is Y

B
Rs
100 V The power wattmeter reading is given by
PMMC
AC
input
millimeter P1 = IPVP cos q1
VL
400 = I L ⋅ ⋅ cos q1
3
(a) 63.56 Ω (b) 89.93 Ω IL VL
(c) 89.93 kΩ (d) 141.3 kΩ 400 = × 0.8 [∵ cos q1 = 0.8]
3
(GATE 2003: 2 Marks)
400 × 3
VL IL =
0.8
Solution:  Given that for full scale deflection
IDC = I mA. Case (ii): When phase sequence is RYB and pres-
sure coil is connected between B and Y phases.
For a full wave rectifier
2I R W
IDC = m
p
2I m
1 mA = ⇒ I m = 1.57 mA
p
Im 1.57
I rms = = = 1.11 mA Y
2 2
V = (Rs + Rm )(I rms ) B
−3
100 = (Rs + 100) (1.11×10 ) q2 = (30° + 60° − q1 )
Rs = 89.8 kΩ [∵ V = 100 and Rm = 100 ] = 90° − 36.86 (∵ q1 = cos−1 0.8 = 36.86)
Ans. (c) = 53.14
6. A wattmeter reads 400 W when its current coil is
The power wattmeter reading is given by
connected in the R phase and its pressure coil is
connected between this phase and the neutral of a
P2 = V YB I L cos q2
symmetrical three-phase system supplying a bal-
anced star connected 0.8 pF inductive load. The 400 3
= × cos 53.14
phase sequence is RYB. What will be the reading 0. 8
of this wattmeter if its pressure coil alone is recon- = 519 W
nected between the B and Y phases, all other con- Ans. (b)
nections remaining as before? 7. The inductance of a certain moving-iron ammeter
(a) 400.0 (b) 519.6 is expressed as
q2
(c) 300.0 (d) 692.8 L =10 + 3q − µH ,
4
(GATE 2003: 2 Marks)

Chapter 7 Solved Question Paper_1 .indd 669 3/23/2016 1:33:29 PM


670        CHAPTER 7:  ELECTRICAL AND ELECTRONIC MEASUREMENTS

where q is deflection in radians from the zero ­position. Percentage error is


The control spring torque is 25 × l0−6 N-m/radian. n − n′ 100 − 111.8
∆n = ×100 = ×100 = − 10.55%
The deflection of the pointer in radian when the n′ 111.8
meter carries a current of 5 A, is Ans. (b)
(a) 2.4 (b) 2.0
(c) 1.2 (d) 1.0 9. The voltage-flux adjustment of a certain single-
phase 220 V induction watt hour meter is altered
(GATE 2003: 2 Marks) so that the phase angle between the applied volt-
age and the flux due to it is 85° (instead of 90°).
Solution:  Given that The errors introduced in the reading of this meter
q2
when the current is 5 A at power factors of unity
L = 10 + 3q − µH and kc = 25 × 10−6 and 0.5 lagging are, respectively,
4
Then (a) 3.8 mW, 77.4 mW
dL  q  (b) −3.8 mW, −77.4 mW
= 3 −  (c) −4.2 mW, −85.1 mW
dq  2
(d) 4.2 mW, 85.1 mW
We know that the control torque for moving iron
ammeter is given by (GATE 2003: 2 Marks)
Solution:  Let θ be the phase angle between supply
Tc = kq = I 2 ⋅
1 dL
2 dq voltage and pressure coil flux and φ be the load
phase angle. The power wattmeter reading is
 
(25×10−6 )q = 12 52 3 − q2 ×10−6 Pm = VI sin (q − f )
Pm = 200 ×5 sin(85°− 60°) [ Q cos f = 0.5 ⇒ f = 60°]
q
2q = 3 − ⇒ q = = 1.2 rad
6
= 110 sin25°
2 5
Ans. (c) = 464.88 W

8. A 500 A/5 A, 50 Hz current transformer has a bar Actual power is given by


primary. The secondary burden is a pure resistance Po = VI cos f = 220 ×5× 0.5 = 550 W
of 1 Ω and it draws a current of 5 A. If the mag-
netic core requires 250 AT for magnetisation, the Therefore, error is given by
percentage ratio error is
Pm − Po = 464.88 − 550 =− 85.12 W
(a) 10.56 (b) −10.56
(c) 11.80 (d) −11.80
For unity power factor, cos φ = 1⇒ φ = 0. Therefore,
(GATE 2003: 2 Marks) Pm = 220 ×5 sin(85°− 0) = 1095.8 W
Po = 220 ×5 cos0° = 1100 W
Solution:  Given that: Error = Pm − Po = 1095.8 − 1100 =− 4.19 W
250
Magnetising current I m = = 250 A Ans. (c)
1
Primary current IP = 500 A 10. Group II represents the figures obtained on a CRO
Secondary current IS = 5 A screen when the voltage signals V x = V xm sin w t and V y = V ym si
I V x = V xm sin w t and V y = V ym sin(w t + F) are given to its X and Y
Turns ratio n = P = 100 plates respectively and Φ is changed. Choose the
IS
correct value of Φ from Group I to match with the
Therefore, total primary current,
corresponding figure of Group II.
I total = IP2 + I m2 Group I
P. F = 0
I total = 5002 + 2502 = 559 A p
Q. Φ =
2
Therefore, the new turns ratio is 3p
R. p < Φ <
I
n ′ = total =
559 3p 2
IS 5
= 111.8 A S. Φ =
2

Chapter 7 Solved Question Paper_1 .indd 670 3/23/2016 1:34:11 PM


SOLVED GATE PREVIOUS YEARS’ QUESTIONS        671

Group II 11. A DC potentiometer is designed to measure up to


about 2 V with a slide wire of 800 mm. A standard
cell of emf 1.18 V obtains balance at 600 mm. A
test cell is seen to obtain balance at 680 mm. The
emf of the test-cell is
(a) 1.00 V (b) 1.34 V
1 2 3 (c) 1.50 V (d) 1.70 V
(GATE 2004: 1 Mark)

Solution:  For DC potentiometer, standard cell emf


is proportional to length of wire, E ∝ l. Therefore
E1 l1
=
4 5 6 E2 l2
(a) P — 1  Q — 3 R — 6 S — 5 Therefore, emf of test cell is
(b) P — 2  Q — 6 R — 4 S — 5 l   680 
E2 = E1  1  = (1.18)  = 1.34 V
 600 
(c) P — 2  Q — 3 R — 5 S — 4
(d) P — 1  Q — 5 R — 6 S — 5  l 
2
Ans. (b)
(GATE 2003: 2 Marks)
12. The circuit in the given figure is used to measure
Solution: When Φ = 0° Vy = sinwt the power consumed by the load. The current coil
and the voltage coil of the wattmeter have 0.02 Ω
We obtain the Lissajous pattern in x —y mode as and 1000 Ω resistances, respectively. The measured
follows: power compared to the load power will be
2 2
0.02 Ω 20 A
3 5 1, 3, 5
1
1000 Ω upf
200 V load
4 4
Vy = Vy msin wt

1
2 (a) 0.4% less (b) 0.2% less
3 (c) 0.2% more (d) 0.4% more
Vx = Vxm sin w t 4
5
(GATE 2004: 1 Mark)
When Φ = 90°; Vy = Vym sin(wt + 90°)
Solution:  From the circuit, the current and volt-
The Lissajous pattern in x — y mode is as follows age can be determined as follows
1 5 1  1000 
20 = I   ⇒ I = 20 A
1000 + 0.02 
6 4
  200 = V − (0.02×200) ⇒ V = 200.4 V
2 4 3
Measured power Pm = VI = 20(200.4) = 4008 W.
3 7 2
Load power PL = 20 × 200 = 4000 W
Pm − PL 4008 − 4000
Percent change in power = = ×100
1 PL 4000
2 P − PL 4008 − 4000
×100
3
= m =
4 PL
4000
5
3p = 8 ×100 = 0.2%
Similarly, we can solve for Φ = 4000
2 Ans. (a) Ans. (c)

Chapter 7 Solved Question Paper_1 .indd 671 3/23/2016 1:34:35 PM


672        CHAPTER 7:  ELECTRICAL AND ELECTRONIC MEASUREMENTS

13. A galvanometer with a full scale current of 10 mA On solving for VP, we get
has a resistance of 1000 Ω. The multiplying power 10 = (2.2 − j0.628) VP
(the ratio of measured current to galvanometer
current) of a 100 Ω shunt with this galvanometer is
10
VP = = 4.38 V
2.28 Ans. (b)
(a) 110 (b) 100
(c) 11 (d) 10 15. A moving coil of a meter has 100 turns, and a
(GATE 2004: 1 Mark) length and depth of 10 mm and 20 mm, respec-
tively. It is positioned in a uniform radial flux den-
Solution:  From the given circuit for the galva- sity of 200 mT. The coil carries a current of 50 mA.
nometer, the multiplying power or turn ratio is The torque on the coil is

given by n =
I (a) 200 µNm (b) 100 µNm
I1 (c) 2 µNm (d) 1 µNm
Rm = 1000 Ω (GATE 2004: 2 Marks)
I I1
Solution:  Given that number. of turns (N ) = 100;
current (I ) = 50 mA; magnetic field (B) = 200 mT
and area (A) = 10 mm × 20 mm.
We know that the deflecting torque is given by
I2 Rsh = 100 Ω
T = NIBA
I1 Rsh 100 1
= = = = 100 × 50 × 10−3 × 200 × 10−3 × 200 × 10−6
I2 Rm 1000 10
Applying KCL, we have = 200 × 10−6 Nm
I = I1 + I2 = I1 + 10I1 = 11I1 Ans. (a)
Therefore
16. A DC ampere hour meter is rated for 15 A, 250 V.
11I1
n= = 11 The meter constant is 14.4 A seconds/rev. The
I1 meter constant at rated voltage may be expressed as
Ans. (c)
14. A CRO probe has an impedance of 500 kΩ in paral- (a) 3750 rev/kWh (b) 3600 rev/kWh
lel with a capacitance of 10 pF. The probe is used to (c) 1000 rev/kWh (d) 960 rev/kWh
measure the voltage between P and Q as shown in (GATE 2004: 2 Marks)
the following figure. The measured voltage will be
Solution:  Given that meter constant = 14.4 A
100 kΩ P
seconds/rev; I = 15 A and V = 250 V.

To CRO I
100 V rms We know that meter constant =
100 kΩ through Speed
100 kHz
probe If K is meter constant in rev/kWh, then
I
Q 14.4 =
K × power
(a) 3.53 V (b) 4.37 V
I
(c) 4.54 V (d) 5.00 V 14.4 = [∵ P = VI ]
  KVI
(GATE 2004: 2 Marks) 1
K= = 3600
14.4×250
Solution:  For the given circuit, reactance,
1 1 To obtain kWh, multiply value of K by 1000 and
XC = = to obtain value per revolution divide by 60 × 60.
jw C 2p ×100 ×103 ×10 ×10−12
Hence
Applying nodal analysis at point P, we have 1000
K = 3600 × = 1000 rev/kWh
VP − 10  1 j  60 ×60
+ VP 
1
− =0
100 500 159 

+
100 Ans. (c)

Chapter 7 Solved Question Paper_1 .indd 672 3/23/2016 1:35:07 PM


SOLVED GATE PREVIOUS YEARS’ QUESTIONS        673

17. A moving-iron ammeter produces a full scale torque 19. A 50 Hz, bar primary CT has a secondary with
of 240 µNm with a deflection of 120° at a current 500 turns. The secondary supplies 5 A current into
of 10 A. The rate of change of self-inductance (µH/ a purely resistive burden of 1 Ω. The magnetising
radian) of the instrument at full scale is ampere-turns is 200. The phase angle between the
(a) 2.0 µH/radian (b) 4.8 µH/radian
primary and secondary current is
(c) 12.0 µH/radian (d) 114.6 µH/radian (a) 4.6° (b) 85.4°
(c) 94.6° (d) 175.4°
(GATE 2004: 2 Marks)
Solution:  Moving-iron ammeter full torque is (GATE 2004: 2 Marks)
given as, Solution:  Given that number of turns n = 500;
1 dL primary current IP = 200 × 1 = 200 A; secondary
Tc = I 2
2 dq current IS = 5 A. Let f = phase angle (in rad).
dL 240 ×10−6 ×2
= 48×10−6 H/radian
I
= nf = P
dq 102 IS
Ans. (b)
= 500f
200
18. A single-phase load is connected between R and 5
Y terminals of a 415 V, symmetrical, three-phase,
Therefore, f =
200 180
four wire system with phase sequence RYB. A × (rad) = 4.58°
wattmeter is connected in the system as shown in 5×500 p
the following figure. The power factor of the load Ans. (a)
is 0.8 lagging. The wattmeter will read 20. The core flux in the CT of Question 19 under the
given operating condition is
W
R (a) 0 (b) 45.0 µWb
(c) 22.5 mWb (d) 100.0 mWb
(GATE 2004: 2 Marks)
Solution:  Secondary winding voltage
100 Ω
Z VS = I S × Z L
0.8 pf lag
= 5×1 = 5 V
Y Induced voltage
B VS = 2 p f Nf
5 = 2 ×3.14×50 ×500 ×f
= 45×10−6 Wb ⇒ 45 µWb
N
Ans. (b)
(a) —795 W (b) —597 W
(c) +597 W (d) +795 W 21. The Q-meter works on the principle of
(a) mutual inductance
(GATE 2004: 2 Marks)
(b) self inductance
415
Solution:  Given that VRY = 415∠30° and VBN = ∠120° (c) series resonance
415 3 (d) parallel resonance
VBN = ∠120° (GATE 2005: 1 Mark)
3
Current coil will carry a current of Ans. (c)
VRY 415∠30° 22. A PMMC voltmeter is connected across a series
Ic = = [pf = cos q = 0.8 = f = 36.87°]
Z 100∠36.87° combination of a DC voltage source V1 = 2 V and
= 4 .15∠− 6.87 an AC voltage source v2(t) = 3sin(4t)V. The meter
reads
 
Power = VI =  415 ⋅ ∠120° ×(415∠6.87 ) (a) 2 V (b) 5 V
 3  (c) (2 + 3 /2)V (d) ( 17 /2)V
= 994.3∠126.87° (GATE 2005: 1 Mark)
Wattmeter reading is given by Solution:  PMMC instrument reads or allows only
power × cos q = 994.3 cos 126.87 = 994.3 (−0.6) DC value, therefore it reads only 2 V.
= −597 W Ans. (a)
Ans. (b)

Chapter 7 Solved Question Paper_1 .indd 673 3/23/2016 1:35:25 PM


674        CHAPTER 7:  ELECTRICAL AND ELECTRONIC MEASUREMENTS

23. The simultaneous application of signals x(t) and R


y(t) to the horizontal and vertical plates, respec- A
tively, of an oscilloscope, produces a vertical figure-
of-8 display. If P and Q are constants, and
x(t) = Psin(4t + 30), then y(t) is equal to V
(a) Qsin(4t − 30) (b) Qsin(2t + 15)
(a) 2.25% (b) 2.35%
(c) Qsin(8t + 60) (d) Qsin(4t + 30)
(c) 4.5% (d) 4.71%
(GATE 2005: 2 Marks) (GATE 2005: 2 Marks)

Solution:  Frequency ratio Solution:  For the given circuit


fv Meeting point of horizontal tangencies 2
= = R I I + IV
fh Meeting point of verticcal tangencies 4
A
1
fv = f IV
2 h
V
Since x(t) = P sin(4t+ 30), therefore,
y(t) = Qsin(4t + 30)/2 = Q sin(2t + 15). E
Hence, there should be a phase difference of 15° to
The voltmeter current is
produce the exact figure of 8.
E 180
Ans. (b) IV = = = 0.09 A
2000 2000
24. A DC ammeter has a resistance of 0.1 Ω and its
Also,
current range is 0 — 100 A. If the range is to be
I = I V = 2 A ⇒ I = 1.91 A
extended to 0 — 500 A, then meter requires the fol-
The actual value of resistance is
lowing shunt resistance
E = IRo
(a) 0.010 Ω (b) 0.011 Ω
= 94.24 Ω
180
(c) 0.025 Ω (d) 1.0 Ω Ro =
1.91
The measured resistance is
(GATE 2005: 2 Marks)
= 90 Ω
E 180
Rm = =
Solution:  From the circuit of the given ammeter, I + IV 2
94.24 − 90
we have Therefore, percent error = ×100 = 4.71%
90 Ans. (d)
I Im Rm
26. A 1000 V DC supply has two single-core cables as its
6.1 Ω positive and negative leads; their insulation resis-
tances to earth are 4 MΩ and 6 MΩ, respectively,
as shown in the given figure. A voltmeter with
I − Im Rsh resistance 50 kΩ is used to ­measure the ­insulation
of the cable. When connected between the positive
Im = 100 A and I = 500 A (for 0 — 500 A meter) core and earth, then voltmeter reads
Therefore, 1000 V
I m R m = (I − I m)Rsh
100 × 0.1 = (500 − 100)Rsh 4MΩ 6 MΩ
= 0.025 Ω
10
Rsh =
  400
Ans. (c)
V
25. The set-up in the given figure is used to measure
resistance R. The ammeter and voltmeter resis-
tances are 0.01 Ω and 2000 Ω, respectively. Their
readings are 2 A and 180 V, respectively, giving a (a) 8 V (b) 16 V
measured resistance of 90 Ω. The percentage error (c) 24 V (d) 40 V
in the measurement is (GATE 2005: 2 Marks)

Chapter 7 Solved Question Paper_1 .indd 674 3/23/2016 1:35:39 PM


SOLVED GATE PREVIOUS YEARS’ QUESTIONS        675

Solution:  For the given circuit, the voltmeter (a) 5 V, 1 ms (b) 5 V, 2 ms


reading will be (c) 7.5 V, 2 ms (d) 10 V, 1 ms

6 MΩ (GATE 2006: 1 Mark)

Solution:  For the calibration pulse, the value of


1000 V V 50 kΩ 4 MΩ peak to peak (p-p) voltage = 5 V and the number
of divisions = 2. Therefore
5
∆V = = 2.5 V
2
 50 k Ω× 4 MΩ 
× 
1000 Therefore, for channel 2, since the number of divi-
V=
 50 k Ω× 4 MΩ   50 kΩ + 4 MΩ  sions for (p-p) voltage = 3, so unknown voltage
6 MΩ +   = 3 × 2.5 = 7 V.
 50 k Ω + 4 MΩ 
  Given that frequency = 1 kHz, therefore time
1000 period
V ≈ × 0.05 ≈ 8.2
6 + 0.05 T=
1
= 1 ms
Ans. (a) 1×103
27. Two wattmeters, which are connected to measure The number of divisions in calibration pulse = 4,
the total power on a three-phase system supply- therefore
1
ing a balanced load, read 10.5 kW and — 2.5 kW, ∆T = ms
respectively. The total power and the power factor, 4
respectively, are For channel —2, the number of divisions = 8,
therefore
1
T = ΔT × 8 = × 8 = 2 ms
(a) 13.0 kW, 0.334 (b) 13.0 kW, 0.684
(c) 8.0 kW, 0.52 (d) 8.0 kW, 0.334 4 Ans. (c)
(GATE 2005: 2 Marks) 29. A sampling wattmeter (that computes power from
simultaneously sampled values of voltage and cur-
Solution:  Total power is given by rent) is used to measure the average power of a
P = P1 + P2 = 10.5 — 2.5 = 8 kW load. The peak to peak voltage of the square wave
The power factor is given by cos f, where is 10 V and the current is a triangular wave of 5 A
  P − P 
p-p as shown in the given figure. The period is
φ = tan−1  3  2 1
 20 ms. The reading in W will be
  P1 + P2 
 −13 
= tan−1  3 ×  =− 70.43°
0
 8 
0
Power factor, cosf = cos (−70.43°) = 0.334
Ans. (d) (a) 0 W (b) 25 W
28. The time/div and voltage/div axes of an oscillo- (c) 50 W (d) 100 W
scope have been erased. A student connects a 1 kHz,
5 V p-p square wave calibration pulse to channel 1 (GATE 2006: 1 Mark)
of the scope and observes the screen to be as shown
in the upper trace of the given figure. An unknown Solution:  Average power is given by
signal is connected to channel 2 (lower trace) of the T

∫ VI dt
1
scope. If the time/div and V/div on both channels Pavg =
T
are the same, the amplitude (p-p) and period of the 0
unknown signal are, respectively that is, the total area under the V-I curve. From
the graph we have
Total Area = Area covered in positive half cycle
+ area covered in negative half cycle
= 0 (as both are equal)
So net power = 0 W
Ans. (a)

Chapter 7 Solved Question Paper_1 .indd 675 3/23/2016 1:35:52 PM


676        CHAPTER 7:  ELECTRICAL AND ELECTRONIC MEASUREMENTS

to be —0.5% and 30 minutes, respectively. If the


30. A current of −8 + 6 2(sinwt + 30°) A is passed number of secondary turns is reduced by 1, the
through three meters. They are a centre zero new ratio error (%) and phase error (min) will be,
PMMC meter, a true rms meter and a moving iron respectively,
instrument. The respective readings (in A) will be
(a) 0.0, 30 (b) −0.5, 35
(a) 8, 6, 10 (b) 8, 6, 8 (c) −1.0, 30 (d) −1.0, 25
(c) −8, 10, 10 (d) −8, 2, 2
(GATE 2006: 2 Marks) (GATE 2006: 2 Marks)
Solution:  The nominal ratio of CT, n = 200. Let
Solution:  The PMMC meter will only read DC
value and since it is centre zero type, I = −8 A.
the actual ratio be R. Then error ratio is given by
n −R
When passed through true rms meter, ×100
(6 2 )2 R
I rms = (−8)2 + = 64 + 36 = 10 A
2 Substituting the given ratio error —0.5 %, we have
200 − R
×100 =− 0.5% =− 0.005
The moving iron (MI) instrument measures rms
values, so R
IMI = 10 A R = 201
Ans. (c)
 I 
31. A variable w is related to three other variables x, Also, R = n 1 + E 
y, z as w = xy/z. The variables are measured with  IP 
meters of accuracy ± 0.5% reading, ± 1% of full where IP is the primary current. Substituting values,
scale value and ± 1.5% reading. The actual read- we get loss component of exciting current as
ings of the three meters are 80, 20 and 50 with 100
 I 
being the full scale value for all three. The maxi- 201 = 200 1 + E 
mum uncertainty in the measurement of w will be  160 
(a) ± 0.5% rdg (b) ± 5.5% rdg I E = 0. 8
(c) ± 6.7% rdg (d) ± 7.0 rdg
When the number of turns in the secondary is
(GATE 2006: 2 Marks) reduced by 1, the nominal ratio n′ = 199.
Therefore, new actual ratio is
xy
Solution:  Given that w =  I 
z R ′ = n ′ 1 + E 
 IP 
On applying log on both sides,
 0.8 
log w = log x + log y — log z
Percentage error in w, = 199 1 + 
 160 
dw dx dy dz
=± ± ± ≈ 200
w x y z
Given that Therefore, ratio error is 0 and the phase angle also
dx remains the same.
= ± 0.5%
x Ans. (a)
dy 1
=± ×100 = ± 1% 33. R1 and R4 are the opposite arms of a Wheatstone
y 100 bridge as are R3 and R2. The source voltage is
dz 1 applied across R1 and R3. Under balanced condi-
= ± ×100 = ± 5% tions which one of the following is true?
z 20
Therefore, R3R4 R2R3
dw (a) R1 = (b) R1 =
% = ± 0.5% ± 5% ± 1.5% = ± 7% R2 R4
w R2R4
Ans. (d) (c) R1 = (d) R1 = R2 + R3 + R4
R3
32. A 200/1 current transformer (CT) is wound with
200 turns on the secondary on a toroidal core. (GATE 2006: 2 Marks)
When it carries a current of 160 A on the primary,
the ratio and phase errors of the CT are found Solution:  From the circuit for Wheatstone bridge

Chapter 7 Solved Question Paper_1 .indd 676 3/23/2016 1:36:11 PM


SOLVED GATE PREVIOUS YEARS’ QUESTIONS        677

B 35. A bridge circuit is shown in the given figure. Which


one of the sequences given below is most suitable
R1 R2 for balancing the bridge?
R1
C R2
V A G jX1

R3 R4
D R4
R3
VA − V VA −jX4
+ =0
R1 R3
Therefore,
 R 
VA = V   (a) First adjust R4 and then adjust R1
 R1 + R3 
3
(i)
(b) First adjust R2 and then adjust R3
VC − V VC  R  (c) First adjust R2 and then adjust R4
+ = 0 ⇒ VC = V  4  (ii)
 R2 + R4 
(d) First adjust R4 and then adjust R2
R2 R4
(GATE 2007: 2 Marks)
Since VA = VC, equating Eqs. (i) and (ii), we have Solution:  For the bridge to be balanced,
 R   R  (R1 + jX1) (R4 — jX4) = R2 R3
V  3  = V 
 R + R 
4 
 R1 + R3  2 4
R1R4 — jR1 X4 + jX1R4 + X1X4 = R2R3
(R1R4 + X1X4) + j(X1R4 — R1X4) = R2R3
R2R3 + R3R4 = R1R4 + R3R4
Equating real and imaginary parts
RR
R1 = 2 3 R1R4 + X1X4 = R2R3
R4
X1R4 — R1X4 = 0
Ans. (b)
Therefore,
34. The probes of a non-isolated, two-channel oscil- X1 R1
=
loscope are clipped to points A, B and C in the X4 R4
circuit of the given figure. Vin is a square wave of Therefore, first balance R4 and then R1.
a suitable low frequency. The display on Ch1 and Ans. (a)
Ch2 are as shown on the right. Then the “signal”
and “ground” probes S1, G1 and S2, G2 of Ch1 and 36. Two sinusoidal signals p(w1t) = Asinw1t and q(w2t)
Ch2, respectively, are connected to points are applied to X and Y inputs of a dual channel
CRO. The Lissajous figure displayed on the screen
A is shown in the following figure.
B
R Y
Ch1
L GND12
Vin
Ch2
C X

(a) A, B, C, A (b) A, B, C, B
(c) C, B, A, B (d) B, A, B, C
(GATE 2007: 1 Mark)
Solution:  Here B is the common point, then the The signal q(w2t) will be represented as
signal and ground probes are connected at the fol- (a) q(w 2 t) = A sinw 2 t, w 2 = 2w 1
w
lowing points: (b) q(w 2 t) = A sin w 2 t, w 2 = 1
S1 → A 2
G1 → B (c) q(w 2 t) = A cosw 2 t, w 2 = 2w 1
S2 → C w
(d) q(w 2 t) = A cos w 2 t, w 2 = 1
G2 → B 2
Ans. (b) (GATE 2008: 2 Marks)

Chapter 7 Solved Question Paper_1 .indd 677 3/23/2016 1:36:28 PM


678        CHAPTER 7:  ELECTRICAL AND ELECTRONIC MEASUREMENTS

Solution:  Frequency ratio 38. The pressure coil of a dynamometer type wattmeter
is
fv Meeting point of horizontal tangencies
= (a) highly inductive (b) highly resistive
fh Meeting point of verticcal tangencies (c) purely resistive (d) purely inductive

fv 2 1 (GATE 2009: 1 Mark)


= ⇒ fv = fh
fh 4 2 Solution:  Potential coil connected across the load
Thus, terminal should be highly resistive as the full volt-
w age appear across load.
w y = w x ⇒ w2 = 1
1
2 2 Ans. (b)
Given Lissajous pattern is an ellipse, so there exists 39. The two inputs of a CRO are fed with two station-
a phase difference of 90° between vertical and hori- ary periodic signals. In the X-Y mode, the screen
zontal inputs. Therefore, shows a figure which changes from ellipse to circle
w1 and back to ellipse with its major axis changing
q(w 2 t) = A cosw 2 t; w 2 = orientation slowly and repeatedly. Which of the
2
following inference can be made from this?
Ans. (d)
(a) The signals are not sinusoidal
37. The AC bridge shown in the given figure is used to (b) The amplitudes of the signals are very close but
measure the impedance Z. If the bridge is balanced not equal
for oscillator frequency f = 2 kHz, then impedance (c) The signals are sinusoidal with their frequencies
Z will be very close but not equal
(d) There is a constant but small phase difference
B between the signals
0.398 µF (GATE 2009: 1 Mark)
500 Ω
300 Ω Solution:  The phase difference (f) between the
two signals is varying linearly with time. So the
Oscillator A D C signals are:
15.91 mH X = A sin w t
300 Ω Y = A sin(w t − f )
With increase in phase difference, the ellipse changes
D
into a circle for phase difference of 90° between verti-
(a) (260 + j0) Ω cal and horizontal inputs. Thus the two input signals
(b) (0 + j200) Ω are sinusoidal with their frequency close but not equal.
(c) (260 − j200) Ω
(d) (260 + j200) Ω
Ans. (d)
(GATE 2008: 2 Marks) 40. The given figure shows a three-phase delta con-
nected load supplied from a 400 V, 50 Hz, three-
Solution:  From the given circuit, we have phase balanced source. The pressure coil (PC) and
ZAB = 500 Ω current coil (CC) of a wattmeter are connected to
1 the load as shown, with the coil polarities suitably
ZBC = + 300
j ×2p ×2×103 × 0.398 mF selected to ensure a positive deflection. The watt-
meter reading will be
= 300 − 200 j W
Z2

ZAD = j ×2p ×2×103 ×15.91 mH + 300


=

a

(1

= 300 + 200 j W

Potrebbero piacerti anche